Academia.eduAcademia.edu
E1C42 11/09/2009 19:34:52 Page 1002 E1FFIRS 11/03/2009 15:27:43 Page 1 FUNDAMENTALS OF MODERN MANUFACTURING Materials,Processes,andSystems Fourth Edition Mikell P. Groover Professor of Industrial and Systems Engineering Lehigh University The author and publisher gratefully acknowledge the contributions of Dr. Gregory L. Tonkay, Associate Professor of Industrial and Systems Engineering, Lehigh University. JOHN WILEY & SONS, INC. E1FFIRS 11/03/2009 15:27:43 Page 2 ACQUISITIONS EDITOR EDITORIAL ASSISTANT SENIOR PRODUCTION EDITOR MARKETING MANAGER SENIOR DESIGNER MEDIA EDITOR OUTSIDE PRODUCTION MANAGMENT COVER PHOTO Michael McDonald Renata Marchione Anna Melhorn Christopher Ruel James O’Shea Lauren Sapira Thomson Digital Courtesy of Kennametal, Inc. This book was set in Times New Roman by Thomson Digital and printed and bound by World Color. The cover was printed by World Color. 1 This book is printed on acid-free paper.  Copyright ª 2010 John Wiley & Sons, Inc. All rights reserved. No part of this publication may be reproduced, stored in a retrieval system or transmitted in any form or by any means, electronic, mechanical, photocopying, recording, scanning or otherwise, except as permitted under Section 107 or 108 of the 1976 United States Copyright Act, without either the prior written permission of the Publisher or authorization through payment of the appropriate per-copy fee to the Copyright Clearance Center, Inc. 222 Rosewood Drive, Danvers, MA 01923, website www.copyright.com. Requests to the Publisher for permission should be addressed to the Permissions Department, John Wiley & Sons, Inc., 111 River Street, Hoboken, NJ 07030-5774, (201)748-6011, fax (201)748-6008, website http://www.wiley.com/go/permissions. Evaluation copies are provided to qualified academics and professionals for review purposes only, for use in their courses during the next academic year. These copies are licensed and my not be sold or transferred to a third party. Upon completion of the review period, please return the evaluation copy to Wiley. Return instructions and a free of charge return shipping label are available at www.wiley.com/go/returnlabel. Outside of the United States, please contact your local representative. Groover, Mikell P. Fundamentals of modern manufacturing: materials, processes and systems, 4th ed. ISBN 978-0470-467002 Printed in the United States of America 10 9 8 7 6 5 4 3 2 1 E1FPREF 11/03/2009 17:13:8 Page 3 PREFACE Fundamentals of Modern Manufacturing: Materials, Processes, and Systems is designed for a first course or two-course sequence in manufacturing at the junior level in mechanical, industrial, and manufacturing engineering curricula. Given its coverage of engineering materials, it is also suitable for materials science and engineering courses that emphasize materials processing. Finally, it may be appropriate for technology programs related to the preceding engineering disciplines. Most of the book’s content is concerned with manufacturing processes (about 65% of the text), but it also provides significant coverage of engineering materials and production systems. Materials, processes, and systems are the basic building blocks of modern manufacturing and the three broad subject areas covered in the book. APPROACH The author’s objective in this edition and its predecessors is to provide a treatment of manufacturing that is modern and quantitative. Its claim to be ‘‘modern’’ is based on (1) its balanced coverage of the basic engineering materials (metals, ceramics, polymers, and composite materials), (2) its inclusion of recently developed manufacturing processes in addition to the traditional processes that have been used and refined over many years, and (3) its comprehensive coverage of electronics manufacturing technologies. Competing textbooks tend to emphasize metals and their processing at the expense of the other engineering materials, whose applications and methods of processing have grown significantly in the last several decades. Also, most competing books provide minimum coverage of electronics manufacturing. Yet the commercial importance of electronics products and their associated industries have increased substantially during recent decades. The book’s claim to be more ‘‘quantitative’’ is based on its emphasis on manufacturing science and its greater use of mathematical models and quantitative (end-of-chapter) problems than other manufacturing textbooks. In the case of some processes, it was the first manufacturing processes book to ever provide a quantitative engineering coverage of the topic. NEW TO THIS EDITION This fourth edition is an updated version of the third edition. The publisher’s instructions to the author were to increase content but reduce page count. As this preface is being written, it is too early to tell whether the page count is reduced, but the content has definitely been increased. Additions and changes in the fourth edition include the following: å The chapter count has been reduced from 45 to 42 through consolidation of several chapters. å Selected end-of-chapter problems have been revised to make use of PC spread sheet calculations. å A new section on trends in manufacturing has been added in Chapter 1. iii E1FPREF 11/03/2009 iv 17:13:8 Page 4 Preface å Chapter 5 on dimensions, tolerances, and surfaces has been modified to include measuring and gauging techniques used for these part features. å A new section on specialty steels has been added to Chapter 8 on metals. å Sections on polymer recycling and biodegradable plastics have been added in Chapter 8 on polymers. å Several new casting processes are discussed in Chapter 11. å Sections on thread cutting and gear cutting have been added in Chapter 22 on machining operations and machine tools. å Several additional hole-making tools have been included in Chapter 23 on cutting tool technology. å Former Chapters 28 and 29 on industrial cleaning and coating processes have been consolidated into a single chapter. å A new section on friction-stir welding has been added to Chapter 30 on welding processes. å Chapter 37 on nanotechnology has been reorganized with several new topics and processes added. å The three previous Chapters 39, 40, and 41on manufacturing systems have been consolidated into two chapters: Chapter 38 titled Automation for Manufacturing Systems and Chapter 39 on Integrated Manufacturing Systems. New topics covered in these chapters include automation components and material handling technologies. å Former Chapters 44 on Quality Control and 45 on Measurement and Inspection have been consolidated into a single chapter, Chapter 42 titled Quality Control and Inspection. New sections have been added on Total Quality Management, Six Sigma, and ISO 9000. The text on conventional measuring techniques has been moved to Chapter 5. OTHER KEY FEATURES Additional features of the book continued from the third edition include the following: å A DVD showing action videos of many of the manufacturing processes is included with the book. å A large number of end-of-chapter problems, review questions, and multiple choice questions are available to instructors to use for homework exercises and quizzes. å Sections on Guide to Processing are included in each of the chapters on engineering materials. å Sections on Product Design Considerations are provided in many of the manufacturing process chapters. å Historical Notes on many of the technologies are included throughout the book. å The principal engineering units are System International (metric), but both metric and U.S. Customary Units are used throughout the text. SUPPORT MATERIAL FOR INSTRUCTORS For instructors who adopt the book for their courses, the following support materials are available: E1FPREF 11/03/2009 17:13:8 Page 5 Preface v å A Solutions Manual (in digital format) covering all problems, review questions, and multiple-choice quizzes. å A complete set of PowerPoint slides for all chapters. These support materials may be found at the website www.wiley.com/college/ groover. Evidence that the book has been adopted as the main textbook for the course must be verified. Individual questions or comments may be directed to the author personally at Mikell.Groover@Lehigh.edu. E1FLAST01 11/03/2009 17:13:50 Page 6 ACKNOWLEDGEMENTS I would like to express my appreciation to the following people who served as technical reviewers of individual sets of chapters for the first edition: Iftikhar Ahmad (George Mason University), J. T. Black (Auburn University), David Bourell (University of Texas at Austin), Paul Cotnoir (Worcester Polytechnic Institute), Robert E. Eppich (American Foundryman’s Society), Osama Eyeda (Virginia Polytechnic Institute and State University), Wolter Fabricky (Virginia Polytechnic Institute and State University), Keith Gardiner (Lehigh University), R. Heikes (Georgia Institute of Technology), Jay R. Geddes (San Jose State University), Ralph Jaccodine (Lehigh University), Steven Liang (Georgia Institute of Technology), Harlan MacDowell (Michigan State University), Joe Mize (Oklahoma State University), Colin Moodie (Purdue University), Michael Philpott (University of Illinois at Urbana-Champaign), Corrado Poli (University of Massachusetts at Amherst), Chell Roberts (Arizona State University), Anil Saigal (Tufts University), G. Sathyanarayanan (Lehigh University), Malur Srinivasan (Texas A&M University), A. Brent Strong (Brigham Young University), Yonglai Tian (George Mason University), Gregory L. Tonkay (Lehigh University), Chester VanTyne (Colorado School of Mines), Robert Voigt (Pennsylvania State University), and Charles White (GMI Engineering and Management Institute). For their reviews of certain chapters in the second edition, I would like to thank John T. Berry (Mississippi State University), Rajiv Shivpuri (The Ohio State University), James B. Taylor (North Carolina State University), Joel Troxler (Montana State University), and Ampere A. Tseng (Arizona State University). For their advice and encouragement on the third edition, I would like to thank several of my colleagues at Lehigh, including John Coulter, Keith Gardiner, Andrew Herzing, Wojciech Misiolek, Nicholas Odrey, Gregory Tonkay, and Marvin White. I am especially grateful to Andrew Herzing in the Materials Science and Engineering Department at Lehigh for his review of the new nanofabrication chapter and to Greg Tonkay in my own department for developing many of the new and revised problems and questions in this new edition. For their reviews of the third edition, I would like to thank Mica Grujicic (Clemson University), Wayne Nguyen Hung (Texas A&M University), Patrick Kwon (Michigan State University), Yuan-Shin Lee (North Carolina State University), T. Warren Liao (Louisiana State University), Fuewen Frank Liou (Missouri University of Science and Technology), Val Marinov (North Dakota State University), William J. Riffe (Kettering University), John E. Wyatt (Mississippi State University), Y. Lawrence Yao (Columbia University), Allen Yi (The Ohio State University), and Henry Daniel Young (Wright State University). For their advice on this fourth edition, I would like to thank the following people: Barbara Mizdail (The Pennsylvania State University – Berks campus) and Jack Feng (formerly of Bradley University and now at Caterpillar, Inc.) for conveying questions and feedback from their students, Larry Smith (St. Clair College, Windsor, Ontario) for his advice on using the ASME standards for hole drilling, Richard Budihas (Voltaic LLC) for his contributed research on nanotechnology and integrated circuit processing, and colleague Marvin White at Lehigh for his insights on integrated circuit technology. In addition, it seems appropriate to acknowledge my colleagues at Wiley, Senior Acquisition Editor Michael McDonald and Production Editor Anna Melhorn. Last but certainly not least, I appreciate the kind efforts of editor Sumit Shridhar of Thomson Digital. vi E1FLAST02 11/03/2009 17:14:28 Page 7 ABOUT THE AUTHOR Mikell P. Groover is Professor of Industrial and Systems Engineering at Lehigh University, where he also serves as faculty member in the Manufacturing Systems Engineering Program. He received his B.A. in Arts and Science (1961), B.S. in Mechanical Engineering (1962), M.S. in Industrial Engineering (1966), and Ph.D. (1969), all from Lehigh. He is a Registered Professional Engineer in Pennsylvania. His industrial experience includes several years as a manufacturing engineer with Eastman Kodak Company. Since joining Lehigh, he has done consulting, research, and project work for a number of industrial companies. His teaching and research areas include manufacturing processes, production systems, automation, material handling, facilities planning, and work systems. He has received a number of teaching awards at Lehigh University, as well as the Albert G. Holzman Outstanding Educator Award from the Institute of Industrial Engineers (1995) and the SME Education Award from the Society of Manufacturing Engineers (2001). His publications include over 75 technical articles and ten books (listed below). His books are used throughout the world and have been translated into French, German, Spanish, Portuguese, Russian, Japanese, Korean, and Chinese. The first edition of the current book Fundamentals of Modern Manufacturing received the IIE Joint Publishers Award (1996) and the M. Eugene Merchant Manufacturing Textbook Award from the Society of Manufacturing Engineers (1996). Dr. Groover is a member of the Institute of Industrial Engineers, American Society of Mechanical Engineers (ASME), the Society of Manufacturing Engineers (SME), the North American Manufacturing Research Institute (NAMRI), and ASM International. He is a Fellow of IIE (1987) and SME (1996). PREVIOUS BOOKS BY THE AUTHOR Automation, Production Systems, and Computer-Aided Manufacturing, Prentice Hall, 1980. CAD/CAM: Computer-Aided Design and Manufacturing, Prentice Hall, 1984 (coauthored with E. W. Zimmers, Jr.). Industrial Robotics: Technology, Programming, and Applications, McGraw-Hill Book Company, 1986 (co-authored with M. Weiss, R. Nagel, and N. Odrey). Automation, Production Systems, and Computer Integrated Manufacturing, Prentice Hall, 1987. Fundamentals of Modern Manufacturing: Materials, Processes, and Systems, originally published by Prentice Hall in 1996, and subsequently published by John Wiley & Sons, Inc., 1999. Automation, Production Systems, and Computer Integrated Manufacturing, Second Edition, Prentice Hall, 2001. Fundamentals of Modern Manufacturing: Materials, Processes, and Systems, Second Edition, John Wiley & Sons, Inc., 2002. vii E1FLAST02 viii 11/03/2009 17:14:28 Page 8 About the Author Work Systems and the Methods, Measurement, and Management of Work, Pearson Prentice Hall, 2007. Fundamentals of Modern Manufacturing: Materials, Processes, and Systems, Third Edition, John Wiley & Sons, Inc., 2007. Automation, Production Systems, and Computer Integrated Manufacturing, Third Edition, Pearson Prentice Hall, 2008. E1FTOC 11/11/2009 16:39:41 Page 9 CONTENTS 1 INTRODUCTION AND OVERVIEW OF MANUFACTURING 1 1.1 1.2 1.3 1.4 1.5 1.6 What Is Manufacturing? 2 Materials in Manufacturing 7 Manufacturing Processes 10 Production Systems 16 Trends in Manufacturing 20 Organization of the Book 23 Part I Material Properties and Product Attributes 25 2 THE NATURE OF MATERIALS 25 2.1 2.2 2.3 2.4 2.5 Atomic Structure and the Elements 26 Bonding between Atoms and Molecules 28 Crystalline Structures 30 Noncrystalline (Amorphous) Structures 35 Engineering Materials 37 3 MECHANICAL PROPERTIES OF MATERIALS 40 3.1 3.2 3.3 3.4 3.5 Stress–Strain Relationships 40 Hardness 52 Effect of Temperature on Properties 56 Fluid Properties 58 Viscoelastic Behavior of Polymers 60 4 PHYSICAL PROPERTIES OF MATERIALS 67 4.1 4.2 4.3 4.4 4.5 Volumetric and Melting Properties 67 Thermal Properties 70 Mass Diffusion 72 Electrical Properties 73 Electrochemical Processes 75 5 DIMENSIONS, SURFACES, AND THEIR MEASUREMENT 78 5.1 5.2 5.3 5.4 5.5 Dimensions, Tolerances, and Related Attributes 78 Conventional Measuring Instruments and Gages 79 Surfaces 87 Measurement of Surfaces 92 Effect of Manufacturing Processes 94 Part II Engineering Materials 98 6 METALS 98 6.1 6.2 6.3 6.4 6.5 Alloys and Phase Diagrams 99 Ferrous Metals 103 Nonferrous Metals 120 Superalloys 131 Guide to the Processing of Metals 132 7 CERAMICS 136 7.1 7.2 7.3 7.4 7.5 7.6 Structure and Properties of Ceramics 137 Traditional Ceramics 139 New Ceramics 142 Glass 144 Some Important Elements Related to Ceramics 148 Guide to Processing Ceramics 150 8 POLYMERS 153 8.1 8.2 8.3 8.4 8.5 8.6 Fundamentals of Polymer Science and Technology 155 Thermoplastic Polymers 165 Thermosetting Polymers 171 Elastomers 175 Polymer Recycling and Biodegradability 182 Guide to the Processing of Polymers 184 9 COMPOSITE MATERIALS 187 9.1 9.2 9.3 9.4 9.5 Technology and Classification of Composite Materials 188 Metal Matrix Composites 196 Ceramic Matrix Composites 198 Polymer Matrix Composites 199 Guide to Processing Composite Materials 201 Part III Solidification Processes 205 10 FUNDAMENTALS OF METAL CASTING 205 10.1 10.2 10.3 Overview of Casting Technology 207 Heating and Pouring 210 Solidification and Cooling 213 11 METAL CASTING PROCESSES 225 11.1 11.2 Sand Casting 225 Other Expendable-Mold Casting Processes 230 ix E1FTOC 11/11/2009 x 16:39:42 Page 10 Contents 11.3 11.4 11.5 11.6 11.7 Permanent-Mold Casting Processes 237 Foundry Practice 245 Casting Quality 249 Metals for Casting 251 Product Design Considerations 253 12 GLASSWORKING 258 12.1 12.2 12.3 12.4 Raw Materials Preparation and Melting 258 Shaping Processes in Glassworking 259 Heat Treatment and Finishing 264 Product Design Considerations 266 13 SHAPING PROCESSES FOR PLASTICS 268 13.1 13.2 13.3 13.4 13.5 13.6 13.7 13.8 13.9 13.10 13.11 13.12 Properties of Polymer Melts 269 Extrusion 271 Production of Sheet and Film 281 Fiber and Filament Production (Spinning) 284 Coating Processes 285 Injection Molding 286 Compression and Transfer Molding 295 Blow Molding and Rotational Molding 298 Thermoforming 302 Casting 306 Polymer Foam Processing and Forming 307 Product Design Considerations 308 14 RUBBER-PROCESSING TECHNOLOGY 315 14.1 14.2 14.3 Rubber Processing and Shaping 315 Manufacture of Tires and Other Rubber Products 320 Product Design Considerations 324 15 SHAPING PROCESSES FOR POLYMER MATRIX COMPOSITES 327 15.1 15.2 15.3 15.4 15.5 15.6 Starting Materials for PMCs 329 Open Mold Processes 331 Closed Mold Processes 335 Filament Winding 337 Pultrusion Processes 339 Other PMC Shaping Processes 341 Part IV Particulate Processing of Metals and Ceramics 344 16 POWDER METALLURGY 344 16.1 16.2 16.3 16.4 16.5 Characterization of Engineering Powders 347 Production of Metallic Powders 350 Conventional Pressing and Sintering 352 Alternative Pressing and Sintering Techniques 358 Materials and Products for Powder Metallurgy 361 16.6 Design Considerations in Powder Metallurgy 362 17 PROCESSING OF CERAMICS AND CERMETS 368 17.1 17.2 17.3 17.4 Processing of Traditional Ceramics 368 Processing of New Ceramics 376 Processing of Cermets 378 Product Design Considerations 380 Part V Metal Forming and Sheet Metalworking 383 18 FUNDAMENTALS OF METAL FORMING 383 18.1 18.2 18.3 18.4 18.5 Overview of Metal Forming 383 Material Behavior in Metal Forming 386 Temperature in Metal Forming 387 Strain Rate Sensitivity 389 Friction and Lubrication in Metal Forming 391 19 BULK DEFORMATION PROCESSES IN METAL WORKING 395 19.1 19.2 19.3 19.4 19.5 19.6 Rolling 396 Other Deformation Processes Related to Rolling 403 Forging 405 Other Deformation Processes Related to Forging 416 Extrusion 420 Wire and Bar Drawing 430 20 SHEET METALWORKING 443 20.1 20.2 20.3 20.4 20.5 20.6 20.7 Cutting Operations 444 Bending Operations 450 Drawing 454 Other Sheet-Metal-Forming Operations 461 Dies and Presses for Sheet-Metal Processes 464 Sheet-Metal Operations Not Performed on Presses 471 Bending of Tube Stock 476 Part VI Material Removal Processes 483 21 THEORY OF METAL MACHINING 483 21.1 21.2 21.3 21.4 21.5 Overview of Machining Technology 485 Theory of Chip Formation in Metal Machining 488 Force Relationships and the Merchant Equation 492 Power and Energy Relationships in Machining 497 Cutting Temperature 500 E1FTOC 11/11/2009 16:39:42 Page 11 Contents 22 MACHINING OPERATIONS AND MACHINE TOOLS 507 22.1 22.2 22.3 22.4 22.5 22.6 22.7 22.8 Machining and Part Geometry 507 Turning and Related Operations 510 Drilling and Related Operations 519 Milling 523 Machining Centers and Turning Centers 530 Other Machining Operations 533 Machining Operations for Special Geometries 537 High-Speed Machining 545 23 CUTTING-TOOL TECHNOLOGY 552 23.1 23.2 23.3 23.4 Tool Life 552 Tool Materials 559 Tool Geometry 567 Cutting Fluids 577 24 ECONOMIC AND PRODUCT DESIGN CONSIDERATIONS IN MACHINING 585 24.1 24.2 24.3 24.4 Machinability 585 Tolerances and Surface Finish 587 Selection of Cutting Conditions 591 Product Design Considerations in Machining 597 25 GRINDING AND OTHER ABRASIVE PROCESSES 604 25.1 25.2 Grinding 604 Related Abrasive Processes 621 26 NONTRADITIONAL MACHINING AND THERMAL CUTTING PROCESSES 628 26.1 26.2 26.3 26.4 26.5 Mechanical Energy Processes 629 Electrochemical Machining Processes 632 Thermal Energy Processes 636 Chemical Machining 644 Application Considerations 650 Part VII Property Enhancing and Surface Processing Operations 656 27 HEAT TREATMENT OF METALS 656 27.1 27.2 27.3 27.4 27.5 Annealing 657 Martensite Formation in Steel 657 Precipitation Hardening 661 Surface Hardening 663 Heat Treatment Methods and Facilities 664 28 SURFACE PROCESSING OPERATIONS 668 28.1 Industrial Cleaning Processes 668 28.2 28.3 28.4 28.5 28.6 28.7 28.8 Diffusion and Ion Implantation 673 Plating and Related Processes 674 Conversion Coating 678 Vapor Deposition Processes 680 Organic Coatings 685 Porcelain Enameling and Other Ceramic Coatings 688 Thermal and Mechanical Coating Processes 689 Part VIII Joining and Assembly Processes 693 29 FUNDAMENTALS OF WELDING 693 29.1 29.2 29.3 29.4 Overview of Welding Technology 695 The Weld Joint 697 Physics of Welding 700 Features of a Fusion-Welded Joint 704 30 WELDING PROCESSES 30.1 30.2 30.3 30.4 30.5 30.6 30.7 30.8 709 Arc Welding 709 Resistance Welding 719 Oxyfuel Gas Welding 726 Other Fusion-Welding Processes 729 Solid-State Welding 732 Weld Quality 738 Weldability 742 Design Considerations in Welding 742 31 BRAZING, SOLDERING, AND ADHESIVE BONDING 748 31.1 31.2 31.3 Brazing 748 Soldering 754 Adhesive Bonding 758 32 MECHANICAL ASSEMBLY 766 32.1 32.2 32.3 32.4 32.5 32.6 Threaded Fasteners 767 Rivets and Eyelets 773 Assembly Methods Based on Interference Fits 774 Other Mechanical Fastening Methods 777 Molding Inserts and Integral Fasteners 778 Design for Assembly 779 Part IX Special Processing and Assembly Technologies 786 33 RAPID PROTOTYPING 786 33.1 33.2 33.3 Fundamentals of Rapid Prototyping 787 Rapid Prototyping Technologies 788 Application Issues in Rapid Prototyping 795 xi E1FTOC 11/11/2009 xii 16:39:42 Page 12 Contents 34 PROCESSING OF INTEGRATED CIRCUITS 800 34.1 34.2 34.3 34.4 34.5 34.6 34.7 Overview of IC Processing 800 Silicon Processing 805 Lithography 809 Layer Processes Used in IC Fabrication 812 Integrating the Fabrication Steps 818 IC Packaging 820 Yields in IC Processing 824 35 ELECTRONICS ASSEMBLY AND PACKAGING 830 35.1 35.2 35.3 35.4 35.5 Electronics Packaging 830 Printed Circuit Boards 832 Printed Circuit Board Assembly 840 Surface-Mount Technology 843 Electrical Connector Technology 847 36 MICROFABRICATION TECHNOLOGIES 853 36.1 36.2 Microsystem Products 853 Microfabrication Processes 859 37 NANOFABRICATION TECHNOLOGIES 869 37.1 37.2 37.3 Nanotechnology Products 870 Introduction to Nanoscience 873 Nanofabrication Processes 877 Part X Manufacturing Systems 886 38 AUTOMATION TECHNOLOGIES FOR MANUFACTURING SYSTEMS 886 38.1 38.2 38.3 38.4 Automation Fundamentals 887 Hardware Components for Automation 890 Computer Numerical Control 894 Industrial Robotics 907 39 INTEGRATED MANUFACTURING SYSTEMS 918 39.1 39.2 39.3 39.4 39.5 39.6 39.7 Material Handling 918 Fundamentals of Production Lines 920 Manual Assembly Lines 923 Automated Production Lines 927 Cellular Manufacturing 931 Flexible Manufacturing Systems and Cells 935 Computer Integrated Manufacturing 939 Part XI Manufacturing Support Systems 945 40 MANUFACTURING ENGINEERING 945 40.1 40.2 40.3 Process Planning 946 Problem Solving and Continuous Improvement 953 Concurrent Engineering and Design for Manufacturability 954 41 PRODUCTION PLANNING AND CONTROL 959 41.1 41.2 41.3 41.4 41.5 Aggregate Planning and the Master Production Schedule 960 Inventory Control 962 Material and Capacity Requirements Planning 965 Just-In-Time and Lean Production 969 Shop Floor Control 971 42 QUALITY CONTROL AND INSPECTION 977 42.1 42.2 42.3 42.4 42.5 42.6 Product Quality 977 Process Capability and Tolerances 978 Statistical Process Control 980 Quality Programs in Manufacturing 984 Inspection Principles 990 Modern Inspection Technologies 992 INDEX 1003 E1C01 11/11/2009 13:31:33 1 Page 1 INTRODUCTION AND OVERVIEW OF MANUFACTURING Chapter Contents 1.1 What 1.1.1 1.1.2 1.1.3 Is Manufacturing? Manufacturing Defined Manufacturing Industries and Products Manufacturing Capability 1.2 Materials in Manufacturing 1.2.1 Metals 1.2.2 Ceramics 1.2.3 Polymers 1.2.4 Composites 1.3 Manufacturing Processes 1.3.1 Processing Operations 1.3.2 Assembly Operations 1.3.3 Production Machines and Tooling 1.4 Production Systems 1.4.1 Production Facilities 1.4.2 Manufacturing Support Systems 1.5 Trends in Manufacturing 1.5.1 Lean Production and Six Sigma 1.5.2 Globalization and Outsourcing 1.5.3 Environmentally Conscious Manufacturing 1.5.4 Microfabrication and Nanotechnology 1.6 Organization of the Book Making things has been an essential activity of human civilizations since before recorded history. Today, the term manufacturing is used for this activity. For technological and economic reasons, manufacturing is important to the welfare of the United States and most other developed and developing nations. Technology can be defined as the application of science to provide society and its members with those things that are needed or desired. Technology affects our daily lives, directly and indirectly, in many ways. Consider the list of products in Table 1.1. They represent various technologies that help society and its members to live better. What do all these products have in common? They are all manufactured. These technological wonders would not be available to society if they could not be manufactured. Manufacturing is the critical factor that makes technology possible. Economically, manufacturing is an important means by which a nation creates material wealth. In the United States, the manufacturing industries account for about 15% of gross domestic product (GDP). A country’s natural resources, such as agricultural lands, mineral deposits, and oil reserves, also create wealth. In the U.S., agriculture, mining, and similar industries account for less than 5% of GDP (agriculture alone is only about 1%). Construction and public utilities make up around 5%. The rest is service industries, which include retail, transportation, banking, communication, education, and government. The service sector accounts for more than 75% of U.S. GDP. Government alone accounts for about as much of GDP as the manufacturing sector; however, government services do not create wealth. In the modern global economy, a nation must have a strong manufacturing base (or it must have significant natural resources) if it is to provide a strong economy and a high standard of living for its people. In this opening chapter, we consider some general topics about manufacturing. What is manufacturing? How is it organized in industry? What are the materials, processes, and systems by which it is accomplished? 1 E1C01 11/11/2009 2 13:31:33 Page 2 Chapter 1/Introduction and Overview of Manufacturing TABLE 1.1 Products representing various technologies, most of which affect nearly everyone. Athletic shoes Automatic teller machine Automatic dishwasher Ballpoint pen Cell phone Compact disc (CD) Compact disc player Compact fluorescent light bulb Contact lenses Digital camera Digital video disc (DVD) Digital video disc player 1.1 Fax machine Flat-screen high-definition television Hand-held electronic calculator High density PC diskette Home security system Hybrid gas-electric automobile Industrial robot Ink-jet color printer Integrated circuit Magnetic resonance imaging (MRI) machine for medical diagnosis Microwave oven One-piece molded plastic patio chair Optical scanner Personal computer (PC) Photocopying machine Pull-tab beverage cans Quartz crystal wrist watch Self-propelled mulching lawnmower Supersonic aircraft Tennis racket of composite materials Video games Washing machine and dryer WHAT IS MANUFACTURING? The word manufacture is derived from two Latin words, manus (hand) and factus (make); the combination means made by hand. The English word manufacture is several centuries old, and ‘‘made by hand’’ accurately described the manual methods used when the word was first coined.1 Most modern manufacturing is accomplished by automated and computer-controlled machinery (Historical Note 1.1). Historical Note 1.1 T History of manufacturing he history of manufacturing can be separated into two subjects: (1) human’s discovery and invention of materials and processes to make things, and (2) development of the systems of production. The materials and processes to make things predate the systems by several millennia. Some of the processes—casting, hammering (forging), and grinding—date back 6000 years or more. The early fabrication of implements and weapons was accomplished more as crafts and trades than manufacturing as it is known today. The ancient Romans had what might be called factories to produce weapons, scrolls, pottery and glassware, and other products of the time, but the procedures were largely based on handicraft. The systems aspects of manufacturing are examined here, and the materials and processes are postponed until Historical Note 1.2. Systems of manufacturing refer to the ways of organizing people and equipment so that production can be performed more efficiently. Several historical events and discoveries stand out as having had a major impact on the development of modern manufacturing systems. Certainly one significant discovery was the principle of division of labor—dividing the total work into tasks and having individual workers each become a specialist at performing only one task. This principle had been practiced for centuries, but the economist Adam Smith (1723–1790) is credited with first explaining its economic significance in The Wealth of Nations. The Industrial Revolution (circa 1760–1830) had a major impact on production in several ways. It marked the change from an economy based on agriculture and handicraft to one based on industry and manufacturing. The change began in England, where a series of machines were invented and steam power replaced water, wind, and animal power. These advances gave British industry significant advantages over other nations, and England attempted to restrict export of the new technologies. However, the revolution eventually spread to other European countries and the United States. 1 As a noun, the word manufacture first appeared in English around 1567 AD. As a verb, it first appeared around 1683 AD. E1C01 11/11/2009 13:31:33 Page 3 Section 1.1/What Is Manufacturing? Several inventions of the Industrial Revolution greatly contributed to the development of manufacturing: (1) Watt’s steam engine, a new power-generating technology for industry; (2) machine tools, starting with John Wilkinson’s boring machine around 1775 (Historical Note 22.1); (3) the spinning jenny, power loom, and other machinery for the textile industry that permitted significant increases in productivity; and (4) the factory system, a new way of organizing large numbers of production workers based on division of labor. While England was leading the industrial revolution, an important concept was being introduced in the United States: interchangeable parts manufacture. Much credit for this concept is given to Eli Whitney (1765–1825), although its importance had been recognized by others [9]. In 1797, Whitney negotiated a contract to produce 10,000 muskets for the U.S. government. The traditional way of making guns at the time was to custom fabricate each part for a particular gun and then hand-fit the parts together by filing. Each musket was unique, and the time to make it was considerable. Whitney believed that the components could be made accurately enough to permit parts assembly without fitting. After several years of development in his Connecticut factory, he traveled to Washington in 1801 to demonstrate the principle. He laid out components for 10 muskets before government officials, including Thomas Jefferson, and proceeded to select parts randomly to assemble the guns. No special filing or fitting was required, and all of the guns worked perfectly. The secret behind his achievement was the collection of special machines, fixtures, and gages that he had developed in his factory. Interchangeable parts manufacture required many years of development before becoming a practical reality, but it revolutionized methods of manufacturing. It is a prerequisite for mass production. Because its origins were in the United States, interchangeable parts production came to be known as the American System of manufacture. The mid- and late 1800s witnessed the expansion of railroads, steam-powered ships, and other machines that created a growing need for iron and steel. New steel 3 production methods were developed to meet this demand (Historical Note 6.1). Also during this period, several consumer products were developed, including the sewing machine, bicycle, and automobile. To meet the mass demand for these products, more efficient production methods were required. Some historians identify developments during this period as the Second Industrial Revolution, characterized in terms of its effects on manufacturing systems by: (1) mass production, (2) scientific management movement, (3) assembly lines, and (4) electrification of factories. In the late 1800s, the scientific management movement was developing in the United States in response to the need to plan and control the activities of growing numbers of production workers. The movement’s leaders included Frederick W. Taylor (1856–1915), Frank Gilbreth (1868–1924), and his wife Lilian (1878–1972). Scientific management included several features [2]: (1) motion study, aimed at finding the best method to perform a given task; (2) time study, to establish work standards for a job; (3) extensive use of standards in industry; (4) the piece rate system and similar labor incentive plans; and (5) use of data collection, record keeping, and cost accounting in factory operations. Henry Ford (1863–1947) introduced the assembly line in 1913 at his Highland Park, MI plant. The assembly line made possible the mass production of complex consumer products. Use of assembly line methods permitted Ford to sell a Model T automobile for as little as $500, thus making ownership of cars feasible for a large segment of the U.S. population. In 1881, the first electric power generating station had been built in New York City, and soon electric motors were being used as a power source to operate factory machinery. This was a far more convenient power delivery system than steam engines, which required overhead belts to distribute power to the machines. By 1920, electricity had overtaken steam as the principal power source in U.S. factories. The twentieth century was a time of more technological advances than in all other centuries combined. Many of these developments resulted in the automation of manufacturing. 1.1.1 MANUFACTURING DEFINED As a field of study in the modern context, manufacturing can be defined two ways, one technologic and the other economic. Technologically, manufacturing is the application of physical and chemical processes to alter the geometry, properties, and/or appearance of a given starting material to make parts or products; manufacturing also includes assembly of multiple parts to make products. The processes to accomplish manufacturing involve a combination of machinery, tools, power, and labor, as depicted in Figure 1.1(a). 11/11/2009 Page 4 Chapter 1/Introduction and Overview of Manufacturing Manufacturing process ac h To ine ol r y in Po g w e La r bo r 4 13:31:33 M E1C01 Starting material Processed part Value added $$ $ $$$ Manufacturing process Scrap and waste Starting material (a) FIGURE 1.1 Material in processing Processed part (b) Two ways to define manufacturing: (a) as a technical process, and (b) as an economic process. Manufacturing is almost always carried out as a sequence of operations. Each operation brings the material closer to the desired final state. Economically, manufacturing is the transformation of materials into items of greater value by means of one or more processing and/or assembly operations, as depicted in Figure 1.1(b). The key point is that manufacturing adds value to the material by changing its shape or properties, or by combining it with other materials that have been similarly altered. The material has been made more valuable through the manufacturing operations performed on it. When iron ore is converted into steel, value is added. When sand is transformed into glass, value is added. When petroleum is refined into plastic, value is added. And when plastic is molded into the complex geometry of a patio chair, it is made even more valuable. The words manufacturing and production are often used interchangeably. The author’s view is that production has a broader meaning than manufacturing. To illustrate, one might speak of ‘‘crude oil production,’’ but the phrase ‘‘crude oil manufacturing’’ seems out of place. Yet when used in the context of products such as metal parts or automobiles, either word seems okay. 1.1.2 MANUFACTURING INDUSTRIES AND PRODUCTS Manufacturing is an important commercial activity performed by companies that sell products to customers. The type of manufacturing done by a company depends on the kind of product it makes. Let us explore this relationship by examining the types of industries in manufacturing and identifying the products they make. Manufacturing Industries Industry consists of enterprises and organizations that produce or supply goods and services. Industries can be classified as primary, secondary, or tertiary. Primary industries cultivate and exploit natural resources, such as agriculture and mining. Secondary industries take the outputs of the primary industries and convert them into consumer and capital goods. Manufacturing is the principal activity in this category, but construction and power utilities are also included. Tertiary industries constitute the service sector of the economy. A list of specific industries in these categories is presented in Table 1.2. This book is concerned with the secondary industries in Table 1.2, which include the companies engaged in manufacturing. However, the International Standard Industrial Classification (ISIC) used to compile Table 1.2 includes several industries whose production technologies are not covered in this text; for example, beverages, chemicals, and food processing. In this book, manufacturing means production of hardware, which ranges from nuts and bolts to digital computers and military weapons. Plastic and ceramic E1C01 11/11/2009 13:31:34 Page 5 Section 1.1/What Is Manufacturing? TABLE 1.2 5 Specific industries in the primary, secondary, and tertiary categories. Primary Agriculture Forestry Fishing Livestock Quarries Mining Petroleum Secondary Tertiary (Service) Aerospace Apparel Automotive Basic metals Beverages Building materials Chemicals Computers Food processing Glass, ceramics Heavy machinery Paper Petroleum refining Pharmaceuticals Plastics (shaping) Power utilities Banking Communications Education Entertainment Financial services Government Health and medical Insurance Legal Real estate Repair and maintenance Restaurant Retail trade Tourism Construction Publishing Hotel Transportation Textiles Information Wholesale trade Consumer appliances Electronics Tire and rubber Wood and furniture Equipment Fabricated metals products are included, but apparel, paper, pharmaceuticals, power utilities, publishing, and wood products are excluded. Manufactured Products Final products made by the manufacturing industries can be divided into two major classes: consumer goods and capital goods. Consumer goods are products purchased directly by consumers, such as cars, personal computers, TVs, tires, and tennis rackets. Capital goods are those purchased by companies to produce goods and/or provide services. Examples of capital goods include aircraft, computers, communication equipment, medical apparatus, trucks and buses, railroad locomotives, machine tools, and construction equipment. Most of these capital goods are purchased by the service industries. It was noted in the Introduction that manufacturing accounts for about 15% of GDP and services about 75% of GDP in the United States. Yet the manufactured capital goods purchased by the service sector are the enablers of that sector. Without the capital goods, the service industries could not function. In addition to final products, other manufactured items include the materials, components, and supplies used by the companies that make the final products. Examples of these items include sheet steel, bar stock, metal stampings, machined parts, plastic moldings and extrusions, cutting tools, dies, molds, and lubricants. Thus, the manufacturing industries consist of a complex infrastructure with various categories and layers of intermediate suppliers with whom the final consumer never deals. This book is generally concerned with discrete items—individual parts and assembled products, rather than items produced by continuous processes. A metal stamping is a discrete item, but the sheet-metal coil from which it is made is continuous (almost). Many discrete parts start out as continuous or semicontinuous products, such as extrusions and electrical wire. Long sections made in almost continuous lengths are cut to the desired size. An oil refinery is a better example of a continuous process. Production Quantity and Product Variety The quantity of products made by a factory has an important influence on the way its people, facilities, and procedures are organized. Annual production quantities can be classified into three ranges: (1) low production, quantities in the range 1 to 100 units per year; (2) medium production, from 100 to 10,000 units annually; and (3) high production, 10,000 to millions of units. The boundaries E1C01 11/11/2009 6 13:31:34 Page 6 Chapter 1/Introduction and Overview of Manufacturing FIGURE 1.2 Relationship between product variety and production quantity in discrete product manufacturing. between the three ranges are somewhat arbitrary (in the author’s judgment). Depending on the kinds of products, these boundaries may shift by an order of magnitude or so. Production quantity refers to the number of units produced annually of a particular product type. Some plants produce a variety of different product types, each type being made in low or medium quantities. Other plants specialize in high production of only one product type. It is instructive to identify product variety as a parameter distinct from production quantity. Product variety refers to different product designs or types that are produced in the plant. Different products have different shapes and sizes; they perform different functions; they are intended for different markets; some have more components than others; and so forth. The number of different product types made each year can be counted. When the number of product types made in the factory is high, this indicates high product variety. There is an inverse correlation between product variety and production quantity in terms of factory operations. If a factory’s product variety is high, then its production quantity is likely to be low; but if production quantity is high, then product variety will be low, as depicted in Figure 1.2. Manufacturing plants tend to specialize in a combination of production quantity and product variety that lies somewhere inside the diagonal band in Figure 1.2. Although product variety has been identified as a quantitative parameter (the number of different product types made by the plant or company), this parameter is much less exact than production quantity, because details on how much the designs differ are not captured simply by the number of different designs. Differences between an automobile and an air conditioner are far greater than between an air conditioner and a heat pump. Within each product type, there are differences among specific models. The extent of the product differences may be small or great, as illustrated in the automotive industry. Each of the U.S. automotive companies produces cars with two or three different nameplates in the same assembly plant, although the body styles and other design features are virtually the same. In different plants, the company builds heavy trucks. The terms ‘‘soft’’ and ‘‘hard’’ might be used to describe these differences in product variety. Soft product variety occurs when there are only small differences among products, such as the differences among car models made on the same production line. In an assembled product, soft variety is characterized by a high proportion of common parts among the models. Hard product variety occurs when the products differ substantially, and there are few common parts, if any. The difference between a car and a truck exemplifies hard variety. 1.1.3 MANUFACTURING CAPABILITY A manufacturing plant consists of a set of processes and systems (and people, of course) designed to transform a certain limited range of materials into products of increased value. These three building blocks—materials, processes, and systems—constitute the E1C01 11/11/2009 13:31:34 Page 7 Section 1.2/Materials in Manufacturing 7 subject of modern manufacturing. There is a strong interdependence among these factors. A company engaged in manufacturing cannot do everything. It must do only certain things, and it must do those things well. Manufacturing capability refers to the technical and physical limitations of a manufacturing firm and each of its plants. Several dimensions of this capability can be identified: (1) technological processing capability, (2) physical size and weight of product, and (3) production capacity. Technological Processing Capability The technological processing capability of a plant (or company) is its available set of manufacturing processes. Certain plants perform machining operations, others roll steel billets into sheet stock, and others build automobiles. A machine shop cannot roll steel, and a rolling mill cannot build cars. The underlying feature that distinguishes these plants is the processes they can perform. Technological processing capability is closely related to material type. Certain manufacturing processes are suited to certain materials, whereas other processes are suited to other materials. By specializing in a certain process or group of processes, the plant is simultaneously specializing in certain material types. Technological processing capability includes not only the physical processes, but also the expertise possessed by plant personnel in these processing technologies. Companies must concentrate on the design and manufacture of products that are compatible with their technological processing capability. Physical Product Limitations A second aspect of manufacturing capability is imposed by the physical product. A plant with a given set of processes is limited in terms of the size and weight of the products that can be accommodated. Large, heavy products are difficult to move. To move these products about, the plant must be equipped with cranes of the required load capacity. Smaller parts and products made in large quantities can be moved by conveyor or other means. The limitation on product size and weight extends to the physical capacity of the manufacturing equipment as well. Production machines come in different sizes. Larger machines must be used to process larger parts. The production and material handling equipment must be planned for products that lie within a certain size and weight range. Production Capacity A third limitation on a plant’s manufacturing capability is the production quantity that can be produced in a given time period (e.g., month or year). This quantity limitation is commonly called plant capacity, or production capacity, defined as the maximum rate of production that a plant can achieve under assumed operating conditions. The operating conditions refer to number of shifts per week, hours per shift, direct labor manning levels in the plant, and so on. These factors represent inputs to the manufacturing plant. Given these inputs, how much output can the factory produce? Plant capacity is usually measured in terms of output units, such as annual tons of steel produced by a steel mill, or number of cars produced by a final assembly plant. In these cases, the outputs are homogeneous. In cases in which the output units are not homogeneous, other factors may be more appropriate measures, such as available labor hours of productive capacity in a machine shop that produces a variety of parts. Materials, processes, and systems are the basic building blocks of manufacturing and the three broad subject areas of this book. This introductory chapter provides an overview of these three subjects before embarking on detailed coverage in the remaining chapters. 1.2 MATERIALS IN MANUFACTURING Most engineering materials can be classified into one of three basic categories:(1) metals, (2) ceramics, and (3) polymers.Their chemistries are different, their mechanical and physical properties are different, and these differences affect the manufacturing processes that can be used to produce products from them. In addition to the three basic categories, there are E1C01 11/11/2009 8 13:31:34 Page 8 Chapter 1/Introduction and Overview of Manufacturing Ferrous Metals Metals Nonferrous Metals Crystalline Ceramics Ceramics Glasses Engineering Materials Thermoplastics Polymers Thermosets Elastomers Metal Matrix Composites Composites FIGURE 1.3 Classification of the four engineering materials. Ceramic Matrix Composites Polymer Matrix Composites (4) composites—nonhomogeneous mixtures of the other three basic types rather than a unique category. The classification of the four groups is pictured in Figure 1.3. This section surveys these materials. Chapters 6 through 9 cover the four material types in more detail. 1.2.1 METALS Metals used in manufacturing are usually alloys, which are composed of two or more elements, with at least one being a metallic element. Metals and alloys can be divided into two basic groups: (1) ferrous and (2) nonferrous. Ferrous Metals Ferrous metals are based on iron; the group includes steel and cast iron. These metals constitute the most important group commercially, more than three fourths of the metal tonnage throughout the world. Pure iron has limited commercial use, but when alloyed with carbon, iron has more uses and greater commercial value than any other metal. Alloys of iron and carbon form steel and cast iron. Steel can be defined as an iron–carbon alloy containing 0.02% to 2.11% carbon. It is the most important category within the ferrous metal group. Its composition often includes other alloying elements as well, such as manganese, chromium, nickel,and molybdenum, to enhance the properties of the metal. Applications of steel include construction (bridges, I-beams, and E1C01 11/11/2009 13:31:34 Page 9 Section 1.2/Materials in Manufacturing 9 nails), transportation (trucks, rails, and rolling stock for railroads), and consumer products (automobiles and appliances). Cast iron is an alloy of iron and carbon (2% to 4%) used in casting (primarily sand casting). Silicon is also present in the alloy (in amounts from 0.5% to 3%), and other elements are often added also, to obtain desirable properties in the cast part. Cast iron is available in several different forms, of which gray cast iron is the most common; its applications include blocks and heads for internal combustion engines. Nonferrous Metals Nonferrous metals include the other metallic elements and their alloys. In almost all cases, the alloys are more important commercially than the pure metals. The nonferrous metals include the pure metals and alloys of aluminum, copper, gold, magnesium, nickel, silver, tin, titanium, zinc, and other metals. 1.2.2 CERAMICS A ceramic is defined as a compound containing metallic (or semimetallic) and nonmetallic elements. Typical nonmetallic elements are oxygen, nitrogen, and carbon. Ceramics include a variety of traditional and modern materials. Traditional ceramics, some of which have been used for thousands of years, include: clay (abundantly available, consisting of fine particles of hydrous aluminum silicates and other minerals used in making brick, tile, and pottery); silica (the basis for nearly all glass products); and alumina and silicon carbide (two abrasive materials used in grinding). Modern ceramics include some of the preceding materials, such as alumina, whose properties are enhanced in various ways through modern processing methods. Newer ceramics include: carbides—metal carbides such as tungsten carbide and titanium carbide, which are widely used as cutting tool materials; and nitrides—metal and semimetal nitrides such as titanium nitride and boron nitride, used as cutting tools and grinding abrasives. For processing purposes, ceramics can be divided into crystalline ceramics and glasses. Different methods of manufacturing are required for the two types. Crystalline ceramics are formed in various ways from powders and then fired (heated to a temperature below the melting point to achieve bonding between the powders). The glass ceramics (namely, glass) can be melted and cast, and then formed in processes such as traditional glass blowing. 1.2.3 POLYMERS A polymer is a compound formed of repeating structural units called mers, whose atoms share electrons to form very large molecules. Polymers usually consist of carbon plus one or more other elements, such as hydrogen, nitrogen, oxygen, and chlorine. Polymers are divided into three categories: (1) thermoplastic polymers, (2) thermosetting polymers, and (3) elastomers. Thermoplasticpolymerscan besubjected tomultipleheatingand coolingcycles without substantially altering the molecular structure of the polymer. Common thermoplastics include polyethylene, polystyrene, polyvinylchloride, and nylon. Thermosetting polymers chemically transform (cure) into a rigid structure on cooling from a heated plastic condition; hence the name thermosetting. Members of this type include phenolics, amino resins, and epoxies. Although the name thermosetting is used, some of these polymers cure by mechanisms other than heating. Elastomers are polymers that exhibit significant elastic behavior; hence the name elastomer. They include natural rubber, neoprene, silicone, and polyurethane. 1.2.4 COMPOSITES Composites do not really constitute a separate category of materials; they are mixtures of the other three types. A composite is a material consisting of two or more phases that are E1C01 11/11/2009 10 13:31:34 Page 10 Chapter 1/Introduction and Overview of Manufacturing processed separately and then bonded together to achieve properties superior to those of its constituents. The term phase refers to a homogeneous mass of material, such as an aggregation of grains of identical unit cell structure in a solid metal. The usual structure of a composite consists of particles or fibers of one phase mixed in a second phase, called the matrix. Composites are found in nature (e.g., wood), and they can be produced synthetically. The synthesized type is of greater interest here, and it includes glass fibers in a polymer matrix, such as fiber-reinforced plastic; polymer fibers of one type in a matrix of a second polymer, such as an epoxy-Kevlar composite; and ceramic in a metal matrix, such as a tungsten carbide in a cobalt binder to form a cemented carbide cutting tool. Properties of a composite depend on its components, the physical shapes of the components, and the way they are combined to form the final material. Some composites combine high strength with light weight and are suited to applications such as aircraft components, car bodies, boat hulls, tennis rackets, and fishing rods. Other composites are strong, hard, and capable of maintaining these properties at elevated temperatures, for example, cemented carbide cutting tools. 1.3 MANUFACTURING PROCESSES A manufacturing process is a designed procedure that results in physical and/or chemical changes to a starting work material with the intention of increasing the value of that material. A manufacturing process is usually carried out as a unit operation , which means that it is a single step in the sequence of steps required to transform the starting material into a final product. Manufacturing operations can be divided into two basic types: (1) processing operations and (2) assembly operations. A processing operation transforms a work material from one state of completion to a more advanced state that is closer to the final desired product. It adds value by changing the geometry, properties, or appearance of the starting material. In general, processing operations are performed on discrete workparts, but certain processing operations are also applicable to assembled items (e.g., painting a spot-welded car body). An assembly operation joins two or more components to create a new entity, called an assembly, subassembly, or some other term that refers to the joining process (e.g., a welded assembly is called a weldment). A classification of manufacturing processes is presented in Figure 1.4. Many of the manufacturing processes covered in this text can be viewed on the DVD that comes with this book. Alerts are provided on these video clips throughout the text. Some of the basic processes used in modern manufacturing date from antiquity (Historical Note 1.2). 1.3.1 PROCESSING OPERATIONS A processing operation uses energy to alter a workpart’s shape, physical properties, or appearance to add value to the material. The forms of energy include mechanical, thermal, electrical, and chemical. The energy is applied in a controlled way by means of machinery and tooling. Human energy may also be required, but the human workers are generally employed to control the machines, oversee the operations, and load and unload parts before and after each cycle of operation. A general model of a processing operation is illustrated in Figure 1.1(a). Material is fed into the process, energy is applied by the machinery and tooling to transform the material, and the completed workpart exits the process. Most production operations produce waste or scrap, either as a natural aspect of the process (e.g., removing material, as in machining) or in the form of occasional defective pieces. It is an important objective in manufacturing to reduce waste in either of these forms. E1C01 11/11/2009 13:31:34 Page 11 Section 1.3/Manufacturing Processes 11 Solidification processes Shaping processes Processing operations Particulate processing Deformation processes Material removal Property enhancing processes Surface processing operations Manufacturing processes Heat treatment Cleaning and surface treatments Coating and deposition processes Welding Permanent joining processes Adhesive bonding Assembly operations FIGURE 1.4 Classification of manufacturing processes. Historical Note 1.2 A Brazing and soldering Mechanical fastening Threaded fasteners Permanent fastening methods Manufacturing materials and processes lthough most of the historical developments that form the modern practice of manufacturing have occurred only during the last few centuries (Historical Note 1.1), several of the basic fabrication processes date as far back as the Neolithic period (circa 8000–3000 BCE.). It was during this period that processes such as the following were developed: carving and other woodworking, hand forming and firing of clay pottery, grinding and polishing of stone, spinning and weaving of textiles, and dyeing of cloth. Metallurgy and metalworking also began during the Neolithic period, in Mesopotamia and other areas around the Mediterranean. It either spread to, or developed independently in, regions of Europe and Asia. Gold was found by early humans in relatively pure form in nature; it could be hammered into shape. Copper was probably the first metal to be extracted from ores, thus requiring smelting as a processing technique. Copper could not be hammered readily because it strain hardened; instead, it was shaped by casting (Historical Note 10.1). Other metals used during this period were silver and tin. It was discovered that copper alloyed with tin produced a more workable metal than copper alone (casting and hammering could both be used). This heralded the important period known as the Bronze Age (circa 3500–1500 BCE.). Iron was also first smelted during the Bronze Age. Meteorites may have been one source of the metal, but iron ore was also mined. Temperatures required to reduce iron ore to metal are significantly higher than for copper, which made furnace operations more difficult. Other processing methods were also more difficult for the same reason. Early blacksmiths learned that when certain irons (those containing small amounts of carbon) were sufficiently heated and then quenched, they became very hard. This permitted grinding a very sharp cutting edge on knives and weapons, but it also made the metal brittle. Toughness could be increased by reheating at a lower temperature, a process known as tempering. E1C01 11/11/2009 12 13:31:35 Page 12 Chapter 1/Introduction and Overview of Manufacturing What we have described is, of course, the heat treatment of steel. The superior properties of steel caused it to succeed bronze in many applications (weaponry, agriculture, and mechanical devices). The period of its use has subsequently been named the Iron Age (starting around 1000 BCE.). It was not until much later, well into the nineteenth century, that the demand for steel grew significantly and more modern steelmaking techniques were developed (Historical Note 6.1). The beginnings of machine tool technology occurred during the Industrial Revolution. During the period 1770–1850, machine tools were developed for most of the conventional material removal processes, such as boring, turning, drilling, milling, shaping, and planing (Historical Note 22.1). Many of the individual processes predate the machine tools by centuries; for example, drilling and sawing (of wood) date from ancient times, and turning (of wood) from around the time of Christ. Assembly methods were used in ancient cultures to make ships, weapons, tools, farm implements, machinery, chariots and carts, furniture, and garments. The earliest processes included binding with twine and rope, riveting and nailing, and soldering. Around 2000 years ago, forge welding and adhesive bonding were developed. Widespread use of screws, bolts, and nuts as fasteners—so common in today’s assembly—required the development of machine tools that could accurately cut the required helical shapes (e.g., Maudsley’s screw cutting lathe, 1800). It was not until around 1900 that fusion welding processes started to be developed as assembly techniques (Historical Note 29.1). Natural rubber was the first polymer to be used in manufacturing (if we overlook wood, which is a polymer composite). The vulcanization process, discovered by Charles Goodyear in 1839, made rubber a useful engineering material (Historical Note 8.2). Subsequent developments included plastics such as cellulose nitrate in 1870, Bakelite in 1900, polyvinylchloride in 1927, polyethylene in 1932, and nylon in the late 1930s (Historical Note 8.1). Processing requirements for plastics led to the development of injection molding (based on die casting, one of the metal casting processes) and other polymer-shaping techniques. Electronics products have imposed unusual demands on manufacturing in terms of miniaturization. The evolution of the technology has been to package more and more devices into smaller and smaller areas—in some cases millions of transistors onto a flat piece of semiconductor material that is only 12 mm (0.50 in.) on a side. The history of electronics processing and packaging dates from only a few decades (Historical Notes 34.1, 35.1, and 35.2). More than one processing operation is usually required to transform the starting material into final form. The operations are performed in the particular sequence required to achieve the geometry and condition defined by the design specification. Three categories of processing operations are distinguished: (1) shaping operations, (2) property-enhancing operations, and (3) surface processing operations. Shaping operations alter the geometry of the starting work material by various methods. Common shaping processes include casting, forging, and machining. Property-enhancing operations add value to the material by improving its physical properties without changing its shape. Heat treatment is the most common example. Surface processing operations are performed to clean, treat, coat, or deposit material onto the exterior surface of the work. Common examples of coating are plating and painting. Shaping processes are covered in Parts III through VI, corresponding to the four main categories of shaping processes in Figure 1.4. Property-enhancing processes and surface processing operations are covered in Part VII. Shaping Processes Most shape processing operations apply heat, mechanical force, or a combination of these to effect a change in geometry of the work material. There are various ways to classify the shaping processes. The classification used in this book is based on the state of the starting material, by which we have four categories: (1) solidification processes, in which the starting material is a heated liquid or semifluid that cools and solidifies to form the part geometry; (2) particulate processing, in which the starting material is a powder, and the powders are formed and heated into the desired geometry; (3) deformation processes, in which the starting material is a ductile solid (commonly metal) that is deformed to shape the part; and (4) material removal processes, in which E1C01 11/11/2009 13:31:35 Page 13 Section 1.3/Manufacturing Processes 13 FIGURE 1.5 Casting and molding processes start with a work material heated to a fluid or semifluid state. The process consists of: (1) pouring the fluid into a mold cavity and (2) allowing the fluid to solidify, after which the solid part is removed from the mold. the starting material is a solid (ductile or brittle), from which material is removed so that the resulting part has the desired geometry. In the first category, the starting material is heated sufficiently to transform it into a liquid or highly plastic (semifluid) state. Nearly all materials can be processed in this way. Metals, ceramic glasses, and plastics can all be heated to sufficiently high temperatures to convert them into liquids. With the material in a liquid or semifluid form, it can be poured or otherwise forced to flow into a mold cavity and allowed to solidify, thus taking a solid shape that is the same as the cavity. Most processes that operate this way are called casting or molding. Casting is the name used for metals, and molding is the common term used for plastics. This category of shaping process is depicted in Figure 1.5. In particulate processing, the starting materials are powders of metals or ceramics. Although these two materials are quite different, the processes to shape them in particulate processing are quite similar. The common technique involves pressing and sintering, illustrated in Figure 1.6, in which the powders are first squeezed into a die cavity under high pressure and then heated to bond the individual particles together. In deformation processes, the starting workpart is shaped by the application of forces that exceed the yield strength of the material. For the material to be formed in this way, it must be sufficiently ductile to avoid fracture during deformation. To increase ductility (and for other reasons), the work material is often heated before forming to a temperature below the melting point. Deformation processes are associated most closely with metalworking and include operations such as forging and extrusion, shown in Figure 1.7. FIGURE 1.6 Particulate processing: (1) the starting material is powder; the usual process consists of (2) pressing and (3) sintering. E1C01 11/11/2009 14 13:31:35 Page 14 Chapter 1/Introduction and Overview of Manufacturing FIGURE 1.7 Some common deformation processes: (a) forging, in which two halves of a die squeeze the workpart, causing it to assume the shape of the die cavity; and (b) extrusion, in which a billet is forced to flow through a die orifice, thus taking the crosssectional shape of the orifice. Materialremovalprocessesareoperations thatremoveexcessmaterialfromthestarting workpiece so that the resulting shape is the desired geometry. The most important processes in this category are machining operations such as turning, drilling, and milling, shown in Figure 1.8. These cutting operations are most commonly applied to solid metals, performed using cutting tools that are harder and stronger than the work metal. Grinding is another common process in this category. Other material removal processes are known as nontraditional processes because they use lasers, electron beams, chemical erosion, electric discharges,andelectrochemicalenergytoremovematerialratherthancuttingorgrindingtools. It is desirable to minimize waste and scrap in converting a starting workpart into its subsequent geometry. Certain shaping processes are more efficient than others in terms of material conservation. Material removal processes (e.g., machining) tend to be wasteful of material, simply by the way they work. The material removed from the starting shape is waste, at least in terms of the unit operation. Other processes, such as certain casting and molding operations, often convert close to 100% of the starting material into final product. Manufacturing processes that transform nearly all of the starting material into product and require no subsequent machining to achieve final part geometry are called net shape processes. Other processes require minimum machining to produce the final shape and are called near net shape processes. Property-Enhancing Processes The second major type of part processing is performed to improve mechanical or physical properties of the work material. These processes do not alter the shape of the part, except unintentionally in some cases. The most important property-enhancing processes involve heat treatments, which include various annealing Workpiece Starting diameter Diameter Chip after turning Rotation Feed Rotation (work) Rotation Milling cutter Drill bit Material removed Work part Work Single point cutting tool Feed tool (a) Hole Feed (b) (c) FIGURE 1.8 Common machining operations: (a) turning, in which a single-point cutting tool removes metal from a rotating workpiece to reduce its diameter; (b) drilling, in which a rotating drill bit is fed into the work to create a round hole; and (c) milling, in which a workpart is fed past a rotating cutter with multiple edges. E1C01 11/11/2009 13:31:35 Page 15 Section 1.3/Manufacturing Processes 15 and strengthening processes for metals and glasses. Sintering of powdered metals and ceramics is also a heat treatment that strengthens a pressed powder metal workpart. Surface Processing Surface processing operations include (1) cleaning, (2) surface treatments, and (3) coating and thin film deposition processes. Cleaning includes both chemical and mechanical processes to remove dirt, oil, and other contaminants from the surface. Surface treatments include mechanical working such as shot peening and sand blasting, and physical processes such as diffusion and ion implantation. Coating and thin film deposition processes apply a coating of material to the exterior surface of the workpart. Common coating processes include electroplating, anodizing of aluminum, organic coating (call it painting), and porcelain enameling. Thin film deposition processes include physical vapor deposition and chemical vapor deposition to form extremely thin coatings of various substances. Several surface-processing operations have been adapted to fabricate semiconductor materials into integrated circuits for microelectronics. These processes include chemical vapor deposition, physical vapor deposition, and oxidation. They are applied to very localized areas on the surface of a thin wafer of silicon (or other semiconductor material) to create the microscopic circuit. 1.3.2 ASSEMBLY OPERATIONS The second basic type of manufacturing operation is assembly, in which two or more separate parts are joined to form a new entity. Components of the new entity are connected either permanently or semipermanently. Permanent joining processes include welding, brazing, soldering, and adhesive bonding. They form a joint between components that cannot be easily disconnected. Certain mechanical assembly methods are available to fasten two (or more) parts together in a joint that can be conveniently disassembled. The use of screws, bolts, and other threaded fasteners are important traditional methods in this category. Other mechanical assembly techniques form a more permanent connection; these include rivets, press fitting,and expansion fits. Special joining and fastening methods are used in the assembly of electronic products.Someofthemethods areidenticaltoorareadaptations oftheprecedingprocesses,for example, soldering. Electronics assembly is concerned primarily with the assembly of components such as integrated circuit packages to printed circuit boards to produce the complex circuits used in so many of today’s products. Joining and assembly processes are discussed in Part VIII, and the specialized assembly techniques for electronics are described in Part IX. 1.3.3 PRODUCTION MACHINES AND TOOLING Manufacturing operations are accomplished using machinery and tooling (and people). The extensive use of machinery in manufacturing began with the Industrial Revolution. It was at that time that metal cutting machines started to be developed and widely used. These were called machine tools—power-driven machines used to operate cutting tools previously operated by hand. Modern machine tools are described by the same basic definition, except that the power is electrical rather than water or steam, and the level of precision and automation is much greater today. Machine tools are among the most versatile of all production machines. They are used to make not only parts for consumer products, but also components for other production machines. Both in a historic and a reproductive sense, the machine tool is the mother of all machinery. Other production machines include presses for stamping operations, forge hammers for forging, rolling mills for rolling sheet metal, welding machines for welding, and insertion machines for inserting electronic components into printed circuit boards. The name of the equipment usually follows from the name of the process. E1C01 11/11/2009 16 13:31:36 Page 16 Chapter 1/Introduction and Overview of Manufacturing TABLE 1.3 Production equipment and tooling used for various manufacturing processes. Process Equipment Special Tooling (Function) Casting Molding Rolling Forging Extrusion Stamping Machining a Molding machine Rolling mill Forge hammer or press Press Press Machine tool Mold (cavity for molten metal) Mold (cavity for hot polymer) Roll (reduce work thickness) Die (squeeze work to shape) Extrusion die (reduce cross-section) Die (shearing, forming sheet metal) Cutting tool (material removal) Fixture (hold workpart) Jig (hold part and guide tool) Grinding Welding Grinding machine Welding machine Grinding wheel (material removal) Electrode (fusion of work metal) Fixture (hold parts during welding) a Various types of casting setups and equipment (Chapter 11). Production equipment can be general purpose or special purpose. General purpose equipment is more flexible and adaptable to a variety of jobs. It is commercially available for any manufacturing company to invest in. Special purpose equipment is usually designed to produce a specific part or product in very large quantities. The economics of mass production justify large investments in special purpose machinery to achieve high efficiencies and short cycle times. This is not the only reason for special purpose equipment, but it is the dominant one. Another reason may be because the process is unique and commercial equipment is not available. Some companies with unique processing requirements develop their own special purpose equipment. Production machinery usually requires tooling that customizes the equipment for the particular part or product. In many cases, the tooling must be designed specifically for the part or product configuration. When used with general purpose equipment, it is designed to be exchanged. For each workpart type, the tooling is fastened to the machine and the production run is made. When the run is completed, the tooling is changed for the next workpart type. When used with special purpose machines, the tooling is often designed as an integral part of the machine. Because the special purposemachine is likely being used for mass production, the tooling may never need changing except for replacement of worn components or for repair of worn surfaces. The type of tooling depends on the type of manufacturing process. Table 1.3 lists examples of special tooling used in various operations. Details are provided in the chapters that discuss these processes. 1.4 PRODUCTION SYSTEMS To operate effectively, a manufacturing firm must have systems that allow it to efficiently accomplish its type of production. Production systems consist of people, equipment, and procedures designed for the combination of materials and processes that constitute a firm’s manufacturing operations. Production systems can be divided into two categories: (1) production facilities and (2) manufacturing support systems, as shown in Figure 1.10. Production facilities refer to the physical equipment and the arrangement of equipment in the factory. Manufacturing support systems are the procedures used by the company to manage production and solve the technical and logistics problems encountered in ordering materials, moving work through the factory, and ensuring that products meet quality E1C01 11/11/2009 13:31:36 Page 17 Section 1.4/Production Systems 17 standards. Both categories include people. People make these systems work. In general, direct labor workers are responsible for operating the manufacturing equipment; and professional staff workers are responsible for manufacturing support. 1.4.1 PRODUCTION FACILITIES Production facilities consist of the factory and the production, material handling, and other equipment in the factory. The equipment comes in direct physical contact with the parts and/or assemblies as they are being made. The facilities ‘‘touch’’ the product. Facilities also include the way the equipment is arranged in the factory—the plant layout. The equipment is usually organized into logical groupings; which can be called manufacturing systems, such as an automated production line, or a machine cell consisting of an industrial robot and two machine tools. A manufacturing company attempts to design its manufacturing systems and organize its factories to serve the particular mission of each plant in the most efficient way. Over the years, certain types of production facilities have come to be recognized as the most appropriate way to organize for a given combination of product variety and production quantity, as discussed in Section 1.1.2. Different types of facilities are required for each of the three ranges of annual production quantities. Low-Quantity Production In the low-quantity range (1–100 units/year), the term job shop is often used to describe the type of production facility. A job shop makes low quantities of specialized and customized products. The products are typically complex, such as space capsules, prototype aircraft, and special machinery. The equipment in a job shop is general purpose, and the labor force is highly skilled. A job shop must be designed for maximum flexibility to deal with the wide product variations encountered (hard product variety). If the product is large and heavy, and therefore difficult to move, it typically remains in a single location during its fabrication or assembly. Workers and processing equipment are brought to the product, rather than moving the product to the equipment. This type of layout is referred to as a fixed-position layout, shown in Figure 1.9(a). In a pure situation, the product remains in a single location during its entire production. Examples of such products include ships, aircraft, locomotives, and heavy machinery. In actual practice, these items are usually built in large modules at single locations, and then the completed modules are brought together for final assembly using large-capacity cranes. The individual components of these large products are often made in factories in which the equipment is arranged according to function or type. This arrangement is called a process layout. The lathes are in one department, the milling machines are in another department, and so on, as in Figure 1.9(b). Different parts, each requiring a different operation sequence, are routed through the departments in the particular order needed for their processing, usually in batches. The process layout is noted for its flexibility; it can accommodate a great variety of operation sequences for different part configurations. Its disadvantage is that the machinery and methods to produce a part are not designed for high efficiency. Medium Quantity Production In the medium-quantity range (100–10,000 units annually), two different types of facility are distinguished, depending on product variety. When productvarietyis hard, theusual approach is batchproduction, inwhich abatch of one product is made, after which the manufacturing equipment is changed over to produce a batch of the next product, and so on. The production rate of the equipment is greater than the demand rate foranysingleproducttype,andsothesameequipmentcanbesharedamongmultipleproducts. The changeover between production runs takes time—time to change tooling and set up the machinery. This setup time is lost production time, and this is a disadvantage of batch manufacturing. Batch production is commonly used for make-to-stock situations, in which E1C01 11/11/2009 18 13:31:36 Page 18 Chapter 1/Introduction and Overview of Manufacturing Departments Work unit Production machines Product Equipment (modile) Workers (a) (b) Worker Workstation Equipment Conveyor v Cell Cell (c) Workers (d) FIGURE 1.9 Various types of plant layout: (a) fixed-position layout, (b) process layout, (c) cellular layout, and (d) product layout. items are manufactured to replenish inventory that has been gradually depleted by demand. The equipment is usually arranged in a process layout, as in Figure 1.9(b). An alternative approach to medium-range production is possible if product variety is soft. In this case, extensive changeovers between one product style and the next may not be necessary. It is often possible to configure the manufacturing system so that groups of similar products can be made on the same equipment without significant lost time because of setup. The processing or assembly of different parts or products is accomplished in cells consisting of several workstations or machines. The term cellular manufacturing is often associated with this type of production. Each cell is designed to produce a limited variety of part configurations; that is, the cell specializes in the production of a given set of similar parts, according to the principles of group technology (Section 39.5). The layout is called a cellular layout, depicted in Figure 1.9(c). High Production The high-quantity range (10,000 to millions of units per year) is referred to as mass production. The situation is characterized by a high demand rate for the product, and the manufacturing system is dedicated to the production of that single item. Two categories of mass production can be distinguished: quantity production and flow line production. Quantity production involves the mass production of single parts on single pieces of equipment. It typically involves standard machines (e.g., stamping presses) equipped with special tooling (e.g., dies and material handling devices), in effect dedicating the equipment to the production of one part type. Typical layouts used in quantity production are the process layout and cellular layout. Flow line production involves multiple pieces of equipment or workstations arranged in sequence, and the work units are physically moved through the sequence to complete the product. The workstations and equipment are designed specifically for the product to maximize efficiency. The layout is called a product layout, and the workstations are arranged E1C01 11/11/2009 13:31:36 Page 19 Section 1.4/Production Systems 19 into one long line, as in Figure 1.9(d), or into a series of connected line segments. The work is usually moved between stations by mechanized conveyor. At each station, a small amount of the total work is completed on each unit of product. The most familiar example of flow line production is the assembly line, associated with products such as cars and household appliances. The pure case of flow line production occurs when there is no variation in the products made on the line. Every product is identical, and the line is referred to as a single model production line. To successfully market a given product, it is often beneficial to introduce feature and model variations so that individual customers can choose the exact merchandise that appeals to them. From a production viewpoint, the feature differences represent a case of soft product variety. The term mixed-model production line applies to situations in which there is soft variety in the products made on the line. Modern automobile assembly is an example. Cars coming off the assembly line have variations in options and trim representing different models and in many cases different nameplates of the same basic car design. 1.4.2 MANUFACTURING SUPPORT SYSTEMS To operate its facilities efficiently, a company must organize itself to design the processes and equipment, plan and control the production orders, and satisfy product quality requirements. These functions are accomplished by manufacturing support systems— people and procedures by which a company manages its production operations. Most of these support systems do not directly contact the product, but they plan and control its progress through the factory. Manufacturing support functions are often carried out in the firm by people organized into departments such as the following: å Manufacturing engineering. The manufacturing engineering department is responsible for planning the manufacturing processes—deciding what processes should be used to make the parts and assemble the products. This department is also involved in designing and ordering the machine tools and other equipment used by the operating departments to accomplish processing and assembly. å Production planning and control. This department is responsible for solving the logistics problem in manufacturing—ordering materials and purchased parts, scheduling production, and making sure that the operating departments have the necessary capacity to meet the production schedules. å Quality control. Producing high-quality products should be a top priority of any manufacturing firm in today’s competitive environment. It means designing and Production system Manufacturing support Manufacturing support systems Quality control systems Manufacturing systems Facilities FIGURE 1.10 Overview of major topics covered in the book. Engineering materials Manufacturing processes and assembly operations Finished products E1C01 11/11/2009 20 13:31:36 Page 20 Chapter 1/Introduction and Overview of Manufacturing building products that conform to specifications and satisfy or exceed customer expectations. Much of this effort is the responsibility of the QC department. 1.5 TRENDS IN MANUFACTURING This section considers several trends that are affecting the materials, processes, and systems used in manufacturing. These trends are motivated by technological and economic factors occurring throughout the world. Their effects are not limited to manufacturing; they impact society as a whole. The discussion is organized into the following topic areas: (1) lean production and Six Sigma, (2) globalization, (3) environmentally conscious manufacturing, and (4) microfabrication and nanotechnology. 1.5.1 LEAN PRODUCTION AND SIX SIGMA These are two programs aimed at improving efficiency and quality in manufacturing. They address the demands by customers for the products they buy to be both low in cost and high in quality. The reason why lean and Six Sigma are trends is because they are being so widely adopted by companies, especially in the United States. Lean production is based on the Toyota Production System developed by Toyota Motors in Japan. Its origins date from the 1950s, when Toyota began using unconventional methods to improve quality, reduce inventories, and increase flexibility in its operations. Lean production can be defined simply as ‘‘doing more work with fewer resources.’’2 It means that fewer workers and less equipment are used to accomplish more production in less time, and yet achieve higher quality in the final product. The underlying objective of lean production is the elimination of waste. In the Toyota Production System, the seven forms of waste in production are (1) production of defective parts, (2) production of more parts than required, (3) excessive inventories, (4) unnecessary processing steps, (5) unnecessary movement of workers, (6) unnecessary movement and handling of materials, and (7) workers waiting. The methods used by Toyota to reduce waste include techniques for preventing errors, stopping a process when something goes wrong, improved equipment maintenance, involving workers in process improvements (so-called continuous improvement), and standardized work procedures. Probably the most important development was the just-in-time delivery system, which is described in Section 41.4 in the chapter on production and inventory control. Six Sigma was started in the 1980s at Motorola Corporation in the United States. The objective was to reduce variability in the company’s processes and products to increase customer satisfaction. Today, Six Sigma can be defined as ‘‘a quality-focused program that utilizes worker teams to accomplish projects aimed at improving an organization’s operational performance.’’3 Six Sigma is discussed in more detail in Section 42.4.2. 1.5.2 GLOBALIZATION AND OUTSOURCING The world is becoming more and more integrated, creating an international economy in which barriers once established by national boundaries have been reduced or eliminated. This has enabled a freer flow of goods and services, capital, technology, and people among regions and countries. Globalization is the term that describes this trend, which was recognized in the late 1980s and is now a dominant economic reality. Of interest here is that once underdeveloped 2 M. P. Groover, Work Systems and the Methods, Measurement, and Management of Work [7], p. 514. The term lean production was coined by researchers at the Massachusetts Institute of Technology who studied the production operations at Toyota and other automobile companies in the 1980s. 3 Ibid, p. 541. E1C01 11/11/2009 13:31:36 Page 21 Section 1.5/Trends in Manufacturing 21 nations such as China, India, and Mexico have developed their manufacturing infrastructures and technologies to a point where they are now important producers in the global economy. The advantages of these three countries in particular are their large populations (therefore, largeworkforcepool)andlowlaborcosts.Hourlywages arecurrentlyan order ofmagnitudeor more higher in the United States than in these countries, making it difficult for domestic U.S. companies to compete in many products requiring a high labor content. Examples include garments, furniture, many types of toys, and electronic gear. The result has been a loss of manufacturing jobs in the United States and a gain of related work to these countries. Globalization is closely related to outsourcing. In manufacturing, outsourcing refers to the use of outside contractors to perform work that was traditionally accomplished inhouse. Outsourcing can be done in several ways, including the use of local suppliers. In this case the jobs remain in the United States. Alternatively, U.S. companies can outsource to foreign countries, so that parts and products once made in the United States are now made outside the country. In this case U.S. jobs are displaced. Two possibilities can be distinguished: (1) offshore outsourcing, which refers to production in China or other overseas locations and transporting the items by cargo ship to the United States, and (2) near-shore outsourcing, which means the items are made in Canada, Mexico, or Central America and shipped by rail or truck into the United States. China is a country of particular interest in this discussion of globalization because of its fast-growing economy, the importance of manufacturing in that economy, and the extent to which U.S. companies have outsourced work to China. To take advantage of the low labor rates, U.S. companies have outsourced much of their production to China (and other east Asian countries). Despite the logistics problems and costs of shipping the goods back into the United States, the result has been lower costs and higher profits for the outsourcing companies, as well as lower prices and a wider variety of available products for U.S. consumers. The downside has been the loss of well-paying manufacturing jobs in the United States. Another consequence of U.S. outsourcing to China has been a reduction in the relative contribution of the manufacturing sector to GDP. In the 1990s, the manufacturing industries accounted for about 20% of GDP in the United States. Today that contribution is less than 15%. At the same time, the manufacturing sector in China has grown (along with the rest of its economy), now accounting for almost 35% of Chinese GDP. Because the U.S. GDP is roughly three times China’s, the United States’ manufacturing sector is still larger. However, China is the world leader in several industries. Its tonnage output of steel is greater than the combined outputs of the next six largest steel producing nations (in order, Japan, United States, Russia, India, South Korea, and Germany).4 China is also the largest producer of metal castings, accounting for more tonnage than the next three largest producers (in order, United States, Japan, and India) [5]. Steel production and casting are considered ‘‘dirty’’ industries, and environmental pollution is an issue not only in China, but in many places throughout the World. This issue is addressed in the next trend. 1.5.3 ENVIRONMENTALLY CONSCIOUS MANUFACTURING An inherent feature of virtually all manufacturing processes is waste (Section 1.3.1). The most obvious examples are material removal processes, in which chips are removed from a starting workpiece to create the desired part geometry. Waste in one form or another is a by-product of nearly all production operations. Another unavoidable aspect of manufacturing is that power is required to accomplish any given process. Generating that power requires fossil fuels (at least in the United States and China), the burning of which results in pollution of the environment. At the end of the manufacturing sequence, a product is created that is sold to a 4 Source: World Steel Association, 2008 data. E1C01 11/11/2009 22 13:31:36 Page 22 Chapter 1/Introduction and Overview of Manufacturing customer. Ultimately, the product wears out and is disposed of, perhaps in some landfill, with the associated environmental degradation. More and more attention is being paid by society to the environmental impact of human activities throughout the world and how modern civilization is using our natural resources at an unsustainable rate. Global warming is presently a major concern. The manufacturing industries contribute to these problems. Environmentally conscious manufacturing refers to programs that seek to determine the most efficient use of materials and natural resources in production, and minimize the negative consequences on the environment. Other associated terms for these programs include green manufacturing, cleaner production, and sustainable manufacturing. They all boil down to two basic approaches: (1) design products that minimize their environmental impact, and (2) design processes that are environmentally friendly. Product design is the logical starting point in environmentally conscious manufacturing. The term design for environment (DFE) is sometimes used for the techniques that attempt to consider environmental impact during product design prior to production. Considerations in DFE include the following: (1) select materials that require minimum energy to produce, (2) select processes that minimize waste of materials and energy, (3) design parts that can be recycled or reused, (4) design products that can be readily disassembled to recover the parts, (5) design products that minimize the use of hazardous and toxic materials, and (6) give attention to how the product will be disposed of at the end of its useful life. To a great degree, decisions made during design dictate the materials and processes that are used to make the product. These decisions limit the options available to the manufacturing departments to achieve sustainability. However, various approaches can be applied to make plant operations more environmentally friendly. They include the following: (1) adopt good housekeeping practices—keep the factory clean, (2) prevent pollutants from escaping into the environment (rivers and atmosphere), (3) minimize waste of materials in unit operations, (4) recycle rather than discard waste materials, (5) use net shape processes, (6) use renewable energy sources when feasible, (7) provide maintenance to production equipment so that it operates at maximum efficiency, and (8) invest in equipment that minimizes power requirements. Various topics related to environmentally conscious manufacturing are discussed in the text. The topics of polymer recycling and biodegradable plastics are covered in Section 8.5. Cutting fluid filtration and dry machining, which reduce the adverse effects of contaminated cutting fluids, are considered in Section 23.4.2. 1.5.4 MICROFABRICATION AND NANOTECHNOLOGY Another trend in manufacturingistheemergenceof materialsand products whosedimensions are sometimes so small that they cannot be seen by the naked eye. In extreme cases, the items cannot even be seen under an optical microscope. Products that are so miniaturized require special fabrication technologies. Microfabrication refers to the processes needed to make parts and products whose features sizes are in the micrometer range 1 mm ¼ 103 mm ¼ 106 mÞ. Examples include ink-jet printing heads, compact discs (CDs and DVDs), and microsensors used in automotive applications (e.g., air-bag deployment sensors). Nanotechnology refers to materials and products whose feature sizes are in the nanometer scale 1 nm ¼ 103 mm ¼ 106 mm ¼ 109 m , a scale that approaches the size of atoms and molecules. Ultra-thin coatingsfor catalytic converters, flatscreen TVmonitors,and cancer drugs are examples of products based on nanotechnology. Microscopic and nanoscopic materials and products are expected to increase in importance in the future, both technologically and economically, and processes are needed to produce them commercially. The purpose here is to make the reader aware of this trend toward miniaturization. Chapters 36 and 37 are devoted to these technologies. E1C01 11/11/2009 13:31:36 Page 23 References 1.6 23 ORGANIZATION OF THE BOOK The preceding sections provide an overview of the book. The remaining 41 chapters are organized into 11 parts. The block diagram in previous Figure 1.10 summarizes the major topics that are covered. It shows the production system (outlined in dashed lines) with engineering materials entering from the left and finished products exiting at the right. Part I, Material Properties and Product Attributes, consists of four chapters that describe the important characteristics and specifications of materials and the products made from them. Part II discusses the four basic engineering materials: metals, ceramics, polymers, and composites. The largest block in Figure 1.10 is labeled ‘‘Manufacturing processes and assembly operations.’’ The processes and operations included in the text are those identified in Figure 1.4. Part III begins the coverage of the four categories of shaping processes. Part III consists of six chapters on the solidification processes that include casting of metals, glassworking, and polymer shaping. In Part IV, the particulate processing of metals and ceramics is covered in two chapters. Part V deals with metal deformation processes such as rolling, forging, extrusion, and sheet metalworking. Finally, Part VI discusses the material removal processes. Four chapters are devoted to machining, and two chapters cover grinding (and related abrasive processes) and the nontraditional material removal technologies. The other types of processing operations, property enhancing and surface processing, are covered in two chapters in Part VII. Property enhancing is accomplished by heat treatment, and surface processing includes operations such as cleaning, electroplating, and coating (painting). Joining and assembly processes are considered in Part VIII, which is organized into four chapters on welding, brazing, soldering, adhesive bonding, and mechanical assembly. Several unique processes that do not neatly fit into the classification scheme of Figure 1.4 are covered in Part IX, Special Processing and Assembly Technologies. Its five chapters cover rapid prototyping, processing of integrated circuits, electronics, microfabrication, and nanofabrication. The remaining blocks in Figure 1.10 deal with the systems of production. Part X, ‘‘Manufacturing Systems,’’ covers the major systems technologies and equipment groupings located in the factory: numerical control, industrial robotics, group technology, cellular manufacturing, flexible manufacturing systems, and production lines. Finally, Part XI deals with manufacturing support systems: manufacturing engineering, production planning and control, and quality control and inspection. REFERENCES [1] Black, J., and Kohser, R. DeGarmo’s Materials and Processes in Manufacturing, 10th ed. John Wiley & Sons, Hoboken, New Jersey, 2008. [2] Emerson, H. P., and Naehring, D. C. E. Origins of Industrial Engineering. Industrial Engineering & Management Press, Institute of Industrial Engineers, Norcross, Georgia, 1988. [3] Flinn, R. A., and Trojan, P. K. Engineering Materials and Their Applications, 5th ed. John Wiley & Sons, New York, 1995. [4] Garrison, E. A History of Engineering and Technology. CRC Taylor & Francis, Boca Raton, Florida, 1991. [5] Gray, A.‘‘Global Automotive Metal Casting,’’ Advanced Materials & Processes, April 2009, pp. 33– 35. [6] Groover, M. P. Automation, Production Systems, and Computer Integrated Manufacturing, 3rd ed. Pearson Prentice-Hall, Upper Saddle River, New Jersey, 2008. [7] Groover, M. P. Work Systems and the Methods, Measurement, and Management of Work, Pearson Prentice-Hall, Upper Saddle River, New Jersey, 2007. [8] Hornyak, G. L., Moore, J. J., Tibbals, H. F., and Dutta, J., Fundamentals of Nanotechnology, CRC Taylor & Francis, Boca Raton, Florida, 2009. E1C01 11/11/2009 24 13:31:37 Page 24 Chapter 1/Introduction and Overview of Manufacturing [9] Hounshell, D. A. From the American System to Mass Production, 1800–1932. The Johns Hopkins University Press, Baltimore, Maryland, 1984. [10] Kalpakjian, S., and Schmid S. R. Manufacturing Processes for Engineering Materials, 6th ed. Pearson Prentice Hall, Upper Saddle River, New Jersey, 2010. [11] wikipedia.org/wiki/globalization [12] www.bsdglobal.com/tools REVIEW QUESTIONS 1.1. What are the differences among primary, secondary, and tertiary industries? Give an example of each category. 1.2. What is a capital good? Provide an example. 1.3. How are product variety and production quantity related when comparing typical factories? 1.4. Define manufacturing capability. 1.5. Name the three basic categories of materials. 1.6. How does a shaping process differ from a surface processing operation? 1.7. What are two subclasses of assembly processes? Provide an example process for each subclass. 1.8. Define batch production and describe why it is often used for medium-quantity production products. 1.9. What is the difference between a process layout and a product layout in a production facility? 1.10. Name two departments that are typically classified as manufacturing support departments. MULTIPLE CHOICE QUIZ There are 18 correct answers in the following multiple choice questions (some questions have multiple answers that are correct). To attain a perfect score on the quiz, all correct answers must be given. Each correct answer is worth 1 point. Each omitted answer or wrong answer reduces the score by 1 point, and each additional answer beyond the correct number of answers reduces the score by 1 point. Percentage score on the quiz is based on the total number of correct answers. 1.1. Which of the following industries are classified as secondary industries (three correct answers): (a) beverages (b) financial services, (c) fishing, (d) mining, (e) power utilities, (f) publishing, and, (g) transportation? 1.2. Mining is classified in which one of the following industry categories: (a) agricultural industry, (b) manufacturing industry, (c) primary industry, (d) secondary industry, (e) service industry, or, (f) tertiary industry? 1.3. Inventions of the Industrial Revolution include which one of the following: (a) automobile, (b) cannon, (c) printing press, (d) steam engine, or, (e) sword? 1.4. Ferrous metals include which of the following (two correct answers): (a) aluminum, (b) cast iron, (c) copper, (d) gold, and, (e) steel? 1.5. Which one of the following engineering materials is defined as a compound containing metallic and nonmetallic elements: (a) ceramic, (b) composite, (c) metal, or, (d) polymer? 1.6. Which of the following processes start with a material that is in a fluid or semifluid state and solidifies the material in a cavity (two best answers): (a) casting, (b) forging, (c) machining, (d) molding, (e) pressing, and, (f) turning? 1.7. Particulate processing of metals and ceramics involves which of the following steps (two best answers): (a) adhesive bonding, (b) deformation, (c) forging, (d) material removal, (e) melting, (f) pressing, and, (g) sintering? 1.8. Deformation processes include which of the following (two correct answers): (a) casting, (b) drilling, (c) extrusion, (d) forging, (e) milling, (f) painting, and, (g) sintering? 1.9. Which one of the following is a machine used to perform extrusion: (a) forge hammer, (b) milling machine, (c) rolling mill, (d) press, (e) torch? 1.10. High-volume production of assembled products is most closely associated with which one of the following layout types: (a) cellular layout, (b) fixed position layout, (c) process layout, or, (d) product layout? 1.11. A production planning and control department accomplishes which of the following functions in its role of providing manufacturing support (two best answers): (a) designs and orders machine tools, (b) develops corporate strategic plans, (c) orders materials and purchased parts, (d) performs quality inspections, and, (e) schedules the order of products on a machine? E1C02 11/02/2009 14:15:23 Page 25 Part I Material Properties and Product Attributes 2 THE NATURE OF MATERIALS Chapter Contents 2.1 Atomic Structure and the Elements 2.2 Bonding between Atoms and Molecules 2.3 Crystalline Structures 2.3.1 Types of Crystal Structures 2.3.2 Imperfections in Crystals 2.3.3 Deformation in Metallic Crystals 2.3.4 Grains and Grain Boundaries in Metals 2.4 Noncrystalline (Amorphous) Structures 2.5 Engineering Materials An understanding of materials is fundamental in the study of manufacturing processes. In Chapter 1, manufacturing was defined as a transformation process. It is the material that is transformed; and it is the behavior of the material when subjected to the particular forces, temperatures, and other physical parameters of the process that determines the success of the operation. Certain materials respond well to certain types of manufacturing processes, and poorly or not at all to others. What are the characteristics and properties of materials that determine their capacity to be transformed by the different processes? Part I of this book consists of four chapters that address this question. The current chapter considers the atomic structure of matter and the bonding between atoms and molecules. It also shows how atoms and molecules in engineering materials organize themselves into two structural forms: crystalline and noncrystalline. It turns out that the basic engineering materials—metals,ceramics,and polymers—can existineither form, although a preference for a particular form is usually exhibited by a given material. For example, metals almost always exist as crystals in their solid state. Glass (e.g., window glass), a ceramic, assumes a noncrystalline form. Some polymers are mixtures of crystalline and amorphous structures. Chapters 3 and 4 discuss the mechanical and physical properties that are relevant in manufacturing. Of course, these properties are also important in product design. Chapter 5 is concerned with several part and product attributes that are specified during product design and must be achieved in 25 E1C02 11/02/2009 26 14:15:24 Page 26 Chapter 2/The Nature of Materials manufacturing: dimensions, tolerances, and surface finish. Chapter 5 also describes how these attributes are measured. 2.1 ATOMIC STRUCTURE AND THE ELEMENTS The basic structural unit of matter is the atom. Each atom is composed of a positively charged nucleus, surrounded by a sufficient number of negatively charged electrons so that the charges are balanced. The number of electrons identifies the atomic number and the element of the atom. There are slightly more than 100 elements (not counting a few extras that have been artificially synthesized), and these elements are the chemical building blocks of all matter. Just as there are differences among the elements, there are also similarities. The elements can be grouped into families and relationships established between and within the families by means of the Periodic Table, shown in Figure 2.1. In the horizontal direction there is a certain repetition, or periodicity, in the arrangement of elements. Metallic elements occupy the left and center portions of the chart, and nonmetals are located to the right. Between them, along a diagonal, is a transition zone containing elements called metalloids or semimetals. In principle, each of the elements can exist as a solid, liquid, or gas, depending on temperature and pressure. At room temperature and atmospheric pressure, they each have a natural phase; e.g., iron (Fe) is a solid, mercury (Hg) is a liquid, and nitrogen (N) is a gas. In the table, the elements are arranged into vertical columns and horizontal rows in such a way that similarities exist among elements in the same columns. For example, in the extreme right column are the noble gases (helium, neon, argon, krypton, xenon, and radon), all of which exhibit great chemical stability and low reaction rates. The halogens (fluorine, chlorine, bromine, iodine, and astatine) in column VIIA share similar properties (hydrogen is not included among the halogens). The noble metals (copper, silver, and gold) in column IB have similar properties. Generally there are correlations in properties among elements within a given column, whereas differences exist among elements in different columns. FIGURE 2.1 Periodic Table of Elements. The atomic number and symbol are listed for the 103 elements. E1C02 11/02/2009 14:15:24 Page 27 Section 2.1/Atomic Structure and the Elements 27 FIGURE 2.2 Simple model of atomic structure for several elements: (a) hydrogen, (b) helium, (c) fluorine, (d) neon, and (e) sodium. Many of the similarities and differences among the elements can be explained by their respective atomic structures. The simplest model of atomic structure, called the planetary model, shows the electrons of the atom orbiting around the nucleus at certain fixed distances, called shells, as shown in Figure 2.2. The hydrogen atom (atomic number 1) has one electron in theorbitclosest to thenucleus. Helium (atomicnumber 2) has two. Also shown in the figure are the atomic structures for fluorine (atomic number 9), neon (atomic number 10), and sodium (atomic number 11). One might infer from these models that there is a maximum number of electronsthatcanbecontainedinagivenorbit.This turnsouttobecorrect,andthemaximumis defined by Maximum number of electrons in an orbit ¼ 2n2 ð2:1Þ where n identifies the orbit, with n ¼ 1 closest to the nucleus. Thenumberofelectronsintheoutermostshell,relativetothemaximumnumberallowed, determines to a large extent the atom’s chemical affinity for other atoms. These outer-shell electrons are called valence electrons. For example, because a hydrogen atom has only one electron in its single orbit, it readily combines with another hydrogen atom to form a hydrogen molecule H2. For the same reason, hydrogen also reacts readily with various other elements (e.g., to form H2O). In the helium atom, the two electrons in its only orbit are the maximum allowed (2n2 ¼ 2(1)2 ¼ 2), and so helium is very stable. Neon is stable for the same reason: Its outermost orbit (n ¼ 2) has eight electrons (the maximum allowed), so neon is an inert gas. In contrast to neon, fluorine has one fewer electron in its outer shell (n ¼ 2) than the maximum allowed and is readily attracted to other elements that might share an electron to make a more stable set. The sodium atom seems divinely made for the situation, with one electron in its outermost orbit. It reacts strongly with fluorine to form the compound sodium fluoride, as pictured in Figure 2.3. FIGURE 2.3 The sodium fluoride molecule, formed by the transfer of the ‘‘extra’’ electron of the sodium atom to complete the outer orbit of the fluorine atom. E1C02 11/02/2009 28 14:15:24 Page 28 Chapter 2/The Nature of Materials At the low atomic numbers considered here, the prediction of the number of electrons in the outer orbit is straightforward. As the atomic number increases to higher levels, the allocation of electrons to the different orbits becomes somewhat more complicated. There are rules and guidelines, based on quantum mechanics, that can be used to predict the positions of the electrons among the various orbits and explain their characteristics. A discussion of these rules is somewhat beyond the scope of the coverage of materials for manufacturing. 2.2 BONDING BETWEEN ATOMS AND MOLECULES Atoms are held together in molecules by various types of bonds that depend on the valence electrons. By comparison, molecules are attracted to each other by weaker bonds, which generally result from the electron configuration in the individual molecules. Thus, we have two types of bonding: (1) primary bonds, generally associated with the formation of molecules; and (2) secondary bonds, generally associated with attraction between molecules. Primary bonds are much stronger than secondary bonds. Primary Bonds Primary bonds are characterized by strong atom-to-atom attractions that involve the exchange of valence electrons. Primary bonds include the following forms: (a) ionic, (b) covalent, and (c) metallic, as illustrated in Figure 2.4. Ionic and covalent bonds are called intramolecular bonds because they involve attractive forces between atoms within the molecule. In the ionic bond, the atoms of one element give up their outer electron(s), which are in turn attracted to the atoms of some other element to increase their electron count in the outermost shell to eight. In general, eight electrons in the outer shell is the most stable atomic configuration (except for the very light atoms), and nature provides a very strong bond between atoms that achieves this configuration. The previous example of the reaction of sodium and fluorine to form sodium fluoride (Figure 2.3) illustrates this form of atomic bond. Sodium chloride (table salt) is a more common example. Because of the transfer of electrons between the atoms, sodium and fluorine (or sodium and chlorine) ions are formed, from which this bonding derives its name. Properties of solid materials with ionic bonding include low electrical conductivity and poor ductility. The covalent bond is one in which electrons are shared (as opposed to transferred) between atoms in their outermost shells to achieve a stable set of eight. Fluorine and diamond are two examples of covalent bonds. In fluorine, one electron from each of two atoms is shared to form F2 gas, as shown in Figure 2.5(a). In the case of diamond, which is carbon (atomic number 6), each atom has four neighbors with which it shares electrons. This produces a very rigid three-dimensional structure, not adequately represented in Figure 2.5(b), and accounts for the extreme high hardness of this material. Other forms of FIGURE 2.4 Three forms of primary bonding: (a) ionic, (b) covalent, and (c) metallic. E1C02 11/02/2009 14:15:24 Page 29 Section 2.2/Bonding Between Atoms and Molecules 29 FIGURE 2.5 Two examples of covalent bonding: (a) fluorine gas F2, and (b) diamond. carbon (e.g., graphite) do not exhibit this rigid atomic structure. Solids with covalent bonding generally possess high hardness and low electrical conductivity. The metallic bond is, of course, the atomic bonding mechanism in pure metals and metal alloys. Atoms of the metallic elements generally possess too few electrons in their outermost orbits to complete the outer shells for all of the atoms in, say, a given block of metal. Accordingly, instead of sharing on an atom-to-atom basis, metallic bonding involves the sharing of outer-shell electrons by all atoms to form a general electron cloud that permeates the entire block. This cloud provides the attractive forces to hold the atoms together and forms a strong, rigid structure in most cases. Because of the general sharing of electrons, and their freedom to move within the metal, metallic bonding provides for good electrical conductivity. Other typical properties of materials characterized by metallic bonding include good conduction of heat and good ductility. (Although some of these terms are yet to be defined, the text relies on the reader’s general understanding of material properties.) Secondary Bonds Whereas primary bonds involve atom-to-atom attractive forces, secondary bonds involve attraction forces between molecules, or intermolecular forces. There is no transfer or sharing of electrons in secondary bonding, and these bonds are therefore weaker than primary bonds. There are three forms of secondary bonding: (a) dipole forces, (b) London forces, and (c) hydrogen bonding, illustrated in Figure 2.6. Types (a) and (b) are often referred to as van der Waals forces, after the scientist who first studied and quantified them. Dipole forces arise in a molecule comprised of two atoms that have equal and opposite electrical charges. Each molecule therefore forms a dipole, as shown in Figure 2.6(a) for hydrogen chloride. Although the material is electrically neutral in its aggregate form, on a molecular scale the individual dipoles attract each other, given the proper orientation of positive and negative ends of the molecules. These dipole forces provide a net intermolecular bonding within the material. London forces involve attractive forces between nonpolar molecules; that is, the atoms in the molecule do not form dipoles in the sense of the preceding paragraph. However, owing to the rapid motion of the electrons in orbit around the molecule, temporary dipoles form when more electrons happen to be on one side of the molecule than the other, as suggested by FIGURE 2.6 Three types of secondary bonding: (a) dipole forces, (b) London forces, and (c) hydrogen bonding. E1C02 11/02/2009 30 14:15:25 Page 30 Chapter 2/The Nature of Materials Figure 2.6(b). These instantaneous dipoles provide a force of attraction between molecules in the material. Finally, hydrogen bonding occurs in molecules containing hydrogen atoms that are covalently bonded to another atom (e.g., oxygen in H2O). Because the electrons needed to complete the shell of the hydrogen atom are aligned on one side of its nucleus, the opposite side has a net positive charge that attracts the electrons of atoms in neighboring molecules. Hydrogen bonding is illustrated in Figure 2.6(c) for water, and is generally a stronger intermolecular bonding mechanism than the other two forms of secondary bonding. It is important in the formation of many polymers. 2.3 CRYSTALLINE STRUCTURES Atoms and molecules are used as building blocks for the more macroscopic structure of matter that is considered here and in the following section. When materials solidify from the molten state, they tend to close ranks and pack tightly, in many cases arranging themselves into a very orderly structure, and in other cases, not quite so orderly. Two fundamentally different material structures can be distinguished:(1) crystalline and (2) noncrystalline. Crystalline structures are examined in this section, and noncrystalline in the next. The video clip on heat treatment shows how metals naturally form into crystal structures. VIDEO CLIP Heat treatment: View the segment titled ‘‘metal and alloy structures.’’ Many materials form into crystals on solidification from the molten or liquid state. It is characteristic of virtually all metals, as well as many ceramics and polymers. A crystalline structure is one in which the atoms are located at regular and recurring positions in three dimensions. The pattern may be replicated millions of times within a given crystal. The structure can be viewed in the form of a unit cell, which is the basic geometric grouping of atoms that is repeated. To illustrate, consider the unit cell for the body-centered cubic (BCC) crystal structure shown in Figure 2.7, one of the common structures found in metals. The simplest model of the BCC unit cell is illustrated in Figure 2.7(a). Although this model clearly depicts the locations of the atoms within the cell, it does not indicate the close packing of the atoms that occurs in the real crystal, as in Figure 2.7(b). Figure 2.7(c) shows the repeating nature of the unit cell within the crystal. FIGURE 2.7 Body-centered cubic (BCC) crystal structure: (a) unit cell, with atoms indicated as point locations in a three-dimensional axis system; (b) unit cell model showing closely packed atoms (sometimes called the hard-ball model); and (c) repeated pattern of the BCC structure. E1C02 11/02/2009 14:15:26 Page 31 Section 2.3/Crystalline Structures 31 FIGURE 2.8 Three types of crystal structures in metals: (a) body-centered cubic, (b) face-centered cubic, and (c) hexagonal close-packed. 2.3.1 TYPES OF CRYSTAL STRUCTURES In metals, three lattice structures are common: (1) body-centered cubic (BCC), (2) facecentered cubic (FCC), and (3) hexagonal close-packed (HCP), illustrated in Figure 2.8. Crystal structures for the common metals are presented in Table 2.1. Note that some metals undergo a change of structure at different temperatures. Iron, for example, is BCC at room temperature; it changes to FCC above 912 C (1674 F) and back to BCC at temperatures above 1400 C (2550 F). When a metal (or other material) changes structure like this, it is referred to as being allotropic. 2.3.2 IMPERFECTIONS IN CRYSTALS Thus far, crystal structures have been discussed as if they were perfect—the unit cell repeated in the material over and over in all directions. A perfect crystal is sometimes desirable to satisfy aesthetic or engineering purposes. For instance, a perfect diamond (contains no flaws) is more valuable than one containing imperfections. In the production of integrated circuit chips, large single crystals of silicon possess desirable processing characteristics for forming the microscopic details of the circuit pattern. However, there are various reasons why a crystal’s lattice structure may not be perfect. The imperfections often arise naturally because of the inability of the solidifying material to continue the replication of the unit cell indefinitely without interruption. Grain boundaries in metals are an example. In other cases, the imperfections are introduced purposely during the TABLE 2.1 Crystal structures for the common metals (at room temperature). Body-Centered Cubic (BCC) Chromium (Cr) Iron (Fe) Molybdenum (Mo) Tantalum (Ta) Tungsten (W) Face-Centered Cubic (FCC) Aluminum (Al) Copper (Cu) Gold (Au) Lead (Pb) Silver (Ag) Nickel (Ni) Hexagonal Close-Packed (HCP) Magnesium (Mg) Titanium (Ti) Zinc (Zn) E1C02 11/02/2009 32 14:15:26 Page 32 Chapter 2/The Nature of Materials FIGURE 2.9 Point defects: (a) vacancy, (b) ion-pair vacancy, (c) interstitialcy, and (d) displaced ion. manufacturing process; for example, the addition of an alloying ingredient in a metal to increase its strength. The various imperfections in crystalline solids are also called defects. Either term, imperfection or defect, refers to deviations in the regular pattern of the crystalline lattice structure. They can be catalogued as (1) point defects, (2) line defects, and (3) surface defects. Point defects are imperfections in the crystal structure involving either a single atom or a few atoms. The defects can take various forms including, as shown in Figure 2.9: (a) vacancy, the simplest defect, involving a missing atom within the lattice structure; (b) ion-pair vacancy, also called a Schottky defect, which involves a missing pair of ions of opposite charge in a compound that has an overall charge balance; (c) interstitialcy, a lattice distortion produced by the presence of an extra atom in the structure; and (d) displaced ion, known as a Frenkel defect, which occurs when an ion becomes removed from a regular position in the lattice structure and inserted into an interstitial position not normally occupied by such an ion. A line defect is a connected group of point defects that forms a line in the lattice structure. The most important line defect is the dislocation, which can take two forms: (a) edge dislocation and (b) screw dislocation. An edge dislocation is the edge of an extra plane of atoms that exists in the lattice, as illustrated in Figure 2.10(a). A screw dislocation, Figure 2.10(b), is a spiral within the lattice structure wrapped around an imperfection line, like a screw is wrapped around its axis. Both types of dislocations can arise in the crystal structure during solidification (e.g., casting), or they can be initiated during a FIGURE 2.10 Line defects: (a) edge dislocation and (b) screw dislocation. (a) (b) E1C02 11/02/2009 14:15:26 Page 33 Section 2.3/Crystalline Structures 33 deformation process (e.g., metal forming) performed on the solid material. Dislocations are useful in explaining certain aspects of mechanical behavior in metals. Surface defects are imperfections that extend in two directions to form a boundary. The most obvious example is the external surface of a crystalline object that defines its shape. The surface is an interruption in the lattice structure. Surface boundaries can also lie inside the material. Grain boundaries are the best example of these internal surface interruptions. Metallic grains are discussed in a moment, but first consider how deformation occurs in a crystal lattice, and how the process is aided by the presence of dislocations. 2.3.3 DEFORMATION IN METALLIC CRYSTALS When a crystal is subjected to a gradually increasing mechanical stress, its initial response is to deform elastically. This can be likened to a tilting of the lattice structure without any changes of position among the atoms in the lattice, in the manner depicted in Figure 2.11(a) and (b). If the force is removed, the lattice structure (and therefore the crystal) returns to its original shape. If the stress reaches a high value relative to the electrostatic forces holding the atoms in their lattice positions, a permanent shape change occurs, called plastic deformation. What has happened is that the atoms in the lattice have permanently moved from their previous locations, and a new equilibrium lattice has been formed, as suggested by Figure 2.11(c). The lattice deformation shown in (c) of the figure is one possible mechanism, called slip, by which plastic deformation can occur in a crystalline structure. The other is called twinning, discussed later. Slip involves the relative movement of atoms on opposite sides of a plane in the lattice, called the slip plane. The slip plane must be somehow aligned with the lattice structure (as indicated in the sketch), and so there are certain preferred directions along which slip is more likely to occur. The number of these slip directions depends on the lattice type. The three common metal crystal structures are somewhat more complicated, especially in three dimensions, than the square lattice depicted in Figure 2.11. It turns out that HCP has the fewest slip directions, BCC the most, and FCC falls in between. HCP metals show poor ductility and are generally difficult to deform at room temperature. Metals with BCC structure would figure to have the highest ductility, if the number of slip directions were the only criterion. However, nature is not so simple. These metals are generally stronger than the others, which complicates the issue; and the BCC metals usually require higher stresses to cause slip. In fact, some of the BCC metals exhibit poor ductility. Low carbon steel is a notable exception; although relatively strong, it is widely used with great commercial success in sheetmetal-forming operations, in which it exhibits good ductility. The FCC metals are generally the most ductile of the three crystal structures, combining a good number of slip directions with (usually) relatively low to moderate strength. All three of these metal structures become more ductile at elevated temperatures, and this fact is often exploited in shaping them. Dislocations play an important role in facilitating slip in metals. When a lattice structure containing an edge dislocation is subjected to a shear stress, the material deforms FIGURE 2.11 Deformation of a crystal structure: (a) original lattice; (b) elastic deformation,withnopermanent change in positions of atoms; and (c) plastic deformation, in which atoms in the lattice are forced to move to new ‘‘homes.’’ E1C02 11/02/2009 34 14:15:26 Page 34 Chapter 2/The Nature of Materials FIGURE 2.12 Effect of dislocations in the lattice structure under stress. In the series of diagrams, the movement of the dislocation allows deformation to occur under a lower stress than in a perfect lattice. much more readily than in a perfect structure. This is explained by the fact that the dislocation is put into motion within the crystal lattice in the presence of the stress, as shown in the series of sketches in Figure 2.12. Why is it easier to move a dislocation through the lattice than it is to deform the lattice itself? The answer is that the atoms at the edge dislocation require a smaller displacement within the distorted lattice structure to reach a new equilibrium position. Thus, a lower energy level is needed to realign the atoms into the new positions than if the lattice were missing the dislocation. A lower stress level is therefore required to effect the deformation. Because the new position manifests a similar distorted lattice, movement of atoms at the dislocation continues at the lower stress level. The slip phenomenon and the influence of dislocations have been explained here on a very microscopic basis. On a larger scale, slip occurs many times over throughout the metal when subjected to a deforming load, thus causing it to exhibit the familiar macroscopic behavior. Dislocations represent a good-news–bad-news situation. Because of dislocations, the metal is more ductile and yields more readily to plastic deformation (forming) during manufacturing. However, from a product design viewpoint, the metal is not nearly as strong as it would be in the absence of dislocations. Twinning is a second way in which metal crystals plastically deform. Twinning can be defined as a mechanism of plastic deformation in which atoms on one side of a plane (called the twinning plane) are shifted to form a mirror image of the other side of the plane. It is illustrated in Figure 2.13. The mechanism is important in HCP metals (e.g., magnesium, zinc) FIGURE 2.13 Twinning involves the formation of an atomic mirror image (i.e., a ‘‘twin’’) on the opposite side of the twinning plane: (a) before, and (b) after twinning. (a) (b) E1C02 11/02/2009 14:15:26 Page 35 Section 2.4/Noncrystalline (Amorphous) Structures 35 because they do not slip readily. Besides structure, another factor in twinning is the rate of deformation. The slip mechanism requires more time than twinning, which can occur almost instantaneously. Thus, in situations in which the deformation rate is very high, metals twin that would otherwise slip. Low carbon steel is an example that illustrates this rate sensitivity; when subjected to high strain rates it twins, whereas at moderate rates it deforms by slip. 2.3.4 GRAINS AND GRAIN BOUNDARIES IN METALS A given block of metal may contain millions of individual crystals, called grains. Each grain has its own unique lattice orientation; but collectively, the grains are randomly oriented within the block. Such a structure is referred to as polycrystalline. It is easy to understand how such a structureis thenaturalstateofthematerial.When theblockiscooled from themolten stateand begins to solidify, nucleation of individual crystals occurs at random positions and orientations throughout the liquid. As these crystals grow they finally interfere with each other, forming at their interface a surface defect—a grain boundary. The grain boundary consists of a transition zone, perhaps only a few atoms thick, in which the atoms are not aligned with either grain. The size of the grains in the metal block is determined by the number of nucleation sites in the molten material and the cooling rate of the mass, among other factors. In a casting process, the nucleation sites are often created by the relatively cold walls of the mold, which motivate a somewhat preferred grain orientation at these walls. Grain size is inversely related to cooling rate: Faster cooling promotes smaller grain size, whereas slower cooling has the opposite effect. Grain size is important in metals because it affects mechanical properties. Smaller grain size is generally preferable from a design viewpoint because it means higher strength and hardness. It is also desirable in certain manufacturing operations (e.g., metal forming), because it means higher ductility during deformation and a better surface on the finished product. Another factor influencing mechanical properties is the presence of grain boundaries in the metal. They represent imperfections in the crystalline structure that interrupt the continued movement of dislocations. This helps to explain why smaller grain size— therefore more grains and more grain boundaries—increases the strength of the metal. By interfering with dislocation movement, grain boundaries also contribute to the characteristic property of a metal to become stronger as it is deformed. The property is called strain hardening, and it is examined more closely in the discussion of mechanical properties in Chapter 3. 2.4 NONCRYSTALLINE (AMORPHOUS) STRUCTURES Many important materials are noncrystalline—liquids and gases, for example. Water and air have noncrystalline structures. A metal loses its crystalline structure when it is melted. Mercury is a liquid metal at room temperature, with its melting point of 38 C (37 F). Important classes of engineering materials have a noncrystalline form in their solid state; the term amorphous is often used to describe these materials. Glass, many plastics, and rubber fall into this category. Many important plastics are mixtures of crystalline and noncrystalline forms. Even metals can be amorphous rather than crystalline, given that the cooling rate during transformation from liquid to solid is fast enough to inhibit the atoms from arranging themselves into their preferred regular patterns. This can happen, for instance, if the molten metal is poured between cold, closely spaced, rotating rolls. Two closely related features distinguish noncrystalline from crystalline materials: (1) absence of a long-range order in the molecular structure, and (2) differences in melting and thermal expansion characteristics. E1C02 11/02/2009 36 14:15:26 Page 36 Chapter 2/The Nature of Materials FIGURE 2.14 Illustration of difference in structure between: (a) crystalline and (b) noncrystalline materials. The crystal structure is regular, repeating, and denser, whereas the noncrystalline structure is more loosely packed and random. The difference in molecular structure can be visualized with reference to Figure 2.14. The closely packed and repeating pattern of the crystal structure is shown on the left; and the less dense and random arrangement of atoms in the noncrystalline material on the right. The difference is demonstrated by a metal when it melts. The more loosely packed atoms in the molten metal show an increase in volume (reduction in density) compared with the material’s solid crystalline state. This effect is characteristic of most materials when melted. (Ice is a notable exception; liquid water is denser than solid ice.) It is a general characteristic of liquids and solid amorphous materials that they are absent of long-range order as on the right in our figure. The melting phenomenon will now be examined in more detail, and in doing so, the second important difference between crystalline and noncrystalline structures will be defined. As indicated, a metal experiences an increase in volume when it melts from the solid to the liquid state. For a pure metal, this volumetric change occurs rather abruptly, at a constant temperature (i.e., the melting temperature Tm), as indicated in Figure 2.15. The change represents a discontinuity from the slopes on either side in the plot. The gradual slopes characterize the metal’s thermal expansion—the change in volume as a function of temperature, which isusually different in the solid and liquid states. Associated with the sudden volume increase as the metal transforms from solid to liquid at the melting point is the addition of a certain quantity of heat, called the heat of fusion, which causes the atoms to lose the dense, regular arrangement of the crystalline structure. The process is reversible; it operates in both directions. If the molten metal is cooled through its melting temperature, the same abrupt change in volume occurs (except that it is a decrease), and the same quantity of heat is given off by the metal. An amorphous material exhibits quite different behavior than that of a pure metal when it changes from solid to liquid, as shown in Figure 2.15. The process is again reversible, but observe the behavior of the amorphous material during cooling from the liquid state, rather FIGURE 2.15 Characteristic change in volume for a pure metal (a crystalline structure), compared to the same volumetric changes in glass (a noncrystalline structure). E1C02 11/02/2009 14:15:26 Page 37 Section 2.5/Engineering Materials 37 than during melting from the solid, as before. Glass (silica, SiO2) is used to illustrate. At high temperatures, glass is a true liquid, and the molecules are free to move about as in the usual definition of a liquid. As the glass cools, it gradually transforms into the solid state, going through a transition phase, called a supercooled liquid, before finally becoming rigid. It does not show the sudden volumetric change that is characteristic of crystalline materials; instead, it passes through its melting temperature Tm without a change in its thermal expansion slope. In this supercooled liquid region, the material becomes increasingly viscous as the temperature continues to decrease. As it cools further, a point is finally reached at which the supercooled liquid converts to a solid. This is called the glass-transition temperature Tg. At this point, there is a change in the thermal expansion slope. (It might be more precise to refer to it as the thermal contraction slope; however, the slope is the same for expansion and contraction.) The rate of thermal expansion is lower for the solid material than for the supercooled liquid. The difference in behavior between crystalline and noncrystalline materials can be traced to the response of their respective atomic structures to changes in temperature. When a pure metal solidifies from the molten state, the atoms arrange themselves into a regular and recurring structure. This crystal structure is much more compact than the random and loosely packed liquid from which it formed. Thus, the process of solidification produces the abrupt volumetric contraction observed in Figure 2.15 for the crystalline material. By contrast, amorphous materials do not achieve this repeating and closely packed structure at low temperatures. The atomic structure is the same random arrangement as in the liquid state; thus, there is no abrupt volumetric change as these materials transition from liquid to solid. 2.5 ENGINEERING MATERIALS Let us summarize how atomic structure, bonding, and crystal structure (or absence thereof) are related to the type of engineering material—metals, ceramics, and polymer. Metals Metals have crystalline structures in the solid state, almost without exception. The unit cells of these crystal structures are almost always BCC, FCC, or HCP. The atoms of the metals are held together by metallic bonding, which means that their valence electrons can move about with relative freedom (compared with the other types of atomic and molecular bonding). These structures and bonding generally make the metals strong and hard. Many of the metals are quite ductile (capable of being deformed, which is useful in manufacturing), especially the FCC metals. Other general properties of metals related to structure and bonding include: high electrical and thermal conductivity, opaqueness (impervious to light rays), and reflectivity (capacity to reflect light rays). Ceramics Ceramic molecules are characterized by ionic or covalent bonding, or both. The metallic atoms release or share their outermost electrons to the nonmetallic atoms, and a strong attractive force exists within the molecules. The general properties that result from these bonding mechanisms include: high hardness and stiffness (even at elevated temperatures), brittleness (no ductility), electrical insulation (nonconducting) properties, refractoriness (being thermally resistant), and chemical inertness. Ceramics possess either a crystalline or noncrystalline structure. Most ceramics have a crystal structure, whereas glasses based on silica (SiO2) are amorphous. In certain cases, either structure can exist in the same ceramic material. For example, silica occurs in nature as crystalline quartz. When this mineral is melted and then cooled, it solidifies to form fused silica, which has a noncrystalline structure. Polymers A polymer molecule consists of many repeating mers to form very large molecules held together by covalent bonding. Elements in polymers are usually carbon E1C02 11/02/2009 38 14:15:27 Page 38 Chapter 2/The Nature of Materials plus one or more other elements such as hydrogen, nitrogen, oxygen, and chlorine. Secondary bonding (van der Waals) holds the molecules together within the aggregate material (intermolecular bonding). Polymers have either a glassy structure or mixture of glassy and crystalline. There are differences among the three polymer types. In thermoplastic polymers, the molecules consist of long chains of mers in a linear structure. These materials can be heated and cooled without substantially altering their linear structure. In thermosetting polymers, the molecules transform into a rigid, three-dimensional structure on cooling from a heated plastic condition. If thermosetting polymers are reheated, they degrade chemically rather than soften. Elastomers have large molecules with coiled structures. The uncoiling and recoiling of the molecules when subjected to stress cycles motivate the aggregate material to exhibit its characteristic elastic behavior. The molecular structure and bonding of polymers provide them with the following typical properties: low density, high electrical resistivity (some polymers are used as insulating materials), and low thermal conductivity. Strength and stiffness of polymers vary widely. Some are strong and rigid (although not matching the strength and stiffness of metals or ceramics), whereas others exhibit highly elastic behavior. REFERENCES [1] Callister, W. D., Jr., Materials Science and Engineering: An Introduction, 7th ed. John Wiley & Sons, Hoboken, New Jersey, 2007. [2] Dieter, G. E. Mechanical Metallurgy, 3rd ed. McGraw-Hill, New York, 1986. [3] Flinn, R. A., and Trojan, P. K. Engineering Materials and Their Applications, 5th ed. John Wiley & Sons, New York, 1995. [4] Guy, A. G., and Hren, J. J. Elements of Physical Metallurgy, 3rd ed. Addison-Wesley, Reading, Massachusetts, 1974. [5] Van Vlack, L. H. Elements of Materials Science and Engineering, 6th ed. Addison-Wesley, Reading, Massachusetts, 1989. REVIEW QUESTIONS 2.1. The elements listed in the Periodic Table can be divided into three categories. What are these categories? Give an example of each. 2.2. Which elements are the noble metals? 2.3. What is the difference between primary and secondary bonding in the structure of materials? 2.4. Describe how ionic bonding works. 2.5. What is the difference between crystalline and noncrystalline structures in materials? 2.6. What are some common point defects in a crystal lattice structure? 2.7. Define the difference between elastic and plastic deformation in terms of the effect on the crystal lattice structure. 2.8. How do grain boundaries contribute to the strain hardening phenomenon in metals? 2.9. Identify some materials that have a crystalline structure. 2.10. Identify some materials that possess a noncrystalline structure. 2.11. What is the basic difference in the solidification (or melting) process between crystalline and noncrystalline structures? MULTIPLE CHOICE QUIZ There are 20 correct answers in the following multiple choice questions (some questions have multiple answers that are correct). To attain a perfect score on the quiz, all correct answers must be given. Each correct answer is worth 1 point. Each E1C02 11/02/2009 14:15:28 Page 39 Multiple Choice Quiz 39 omitted answer or wrong answer reduces the score by 1 point, and each additional answer beyond the correct number of answers reduces the score by 1 point. Percentage score on the quiz is based on the total number of correct answers. 2.1. The basic structural unit of matter is which one of the following: (a) atom, (b) electron, (c) element, (d) molecule, or (e) nucleus? 2.2. Approximately how many different elements have been identified (one best answer): (a) 10, (b) 50, (c) 100, (d) 200, or (e) 500? 2.3. In the Periodic Table, the elements can be divided into which of the following categories (three best answers): (a) ceramics, (b) gases, (c) liquids, (d) metals, (e) nonmetals, (f) polymers, (g) semimetals, and (h) solids? 2.4. The element with the lowest density and smallest atomic weight is which one of the following: (a) aluminum, (b) argon, (c) helium, (d) hydrogen, or (e) magnesium? 2.5. Which of the following bond types are classified as primary bonds (three correct answers): (a) covalent bonding, (b) hydrogen bonding, (c) ionic bonding, (d) metallic bonding, and (e) van der Waals forces? 2.6. How many atoms are there in the face-centered cubic (FCC) unit cell (one correct answer): (a) 8, (b) 9, (c) 10, (d) 12, or (e) 14? 2.7. Which of the following are not point defects in a crystal lattice structure (three correct answers): (a) edge dislocation, (b) grain boundaries, (c) interstitialcy, (d) Schottky defect, (e) screw dislocation, or (f) vacancy? 2.8. Which one of the following crystal structures has the fewest slip directions, thus making the metals with this structure generally more difficult to deform at room temperature: (a) BCC, (b) FCC, or (c) HCP? 2.9. Grain boundaries are an example of which one of the following types of crystal structure defects: (a) dislocation, (b) Frenkel defect, (c) line defects, (d) point defects, or (e) surface defects? 2.10. Twinning is which of the following (three bestanswers): (a) elastic deformation, (b) mechanism of plastic deformation, (c) more likely at high deformation rates, (d) more likely in metals with HCP structure, (e) slip mechanism, and (f) type of dislocation? 2.11. Polymers are characterized by which of the following bonding types (two correct answers): (a) adhesive, (b) covalent, (c) hydrogen, (d) ionic, (e) metallic, and (f) van der Waals? E1C03 11/10/2009 3 13:10:21 Page 40 MECHANICAL PROPERTIES OF MATERIALS Chapter Contents 3.1 Stress–Strain Relationships 3.1.1 Tensile Properties 3.1.2 Compression Properties 3.1.3 Bending and Testing of Brittle Materials 3.1.4 Shear Properties 3.2 Hardness 3.2.1 Hardness Tests 3.2.2 Hardness of Various Materials 3.3 Effect of Temperature on Properties 3.4 Fluid Properties 3.5 Viscoelastic Behavior of Polymers Mechanical properties of a material determine its behavior when subjected to mechanical stresses. These properties include elastic modulus, ductility, hardness, and various measures of strength. Mechanical properties are important in design because the function and performance of a product depend on its capacity to resist deformation under the stresses encountered in service. In design, the usual objective is for the product and its components to withstand these stresses without significant change in geometry. This capability depends on properties such as elastic modulus and yield strength. In manufacturing, the objective is just the opposite. Here, stresses that exceed the yield strength of the material must be applied to alter its shape. Mechanical processes such as forming and machining succeed by developing forces that exceed the material’s resistance to deformation. Thus, there is the following dilemma: Mechanical properties that are desirable to the designer, such as high strength, usually make the manufacture of the product more difficult. It is helpful for the manufacturing engineer to appreciate the design viewpoint and for the designer to be aware of the manufacturing viewpoint. This chapter examines the mechanical properties of materials that are most relevant in manufacturing. 3.1 STRESS–STRAIN RELATIONSHIPS There are three types of static stresses to which materials can be subjected: tensile, compressive, and shear. Tensile stresses tend to stretch the material, compressive stresses tend to squeeze it, and shear involves stresses that tend to cause adjacent portions of the material to slide against each other. The stress–strain curve is the basic relationship that describes the mechanical properties of materials for all three types. 40 E1C03 11/10/2009 13:10:21 Page 41 Section 3.1/Stress–Strain Relationships 41 FIGURE 3.1 Tensile test: (a) tensile force applied in (1) and (2) resulting elongation of material; (b) typical test specimen; and (c) setup of the tensile test. 3.1.1 TENSILE PROPERTIES The tensile test is the most common procedure for studying the stress–strain relationship, particularly for metals. In the test, a force is applied that pulls the material, tending to elongate it and reduce its diameter, as shown in Figure 3.1(a). Standards by ASTM (American Society for Testing and Materials) specify the preparation of the test specimen and the conduct of the test itself. The typical specimen and general setup of the tensile test is illustrated in Figure 3.1(b) and (c), respectively. The starting test specimen has an original length Lo and area Ao. The length is measured as the distance between the gage marks, and the area is measured as the (usually round) cross section of the specimen. During the testing of a metal, the specimen stretches, then necks, and finally fractures, as shown in Figure 3.2. The load and the change in length of the specimen are recorded as testing proceeds, to provide the data required to determine FIGURE 3.2 Typical progress of a tensile test: (1) beginning of test, no load; (2) uniform elongation and reduction of cross-sectional area; (3) continued elongation, maximum load reached; (4) necking begins, load begins to decrease; and (5) fracture. If pieces are put back together as in, (6) final length can be measured. E1C03 11/10/2009 42 13:10:21 Page 42 Chapter 3/Mechanical Properties of Materials FIGURE 3.3 Typical engineering stress–strain plot in a tensile test of a metal. the stress–strain relationship. There are two different types of stress–strain curves: (1) engineering stress–strain and (2) true stress–strain. The first is more important in design, and the second is more important in manufacturing. Engineering Stress–Strain The engineering stress and strain in a tensile test are defined relative to the original area and length of the test specimen. These values are of interest in design because the designer expects that the strains experienced by any component of the product will not significantly change its shape. The components are designed to withstand the anticipated stresses encountered in service. A typical engineering stress–strain curve from a tensile test of a metallic specimen is illustrated in Figure 3.3. The engineering stress at any point on the curve is defined as the force divided by the original area: s¼ F Ao ð3:1Þ where s ¼ engineering stress, MPa (lb/in2), F ¼ applied force in the test, N (lb), and Ao ¼ original area of the test specimen, mm2 (in2). The engineering strain at any point in the test is given by L  Lo ð3:2Þ e¼ Lo where e ¼ engineering strain, mm/mm (in/in); L ¼ length at any point during the elongation, mm (in); and Lo ¼ original gage length, mm (in). The units of engineering strain are given as mm/mm (in/in), but think of it as representing elongation per unit length, without units. The stress–strain relationship in Figure 3.3 has two regions, indicating two distinct forms of behavior: (1) elastic and (2) plastic. In the elastic region, the relationship between stress and strain is linear, and the material exhibits elastic behavior by returning to its original length when the load (stress) is released. The relationship is defined by Hooke’s law: s ¼ Ee ð3:3Þ where E ¼ modulus of elasticity, MPa (lb/in2), a measure of the inherent stiffness of a material. E1C03 11/10/2009 13:10:22 Page 43 Section 3.1/Stress–Strain Relationships TABLE 3.1 43 Elastic modulus for selected materials. Modulus of Elasticity Metals Aluminum and alloys Cast iron Copper and alloys Iron Lead Magnesium Nickel Steel Titanium Tungsten 2 MPa lb/in 69  103 138  103 110  103 209  103 21  103 48  103 209  103 209  103 117  103 407  103 10  106 20  106 16  106 30  106 3  106 7  106 30  106 30  106 17  106 59  106 Modulus of Elasticity Ceramics and Polymers Alumina Diamonda Plate glass Silicon carbide Tungsten carbide Nylon Phenol formaldehyde Polyethylene (low density) Polyethylene (high density) Polystyrene MPa lb/in2 345  103 1035  103 69  103 448  103 552  103 3.0  103 7.0  103 0.2  103 0.7  103 3.0  103 50  106 150  106 10  106 65  106 80  106 0.40  106 1.00  106 0.03  106 0.10  106 0.40  106 a Compiled from [8], [10], [11], [15], [16], and other sources. Although diamond is not a ceramic, it is often compared with the ceramic materials. It is a constant of proportionality whose value is different for different materials. Table 3.1 presents typical values for several materials, metals and nonmetals. As stress increases, some point in the linear relationship is finally reached at which the material begins to yield. This yield point Y of the material can be identified in the figure by the change in slope at the end of the linear region. Because the start of yielding is usually difficult to see in a plot of test data (it does not usually occur as an abrupt change in slope), Y is typically defined as the stress at which a strain offset of 0.2% from the straight line has occurred. More specifically, it is the point where the stress–strain curve for the material intersects a line that is parallel to the straight portion of the curve but offset from it by a strain of 0.2%. The yield point is a strength characteristic of the material, and is therefore often referred to as the yield strength (other names include yield stress and elastic limit). The yield point marks the transition to the plastic region and the start of plastic deformation of the material. The relationship between stress and strain is no longer guided by Hooke’s law. As the load is increased beyond the yield point, elongation of the specimen proceeds, but at a much faster rate than before, causing the slope of the curve to change dramatically, as shown in Figure 3.3. Elongation is accompanied by a uniform reduction in cross-sectional area, consistent with maintaining constant volume. Finally, the applied load F reaches a maximum value, and the engineering stress calculated at this point is called the tensile strength or ultimate tensile strength of the material. It is denoted as TS where TS ¼ F max =Ao . TS and Y are important strength properties in design calculations. (They are also used in manufacturing calculations.) Some typical values of yield strength and tensile strength are listed in Table 3.2 for selected metals. Conventional tensile testing of ceramics is difficult, and an alternative test is used to measure the strength of these brittle materials (Section 3.1.3). Polymers differ in their strength properties from metals and ceramics because of viscoelasticity (Section 3.5). To the right of the tensile strength on the stress–strain curve, the load begins to decline, and the test specimen typically begins a process of localized elongation known as necking. Instead of continuing to strain uniformly throughout its length, straining becomes concentrated in one small section of the specimen. The area of that section narrows down (necks) significantly until failure occurs. The stress calculated immediately before failure is known as the fracture stress. The amount of strain that the material can endure before failure is also a mechanical property of interest in many manufacturing processes. The common measure of this property is ductility, the ability of a material to plastically strain without fracture. This E1C03 11/10/2009 44 13:10:22 Page 44 Chapter 3/Mechanical Properties of Materials TABLE 3.2 Yield strength and tensile strength for selected metals. Yield Strength Tensile Strength Tensile Strength Yield Strength Metal MPa lb/in2 MPa lb/in2 Metal MPa lb/in2 MPa lb/in2 Aluminum, annealed Aluminum, CWa Aluminum alloysa Cast irona Copper, annealed Copper alloysa Magnesium alloysa 28 105 175 275 70 205 175 4,000 15,000 25,000 40,000 10,000 30,000 25,000 69 125 350 275 205 410 275 10,000 18,000 50,000 40,000 30,000 60,000 40,000 Nickel, annealed Steel, low Ca Steel, high Ca Steel, alloya Steel, stainlessa Titanium, pure Titanium alloy 150 175 400 500 275 350 800 22,000 25,000 60,000 75,000 40,000 50,000 120,000 450 300 600 700 650 515 900 65,000 45,000 90,000 100,000 95,000 75,000 130,000 Compiled from [8], [10], [11], [16], and other sources. Values given are typical. For alloys, there is a wide range in strength values depending on composition and treatment (e.g., heat treatment, work hardening). a measure can be taken as either elongation or area reduction. Elongation is defined as Lf  L o ð3:4Þ Lo where EL ¼ elongation, often expressed as a percent; Lf ¼ specimen length at fracture, mm (in), measured as the distance between gage marks after the two parts of the specimen have been put back together; and Lo ¼ original specimen length, mm (in). Area reduction is defined as EL ¼ Ao  Af ð3:5Þ Ao where AR ¼ area reduction, often expressed as a percent; Af ¼ area of the cross section at the point of fracture, mm2(in2); and Ao ¼ original area, mm2 (in2). There are problems with both of these ductility measures because of necking that occurs in metallic test specimens and the associated nonuniform effect on elongation and area reduction. Despite these difficulties, percent elongation and percent area reduction are the most commonly used measures of ductility in engineering practice. Some typical values of percent elongation for various materials (mostly metals) are listed in Table 3.3. AR ¼ True Stress–Strain Thoughtful readers may be troubled by the use of the original area of the test specimen to calculate engineering stress, rather than the actual (instantaneous) area that becomes increasingly smaller as the test proceeds. If the actual area were used, the calculated stress value would be higher. The stress value obtained by dividing the instantaneous value of area into the applied load is defined as the true stress: F ð3:6Þ A where s ¼ true stress, MPa (lb/in2); F ¼ force, N (lb); and A ¼ actual (instantaneous) area resisting the load, mm2 (in2). Similarly, true strain provides a more realistic assessment of the ‘‘instantaneous’’ elongation per unit length of the material. The value of true strain in a tensile test can be estimated by dividing the total elongation into small increments, calculating the engineering strain for each increment on the basis of its starting length, and then adding up the strain values. In the limit, true strain is defined as s¼ ZL e¼ Lo dL L ¼ ln L Lo ð3:7Þ E1C03 11/10/2009 13:10:23 Page 45 Section 3.1/Stress–Strain Relationships TABLE 3.3 materials. Material 45 Ductility as a percent of elongation (typical values) for various selected Metals Aluminum, annealed Aluminum, cold worked Aluminum alloys, annealeda Aluminum alloys, heat treateda Aluminum alloys, casta Cast iron, graya Copper, annealed Copper, cold worked Copper alloy: brass, annealed Magnesium alloysa Nickel, annealed Elongation Material 40% 8% 20% 8% 4% 0.6% 45% 10% 60% 10% 45% Metals, continued Steel, low Ca Steel, high Ca Steel, alloya Steel, stainless, austenitica Titanium, nearly pure Zinc alloy Ceramics Polymers Thermoplastic polymers Thermosetting polymers Elastomers (e.g., rubber) Elongation 30% 10% 20% 55% 20% 10% 0b 100% 1% 1%c Compiled from [8], [10], [11], [16], and other sources. Values given are typical. For alloys, there is a range of ductility that depends on composition and treatment (e.g., heat treatment, degree of work hardening). b Ceramic materials are brittle; they withstand elastic strain but virtually no plastic strain. c Elastomers endure significant elastic strain, but their plastic strain is very limited, only around 1% being typical. a where L ¼ instantaneous length at any moment during elongation. At the end of the test (or other deformation), the final strain value can be calculated using L ¼ Lf. When the engineering stress–strain data in Figure 3.3 are plotted using the true stress and strain values, the resulting curve would appear as in Figure 3.4. In the elastic region, the plot is virtually the same as before. Strain values are small, and true strain is nearly equal to engineering strain for most metals of interest. The respective stress values are also very close to each other. The reason for these near equalities is that the cross-sectional area of the test specimen is not significantly reduced in the elastic region. Thus, Hooke’s law can be used to relate true stress to true strain: s ¼ E e. The difference between the true stress–strain curve and its engineering counterpart occurs in the plastic region. The stress values are higher in the plastic region because the FIGURE 3.4 True stress–strain curve for the previous engineering stress–strain plot in Figure 3.3. E1C03 11/10/2009 46 13:10:23 Page 46 Chapter 3/Mechanical Properties of Materials instantaneous cross-sectional area of the specimen, which has been continuously reduced during elongation, is now used in the computation. As in the previous curve, a downturn finally occurs as a result of necking. A dashed line is used in the figure to indicate the projected continuation of the true stress–strain plot if necking had not occurred. As strain becomes significant in the plastic region, the values of true strain and engineering strain diverge. True strain can be related to the corresponding engineering strain by e ¼ lnð1 þ eÞ ð3:8Þ Similarly, true stress and engineering stress can be related by the expression s ¼ sð1 þ eÞ ð3:9Þ In Figure 3.4, note that stress increases continuously in the plastic region until necking begins. When this happened in the engineering stress–strain curve, its significance was lost because an admittedly erroneous area value was used to calculate stress. Now when the true stress also increases, it cannot be dismissed so lightly. What it means is that the metal is becoming stronger as strain increases. This is the property called strain hardening that was mentioned in the previous chapter in the discussion of metallic crystal structures, and it is a property that most metals exhibit to a greater or lesser degree. Strain hardening, or work hardening as it is often called, is an important factor in certain manufacturing processes, particularly metal forming. Consider the behavior of a metal as it is affected by this property. If the portion of the true stress–strain curve representing the plastic region were plotted on a log–log scale, the result would be a linear relationship, as shown in Figure 3.5. Because it is a straight line in this transformation of the data, the relationship between true stress and true strain in the plastic region can be expressed as ð3:10Þ s ¼ Ken This equation is called the flow curve, and it provides a good approximation of the behavior of metals in the plastic region, including their capacity for strain hardening. The constant K is called the strength coefficient, MPa (lb/in2), and it equals the value of true stress at a true strain value equal to one. The parameter n is called the strain hardening exponent, and it is the slope of the line in Figure 3.5. Its value is directly related to a metal’s tendency to work harden. Typical values of K and n for selected metals are given in Table 3.4. Necking in a tensile test and metal-forming operations that stretch the workpart is closely related to strain hardening. As the test specimen is elongated during the initial part of the test (before necking begins), uniform straining occurs throughout the length because if any element in the specimen becomes strained more than the surrounding metal, its strength increases because of work hardening, thus making it more resistant to additional strain until FIGURE 3.5 True stress–strain curve plotted on log–log scale. E1C03 11/10/2009 13:10:23 Page 47 Section 3.1/Stress–Strain Relationships 47 TABLE 3.4 Typical values of strength coefficient K and strain hardening exponent n for selected metals. Strength Coefficient, K Material MPa lb/in2 Aluminum, pure, annealed Aluminum alloy, annealeda Aluminum alloy, heat treated Copper, pure, annealed Copper alloy: brassa Steel, low C, annealeda Steel, high C, annealeda Steel, alloy, annealeda Steel, stainless, austenitic, annealed 175 240 400 300 700 500 850 700 1200 25,000 35,000 60,000 45,000 100,000 75,000 125,000 100,000 175,000 Strain Hardening Exponent, n 0.20 0.15 0.10 0.50 0.35 0.25 0.15 0.15 0.40 Compiled from [9], [10], [11], and other sources. Values of K and n vary according to composition, heat treatment, and work hardening. a the surrounding metal has been strained an equal amount. Finally, the strain becomes so large that uniform straining cannot be sustained. A weak point in the length develops (because of buildup of dislocations at grain boundaries, impurities in the metal, or other factors), and necking is initiated, leading to failure. Empirical evidence reveals that necking begins for a particular metal when the true strain reaches a value equal to the strain-hardening exponent n. Therefore, a higher n value means that the metal can be strained further before the onset of necking during tensile loading. Types of Stress–Strain Relationships Much information about elastic–plastic behavior is provided by the true stress–strain curve. As indicated, Hooke’s law ðs ¼ EeÞ governs the metal’s behavior in the elastic region, and the flow curve ðs ¼ Ken Þ determines the behavior in the plastic region. Three basic forms of stress–strain relationship describe the behavior of nearly all types of solid materials, shown in Figure 3.6: 1. Perfectly elastic. The behavior of this material is defined completely by its stiffness, indicated by the modulus of elasticity E. It fractures rather than yielding to plastic flow. Brittle materials such as ceramics, many cast irons, and thermosetting polymers possess stress–strain curves that fall into this category. These materials are not good candidates for forming operations. 2. Elastic and perfectly plastic. This material has a stiffness defined by E. Once the yield strength Y is reached, the material deforms plastically at the same stress level. The flow curve is given by K ¼ Y and n ¼ 0. Metals behave in this fashion when they have been FIGURE 3.6 Three categories of stress– strain relationship: (a) perfectly elastic, (b) elastic and perfectly plastic, and (c) elastic and strain hardening. E1C03 11/10/2009 48 13:10:23 Page 48 Chapter 3/Mechanical Properties of Materials heated to sufficiently high temperatures that they recrystallize rather than strain harden during deformation. Lead exhibits this behavior at room temperature because room temperature is above the recrystallization point for lead. 3. Elastic and strain hardening. This material obeys Hooke’s law in the elastic region. It begins to flow at its yield strength Y. Continued deformation requires an ever-increasing stress, given by a flow curve whose strength coefficient K is greater than Y and whose strain-hardening exponent n is greater than zero. The flow curve is generally represented as a linear function on a natural logarithmic plot. Most ductile metals behave this way when cold worked. Manufacturing processes that deform materials through the application of tensile stresses include wire and bar drawing (Section 19.6) and stretch forming (Section 20.6.1). 3.1.2 COMPRESSION PROPERTIES A compression test applies a load that squeezes a cylindrical specimen between two platens, as illustrated in Figure 3.7. As the specimen is compressed, its height is reduced and its cross-sectional area is increased. Engineering stress is defined as s¼ F Ao ð3:11Þ where Ao ¼ original area of the specimen. This is the same definition of engineering stress used in the tensile test. The engineering strain is defined as e¼ h  ho ho ð3:12Þ where h ¼ height of the specimen at a particular moment into the test, mm (in); and ho ¼ starting height, mm (in). Because the height is decreased during compression, the value of e will be negative. The negative sign is usually ignored when expressing values of compression strain. When engineering stress is plotted against engineering strain in a compression test, the results appear as in Figure 3.8. The curve is divided into elastic and plastic regions, as before, FIGURE 3.7 Compression test: (a) compression force applied to test piece in (1), and (2) resulting change in height; and (b) setup for the test, with size of test specimen exaggerated. E1C03 11/10/2009 13:10:23 Page 49 Section 3.1/Stress–Strain Relationships 49 FIGURE 3.8 Typical engineering stress– strain curve for a compression test. but the shape of the plastic portion of the curve is different from its tensile test complement. Because compression causes the cross section to increase (rather than decrease as in the tensile test), the load increases more rapidly than previously. This results in a higher value of calculated engineering stress. Something else happens in the compression test that contributes to the increase in stress. As the cylindrical specimen is squeezed, friction at the surfaces in contact with the platens tends to prevent the ends of the cylinder from spreading. Additional energy is consumed by this friction during the test, and this results in a higher applied force. It also shows up as an increase in the computed engineering stress. Hence, owing to the increase in cross-sectional area and friction between the specimen and the platens, the characteristic engineering stress–strain curve is obtained in a compression test as seen in the figure. Another consequence of the friction between the surfaces is that the material near the middle of the specimen is permitted to increase in area much more than at the ends. This results in the characteristic barreling of the specimen, as seen in Figure 3.9. Although differences exist between the engineering stress–strain curves in tension and compression, when the respective data are plotted as true stress–strain, the relationships are nearly identical (for almost all materials). Because tensile test results are more abundant in the literature, values of the flow curve parameters (K and n) can be derived from tensile test data FIGURE 3.9 Barreling effect in a compression test: (1) start of test; and (2) after considerable compression has occurred. E1C03 11/10/2009 50 13:10:23 Page 50 Chapter 3/Mechanical Properties of Materials and applied with equal validity to a compression operation. What must be done in using the tensile test results for a compression operation is to ignore the effect of necking, a phenomenon that is peculiar to straining induced by tensile stresses. In compression, there is no corresponding collapse of the work. In previous plots of tensile stress–strain curves, the data were extended beyond the point of necking by means of the dashed lines. The dashed lines better represent the behavior of the material in compression than the actual tensile test data. Compression operations in metal forming are much more common than stretching operations. Important compression processes in industry include rolling, forging, and extrusion (Chapter 19). 3.1.3 BENDING AND TESTING OF BRITTLE MATERIALS Bending operations are used to form metal plates and sheets. As shown in Figure 3.10, the process of bending a rectangular cross section subjects the material to tensile stresses (and strains) in the outer half of the bent section and compressive stresses (and strains) in the inner half. If the material does not fracture, it becomes permanently (plastically) bent as shown in (3.1) of Figure 3.10. Hard, brittle materials (e.g., ceramics), which possess elasticity but little or no plasticity, are often tested by a method that subjects the specimen to a bending load. These materials do not respond well to traditional tensile testing because of problems in preparing the test specimens and possible misalignment of the press jaws that hold the specimen. The bending test (also known as the flexure test) is used to test the strength of these materials, using a setup illustrated in the first diagram in Figure 3.10. In this procedure, a specimen of rectangular cross section is positioned between two supports, and a load is applied at its center. In this configuration, the test is called a three-point bending test. A four-point configuration is also sometimes used. These brittle materials do not flex to the exaggerated extent shown in Figure 3.10; instead they deform elastically until immediately before fracture. Failure usually occurs because the ultimate tensile strength of the outer fibers of the specimen has been exceeded. This results in cleavage, a failure mode associated with ceramics and metals operating at low service temperatures, in which separation rather than slip occurs along certain crystallographic planes. The strength value derived from this test is called the transverse rupture strength, calculated from the formula TRS ¼ 1:5 FL bt2 ð3:13Þ FIGURE 3.10 Bending of a rectangular cross section results in both tensile and compressive stresses in the material: (1) initial loading; (2) highly stressed and strained specimen; and (3) bent part. E1C03 11/10/2009 13:10:23 Page 51 Section 3.1/Stress–Strain Relationships 51 where TRS ¼ transverse rupture strength, MPa (lb/in2); F ¼ applied load at fracture, N (lb); L ¼ length of the specimen between supports, mm (in); and b and t are the dimensions of the cross section of the specimen as shown in the figure, mm (in). The flexure test is also used for certain nonbrittle materials such as thermoplastic polymers. In this case, because the material is likely to deform rather than fracture, TRS cannot be determined based on failure of the specimen. Instead, either of two measures is used: (1) the load recorded at a given level of deflection, or (2) the deflection observed at a given load. 3.1.4 SHEAR PROPERTIES Shear involves application of stresses in opposite directions on either side of a thin element to deflect it, as shown in Figure 3.11. The shear stress is defined as F ð3:14Þ t¼ A where t ¼ shear stress, lb/in2 (MPa); F ¼ applied force, N (lb); and A ¼ area over which the force is applied, in2 (mm2). Shear strain can be defined as d g¼ ð3:15Þ b where g ¼ shear strain, mm/mm (in/in); d ¼ the deflection of the element, mm (in); and b ¼ the orthogonal distance over which deflection occurs, mm (in). Shear stress and strain are commonly tested in a torsion test, in which a thin-walled tubular specimen is subjected to a torque as shown in Figure 3.12. As torque is increased, the tube deflects by twisting, which is a shear strain for this geometry. The shear stress can be determined in the test by the equation t¼ FIGURE 3.11 (a) stress and (b) strain. FIGURE 3.12 test setup. Torsion Shear T 2pR2 t ð3:16Þ E1C03 11/10/2009 52 13:10:23 Page 52 Chapter 3/Mechanical Properties of Materials FIGURE 3.13 Typical shear stress– strain curve from a torsion test. where T ¼ applied torque, N-mm (lb-in); R ¼ radius of the tube measured to the neutral axis of the wall, mm (in); and t ¼ wall thickness, mm (in). The shear strain can be determined by measuring the amount of angular deflection of the tube, converting this into a distance deflected, and dividing by the gauge length L. Reducing this to a simple expression g¼ Ra L ð3:17Þ where a ¼ the angular deflection (radians). A typical shear stress–strain curve is shown in Figure 3.13. In the elastic region, the relationship is defined by t ¼ Gg ð3:18Þ where G ¼ the shear modulus, or shear modulus of elasticity, MPa (lb/in ). For most materials, the shear modulus can be approximated by G ¼ 0.4E, where E is the conventional elastic modulus. In the plastic region of the shear stress–strain curve, the material strain hardens to cause the applied torque to continue to increase until fracture finally occurs. The relationship in this region is similar to the flow curve. The shear stress at fracture can be calculated and this is used as the shear strength S of the material. Shear strength can be estimated from tensile strength data by the approximation: S ¼ 0.7(TS). Because the cross-sectional area of the test specimen in the torsion test does not change as it does in the tensile and compression tests, the engineering stress–strain curve for shear derived from the torsion test is virtually the same as the true stress–strain curve. Shear processes are common in industry. Shearing action is used to cut sheet metal in blanking, punching, and other cutting operations (Section 20.1). In machining, the material is removed by the mechanism of shear deformation (Section 21.2). 2 3.2 HARDNESS The hardness of a material is defined as its resistance to permanent indentation. Good hardness generally means that the material is resistant to scratching and wear. For many engineering applications, including most of the tooling used in manufacturing, scratch E1C03 11/10/2009 13:10:23 Page 53 Section 3.2/Hardness 53 and wear resistance are important characteristics. As the reader shall see later in this section, there is a strong correlation between hardness and strength. 3.2.1 HARDNESS TESTS Hardness tests are commonly used for assessing material properties because they are quick and convenient. However, a variety of testing methods are appropriate because of differences in hardness among different materials. The best-known hardness tests are Brinell and Rockwell. Brinell Hardness Test The Brinell hardness test is widely used for testing metals and nonmetals of low to medium hardness. It is named after the Swedish engineer who developed it around 1900. In the test, a hardened steel (or cemented carbide) ball of 10-mm diameter is pressed into the surface of a specimen using a load of 500, 1500, or 3000 kg. The load is then divided into the indentation area to obtain the Brinell Hardness Number (BHN). In equation form HB ¼ 2F  qffiffiffiffiffiffiffiffiffiffiffiffiffiffiffiffiffiffi pDb Db  D2b  D2i ð3:19Þ where HB ¼ Brinell Hardness Number (BHN); F ¼ indentation load, kg; Db ¼ diameter of the ball, mm; and Di ¼ diameter of the indentation on the surface, mm. These dimensions are indicated in Figure 3.14(a). The resulting BHN has units of kg/ mm2, but the units are usually omitted in expressing the number. For harder materials (above 500 BHN), the cemented carbide ball is used because the steel ball experiences elastic deformation that compromises the accuracy of the reading. Also, higher loads (1500 and 3000 kg) are typically used for harder materials. Because of differences in results under different loads, it is considered good practice to indicate the load used in the test when reporting HB readings. FIGURE 3.14 Hardness testing methods: (a) Brinell; (b) Rockwell: (1) initial minor load and (2) major load, (c) Vickers, and (d) Knoop. E1C03 11/10/2009 54 13:10:24 Page 54 Chapter 3/Mechanical Properties of Materials TABLE 3.5 Common Rockwell hardness scales. Rockwell Scale Hardness Symbol Indenter A B C Cone 1.6 mm ball Cone HRA HRB HRC Load (kg) 60 100 150 Typical Materials Tested Carbides, ceramics Nonferrous metals Ferrous metals, tool steels Rockwell Hardness Test This is another widely used test, named after the metallurgist who developed it in the early 1920s. It is convenient to use, and several enhancements over the years have made the test adaptable to a variety of materials. In the Rockwell Hardness Test, a cone-shaped indenter or small-diameter ball, with diameter ¼ 1.6 or 3.2 mm (1/16 or 1/8 in) is pressed into the specimen using a minor load of 10 kg, thus seating the indenter in the material. Then, a major load of 150 kg (or other value) is applied, causing the indenter to penetrate into the specimen a certain distance beyond its initial position. This additional penetration distance d is converted into a Rockwell hardness reading by the testing machine. The sequence is depicted in Figure 3.14(b). Differences in load and indenter geometry provide various Rockwell scales for different materials. The most common scales are indicated in Table 3.5. Vickers Hardness Test This test, also developed in the early 1920s, uses a pyramidshaped indenter made of diamond. It is based on the principle that impressions made by this indenter are geometrically similar regardless of load. Accordingly, loads of various size are applied, depending on the hardness of the material to be measured. The Vickers Hardness (HV) is then determined from the formula HV ¼ 1:854 F D2 ð3:20Þ where F ¼ applied load, kg, and D ¼ the diagonal of the impression made by the indenter, mm, as indicated in Figure 3.14(c). The Vickers test can be used for all metals and has one of the widest scales among hardness tests. Knoop Hardness Test The Knoop test, developed in 1939, uses a pyramid-shaped diamond indenter, but the pyramid has a length-to-width ratio of about 7:1, as indicated in Figure 3.14(d), and the applied loads are generally lighter than in the Vickers test. It is a microhardness test, meaning that it is suitable for measuring small, thin specimens or hard materials that might fracture if a heavier load were applied. The indenter shape facilitates reading of the impression under the lighter loads used in this test. The Knoop hardness value (HK) is determined according to the formula HK ¼ 14:2 F D2 ð3:21Þ where F ¼ load, kg; and D ¼ the long diagonal of the indentor, mm. Because the impression made in this test is generally very small, considerable care must be taken in preparing the surface to be measured. Scleroscope The previous tests base their hardness measurements either on the ratio of applied load divided by the resulting impression area (Brinell, Vickers, and Knoop) or by the depth of the impression (Rockwell). The Scleroscope is an instrument that measures the rebound height of a ‘‘hammer’’ dropped from a certain distance above the surface of the material to be tested. The hammer consists of a weight with diamond indenter attached to it. E1C03 11/10/2009 13:10:24 Page 55 55 Section 3.2/Hardness The Scleroscope therefore measures the mechanical energy absorbed by the material when the indenter strikes the surface. The energy absorbed gives an indication of resistance to penetration, which matches the definition of hardness given here. If more energy is absorbed, the rebound will be less, meaning a softer material. If less energy is absorbed, the rebound will be higher—thus a harder material. The primary use of the Scleroscope seems to be in measuring the hardness of large parts of steel and other ferrous metals. Durometer The previous tests are all based on resistance to permanent or plastic deformation (indentation). The durometer is a device that measures the elastic deformation of rubber and similar flexible materials by pressing an indenter into the surface of the object. The resistance to penetration is an indication of hardness, as the term is applied to these types of materials. 3.2.2 HARDNESS OF VARIOUS MATERIALS This section compares the hardness values of some common materials in the three engineering material classes: metals, ceramics, and polymers. Metals The Brinell and Rockwell hardness tests were developed at a time when metals were the principal engineering materials. A significant amount of data has been collected using these tests on metals. Table 3.6 lists hardness values for selected metals. For most metals, hardness is closely related to strength. Because the method of testing for hardness is usually based on resistance to indentation, which is a form of compression, one would expect a good correlation between hardness and strength properties determined in a compression test. However, strength properties in a compression test are nearly the same as those from a tension test, after allowances for changes in cross-sectional area of the respective test specimens; so the correlation with tensile properties should also be good. Brinell hardness (HB) exhibits a close correlation with the ultimate tensile strength TS of steels, leading to the relationship [9, 15]: TS ¼ Kh ðHBÞ ð3:22Þ where Kh is a constant of proportionality. If TS is expressed in MPa, then Kh ¼ 3.45; and if TS is in lb/in2, then Kh ¼ 500. TABLE 3.6 Typical hardness of selected metals. Metal Aluminum, annealed Aluminum, cold worked Aluminum alloys, annealedb Aluminum alloys, hardenedb Aluminum alloys, castb Cast iron, gray, as castb Copper, annealed Copper alloy: brass, annealed Lead Brinell Hardness, HB 20 35 40 90 80 175 45 100 4 Rockwell Hardness, HRa 52B 44B 10C 60B Metal Magnesium alloys, hardenedb Nickel, annealed Steel, low C, hot rolledb Steel, high C, hot rolledb Steel, alloy, annealedb Steel, alloy, heat treatedb Steel, stainless, austeniticb Titanium, nearly pure Zinc Brinell Hardness, HB Rockwell Hardness, HRa 70 75 100 200 175 300 150 200 30 35B 40B 60B 95B, 15C 90B, 10C 33C 85B 95B Compiled from [10], [11], [16], and other sources. HR values are given in the B or C scale as indicated by the letter designation. Missing values indicate that the hardness is too low for Rockwell scales. b HB values given are typical. Hardness values will vary according to composition, heat treatment, and degree of work hardening. a E1C03 11/10/2009 56 13:10:24 Page 56 Chapter 3/Mechanical Properties of Materials TABLE 3.7 Hardness of selected ceramics and other hard materials, arranged in ascending order of hardness. Material Hardened tool steela Cemented carbide (WC – Co)a Alumina, Al2O3 Tungsten carbide, WC Silicon carbide, SiC Vickers Hardness, HV Knoop Hardness, HK 800 2000 2200 2600 2600 850 1400 1500 1900 1900 Material Titanium nitride, TiN Titanium carbide, TiC Cubic boron nitride, BN Diamond, sintered polycrystal Diamond, natural Vickers Hardness, HV Knoop Hardness, HK 3000 3200 6000 7000 10,000 2300 2500 4000 5000 8000 Compiled from [14], [16], and other sources. Hardened tool steel and cemented carbide are the two materials commonly used in the Brinell hardness test. a TABLE 3.8 Hardness of selected polymers. Polymer Brinell Hardness, HB Nylon Phenol formaldehyde Polyethylene, low density Polyethylene, high density 12 50 2 4 Polymer Polypropylene Polystyrene Polyvinyl-chloride Brinell Hardness, HB 7 20 10 Compiled from [5], [8], and other sources. Ceramics The Brinell hardness test is not appropriate for ceramics because the materials being tested are often harder than the indenter ball. The Vickers and Knoop hardness tests are used to test these hard materials. Table 3.7 lists hardness values for several ceramics and hard materials. For comparison, the Rockwell C hardness for hardened tool steel is 65 HRC. The HRC scale does not extend high enough to be used for the harder materials. Polymers Polymers have the lowest hardness among the three types of engineering materials. Table 3.8 lists several of the polymers on the Brinell hardness scale, although this testing method is not normally used for these materials. It does, however, allow comparison with the hardness of metals. 3.3 EFFECT OF TEMPERATURE ON PROPERTIES Temperature has a significant effect on nearly all properties of a material. It is important for the designer to know the material properties at the operating temperatures of the product when in service. It is also important to know how temperature affects mechanical properties in manufacturing. At elevated temperatures, materials are lower in strength and higher in ductility. The general relationships for metals are depicted in Figure 3.15. Thus, most metals can be formed more easily at elevated temperatures than when they are cold. Hot Hardness A property often used to characterize strength and hardness at elevated temperatures is hot hardness. Hot hardness is simply the ability of a material to retain hardness at elevated temperatures; it is usually presented as either a listing of hardness values at different temperatures or as a plot of hardness versus temperature, as in Figure 3.16. Steels can be alloyed to achieve significant improvements in hot hardness, as shown in the figure. E1C03 11/10/2009 13:10:24 Page 57 Section 3.3/Effect of Temperature on Properties 57 FIGURE 3.15 General effect of temperature on strength and ductility. Ceramics exhibit superior properties at elevated temperatures. These materials are often selected for high temperature applications, such as turbine parts, cutting tools, and refractory applications. The outside skin of a shuttle spacecraft is lined with ceramic tiles to withstand the friction heat of high-speed re-entry into the atmosphere. Good hot hardness is also desirable in the tooling materials used in many manufacturing operations. Significant amounts of heat energy are generated in most metalworking processes, and the tools must be capable of withstanding the high temperatures involved. Recrystallization Temperature Most metals behave at room temperature according to the flow curve in the plastic region. As the metal is strained, it increases in strength because of strain hardening (the strain-hardening exponent n > 0). However, if the metal is heated to a sufficiently elevated temperature and then deformed, strain hardening does not occur. Instead, new grains are formed that are free of strain, and the metal behaves as a perfectly plastic material; that is, with a strain-hardening exponent n ¼ 0. The formation of new strainfree grains is a process called recrystallization, and the temperature at which it occurs is about one-half the melting point (0.5 Tm), as measured on an absolute scale (R or K). This is called the recrystallization temperature. Recrystallization takes time. The recrystallization temperature for a particular metal is usually specified as the temperature at which complete formation of new grains requires about 1 hour. FIGURE 3.16 Hot hardness—typical hardness as a function of temperature for several materials. E1C03 11/10/2009 58 13:10:24 Page 58 Chapter 3/Mechanical Properties of Materials Recrystallization is a temperature-dependent characteristic of metals that can be exploited in manufacturing. By heating the metal to the recrystallization temperature before deformation, the amount of straining that the metal can endure is substantially increased, and the forces and power required to carry out the process are significantly reduced. Forming metals at temperatures above the recrystallization temperature is called hot working (Section 18.3). 3.4 FLUID PROPERTIES Fluids behave quite differently than solids. A fluid flows; it takes the shape of the container that holds it. A solid does not flow; it possesses a geometric form that is independent of its surroundings. Fluids include liquids and gases; the interest in this section is on the former. Many manufacturing processes are accomplished on materials that have been converted from solid to liquid state by heating. Metals are cast in the molten state; glass is formed in a heated and highly fluid state; and polymers are almost always shaped as thick fluids. Viscosity Although flow is a defining characteristic of fluids, the tendency to flow varies for different fluids. Viscosity is the property that determines fluid flow. Roughly, viscosity can be defined as the resistance to flow that is characteristic of a fluid. It is a measure of the internal friction that arises when velocity gradients are present in the fluid—the more viscous the fluid is, the higher the internal friction and the greater the resistance to flow. The reciprocal of viscosity is fluidity—the ease with which a fluid flows. Viscosity is defined more precisely with respect to the setup in Figure 3.17, in which two parallel plates are separated by a distance d. One of the plates is stationary while the other is moving at a velocity v, and the space between the plates is occupied by a fluid. Orienting these parameters relative to an axis system, d is in the y-axis direction and v is in the x-axis direction. The motion of the upper plate is resisted by force F that results from the shear viscous action of the fluid. This force can be reduced to a shear stress by dividing F by the plate area A t¼ F A ð3:23Þ where t ¼ shear stress, N/m2 or Pa (lb/in2). This shear stress is related to the rate of shear, which is defined as the change in velocity dv relative to dy. That is dv g_ ¼ ð3:24Þ dy where g_ ¼ shear rate, 1/s; dv ¼ incremental change in velocity, m/s (in/sec); and dy ¼ incremental change in distance y, m (in). FIGURE 3.17 Fluid flow between two parallel plates, one stationary and the other moving at velocity v. E1C03 11/10/2009 13:10:25 Page 59 59 Section 3.4/Fluid Properties The shear viscosity is the fluid property that defines the relationship between F/A and dv/dy; that is F dv ¼h A dy or t ¼ hg_ ð3:25Þ where h ¼ a constant of proportionality called the coefficient of viscosity, Pa-s (lb-sec/in2). Rearranging Equation 3.25, the coefficient of viscosity can be expressed as follows t ð3:26Þ h¼ g_ Thus, the viscosity of a fluid can be defined as the ratio of shear stress to shear rate during flow, where shear stress is the frictional force exerted by the fluid per unit area, and shear rate is the velocity gradient perpendicular to the flow direction. The viscous characteristics of fluids defined by Equation 3.26 were first stated by Newton. He observed that viscosity was a constant property of a given fluid, and such a fluid is referred to as a Newtonian fluid. The units of coefficient of viscosity require explanation. In the International System of units (SI), because shear stress is expressed in N/m2 or Pascals and shear rate in 1/s, it follows that h has units of N-s/m2 or Pascal-seconds, abbreviated Pa-s. In the U.S. customary units, the corresponding units are lb/in2 and 1/sec, so that the units for coefficient of viscosity are lb-sec/ in2. Other units sometimes given for viscosity are poise, which ¼ dyne-sec/cm2 (10 poise ¼ 1 Pa-s and 6895 Pa-s ¼ 1 lb-sec/in2). Some typical values of coefficient of viscosity for various fluids are given in Table 3.9. One can observe in several of the materials listed that viscosity varies with temperature. Viscosity in Manufacturing Processes For many metals, the viscosity in the molten state compares with that of water at room temperature. Certain manufacturing processes, notably casting and welding, are performed on metals in their molten state, and success in these operations requires low viscosity so that the molten metal fills the mold cavity or weld seam before solidifying. In other operations, such as metal forming and machining, lubricants and coolants are used in the process, and again the success of these fluids depends to some extent on their viscosities. Glass ceramics exhibit a gradual transition from solid to liquid states as temperature is increased; they do not suddenly melt as pure metals do. The effect is illustrated by the viscosity values for glass at different temperatures in Table 3.9. At room temperature, glass is solid and brittle, exhibiting no tendency to flow; for all practical purposes, its viscosity is infinite. As glass is heated, it gradually softens, becoming less and less viscous (more and more fluid), until it can finally be formed by blowing or molding at around 1100 C (2000 F). TABLE 3.9 Viscosity values for selected fluids. Coefficient of Viscosity 2 Material Pa-s lb-sec/in Glassb, 540 C (1000 F) Glassb, 815 C (1500 F) Glassb, 1095 C (2000 F) Glassb, 1370 C (2500 F) Mercury, 20 C (70 F) Machine oil (room temp.) 1012 105 103 15 0.0016 0.1 108 14 0.14 22  104 0.23  106 0.14  104 Coefficient of Viscosity Material Pa-s lb-sec/in2 Pancake syrup (room temp) Polymer,a 151 C (300 F) Polymer,a 205 C (400 F) Polymer,a 260 C (500 F) Water, 20 C (70 F) Water, 100 C (212 F) 50 115 55 28 0.001 0.0003 73  104 167  104 80  104 41  104 0.15  106 0.04  106 Compiled from various sources. Low-density polyethylene is used as the polymer example here; most other polymers have slightly higher viscosities. b Glass composition is mostly SiO2; compositions and viscosities vary; values given are representative. a E1C03 11/10/2009 60 13:10:25 Page 60 Chapter 3/Mechanical Properties of Materials FIGURE 3.18 Viscous behaviors of Newtonian and pseudoplastic fluids. Polymer melts exhibit pseudoplastic behavior. For comparison, the behavior of a plastic solid material is shown. Most polymer-shaping processes are performed at elevated temperatures, at which the material is in a liquid or highly plastic condition. Thermoplastic polymers represent the most straightforward case, and they are also the most common polymers. At low temperatures, thermoplastic polymers are solid; as temperature is increased, they typically transform first into a soft rubbery material, and then into a thick fluid. As temperature continues to rise, viscosity decreases gradually, as in Table 3.9 for polyethylene, the most widely used thermoplastic polymer. However, with polymers the relationship is complicated by other factors. For example, viscosity is affected by flow rate. The viscosity of a thermoplastic polymer is not a constant. A polymer melt does not behave in a Newtonian fashion. Its relationship between shear stress and shear rate can be seen in Figure 3.18. A fluid that exhibits this decreasing viscosity with increasing shear rate is called pseudoplastic. This behavior complicates the analysis of polymer shaping. 3.5 VISCOELASTIC BEHAVIOR OF POLYMERS Another property that is characteristic of polymers is viscoelasticity. Viscoelasticity is the property of a material that determines the strain it experiences when subjected to combinations of stress and temperature over time. As the name suggests, it is a combination of viscosity and elasticity. Viscoelasticity can be explained with reference to Figure 3.19. The two parts of the figure show the typical response of two materials to an applied stress below the yield point during some time period. The material in (a) exhibits perfect elasticity; when the stress is removed, the material returns to its original shape. By contrast, the material in (b) shows viscoelastic behavior. The amount of strain gradually increases over time under the applied stress. When stress is removed, the material does not immediately return to its original shape; instead, the strain decays gradually. If the stress had been applied and then immediately removed, the material would have returned immediately to its starting shape. However, time has entered the picture and played a role in affecting the behavior of the material. A simple model of viscoelasticity can be developed using the definition of elasticity as a starting point. Elasticity is concisely expressed by Hooke’s law, s ¼ Ee, which simply relates stress to strain through a constant of proportionality. In a viscoelastic solid, the E1C03 11/10/2009 13:10:25 Page 61 Section 3.5/Viscoelastic Behavior of Polymers 61 FIGURE 3.19 Comparison of elastic and viscoelastic properties: (a) perfectly elastic response of material to stress applied over time; and (b) response of a viscoelastic material under same conditions. The material in (b) takes a strain that is a function of time and temperature. relationship between stress and strain is time dependent; it can be expressed as s ðtÞ ¼ f ðtÞe ð3:27Þ The time function f(t) can be conceptualized as a modulus of elasticity that depends on time. It might be written E(t) and referred to as a viscoelastic modulus. The form of this time function can be complex, sometimes including strain as a factor. Without getting into the mathematical expressions for it, nevertheless the effect of the time dependency can be explored. One common effect can be seen in Figure 3.20, which shows the stress–strain behavior of a thermoplastic polymer under different strain rates. At low strain rate, the material exhibits significant viscous flow. At high strain rate, it behaves in a much more brittle fashion. Temperature is a factor in viscoelasticity. As temperature increases, the viscous behavior becomes more and more prominent relative to elastic behavior. The material becomes more like a fluid. Figure 3.21 illustrates this temperature dependence for a thermoplastic polymer. At low temperatures, the polymer shows elastic behavior. As T increases above the glass transition temperature Tg, the polymer becomes viscoelastic. As temperature increases further, it becomes soft and rubbery. At still higher temperatures, it exhibits viscous characteristics. The temperatures at which these modes of behavior are observed vary, depending on the plastic. Also, the shapes of the modulus versus temperature curve differ according to the FIGURE 3.20 Stress–strain curve of a viscoelastic material (thermoplastic polymer) at high and low strain rates. E1C03 11/10/2009 62 13:10:25 Page 62 Chapter 3/Mechanical Properties of Materials FIGURE 3.21 Viscoelastic modulus as a function of temperature for a thermoplastic polymer. proportions of crystalline and amorphous structures in the thermoplastic. Thermosetting polymers and elastomers behave differently than shown in the figure; after curing, these polymers do not soften as thermoplastics do at elevated temperatures. Instead, they degrade (char) at high temperatures. Viscoelastic behavior manifests itself in polymer melts in the form of shape memory. As the thick polymer melt is transformed during processing from one shape to another, it ‘‘remembers’’ its previous shape and attempts to return to that geometry. For example, a common problem in extrusion of polymers is die swell, in which the profile of the extruded material grows in size, reflecting its tendency to return to its larger cross section in the extruder barrel immediately before being squeezed through the smaller die opening. The properties of viscosity and viscoelasticity are examined in more detail in the discussion of plastic shaping (Chapter 13). REFERENCES [1] Avallone, E. A., and Baumeister III, T. (eds.). Mark’s Standard Handbook for Mechanical Engineers, 11th ed. McGraw-Hill, New York, 2006. [2] Beer, F. P., Russell, J. E., Eisenberg, E., and Mazurek, D., Vector Mechanics for Engineers: Statics, 9th ed. McGraw-Hill, New York, 2009. [3] Black, J. T., and Kohser, R. A. DeGarmo’s Materials and Processes in Manufacturing, 10th ed. John Wiley & Sons, Hoboken, New Jersey, 2008. [4] Budynas, R. G. Advanced Strength and Applied Stress Analysis, 2nd ed. McGraw-Hill, New York, 1998. [5] Chandra, M., and Roy, S. K. Plastics Technology Handbook, 4th ed. CRC Press, Inc., Boca Raton, Florida, 2006. [6] Dieter, G. E. Mechanical Metallurgy, 3rd ed. McGraw-Hill, New York, 1986. [7] Engineering Plastics. Engineered Materials Handbook, Vol. 2. ASM International, Metals Park, Ohio, 1987. [8] Flinn, R. A., and Trojan, P. K. Engineering Materials and Their Applications, 5th ed. John Wiley & Sons, Hoboken, New Jersey, 1995. [9] Kalpakjian, S., and Schmid S. R. Manufacturing Processes for Engineering Materials, 5th ed. Prentice Hall, Upper Saddle River, New Jersey, 2007. [10] Metals Handbook, Vol. 1, Properties and Selection: Iron, Steels, and High Performance Alloys. ASM International, Metals Park, Ohio, 1990. [11] Metals Handbook, Vol. 2, Properties and Selection: Nonferrous Alloys and Special Purpose Materials, ASM International, Metals Park, Ohio, 1991. [12] Metals Handbook, Vol. 8, Mechanical Testing and Evaluation, ASM International, Metals Park, Ohio, 2000. [13] Morton-Jones, D. H. Polymer Processing. Chapman and Hall, London, 2008. E1C03 11/10/2009 13:10:25 Page 63 Multiple Choice Quiz [14] Schey, J. A. Introduction to Manufacturing Processes. 3rd ed. McGraw-Hill, New York, 2000. [15] Van Vlack, L. H. Elements of Materials Science and Engineering, 6th ed. Addison-Wesley, Reading, Massachusetts, 1991. 63 [16] Wick, C., and Veilleux, R. F. (eds.). Tool and Manufacturing Engineers Handbook, 4th ed. Vol. 3, Materials, Finishing, and Coating. Society of Manufacturing Engineers, Dearborn, Michigan, 1985. REVIEW QUESTIONS 3.1. What is the dilemma between design and manufacturing in terms of mechanical properties? 3.2. What are the three types of static stresses to which materials are subjected? 3.3. State Hooke’s law. 3.4. What is the difference between engineering stress and true stress in a tensile test? 3.5. Define tensile strength of a material. 3.6. Define yield strength of a material. 3.7. Why cannot a direct conversion be made between the ductility measures of elongation and reduction in area using the assumption of constant volume? 3.8. What is work hardening? 3.9. In what case does the strength coefficient have the same value as the yield strength? 3.10. How does the change in cross-sectional area of a test specimen in a compression test differ from its counterpart in a tensile test specimen? 3.11. What is the complicating factor that occurs in a compression test? 3.12. Tensile testing is not appropriate for hard brittle materials such as ceramics. What is the test commonly used to determine the strength properties of such materials? 3.13. How is the shear modulus of elasticity G related to the tensile modulus of elasticity E, on average? 3.14. How is shear strength S related to tensile strength TS, on average? 3.15. What is hardness, and how is it generally tested? 3.16. Why are different hardness tests and scales required? 3.17. Define the recrystallization temperature for a metal. 3.18. Define viscosity of a fluid. 3.19. What is the defining characteristic of a Newtonian fluid? 3.20. What is viscoelasticity, as a material property? MULTIPLE CHOICE QUIZ There are 15 correct answers in the following multiple choice questions (some questions have multiple answers that are correct). To attain a perfect score on the quiz, all correct answers must be given. Each correct answer is worth 1 point. Each omitted answer or wrong answer reduces the score by 1 point, and each additional answer beyond the correct number of answers reduces the score by 1 point. Percentage score on the quiz is based on the total number of correct answers. 3.1. Which of the following are the three basic types of static stresses to which a material can be subjected (three correct answers): (a) compression, (b) hardness, (c) reduction in area, (d) shear, (e) tensile, (f) true stress, and (g) yield? 3.2. Which one of the following is the correct definition of ultimate tensile strength, as derived from the results of a tensile test on a metal specimen: (a) the stress encountered when the stress–strain curve transforms from elastic to plastic behavior, (b) the maximum load divided by the final area of the specimen, (c) the maximum load divided by the original area of the specimen, or (d) the stress observed when the specimen finally fails? 3.3. If stress values were measured during a tensile test, which of the following would have the higher value: (a) engineering stress or (b) true stress? 3.4. If strain measurements were made during a tensiletest, which of the following would have the higher value: (a) engineering strain, or (b) true strain? 3.5. The plastic region of the stress–strain curve for a metal is characterized by a proportional relationship between stress and strain: (a) true or (b) false? 3.6. Which one of the following types of stress–strain relationship best describes the behavior of brittle materials such as ceramics and thermosetting plastics: (a) elastic and perfectly plastic, (b) elastic and strain hardening, (c) perfectly elastic, or (d) none of the above? 3.7. Which one of the following types of stress–strain relationship best describes the behavior of most metals at room temperature: (a) elastic and perfectly plastic, (b) elastic and strain hardening, (c) perfectly elastic, or (d) none of the above? E1C03 11/10/2009 64 13:10:25 Page 64 Chapter 3/Mechanical Properties of Materials 3.8. Which one of the following types of stress–strain relationship best describes the behavior of metals at temperatures above their respective recrystallization points: (a) elastic and perfectly plastic, (b) elastic and strain hardening, (c) perfectly elastic, or (d) none of the above? 3.9. Which one of the following materials has the highest modulus of elasticity: (a) aluminum, (b) diamond, (c) steel, (d) titanium, or (e) tungsten? 3.10. The shear strength of a metal is usually (a) greater than or (b) less than its tensile strength? 3.11. Most hardness tests involve pressing a hard object into the surface of a test specimen and measuring the indentation (or its effect) that results: (a) true or (b) false? 3.12. Which one of the following materials has the highest hardness: (a) alumina ceramic, (b) gray cast iron, (c) hardened tool steel, (d) high carbon steel, or (e) polystyrene? 3.13. Viscosity can be defined as the ease with which a fluid flows: (a) true or (b) false? PROBLEMS Strength and Ductility in Tension 3.1. A tensile test uses a test specimen that has a gage length of 50 mm and an area ¼ 200 mm 2 . During the test the specimen yields under a load of 98,000 N. The corresponding gage length ¼ 50.23 mm. This is the 0.2% yield point. The maximum load of 168,000 N is reached at a gage length ¼ 64.2 mm. Determine (a) yield strength, (b) modulus of elasticity, and (c) tensile strength. (d) If fracture occurs at a gage length of 67.3 mm, determine the percent elongation. (e) If the specimen necked to an area ¼ 92 mm2, determine the percent reduction in area. 3.2. A test specimen in a tensile test has a gage length of 2.0 in and an area ¼ 0.5 in2. During the test the specimen yields under a load of 32,000 lb. The corresponding gage length ¼ 2.0083 in. This is the 0.2 percent yield point. The maximum load of 60,000 lb is reached at a gage length ¼ 2.60 in. Determine (a) yield strength, (b) modulus of elasticity, and (c) tensile strength. (d) If fracture occurs at a gage length of 2.92 in, determine the percent elongation. (e) If the specimen necked to an area ¼ 0.25 in2, determine the percent reduction in area. 3.3. During a tensile test in which the starting gage length ¼ 125.0 mm and the cross-sectional area ¼ 62.5 mm2, the following force and gage length data are collected (1) 17,793 N at 125.23 mm, (2) 23,042 N at 131.25 mm, (3) 27,579 N at 140.05 mm, (4) 28, 913 N at 147.01 mm, (5) 27,578 N at 153.00 mm, and (6) 20,462 N at 160.10 mm. The maximum load is 28,913 N and the final data point occurred immediately before failure. (a) Plot the engineering stress strain curve. Determine (b) yield strength, (c) modulus of elasticity, and (d) tensile strength. Flow Curve 3.4. In Problem 3.3, determine the strength coefficient and the strain-hardening exponent in the flow curve equation. Be sure not to use data after the point at which necking occurred. 3.5. In a tensile test on a metal specimen, true strain ¼ 0.08 at a stress ¼ 265 MPa. When true stress ¼ 325 MPa, true strain ¼ 0.27. Determine the strength coefficient and the strain-hardening exponent in the flow curve equation. 3.6. During a tensile test, a metal has a true strain ¼ 0.10 at a true stress ¼ 37,000 lb/in2. Later, at a true stress ¼ 55,000 lb/in2, true strain ¼ 0.25. Determine the strength coefficient and strain-hardening exponent in the flow curve equation. 3.7. In a tensile test a metal begins to neck at a true strain ¼ 0.28 with a corresponding true stress ¼ 345.0 MPa. Without knowing any more about the test, can you estimate the strength coefficient and the strainhardening exponent in the flow curve equation? 3.8. A tensile test for a certain metal provides flow curve parameters: strain-hardening exponent is 0.3 and strength coefficient is 600 MPa. Determine (a) the flow stress at a true strain ¼ 1.0 and (b) true strain at a flow stress ¼ 600 MPa. 3.9. The flow curve for a certain metal has a strainhardening exponent of 0.22 and strength coefficient of 54,000 lb/in2. Determine (a) the flow stress at a true strain ¼ 0.45 and (b) the true strain at a flow stress ¼ 40,000 lb/in2. 3.10. A metal is deformed in a tension test into its plastic region. The starting specimen had a gage length ¼ 2.0 in and an area ¼ 0.50 in2. At one point in the tensile test, the gage length ¼ 2.5 in, and the corresponding engineering stress ¼ 24,000 lb/in2; at another point in the test before necking, the gage length ¼ 3.2 in, and the corresponding engineering stress ¼ 28,000 lb/in2. Determine the strength E1C03 11/10/2009 13:10:26 Page 65 Problems coefficient and the strain-hardening exponent for this metal. 3.11. A tensile test specimen has a starting gage length ¼ 75.0 mm. It is elongated during the test to a length ¼ 110.0 mm before necking occurs. Determine (a) the engineering strain and (b) the true strain. (c) Compute and sum the engineering strains as the specimen elongates from: (1) 75.0 to 80.0 mm, (2) 80.0 to 85.0 mm, (3) 85.0 to 90.0 mm, (4) 90.0 to 95.0 mm, (5) 95.0 to 100.0 mm, (6) 100.0 to 105.0 mm, and (7) 105.0 to 110.0 mm. (d) Is the result closer to the answer to part (a) or part (b)? Does this help to show what is meant by the term true strain? 3.12. A tensile specimen is elongated to twice its original length. Determine the engineering strain and true strain for this test. If the metal had been strained in compression, determine the final compressed length of the specimen such that (a) the engineering strain is equal to the same value as in tension (it will be negative value because of compression), and (b) the true strain would be equal to the same value as in tension (again, it will be negative value because 3.13. 3.14. 3.15. 3.16. 3.17. 65 of compression). Note that the answer to part (a) is an impossible result. True strain is therefore a better measure of strain during plastic deformation. Derive an expression for true strain as a function of D and Do for a tensile test specimen of round cross section, where D ¼ the instantaneous diameter of the specimen and Do is its original diameter. Show that true strain ¼ ln(1 þ e), where e ¼ engineering strain. Based on results of a tensile test, the flow curve strainhardening exponent ¼ 0.40 and strength coefficient ¼ 551.6 MPa. Based on this information, calculate the (engineering) tensile strength for the metal. A copper wire of diameter 0.80 mm fails at an engineering stress ¼ 248.2 MPa. Its ductility is measured as 75% reduction of area. Determine the true stress and true strain at failure. A steel tensile specimen with starting gage length ¼ 2.0 in and cross-sectional area ¼ 0.5 in2 reaches a maximum load of 37,000 lb. Its elongation at this point is 24%. Determine the true stress and true strain at this maximum load. Compression 3.18. A metal alloy has been tested in a tensile test with the following results for the flow curve parameters: strength coefficient ¼ 620.5 MPa and strainhardening exponent ¼ 0.26. The same metal is now tested in a compression test in which the starting height of the specimen ¼ 62.5 mm and its diameter ¼ 25 mm. Assuming that the cross section increases uniformly, determine the load required to compress the specimen to a height of (a) 50 mm and (b) 37.5 mm. 3.19. The flow curve parameters for a certain stainless steel are strength coefficient ¼ 1100 MPa and strain-hardening exponent ¼ 0.35. A cylindrical specimen of starting cross-sectional area ¼ 1000 mm2 and height ¼ 75 mm is compressed to a height of 58 mm. Determine the force required to achieve this compression, assuming that the cross section increases uniformly. 3.20. A steel test specimen (modulus of elasticity ¼ 30  106 lb/in2) in a compression test has a starting height ¼ 2.0 in and diameter ¼ 1.5 in. The metal yields (0.2% offset) at a load ¼ 140,000 lb. At a load of 260,000 lb, the height has been reduced to 1.6 in. Determine (a) yield strength and (b) flow curve parameters (strength coefficient and strainhardening exponent). Assume that the crosssectional area increases uniformly during the test. Bending and Shear 3.21. A bend test is used for a certain hard material. If the transverse rupture strength of the material is known to be 1000 MPa, what is the anticipated load at which the specimen is likely to fail, given that its width ¼ 15 mm, thickness ¼ 10 mm, and length ¼ 60 mm? 3.22. A special ceramic specimen is tested in a bend test. Its width ¼ 0.50 in and thickness ¼ 0.25 in. The length of the specimen between supports ¼ 2.0 in. Determine the transverse rupture strength if failure occurs at a load ¼ 1700 lb. 3.23. A torsion test specimen has a radius ¼ 25 mm, wall thickness ¼ 3 mm, and gage length ¼ 50 mm. In testing, a torque of 900 N-m results in an angular deflection ¼ 0.3 Determine (a) the shear stress, (b) shear strain, and (c) shear modulus, assuming the specimen had not yet yielded. (d) If failure of thespecimen occurs at a torque ¼ 1200 N-m and a corresponding angular deflection ¼ 10 , what is the shear strength of the metal? 3.24. In a torsion test, a torque of 5000 ft-lb is applied which causes an angular deflection ¼ 1 on a thinwalled tubular specimen whose radius ¼ 1.5 in, wall thickness ¼ 0.10 in, and gage length ¼ 2.0 in. Determine (a) the shear stress, (b) shear strain, and (c) shear modulus, assuming the specimen had not yet yielded. (d) If the specimen fails at a torque ¼ 8000 ft-lb and an angular deflection ¼ 23 , calculate the shear strength of the metal. E1C03 11/10/2009 66 13:10:26 Page 66 Chapter 3/Mechanical Properties of Materials Hardness 3.25. In a Brinell hardness test, a 1500-kg load is pressed into a specimen using a 10-mm-diameter hardened steel ball. The resulting indentation has a diameter ¼ 3.2 mm. (a) Determine the Brinell hardness number for the metal. (b) If the specimen is steel, estimate the tensile strength of the steel. 3.26. One of the inspectors in the quality control department has frequently used the Brinell and Rockwell hardness tests, for which equipment is available in the company. He claims that all hardness tests are based on the same principle as the Brinell test, which is that hardness is always measured as the applied load divided by the area of the impressions made by an indentor. (a) Is he correct? (b) If not, what are some of the other principles involved in hardness testing, and what are the associated tests? 3.27. A batch of annealed steel has just been received from the vendor. It is supposed to have a tensile strength in the range 60,000 to 70,000 lb/in2. A Brinell hardness test in the receiving department yields a value of HB ¼ 118. (a) Does the steel meet the specification on tensile strength? (b) Estimate the yield strength of the material. Viscosity of Fluids 3.28. Two flat plates, separated by a space of 4 mm, are moving relative to each other at a velocity of 5 m/sec. The space between them is occupied by a fluid of unknown viscosity. The motion of the plates is resisted by a shear stress of 10 Pa because of the viscosity of the fluid. Assuming that the velocity gradient of the fluid is constant, determine the coefficient of viscosity of the fluid. 3.29. Two parallel surfaces, separated by a space of 0.5 in that is occupied by a fluid, are moving relative to each other at a velocity of 25 in/sec. The motion is resisted by a shear stress of 0.3 lb/in2 because of the viscosity of the fluid. If the velocity gradient in the space between the surfaces is constant, determine the viscosity of the fluid. 3.30. A 125.0-mm-diameter shaft rotates inside a stationary bushing whose inside diameter ¼ 125.6 mm and length ¼ 50.0 mm. In the clearance between the shaft and the bushing is a lubricating oil whose viscosity ¼ 0.14 Pa-s. The shaft rotates at a velocity of 400 rev/ min; this speed and the action of the oil are sufficient to keep the shaft centered inside the bushing. Determine the magnitude of the torque due to viscosity that acts to resist the rotation of the shaft. E1C04 11/10/2009 13:13:22 4 Page 67 PHYSICAL PROPERTIES OF MATERIALS Chapter Contents 4.1 Volumetric and Melting Properties 4.1.1 Density 4.1.2 Thermal Expansion 4.1.3 Melting Characteristics 4.2 Thermal Properties 4.2.1 Specific Heat and Thermal Conductivity 4.2.2 Thermal Properties in Manufacturing 4.3 Mass Diffusion 4.4 Electrical Properties 4.4.1 Resistivity and Conductivity 4.4.2 Classes of Materials by Electrical Properties 4.5 Electrochemical Processes Physical properties, as the term is used here, defines the behavior of materials in response to physical forces other than mechanical. They include volumetric, thermal, electrical, and electrochemical properties. Components in a product must do more than simply withstand mechanical stresses. They must conduct electricity (or prevent its conduction), allow heat to be transferred (or allow it to escape), transmit light (or block its transmission), and satisfy myriad other functions. Physical properties are important in manufacturing because they often influence the performance of the process. For example, thermal properties of the work material in machining determine the cutting temperature, which affects how long the tool can be used before it fails. In microelectronics, electrical properties of silicon and the way in which these properties can be altered by various chemical and physical processes comprise the basis of semiconductor manufacturing. This chapter discusses the physical properties that are most important in manufacturing—properties that will be encountered in subsequent chapters of the book. They are divided into major categories such as volumetric, thermal, electrical, and so on. We also relate these properties to manufacturing, as we did in the previous chapter on mechanical properties. 4.1 VOLUMETRIC AND MELTING PROPERTIES These properties are related to the volume of solids and how they are affected by temperature. The properties include density, thermal expansion, and melting point. They are explained in the following, and a listing of typical values for selected engineering materials is presented in Table 4.1. 4.1.1 DENSITY In engineering, the density of a material is its weight per unit volume. Its symbol is r, and typical units are g/cm3 (lb/in3). 67 E1C04 11/10/2009 68 13:13:25 Page 68 Chapter 4/Physical Properties of Materials TABLE 4.1 Volumetric properties in U.S. customary units for selected engineering materials. Coefficient of Thermal Expansion, a Density, r 3 lb/in 3  C 1 6  10  1 F  10 Melting Point, Tm 6  C  Material g/cm F Metals Aluminum Copper Iron Lead Magnesium Nickel Steel Tin Tungsten Zinc 2.70 8.97 7.87 11.35 1.74 8.92 7.87 7.31 19.30 7.15 0.098 0.324 0.284 0.410 0.063 0.322 0.284 0.264 0.697 0.258 24 17 12.1 29 26 13.3 12 23 4.0 40 13.3 9.4 6.7 16.1 14.4 7.4 6.7 12.7 2.2 22.2 660 1083 1539 327 650 1455 1220 1981 2802 621 1202 2651 a a 232 3410 420 449 6170 787 Ceramics Glass Alumina Silica 2.5 3.8 2.66 0.090 0.137 0.096 1.8–9.0 9.0 NA 1.0–5.0 5.0 NA b b NA NA b b Polymers Phenol resins Nylon Teflon Natural rubber Polyethylene (low density) Polystyrene 1.3 1.16 2.2 1.2 0.92 1.05 0.047 0.042 0.079 0.043 0.033 0.038 60 100 100 80 180 60 33 55 55 45 100 33 c c b b b b b b b b b b Compiled from, [2], [3], [4], and other sources. Melting characteristics of steel depend on composition. b Softens at elevated temperatures and does not have a well-defined melting point. c Chemically degrades at high temperatures. NA ¼ not available; value of property for this material could not be obtained. a The density of an element is determined by its atomic number and other factors, such as atomic radius and atomic packing. The term specific gravity expresses the density of a material relative to the density of water and is therefore a ratio with no units. Density is an important consideration in the selection of a material for a given application, but it is generally not the only property of interest. Strength is also important, and the two properties are often related in a strength-to-weight ratio, which is the tensile strength of the material divided by its density. The ratio is useful in comparing materials for structural applications in aircraft, automobiles, and other products in which weight and energy are of concern. 4.1.2 THERMAL EXPANSION The density of a material is a function of temperature. The general relationship is that density decreases with increasing temperature. Put another way, the volume per unit weight increases with temperature. Thermal expansion is the name given to this effect that temperature has on density. It is usually expressed as the coefficient of thermal expansion, which measures the change in length per degree of temperature, as mm/mm/ C (in/in/ F). It is a length ratio rather than a volume ratio because this is easier to measure and apply. It is E1C04 11/10/2009 13:13:25 Page 69 Section 4.1/Volumetric and Melting Properties 69 consistent with the usual design situation in which dimensional changes are of greater interest than volumetric changes. The change in length corresponding to a given temperature change is given by L2  L1 ¼ aL1 (T 2  T 1 ) ð4:1Þ where a ¼ coefficient of thermal expansion,  C1( F1); and L1 and L2 are lengths, mm (in), corresponding, respectively, to temperatures T1 and T2,  C ( F). Values of coefficient of thermal expansion given in Table 4.1 suggest that it has a linear relationship with temperature. This is only an approximation. Not only is length affected by temperature, but the thermal expansion coefficient itself is also affected. For some materials it increases with temperature; for other materials it decreases. These changes are usually not significant enough to be of much concern, and values like those in the table are quite useful in design calculations for the range of temperatures contemplated in service. Changes in the coefficient are more substantial when the metal undergoes a phase transformation, such as from solid to liquid, or from one crystal structure to another. In manufacturing operations, thermal expansion is put to good use in shrink fit and expansion fit assemblies (Section 32.3) in which a part is heated to increase its size or cooled to decrease its size to permit insertion into some other part. When the part returns to ambient temperature, a tightly fitted assembly is obtained. Thermal expansion can be a problem in heat treatment (Chapter 27) and welding (Section 30.6) because of thermal stresses that develop in the material during these processes. 4.1.3 MELTING CHARACTERISTICS For a pure element, the melting point Tm is the temperature at which the material transforms from solid to liquid state. The reverse transformation, from liquid to solid, occurs at the same temperature and is called the freezing point. For crystalline elements, such as metals, the melting and freezing temperatures are the same. A certain amount of heat energy, called the heat of fusion, is required at this temperature to accomplish the transformation from solid to liquid. Melting of a metal element at a specific temperature, as it has been described, assumes equilibrium conditions. Exceptions occur in nature; for example, when a molten metal is cooled, it may remain in the liquid state below its freezing point if nucleation of crystals does not initiate immediately. When this happens, the liquid is said to be supercooled. There are other variations in the melting process—differences in the way melting occurs in different materials. For example, unlike pure metals, most metal alloys do not have a single melting point. Instead, melting begins at a certain temperature, called the solidus, and continues as the temperature increases until finally converting completely to the liquid state at a temperature called the liquidus. Between the two temperatures, the alloy is a mixture of solid and molten metals, the amounts of each being inversely proportional to their relative distances from the liquidus and solidus. Although most alloys behave in this way, exceptions are eutectic alloys that melt (and freeze) at a single temperature. These issues are examined in the discussion of phase diagrams in Chapter 6. Another difference in melting occurs with noncrystalline materials (glasses). In these materials, there is a gradual transition from solid to liquid states. The solid material gradually softens as temperature increases, finally becoming liquid at the melting point. During softening, the material has a consistency of increasing plasticity (increasingly like a fluid) as it gets closer to the melting point. These differences in melting characteristics among pure metals, alloys, and glass are portrayed in Figure 4.1. The plots show changes in density as a function of temperature for three hypothetical materials: a pure metal, an alloy, and glass. Plotted in the figure is the volumetric change, which is the reciprocal of density. E1C04 11/10/2009 70 13:13:25 Page 70 Chapter 4/Physical Properties of Materials FIGURE 4.1 Changes in volume per unit weight (1/density) as a function of temperature for a hypothetical pure metal, alloy, and glass; all exhibiting similar thermal expansion and melting characteristics. The importance of melting in manufacturing is obvious. In metal casting (Chapters 10 and 11), the metal is melted and then poured into a mold cavity. Metals with lower melting points are generally easier to cast, but if the melting temperature is too low, the metal loses its applicability as an engineering material. Melting characteristics of polymers are important in plastic molding and other polymer shaping processes (Chapter 13). Sintering of powdered metals and ceramics requires knowledge of melting points. Sintering does not melt the materials, but the temperatures used in the process must approach the melting point to achieve the required bonding of the powders. 4.2 THERMAL PROPERTIES Much of the previous section is concerned with the effects of temperature on volumetric properties of materials. Certainly, thermal expansion, melting, and heat of fusion are thermal properties because temperature determines the thermal energy level of the atoms, leading to the changes in the materials. The current section examines several additional thermal properties—ones that relate to the storage and flow of heat within a substance. The usual properties of interest are specific heat and thermal conductivity, values of which are compiled for selected materials in Table 4.2. 4.2.1 SPECIFIC HEAT AND THERMAL CONDUCTIVITY The specific heat C of a material is defined as the quantity of heat energy required to increase the temperature of a unit mass of the material by one degree. Some typical values are listed in Table 4.2. To determine the amount of energy needed to heat a certain weight of a metal in a furnace to a given elevated temperature, the following equation can be used H ¼ CW(T 2  T 1 ) ð4:2Þ where H ¼ amount of heat energy, J (Btu); C ¼ specific heat of the material, J/kg  C (Btu/lb F); W ¼ its weight, kg (lb); and (T2  T1) ¼ change in temperature,  C ( F).  E1C04 11/10/2009 13:13:26 Page 71 Section 4.2/Thermal Properties 71 TABLE 4.2 Values of common thermal properties for selected materials. Values are at room temperature, and these values change for different temperatures. Specific Heat Material Cal/g  Ca or Btu/lbm  F Thermal Conductivity Specific Heat J/s mm  C Btu/hr in  F Material Cal/g  Ca or Btu/lbm  F Ceramics Alumina Concrete Thermal Conductivity J/s mm  C Btu/hr in  F 0.18 0.2 0.029 0.012 1.4 0.6 Metals Aluminum Cast iron 0.21 0.11 0.22 0.06 9.75 2.7 Copper Iron Lead Magnesium Nickel 0.092 0.11 0.031 0.25 0.105 0.40 0.072 0.033 0.16 0.070 18.7 2.98 1.68 7.58 2.88 Polymers Phenolics Polyethylene Teflon Natural rubber 0.4 0.5 0.25 0.48 0.00016 0.00034 0.00020 0.00012 0.0077 0.016 0.0096 0.006 Steel Stainless steelb Tin Zinc 0.11 0.11 0.054 0.091 0.046 0.014 0.062 0.112 2.20 0.67 3.0 5.41 Other Water (liquid) Ice 1.00 0.46 0.0006 0.0023 0.029 0.11 Compiled from [2], [3], [6], and other sources. Specific heat has the same numerical value in Btu/lbm-F or Cal/g-C. 1.0 Calory ¼ 4.186 Joule. b Austenitic (18-8) stainless steel. a The volumetric heat storage capacity of a material is often of interest. This is simply density multiplied by specific heat rC. Thus, volumetric specific heat is the heat energy required to raise the temperature of a unit volume of material by one degree, J/mm3  C (Btu/in3  F). Conduction is a fundamental heat-transfer process. It involves transfer of thermal energy within a material from molecule to molecule by purely thermal motions; no transfer of mass occurs. The thermal conductivity of a substance is therefore its capability to transfer heat through itself by this physical mechanism. It is measured by the coefficient of thermal conductivity k, which has typical units of J/s mm  C (Btu/in hr  F). The coefficient of thermal conductivity is generally high in metals, low in ceramics and plastics. The ratio of thermal conductivity to volumetric specific heat is frequently encountered in heat transfer analysis. It is called the thermal diffusivity K and is determined as k K¼ ð4:3Þ rC It can be used to calculate cutting temperatures in machining (Section 21.5.1). 4.2.2 THERMAL PROPERTIES IN MANUFACTURING Thermal properties play an important role in manufacturing because heat generation is common in so many processes. In some operations heat is the energy that accomplishes the process; in others heat is generated as a consequence of the process. Specific heat is of interest for several reasons. In processes that require heating of the material (e.g., casting, heat treating, and hot metal forming), specific heat determines the amount of heat energy needed to raise the temperature to a desired level, according to Eq. (4.2). In many processes carried out at ambient temperature, the mechanical energy to perform the operation is converted to heat, which raises the temperature of the workpart. This is common in machining and cold forming of metals. The temperature rise is a function of the metal’s specific heat. Coolants are often used in machining to reduce these temperatures, and here the fluid’s heat capacity is critical. Water is almost always employed as the base for these fluids because of its high heat-carrying capacity. E1C04 11/10/2009 72 13:13:26 Page 72 Chapter 4/Physical Properties of Materials Thermal conductivity functions to dissipate heat in manufacturing processes, sometimes beneficially, sometimes not. In mechanical processes such as metal forming and machining, much of the power required to operate the process is converted to heat. The ability of the work material and tooling to conduct heat away from its source is highly desirable in these processes. On the other hand, high thermal conductivity of the work metal is undesirable in fusion welding processes such as arc welding. In these operations, the heat input must be concentrated at the joint location so that the metal can be melted. For example, copper is generally difficult to weld because its high thermal conductivity allows heat to be conducted from the energy source into the work too rapidly, inhibiting heat buildup for melting at the joint. 4.3 MASS DIFFUSION In addition to heat transfer in a material, there is also mass transfer. Mass diffusion involves movement of atoms or molecules within a material or across a boundary between two materials in contact. It is perhaps more appealing to one’s intuition that such a phenomenon occurs in liquids and gases, but it also occurs in solids. It occurs in pure metals, in alloys, and between materials that share a common interface. Because of thermal agitation of the atoms in a material (solid, liquid, or gas), atoms are continuously moving about. In liquids and gases, where the level of thermal agitation is high, it is a free-roaming movement. In solids (metals in particular), the atomic motion is facilitated by vacancies and other imperfections in the crystal structure. Diffusion can be illustrated by the series of sketches in Figure 4.2 for the case of two metals suddenly brought into intimate contact with each other. At the start, both metals have their own atomic structure; but with time there is an exchange of atoms, not only across the boundary, but within the separate pieces. Given enough time, the assembly of two pieces will finally reach a uniform composition throughout. Temperature is an important factor in diffusion. At higher temperatures, thermal agitation is greater and the atoms can move about more freely. Another factor is the concentration gradient dc=dx, which indicates the concentration of the two types of atoms in a direction of interest defined by x. The concentration gradient is plotted in Figure 4.2(b) to correspond to the instantaneous distribution of atoms in the assembly. The relationship often used to describe mass diffusion is Fick’s first law:   dc dm ¼ D A dt ð4:4Þ dt where dm ¼ small amount of material transferred, D ¼ diffusion coefficient of the metal, which increases rapidly with temperature, dc=dx ¼ concentration gradient, A ¼ area of the boundary, and dt represents a small time increment. An alternative expression of Eq. (4.4) gives the mass diffusion rate:   dm dc ¼ D A ð4:5Þ dt dt Although these equations are difficult to use in calculations because of the problem of assessing D, they are helpful in understanding diffusion and the variables on which D depends. Mass diffusion is used in several processes. A number of surface-hardening treatments are based on diffusion (Section 27.4), including carburizing and nitriding. Among the welding processes, diffusion welding (Section 30.5.2) is used to join two components by pressing them together and allowing diffusion to occur across the boundary to create a permanent bond. Diffusion is also used in electronics manufacturing to alter the surface chemistry of a semiconductor chip in very localized regions to create circuit details (Section 34.4.3). E1C04 11/10/2009 13:13:27 Page 73 Section 4.4/Electrical Properties 73 Interface Pure A Pure B (1) A A and B (2) B Uniform mixture of A and B (3) (a) FIGURE 4.2 Mass diffusion: (a) model of atoms in two solid blocks in contact: (1) at the start when two pieces are brought together, they each have their individual compositions; (2) after some time, an exchange of atoms has occurred; and (3) eventually, a condition of uniform concentration occurs. The concentration gradient dc=dx for metal A is plotted in (b) of the figure. 4.4 ELECTRICAL PROPERTIES Engineering materials exhibit a great variation in their capacity to conduct electricity. This section defines the physical properties by which this capacity is measured. 4.4.1 RESISTIVITY AND CONDUCTIVITY The flow of electrical current involves movement of charge carriers—infinitesimally small particles possessing an electrical charge. In solids, these charge carriers are electrons. In a liquid solution, charge carriers are positive and negative ions. The movement of charge carriers is driven by the presence of an electric voltage and resisted by the inherent characteristics of the material, such as atomic structure and bonding between atoms and molecules. This is the familiar relationship defined by Ohm’s law I¼ E R where I ¼ current, A; E ¼ voltage, V; and R ¼ electrical resistance, V. ð4:6Þ E1C04 11/10/2009 74 13:13:27 Page 74 Chapter 4/Physical Properties of Materials The resistance in a uniform section of material (e.g., a wire) depends on its length L, cross-sectional area A, and the resistivity of the material r; thus, L A R¼r or r ¼ R ð4:7Þ A L where resistivity has units of V-m2/m or V-m (V-in). Resistivity is the basic property that defines a material’s capability to resist current flow. Table 4.3 lists values of resistivity for selected materials. Resistivity is not a constant; instead it varies, as do so many other properties, with temperature. For metals, it increases with temperature. It is often more convenient to consider a material as conducting electrical current rather than resisting its flow. The conductivity of a material is simply the reciprocal of resistivity: Electrical conductivity ¼ 1 r ð4:8Þ where conductivity has units of (V-m)1 ((V-in)1). 4.4.2 CLASSES OF MATERIALS BY ELECTRICAL PROPERTIES Metals are the best conductors of electricity, because of their metallic bonding. They have the lowest resistivity (Table 4.3). Most ceramics and polymers, whose electrons are tightly bound by covalent and/or ionic bonding, are poor conductors. Many of these materials are used as insulators because they possess high resistivities. An insulator is sometimes referred to as a dielectric, because the term dielectric means nonconductor of direct current. It is a material that can be placed between two electrodes without conducting current between them. However, if the voltage is high enough, the current will suddenly pass through the material; for example, in the form of an arc. The dielectric strength of an insulating material, then, is the electrical potential required to break down the insulator per unit thickness. Appropriate units are volts/m (volts/in). In addition to conductors and insulators (or dielectrics), there are also superconductors and semiconductors. A superconductor is a material that exhibits zero resistivity. It is a phenomenon that has been observed in certain materials at low temperatures TABLE 4.3 Resistivity of selected materials. Resistivity Material Conductors Aluminum Aluminum alloys Cast iron Copper Gold Iron Lead Magnesium Nickel Silver V-m 6 V-in 8 4 Material 7 V-m 10 – 10 2.8  108 4.0  108a 65.0  108a 1.7  108 2.4  108 9.5  108 20.6  108 10 – 10 1.1  106 1.6  106a 25.6  106a 0.67  106 0.95  106 3.7  106 8.1  106 Conductors, continued Steel, low C 17.0  108 Steel, stainless 70.0  108a Tin 11.5  108 Zinc 6.0  108 Carbon 5000  108b Semiconductors 101 – 105 Silicon 1.0  103 4.5  108 6.8  108 1.6  108 1.8  106 2.7  106 0.63  106 Insulators Natural rubber Polyethylene Compiled from various standard sources. Value varies with alloy composition. b Value is approximate. a Resistivity 1012 – 1015 1.0  1012b 100  1012b V-in 6.7  106 27.6  106 4.5  106 2.4  106 2000  106b 102 – 107 1013 – 1017 0.4  1014b 40  1014b E1C04 11/10/2009 13:13:27 Page 75 Section 4.5/Electrochemical Processes 75 approaching absolute zero. One might expect the existence of this phenomenon, because of the significant effect that temperature has on resistivity. That these superconducting materials exist is of great scientific interest. If materials could be developed that exhibit this property at more normal temperatures, there would be significant practical implications in power transmission, electronic switching speeds, and magnetic field applications. Semiconductors have already proved their practical worth: Their applications range from mainframe computers to household appliances and automotive engine controllers. As one would guess, a semiconductor is a material whose resistivity lies between insulators and conductors. The typical range is shown in Table 4.3. The most commonly used semiconductor material today is silicon (Section 7.5.2), largely because of its abundance in nature, relative low cost, and ease of processing. What makes semiconductors uniqueisthe capacityto significantly alter conductivities in their surface chemistries in very localized areas to fabricate integrated circuits (Chapter 34). Electrical properties play an important role in various manufacturing processes. Some of the nontraditional processes use electrical energy to remove material. Electric discharge machining (Section 26.3.1) uses the heat generated by electrical energy in the form of sparks to remove material from metals. Most of the important welding processes use electrical energy to melt the joint metal. Finally, the capacity to alter the electrical properties of semiconductor materials is the basis for microelectronics manufacturing. 4.5 ELECTROCHEMICAL PROCESSES Electrochemistry is a field of science concerned with the relationship between electricity and chemical changes, and with the conversion of electrical and chemical energy. In a water solution, the molecules of an acid, base, or salt are dissociated into positively and negatively charged ions. These ions are the charge carriers in the solution— they allow electric current to be conducted, playing the same role that electrons play in metallic conduction. The ionized solution is called an electrolyte; and electrolytic conduction requires that current enter and leave the solution at electrodes. The positive electrode is called the anode, and the negative electrode is the cathode. The whole arrangement is called an electrolytic cell. At each electrode, some chemical reaction occurs, such as the deposition or dissolution of material, or the decomposition of gas from the solution. Electrolysis is the name given to these chemical changes occurring in the solution. Consider a specific case of electrolysis: decomposition of water, illustrated in Figure 4.3. To accelerate the process, dilute sulfuric acid (H2SO4) is used as the electrolyte, and platinum and carbon (both chemically inert) are used as electrodes. The electrolyte dissociates in the ions Hþ and SO4¼. The Hþ ions are attracted to the negatively charged cathode; upon FIGURE 4.3 Example of electrolysis: decomposition of water. E1C04 11/10/2009 76 13:13:28 Page 76 Chapter 4/Physical Properties of Materials reaching it they acquire an electron and combine into molecules of hydrogen gas: 2Hþ þ 2e ! H2 (gas) ð4:9aÞ ¼ The SO4 ions are attracted to the anode, transferring electrons to it to form additional sulfuric acid and liberate oxygen: 2SO4 ¼  4e þ 2H2 O ! 2H2 SO4 þ O2 (gas) ð4:9bÞ The product H2SO4 is dissociated into ions of H and SO4 ¼ again and so the process continues. In addition totheproductionofhydrogen andoxygengases,as illustrated bytheexample, electrolysis is also used in several other industrial processes. Two examples are (1) electroplating (Section 28.3.1), an operation that adds a thin coating of one metal (e.g., chromium) to the surface of a second metal (e.g., steel) for decorative or other purposes; and (2) electrochemicalmachining (Section26.2),aprocessinwhichmaterialisremoved fromthesurfaceofa metal part. Both these operations rely on electrolysis to either add or remove material from the surface of a metal part. In electroplating, the workpart is set up in the electrolytic circuit as the cathode, so that the positive ions of the coating metal are attracted to the negatively charged part. In electrochemical machining, the workpart is the anode, and a tool with the desired shape is the cathode. The action of electrolysis in this setup is to remove metal from the part surface in regions determined by the shape of the tool as it slowly feeds into the work. The two physical laws that determine the amount of material deposited or removed from a metallic surface were first stated by the British scientist Michael Faraday: + 1. The mass of a substance liberated in an electrolytic cell is proportional to the quantity of electricity passing through the cell. 2. When the same quantity of electricity is passed through different electrolytic cells, the masses of the substances liberated are proportional to their chemical equivalents. Faraday’s laws are used in the subsequent coverage of electroplating and electrochemical machining. REFERENCES [1] Guy, A. G., and Hren, J. J. Elements of Physical Metallurgy, 3rd ed. Addison-Wesley Publishing Company, Reading, Massachusetts, 1974. [2] Flinn, R. A., and Trojan, P. K. Engineering Materials and Their Applications, 5th ed. John Wiley & Sons, New York, 1995. [3] Kreith, F., and Bohn, M. S., Principles of Heat Transfer, 6th ed. CL-Engineering, New York, 2000. [4] Metals Handbook, 10th ed., Vol. 1, Properties and Selection: Iron, Steel, and High Performance Alloys. ASM International, Metals Park, Ohio, 1990. [5] Metals Handbook, 10th ed., Vol. 2, Properties and Selection: Nonferrous Alloys and Special Purpose Materials. ASM International, Metals Park, Ohio, 1990. [6] Van Vlack, L. H. Elements of Materials Science and Engineering, 6th ed. Addison-Wesley, Reading, Massachusetts, 1989. REVIEW QUESTIONS 4.1. Define density as a material property. 4.2. What is the difference in melting characteristics between a pure metal element and an alloy metal? 4.3. Describe the melting characteristics of a noncrystalline material such as glass. 4.4. Define specific heat as a material property. 4.5. What is thermal conductivity as a material property? 4.6. Define thermal diffusivity. 4.7. What are the important variables that affect mass diffusion? 4.8. Define resistivity as a material property. 4.9. Why are metals better conductors of electricity than ceramics and polymers? 4.10. What is dielectric strength as a material property? 4.11. What is an electrolyte? E1C04 11/10/2009 13:13:28 Page 77 Problems 77 MULTIPLE CHOICE QUIZ There are 12 correct answers in the following multiple choice questions (some questions have multiple answers that are correct). To attain a perfect score on the quiz, all correct answers must be given. Each correct answer is worth 1 point. Each omitted answer or wrong answer reduces the score by 1 point, and each additional answer beyond the correct number of answers reduces the score by 1 point. Percentage score on the quiz is based on the total number of correct answers. 4.1. Which one of the following metals has the lowest density: (a) aluminum, (b) copper, (c) magnesium, or (d) tin? 4.2. The thermal expansion properties of polymers are generally (a) greater than, (b) less than, or (c) the same as those of metals? 4.3. In the heating of most metal alloys, melting begins at a certain temperature and concludes at a higher temperature. In these cases, which of the following temperatures marks the beginning of melting: (a) liquidus or (b) solidus? 4.4. Which one of the following materials has the highest specific heat: (a) aluminum, (b) concrete, (c) polyethylene, or (d) water? 4.5. Copper is generally considered easy to weld because of its high thermal conductivity: (a) true or (b) false? 4.6. The mass diffusion rate dm=dt across a boundary between two different metals is a function of which of the following variables (four best answers): (a) concentration gradient dc=dx, (b) contact area, (c) density, (d) melting point, (e) thermal expansion, (f) temperature, and (g) time? 4.7. Which of the following pure metals is the best conductor of electricity: (a) aluminum, (b) copper, (c) gold, or (d) silver? 4.8. A superconductor is characterized by which of the following (one best answer): (a) high conductivity, (b) resistivity properties between those of conductors and semiconductors, (c) very low resistivity, or (d) zero resistivity? 4.9. In an electrolytic cell, the anode is the electrode that is (a) positive or (b) negative. PROBLEMS 4.1. The starting diameter of a shaft is 25.00 mm. This shaft is to be inserted into a hole in an expansion fit assembly operation. To be readily inserted, the shaft must be reduced in diameter by cooling. Determine the temperature to which the shaft must be reduced from room temperature (20 C) in order to reduce its diameter to 24.98 mm. Refer to Table 4.1. 4.2. A bridge built with steel girders is 500 m in length and 12 m in width. Expansion joints are provided to compensate for the change in length in the support girders as the temperature fluctuates. Each expansion joint can compensate for a maximum of 40 mm of change in length. From historical records it is estimated that the minimum and maximum temperatures in the region will be 35 C and 38 C, respectively. What is the minimum number of expansion joints required? 4.3. Aluminum has a density of 2.70 g/cm3 at room temperature (20 C). Determine its density at 650 C, using data in Table 4.1 as a reference. 4.4. With reference to Table 4.1, determine the increase in length of a steel bar whose length ¼ 10.0 in, if the bar is heated from room temperature of 70 F to 500 F. 4.5. With reference to Table 4.2, determine the quantity of heat required to increase the temperature of an aluminum block that is 10 cm  10 cm  10 cm from room temperature (21 C) to 300 C. 4.6. What is the resistance R of a length of copper wire whose length = 10 m and whose diameter = 0.10 mm? Use Table 4.3 as a reference. 4.7. A 16-gage nickel wire (0.0508-in diameter) connects a solenoid to a control circuit that is 32.8 ft away. (a) What is the resistance of the wire? Use Table 4.3 as a reference. (b) If a current was passed through the wire, it would heat up. How does this affect the resistance? 4.8. Aluminum wiring was used in many homes in the 1960s because of the high cost of copper at the time. Aluminum wire that was 12 gauge (a measure of cross-sectional area) was rated at 15 A of current. If copper wire of the same gauge were used to replace the aluminum wire, what current should the wire be capable of carrying if all factors except resistivity are considered equal? Assume that the resistance of the wire is the primary factor that determines the current it can carry and the cross-sectional area and length are the same for the aluminum and copper wires. E1C05 11/10/2009 5 13:16:31 Page 78 DIMENSIONS, SURFACES, AND THEIR MEASUREMENT Chapter Contents 5.1 Dimensions, Tolerances, and Related Attributes 5.1.1 Dimensions and Tolerances 5.1.2 Other Geometric Attributes 5.2 Conventional Measuring Instruments and Gages 5.2.1 Precision Gage Blocks 5.2.2 Measuring Instruments for Linear Dimensions 5.2.3 Comparative Instruments 5.2.4 Fixed Gages 5.2.5 Angular Measurements 5.3 Surfaces 5.3.1 Characteristics of Surfaces 5.3.2 Surface Texture 5.3.3 Surface Integrity 5.4 Measurement of Surfaces 5.4.1 Measurement of Surface Roughness 5.4.2 Evaluation of Surface Integrity 5.5 Effect of Manufacturing Processes In addition to mechanical and physical properties of materials, other factors that determine the performance of a manufactured product include the dimensions and surfaces of its components. Dimensions are the linear or angular sizes of a component specified on the part drawing. Dimensions are important because they determine how well the components of a product fit together during assembly. When fabricating a given component, it is nearly impossible and very costly to make the part to the exact dimension given on the drawing. Instead a limited variation is allowed from the dimension, and that allowable variation is called a tolerance. The surfaces of a component are also important. They affect product performance, assembly fit, and aesthetic appeal that a potential customer might have for the product. A surface is the exterior boundary of an object with its surroundings, which may be another object, a fluid, or space, or combinations of these. The surface encloses the object’s bulk mechanical and physical properties. This chapter discusses dimensions, tolerances, and surfaces—three attributes specified by the product designer and determined by the manufacturing processes used to make the parts and products. It also considers how these attributes are assessed using measuring and gaging devices. A closely related topic is inspection, covered in Chapter 42. 5.1 DIMENSIONS, TOLERANCES, AND RELATED ATTRIBUTES The basic parameters used by design engineers to specify sizes of geometric features on a part drawing are defined in this section. The parameters include dimensions and tolerances, flatness, roundness, and angularity. 78 E1C05 11/10/2009 13:16:32 Page 79 Section 5.2/Conventional Measuring Instruments and Gages 79 FIGURE 5.1 Three ways to specify tolerance limits for a nominal dimension of 2.500: (a) bilateral, (b) unilateral, and (c) limit dimensions. 5.1.1 DIMENSIONS AND TOLERANCES ANSI [3] defines a dimension as ‘‘a numerical value expressed in appropriate units of measure and indicated on a drawing and in other documents along with lines, symbols, and notes to define the size or geometric characteristic, or both, of a part or part feature.’’ Dimensions on part drawings represent nominal or basic sizes of the part and its features. These are the values that the designer would like the part size to be, if the part could be made to an exact size with no errors or variations in the fabrication process. However, there are variations in the manufacturing process, which are manifested as variations in the part size. Tolerances are used to define the limits of the allowed variation. Quoting again from the ANSI standard [3], a tolerance is ‘‘the total amount by which a specific dimension is permitted to vary. The tolerance is the difference between the maximum and minimum limits.’’ Tolerances can be specified in several ways, illustrated in Figure 5.1. Probably most common is the bilateral tolerance, in which the variation is permitted in both positive and negative directions from the nominal dimension. For example, in Figure 5.1(a), the nominal dimension ¼ 2.500 linear units (e.g., mm, in), with an allowable variation of 0.005 units in either direction. Parts outside these limits are unacceptable. It is possible for a bilateral tolerance to be unbalanced; for example, 2.500 +0.010, –0.005 dimensional units. A unilateral tolerance is one in which the variation from the specified dimension is permitted in only one direction, either positive or negative, as in Figure 5.1(b). Limit dimensions are an alternative method to specify the permissible variation in a part feature size; they consist of the maximum and minimum dimensions allowed, as in Figure 5.1(c). 5.1.2 OTHER GEOMETRIC ATTRIBUTES Dimensions and tolerances are normally expressed as linear (length) values. There are other geometric attributes of parts that are also important, such as flatness of a surface, roundness of a shaft or hole, parallelism between two surfaces, and so on. Definitions of these terms are listed in Table 5.1. 5.2 CONVENTIONAL MEASURING INSTRUMENTS AND GAGES Measurement is a procedure in which an unknown quantity is compared with a known standard, using an accepted and consistent system of units. Two systems of units have evolved in the world: (1) the U.S. customary system (U.S.C.S.), and (2) the International System of Units (or SI, for Systeme Internationale d’Unites), more popularly known as the metric system. Both systems are used in parallel throughout this book. The metric system is widely accepted in nearly every part of the industrialized world except the United States, which has stubbornly clung to its U.S.C.S. Gradually, the United States is adopting SI. Measurement provides a numerical value of the quantity of interest, within certain limits of accuracy and precision. Accuracy is the degree to which the measured value agrees with the true value of the quantity of interest. A measurement procedure is accurate when it is E1C05 11/10/2009 80 13:16:33 Page 80 Chapter 5/Dimensions, Surfaces, and their Measurement TABLE 5.1 Definitions of geometric attributes of parts. Angularity—The extent to which a part feature such as a surface or axis is at a specified angle relative to a reference surface. If the angle = 90 , then the attribute is called perpendicularity or squareness. Circularity—For a surface of revolution such as a cylinder, circular hole, or cone, circularity is the degree to which all points on the intersection of the surface and a plane perpendicular to the axis of revolution are equidistant from the axis. For a sphere, circularity is the degree to which all points on the intersection of the surface and a plane passing through the center are equidistant from the center. Concentricity—The degree to which any two (or more) part features such as a cylindrical surface and a circular hole have a common axis. Cylindricity—The degree to which all points on a surface of revolution such as a cylinder are equidistant from the axis of revolution. Flatness—The extent to which all points on a surface lie in a single plane. Parallelism—The degree to which all points on a part feature such as a surface, line, or axis are equidistant from a reference plane or line or axis. Perpendicularity—The degree to which all points on a part feature such as a surface, line, or axis are 90 from a reference plane or line or axis. Roundness—Same as circularity. Squareness—Same as perpendicularity. Straightness—The degree to which a part feature such as a line or axis is a straight line. absent of systematic errors, which are positive or negative deviations from the true value that are consistent from one measurement to the next. Precision is the degree of repeatability in the measurement process. Good precision means that random errors in the measurement procedure are minimized. Random errors are usually associated with human participation in the measurement process. Examples include variations in the setup, imprecise reading of the scale, round-off approximations, and so on. Nonhuman contributors to random error include temperature changes, gradual wear and/or misalignment in the working elements of the device, and other variations. Closely related to measurement is gaging. Gaging (also spelled gauging) determines simply whether the part characteristic meets or does not meet the design specification. It is usually faster than measuring, but scant information is provided about the actual value of the characteristic of interest. The video clip on measurement and gaging illustrates some of the topics discussed in this chapter. VIDEO CLIP Measurement and Gaging. This clip contains three segments: (1) precision, resolution, and accuracy, (2) how to read a vernier caliper, and (3) how to read a micrometer. This section considers the variety of manually operated measuring instruments and gages used to evaluate dimensions such as length and diameter, as well as features such as angles, straightness, and roundness. This type of equipment is found in metrology labs, inspection departments, and tool rooms. The logical starting topic is precision gage blocks. 5.2.1 PRECISION GAGE BLOCKS Precision gage blocks are the standards against which other dimensional measuring instrumentsandgages are compared. Gageblocks are usuallysquare or rectangular. Themeasuring surfaces are finished to be dimensionally accurate and parallel to within several millionths of an inch and are polished to a mirror finish. Several grades of precision gage blocks are available, with closer tolerances for higher precision grades. The highest grade—the master laboratory standard—is made to a tolerance of 0.000,03 mm (0.000,001 in). Depending E1C05 11/10/2009 13:16:33 Page 81 Section 5.2/Conventional Measuring Instruments and Gages 81 on degree of hardness desired and price the user is willing to pay, gage blocks can be made out of any of several hard materials, including tool steel, chrome-plated steel, chromium carbide, or tungsten carbide. Precision gage blocks are available in certain standard sizes or in sets, the latter containing a variety of different-sized blocks. The sizes in a set are systematically determined so they can be stacked to achieve virtually any dimension desired to within 0.0025 mm (0.0001 in). For best results, gage blocks must be used on a flat reference surface, such as a surface plate. A surface plate is a large solid block whose top surface is finished to a flat plane. Most surface plates today are made of granite. Granite has the advantage of being hard, nonrusting, nonmagnetic, long wearing, thermally stable, and easy to maintain. Gage blocks and other high-precision measuring instruments must be used under standard conditions of temperature and other factors that might adversely affect the measurement. By international agreement, 20 C (68 F) has been established as the standard temperature. Metrology labs operate at this standard. If gage blocks or other measuring instruments are used in a factory environment in which the temperature differs from this standard, corrections for thermal expansion or contraction may be required. Also, working gage blocks used for inspection in the shop are subject to wear and must be calibrated periodically against more precise laboratory gage blocks. 5.2.2 MEASURING INSTRUMENTS FOR LINEAR DIMENSIONS Measuring instruments can be divided into two types: graduated and nongraduated. Graduated measuring devices include a set of markings (called graduations) on a linear or angular scale to which the object’s feature of interest can be compared for measurement. Nongraduated measuring devices possess no such scale and are used to make comparisons between dimensions or to transfer a dimension for measurement by a graduated device. The most basic of the graduated measuring devices is the rule (made of steel, and often called a steel rule), used to measure linear dimensions. Rules are available in various lengths. Metric rule lengths include 150, 300, 600, and 1000 mm, with graduations of 1 or 0.5 mm. Common U.S. sizes are 6, 12, and 24 in, with graduations of 1/32, 1/64, or 1/100 in. Calipers are available in either nongraduated or graduated styles. A nongraduated caliper (referred to simply as a caliper) consists of two legs joined by a hinge mechanism, as in Figure 5.2. The ends of the legs are made to contact the surfaces of the object being measured, FIGURE 5.2 Two sizes of outside calipers. (Courtesy of L.S. Starrett Co.) E1C05 11/10/2009 82 13:16:34 Page 82 Chapter 5/Dimensions, Surfaces, and their Measurement FIGURE 5.3 Slide caliper, opposite sides of instrument shown. (Courtesy of L.S. Starrett Co.) and the hinge is designed to hold the legs in position during use. The contacts point either inward or outward. When they point inward, as in Figure 5.2, the instrument is an outside caliper and is used for measuring outside dimensions such as a diameter. When the contacts point outward, it is an inside caliper, which is used to measure the distance between two internal surfaces. An instrument similar in configuration to the caliper is a divider, except that both legs are straight and terminate in hard, sharply pointed contacts. Dividers are used for scaling distances between two points or lines on a surface, and for scribing circles or arcs onto a surface. A variety of graduated calipers are available for various measurement purposes. The simplest is the slide caliper, which consists of a steel rule to which two jaws are added, one fixed at the end of the rule and the other movable, shown in Figure 5.3. Slide calipers can be used for inside or outside measurements, depending on whether the inside or outside jaw faces are used. In use, the jaws are forced into contact with the part surfaces to be measured, and the location of the movable jaw indicates the dimension of interest. Slide calipers permit more accurate and precise measurements than simple rules. A refinement of the slide caliper is the vernier caliper, shown in Figure 5.4. In this device, the movable jaw includes a vernier scale, named after P. Vernier (1580–1637), a French mathematician who invented it. The vernier provides graduations of 0.01 mm in the SI (and 0.001 inch in the U.S. customary scale), much more precise than the slide caliper. The micrometer is a widely used and very accurate measuring device, the most common form of which consists of a spindle and a C-shaped anvil, as in Figure 5.5. The spindle is moved relative to the fixed anvil by means of an accurate screw thread. On a typical U.S. micrometer, each rotation of the spindle provides 0.025 in of linear travel. Attached to the spindle is a thimble graduated with 25 marks around its circumference, each mark corresponding to 0.001 in. The micrometer sleeve is usually equipped with a vernier, E1C05 11/10/2009 13:16:35 Page 83 Section 5.2/Conventional Measuring Instruments and Gages 83 FIGURE 5.4 Vernier caliper. (Courtesy of L.S. Starrett Co.) allowing resolutions as close as 0.0001 in. On a micrometer with metric scale, graduations are 0.01 mm. Modern micrometers (and graduated calipers) are available with electronic devices that display a digital readout of the measurement (as in the figure). These instruments are easier to read and eliminate much of the human error associated with reading conventional graduated devices. The most common micrometer types are (1) external micrometer, Figure 5.5, also called an outside micrometer, which comes in a variety of standard anvil sizes; (2) internal micrometer, or inside micrometer, which consists of a head assembly and a set of rods of different lengths to measure various inside dimensions that might be encountered; and (3) depth micrometer, similar to an inside micrometer but adapted to measure hole depths. FIGURE 5.5 External micrometer, standard 1-in size with digital readout. (Courtesy of L. S. Starrett Co.) E1C05 11/10/2009 84 13:16:38 Page 84 Chapter 5/Dimensions, Surfaces, and their Measurement FIGURE 5.6 Dial indicator: top view shows dial and graduated face; bottom view shows rear of instrument with cover plate removed. (Courtesy of Federal Products Co., Providence, RI.) 5.2.3 COMPARATIVE INSTRUMENTS Comparative instruments are used to make dimensional comparisons between two objects, such as a workpart and a reference surface. They are usually not capable of providing an absolute measurement of the quantity of interest; instead, they measure the magnitude and direction of the deviation between two objects. Instruments in this category include mechanical and electronic gages. Mechanical Gages: Dial Indicators Mechanical gages are designed to mechanically magnify the deviation to permit observation. The most common instrument in this category is the dial indicator (Figure 5.6), which converts and amplifies the linear movement of a contact pointer into rotation of a dial needle. The dial is graduated in small units such as 0.01 mm (or 0.001 in). Dial indicators are used in many applications to measure straightness, flatness, parallelism, squareness, roundness, and runout. A typical setup for measuring runout is illustrated in Figure 5.7. Electronic Gages Electronic gages are a family of measuring and gaging instruments based on transducers capable of converting a linear displacement into an electrical signal. The electrical signal is then amplified and transformed into a suitable data format such as a digital readout, as in Figure 5.5. Applications of electronic gages have grown rapidly in recent years, driven by advances in microprocessor technology. They are gradually replacing many of the conventional measuring and gaging devices. Advantages of electronic gages include (1) good sensitivity, accuracy, precision, repeatability, and speed of response; (2) ability to sense very small dimensions—down to 0.025 mm (1 m-in.); (3) ease of operation; (4) reduced FIGURE 5.7 Dial indicator setup to measure runout; as part is rotated about its center, variations in outside surface relative to center are indicated on the dial. E1C05 11/10/2009 13:16:39 Page 85 Section 5.2/Conventional Measuring Instruments and Gages 85 human error; (5) electrical signal that can be displayed in various formats; and (6) capability to be interfaced with computer systems for data processing. 5.2.4 FIXED GAGES A fixed gage is a physical replica of the part dimension to be assessed. There are two basic categories: master gage and limit gage. A master gage is fabricated to be a direct replica of the nominal size of the part dimension. It is generally used for setting up a comparative measuring instrument, such as a dial indicator; or for calibrating a measuring device. A limit gage is fabricated to be a reverse replica of the part dimension and is designed to check the dimension at one or more of its tolerance limits. A limit gage often consists of two gages in one piece, the first for checking the lower limit of the tolerance on the part dimension, and the other for checking the upper limit. These gages are popularly known as GO/NO-GO gages, because one gage limit allows the part to be inserted, whereas the other limit does not. The GO limit is used to check the dimension at its maximum material condition; this is the minimum size for an internal feature such as a hole, and it is the maximum size for an external feature such as an outside diameter. The NO-GO limit is used to inspect the minimum material condition of the dimension in question. Common limit gages are snap gages and ring gages for checking outside part dimensions, and plug gages for checking inside dimensions. A snap gage consists of a C-shaped frame with gaging surfaces located in the jaws of the frame, as in Figure 5.8. It has two gage buttons, the first being the GO gage, and the second being the NO-GO gage. Snap gages are used for checking outside dimensions such as diameter, width, thickness, and similar surfaces. Ring gages are used for checking cylindrical diameters. For a given application, a pair of gages is usually required, one GO and the other NO-GO. Each gage is a ring whose opening is machined to one of the tolerance limits of the part diameter. For ease of handling, the outside of the ring is knurled. The two gages are distinguished by the presence of a groove around the outside of the NO-GO ring. The most common limit gage for checking hole diameter is the plug gage. The typical gage consists of a handle to which are attached two accurately ground cylindrical pieces (plugs) of hardened steel, as in Figure 5.9. The cylindrical plugs serve as the GO and NO-GO FIGURE 5.8 Snap gage for measuring diameter of a part; difference in height of GO and NOGO gage buttons is exaggerated. FIGURE 5.9 Plug gage; difference in diameters of GO and NO-GO plugs is exaggerated. E1C05 11/10/2009 86 13:16:40 Page 86 Chapter 5/Dimensions, Surfaces, and their Measurement gages. Other gages similar to the plug gage include taper gages, consisting of a tapered plug for checking tapered holes; and thread gages, in which the plug is threaded for checking internal threads on parts. Fixed gages are easy to use, and the time required to complete an inspection is almost always less than when a measuring instrument is employed. Fixed gages were a fundamental element in the development of interchangeable parts manufacturing (Historical Note 1.1). They provided the means by which parts could be made to tolerances that were sufficiently close for assembly without filing and fitting. Their disadvantage is that they provide little if any information on the actual part size; they only indicate whether the size is within tolerance. Today, with the availability of high-speed electronic measuring instruments, and with the need for statistical process control of part sizes, use of gages is gradually giving way to instruments that provide actual measurements of the dimension of interest. 5.2.5 ANGULAR MEASUREMENTS Angles can be measured using any of several styles of protractor. A simple protractor consists of a blade that pivots relative to a semicircular head that is graduated in angular units (e.g., degrees, radians). To use, the blade is rotated to a position corresponding to some part angle to be measured, and the angle is read off the angular scale. A bevel protractor (Figure 5.10) consists of two straight blades that pivot relative to each other. The pivot assembly has a protractor scale that permits the angle formed by the blades to be read. When equipped with a vernier, the bevel protractor can be read to about 5 min; without a vernier the resolution is only about 1 degree. High precision in angular measurements can be made using a sine bar, illustrated in Figure 5.11. One possible setup consists of a flat steel straight edge (the sine bar), and two precision rolls set a known distance apart on the bar. The straight edge is aligned with the part angle to be measured, and gage blocks or other accurate linear measurements are made to determine height. The procedure is carried out on a surface plate to achieve most accurate results. This height H and the length L of the sine bar between rolls are used to calculate the angle A using sin A ¼ FIGURE 5.10 Bevel protractor with vernier scale. (Courtesy of L.S. Starrett Co.) H L ð5:1Þ E1C05 11/10/2009 13:16:40 Page 87 Section 5.3/Surfaces 87 FIGURE 5.11 Setup for using a sine bar. 5.3 SURFACES A surface is what one touches when holding an object, such as a manufactured part. The designer specifies the part dimensions, relating the various surfaces to each other. These nominal surfaces, representing the intended surface contour of the part, are defined by lines in the engineering drawing. The nominal surfaces appear as absolutely straight lines, ideal circles, round holes, and other edges and surfaces that are geometrically perfect. The actual surfaces of a manufactured part are determined by the processes used to make it. The variety of processes available in manufacturing result in wide variations in surface characteristics, and it is important for engineers to understand the technology of surfaces. Surfaces are commercially and technologically important for a number of reasons, different reasons for different applications: (1) Aesthetic reasons—surfaces that are smooth and free of scratches and blemishes are more likely to give a favorable impression to the customer. (2) Surfaces affect safety. (3) Friction and wear depend on surface characteristics. (4) Surfaces affect mechanical and physical properties; for example, surface flaws can be points of stress concentration. (5) Assembly of parts is affected by their surfaces; for example, the strength of adhesively bonded joints (Section 31.3) is increased when the surfaces are slightly rough. (6) Smooth surfaces make better electrical contacts. Surface technology is concerned with (1) defining the characteristics of a surface, (2) surface texture, (3) surface integrity, and (4) the relationship between manufacturing processes and the characteristics of the resulting surface. The first three topics are covered in this section; the final topic is presented in Section 5.5. 5.3.1 CHARACTERISTICS OF SURFACES A microscopic view of a part’s surface reveals its irregularities and imperfections. The features of a typical surface are illustrated in the highly magnified cross section of the surface of a metal part in Figure 5.12. Although the discussion here is focused on metallic surfaces, FIGURE 5.12 A magnified cross section of a typical metallic part surface. E1C05 11/10/2009 88 13:16:41 Page 88 Chapter 5/Dimensions, Surfaces, and their Measurement these comments apply to ceramics and polymers, with modifications owing to differences in structure of these materials. The bulk of the part, referred to as the substrate, has a grain structure that depends on previous processing of the metal; for example, the metal’s substrate structure is affected by its chemical composition, the casting process originally used on the metal, and any deformation operations and heat treatments performed on the casting. The exterior of the part is a surface whose topography is anything but straight and smooth. In this highly magnified cross section, the surface has roughness, waviness, and flaws. Although not shown here, it also possesses a pattern and/or direction resulting from the mechanical process that produced it. All of these geometric features are included in the term surface texture. Just below the surface is a layer of metal whose structure differs from that of the substrate. This is called the altered layer, and it is a manifestation of the actions that have been visited upon the surface during its creation and afterward. Manufacturing processes involve energy, usually in large amounts, which operates on the part against its surface. The altered layer may result from work hardening (mechanical energy), heating (thermal energy), chemical treatment, or even electrical energy. The metal in this layer is affected by the application of energy, and its microstructure is altered accordingly. This altered layer falls within the scope of surface integrity, which is concerned with the definition, specification, and control of the surface layers of a material (most commonly metals) in manufacturing and subsequent performance in service. The scope of surface integrity is usually interpreted to include surface texture as well as the altered layer beneath. In addition, most metal surfaces are coated with an oxide film, given sufficient time after processing for the film to form. Aluminum forms a hard, dense, thin film of Al2O3 on its surface (which serves to protect the substrate from corrosion), and iron forms oxides of several chemistries on its surface (rust, which provides virtually no protection at all). There is also likely to be moisture, dirt, oil, adsorbed gases, and other contaminants on the part’s surface. 5.3.2 SURFACE TEXTURE Surface texture consists of the repetitive and/or random deviations from the nominal surface of an object; it is defined by four features: roughness, waviness, lay, and flaws, shown in Figure 5.13. Roughness refers to the small, finely spaced deviations from the nominal surface that are determined by the material characteristics and the process that formed the surface. Waviness is defined as the deviations of much larger spacing; they occur because of work FIGURE 5.13 Surface texture features. E1C05 11/10/2009 13:16:41 Page 89 Section 5.3/Surfaces FIGURE 5.14 89 Possible lays of a surface. (Source: [1]). deflection, vibration, heat treatment, and similar factors. Roughness is superimposed on waviness. Lay is the predominant direction or pattern of the surface texture. It is determined by the manufacturing method used to create the surface, usually from the action of a cutting tool. Figure 5.14 presents most of the possible lays a surface can take, together with the symbol used by a designer to specify them. Finally, flaws are irregularities that occur occasionally on the surface; these include cracks, scratches, inclusions, and similar defects in the surface. Although some of the flaws relate to surface texture, they also affect surface integrity (Section 5.2.3). Surface Roughness and Surface Finish Surface roughness is a measurable characteristic based on the roughness deviations as defined in the preceding. Surface finish is a more subjective term denoting smoothness and general quality of a surface. In popular usage, surface finish is often used as a synonym for surface roughness. The most commonly used measure of surface texture is surface roughness. With respect to Figure 5.15, surface roughness can be defined as the average of the vertical deviations from the nominal surface over a specified surface length. An arithmetic average (AA) is generally used, based on the absolute values of the deviations, and this roughness value is referred to by the name average roughness. In equation form ZLm Ra ¼ jyj dx Lm ð5:2Þ 0 where Ra ¼ arithmetic mean value of roughness, m (in); y ¼ the vertical deviation from nominal surface (converted to absolute value), m (in); and Lm ¼ the specified distance over which the surface deviations are measured. FIGURE 5.15 Deviations from nominal surface used in the two definitions of surface roughness. E1C05 11/10/2009 90 13:16:41 Page 90 Chapter 5/Dimensions, Surfaces, and their Measurement An approximation of Eq. (5.2), perhaps easier to comprehend, is given by Ra ¼ n X jy j i i¼1 n ð5:3Þ where Ra has the same meaning as above; yi ¼ vertical deviations converted to absolute value and identified by the subscript i, m (in); and n ¼ the number of deviations included in Lm. The units in these equations are meters and inches. In fact, the scale of the deviations is very small, so more appropriate units are mm (mm ¼ m  106 ¼ mm  103) or m-in (m-in ¼ inch  106). These are the units commonly used to express surface roughness. The AA method is the most widely used averaging method for surface roughness today. An alternative, sometimes used in the United States, is the root-mean-square (RMS) average, which is the square root of the mean of the squared deviations over the measuring length. RMS surface roughness values will almost always be greater than the AA values because the larger deviations will figure more prominently in the calculation of the RMS value. Surface roughness suffers the same kinds of deficiencies of any single measure used to assess a complex physical attribute. For example, it fails to account for the lay of the surface pattern; thus, surface roughness may vary significantly, depending on the direction in which it is measured. Another deficiency is that waviness can be included in the Ra computation. To deal with this problem, a parameter called the cutoff length is used as a filter that separates the waviness in a measured surface from the roughness deviations. In effect, the cutoff length is a sampling distance along the surface. A sampling distance shorter than the waviness width will eliminate the vertical deviations associated with waviness and only include those associated with roughness. The most common cutoff length used in practice is 0.8 mm (0.030 in). The measuring length Lm is normally set at about five times the cutoff length. The limitations of surface roughness have motivated the development of additional measures that more completely describe the topography of a given surface. These measures include three-dimensional graphical renderings of the surface, as described in [17]. Symbols for Surface Texture Designers specify surface texture on an engineering drawing by means of symbols as in Figure 5.16. The symbol designating surface texture parameters is a check mark (looks like a square root sign), with entries as indicated for average roughness, waviness, cutoff, lay, and maximum roughness spacing. The symbols for lay are from Figure 5.14. FIGURE 5.16 Surface texture symbols in engineering drawings: (a) the symbol, and (b) symbol with identification labels. Values of Ra are given in microinches; units for other measures are given in inches. Designers do not always specify all of the parameters on engineering drawings. E1C05 11/10/2009 13:16:41 Page 91 Section 5.3/Surfaces 91 5.3.3 SURFACE INTEGRITY Surface texture alone does not completely describe a surface. There may be metallurgical or other changes in the material immediately beneath the surface that can have a significant effect on its mechanical properties. Surface integrity is the study and control of this subsurface layer and any changes in it because of processing that may influence the performance of the finished part or product. This subsurface layer was previously referred to as the altered layer when its structure differs from the substrate, as in Figure 5.12. The possible alterations and injuries to the subsurface layer that can occur in manufacturing are listed in Table 5.2. The surface changes are caused by the application of various forms of energy during processing—mechanical, thermal, chemical, and electrical. Mechanical energy is the most common form used in manufacturing; it is applied against the work material in operations such as metal forming (e.g., forging, extrusion), pressworking, and machining. Although its primary function in these processes is to change the geometry of the workpart, mechanical energy can also cause residual stresses, work hardening, and cracks TABLE 5.2 Surface and subsurface alterations that define surface integrity.a Absorption are impurities that are absorbed and retained in surface layers of the base material, possibly leading to embrittlement or other property changes. Alloy depletion occurs when critical alloying elements are lost from the surface layers, with possible loss of properties in the metal. Cracks are narrow ruptures or separations either at or below the surface that alter the continuity of the material. Cracks are characterized by sharp edges and length-to-width ratios of 4:1 or more. They are classified as macroscopic (can be observed with magnification of 10 or less) and microscopic (requires magnification of more than 10). Craters are rough surface depressions left in the surface by short circuit discharges; associated with electrical processing methods such as electric discharge machining and electrochemical machining (Chapter 26). Hardness changes refer to hardness differences at or near the surface. Heat affected zone are regions of the metal that are affected by the application of thermal energy; the regions are not melted but are sufficiently heated that they undergo metallurgical changes that affect properties. Abbreviated HAZ, the effect is most prominent in fusion welding operations (Chapter 31). Inclusions are small particles of material incorporated into the surface layers during processing; they are a discontinuity in the base material. Their composition usually differs from the base material. a Compiled from [2]. Intergranular attack refers to various forms of chemical reactions at the surface, including intergranular corrosion and oxidation. Laps, folds, seams are irregularities and defects in the surface caused by plastic working of overlapping surfaces. Pits are shallow depressions with rounded edges formed by any of several mechanisms, including selective etching or corrosion; removal of surface inclusions; mechanically formed dents; or electrochemical action. Plastic deformation refers to microstructural changes from deforming the metal at the surface; it results in strain hardening. Recrystallization involves the formation of new grains in strain hardened metals; associated with heating of metal parts that have been deformed. Redeposited metal is metal that is removed from the surface in the molten state and then reattached prior to solidification. Resolidified metal is a portion of the surface that is melted during processing and then solidified without detaching from the surface. The name remelted metal is also used for resolidified metal. Recast metal is a term that includes both redeposited and resolidified metal. Residual stresses are stresses remaining in the material after processing. Selective etch is a form of chemical attack that concentrates on certain components in the base material. E1C05 11/10/2009 92 13:16:41 Page 92 Chapter 5/Dimensions, Surfaces, and their Measurement TABLE 5.3 Forms of energy applied in manufacturing and the resulting possible surface and subsurface alterations that can occur.a Mechanical Thermal Chemical Electrical Residual stresses in subsurface layer Intergranular attack Changes in conductivity and/or magnetism Cracks—microscopic and macroscopic Metallurgical changes (recrystallization, grain size changes, phase changes at surface) Redeposited or resolidified material Chemical contamination Craters resulting from short circuits during certain electrical processing techniques Plastic deformation Heat-affected zone Laps, folds, or seams Hardness changes Voids or inclusions Hardness variations (e.g., work hardening) a Absorption of elements such as H and Cl Corrosion, pitting, and etching Dissolving of microconstituents Alloy depletion Based on [2]. in the surface layers. Table 5.3 indicates the various types of surface and subsurface alterations that are attributable to the different forms of energy applied in manufacturing. Most of the alterations in the table refer to metals, for which surface integrity has been most intensively studied. 5.4 MEASUREMENT OF SURFACES Surfaces are described as consisting of two parameters: (1) surface texture and (2) surface integrity. This section is concerned with the measurement of these two parameters. 5.4.1 MEASUREMENT OF SURFACE ROUGHNESS Various methods are used to assess surface roughness. They can be divided into three categories: (1) subjective comparison with standard test surfaces, (2) stylus electronic instruments, and (3) optical techniques. Standard Test Surfaces Sets of standard surface finish blocks are available, produced to specified roughness values.1 To estimate the roughness of a given test specimen, the surface is compared with the standard both visually and by the ‘‘fingernail test.’’ In this test, the user gently scratches the surfaces of the specimen and the standards, judging which standard is closest to the specimen. Standard test surfaces are a convenient way for a machine operator to obtain an estimate of surface roughness. They are also useful for design engineers in judging what value of surface roughness to specify on a part drawing. Stylus Instruments The disadvantage of the fingernail test is its subjectivity. Several stylus-type instruments are commercially available to measure surface roughness—similar to 1 In the U.S.C.S., these blocks have surfaces with roughness values of 2, 4, 8, 16, 32, 64, or 128 microinches. E1C05 11/10/2009 13:16:41 Page 93 Section 5.4/Measurement of Surfaces 93 FIGURE 5.17 Stylustype instrument for measuring surface roughness. (Courtesy of Giddings & Lewis, Measurement Systems Division.) the fingernail test, but more scientific. An example is the Profilometer, shown in Figure 5.17. In these electronic devices, a cone-shaped diamond stylus with point radius of about 0.005 mm (0.0002 in) and 90 tip angle is traversed across the test surface at a constant slow speed. The operation is depicted in Figure 5.18. As the stylus head is traversed horizontally, it also moves vertically to follow the surface deviations. The vertical movement is converted into an electronic signal that represents the topography of the surface. This can be displayed as either a profile of the actual surface or an average roughness value. Profiling devices use a separate flat plane as the nominal reference against which deviations are measured. The output is a plot of the surface contour along the line traversed by the stylus. This type of system can identify both roughness and waviness in the test surface. Averaging devices reduce the roughness deviations to a single value Ra. They use skids riding on the actual surface to establish the nominal reference plane. The skids act as a mechanical filter to reduce the effect of waviness in the surface; in effect, these averaging devices electronically perform the computations in Eq. (5.1). Optical Techniques Most other surface-measuring instruments employ optical techniques to assess roughness. These techniques are based on light reflectance from the surface, light scatter or diffusion, and laser technology. They are useful in applications where stylus contact with the surface is undesirable. Some of the techniques permit very-high-speed operation, thus making 100% inspection feasible. However, the optical techniques yield values that do not always correlate well with roughness measurements made by stylus-type instruments. FIGURE 5.18 Sketch illustrating the operation of stylus-type instrument. Stylus head traverses horizontally across surface, while stylus moves vertically to follow surface profile. Vertical movement is converted into either (1) a profile of the surface or (2) the average roughness value. E1C05 11/10/2009 94 13:16:42 Page 94 Chapter 5/Dimensions, Surfaces, and their Measurement 5.4.2 EVALUATION OF SURFACE INTEGRITY Surface integrity is more difficult to assess than surface roughness. Some of the techniques to inspect for subsurface changes are destructive to the material specimen. Evaluation techniques for surface integrity include the following: å Surface texture. Surface roughness, designation of lay, and other measures provide superficial data on surface integrity. This type of testing is relatively simple to perform and is always included in the evaluation of surface integrity. å Visual examination. Visual examination can reveal various surface flaws such as cracks, craters, laps, and seams. This type of assessment is often augmented by fluorescent and photographic techniques. å Microstructural examination. This involves standard metallographic techniques for preparing cross sections and obtaining photomicrographs for examination of microstructure in the surface layers compared with the substrate. å Microhardness profile. Hardness differences near the surface can be detected using microhardness measurement techniques such as Knoop and Vickers (Section 3.2.1). The part is sectioned, and hardness is plotted against distance below the surface to obtain a hardness profile of the cross section. å Residual stress profile. X-ray diffraction techniques can be employed to measure residual stresses in the surface layers of a part. 5.5 EFFECT OF MANUFACTURING PROCESSES The ability to achieve a certain tolerance or surface is a function of the manufacturing process. This section describes the general capabilities of various processes in terms of tolerance and surface roughness and surface integrity. Some manufacturing processes are inherently more accurate than others. Most machining processes are quite accurate, capable of tolerances of 0.05 mm (0.002 in) or better. By contrast, sand castings are generally inaccurate, and tolerances of 10 to 20 times those used for machined parts should be specified. Table 5.4 lists a variety of manufacturing processes and indicates the typical tolerances for each process. Tolerances are TABLE 5.4 Typical tolerance limits, based on process capability (Section 42.2), for various manufacturing processes.b Process Sand casting Cast iron Steel Aluminum Die casting Plastic molding: Polyethylene Polystyrene Machining: Drilling, 6 mm (0.25 in) Milling Turning b Typical Tolerance, mm (in) 1.3 (0.050) 1.5 (0.060) 0.5 (0.020) 0.12 (0.005) 0.3 (0.010) 0.15 (0.006) 0.080.03 (+0.003/0.001) 0.08 (0.003) 0.05 (0.002) Process Typical Tolerance, mm (in) Abrasive Grinding Lapping Honing Nontraditional and thermal Chemical machining Electric discharge Electrochem. grind Electrochem. machine Electron beam cutting Laser beam cutting Plasma arc cutting 0.008 (0.0003) 0.005 (0.0002) 0.005 (0.0002) 0.08 (0.003) 0.025 (0.001) 0.025 (0.001) 0.05 (0.002) 0.08 (0.003) 0.08 (0.003) 1.3 (0.050) Compiled from [4], [5], and other sources. For each process category, tolerances vary depending on process parameters. Also, tolerances increase with part size. E1C05 11/10/2009 13:16:42 Page 95 References TABLE 5.5 Surface roughness values produced by the various manufacturing processes.a Process Typical Finish Roughness Rangeb Process Typical Finish Roughness Rangeb Casting: Die casting Investment Sand casting Good Good Poor 1–2 (30–65) 1.5–3 (50–100) 12–25 (500–1000) Abrasive: Grinding Honing Lapping Very good Very good Excellent 0.1–2 (5–75) 0.1–1 (4–30) 0.05–0.5 (2–15) Metal forming: Cold rolling Good 1–3 (25–125) Excellent Excellent 0.1–0.5 (5–15) 0.02–0.3 (1–10) Good Good Poor 1–3 (25–125) 1–4 (30–150) 12–25 (500–1000) Nontraditional: Chemical milling Electrochemical Medium Good 1.5–5 (50–200) 0.2–2 (10–100) Good 0.5–6 (15–250) Medium Good Good Medium 1.5–6 (60–250) 1–6 (30–250) 1–3 (30–125) 1.5–12 (60–500) Electric discharge Electron beam Laser beam Thermal: Arc welding Flame cutting Medium Medium Medium 1.5–15 (50–500) 1.5–15 (50–500) 1.5–15 (50–500) Poor Poor 5–25 (250–1000) 12–25 (500–1000) Poor 3–25 (100–1000) Good 0.5–6 (15–250) Sheet metal draw Cold extrusion Hot rolling Machining: Boring Drilling Milling Reaming Shaping and planing Sawing Turning a 95 Polishing Superfinish Plasma arc cutting Poor 12–25 (500–1000) Compiled from [1], [2], and other sources. Roughness range values are given, mm (m-in). Roughness can vary significantly for a given process, depending on process parameters. b based on the process capability for the particular manufacturing operation, as defined in Section 42.2. The tolerance that should be specified is a function of part size; larger parts require more generous tolerances. The table lists tolerance for moderately sized parts in each processing category. The manufacturing process determines surface finish and surface integrity. Some processes are capable of producing better surfaces than others. In general, processing cost increases with improvement in surface finish. This is because additional operations and more time are usually required to obtain increasingly better surfaces. Processes noted for providing superior finishes include honing, lapping, polishing, and superfinishing (Chapter 25). Table 5.5 indicates the usual surface roughness that can be expected from various manufacturing processes. REFERENCES [1] American National Standards Institute, Inc. Surface Texture, ANSI B46.1-1978. American Society of Mechanical Engineers, New York, 1978. [2] American National Standards Institute, Inc. Surface Integrity, ANSI B211.1-1986. Society of Manufacturing Engineers, Dearborn, Michigan, 1986. [3] American National Standards Institute, Inc. Dimensioning and Tolerancing, ANSI Y14.5M-1982. American Society of Mechanical Engineers, New York, 1982. [4] Bakerjian, R. and Mitchell, P. Tool and Manufacturing Engineers Handbook, 4th ed., Vol. VI, Design for Manufacturability. Society of Manufacturing Engineers, Dearborn, Michigan, 1992. [5] Brown & Sharpe. Handbook of Metrology. North Kingston, Rhode Island, 1992. [6] Curtis, M., Handbook of Dimensional Measurement, 4th ed. Industrial Press, New York, 2007. [7] Drozda, T. J. and Wick, C. Tool and Manufacturing EngineersHandbook,4thed.,Vol. I, Machining. Society ofManufacturingEngineers,Dearborn,Michigan,1983. E1C05 11/10/2009 96 13:16:43 Page 96 Chapter 5/Dimensions, Surfaces, and their Measurement [8] Farago, F. T. Handbook of Dimensional Measurement, 3rd ed. Industrial Press Inc., New York, 1994. [9] Machining Data Handbook, 3rd ed., Vol. II. Machinability Data Center, Cincinnati, Ohio, 1980, Ch. 18. [10] Mummery, L. Surface Texture Analysis—The Handbook. Hommelwerke Gmbh, Germany, 1990. [11] Oberg, E., Jones, F. D., Horton, H. L., and Ryffel, H. Machinery’s Handbook, 26th ed. Industrial Press, New York, 2000. [12] Schaffer, G. H.‘‘The Many Faces of Surface Texture,’’ Special Report 801, American Machinist and Automated Manufacturing, June 1988, pp. 61–68. [13] Sheffield Measurement, a Cross & Trecker Company, Surface Texture and Roundness Measurement Handbook, Dayton, Ohio, 1991. [14] Spitler, D., Lantrip, J., Nee, J., and Smith, D. A. Fundamentals of Tool Design, 5th ed. Society of Manufacturing Engineers, Dearborn, Michigan, 2003. [15] S. Starrett Company. Tools and Rules. Athol, Massachusetts, 1992. [16] Wick, C., and Veilleux, R. F. Tool and Manufacturing Engineers Handbook, 4th ed., Vol. IV, Quality Control and Assembly. Society of Manufacturing Engineers, Dearborn, Michigan, 1987, Section 1. [17] Zecchino, M.‘‘Why Average Roughness Is Not Enough,’’ Advanced Materials & Processes, March 2003, pp. 25–28. REVIEW QUESTIONS 5.1. What is a tolerance? 5.2. What is the difference between a bilateral tolerance and a unilateral tolerance? 5.3. What is accuracy in measurement? 5.4. What is precision in measurement? 5.5. What is meant by the term graduated measuring device? 5.6. What are some of the reasons why surfaces are important? 5.7. Define nominal surface. 5.8. Define surface texture. 5.9. How is surface texture distinguished from surface integrity? 5.10. Within the scope of surface texture, how is roughness distinguished from waviness? 5.11. Surface roughness is a measurable aspect of surface texture; what does surface roughness mean? 5.12. Indicate some of the limitations of using surface roughness as a measure of surface texture. 5.13. Identify some of the changes and injuries that can occur at or immediately below the surface of a metal. 5.14. What causes the various types of changes that occur in the altered layer just beneath the surface? 5.15. What are the common methods for assessing surface roughness? 5.16. Name some manufacturing processes that produce very poor surface finishes. 5.17. Name some manufacturing processes that produce very good or excellent surface finishes. 5.18. (Video) Based on the video about vernier calipers, are the markings on the vernier plate (moveable scale) the same spacing, slightly closer, or slightly further apart compared to the stationary bar? 5.19. (Video) Based on the video about vernier calipers, explain how to read the scale on a vernier caliper. 5.20. (Video) Based on the video about micrometers, explain the primary factor that makes an English micrometer different from a metric micrometer. MULTIPLE CHOICE QUIZ There are 19 correct answers in the following multiple choice questions (some questions have multiple answers that are correct). To attain a perfect score on the quiz, all correct answers must be given. Each correct answer is worth 1 point. Each omitted answer or wrong answer reduces the score by 1 point, and each additional answer beyond the correct number of answers reduces the score by 1 point. Percentage score on the quiz is based on the total number of correct answers. 5.1. A tolerance is which one of the following: (a) clearance between a shaft and a mating hole, (b) measurement error, (c) total permissible variation from a specified dimension, or (d) variation in manufacturing? 5.2. Which of the following two geometric terms have the same meaning: (a) circularity, (b) concentricity, (c) cylindricity, and (d) roundness? 5.3. A surface plate is most typically made of which one of the following materials: (a) aluminum oxide ceramic, (b) cast iron, (c) granite, (d) hard polymers, or (e) stainless steel? 5.4. An outside micrometer would be appropriate for measuring which of the following (two correct answers): (a) hole depth, (b) hole diameter, (c) E1C05 11/10/2009 13:16:43 Page 97 Problems part length, (d) shaft diameter, and (e) surface roughness? 5.5. In a GO/NO-GO gage, which one of the following best describes the function of the GO gage: (a) checks limit of maximum tolerance, (b) checks maximum material condition, (c) checks maximum size, (d) checks minimum material condition, or (e) checks minimum size? 5.6. Which of the following are likely to be GO/NO-GO gages (three correct answers): (a) gage blocks, (b) limit gage, (c) master gage, (d) plug gage, and (e) snap gage? 5.7. Surface texture includes which of the following characteristics of a surface (three correct answers): (a) deviations from the nominal surface, (b) feed marks of the tool that produced the surface, (c) 5.8. 5.9. 5.10. 5.11. 97 hardness variations, (d) oil films, and (e) surface cracks? Surface texture is included within the scope of surface integrity: (a) true or (b) false? Thermal energy is normally associated with which of the following changes in the altered layer (three best answers): (a) cracks, (b) hardness variations, (c) heat affected zone, (d) plastic deformation, (e) recrystallization, or (f) voids? Which one of the following manufacturing processes will likely result in the best surface finish: (a) arc welding, (b) grinding, (c) machining, (d) sand casting, or (e) sawing? Which one of the following manufacturing processes will likely result in the worst surface finish: (a) cold rolling, (b) grinding, (c) machining, (d) sand casting, or (e) sawing? PROBLEMS 5.1. Designthe nominalsizesof aGO/NO-GO plug gage to inspecta 1.500  0.030 in diameter hole. Thereisa wear allowance applied only to the GO side of the gage. The wear allowance is 2% of the entire tolerance band for the inspected feature. Determine (a) the nominal size of the GO gage including the wear allowance and (b) the nominal size of the NO-GO gage. 5.2. Design the nominal sizes of a GO/NO-GO snap gage to inspect the diameter of a shaft that is 1.500  0.030. A wear allowance of 2% of the entire tolerance band is applied to the GO side. Determine (a) the nominal size of the GO gage including the wear allowance and (b) the nominal size of the NO-GO gage. 5.3. Design the nominal sizes of a GO/NO-GO plug gage to inspect a 30.00  0.18 mm diameter hole. There is a wear allowance applied only to the GO side of the gage. The wear allowance is 3% of the entire tolerance band for the inspected feature. Determine (a) the nominal size of the GO gage including the wear allowance and (b) the nominal size of the NO-GO gage. 5.4. Design the nominal sizes of a GO/NO-GO snap gage to inspect the diameter of a shaft that is 30.00  0.18 mm. A wear allowance of 3% of the entire tolerance band is applied to the GO side. Determine (a) the nominal size of the GO gage including the wear allowance and (b) the nominal size of the NO-GO gage. 5.5. A sine bar is used to determine the angle of a part feature. The length of the sine bar is 6.000 in. The rolls have a diameter of 1.000 in. All inspection is performed on a surface plate. In order for the sine bar to match the angle of the part, the following gage blocks must be stacked: 2.0000, 0.5000, 0.3550. Determine the angle of the part feature. 5.6. A 200.00 mm sine bar is used to inspect an angle on a part. The angle has a dimension of 35.0  1.8. The sine bar rolls have a diameter of 30.0 mm. A set of gage blocks is available that can form any height from 10.0000 to 199.9975 mm in increments of 0.0025 mm. Determine (a) the height of the gage block stack to inspect the minimum angle, (b) height of the gage block stack to inspect the maximum angle, and (c) smallest increment of angle that can be setup at the nominal angle size. All inspection is performed on a surface plate. E1C06 11/11/2009 14:12:25 Page 98 Part II Engineering Materials 6 METALS Chapter Contents 6.1 Alloys and Phase Diagrams 6.1.1 Alloys 6.1.2 Phase Diagrams 6.2 Ferrous Metals 6.2.1 The Iron–Carbon Phase Diagram 6.2.2 Iron and Steel Production 6.2.3 Steels 6.2.4 Cast Irons 6.3 Nonferrous Metals 6.3.1 Aluminum and Its Alloys 6.3.2 Magnesium and Its Alloys 6.3.3 Copper and Its Alloys 6.3.4 Nickel and Its Alloys 6.3.5 Titanium and Its Alloys 6.3.6 Zinc and Its Alloys 6.3.7 Lead and Tin 6.3.8 Refractory Metals 6.3.9 Precious Metals 6.4 Superalloys 6.5 Guide to the Processing of Metals 98 Part II discusses the four types of engineering materials: (1) metals, (2) ceramics, (3) polymers, and (4) composites. Metals are the most important engineering materials and the topic of this chapter. A metal is a category of materials generally characterized by properties of ductility, malleability, luster, and high electrical and thermal conductivity. The category includes both metallic elements and their alloys. Metals have properties that satisfy a wide variety of design requirements. The manufacturing processes by which they are shaped into products have been developed and refined over many years; indeed, some of the processes date from ancient times (Historical Note 1.2). In addition, the properties of metals can be enhanced through heat treatment (covered in Chapter 27). The technological and commercial importance of metals results from the following general properties possessed by virtually all of the common metals: å High stiffness and strength. Metals can be alloyed for high rigidity, strength, and hardness; thus, they are used to provide the structural framework for most engineered products. å Toughness. Metals have the capacity to absorb energy better than other classes of materials. å Good electrical conductivity. Metals are conductors because of their metallic bonding that permits the free movement of electrons as charge carriers. å Good thermal conductivity. Metallic bonding also explains why metals generally conduct heat better than ceramics or polymers. E1C06 11/11/2009 14:12:25 Page 99 Section 6.1/Alloys and Phase Diagrams 99 In addition, certain metals have specific properties that make them attractive for specialized applications. Many common metals are available at relatively low cost per unit weight and are often the material of choice simply because of their low cost. Metals are converted into parts and products using a variety of manufacturing processes. The starting form of the metal differs, depending on the process. The major categories are (1) cast metal, in which the initial form is a casting; (2) wrought metal, in which the metal has been worked or can be worked (e.g., rolled or otherwise formed) after casting; better mechanical properties are generally associated with wrought metals compared with cast metals; and (3) powdered metal, in which the metal is purchased in the form of very small powders for conversion into parts using powder metallurgy techniques. Most metals are available in all three forms. The discussion in this chapter focuses on categories (1) and (2), which are of greatest commercial and engineering interest. Powder metallurgy techniques are examined in Chapter 16. Metals are classified into two major groups: (1) ferrous—those based on iron; and (2) nonferrous—all other metals. The ferrous group can be further subdivided into steels and cast irons. Most of the discussion in the present chapter is organized around this classification, but first the general topic of alloys and phase diagrams is examined. 6.1 ALLOYS AND PHASE DIAGRAMS Although some metals are important as pure elements (e.g., gold, silver, copper), most engineering applications require the improved properties obtained by alloying. Through alloying, it is possible to enhance strength, hardness, and other properties compared with pure metals. This section defines and classifies alloys; it then discusses phase diagrams, which indicate the phases of an alloy system as a function of composition and temperature. 6.1.1 ALLOYS An alloy is a metal composed of two or more elements, at least one of which is metallic. The two main categories of alloys are (1) solid solutions and (2) intermediate phases. Solid Solutions A solid solution is an alloy in which one element is dissolved in another to form a single-phase structure. The term phase describes any homogeneous mass of material, such as a metal in which the grains all have the same crystal lattice structure. In a solid solution, the solvent or base element is metallic, and the dissolved element can be either metallic or nonmetallic. Solid solutions come in two forms, shown in Figure 6.1. The first is a substitutional solid solution, in which atoms of the solvent element are replaced in its unit cell by the dissolved element. Brass is an example, in which zinc is dissolved in copper. To make the substitution, several rules must be satisfied [3], [6], [7]: (1) the atomic radii of the two elements must be similar, usually within 15%; (2) their lattice types must be the FIGURE 6.1 Two forms of solid solutions: (a) substitutional solid solution, and (b) interstitial solid solution. (a) (b) E1C06 11/11/2009 100 14:12:25 Page 100 Chapter 6/Metals same; (3) if the elements have different valences, the lower valence metal is more likely to be the solvent; and (4) if the elements have high chemical affinity for each other, they are less likely to form a solid solution and more likely to form a compound. The second type of solid solution is an interstitial solid solution, in which atoms of the dissolving element fit into the vacant spaces between base metal atoms in the lattice structure. It follows that the atoms fitting into these interstices must be small compared with those of the solvent metal. The most important example of this second type is carbon dissolved in iron to form steel. In both forms of solid solution, the alloy structure is generally stronger and harder than either of the component elements. Intermediate Phases There are usually limits to the solubility of one element in another. When the amount of the dissolving element in the alloy exceeds the solid solubility limit of the base metal, a second phase forms in the alloy. The term intermediate phase is used to describe it because its chemical composition is intermediate between the two pure elements. Its crystalline structure is also different from those of the pure metals. Depending on composition, and recognizing that many alloys consist of more than two elements, these intermediate phases can be of several types, including (1) metallic compounds consisting of a metal and nonmetal such as Fe3C; and (2) intermetallic compounds—two metals that form a compound, such as Mg2Pb. 6pt?>The composition of the alloy is often such that the intermediate phase is mixed with the primary solid solution to form a two-phase structure, one phase dispersed throughout the second. These two-phase alloys are important because they can be formulated and heat treated for significantly higher strength than solid solutions. 6.1.2 PHASE DIAGRAMS As the term is used in this text, a phase diagram is a graphical means of representing the phases of a metal alloy system as a function of composition and temperature. This discussion of the diagram will be limited to alloy systems consisting of two elements at atmospheric pressures. This type of diagram is called a binary phase diagram. Other forms of phase diagrams are discussed in texts on materials science, such as [6]. The Copper–Nickel Alloy System The best way to introduce the phase diagram is by example. Figure 6.2 presents one of the simplest cases, the Cu–Ni alloy system. Composition is plotted on the horizontal axis and temperature on the vertical axis. Thus, any point in the diagram indicates the overall composition and the phase or phases present at the given temperature. Pure copper melts at 1083 C (1981 F), and pure nickel at 1455 C (2651 F). Alloy compositions between these extremes exhibit gradual melting that commences at the solidus and concludes at the liquidus as temperature is increased. The copper–nickel system is a solid solution alloy throughout its entire range of compositions. Anywhere in the region below the solidus line, the alloy is a solid solution; there are no intermediate solid phases in this system. However, there is a mixture of phases in the region bounded by the solidus and liquidus. Recall from Chapter 4 that the solidus is the temperature at which the solid metal begins to melt as temperature is increased, and the liquidus is the temperature at which melting is completed. It can now be seen from the phase diagram that these temperatures vary with composition. Between the solidus and liquidus, the metal is a solid–liquid mix. Determining Chemical Compositions of Phases Although the overall composition of the alloy is given by its position along the horizontal axis, the compositions of the liquid 11/11/2009 14:12:25 Page 101 Section 6.1/Alloys and Phase Diagrams 101 3000 Liquid solution 1455∞C (2651∞F) Liquid + solid Liquidus 1400 1260∞C (2300∞F) 2800 2600 2400 L C S Solidus 1200 2200 2000 Temperature,∞F 1600 Temperature,∞C E1C06 Solid solution 1000 1083∞C (1981∞F) ~ ~ FIGURE 6.2 Phase diagram for the copper– nickel alloy system. 0 Cu 1800 26% 10 20 36% 30 40 50 ~ ~ 62% 60 % Nickel (Ni) 70 80 90 100 Ni and solid phases are not the same. It is possible to determine these compositions from the phase diagram by drawing a horizontal line at the temperature of interest. The points of intersection between the horizontal line and the solidus and liquidus indicate the compositions of the solid and liquid phases present, respectively. Simply construct the vertical projections from the intersection points to the x-axis and read the corresponding compositions. Example 6.1 Determining Compositions from the Phase Diagram To illustrate the procedure, suppose one wants to analyze the compositions of the liquid and solid phases present in the copper-nickel system at an aggregate composition of 50% nickel and a temperature of 1260 C (2300 F). Solution: A horizontal line is drawn at the given temperature level as shown in Figure 6.2. The line intersects the solidus at a composition of 62% nickel, thus indicating the composition of the solid phase. The intersection with the liquidus occurs n at a composition of 36% Ni, corresponding to the analysis of the liquid phase. As the temperature of the 50–50 Cu–Ni alloy is reduced, the solidus line is reached at about 1221 C (2230 F). Applying the same procedure used in the example, the composition of the solid metal is 50% nickel, and the composition of the last remaining liquid to freeze is about 26% nickel. How is it, the reader might ask, that the last ounce of molten metal has a composition so different from the solid metal into which it freezes? The answer is that the phase diagram assumes equilibrium conditions are allowed to prevail. In fact, the binary phase diagram is sometimes called an equilibrium diagram because of this assumption. What it means is that enough time is permitted for the solid metal to gradually change its composition by diffusion to achieve the composition indicated by the intersection point along the liquidus. In practice, when an alloy freezes (e.g., a casting), segregation occurs in the solid mass because of nonequilibrium conditions. The first liquid to solidify has a composition that is rich in the metal element with the higher melting point. Then as additional metal solidifies, its composition is different from that of the first metal to freeze. As the nucleation sites grow into a solid mass, compositions are distributed within the mass, depending on the temperature and time in the process at which freezing occurred. The overall composition is the average of the distribution. E1C06 11/11/2009 102 14:12:25 Page 102 Chapter 6/Metals Determining Amounts of Each Phase The amounts of each phase present at a given temperature from the phase diagram can also be determined. This is done by the inverse lever rule: (1) using the same horizontal line as before that indicates the overall composition at a given temperature, measure the distances between the aggregate composition and the intersection points with the liquidus and solidus, identifying the distances as CL and CS, respectively (refer back to Figure 6.2); (2) the proportion of liquid phase present is given by L phase proportion ¼ CS ðCS þ CLÞ ð6:1Þ (3) the proportion of solid phase present is given by S phase proportion ¼ Example 6.2 Determining Proportions of Each Phase CL ðCS þ CLÞ ð6:2Þ Determine the proportions of liquid and solid phases for the 50% nickel composition of the copper–nickel system at the temperature of 1260 C (2300 F). Solution: Using the same horizontal line in Figure 6.2 as in previous Example 6.1, the distances CS and CL are measured as 10 mm and 12 mm, respectively. Thus the proportion of the liquid phase is 10=22 ¼ 0.45 (45%), and the proportion of n solid phase is 12=22 ¼ 0.55 (55%). The proportions given by Eqs. (6.1) and (6.2) are by weight, same as the phase diagram percentages. Note that the proportions are based on the distance on the opposite side of the phase of interest; hence the name inverse lever rule. One can see the logic in this by taking the extreme case when, say, CS ¼ 0; at that point, the proportion of the liquid phase is zero because the solidus has been reached and the alloy is therefore completely solidified. The methods for determining chemical compositions of phases and the amounts of each phase are applicable to the solid region of the phase diagram as well as the liquidus–solidus region. Wherever there are regions in the phase diagram in which two phases are present, these methods can be used. When only one phase is present (in Figure 6.2, this is the entire solid region), the composition of the phase is its aggregate composition under equilibrium conditions; and the inverse lever rule does not apply because there is only one phase. The Tin–Lead Alloy System A more complicated phase diagram is the Sn–Pb system, shown in Figure 6.3. Tin–lead alloys have traditionally been used as solders for making electrical and mechanical connections (Section 31.2).1 The phase diagram exhibits several features not included in the previous Cu–Ni system. One feature is the presence of two solid phases, alpha (a) and beta (b). The a phase is a solid solution of tin in lead at the left side of the diagram, and the b phase is a solid solution of lead in tin that occurs only at elevated temperatures around 200 C (375 F) at the right side of the diagram. Between these solid solutions lies a mixture of the two solid phases, a þ b. Another feature of interest in the tin–lead system is how melting differs for different compositions. Pure tin melts at 232 C (449 F), and pure lead melts at 327 C (621 F). Alloys of these elements melt at lower temperatures. The diagram shows two liquidus lines that begin at the melting points of the pure metals and meet at a composition of 61.9% Sn. This is the eutectic composition for the tin–lead system. In general, a eutectic alloy is a particular composition in an alloy system for which the solidus and liquidus are at the same temperature. The corresponding eutectic temperature, the melting point of the eutectic 1 Because lead is a poisonous substance, alternative alloying elements have been substituted for lead in many commercial solders. These are called lead-free solders. 11/11/2009 14:12:25 Page 103 103 Section 6.2/Ferrous Metals Liquid 600 300 +L +L 200 183∞C (362∞F) 100 61.9% Sn (eutectic composition) 400 300 Temperature ∞F 500 Temperature ∞C E1C06 200 + 100 FIGURE 6.3 Phase diagram for the tin–lead alloy system. 0 0 Pb 20 40 60 % Tin (Sn) 80 Sn composition, is 183 C (362 F) in the present case. The eutectic temperature is always the lowest melting point for an alloy system (eutectic is derived from the Greek word eutektos, meaning easily melted). Methods for determining the chemical analysis of the phases and the proportions of phases present can be readily applied to the Sn–Pb system just as it was used in the Cu–Ni system. In fact, these methods are applicable in any region containing two phases, including two solid phases. Most alloy systems are characterized by the existence of multiple solid phases and eutectic compositions, and so the phase diagrams of these systems are often similar to the tin–lead diagram. Of course, many alloy systems are considerably more complex. One of these is the alloy system of iron and carbon. 6.2 FERROUS METALS The ferrous metals are based on iron, one of the oldest metals known to humans (Historical Note 6.1). The properties and other data relating to iron are listed in Table 6.1(a). The ferrous metals of engineering importance are alloys of iron and carbon. These alloys divide into two major groups: steel and cast iron. Together, they constitute approximately 85% of the metal tonnage in the United States [6]. This discussion of the ferrous metals begins with the iron–carbon phase diagram. TABLE 6.1 Basic data on the metallic elements: (a) Iron. Symbol: Atomic number: Specific gravity: Crystal structure: Melting temperature: Elastic modulus: Fe 26 7.87 BCC 1539 C (2802 F) 209,000 MPa (30  106 lb/in2) Compiled from [6], [11], [12], and other references. Principal ore: Hematite (Fe2O3) Alloying elements: Carbon; also chromium, manganese, nickel, molybdenum, vanadium, and silicon Typical applications: Construction, machinery, automotive, railway tracks and equipment E1C06 11/11/2009 104 14:12:25 Page 104 Chapter 6/Metals Historical Note 6.1 Iron and steel I ron was discovered sometime during the Bronze Age. It was probably uncovered from ashes of fires built near iron ore deposits. Use of the metal grew, finally surpassing bronze in importance. The Iron Age is usually dated from about 1200 BCE, although artifacts made of iron have been found in the Great Pyramid of Giza in Egypt, which dates to 2900 BCE. Iron-smelting furnaces have been discovered in Israel dating to 1300 BCE. Iron chariots, swords, and tools were made in ancient Assyria (northern Iraq) around 1000 BCE. The Romans inherited ironworking from their provinces, mainly Greece, and they developed the technology to new heights, spreading it throughout Europe. The ancient civilizations learned that iron was harder than bronze and that it took a sharper, stronger edge. During the Middle Ages in Europe, the invention of the cannon created the first real demand for iron; only then did it finally exceed copper and bronze in usage. Also, the cast iron stove, the appliance of the seventeenth and eighteenth centuries, significantly increased demand for iron (Historical Note 11.3). In the nineteenth century, industries such as railroads, shipbuilding, construction, machinery, and the military created a dramatic growth in the demand for iron and steel in Europe and America. Although large quantities of (crude) pig iron could be produced by blast furnaces, the subsequent processes for producing wrought iron and steel were slow. The necessity to improve productivity of these vital metals was the ‘‘mother of invention.’’ Henry Bessemer in England developed the process of blowing air up through the molten iron that led to the Bessemer converter (patented in 1856). Pierre and Emile Martin in France built the first open hearth furnace in 1864. These methods permitted up to 15 tons of steel to be produced in a single batch (heat), a substantial increase from previous methods. In the United States, expansion of the railroads after the Civil War created a huge demand for steel. In the 1880s and 1890s, steel beams were first used in significant quantities in construction. Skyscrapers came to rely on these steel frames. When electricity became available in abundance in the late 1800s, this energy source was used for steelmaking. The first commercial electric furnace for production of steel was operated in France in 1899. By 1920, this had become the principal process for making alloy steels. The use of pure oxygen in steelmaking was initiated just before World War II in several European countries and the United States. Work in Austria after the war culminated in the development of the basic oxygen furnace (BOF). This has become the leading modern technology for producing steel, surpassing the open hearth method around 1970. The Bessemer converter had been surpassed by the open hearth method around 1920 and ceased to be a commercial steelmaking process in 1971. 6.2.1 THE IRON–CARBON PHASE DIAGRAM The iron–carbon phase diagram is shown in Figure 6.4. Pure iron melts at 1539 C (2802 F). During the rise in temperature from ambient, it undergoes several solid phase transformations, as indicated in the diagram. Starting at room temperature the phase is alpha (a), also called ferrite. At 912 C (1674 F), ferrite transforms to gamma (g), called austenite. This, in turn, transforms at 1394 C (2541 F) to delta (d), which remains until melting occurs. The three phases are distinct; alpha and delta have BCC lattice structures (Section 2.3.1), and between them, gamma is FCC. The video clip on heat treatment describes the iron–carbon phase diagram and how it is used to strengthen steel. VIDEO CLIP Heat Treatment: View the segment on the iron–carbon phase diagram. Iron as a commercial product is available at various levels of purity. Electrolytic iron is the most pure, at about 99.99%, for research and other purposes where the pure metal is required. Ingot iron, containing about 0.1% impurities (including about 0.01% carbon), is 11/11/2009 14:12:26 Page 105 105 Section 6.2/Ferrous Metals 1800 3200 Liquid (L) 2800 1400 +L 1000 L + Fe3C 2000 1130∞C (2066∞F) + 1600 + Fe3C Solid A1 723∞C (1333∞F) 600 1200 800 + Fe3C Solid 400 200 FIGURE 6.4 Phase diagram for iron–carbon system, up to about 6% carbon. 0 Fe Temperature, ∞F 2400 Temperature, ∞C E1C06 1 2 3 4 5 6 C % Carbon (C) used in applications in which high ductility or corrosion resistance are needed. Wrought iron contains about 3% slag but very little carbon, and is easily shaped in hot forming operations such as forging. Solubility limits of carbon in iron are low in the ferrite phase—only about 0.022% at 723 C (1333 F). Austenite can dissolve up to about 2.1% carbon at a temperature of 1130 C (2066 F). This difference in solubility between alpha and gamma leads to opportunities for strengthening by heat treatment (but leave that for Chapter 27). Even without heat treatment, the strength of iron increases dramatically as carbon content increases, and the metal is called steel. More precisely, steel is defined as an iron–carbon alloy containing from 0.02% to 2.11% carbon.2 Of course, steels can also contain other alloying elements as well. A eutectic composition at 4.3% carbon can be seen in the diagram. There is a similar feature in the solid region of the diagram at 0.77% carbon and 723 C (1333 F). This is called the eutectoid composition. Steels below this carbon level are known as hypoeutectoid steels, and above this carbon level, from 0.77% to 2.1%, they are called hypereutectoid steels. In addition to the phases mentioned, one other phase is prominent in the iron–carbon alloy system. This is Fe3C, also known as cementite, an intermediate phase. It is a metallic compound of iron and carbon that is hard and brittle. At room temperature under equilibrium conditions, iron–carbon alloys form a two-phase system at carbon levels even slightly above zero. The carbon content in steel ranges between these very low levels and about 2.1% C. Above 2.1% C, up to about 4% or 5%, the alloy is defined as cast iron. 6.2.2 IRON AND STEEL PRODUCTION Coverage of iron and steel production begins with the iron ores and other raw materials required. Ironmaking is then discussed, in which iron is reduced from the ores, and 2 This is the conventional definition of steel, but exceptions exist. A recently developed steel for sheetmetal forming, called interstitial-free steel, has a carbon content of only 0.005%. It is discussed in Section 6.2.3. E1C06 11/11/2009 106 14:12:26 Page 106 Chapter 6/Metals steelmaking, in which the iron is refined to obtain the desired purity and composition (alloying). The casting processes that are accomplished at the steel mill are then considered. Iron Ores and Other Raw Materials The principal ore used in the production of iron and steel is hematite (Fe2O3). Other iron ores include magnetite (Fe3O4), siderite (FeCO3), and limonite (Fe2O3-xH2O, in which x is typically around 1.5). Iron ores contain from 50% to around 70% iron, depending on grade (hematite is almost 70% iron). In addition, scrap iron and steel are widely used today as raw materials in iron- and steelmaking. Otherrawmaterialsneededtoreduceironfromtheoresarecokeandlimestone.Cokeisa high carbon fuel produced by heating bituminous coal in a limited oxygen atmosphere for several hours, followed by water spraying in special quenching towers. Coke serves two functions in the reduction process: (1) it is a fuel that supplies heat for the chemical reactions; and (2) it produces carbon monoxide (CO) to reduce the iron ore. Limestone is a rock containing high proportions of calcium carbonate (CaCO3). The limestone is used in the process as a flux to react with and remove impurities in the molten iron as slag. Ironmaking To produce iron, a charge of ore, coke, and limestone are dropped into the top of a blast furnace. A blast furnace is a refractory-lined chamber with a diameter of about 9 to 11 m (30–35 ft) at its widest and a height of 40 m (125 ft), in which hot gases are forced into the lower part of the chamber at high rates to accomplish combustion and reduction of the iron. A typical blast furnace and some of its technical details are illustrated in Figures 6.5 and 6.6. The charge slowly descends from the top of the furnace toward the FIGURE 6.5 Cross section of ironmaking blast furnace showing major components. E1C06 11/11/2009 14:12:26 Page 107 Section 6.2/Ferrous Metals Iron ore, coke, and limestone 107 Gas to cleaning and reheating Typical temperature profile 200∞C (400∞F) Direction of motion of charge material 800∞C (1500∞F) 1100∞C (2000∞F) FIGURE 6.6 Schematic diagram indicating details of the blast furnace operation. Direction of motion of hot gases 1400∞C (2500∞F) Hot blast air 1650∞C (3000∞F) Slag Molten pig iron base and is heated to temperatures around 1650 C (3000 F). Burning of the coke is accomplished by the hot gases (CO, H2, CO2, H2O, N2, O2, and fuels) as they pass upward through the layers of charge material. The carbon monoxide is supplied as hot gas, and it is also formed from combustion of coke. The CO gas has a reducing effect on the iron ore; the reaction (simplified) can be written as follows (using hematite as the starting ore) Fe2 O3 þ CO ! 2FeO þ CO2 ð6:3aÞ Carbon dioxide reacts with coke to form more carbon monoxide CO2 þ C(coke) ! 2CO ð6:3bÞ which then accomplishes the final reduction of FeO to iron FeO þ CO ! Fe þ CO2 ð6:3cÞ The molten iron drips downward, collecting at the base of the blast furnace. This is periodically tapped into hot iron ladle cars for transfer to subsequent steelmaking operations. The role played by limestone can be summarized as follows. First the limestone is reduced to lime (CaO) by heating, as follows CaCO3 ! CaO þ CO2 ð6:4Þ The lime combines with impurities such as silica (SiO2), sulfur (S), and alumina (Al2O3) in reactions that produce a molten slag that floats on top of the iron. It is instructive to note that approximately 7 tons of raw materials are required to produce 1 ton of iron. The ingredients are proportioned about as follows: 2.0 tons of iron ore, 1.0 ton of coke, 0.5 ton of limestone, and (here’s the amazing statistic) 3.5 tons of gases. A significant proportion of the byproducts are recycled. The iron tapped from the base of the blast furnace (called pig iron) contains more than 4% C, plus other impurities: 0.3–1.3% Si, 0.5–2.0% Mn, 0.1–1.0% P, and 0.02–0.08% S [11]. Further refinement of the metal is required for both cast iron and steel. A furnace called a cupola (Section 11.4.1) is commonly used for converting pig iron into gray cast iron. For steel, compositions must be more closely controlled and impurities brought to much lower levels. E1C06 11/11/2009 108 14:12:26 Page 108 Chapter 6/Metals FIGURE 6.7 Basic oxygen furnace showing BOF vessel during processing of a heat. Steelmaking Since the mid-1800s, a number of processes have been developed for refining pig iron into steel. Today, the two most important processes are the basic oxygen furnace (BOF) and the electric furnace. Both are used to produce carbon and alloy steels. The basic oxygen furnace accounts for about 70% of U.S. steel production. The BOF is an adaptation of the Bessemer converter. Whereas the Bessemer process used air blown up through the molten pig iron to burn off impurities, the basic oxygen process uses pure oxygen. A diagram of the conventional BOF during the middle of a heat is illustrated in Figure 6.7. The typical BOF vessel is about 5 m (16 ft) inside diameter and can process 150 to 200 tons in a heat. The BOF steelmaking sequence is shown in Figure 6.8. Integrated steel mills transfer the molten pig iron from the blast furnace to the BOF in railway cars called hot-iron ladle cars. In modern practice, steel scrap is added to the pig iron, accounting for about 30% of a typical BOF charge. Lime (CaO) is also added. After charging, the lance is inserted into the vessel so that its tip is about 1.5 m (5 ft) above the surface of the molten iron. Pure O2 is blown at high velocity through the lance, causing combustion and heating at the surface of the molten pool. Carbon dissolved in the iron and other impurities such as silicon, manganese, and phosphorus are oxidized. The reactions are 2C þ O2 ! 2CO (CO2 is also produced) ð6:5aÞ Si þ O2 ! SiO2 ð6:5bÞ 2Mn þ O2 ! 2MnO ð6:5cÞ 4P þ 5O2 ! 2P2 O5 ð6:5dÞ The CO and CO2 gases produced in the first reaction escape through the mouth of the BOF vessel and are collected by the fume hood; the products of the other three reactions are removed as slag, using the lime as a fluxing agent. The C content in the iron decreases almost linearly with time during the process, thus permitting fairly predictable control over carbon levels in the steel. After refining to the desired level, the molten steel is tapped; alloying ingredients and other additives are poured into the heat; then the slag is E1C06 11/11/2009 14:12:26 Page 109 Section 6.2/Ferrous Metals 109 FIGURE 6.8 BOF sequence during processing cycle: (1) charging of scrap and (2) pig iron; (3) blowing (Figure 6.7); (4) tapping the molten steel; and (5) pouring off the slag. poured. A 200-ton heat of steel can be processed in about 20 min, although the entire cycle time (tap-to-tap time) takes about 45 min. Recent advances in the technology of the basic oxygen process include the use of nozzles in the bottom of the vessel through which oxygen is injected into the molten iron. This allows better mixing than the conventional BOF lance, resulting in shorter processing times (a reduction of about 3 min), lower carbon contents, and higher yields. The electric arc furnace accounts for about 30% of U.S. steel production. Although pig iron was originally used as the charge in this type of furnace, scrap iron and scrap steel are the primary raw materials today. Electric arc furnaces are available in several designs; the direct arc type shown in Figure 6.9 is currently the most economical type. These furnaces have removable roofs for charging from above; tapping is accomplished by tilting the entire furnace. Scrap iron and steel selected for their compositions, together with alloying ingredients and limestone (flux), are charged into the furnace and heated by an electric arc that flows between large electrodes and the charge metal. Complete melting requires about 2 hours; tap-to-tap time is 4 hours. Capacities of electric furnaces commonly range between 25 and 100 tons per heat. Electric arc furnaces are noted for better-quality steel but higher cost per ton, compared with the BOF. The electric arc furnace is generally associated with production of alloy steels, tool steels, and stainless steels. Casting of Ingots Steels produced by BOF or electric furnace are solidified for subsequent processing either as cast ingots or by continuous casting. Steel ingots are large discrete castings weighing from less than 1 ton up to around 300 tons (the weight of an entire heat). Ingot molds are made of high carbon iron and are tapered at the top or bottom for removal of the solid casting. A big-end-down mold is illustrated in Figure 6.10. The cross E1C06 11/11/2009 110 14:12:26 Page 110 Chapter 6/Metals FIGURE 6.9 Electric arc furnace for steelmaking. section may be square, rectangular, or round, and the perimeter is usually corrugated to increase surface area for faster cooling. The mold is placed on a platform called a stool; after solidification the mold is lifted, leaving the casting on the stool. The solidification process for ingots as well as other castings is described in the chapter on casting principles (Chapter 10). Because ingots are such large castings, the time required for solidification and the associated shrinkage are significant. Porosity caused by the reaction of carbon and oxygen to form CO during cooling and solidification is a problem that must be addressed in ingot casting. These gases are liberated from the molten steel because of their reduced solubility with decreasing temperature. Cast steels are often treated to limit or prevent CO gas evolution during solidification. The treatment involves adding elements such as Si and Al that react with the oxygen dissolved in the molten steel, so it is not available for CO reaction. The structure of the solid steel is thus free of pores and other defects caused by gas formation. Continuous Casting Continuous casting is widely applied in aluminum and copper production, but its most noteworthy application is in steelmaking. The process is replacing ingot casting because it dramatically increases productivity. Ingot casting is a discrete process. Because the molds are relatively large, solidification time is significant. For a large FIGURE 6.10 A big-end-down ingot mold typical of type used in steelmaking. E1C06 11/11/2009 14:12:27 Page 111 Section 6.2/Ferrous Metals 111 FIGURE 6.11 Continuous casting; steel is poured into tundish and distributed to a water-cooled continuous casting mold; it solidifies as it travels down through the mold. The slab thickness is exaggerated for clarity. steel ingot, it may take 10 to 12 hours for the casting to solidify. The use of continuous casting reduces solidification time by an order of magnitude. The continuous casting process, also called strand casting, is illustrated in Figure 6.11. Molten steel is poured from a ladle into a temporary container called a tundish, which dispenses the metal to one or more continuous casting molds. The steel begins to solidify at the outer regions as it travels down through the water-cooled mold. Water sprays accelerate the cooling process. While still hot and plastic, the metal is bent from vertical to horizontal orientation. It is then cut into sections or fed continuously into a rolling mill (Section 19.1) in which it is formed into plate or sheet stock or other cross sections. 6.2.3 STEELS As defined earlier, Steel is an alloy of iron that contains carbon ranging by weight between 0.02% and 2.11% (most steels range between 0.05% and 1.1%C). It often includes other alloying ingredients, such as manganese, chromium, nickel, and/or molybdenum (see Table 6.2); but it is the carbon content that turns iron into steel. Hundreds of compositions of steel are available commercially. For purposes of organization here, the vast majority of commercially important steels can be grouped into the following categories: (1) plain carbon steels, (2) low alloy steels, (3) stainless steels, (4) tool steels, and (5) specialty steels. Plain Carbon Steels These steels contain carbon as the principal alloying element, with only small amounts of other elements (about 0.4% manganese plus lesser amounts of 11/11/2009 112 14:12:28 Page 112 Chapter 6/Metals TABLE 6.2 AISI-SAE designations of steels. Nominal Chemical Analysis, % Code 10XX 11XX 12XX 13XX 20XX 31XX 40XX 41XX 43XX 46XX 47XX 48XX 50XX 52XX 61XX 81XX 86XX 88XX 92XX 93XX 98XX Name of Steel Cr Plain carbon Resulfurized Resulfurized, rephosphorized Manganese Nickel steels Nickel–chrome Molybdenum Chrome–molybdenum Ni–Cr–Mo Nickel–molybdenum Ni–Cr–Mo Nickel–molybdenum Chromium Chromium Cr–Vanadium Ni–Cr–Mo Ni–Cr–Mo Ni–Cr–Mo Silicon–Manganese Ni–Cr–Mo Ni–Cr–Mo Mn Mo Ni V 0.4 0.9 0.9 1.7 0.5 0.6 1.2 0.6 0.8 0.8 0.7 0.6 0.6 0.6 0.4 0.4 0.8 0.8 0.8 0.8 0.8 0.6 0.8 1.0 0.8 0.4 0.5 1.4 0.8 0.4 0.5 0.5 1.2 0.8 0.25 0.2 0.25 0.25 0.2 0.25 1.8 1.8 1.0 3.5 0.1 0.2 0.35 0.3 0.5 0.5 0.1 0.25 3.2 1.0 0.1 P S Si 0.04 0.01 0.10 0.05 0.12 0.22 0.01 0.01 0.04 0.04 0.04 0.04 0.04 0.04 0.04 0.04 0.04 0.04 0.02 0.04 0.04 0.04 0.04 0.04 0.02 0.04 0.04 0.04 0.04 0.04 0.04 0.04 0.04 0.04 0.04 0.04 0.02 0.04 0.04 0.04 0.04 0.04 0.02 0.04 0.3 0.2 0.3 0.2 0.3 0.2 0.3 0.3 0.3 0.3 0.3 0.3 0.3 0.3 0.3 2.0 0.3 0.3 silicon, phosphorus, and sulfur). The strength of plain carbon steels increases with carbon content. A typical plot of the relationship is illustrated in Figure 6.12. As seen in the phase diagram for iron and carbon (Figure 6.4), steel at room temperature is a mixture of ferrite (a) and cementite (Fe3C). The cementite particles distributed throughout the ferrite act as 160 120 FIGURE 6.12 Tensile strength and hardness as a function of carbon content in plain carbon steel (hot-rolled, unheattreated). Tensile strength, MPa 220 200 120 800 100 Tensile strength 600 80 Hardness 60 400 40 200 20 80 ~ ~ 0 0.2 0.4 0.6 0.8 % Carbon (C) 1.0 Tensile strength, 1000 lb/in2. 240 Hardness, HB E1C06 E1C06 11/11/2009 14:12:28 Page 113 Section 6.2/Ferrous Metals 113 obstacles to the movement of dislocations during slip (Section 2.3.3); more carbon leads to more barriers, and more barriers mean stronger and harder steel. According to a designation scheme developed by the American Iron and Steel Institute (AISI) and the Society of Automotive Engineers (SAE), plain carbon steels are specified by a four-digit number system: 10XX, in which 10 indicates that the steel is plain carbon, and XX indicates the percent of carbon in hundredths of percentage points. For example, 1020 steel contains 0.20% C. The plain carbon steels are typically classified into three groups according to their carbon content: 1. Low carbon steels contain less than 0.20% C and are by far the most widely used steels. Typical applications are automobile sheet-metal parts, plate steel for fabrication, and railroad rails. These steels are relatively easy to form, which accounts for their popularity where high strength is not required. Steel castings usually fall into this carbon range, also. 2. Medium carbon steels range in carbon between 0.20% and 0.50% and are specified for applications requiring higher strength than the low-C steels. Applications include machinery components and engine parts such as crankshafts and connecting rods. 3. High carbon steels contain carbon in amounts greater than 0.50%. They are specified for still higher strength applications and where stiffness and hardness are needed. Springs, cutting tools and blades, and wear-resistant parts are examples. Increasing carbon content strengthens and hardens the steel, but its ductility is reduced. Also, high carbon steels can be heat treated to form martensite, making the steel very hard and strong (Section 27.2). Low Alloy Steels Low alloy steels are iron–carbon alloys that contain additional alloying elements in amounts totaling less than about 5% by weight. Owing to these additions, low alloy steels have mechanical properties that are superior to those of the plain carbon steels for given applications. Superior properties usually mean higher strength, hardness, hot hardness, wear resistance, toughness, and more desirable combinations of these properties. Heat treatment is often required to achieve these improved properties. Common alloying elements added to steel are chromium, manganese, molybdenum, nickel, and vanadium, sometimes individually but usually in combinations. These elements typically form solid solutions with iron and metallic compounds with carbon (carbides), assuming sufficient carbon is present to support a reaction. The effects of the principal alloying ingredients can be summarized as follows: å Chromium (Cr) improves strength, hardness, wear resistance, and hot hardness. It is one of the most effective alloying ingredients for increasing hardenability (Section 27.2.3). In significant proportions, Cr improves corrosion resistance. å Manganese (Mn) improves the strength and hardness of steel. When the steel is heat treated, hardenability is improved with increased manganese. Because of these benefits, manganese is a widely used alloying ingredient in steel. å Molybdenum (Mo) increases toughness and hot hardness. It also improves hardenability and forms carbides for wear resistance. å Nickel (Ni) improves strength and toughness. It increases hardenability but not as much as some of the other alloying elements in steel. In significant amounts it improves corrosion resistance and is the other major ingredient (besides chromium) in certain types of stainless steel. E1C06 11/11/2009 114 14:12:29 Page 114 Chapter 6/Metals TABLE 6.3 Treatments and mechanical properties of selected steels. Tensile Strength Code 1010 1010 1020 1020 1040 1040 1055 1315 2030 3130 4130 4140 4340 4815 9260 HSLA Treatmenta HR CD HR CD HR CD HT None None HT HT HT HT HT HT None MPa lb/in2 Elongation, % 304 366 380 421 517 587 897 545 566 697 890 918 1279 635 994 586 44,000 53,000 55,000 61,000 75,000 85,000 130,000 79,000 82,000 101,000 129,000 133,000 185,000 92,000 144,000 85,000 47 12 28 15 20 10 16 34 32 28 17 16 12 27 18 20 Compiled from [6], [11], and other sources. HR ¼ hot-rolled; CD ¼ cold-drawn; HT ¼ heat treatment involving heating and quenching, followed by tempering to produce tempered martensite (Section 27.2). a å Vanadium (V) inhibits grain growth during elevated temperature processing and heat treatment, which enhances strength and toughness of steel. It also forms carbides that increase wear resistance. The AISI-SAE designations of many of the low alloy steels are presented in Table 6.2, which indicates nominal chemical analysis. As before, carbon content is specified by XX in 1=100% of carbon. For completeness, plain carbon steels (10XX) have been included. To obtain an idea of the properties possessed by some of these steels, Table 6.3 was compiled, which lists the treatment to which the steel is subjected for strengthening and its strength and ductility. Low alloy steels are not easily welded, especially at medium and high carbon levels. Since the 1960s, research has been directed at developing low carbon, low alloy steels that have better strength-to-weight ratios than plain carbon steels but are more weldable than low alloy steels. The products developed out of these efforts are called high-strength low-alloy (HSLA) steels. They generally have low carbon contents (in the range 0.10%–0.30% C) plus relatively small amounts of alloying ingredients (usually only about 3% total of elements such as Mn, Cu, Ni, and Cr). HSLA steels are hot-rolled under controlled conditions designed to provide improved strength compared with plain C steels, yet with no sacrifice in formability or weldability. Strengthening is by solid solution alloying; heat treatment is not feasible because of low carbon content. Table 6.3 lists one HSLA steel, together with properties (chemistry is: 0.12 C, 0.60 Mn, 1.1 Ni, 1.1 Cr, 0.35 Mo, and 0.4 Si). Stainless Steels Stainless steels are a group of highly alloyed steels designed to provide high corrosion resistance. The principal alloying element in stainless steel is chromium, usually above 15%. The chromium in the alloy forms a thin, impervious oxide film in an E1C06 11/11/2009 14:12:29 Page 115 Section 6.2/Ferrous Metals 115 oxidizing atmosphere, which protects the surface from corrosion. Nickel is another alloying ingredient used in certain stainless steels to increase corrosion protection. Carbon is used to strengthen and harden the metal; however, increasing the carbon content has the effect of reducing corrosion protection because chromium carbide forms to reduce the amount of free Cr available in the alloy. In addition to corrosion resistance, stainless steels are noted for their combination of strength and ductility. Although these properties are desirable in many applications, they generally make these alloys difficult to work in manufacturing. Also, stainless steels are significantly more expensive than plain C or low alloy steels. Stainless steels are traditionally divided into three groups, named for the predominant phase present in the alloy at ambient temperature. 1. Austenitic stainless have a typical composition of around 18% Cr and 8% Ni and are the most corrosion resistant of the three groups. Owing to this composition, they are sometimesidentifiedas18-8 stainless.Theyarenonmagneticand very ductile; butthey show significant work hardening. The nickel has the effect of enlarging the austenite region in the iron–carbon phase diagram, making it stable at room temperature. Austenitic stainless steels are used to fabricate chemical and food processing equipment, as well as machinery parts requiring high corrosion resistance. 2. Ferritic stainless have around 15% to 20% chromium, low carbon, and no nickel. This provides a ferrite phase at room temperature. Ferritic stainless steels are magnetic and are less ductile and corrosion resistant than the austenitics. Parts made of ferritic stainless range from kitchen utensils to jet engine components. 3. Martensitic stainless have a higher carbon content than ferritic stainlesses, thus permitting them to be strengthened by heat treatment (Section 27.2). They have as much as 18% Cr but no Ni. They are strong, hard, and fatigue resistant, but not generally as corrosion resistant as the other two groups. Typical products include cutlery and surgical instruments. Most stainless steels are designated by a three-digit AISI numbering scheme. The first digit indicates the general type, and the last two digits give the specific grade within the type. Table 6.4 lists the common stainless steels with typical compositions and mechanical properties. The traditional stainless steels were developed in the early 1900s. Since then, several additional high alloy steels have been developed that have good corrosion resistance and other desirable properties. These are also classified as stainless steels. Continuing the list: 4. Precipitation hardening stainless, which have a typical composition of 17% Cr and 7%Ni, with additional small amounts of alloying elements such as aluminum, copper, titanium, and molybdenum. Their distinguishing feature among stainlesses is that they can be strengthened by precipitation hardening (Section 27.3). Strength and corrosion resistance are maintained at elevated temperatures, which suits these alloys to aerospace applications. 5. Duplex stainless possess a structure that is a mixture of austenite and ferrite in roughly equal amounts. Their corrosion resistance is similar to the austenitic grades, and they show improved resistance to stress-corrosion cracking. Applications include heat exchangers, pumps, and wastewater treatment plants. Tool Steels Tool steels are a class of (usually) highly alloyed steels designed for use as industrial cutting tools, dies, and molds. To perform in these applications, they must possess high strength, hardness, hot hardness, wear resistance, and toughness under impact. To obtain these properties, tool steels are heat treated. Principal reasons for the high levels of alloying elements are (1) improved hardenability, (2) reduced distortion during heat E1C06 11/11/2009 116 14:12:30 Page 116 Chapter 6/Metals TABLE 6.4 Compositions and mechanical properties of selected stainless steels. Chemical Analysis, % Tensile Strength Othera MPa lb/in2 Elongation, % 2.5 Mo 620 515 515 515 515 90,000 75,000 75,000 75,000 75,000 40 40 40 40 40 1 1 415 415 60,000 60,000 20 20 1 1 1 1 1 1 485 825 485 965 725 1790 70,000 120,000 70,000 140,000 105,000 260,000 20 12 20 10 20 5 Type Fe Cr Ni C Mn Austenitic 301 302 304 309 316 73 71 69 61 65 17 18 19 23 17 7 8 9 13 12 0.15 0.15 0.08 0.20 0.08 2 2 2 2 2 Ferritic 405 430 85 81 13 17 — — 0.08 0.12 Martensitic 403 403b 416 416b 440 440b 86 86 85 85 81 81 12 12 13 13 17 17 — — — — — — 0.15 0.15 0.15 0.15 0.65 0.65 Compiled from [11]. All of the grades in the table contain about 1% (or less) Si plus small amounts (well below 1%) of phosphorus, sulfur, and other elements such as aluminum. b Heat treated. a treatment, (3) hot hardness, (4) formation of hard metallic carbides for abrasion resistance, and (5) enhanced toughness. The tool steels divide into major types, according to application and composition. The AISI uses a classification scheme that includes a prefix letter to identify the tool steel. In the following list of tool steel types, the prefix and some typical compositions are presented in Table 6.5: TABLE 6.5 Tool steels by AISI prefix identification, with examples of composition and typical hardness values. Chemical Analysis, %a AISI T M H D A O W S P L a Example T1 M2 H11 D1 A2 O1 W1 S1 P20 L6 C Cr 0.7 0.8 0.4 1.0 1.0 0.9 1.0 0.5 0.4 0.7 4.0 4.0 5.0 12.0 5.0 0.5 Percent composition rounded to nearest tenth. Hardness estimated. b 1.5 1.7 0.8 Mn Mo Ni 5.0 1.5 1.0 1.0 1.0 V W 1.0 2.0 0.4 18.0 6.0 0.5 2.5 0.4 0.2 1.5 Hardness, HRC 65 65 55 60 60 61 63 50 40b 45b E1C06 11/11/2009 14:12:30 Page 117 Section 6.2/Ferrous Metals T, M H D W S P L 117 High-speed tool steels are used as cutting tools in machining processes (Section 23.2.1). They are formulated for high wear resistance and hot hardness. The original high-speed steels (HSS) were developed around 1900. They permitted dramatic increases in cutting speed compared to previously used tools; hence their name. The two AISI designations indicate the principal alloying element: T for tungsten and M for molybdenum. Hot-working tool steels are intended for hot-working dies in forging, extrusion, and die-casting. Cold-work tool steels are die steels used for cold working operations such as sheetmetal pressworking, cold extrusion, and certain forging operations. The designation D stands for die. Closely related AISI designations are A and O. A and O stand for air- and oil-hardening. They all provide good wear resistance and low distortion. Water-hardening tool steels have high carbon with little or no other alloying elements. They can only be hardened by fast quenching in water. They are widely used because of low cost, but they are limited to low temperature applications. Cold heading dies are a typical application. Shock-resistant tool steels are intended for use in applications where high toughness is required, as in many sheetmetal shearing, punching, and bending operations. Mold steels are used to make molds for molding plastics and rubber. Low-alloy tool steels are generally reserved for special applications. Tool steels are not the only tool materials. Plain carbon, low alloy, and stainless steels are used for many tool and die applications. Cast irons and certain nonferrous alloys are also suitable for certain tooling applications. In addition, several ceramic materials (e.g., Al2O3) are used as high-speed cutting inserts, abrasives, and other tools. Specialty Steels To complete this survey, several specialty steels are mentioned that are not included in the previous coverage. One of the reasons why these steels are special is that they possess unique processing characteristics. Maraging steels are low carbon alloys containing high amounts of nickel (15% to 25%) and lesser proportions of cobalt, molybdenum, and titanium. Chromium is also sometimes added for corrosion resistance. Maraging steels are strengthened by precipitation hardening (Section 27.3), but in the unhardened condition, they are quite processable by forming and/or machining. They can also be readily welded. Heat treatment results in very high strength together with good toughness. Tensile strengths of 2000 MPa (290,000 lb/ in2) and 10% elongation are not unusual. Applications include parts for missiles, machinery, dies, and other situations where these properties are required and justify the high cost of the alloy. Free-machining steels are carbon steels formulated to improve machinability (Section 24.1). Alloying elements include sulfur, lead, tin, bismuth, selenium, tellurium, and/or phosphorus. Lead is less-frequently used today because of environmental and health concerns. Added in small amounts, these elements act to lubricate the cutting operation, reduce friction, and break up chips for easier disposal. Although more expensive than non-free-machining steels, they often pay for themselves in higher production rates and longer tool lives. Because of their good ductility, low-carbon sheet steels are widely used in sheet-metal forming operations. Further improvements in formability have been achieved using a new class of sheet steel product called interstitial-free steels. These steels have extremely low carbon levels (0.005% C), which result from the use of alloying elements such as niobium and titanium that combine with C and leave the steel virtually free of interstitial atoms. The result E1C06 11/11/2009 118 14:12:30 Page 118 Chapter 6/Metals FIGURE 6.13 Carbon and silicon compositions for cast irons, with comparison to steels (most steels have relatively low silicon contents—cast steels have the higher Si content). Ductile iron is formed by special melting and pouring treatment of gray cast iron, and malleable iron is formed by heat treatment of white cast iron. is excellent ductility, even greater than low-C steels. Applications include deep-drawing operations in the automotive industry. 6.2.4 CAST IRONS Cast iron is an iron alloy containing from 2.1% to about 4% carbon and from 1% to 3% silicon. Its composition makes it highly suitable as a casting metal. In fact, the tonnage of cast iron castings is several times that of all other cast metal parts combined (excluding cast ingots made during steelmaking, which are subsequently rolled into bars, plates, and similar stock). The overall tonnage of cast iron is second only to steel among metals. There are several types of cast iron, the most important being gray cast iron. Other types include ductile iron, white cast iron, malleable iron, and various alloy cast irons. Typical chemical compositions of gray and white cast irons are shown in Figure 6.13, indicating their relationship with cast steel. Ductile and malleable irons possess chemistries similar to the gray and white cast irons, respectively, but result from special treatments to be described in the following. Table 6.6 presents a listing of chemistries for the principal types together with mechanical properties. Gray Cast Iron Graycast iron accounts for the largest tonnageamongthe castirons.It has a compositionintherange2.5% to 4%carbonand 1% to3%silicon.Thischemistryresults in the formation of graphite (carbon) flakes distributed throughout the cast product upon solidification. The structure causes the surface of the metal to have a gray color when fractured; hence the name gray cast iron. The dispersion of graphite flakes accounts for two attractive properties: (1) good vibration damping, which is desirable in engines and other machinery; and (2) internal lubricating qualities, which makes the cast metal machinable. The strength of gray cast iron spans a significant range. The American Society for Testing of Materials (ASTM) uses a classification method for gray cast iron that is intended to provide a minimum tensile strength (TS) specification for the various classes: Class 20 gray cast iron has a TS of 20,000 lb=in2, Class 30 has a TS of 30,000 lb/in2, and so forth, up to around 70,000 lb=in2 (see Table 6.6 for equivalent TS in metric units). The compressive strength of gray cast iron is significantly greater than its tensile strength. Properties of the casting can be controlled to some extent by heat treatment. Ductility of gray cast iron is very low; it is a relatively brittle material. Products made from gray cast iron include automotive engine blocks and heads, motor housings, and machine tool bases. E1C06 11/11/2009 14:12:30 Page 119 119 Section 6.2/Ferrous Metals TABLE 6.6 Compositions and mechanical properties of selected cast irons. Typical Composition, % Type Gray cast irons ASTM Class 20 ASTM Class 30 ASTM Class 40 ASTM Class 50 Ductile irons ASTM A395 ASTM A476 White cast iron Low-C Malleable irons Ferritic Pearlitic Fe C Si Mn 93.0 93.6 93.8 93.5 3.5 3.2 3.1 3.0 2.5 2.1 1.9 1.6 0.65 0.75 0.85 1.0 94.4 93.8 3.0 3.0 2.5 3.0 92.5 2.5 1.3 0.4 95.3 95.1 2.6 2.4 1.4 1.4 0.4 0.8 Tensile Strength Othera lb/in2 Elongation, % 0.67 Mo 138 207 276 345 20,000 30,000 40,000 50,000 0.6 0.6 0.6 0.6 414 552 60,000 80,000 18 3 1.5Ni, 1Cr, 0.5Mo 276 40,000 0 345 414 50,000 60,000 10 10 MPa Compiled from [11]. Cast irons are identified by various systems. This table attempts to indicate the particular cast iron grade using the most common identification for each type. a Cast irons also contain phosphorus and sulfur usually totaling less than 0.3%. Ductile Iron This is an iron with the composition of gray iron in which the molten metal is chemically treated before pouring to cause the formation of graphite spheroids rather than flakes. This results in a stronger and more ductile iron, hence its name. Applications include machinery components requiring high strength and good wear resistance. White Cast Iron This cast iron has less carbon and silicon than gray cast iron. It is formed by more rapid cooling of the molten metal after pouring, thus causing the carbon to remain chemically combined with iron in the form of cementite (Fe3C), rather than precipitating out of solution in the form of flakes. When fractured, the surface has a white crystalline appearance that gives the iron its name. Owing to the cementite, white cast iron is hard and brittle, and its wear resistance is excellent. Strength is good, with TS of 276 MPa (40,000 lb/in2) being typical. These properties make white cast iron suitable for applications in which wear resistance is required. Railway brake shoes are an example. Malleable Iron When castings of white cast iron are heat treated to separate the carbon out of solution and form graphite aggregates, the resulting metal is called malleable iron. The new microstructure can possess substantial ductility (up to 20% elongation)—a significant difference from the metal out of which it was transformed. Typical products made of malleable cast iron include pipe fittings and flanges, certain machine components, and railroad equipment parts. Alloy Cast Irons Cast irons can be alloyed for special properties and applications. These alloy cast irons are classified as follows: (1) heat-treatable types that can be hardened by martensite formation; (2) corrosion-resistant types, whose alloying elements include nickel and chromium; and (3) heat-resistant types containing high proportions of nickel for hot hardness and resistance to high temperature oxidation. E1C06 11/11/2009 120 14:12:31 Page 120 Chapter 6/Metals 6.3 NONFERROUS METALS The nonferrous metals include metal elements and alloys not based on iron. The most important engineering metals in the nonferrous group are aluminum, copper, magnesium, nickel, titanium, and zinc, and their alloys. Although the nonferrous metals as a group cannot match the strength of the steels, certain nonferrous alloys have corrosion resistance and/or strength-to-weight ratios that make them competitive with steels in moderate-to-high stress applications. In addition, many of the nonferrous metals have properties other than mechanical that make them ideal for applications in which steel would be quite unsuitable. For example, copper has one of the lowest electrical resistivities among metals and is widely used for electrical wire. Aluminum is an excellent thermal conductor, and its applications include heat exchangers and cooking pans. It is also one of the most readily formed metals, and is valued for that reason also. Zinc has a relatively low melting point, so zinc is widely used in die casting operations. The common nonferrous metals have their own combination of properties that make them attractive in a variety of applications. The following nine sections discuss the nonferrous metals that are the most commercially and technologically important. 6.3.1 ALUMINUM AND ITS ALLOYS Aluminum and magnesium are light metals, and they are often specified in engineering applications for this feature. Both elements are abundant on Earth, aluminum on land and magnesium in the sea, although neither is easily extracted from their natural states. Properties and other data on aluminum are listed in Table 6.1(b). Among the major metals, it is a relative newcomer, dating only to the late 1800s (Historical Note 6.2). The coverage in this section includes (1) a brief description of how aluminum is produced and (2) a discussion of the properties and the designation system for the metal and its alloys. Aluminum Production The principal aluminum ore is bauxite, which consists largely of hydrated aluminum oxide (Al2O3-H2O) and other oxides. Extraction of the aluminum from bauxite can be summarized in three steps: (1) washing and crushing the ore into fine powders; (2) the Bayer process, in which the bauxite is converted to pure alumina (Al2O3); and (3) electrolysis, in which the alumina is separated into aluminum and TABLE 6.1 (continued): (b) Aluminum. Symbol: Atomic number: Specific gravity: Crystal structure: Melting temperature: Elastic modulus: Al 13 2.7 FCC 660 C (1220 F) 69,000 MPa (10  106 lb/in2) Principal ore: Bauxite (impure mix of Al2O3 and Al(OH)3) Alloying elements: Copper, magnesium, manganese, silicon, and zinc Typical applications: Containers (aluminum cans), wrapping foil, electrical conductors, pots and pans, parts for construction, aerospace, automotive, and other uses in which light weight is important E1C06 11/11/2009 14:12:31 Page 121 Section 6.3/Nonferrous Metals Historical Note 6.2 121 Aluminum I n 1807, the English chemist Humphrey Davy, believing that the mineral alumina (Al2O3) had a metallic base, attempted to extract the metal. He did not succeed, but was sufficiently convinced that he proceeded to name the metal anyway: alumium, later changing the name to aluminum. In 1825, the Danish physicist/chemist Hans Orsted finally succeeded in separating the metal. He noted that it ‘‘resembles tin.’’ In 1845, the German physicist Friedrich Wohler was the first to determine the specific gravity, ductility, and various other properties of aluminum. The modern electrolytic process for producing aluminum was based on the concurrent but independent work of Charles Hall in the United States and Paul Heroult in France around 1886. In 1888, Hall and a group of businessmen started the Pittsburgh Reduction Co. The first ingot of aluminum was produced by the electrolytic smelting process that same year. Demand for aluminum grew. The need for large amounts of electricity in the production process led the company to relocate in Niagara Falls in 1895, where hydroelectric power was becoming available at very low cost. In 1907, the company changed its name to the Aluminum Company of America (Alcoa). It was the sole producer of aluminum in the United States until World War II. oxygen gas (O2). The Bayer process, named after the German chemist who developed it, involves solution of bauxite powders in aqueous caustic soda (NaOH) under pressure, followed by precipitation of pure Al2O3 from solution. Alumina is commercially important in its own right as an engineering ceramic (Chapter 7). Electrolysis to separate Al2O3 into its constituent elements requires dissolving the precipitate in a molten bath of cryolite (Na3AlF6) and subjecting the solution to direct current between the plates of an electrolytic furnace. The electrolyte dissociates to form aluminum at the cathode and oxygen gas at the anode. Properties and Designation Scheme Aluminum has high electrical and thermal conductivity, and its resistance to corrosion is excellent because of the formation of a hard, thin oxide surface film. It is a very ductile metal and is noted for its formability. Pure aluminum is relatively low in strength, but it can be alloyed and heat treated to compete with some steels, especially when weight is an important consideration. The designation system for aluminum alloys is a four-digit code number. The system has two parts, one for wrought aluminums and the other for cast aluminums. The difference is that a decimal point is used after the third digit for cast aluminums. The designations are presented in Table 6.7(a). TABLE 6.7(a) Designations of wrought and cast aluminum alloys. Alloy Group Aluminum, 99.0% or higher purity Aluminum alloys, by major element(s): Copper Manganese Silicon + copper and/or magnesium Silicon Magnesium Magnesium and silicon Zinc Tin Other Wrought Code Cast Code 1XXX 1XX.X 2XXX 3XXX 2XX.X 4XXX 5XXX 6XXX 7XXX 8XXX 3XX.X 4XX.X 5XX.X 7XX.X 8XX.X 9XX.X E1C06 11/11/2009 122 14:12:31 Page 122 Chapter 6/Metals TABLE 6.7(b) Temper designations for aluminum alloys. Temper Description F As fabricated—no special treatment. H Strain hardened (wrought aluminums). H is followed by two digits, the first indicating a heat treatment, if any; and the second indicating the degree of work hardening remaining; for example: H1X No heat treatment after strain hardening, and X ¼ 1 to 9, indicating degree of work hardening. H2X Partially annealed, and X ¼ degree of work hardening remaining in product. H3X Stabilized, and X ¼ degree of work hardening remaining. Stabilized means heating to slightly above service temperature anticipated. O Annealed to relieve strain hardening and improve ductility; reduces strength to lowest level. T Thermal treatment to produce stable tempers other than F, H, or O. It is followed by a digit to indicate specific treatments; for example: T1 ¼ cooled from elevated temperature, naturally aged. T2 ¼ cooled from elevated temperature, cold worked, naturally aged. T3 ¼ solution heat treated, cold worked, naturally aged. T4 ¼ solution heat treated and naturally aged. T5 ¼ cooled from elevated temperature, artificially aged. T6 ¼ solution heat treated and artificially aged. T7 ¼ solution heat treated and overaged or stabilized. T8 ¼ solution heat treated, cold worked, artificially aged. T9 ¼ solution heat treated, artificially aged, and cold worked. T10 ¼ cooled from elevated temperature, cold worked, and artificially aged. W Solution heat treatment, applied to alloys that age harden in service; it is an unstable temper. Because properties of aluminum alloys are so influenced by work hardening and heat treatment, the temper (strengthening treatment, if any) must be designated in addition to the composition code. The principal temper designations are presented in Table 6.7(b). This designation is attached to the preceding four-digit number, separated from it by a hyphen, to indicate the treatment or absence thereof; for example, 2024-T3. Of course, temper treatments that specify strain hardening do not apply to the cast alloys. Some examples of the remarkable differences in the mechanical properties of aluminum alloys that result from the different treatments are presented in Table 6.8. 6.3.2 MAGNESIUM AND ITS ALLOYS Magnesium (Mg) is the lightest of the structural metals. Its specific gravity and other basic data are presented in Table 6.1(c). Magnesium and its alloys are available in both wrought and cast forms. It is relatively easy to machine. However, in all processing of magnesium, small TABLE 6.1 (continued): (c) Magnesium. Symbol: Atomic number: Specific gravity: Crystal structure: Melting temperature: Elastic modulus: Mg 12 1.74 HCP 650 C (1202 F) 48,000 MPa (7  106 lb/in2) Extracted from: MgCl2 in sea water by electrolysis Alloying elements: See Table 6.9 Typical applications: Aerospace, missiles, bicycles, chain saw housings, luggage, and other applications in which light weight is a primary requirement E1C06 11/11/2009 14:12:32 Page 123 123 Section 6.3/Nonferrous Metals TABLE 6.8 Compositions and mechanical properties of selected aluminum alloys. a Tensile Strength Typical Composition, % Cu Si Temper MPa lb/in2 Elongation 0.4 0.3 0.6 0.3 O H18 O H18 O T3 O H36 O H18 O H38 O T4 76 159 90 165 185 485 180 260 130 285 125 200 90 172 11,000 23,000 13,000 24,000 27,000 70,000 26,000 38,000 19,000 41,000 18,000 29,000 13,000 25,000 39 7 40 10 20 18 22 7 25 1 18 3 25 20 Code Al Fe Mg Mn 1050 99.5 1100 99.0 2024 93.5 4.4 0.5 1.5 0.6 0.5 3004 96.5 0.3 0.7 1.0 1.2 0.3 4043 93.5 0.3 0.8 5050 96.9 0.2 0.7 1.4 6063 98.5 0.3 0.7 5.2 0.1 0.4 0.4 Compiled from [12]. In addition to elements listed, alloy may contain trace amounts of other elements such as copper, magnesium, manganese, vanadium, and zinc. a particles of the metal (such as small metal cutting chips) oxidize rapidly, and care must be taken to avoid fire hazards. Magnesium Production Sea water contains about 0.13% MgCl2, and this is the source of most commercially produced magnesium. To extract Mg, a batch of sea water is mixed with milk of lime–calcium hydroxide (Ca(OH)2). The resulting reaction precipitates magnesium hydroxide (Mg(OH)2) that settles and is removed as a slurry. The slurry is then filtered to increase Mg(OH)2 content and then mixed with hydrochloric acid (HCl), which reacts with the hydroxide to form concentrated MgCl2—much more concentrated than the original sea water. Electrolysis is used to decompose the salt into magnesium (Mg) and chlorine gas (Cl2). The magnesium is then cast into ingots for subsequent processing. The chlorine is recycled to form more MgCl2. Properties and Designation Scheme As a pure metal, magnesium is relatively soft and lacks sufficient strength for most engineering applications. However, it can be alloyed and heat treated to achieve strengths comparable to aluminum alloys. In particular, its strength-to-weight ratio is an advantage in aircraft and missile components. The designation scheme for magnesium alloys uses a three-to-five character alphanumeric code. The first two characters are letters that identify the principal alloying elements (up to two elements can be specified in the code, in order of decreasing percentages, or alphabetically if equal percentages). These code letters are listed in Table 6.9. The letters are followed by a two-digit number that indicates, respectively, the amounts of the two alloying ingredients to the nearest percent. Finally, the last symbol is a letter that indicates some variation in composition, or simply the chronological order in which it was standardized for commercial availability. Magnesium alloys also require specification of a temper, and the same basic scheme presented in Table 6.7(b) for aluminum is used for magnesium alloys. Some examples of magnesium alloys, illustrating the designation scheme and indicating tensile strength and ductility of these alloys, are presented in Table 6.10. E1C06 11/11/2009 124 14:12:32 Page 124 Chapter 6/Metals TABLE 6.9 Code letters used to identify alloying elements in magnesium alloys. A Aluminum (Al) E Rate earth metals TABLE 6.10 H Thorium (Th) K Zirconium (Zr) M Manganese (Mn) P Lead (Pb) Q Silver (Ag) S Silicon (Si) T Tin (Sn) Z Zinc (Zn) Compositions and mechanical properties of selected magnesium alloys. Typical Composition, % Code Mg Al Mn Si Zn AZ10A AZ80A HM31A ZK21A AM60 AZ63A 98.0 91.0 95.8 97.1 92.8 91.0 1.3 8.5 0.2 0.1 0.4 0.5 1.2 6.0 6.0 0.1 0.5 2.3 0.2 3.0 Tensile Strength Other 3.0 Th 6 Zr 0.3 Cu Process MPa lb/in2 Elongation Wrought Forged Wrought Wrought Cast Cast 240 330 283 260 220 200 35,000 48,000 41,000 38,000 32,000 29,000 10 11 10 4 6 6 Compiled from [12]. 6.3.3 COPPER AND ITS ALLOYS Copper (Cu) is one of the oldest metals known (Historical Note 6.3). Basic data on the element copper are presented in Table 6.1(d). Copper Production In ancient times, copper was available in nature as a free element. Today these natural deposits are more difficult to find, and copper is now extracted from ores that are mostly sulfides, such as chalcopyrite (CuFeS2). The ore is crushed (Section 17.1.1), concentrated by flotation, and then smelted (melted or fused, often with an associated chemical reaction to separate a metal from its ore). The resulting copper is called blister copper, which is between 98% and 99% pure. Electrolysis is used to obtain higher purity levels suitable for commercial use. Properties and Designation Scheme Pure copper has a distinctive reddish-pink color, butitsmostdistinguishingengineeringpropertyisitslowelectricalresistivity—oneofthelowest Historical Note 6.3 C Copper opper was one of the first metals used by human cultures (gold was the other). Discovery of the metal was probably around 6000 BCE. At that time, copper was found in the free metallic state. Ancient peoples fashioned implements and weapons out of it by hitting the metal (cold forging). Pounding copper made it harder (strain hardening); this and its attractive reddish color made it valuable in early civilizations. Around 4000 BCE, it was discovered that copper could be melted and cast into useful shapes. It was later found that copper mixed with tin could be more readily cast and worked than the pure metal. This led to the widespread use of bronze and the subsequent naming of the Bronze Age, dated from about 2000 BCE to the time of Christ. To the ancient Romans, the island of Cyprus was almost the only source of copper. They called the metal aes cyprium (ore of Cyprus). This was shortened to Cyprium and subsequently renamed Cuprium. From this derives the chemical symbol Cu. E1C06 11/11/2009 14:12:33 Page 125 Section 6.3/Nonferrous Metals TABLE 6.1 125 (continued): (d) Copper. Symbol: Atomic number: Specific gravity: Crystal structure: Melting temperature: Elastic modulus: Cu 29 8.96 FCC 1083 C (1981 F) 110,000 MPa (16  106 lb/in2) Ore extracted from: Several: e.g., chalcopyrite (CuFeS2). Alloying elements: Tin (bronze), zinc (brass), aluminum, silicon, nickel, and Typical applications: beryllium. Electrical conductors and components, ammunition (brass), pots and pans, jewelry, plumbing, marine applications, heat exchangers, springs (Be-Cu). ofallelements.Becauseofthisproperty,anditsrelativeabundanceinnature,commerciallypure copperiswidelyusedasanelectricalconductor.(Notethattheconductivityofcopper decreases significantly as alloying elements are added.) Cu is also an excellent thermal conductor. Copper isoneofthenoblemetals(goldandsilverarealsonoblemetals),soitiscorrosionresistant.Allof these properties combine to make copper one of the most important metals. On the downside, the strength and hardness of copper are relatively low, especially when weight is taken into account. Accordingly, to improve strength (as well as for other reasons), copper is frequently alloyed. Bronze is an alloy of copper and tin (typically about 90% Cu and 10% Sn), still widely used today despite its ancient ancestry. Additional bronze alloys have been developed, based on other elements than tin; these include aluminum bronzes, and silicon bronzes. Brass is another familiar copper alloy, composed of copper and zinc (typically around 65% Cu and 35% Zn). The highest strength alloy of copper is beryllium-copper (only about 2% Be). It can be heat treated to tensile strengths of 1035 MPa (150,000 lb/in2). Be-Cu alloys are used for springs. The designation of copper alloys is based on the Unified Numbering System for Metals and Alloys (UNS), which uses a five-digit number preceded by the letter C (C for copper). The alloys are processed in wrought and cast forms, and the designation system includes both. Some copper alloys with compositions and mechanical properties are presented in Table 6.11. 6.3.4 NICKEL AND ITS ALLOYS Nickel (Ni) is similar to iron in many respects. It is magnetic, and its modulus of elasticity is virtually the same as that of iron and steel. However, it is much more corrosion resistant, and the high temperature properties of its alloys are generally superior. Because of its corrosion-resistant characteristics, it is widely used as an alloying element in steel, such as stainless steel, and as a plating metal on other metals such as plain carbon steel. TABLE 6.1 (continued): (e) Nickel. Symbol: Atomic number: Specific gravity: Crystal structure: Melting temperature: Elastic Modulus: Ni 28 8.90 FCC 1453 C (2647 F) 209,000 MPa (30  106 lb/in2) Ore extracted from: Pentlandite ((Fe, Ni)9S8) Alloying elements: Copper, chromium, iron, aluminum. Typical applications: Stainless steel alloying ingredient, plating metal for steel, applications requiring high temperature and corrosion resistance. E1C06 11/11/2009 126 14:12:33 Page 126 Chapter 6/Metals TABLE 6.11 Compositions and mechanical properties of selected copper alloys. Typical Composition, % Code Cu C10100 C11000 C17000 C24000 C26000 C52100 C71500 C71500b Be 99.99 99.95 98.0 80.0 70.0 92.0 70.0 70.0 1.7 Ni Tensile Strength Sn Zn MPa lb/in2 Elongation, % 20.0 30.0 235 220 500 290 300 380 380 580 34,000 32,000 70,000 42,000 44,000 55,000 55,000 84,000 45 45 45 52 68 70 45 3 a 8.0 30.0 30.0 Compiled from [12]. Small amounts of Ni and Fe þ 0.3 Co. b Heat treated for high strength. a Nickel Production The most important ore of nickel is pentlandite ((Ni, Fe)9S8). To extract the nickel, the ore is first crushed and ground with water. Flotation techniques are used to separate the sulfides from other minerals mixed with the ore. The nickel sulfide is then heated to burn off some of the sulfur, followed by smelting to remove iron and silicon. Further refinement is accomplished in a Bessemer-style converter to yield high-concentration nickel sulfide (NiS). Electrolysis is then used to recover high-purity nickel from the compound. Ores of nickel are sometimes mixed with copper ores, and the recovery technique described here also yields copper in these cases. Nickel Alloys Alloys of nickel are commercially important in their own right and are noted for corrosion resistance and high temperature performance. Composition, tensile strength, and ductility of some of the nickel alloys are given in Table 6.12. In addition, a number of superalloys are based on nickel (Section 6.4). 6.3.5 TITANIUM AND ITS ALLOYS Titanium (Ti) is fairly abundant in nature, constituting about 1% of Earth’s crust (aluminum, the most abundant, is about 8%). The density of Ti is between aluminum and iron; these and other data are presented in Table 6.1(f). Its importance has grown in recent decades due to TABLE 6.12 Compositions and mechanical properties of selected nickel alloys. Typical Composition, % Code 270 200 400 600 230 Ni 99.9 99.0 66.8 74.0 52.8 Cr 16.0 22.0 Cu Fe a a 0.2 30.0 0.5 0.3 2.5 8.0 3.0 Tensile Strength Mn Si Other 0.2 0.2 1.0 0.4 0.2 0.5 0.5 0.4 C, S C b Compiled from [12]. Trace amounts. b Other alloying ingredients in Grade 230: 5% Co, 2% Mo, 14% W, 0.3% Al, 0.1% C. a MPa lb/in2 345 462 550 655 860 50,000 67,000 80,000 95,000 125,000 Elongation, % 50 47 40 40 47 E1C06 11/11/2009 14:12:34 Page 127 Section 6.3/Nonferrous Metals TABLE 6.1 127 (continued): (f) Titanium. Symbol: Atomic number: Specific gravity: Crystal structure: Melting temperature: Elastic modulus: Ores extracted from: Rutile (TiO2) and Ilmenite (FeTiO3) Alloying elements: Aluminum, tin, vanadium, copper, and magnesium Typical applications: Jet engine components, other aerospace applications, prosthetic implants Ti 22 4.51 HCP 1668 C (3034 F) 117,000 MPa (17  106 lb/in2) its aerospace applications, in which its light weight and good strength-to-weight ratio are exploited. Titanium Production The principal ores of titanium are rutile, which is 98% to 99% TiO2, and ilmenite, which is a combination of FeO and TiO2. Rutile is preferred as an ore because of its higher Ti content. In recovery of the metal from its ores, the TiO2 is converted to titanium tetrachloride (TiCl4) by reacting the compound with chlorine gas. This is followed by a sequence of distillation steps to remove impurities. The highly concentrated TiCl4 is then reduced to metallic titanium by reaction with magnesium; this is known as the Kroll process. Sodium can also be used as a reducing agent. In either case, an inert atmosphere must be maintained to prevent O2, N2, or H2 from contaminating the Ti, owing to its chemical affinity for these gases. The resulting metal is used to cast ingots of titanium and its alloys. Properties of Titanium Ti’s coefficient of thermal expansion is relatively low among metals. It is stiffer and stronger than aluminum, and it retains good strength at elevated temperatures. Pure titanium is reactive, which presents problems in processing, especially in the molten state. However, at room temperature it forms a thin adherent oxide coating (TiO2) that provides excellent corrosion resistance. These properties give rise to two principal application areas for titanium: (1) in the commercially pure state, Ti is used for corrosion resistant components, such as marine components and prosthetic implants; and (2) titanium alloys are used as high-strength components in temperatures ranging from ambient to above 550 C (1000 F), especially where its excellent strength-to-weight ratio is exploited. These latter applications include aircraft and missile components. Some of the alloying elements used with titanium include aluminum, manganese, tin, and vanadium. Some compositions and mechanical properties for several alloys are presented in Table 6.13. TABLE 6.13 Compositions and mechanical properties of selected titanium alloys. Typical Composition, % Codea Ti Al R50250 R56400 R54810 R56620 99.8 89.6 90.0 84.3 6.0 8.0 6.0 Compiled from [1] and [12]. United Numbering System (UNS). b Traces of C, H, O. a Cu Fe 0.2 0.3 0.8 0.8 Tensile Strength V 4.0 1.0 6.0 Other b 1 Mob 2 Snb MPa lb/in2 240 1000 985 1030 35,000 145,000 143,000 150,000 Elongation, % 24 12 15 14 E1C06 11/11/2009 128 14:12:34 Page 128 Chapter 6/Metals TABLE 6.1 (continued): (g) Zinc. Symbol: Atomic number: Specific gravity: Crystal structure: Melting temperature: a Zn 30 7.13 HCP 419 C (786 F) Elastic modulus: Ore extracted from: Alloying elements: Typical applications: 90,000 MPa (13  106 lb/in2)a Sphalerite (ZnS) Aluminum, magnesium, copper Galvanized steel and iron, die castings, alloying element in brass Zinc creeps, which makes it difficult to measure modulus of elasticity; some tables of properties omit E for zinc for this reason. 6.3.6 ZINC AND ITS ALLOYS Table 6.1(g) lists basic data on zinc. Its low melting point makes it attractive as a casting metal. It also provides corrosion protection when coated onto steel or iron; galvanized steel is steel that has been coated with zinc. Production of Zinc Zinc blende or sphalerite is the principal ore of zinc; it contains zinc sulfide (ZnS). Other important ores include smithsonite, which is zinc carbonate (ZnCO3), and hemimorphate, which is hydrous zinc silicate (Zn4Si2O7OH-H2O). Sphalerite must be concentrated (beneficiated, as it is called) because of the small fraction of zinc sulfide present in the ore. This is accomplished by first crushing the ore, then grinding with water in a ball mill (Section 17.1.1) to create a slurry. In the presence of a frothing agent, the slurry is agitated so that the mineral particles float to the top and can be skimmed off (separated from the lower-grade minerals). The concentrated zinc sulfide is then roasted at around 1260 C (2300 F), so that zinc oxide (ZnO) is formed from the reaction. There are various thermochemical processes for recovering zinc from this oxide, all of which reduce zinc oxide by means of carbon. The carbon combines with oxygen in ZnO to form CO and/or CO2, thus freeing Zn in the form of vapor that is condensed to yield the desired metal. An electrolytic process is also widely used, accounting for about half the world’s production of zinc. This process also begins with the preparation of ZnO, which is mixed with dilute sulfuric acid (H2SO4), followed by electrolysis to separate the resulting zinc sulfate (ZnSO4) solution to yield the pure metal. Zinc Alloys and Applications Several alloys of zinc are listed in Table 6.14, with data on composition, tensile strength, and applications. Zinc alloys are widely used in die casting to mass produce components for the automotive and appliance industries. Another major application of zinc is in galvanized steel. As the name suggests, a galvanic cell is created in TABLE 6.14 Compositions, tensile strength, and applications of selected zinc alloys. Typical Composition, % Code Zn Al Cu Mg Fe MPa lb/in2 Application Z33520 Z35540 Z35635 Z35840 Z45330 95.6 93.4 91.0 70.9 98.9 4.0 4.0 8.0 27.0 0.25 2.5 1.0 2.0 1.0 0.04 0.04 0.02 0.02 0.01 0.1 0.1 0.06 0.07 283 359 374 425 227 41,000 52,000 54,000 62,000 33,000 Die casting Die casting Foundry alloy Foundry alloy Rolled alloy Compiled from [12]. UNS, Unified Numbering System for metals. a Tensile Strength E1C06 11/11/2009 14:12:34 Page 129 Section 6.3/Nonferrous Metals TABLE 6.1 129 (continued): (h) Lead and tin Symbol: Atomic number: Specific gravity: Crystal structure: Melting temperature: Modulus of elasticity: Ore from which extracted: Typical alloying elements: Typical applications: Lead Tin Pb 82 11.35 FCC 327 C (621 F) 21,000 MPa (3  106 lb/in2) Galena (PbS) Tin, antimony See text Sn 50 7.30 HCP 232 C (449 F) 42,000 MPa (6  106 lb/in2) Cassiterite (SnO2) Lead, copper Bronze, solder, tin cans galvanized steel (Zn is the anode and steel is the cathode) that protects the steel from corrosive attack. A third important use of zinc is in brass. As previously indicated in the discussion of copper, this alloy consists of copper and zinc, in the ratio of about 2/3 Cu to 1/3 Zn. Finally, readers may be interested to know that the U.S. one cent coin is mostly zinc. The penny is coined out of zinc and then electroplated with copper, so that the final proportions are 97.5% Zn and 2.5% Cu. It costs the U.S. Mint about 1.5 cents to produce each penny. 6.3.7 LEAD AND TIN Lead (Pb) and tin (Sn) are often considered together because of their low melting temperatures, and because they are used in soldering alloys to make electrical connections. The phase diagram for the tin–lead alloy system is depicted in Figure 6.3. Basic data for lead and tin are presented in Table 6.1(h). Lead is a dense metal with a low melting point; other properties include low strength, low hardness (the word ‘‘soft’’ is appropriate), high ductility, and good corrosion resistance. In addition to its use in solder, applications of lead and its alloys include ammunition, type metals, x-ray shielding, storage batteries, bearings, and vibration damping. It has also been widely used in chemicals and paints. Principal alloying elements with lead are tin and antimony. Tin has an even lower melting point than lead; other properties include low strength, low hardness, and good ductility. The earliest use of tin was in bronze, the alloy consisting of copper and tin developed around 3000 BCE in Mesopotamia and Egypt. Bronze is still an important commercial alloy (although its relative importance has declined during 5000 years). Other uses of tin include tin-coated sheet steel containers (‘‘tin cans’’) for storing food and, of course, solder metal. 6.3.8 REFRACTORY METALS The refractory metals are metals capable of enduring high temperatures. The most important metals in this group are molybdenum and tungsten; see Table 6.1(i). Other refractory metals are columbium (Cb) and tantalum (Ta). In general, these metals and their alloys are capable of maintaining high strength and hardness at elevated temperatures. Molybdenum has a high melting point and is relatively dense, stiff, and strong. It is used both as a pure metal (99.9+% Mo) and as an alloy. The principal alloy is TZM, which contains small amounts of titanium and zirconium (less than 1% total). Mo and its alloys possess good high temperature strength, and this accounts for many of its applications, which include heat shields, heating elements, electrodes for resistance welding, dies for high E1C06 11/11/2009 130 14:12:35 Page 130 Chapter 6/Metals TABLE 6.1 (continued): (i) Refractory metals. Symbol: Atomic number: Specific gravity: Crystal structure: Melting point: Elastic modulus: Principal ores: Alloying elements: Applications: a Molybdenum Tungsten Mo 42 10.2 BCC 2619 C (4730 F) 324,000 MPa (47  106 lb/in2) Molybdenite (MoS2) W 74 19.3 BCC 3400 C (6150 F) 407,000 MPa (59  106 lb/in3) Scheelite (CaWO4), Wolframite ((Fe,Mn)WO4) a See text See text Light filaments, rocket engine parts, WC tools. Tungsten is used as a pure metal and as an alloying ingredient, but few alloys are based on W. temperature work (e.g., die casting molds), and parts for rocket and jet engines. In addition to these applications, molybdenum is also widely used as an alloying ingredient in other metals, such as steels and superalloys. Tungsten (W) has the highest melting point among metals and is one of the densest. It is also the stiffest and hardest of all pure metals. Its most familiar application is filament wire in incandescent light bulbs. Applications of tungsten are typically characterized by high operating temperatures, such as parts for rocket and jet engines and electrodes for arc welding. W is also widely used as an element in tool steels, heat resistant alloys, and tungsten carbide (Section 7.3.2). A major disadvantage of both Mo and W is their propensity to oxidize at high temperatures, above about 600 C (1000 F), thus detracting from their high temperature properties. To overcome this deficiency, either protective coatings must be used on these metals in high temperature applications or the metal parts must operate in a vacuum. For example, the tungsten filament must be energized in a vacuum inside the glass light bulb. 6.3.9 PRECIOUS METALS The precious metals, also called the noble metals because they are chemically inactive, include silver, gold, and platinum. They are attractive metals, available in limited supply, and have been used throughout civilized history for coinage and to underwrite paper TABLE 6.1 (continued): ( j) The precious metals. Symbol: Atomic number: Specific gravity: Crystal structure: Melting temperature: Principal ores: Applications: Gold Platinum Silver Au 79 19.3 FCC 1063 C (1945 F) Pt 78 21.5 FCC 1769 C (3216 F) Ag 47 10.5 FCC 961 C (1762 F) a a a See text See text See text a All three precious metals are mined from deposits in which the pure metal is mixed with other ores and metals. Silver is also mined from the ore Argentite (Ag2S). E1C06 11/11/2009 14:12:35 Page 131 Section 6.4/Superalloys 131 currency. They are also widely used in jewelry and similar applications that exploit their high value. As a group, these precious metals possess high density, good ductility, high electrical conductivity, and good corrosion resistance; see Table 6.1(j). Silver (Ag) is less expensive per unit weight than gold or platinum. Nevertheless, its attractive ‘‘silvery’’ luster makes it a highly valued metal in coins, jewelry, and tableware (which even assumes the name of the metal: ‘‘silverware’’). It is also used for fillings in dental work. Silver has the highest electrical conductivity of any metal, which makes it useful for contacts in electronics applications. Finally, it should be mentioned that lightsensitive silver chloride and other silver halides are the basis for photography. Gold (Au) is one of the heaviest metals; it is soft and easily formed, and possesses a distinctive yellow color that adds to its value. In addition to currency and jewelry, its applications include electrical contacts (owing to its good electrical conductivity and corrosion resistance), dental work, and plating onto other metals for decorative purposes. Platinum (Pt) is also used in jewelry and is in fact more expensive than gold. It is the most important of six precious metals known as the platinum group metals, which consists of Ruthenium (Ru), Rhodium (Rh), Palladium (Pd), Osmium (Os), and Iridium (Ir), in addition to Pt. They are clustered in a rectangle in the periodic table (Figure 2.1). Osmium, Iridium, and Platinum are all denser than gold (Ir is the densest material known, at 22.65 g/ cm3). Because the platinum group metals are all scarce and very expensive, their applications are generally limited to situations in which only small amounts are needed and their unique properties are required (e.g., high melting temperatures, corrosion resistance, and catalytic characteristics). The applications include thermocouples, electrical contacts, spark plugs, corrosion resistant devices, and catalytic pollution control equipment for automobiles. 6.4 SUPERALLOYS Superalloys constitute a category that straddles the ferrous and nonferrous metals. Some of them are based on iron, whereas others are based on nickel and cobalt. In fact, many of the superalloys contain substantial amounts of three or more metals, rather than consisting of one base metal plus alloying elements. Although the tonnage of these metals is not significant compared with most of the other metals discussed in this chapter, they are nevertheless commercially important because they are very expensive; and they are technologically important because of what they can do. The superalloys are a group of high-performance alloys designed to meet very demanding requirements for strength and resistance to surface degradation (corrosion and oxidation) at high service temperatures. Conventional room temperature strength is usually not the important criterion for these metals, and most of them possess room temperature strength properties that are good but not outstanding. Their high temperature performance is what distinguishes them; tensile strength, hot hardness, creep resistance, and corrosion resistance at very elevated temperatures are the mechanical properties of interest. Operating temperatures are often in the vicinity of 1100 C (2000 F). These metals are widely used in gas turbines—jet and rocket engines, steam turbines, and nuclear power plants—systems in which operating efficiency increases with higher temperatures. The superalloys are usually divided into three groups, according to their principal constituent: iron, nickel, or cobalt: å Iron-based alloys have iron as the main ingredient, although in some cases the iron is less than 50% of the total composition. å Nickel-based alloys generally have better high temperature strength than alloy steels. Nickel is the base metal. The principal alloying elements are chromium and E1C06 11/11/2009 132 14:12:35 Page 132 Chapter 6/Metals TABLE 6.15 Some typical superalloy compositions together with strength properties at room temperature and elevated temperature. Tensile Strength at Room Temperature a Chemical Analysis, % Superalloy Iron-based Incoloy 802 Haynes 556 Nickel-based Incoloy 718 Rene 41 Hastelloy S Nimonic 75 Cobalt-based Stellite 6B Haynes 188 L-605 Fe Ni Co Cr Mo 46 29 32 20 20 21 22 3 18 53 55 67 76 1 3 3 3 3 22 10 11 53 39 53 19 19 16 20 3 1 15 30 22 20 2 W 5 14 15 Otherb Tensile Strength at 870 C (1600 F) MPa lb/in2 MPa lb/in2 <2 6 690 815 100,000 118,000 195 330 28,000 48,000 6 5 1 <2 1435 1420 845 745 208,000 206,000 130,000 108,000 340 620 340 150 49,000 90,000 50,000 22,000 4 1010 960 1005 146,000 139,000 146,000 385 420 325 56,000 61,000 47,000 2 Compiled from [11] and [12]. Compositions to nearest percent. b Other elements include carbon, niobium, titanium, tungsten, manganese, and silicon. a cobalt; lesser elements include aluminum, titanium, molybdenum, niobium (Nb), and iron. Some familiar names in this group include Inconel, Hastelloy, and Rene 41. å Cobalt-based alloys consist of cobalt (around 40%) and chromium (perhaps 20%) as their main components. Other alloying elements include nickel, molybdenum, and tungsten. In virtually all of the superalloys, including those based on iron, strengthening is accomplished by precipitation hardening. The iron-based superalloys do not use martensite formation for strengthening. Typical compositions and strength properties at room temperature and elevated temperature for some of the alloys are presented in Table 6.15. 6.5 GUIDE TO THE PROCESSING OF METALS A wide variety of manufacturing processes are available to shape metals, enhance their properties, assemble them, and finish them for appearance and protection. Shaping, Assembly, and Finishing Processes Metals are shaped by all of the basic processes, including casting, powder metallurgy, deformation processes, and material removal. In addition, metal parts are joined to form assemblies by welding, brazing, soldering, and mechanical fastening; and finishing processes are commonly used to improve the appearance of metal parts and/or to provide corrosion protection. These finishing operations include electroplating and painting. Enhancement of Mechanical Properties in Metals Mechanical properties of metals can be altered by a number of techniques. Some of these techniques have E1C06 11/11/2009 14:12:35 Page 133 Review Questions 133 been referred to in the discussion of the various metals. Methods for enhancing mechanical properties of metals can be grouped into three categories: (1) alloying, (2) cold working, and (3) heat treatment. Alloying has been discussed throughout the present chapter and is an important technique for strengthening metals. Cold working has previously been referred to as strain hardening; its effect is to increase strength and reduce ductility. The degree to which these mechanical properties are affected depends on the amount of strain and the strain hardening exponent in the flow curve, Eq. (3.10). Cold working can be used on both pure metals and alloys. It is accomplished during deformation of the workpart by one of the shape forming processes, such as rolling, forging, or extrusion. Strengthening of the metal therefore occurs as a by-product of the shaping operation. Heat treatment refers to several types of heating and cooling cycles performed on a metal to beneficially change its properties. They operate by altering the basic microstructure of the metal, which in turn determines mechanical properties. Some heat treatment operations are applicable only to certain types of metals; for example, the heat treatment of steel to form martensite is somewhat specialized because martensite is unique to steel. Heat treatments for steels and other metals are discussed in Chapter 27. REFERENCES [1] Bauccio. M. (ed.). ASM Metals Reference Book, 3rd ed. ASM International, Materials Park, Ohio, 1993. [2] Black, J, and Kohser, R. DeGarmo’s Materials and Processes in Manufacturing, 10th ed., John Wiley & Sons, Hoboken, New Jersey, 2008. [3] Brick, R. M., Pense, A. W., and Gordon, R. B. Structure and Properties of Engineering Materials, 4th ed. McGraw-Hill, New York, 1977. [4] Carnes, R., and Maddock, G., ‘‘Tool Steel Selection,’’ Advanced Materials & Processes, June 2004, pp. 37–40. [5] Encyclopaedia Britannica, Vol. 21, Macropaedia. Encyclopaedia Britannica, Chicago, 1990, under section: Industries, Extraction and Processing. [6] Flinn, R. A., and Trojan, P. K. Engineering Materials and Their Applications, 5th ed. John Wiley & Sons, New York, 1995. [7] Guy, A. G., and Hren, J. J. Elements of Physical Metallurgy, 3rd ed. Addison-Wesley, Reading, Massachusetts, 1974. [8] Hume-Rothery, W., Smallman, R. E., and Haworth, C. W. The Structure of Metals and Alloys. Institute of Materials, London, 1988. [9] Keefe, J.‘‘A Brief Introduction to Precious Metals,’’ The AMMTIAC Quarterly, Vol. 2, No. 1, 2007. [10] Lankford, W. T., Jr., Samways, N. L., Craven, R. F., and McGannon, H. E. The Making, Shaping, and Treating of Steel, 10th ed. United States Steel Co., Pittsburgh, 1985. [11] Metals Handbook, Vol. 1, Properties and Selection: Iron, Steels, and High Performance Alloys. ASM International, Metals Park, Ohio, 1990. [12] Metals Handbook, Vol. 2, Properties and Selection: Nonferrous Alloys and Special Purpose Materials, ASM International, Metals Park, Ohio, 1990. [13] Moore, C., and Marshall, R. I. Steelmaking. The Institute for Metals, The Bourne Press, Ltd., Bournemouth, U.K., 1991. [14] Wick, C., and Veilleux, R. F. (eds.). Tool and Manufacturing Engineers Handbook, 4, Vol. 3, Materials, Finishing, and Coating. Society of Manufacturing Engineers, Dearborn, Michigan, 1985. REVIEW QUESTIONS 6.1. What are some of the general properties that distinguish metals from ceramics and polymers? 6.2. What are the two major groups of metals? Define them. 6.3. What is an alloy? 6.4. What is a solid solution in the context of alloys? 6.5. Distinguish between a substitutional solid solution and an interstitial solid solution. 6.6. What is an intermediate phase in the context of alloys? E1C06 11/11/2009 134 14:12:36 Page 134 Chapter 6/Metals 6.7. The copper-nickel system is a simple alloy system, as indicated by its phase diagram. Why is it so simple? 6.8. What is the range of carbon percentages that defines an iron–carbon alloy as a steel? 6.9. What is the range of carbon percentages that defines an iron–carbon alloy as cast iron? 6.10. Identify some of the common alloying elements other than carbon in low alloy steels. 6.11. What are some of the mechanisms by which the alloying elements other than carbon strengthen steel? 6.12. What is the predominant alloying element in all of the stainless steels? 6.13. Why is austenitic stainless steel called by that name? 6.14. Besides high carbon content, what other alloying element is characteristic of the cast irons? 6.15. Identify some of the properties for which aluminum is noted. 6.16. What are some of the noteworthy properties of magnesium? 6.17. What is the most important engineering property of copper that determines most of its applications? 6.18. What elements are traditionally alloyed with copper to form (a) bronze and (b) brass? 6.19. What are some of the important applications of nickel? 6.20. What are the noteworthy properties of titanium? 6.21. Identify some of the important applications of zinc. 6.22. What important alloy is formed from lead and tin? 6.23. (a) Name the important refractory metals. (b) What does the term refractory mean? 6.24. (a) Name the four principal noble metals. (b) Why are they called noble metals? 6.25. The superalloys divide into three basic groups, according to the base metal used in the alloy. Name the three groups. 6.26. What is so special about the superalloys? What distinguishes them from other alloys? 6.27. What are the three basic methods by which metals can be strengthened? MULTIPLE CHOICE QUIZ There are 20 correct answers in the following multiple choice questions (some questions have multiple answers that are correct). To attain a perfect score on the quiz, all correct answers must be given. Each correct answer is worth 1 point. Each omitted answer or wrong answer reduces the score by 1 point, and each additional answer beyond the correct number of answers reduces the score by 1 point. Percentage score on the quiz is based on the total number of correct answers. 6.1. Which of the following properties or characteristics are inconsistent with the metals (two correct answers): (a) good thermal conductivity, (b) high strength, (c) high electrical resistivity, (d) high stiffness, and (e) ionic bonding? 6.2. Which one of the metallic elements is the most abundant on the earth: (a) aluminum, (b) copper, (c) iron, (d) magnesium, or (e) silicon? 6.3. The predominant phase in the iron–carbon alloy system for a composition with 99% Fe at room temperature is which one of the following: (a) austenite, (b) cementite, (c) delta, (d) ferrite, or (e) gamma? 6.4. A steel with 1.0% carbon is known as which one of the following: (a) eutectoid, (b) hypoeutectoid, (c) hypereutectoid, or (d) wrought iron? 6.5. The strength and hardness of steel increases as carbon content (a) increases or (b) decreases? 6.6. Plain carbon steels are designated in the AISI code system by which of the following: (a) 01XX, (b) 10XX, (c) 11XX, (d) 12XX, or (e) 30XX? 6.7. Which one of the following elements is the most important alloying ingredient in steel: (a) carbon, (b) chromium, (c) nickel, (d) molybdenum, or (e) vanadium? 6.8. Which one of the following is not a common alloying ingredient in steel: (a) chromium, (b) manganese, (c) nickel, (d) vanadium, (e) zinc? 6.9. Solid solution alloying is the principal strengthening mechanism in high-strength low-alloy (HSLA) steels: (a) true or (b) false? 6.10. Which of the following alloying elements are most commonly associated with stainless steel (two best answers): (a) chromium, (b) manganese, (c) molybdenum, (d) nickel, and (e) tungsten? 6.11. Which of the following is the most important cast iron commercially: (a) ductile cast iron, (b) gray cast iron, (c) malleable iron, or (d) white cast iron? 6.12. Which one of the following metals has the lowest density: (a) aluminum, (b) magnesium, (c) tin, or (d) titanium? 6.13. Which of the following metals has the highest density: (a) gold, (b) lead, (c) platinum, (d) silver, or (e) tungsten? 6.14. From which of the following ores is aluminum derived: (a) alumina, (b) bauxite, (c) cementite, (d) hematite, or (e) scheelite? E1C06 11/11/2009 14:12:37 Page 135 Problems 6.15. Which of the following metals is noted for its good electrical conductivity (one best answer): (a) copper, (b) gold, (c) iron, (d) nickel, or (e) tungsten? 6.16. Traditional brass is an alloy of which of the following metallic elements (two correct answers): 135 (a) aluminum, (b) copper, (c) gold, (d) tin, and (e) zinc? 6.17. Which one of the following metals has the lowest melting point: (a) aluminum, (b) lead, (c) magnesium, (d) tin, or (e) zinc? PROBLEMS 6.1. For the copper-nickel phase diagram in Figure 6.2, find the compositions of the liquid and solid phases for a nominal composition of 70% Ni and 30% Cu at 1371 C (2500 F). 6.2. For the preceding problem, use the inverse lever rule to determine the proportions of liquid and solid phases present in the alloy. 6.3. Using the lead–tin phase diagram in Figure 6.3, determine the liquid and solid phase compositions for a nominal composition of 40% Sn and 60% Pb at 204 C (400 F). 6.4. For the preceding problem, use the inverse lever rule to determine the proportions of liquid and solid phases present in the alloy. 6.5. Using the lead–tin phase diagram in Figure 6.3, determine the liquid and solid phase compositions for a nominal composition of 90% Sn and 10% Pb at 204 C (400 F). 6.6. For the preceding problem, use the inverse lever rule to determine the proportions of liquid and solid phases present in the alloy. 6.7. In the iron–iron carbide phase diagram of Figure 6.4, identify the phase or phases present at the following temperatures and nominal compositions: (a) 650 C (1200 F) and 2% Fe3C, (b) 760 C (1400 F) and 2% Fe3C, and (c) 1095 C (2000 F) and 1% Fe3C. E1C07 11/02/2009 7 14:42:42 Page 136 CERAMICS Chapter Contents 7.1 Structure and Properties of Ceramics 7.1.1 Mechanical Properties 7.1.2 Physical Properties 7.2 Traditional Ceramics 7.2.1 Raw Materials 7.2.2 Traditional Ceramic Products 7.3 New 7.3.1 7.3.2 7.3.3 Ceramics Oxide Ceramics Carbides Nitrides 7.4 Glass 7.4.1 Chemistry and Properties of Glass 7.4.2 Glass Products 7.4.3 Glass-Ceramics 7.5 Some Important Elements Related to Ceramics 7.5.1 Carbon 7.5.2 Silicon 7.5.3 Boron 7.6 Guide to Processing Ceramics 136 We usually consider metals to be the most important class of engineering materials. However, it is of interest to note that ceramic materials are actually more abundant and widely used. Included in this category are clay products (e.g., bricks and pottery), glass, cement, and more modern ceramic materials such as tungsten carbide and cubic boron nitride. This is the class of materials discussed in this chapter. We also include coverage of several elements related to ceramics because they are sometimes used in similar applications. These elements are carbon, silicon, and boron. The importance of ceramics as engineering materials derives from their abundance in nature and their mechanical and physical properties, which are quite different from those of metals. A ceramic material is an inorganic compound consisting of a metal (or semimetal) and one or more nonmetals. The word ceramic traces from the Greek keramos meaning potter’s clay or wares made from fired clay. Important examples of ceramic materials are silica, or silicon dioxide (SiO2), the main ingredient in most glass products; alumina, or aluminum oxide (Al2O3), used in applications ranging from abrasives to artificial bones; and more complex compounds such as hydrous aluminum silicate (Al2Si2O5(OH)4), known as kaolinite, the principal ingredient in most clay products. The elements in these compounds are the most common in Earth’s crust; see Table 7.1. The group includes many additional compounds, some of which occur naturally while others are manufactured. The general properties that make ceramics useful in engineered products are high hardness, good electrical and thermal insulating characteristics, chemical stability, and high melting temperatures. Some ceramics are translucent—window glass being the clearest example. They are also brittle and possess virtually no ductility, which can cause problems in both processing and performance of ceramic products. The commercial and technological importance of ceramics is best demonstrated by the variety of products and applications that are based on this class of material. The list includes: E1C07 11/02/2009 14:42:42 Page 137 Section 7.1/Structure and Properties of Ceramics TABLE 7.1 137 Most common elements in the Earth’s crust, with approximate percentages. Oxygen 50% Silicon Aluminum Iron Calcium Sodium Potassium Magnesium 26% 7.6% 4.7% 3.5% 2.7% 2.6% 2.0% Compiled from [6]. å Clay construction products, such as bricks, clay pipe, and building tile å Refractory ceramics, which are capable of high temperature applications such as furnace walls, crucibles, and molds å Cement used in concrete, used for construction and roads (concrete is a composite material, but its components are ceramics) å Whiteware products, including pottery, stoneware, fine china, porcelain, and other tableware, based on mixtures of clay and other minerals å Glass used in bottles, glasses, lenses, window panes, and light bulbs å Glass fibers for thermal insulating wool, reinforced plastics (fiberglass), and fiber optics communications lines å Abrasives, such as aluminum oxide and silicon carbide å Cutting tool materials, including tungsten carbide, aluminum oxide, and cubic boron nitride å Ceramic insulators, which are used in applications such as electrical transmission components, spark plugs, and microelectronic chip substrates å Magnetic ceramics, for example, in computer memories å Nuclear fuels based on uranium oxide (UO2) å Bioceramics, which include materials used in artificial teeth and bones For purposes of organization, we classify ceramic materials into three basic types: (1) traditional ceramics—silicates used for clay products such as pottery and bricks, common abrasives, and cement; (2) new ceramics—more recently developed ceramics based on nonsilicates such as oxides and carbides, and generally possessing mechanical or physical properties that are superior or unique compared to traditional ceramics; and (3) glasses—based primarily on silica and distinguished from the other ceramics by their noncrystalline structure. In addition to the three basic types, we have glass ceramics— glasses that have been transformed into a largely crystalline structure by heat treatment. 7.1 STRUCTURE AND PROPERTIES OF CERAMICS Ceramic compounds are characterized by covalent and ionic bonding. These bonds are stronger than metallic bonding in metals, which accounts for the high hardness and stiffness but low ductility of ceramic materials. Just as the presence of free electrons in the metallic bond explains why metals are good conductors of heat and electricity, the presence of tightly held electrons in ceramic molecules explains why these materials are poor conductors. The strong bonding also provides these materials with high melting temperatures, although some ceramics decompose, rather than melt, at elevated temperatures. Most ceramics take a crystalline structure. The structures are generally more complex than those of most metals. There are several reasons for this. First, ceramic molecules usually consist of atoms that are significantly different in size. Second, the ion charges are often different, as in many of the common ceramics such as SiO2 and Al2O3. Both of these factors tend to force a more complicated physical arrangement of the atoms in the molecule and in the resulting crystal structure. In addition, many ceramic materials consist of more than two E1C07 11/02/2009 138 14:42:43 Page 138 Chapter 7/Ceramics elements, such as (Al2Si2O5(OH)4), also leading to further complexity in the molecular structure. Crystalline ceramics can be single crystals or polycrystalline substances. In the more common second form, mechanical and physical properties are affected by grain size; higher strength and toughness are achieved in the finer-grained materials. Some ceramic materials tend to assume an amorphous structure or glassy phase, rather than a crystalline form. The most familiar example is, of course, glass. Chemically, most glasses consist of fused silica. Variations in properties and colors are obtained by adding other glassy ceramic materials such as oxides of aluminum, boron, calcium, and magnesium. In addition to these pure glasses, many ceramics that have a crystal structure use the glassy phase as a binder for their crystalline phase. 7.1.1 MECHANICAL PROPERTIES Basic mechanical properties of ceramics are presented in Chapter 3. Ceramic materials are rigid and brittle, exhibiting a stress-strain behavior best characterized as perfectly elastic (see Figure 3.6). As seen in Table 7.2, hardness and elastic modulus for many of the new ceramics are greater than those of metals (see Tables 3.1, 3.6, and 3.7). Stiffness and hardness of traditional ceramics and glasses are significantly less than for new ceramics. Theoretically, the strength of ceramics should be higher than that of metals because of their atomic bonding. The covalent and ionic bonding types are stronger than metallic bonding. However, metallic bonding has the advantage that it allows for slip, the basic mechanism by which metals deform plastically when subjected to high stresses. Bonding in ceramics is more rigid and does not permit slip under stress. The inability to slip makes it much more difficult for ceramics to absorb stresses. Yet ceramics contain the same imperfections in their crystal structure as metals—vacancies, interstitialcies, displaced atoms, and microscopic cracks. These internal flaws tend to concentrate the stresses, especially when a tensile, bending, or impact loading is involved. As a result of these factors, ceramics fail by brittle fracture under applied stress much more readily than metals. Their TABLE 7.2 Selected mechanical and physical properties of ceramic materials. Elastic modulus, E Melting Temperature Hardness (Vickers) Gpa Brick-fireclay Cement, Portland Silicon carbide (SiC) New ceramics NA NA 2600 HV 95 50 460 14  106 7  106 68  106 2.3 2.4 3.2 NA NA 27,007a NA NA 48,927a Alumina (Al2O3) Cubic boron nitride (cBN) Titanium carbide (TiC) Tungsten carbide (WC) Glass 2200 HV 6000 HV 3200 HV 2600 HV 345 NA 300 700 50  106 NA 45  106 100  106 3.8 2.3 4.9 15.6 2054 30,007a 3250 2870 3729 54,307a 5880 5198 500 HV 69 10  106 2.2 7b 7b Material 2 (lb/in ) Specific Gravity   C F Traditional ceramics Silica glass (SiO2) NA ¼ Not available or not applicable. The ceramic material chemically dissociates or, in the case of diamond and graphite, sublimes (vaporizes), rather than melts. b Glass, being noncrystalline, does not melt at a specific melting point. Instead, it gradually exhibits fluid properties with increasing temperature. It becomes liquid at around 1400 C (2550 F). Compiled from [3], [4], [5], [6], [9], [10], and other sources. a E1C07 11/02/2009 14:42:44 Page 139 Section 7.2/Traditional Ceramics 139 tensile strength and toughness are relatively low. Also, their performance is much less predictable due to the random nature of the imperfections and the influence of processing variations, especially in products made of traditional ceramics. The frailties that limit the tensile strength of ceramic materials are not nearly so operative when compressive stresses are applied. Ceramics are substantially stronger in compression than in tension. For engineering and structural applications, designers have learned to use ceramic components so that they are loaded in compression rather than tension or bending. Various methods have been developed to strengthen ceramics, nearly all of which have as their fundamental approach the minimization of surface and internal flaws and their effects. These methods include [7]: (1) making the starting materials more uniform; (2) decreasing grain size in polycrystalline ceramic products; (3) minimizing porosity; (4) introducing compressive surface stresses, for example, through application of glazes with low thermal expansions, so that the body of the product contracts after firing more than the glaze, thus putting the glaze in compression; (5) using fiber reinforcement; and (6) heat treatments, such as quenching alumina from temperatures in the slightly plastic region to strengthen it. 7.1.2 PHYSICAL PROPERTIES Several of the physical properties of ceramics are presented in Table 7.2. Most ceramic materials are lighter than metals and heavier than polymers (see Table 4.1). Melting temperatures are higher than for most metals, some ceramics preferring to decompose rather than melt. Electrical and thermal conductivities of most ceramics are lower than for metals; but the range of values is greater, permitting some ceramics to be used as insulators while others are electrical conductors. Thermal expansion coefficients are somewhat less than for the metals, but the effects are more damaging in ceramics because of their brittleness. Ceramic materials with relatively high thermal expansions and low thermal conductivities are especially susceptible to failures of this type, which result from significant temperature gradients and associated volumetric changes in different regions of the same part. The terms thermal shock and thermal cracking are used in connection with such failures. Certain glasses (for example, those containing high proportions of SiO2) and glass ceramics are noted for their low thermal expansion and are particularly resistant to these thermal failures (Pyrex is a familiar example). 7.2 TRADITIONAL CERAMICS These materials are based on mineral silicates, silica, and mineral oxides. The primary products are fired clay (pottery, tableware, brick, and tile), cement, and natural abrasives such as alumina. These products, and the processes used to make them, date back thousands of years (see Historical Note 7.1). Glass is also a silicate ceramic material and is often included within the traditional ceramics group [5], [6]. We cover glass in a later section because it is distinguished from the above crystalline materials by its amorphous or vitreous structure (the term vitreous means glassy, or possessing the characteristics of glass). 7.2.1 RAW MATERIALS Mineral silicates, such as clays of various compositions, and silica, such as quartz, are among the most abundant substances in nature and constitute the principal raw materials for E1C07 11/02/2009 140 14:42:44 Page 140 Chapter 7/Ceramics Historical Note 7.1 M Ancient pottery ceramics aking pottery has been an art since the earliest civilizations. Archeologists examine ancient pottery and similar artifacts to study the cultures of the ancient world. Ceramic pottery does not corrode or disintegrate with age nearly as rapidly as artifacts made of wood, metal, or cloth. Somehow, early tribes discovered that clay is transformed into a hard solid when placed near an open fire. Burnt clay articles have been found in the Middle East that date back nearly 10,000 years. Earthenware pots and similar products became an established commercial trade in Egypt by around 4000 BCE. The greatest advances in pottery making were made in China, where fine white stoneware was first crafted as early as 1400 BCE. By the ninth century, the Chinese were making articles of porcelain, which was fired at higher temperatures than earthenware or stoneware to partially vitrify the more complex mixture of raw materials and produce translucency in the final product. Dinnerware made of Chinese porcelain was highly valued in Europe; it was called ‘‘china.’’ It contributed significantly to trade between China and Europe and influenced the development of European culture. traditional ceramics. These solid crystalline compounds have been formed and mixed in the Earth’s crust over billions of years by complex geological processes. The clays are the raw materials used most widely in ceramics. They consist of fine particles of hydrous aluminum silicate that become a plastic substance that is formable and moldable when mixed with water. The most common clays are based on the mineral kaolinite (Al2Si2O5(OH)4). Other clay minerals vary in composition, both in terms of proportions of the basic ingredients and through additions of other elements such as magnesium, sodium, and potassium. Besides its plasticity when mixed with water, a second characteristic of clay that makes it so useful is that it fuses into a dense, strong material when heated to a sufficiently elevated temperature. The heat treatment is known as firing. Suitable firing temperatures depend on clay composition. Thus, clay can be shaped while wet and soft, and then fired to obtain the final hard ceramic product. Silica (SiO2) is another major raw material for the traditional ceramics. It is the principal component in glass, and an important ingredient in other ceramic products including whiteware, refractories, and abrasives. Silica is available naturally in various forms, the most important of which is quartz. The main source of quartz is sandstone. The abundance of sandstone and its relative ease of processing means that silica is low in cost; it is also hard and chemically stable. These features account for its widespread use in ceramic products. It is generally mixed in various proportions with clay and other minerals to achieve the appropriate characteristics in the final product. Feldspar is one of the other minerals often used. Feldspar refers to any of several crystalline minerals that consist of aluminum silicate combined with either potassium, sodium, calcium, or barium. The potassium blend, for example, has the chemical composition KAlSi3O8. Mixtures of clay, silica, and feldspar are used to make stoneware, china, and other tableware. Still another important raw material for traditional ceramics is alumina. Most alumina is processed from the mineral bauxite, which is an impure mixture of hydrous aluminum oxide and aluminum hydroxide plus similar compounds of iron or manganese. Bauxite is also the principal ore in the production of aluminum metal. A purer but less common form of Al2O3 is the mineral corundum, which contains alumina in massive amounts. Slightly impure forms of corundum crystals are the colored gemstones sapphire and ruby. Alumina ceramic is used as an abrasive in grinding wheels and as a refractory brick in furnaces. Silicon carbide, also used as an abrasive, does not occur as a mineral. Instead, it is produced by heating mixtures of sand (source of silicon) and coke (carbon) to a temperature of around 2200 C (4000 F), so that the resulting chemical reaction forms SiC and carbon monoxide. E1C07 11/02/2009 14:42:45 Page 141 Section 7.2/Traditional Ceramics 141 7.2.2 TRADITIONAL CERAMIC PRODUCTS The minerals discussed above are the ingredients for a variety of ceramic products. We organize our coverage here by major categories of traditional ceramic products. A summary of these products, and the raw materials and ceramics out of which they are made, is presented in Table 7.3. We limit our coverage to materials commonly in with manufactured products, thus omitting certain commercially important ceramics such as cement. Pottery and Tableware This category is one of the oldest, dating back thousands of years; yet it is still one of the most important. It includes tableware products that we all use: earthenware, stoneware, and china. The raw materials for these products are clay usually combined with other minerals such as silica and feldspar. The wetted mixture is shaped and then fired to produce the finished piece. Earthenware is the least refined of the group; it includes pottery and similar articles made in ancient times. Earthenware is relatively porous and is often glazed. Glazing involves application of a surface coating, usually a mixture of oxides such as silica and alumina, to make the product less pervious to moisture and more attractive to the eye. Stoneware has lower porosity than earthenware, resulting from closer control of ingredients and higher firing temperatures. China is fired at even higher temperatures, which produces the translucence in the finished pieces that characterize their fine quality. The reason for this is that much of the ceramic material has been converted to the glassy (vitrified) phase, which is relatively transparent compared to the polycrystalline form. Modern porcelain is nearly the same as china and is produced by firing the components, mainly clay, silica, and feldspar, at still higher temperatures to achieve a very hard, dense, glassy material. Porcelain is used in a variety of products ranging from electrical insulation to bathtub coatings. Brick and Tile Building brick, clay pipe, unglazed roof tile, and drain tile are made from various low-cost clays containing silica and gritty matter widely available in natural deposits. These products are shaped by pressing (molding) and firing at relatively low temperatures. Refractories Refractory ceramics, often in the form of bricks, are critical in many industrial processes that require furnaces and crucibles to heat and/or melt materials. The useful properties of refractory materials are high temperature resistance, thermal insulation, and resistance to chemical reaction with the materials (usually molten metals) being heated. As we have mentioned, alumina is often used as a refractory ceramic, together with silica. Other refractory materials include magnesium oxide (MgO) and calcium oxide (CaO). The refractory lining often contains two layers, the outside layer being more porous because this increases the insulation properties. Abrasives Traditional ceramics used for abrasive products, such as grinding wheels and sandpaper, are alumina and silicon carbide. Although SiC is the harder material (hardness of SiC is 2600 HV vs. 2200 HV for alumina), the majority of grinding wheels are based on TABLE 7.3 Summary of traditional ceramic products. Product Principal Chemistry Minerals and Raw Materials Pottery, tableware Porcelain Brick, tile Refractory Abrasive: silicon carbide Abrasive: aluminum oxide Al2Si2O5(OH)4, SiO2, KAlSi3O8 Al2Si2O5(OH)4, SiO2, KAlSi3O8 Al2 Si2O5(OH)4, SiO2 plus fine stones Al2O3, SiO2 Others: MgO, CaO SiC Al2O3 Clay + silica + feldspar Clay + silica + feldspar Clay + silica + other Alumina and silica Silica + coke Bauxite or alumina E1C07 11/02/2009 14:42:46 Page 143 Section 7.3/New Ceramics 143 7.3.2 CARBIDES The carbide ceramics include silicon carbide (SiC), tungsten carbide (WC), titanium carbide (TiC), tantalum carbide (TaC), and chromium carbide (Cr3C2). Silicon carbide was discussed previously. Although it is a man-made ceramic, the methods for its production were developed a century ago, and therefore it is generally included in the traditional ceramics group. In addition to its use as an abrasive, other SiC applications include resistance heating elements and additives in steelmaking. WC, TiC, and TaC are valued for their hardness and wear resistance in cutting tools and other applications requiring these properties. Tungsten carbide was the first to be developed (Historical Note 7.2) and is the most important and widely used material in the group. WC is typically produced by carburizing tungsten powders that have been reduced from tungsten ores such as wolframite (FeMnWO4) and scheelite (CaWO4). Titanium carbide is produced by carburizing the minerals rutile (TiO2) or ilmenite (FeTiO3). And tantalum carbide is made by carburizing either pure tantalum powders or tantalum pentoxide (Ta2O5) [11]. Chromium carbide is more suited to applications where chemical stability and oxidation resistance are important. Cr3C2 is prepared by carburizing chromium oxide (Cr2O3) as the starting compound. Carbon black is the usual source of carbon in all of these reactions. Historical Note 7.2 T Tungsten carbide he compound WC does not occur in nature. It was first fabricated in the late 1890s by the Frenchman Henri Moissan. However, the technological and commercial importance of the development was not recognized for two decades. Tungsten became an important metal for incandescent lamp filaments in the early 1900s. Wire drawing was required to produce the filaments. The traditional tool steel draw dies of the period were unsatisfactory for drawing tungsten wire due to excessive wear. There was a need for a much harder material. The compound WC was known to possess such hardness. In 1914 in Germany, H. Voigtlander and H. Lohmann developed a fabrication process for hard carbide draw dies by sintering parts pressed from powders of tungsten carbide and/or molybdenum carbide. Lohmann is credited with the first commercial production of sintered carbides. The breakthrough leading to the modern technology of cemented carbides is linked to the work of K. Schroter in Germany in the early and mid-1920s. He used WC powders mixed with about 10% of a metal from the iron group, finally settling on cobalt as the best binder, and sintering the mixture at a temperature close to the melting point of the metal. The hard material was first marketed in Germany as ‘‘Widia’’ in 1926. The Schroter patents were assigned to the General Electric Company under the trade name ‘‘Carboloy’’—first produced in the United States around 1928. Widia and Carboloy were used as cutting tool materials, with cobalt content in the range 4% to 13%. They were effective in the machining of cast iron and many nonferrous metals, but not in the cutting of steel. When steel was machined, the tools would wear rapidly by cratering. In the early 1930s, carbide cutting tool grades with WC and TiC were developed for steel cutting. In 1931, the German firm Krupp started production of Widia X, which had a composition 84% WC, 10% TiC, and 6% cobalt (Co). And Carboloy Grade 831 was introduced in the United States in 1932; it contained 69% WC, 21% TiC, and 10% Co. Except for SiC, each carbide discussed here must be combined with a metallic binder such as cobalt or nickel in order to fabricate a useful solid product. In effect, the carbide powders bonded in a metal framework creates what is known as a cemented carbide—a composite material, specifically a cermet (reduced from ceramic and metal). We examine cemented carbides and other cermets in Section 9.2.1. The carbides have little engineering value except as constituents in a composite system. E1C07 11/02/2009 144 14:42:46 Page 144 Chapter 7/Ceramics 7.3.3 NITRIDES The important nitride ceramics are silicon nitride (Si3N4), boron nitride (BN), and titanium nitride (TiN). As a group, the nitride ceramics are hard and brittle, and they melt at high temperatures (but not generally as high as the carbides). They are usually electrically insulating, except for TiN. Silicon nitride shows promise in high temperature structural applications. Si3N4 oxidizes at about 1200 C (2200 F) and chemically decomposes at around 1900 C (3400 F). It has low thermal expansion, good resistance to thermal shock and creep, and resists corrosion by molten nonferrous metals. These properties have provided applications for this ceramic in gas turbines, rocket engines, and melting crucibles. Boron nitride exists in several structures, similar to carbon. The important forms of BN are (1) hexagonal, similar to graphite; and (2) cubic, same as diamond; in fact, its hardness is comparable to that of diamond. This latter structure goes by the names cubic boron nitride and borazon, symbolized cBN, and is produced by heating hexagonal BN under very high pressures. Owing to its extreme hardness, the principal applications of cBN are in cutting tools (Section 23.2.5) and abrasive wheels (Section 25.1.1). Interestingly, it does not compete with diamond cutting tools and grinding wheels. Diamond is suited to nonsteel machining and grinding, while cBN is appropriate for steel. Titanium nitride has properties similar to those of other nitrides in this group, except for its electrical conductivity; it is a conductor. TiN has high hardness, good wear resistance, and a low coefficient of friction with the ferrous metals. This combination of properties makes TiN an ideal material as a surface coating on cutting tools. The coating is only around 0.006 mm (0.00024 in) thick, so the amounts of material used in this application are low. A new ceramic material related to the nitride group, and also to the oxides, is the oxynitride ceramic called sialon. It consists of the elements silicon, aluminum, oxygen, and nitrogen; and its name derives from these ingredients: Si-Al-O-N. Its chemical composition is variable, a typical composition being Si4Al2O2N6. Properties of sialon are similar to those of silicon nitride, but it has better resistance to oxidation at high temperatures than Si3N4. Its principal application is for cutting tools, but its properties may make it suitable for other high temperature applications in the future. 7.4 GLASS The term glass is somewhat confusing because it describes a state of matter as well as a type of ceramic. As a state of matter, the term refers to an amorphous, or noncrystalline, structure of a solid material. The glassy state occurs in a material when insufficient time is allowed during cooling from the molten condition for the crystalline structure to form. It turns out that all three categories of engineering materials (metals, ceramics, and polymers) can assume the glassy state, although the circumstances for metals to do so are quite rare. As a type of ceramic, glass is an inorganic, nonmetallic compound (or mixture of compounds) that cools to a rigid condition without crystallizing; it is a ceramic that is in the glassy state as a solid material. This is the material we shall discuss in this section—a material that dates back 4500 years (Historical Note 7.3). 7.4.1 CHEMISTRY AND PROPERTIES OF GLASS The principal ingredient in virtually all glasses is silica, most commonly found as the mineral quartz in sandstone and silica sand. Quartz occurs naturally as a crystalline substance; but when melted and then cooled, it forms vitreous silica. Silica glass has a very low thermal expansion coefficient and is therefore quite resistant to thermal shock. These properties are E1C07 11/02/2009 14:42:47 Page 145 Section 7.4/Glass Historical Note 7.3 145 History of glass T he oldest glass specimens, dating from around 2500 BCE, are glass beads and other simple shapes found in Mesopotamia and ancient Egypt. These were made by painstakingly sculpturing glass solids, rather than by molding or shaping molten glass. It was a thousand years before the ancient cultures exploited the fluid properties of hot glass, by pouring it in successive layers over a sand core until sufficient thickness and rigidity had been attained in the product, a cup-shaped vessel. This pouring technique was used until around 200 BCE, when a simple tool was developed that revolutionized glassworking—the blowpipe. Glassblowing was probably first accomplished in Babylon and later by the Romans. It was performed using an iron tube several feet long, with a mouthpiece on one end and a fixture for holding the molten glass on the other. A blob of hot glass in the required initial shape and viscosity was attached to the end of the iron tube, and then blown into shape by an artisan either freely in air or into a mold cavity. Other simple tools were utilized to add the stem and/or base to the object. The ancient Romans showed great skill in their use of various metallic oxides to color glass. Their technology is evident in the stained glass windows of cathedrals and churches of the Middle Ages in Italy and the rest of Europe. The art of glassblowing is still practiced today for certain consumer glassware; and automated versions of glassblowing are used for massproduced glass products such as bottles and light bulbs (Chapter 12). ideal for elevated temperature applications; accordingly, Pyrex and chemical glassware designed for heating are made with high proportions of silica glass. In order to reduce the melting point of glass for easier processing, and to control properties, the composition of most commercial glasses includes other oxides as well as silica. Silica remains as the main component in these glass products, usually comprising 50% to 75% of total chemistry. The reason SiO2 is used so widely in these compositions is because it is the best glass former. It naturally transforms into a glassy state upon cooling from the liquid, whereas most ceramics crystallize upon solidification. Table 7.4 lists typical TABLE 7.4 Typical compositions of selected glass products. Chemical Composition (by weight to nearest %) Product SiO2 Na2O CaO Al2O3 Soda-lime glass 71 14 13 2 Window glass 72 15 8 1 4 Container glass 72 13 10 2a 2 Light bulb glass 73 17 5 1 4 MgO K2O PbO B2O3 Other 1 Laboratory glass Vycor 96 Pyrex 81 4 E-glass (fibers) 54 1 S-glass (fibers) 64 17 1 3 2 13 15 4 26 10 9 Optical glasses Crown glass 67 8 12 Flint glass 46 3 6 Compiled from [4], [5] and [10], and other sources. May include Fe2O3 with Al2O3 a 12 45 ZnO E1C07 11/02/2009 146 14:42:47 Page 146 Chapter 7/Ceramics chemistries for some common glasses. The additional ingredients are contained in a solid solution with SiO2, and each has a function: (1) acting as flux (promoting fusion) during heating; (2) increasing fluidity in the molten glass for processing; (3) retarding devitrification—the tendency to crystallize from the glassy state; (4) reducing thermal expansion in the final product; (5) improving the chemical resistance against attack by acids, basic substances, or water; (6) adding color to the glass; and (7) altering the index of refraction for optical applications (e.g., lenses). 7.4.2 GLASS PRODUCTS Following is a list of the major categories of glass products. We examine the roles played by the different ingredients in Table 7.4 as we discuss these products. Window Glass This glass is represented by two chemistries in Table 7.4: (1) soda-lime glass and (2) window glass. The soda-lime formula dates back to the glass-blowing industry of the 1800s and earlier. It was (and is) made by mixing soda (Na2O) and lime (CaO) with silica (SiO2) as the major ingredient. The blending of ingredients has evolved empirically to achieve a balance between avoiding crystallization during cooling and achieving chemical durability of the final product. Modern window glass and the techniques for making it have required slight adjustments in composition and closer control over its variation. Magnesia (MgO) has been added to help reduce devitrification. Containers In previous times, the same basic soda-lime composition was used for manual glass-blowing to make bottles and other containers. Modern processes for shaping glass containers cool the glass more rapidly than older methods. Also, the importance of chemical stability in container glass is better understood today. Resulting changes in composition have attempted to optimize the proportions of lime (CaO) and soda (Na2O3). Lime promotes fluidity. It also increases devitrification, but since cooling is more rapid, this effect is not as important as in prior processing techniques with slower cooling rates. Soda reduces chemical instability and solubility of the container glass. Light Bulb Glass Glass used in light bulbs and other thin glass items (e.g., drinking glasses, Christmas ornaments) is high in soda and low in lime; it also contains small amounts of magnesia and alumina. The chemistry is dictated largely by the economics of large volumes involved in light bulb manufacture. The raw materials are inexpensive and suited to the continuous melting furnaces used today. Laboratory Glassware These products include containers for chemicals (e.g., flasks, beakers, glass tubing). The glass must be resistant to chemical attack and thermal shock. Glass that is high in silica is suitable because of its low thermal expansion. The trade name ‘‘Vicor’’ is used for this high-silica glass. This product is very insoluble in water and acids. Additions of boric oxide also produce a glass with low coefficient of thermal expansion, so some glass for laboratory ware contains B2O3 in amounts of around 13%. The trade name ‘‘Pyrex’’ is used for the borosilicate glass developed by the Corning Glass Works. Both Vicor and Pyrex are included in our listing as examples of this product category. Glass Fibers Glass fibers are manufactured for a number of important applications, including fiberglass reinforced plastics, insulation wool, and fiber optics. The compositions vary according to function. The most commonly used glass reinforcing fibers in plastics are E-glass. It is high in CaO and Al2O3 content, it is economical, and it possesses good tensile strength in fiber form. Another glass fiber material is S-glass, which has higher strength but is not as economical as E-glass. Compositions are indicated in our table. E1C07 11/02/2009 14:42:48 Page 147 Section 7.4/Glass 147 Insulating fiberglass wool can be manufactured from regular soda-lime-silica glasses. The glass product for fiber optics consists of a long, continuous core of glass with high refractive index surrounded by a sheath of lower refractive glass. The inside glass must have a very high transmittance for light in order to accomplish long distance communication. Optical Glasses Applications for these glasses include lenses for eyeglasses and optical instruments such as cameras, microscopes, and telescopes. To achieve their function, the glasses must have different refractive indices, but each lens must be homogenous in composition. Optical glasses are generally divided into: crowns and flints. Crown glass has a low index of refraction, while flint glass contains lead oxide (PbO) that gives it a high index of refraction. 7.4.3 GLASS-CERAMICS Glass-ceramics are a class of ceramic material produced by conversion of glass into a polycrystalline structure through heat treatment. The proportion of crystalline phase in the final product typically ranges between 90% and 98%, with the remainder being unconverted vitreous material. Grain size is usually between 0.1 and 1.0 mm (4 and 40 m-in), significantly smaller than the grain size of conventional ceramics. This fine crystal microstructure makes glass-ceramics much stronger than the glasses from which they are derived. Also, due to their crystal structure, glass-ceramics are opaque (usually gray or white) rather than clear. The processing sequence for glass-ceramics is as follows: (1) The first step involves heating and forming operations used in glassworking (Section 12.2) to create the desired product geometry. Glass shaping methods are generally more economical than pressing and sintering to shape traditional and new ceramics made from powders. (2) The product is cooled. (3) The glass is reheated to a temperature sufficient to cause a dense network of crystal nuclei to form throughout the material. It is the high density of nucleation sites that inhibits grain growth of individual crystals, thus leading ultimately to the fine grain size in the glass-ceramic material. The key to the propensity for nucleation is the presence of small amounts of nucleating agents in the glass composition. Common nucleating agents are TiO2, P2O5, and ZrO2. (4) Once nucleation is initiated, the heat treatment is continued at a higher temperature to cause growth of the crystalline phases. Several examples of glass-ceramic systems and typical compositions are listed in Table 7.5. The Li2O-Al2O3-SiO2 system is the most important commercially; it includes Corning Ware (Pyroceram), the familiar product of the Corning Glass Works. The significant advantages of glass-ceramics include (1) efficiency of processing in the glassy state, (2) close dimensional control over the final product shape, and (3) good mechanical and physical properties. Properties include high strength (stronger than glass), absence of porosity, low coefficient of thermal expansion, and high resistance to thermal TABLE 7.5 Several glass-ceramic systems. Typical Composition (to nearest %) Glass-Ceramic System Li2O–Al2O3–SiO2 MgO–Al2O3–SiO2 Na2O–BaO–Al2O3–SiO2 Compiled from [5], [6], and [10]. Li2O MgO Na2O BaO 3 13 13 9 Al2O3 SiO2 TiO2 18 70 5 30 47 10 29 41 7 E1C07 11/02/2009 148 14:42:48 Page 148 Chapter 7/Ceramics shock. These properties have resulted in applications in cooking ware, heat exchangers, and missile radomes. Certain systems (e.g., MgO-Al2O3-SiO2 system) are also characterized by high electrical resistance, suitable for electrical and electronics applications. 7.5 SOME IMPORTANT ELEMENTS RELATED TO CERAMICS In this section, several elements of engineering importance are discussed: carbon, silicon, and boron. We encounter these materials on occasion in subsequent chapters. Although they are not ceramic materials according to our definition, they sometimes compete for applications with ceramics. And they have important applications of their own. Basic data on these elements are presented in Table 7.6. 7.5.1 CARBON Carbon occurs in two alternative forms of engineering and commercial importance: graphite and diamond. They compete with ceramics in various applications: graphite in situations where its refractory properties are important, and diamond in industrial applications where hardness is the critical factor (such as cutting and grinding tools). Graphite Graphite has a high content of crystalline carbon in the form of layers. Bonding between atoms in the layers is covalent and therefore strong, but the parallel layers are bonded to each other by weak van der Waals forces. This structure makes graphite quite anisotropic; strength and other properties vary significantly with direction. It explains why graphite can be used both as a lubricant and as a fiber in advanced composite materials. In powder form, graphite possesses low frictional characteristics due to the ease with which it shears between the layers; in this form, graphite is valued as a lubricant. In fiber form, graphite is oriented in the hexagonal planar direction to produce a filament material of very high strength and elastic modulus. These graphite fibers are used in structural composites ranging from tennis rackets to fighter aircraft components. Graphite exhibits certain high temperature properties that are both useful and unusual. It is resistant to thermal shock, and its strength actually increases with temperature. Tensile strength at room temperature is about 100 MPa (14,500 lb/in2), but increases to about twice this value at 2500 C (4500 F) [5]. Theoretical density of carbon is 2.22 g/cm3, but apparent density of bulk graphite is lower due to porosity (around 1.7 g/cm3). This is TABLE 7.6 Some basic data and properties of carbon, silicon, and boron. Symbol Atomic number Specific gravity Melting temperature Elastic modulus, GPa (lb/in2) Hardness (Mohs scale) Carbon Silicon Boron C 6 2.25 3727 Ca (6740 F) b 240 (35  106)c 10357c (150  106)c 1b, 10c Si 14 2.42 1410 C (2570 F) NA B 5 2.34 2030 C (3686 F) 393 (57  106) 7 9.3 NA = not available. Carbon sublimes (vaporizes) rather than melt. b Carbon in the form of graphite (typical value given). c Carbon in the form of diamond. a E1C07 11/02/2009 150 14:42:48 Page 150 Chapter 7/Ceramics 7.5.2 SILICON Silicon is a semimetallic element in the same group in the periodic table as carbon (Figure 2.1). Silicon is one of the most abundant elements in the Earth’s crust, comprising about 26% by weight (Table 7.1). It occurs naturally only as a chemical compound—in rocks, sand, clay, and soil—either as silicon dioxide or as more complex silicate compounds. As an element it has the same crystalline structure as diamond, but its hardness is lower. It is hard but brittle, lightweight, chemically inactive at room temperature, and is classified as a semiconductor. The greatest amounts of silicon in manufacturing are in ceramic compounds (SiO2 in glass and silicates in clays) and alloying elements in steel, aluminum, and copper alloys. It is also used as a reducing agent in certain metallurgical processes. Of significant technological importance is pure silicon as the base material in semiconductor manufacturing in electronics. The vast majority of integrated circuits produced today are made from silicon (Chapter 34). 7.5.3 BORON Boron is a semimetallic element in the same periodic group as aluminum. It is only about 0.001% of the Earth’s crust by weight, commonly occurring as the minerals borax (Na2B4O7–10H2O) and kernite (Na2B4O7–4H2O). Boron is lightweight and very stiff (high modulus of elasticity) in fiber form. In terms of electrical properties, it is classified as a semiconductor (its conductivity varies with temperature; it is an insulator at low temperatures but a conductor at high temperatures). As a material of industrial significance, boron is usually found in compound form. As such, it is used as a solution in nickel electroplating operations, an ingredient (B2O3) in certain glass compositions, a catalyst in organic chemical reactions, and as a nitride (cubic boron nitride) for cutting tools. In nearly pure form it is used as a fiber in composite materials (Sections 9.4.1 and 15.1.2). 7.6 GUIDE TO PROCESSING CERAMICS The processing of ceramics can be divided into two basic categories: molten ceramics and particulate ceramics. The major category of molten ceramics is glassworking (Chapter 12). Particulate ceramics include traditional and new ceramics; their processing methods constitute most of the rest of the shaping technologies for ceramics (Chapter 17). Cermets, such as cemented carbides, are a special case because they are metal matrix composites (Section 17.3). Table 7.7 provides a guide to the processing of ceramic materials and the elements carbon, silicon, and boron. TABLE 7.7 Guide to the processing of ceramic materials and the elements carbon, silicon, and boron. Material Chapter or Section Material Chapter or Section Glass Glass fibers Particulate ceramics Cermets Chapter 12 Section 12.2.3 Chapter 17 Section 17.3 Synthetic diamonds Silicon Carbon fibers Boron fibers Section 23.2.6 Section 35.2 Section 15.1.2 Section 15.1.2 E1C07 11/02/2009 14:42:49 Page 151 Multiple Choice Quiz 151 REFERENCES [1] Carter, C. B., and Norton, M. G. Ceramic Materials: Science and Engineering. Springer, New York, 2007. [2] Chiang, Y-M., Birnie, III, D. P., and Kingery, W. D. Physical Ceramics. John Wiley & Sons, Inc., New York, 1997. [3] Engineered Materials Handbook, Vol. 4, Ceramics and Glasses. ASM International, Materials Park, Ohio, 1991. [4] Flinn, R. A., and Trojan, P. K. Engineering Materials and Their Applications, 5th ed. John Wiley & Sons, Inc., New York, 1995. [5] Hlavac, J. The Technology of Glass and Ceramics. Elsevier Scientific Publishing Company, New York, 1983. [6] Kingery, W. D., Bowen, H. K., and Uhlmann, D. R. Introduction to Ceramics, 2nd ed. John Wiley & Sons, Inc., New York, 1995. [7] Kirchner, H. P. Strengthening of Ceramics. Marcel Dekker, Inc., New York, 1979. [8] Richerson, D. W. Ceramics—Applications in Manufacturing. Society of Manufacturing Engineers, Dearborn, Michigan, 1989. [9] Richerson, D. W. Modern Ceramic Engineering: Properties, Processing, and Use in Design, 3rd ed. CRC Taylor & Francis, Boca Raton, Florida, 2006. [10] Scholes, S. R., and Greene, C. H. Modern Glass Practice, 7th ed. CBI Publishing Company, Boston, 1993. [11] Schwarzkopf, P., and Kieffer, R. Cemented Carbides. The Macmillan Company, New York, 1960. [12] Singer, F., and Singer, S. S. Industrial Ceramics. Chemical Publishing Company, New York, 1963. [13] Somiya, S. (ed.). Advanced Technical Ceramics. Academic Press, San Diego, California,1989. REVIEW QUESTIONS 7.1. What is a ceramic? 7.2. What are the four most common elements in the Earth’s crust? 7.3. What is the difference between the traditional ceramics and the new ceramics? 7.4. What is the feature that distinguishes glass from the traditional and new ceramics? 7.5. What are the general mechanical properties of ceramic materials? 7.6. What are the general physical properties of ceramic materials? 7.7. What type of atomic bonding characterizes the ceramics? 7.8. What do bauxite and corundum have in common? 7.9. What is clay, as used in making ceramic products? 7.10. What is glazing, as applied to ceramics? 7.11. What does the term refractory mean? 7.12. What are some of the principal applications of cemented carbides, such as WC–Co? 7.13. What is one of the important applications of titanium nitride, as mentioned in the text? 7.14. What are the elements in the ceramic material Sialon? 7.15. Define glass. 7.16. What is the primary mineral in glass products? 7.17. What are some of the functions of the ingredients that are added to glass in addition to silica? Name at least three. 7.18. What does the term devitrification mean? 7.19. What is graphite? MULTIPLE CHOICE QUIZ There are 17 correct answers in the following multiple choice questions (some questions have multiple answers that are correct). To attain a perfect score on the quiz, all correct answers must be given. Each correct answer is worth 1 point. Each omitted answer or wrong answer reduces the score by 1 point, and each additional answer beyond the correct number of answers reduces the score by 1 point. Percentage score on the quiz is based on the total number of correct answers. 7.1. Which one of the following is the most common element in the Earth’s crust: (a) aluminum, (b) calcium, (c) iron, (d) oxygen, or (e) silicon? 7.2. Glass products are based primarily on which one of the following minerals: (a) alumina, (b) corundum, (c) feldspar, (d) kaolinite, or (e) silica? 7.3. Which of the following contains significant amounts of aluminum oxide (three correct answers): (a) alumina, (b) bauxite, (c) corundum, (d) feldspar, (e) kaolinite, (f) quartz, (g) sandstone, and (h) silica? 7.4. Which of the following ceramics are commonly used as abrasives in grinding wheels (two best answers): E1C07 11/02/2009 152 7.5. 7.6. 7.7. 7.8. 14:42:49 Page 152 Chapter 7/Ceramics (a) aluminum oxide, (b) calcium oxide, (c) carbon monoxide, (d) silicon carbide, and (e) silicon dioxide? Which one of the following is generally the most porous of the clay-based pottery ware: (a) china, (b) earthenware, (c) porcelain, or (d) stoneware? Which one of the following is fired at the highest temperatures: (a) china, (b) earthenware, (c) porcelain, or (d) stoneware? Which one of the following comes closest to expressing the chemical composition of clay: (a) Al2O3, (b) Al2(Si2O5)(OH)4, (c) 3AL2O3–2SiO2, (d) MgO, or (e) SiO2? Glass ceramics are polycrystalline ceramic structures that have been transformed into the glassy state: (a) true or (b) false? 7.9. Which one of the following materials is closest to diamond in hardness: (a) aluminum oxide, (b) carbon dioxide, (c) cubic boron nitride, (d) silicon dioxide, or (e) tungsten carbide? 7.10. Which of the following best characterizes the structure of glass-ceramics: (a) 95% polycrystalline, (b) 95% vitreous, or (c) 50% polycrystalline? 7.11. Properties and characteristics of the glass-ceramics include which of the following (two best answers): (a) efficiency in processing, (b) electrical conductor, (c) high-thermal expansion, and (d) strong, relative to other glasses? 7.12. Diamond is the hardest material known: (a) true or (b) false? 7.13. Synthetic diamonds date to (a) ancient times, (b) 1800s, (c) 1950s, or (d) 1980. E1C08 11/10/2009 13:19:37 8 Page 153 POLYMERS Chapter Contents 8.1 Fundamentals of Polymer Science and Technology 8.1.1 Polymerization 8.1.2 Polymer Structures and Copolymers 8.1.3 Crystallinity 8.1.4 Thermal Behavior of Polymers 8.1.5 Additives 8.2 Thermoplastic Polymers 8.2.1 Properties of Thermoplastic Polymers 8.2.2 Important Commercial Thermoplastics 8.3 Thermosetting Polymers 8.3.1 General Properties and Characteristics 8.3.2 Important Thermosetting Polymers 8.4 Elastomers 8.4.1 Characteristics of Elastomers 8.4.2 Natural Rubber 8.4.3 Synthetic Rubbers 8.5 Polymer Recycling and Biodegradability 8.5.1 Polymer Recycling 8.5.2 Biodegradable Polymers 8.6 Guide to the Processing of Polymers Of the three basic types of materials, polymers are the newest and at the same time the oldest known to man. Polymers form the living organisms and vital processes of all life on Earth. To ancient man, biological polymers were the source of food, shelter, and many of his implements. However, our interest in this chapter is in polymers other than biological. With the exception of natural rubber, nearly all of the polymeric materials used in engineering today are synthetic. The materials themselves are made by chemical processing, and most of the products are made by solidification processes. A polymer is a compound consisting of long-chain molecules, each molecule made up of repeating units connected together. There may be thousands, even millions of units in a single polymer molecule. The word is derived from the Greek words poly, meaning many, and meros (reduced to mer), meaning part. Most polymers are based on carbon and are therefore considered organic chemicals. Polymers can be separated into plastics and rubbers. As engineering materials, they are relatively new compared to metals and ceramics, dating only from around the mid-1800s (Historical Note 8.1). For our purposes in covering polymers as a technical subject, it is appropriate to divide them into the following three categories, where (1) and (2) are plastics and (3) is the rubber category: 1. Thermoplastic polymers, also called thermoplastics (TP), are solid materials at room temperature, but they become viscous liquids when heated to temperatures of only a few hundred degrees. This characteristic allows them to be easily and economically shaped into products. They can be subjected to this heating and cooling cycle repeatedly without significant degradation of the polymer. 2. Thermosetting polymers, or thermosets (TS), cannot tolerate repeated heating cycles as thermoplastics can. When initially heated, they soften and flow for molding, but the elevated temperatures also produce a chemical reaction 153 E1C08 11/10/2009 154 13:19:37 Page 154 Chapter 8/Polymers Historical Note 8.1 C History of polymers ertainly one of the milestones in the history of polymers was Charles Goodyear’s discovery of vulcanization of rubber in 1839 (Historical Note 8.2). In 1851, his brother Nelson patented hard rubber, called ebonite, which in reality is a thermosetting polymer. It was used for many years for combs, battery cases, and dental prostheses. At the 1862 International Exhibition in London, an English chemist Alexander Parkes demonstrated the possibilities of the first thermoplastic, a form of cellulose nitrate (cellulose is a natural polymer in wood and cotton). He called it Parkesine and described it as a replacement for ivory and tortoiseshell. The material became commercially important due to the efforts of American John W. Hyatt, Jr., who combined cellulose nitrate and camphor (which acts as a plasticizer) together with heat and pressure to form the product he called Celluloid. His patent was issued in 1870. Celluloid plastic was transparent, and the applications subsequently developed for it included photographic and motion picture film and windshields for carriages and early motorcars. Several additional products based on cellulose were developed around the turn of the last century. Cellulose fibers, called Rayon, were first produced around 1890. Packaging film, called Cellophane, was first marketed around 1910. Cellulose acetate was adopted as the base for photographic film around the same time. This material was to become an important thermoplastic for injection molding during the next several decades. The first synthetic plastic was developed in the early 1900s by the Belgian-born American chemist L. H. Baekeland. It involved the reaction and polymerization of phenol and formaldehyde to form what its inventor called Bakelite. This thermosetting resin is still commercially important today. It was followed by other similar polymers: urea-formaldehyde in 1918 and melamineformaldehyde in 1939. The late 1920s and 1930s saw the development of a number of thermoplastics of major importance today. A Russian I. Ostromislensky had patented polyvinylchloride in 1912, but it was first commercialized in 1927 as a wall covering. Around the same time, polystyrene was first produced in Germany. In England, fundamental research was started in 1932 that led to the synthesis of polyethylene; the first production plant came on line just before the outbreak of World War II. This was low density polyethylene. Finally, a major research program initiated in 1928 under the direction of W. Carothers at DuPont in the United States led to the synthesis of the polyamide nylon; it was commercialized in the late 1930s. Its initial use was in ladies’ hosiery; subsequent applications during the war included low-friction bearings and wire insulation. Similar efforts in Germany provided an alternative form of nylon in 1939. Several important special-purpose polymers were developed in the 1940s: fluorocarbons (Teflon), silicones, and polyurethanes in 1943; epoxy resins in 1947, and acrylonitrile-butadiene-styrene copolymer (ABS) in 1948. During the 1950s: polyester fibers in 1950; and polypropylene, polycarbonate, and highdensity polyethylene in 1957. Thermoplastic elastomers were first developed in the 1960s. The ensuing years have witnessed a tremendous growth in the use of plastics. that hardens the material into an infusible solid. If reheated, thermosetting polymers degrade and char rather than soften. 3. Elastomers are the rubbers. Elastomers (E) are polymers that exhibit extreme elastic extensibility when subjected to relatively low mechanical stress. Some elastomers can be stretched by a factor of 10 and yet completely recover to their original shape. Although their properties are quite different from thermosets, they have a similar molecular structure that is different from the thermoplastics. Thermoplastics are commercially the most important of the three types, constituting around 70% of the tonnage of all synthetic polymers produced. Thermosets and elastomers share the remaining 30% about evenly, with a slight edge for the former. Common TP polymers include polyethylene, polyvinylchloride, polypropylene, polystyrene, and nylon. Examples of TS polymers are phenolics, epoxies, and certain polyesters. The most common example given for elastomers is natural (vulcanized) rubber; however, synthetic rubbers exceed the tonnage of natural rubber. E1C08 11/10/2009 13:19:37 Page 155 Section 8.1/Fundamentals of Polymer Science and Technology 155 Although the classification of polymers into the TP, TS, and E categories suits our purposes for organizing the topic in this chapter, we should note that the three types sometimes overlap. Certain polymers that are normally thermoplastic can be made into thermosets. Some polymers can be either thermosets or elastomers (we indicated that their molecular structures are similar). And some elastomers are thermoplastic. However, these are exceptions to the general classification scheme. The growth in applications of synthetic polymers is truly impressive. On a volumetric basis, current annual usage of polymers exceeds that of metals. There are several reasons for the commercial and technological importance of polymers: å Plastics can be formed by molding into intricate part geometries, usually with no further processing required. They are very compatible with net shape processing. å Plastics possess an attractive list of properties for many engineering applications where strength is not a factor: (1) low density relative to metals and ceramics; (2) good strength-to-weight ratios for certain (but not all) polymers; (3) high corrosion resistance; and (4) low electrical and thermal conductivity. å On a volumetric basis, polymers are cost-competitive with metals. å On a volumetric basis, polymers generally require less energy to produce than metals. This is generally true because the temperatures for working these materials are much lower than for metals. å Certain plastics are translucent and/or transparent, which makes them competitive with glass in some applications. å Polymers are widely used in composite materials (Chapter 9). On the negative side, polymers in general have the following limitations: (1) strength is low relative to metals and ceramics; (2) modulus of elasticity or stiffness is also low—in the case of elastomers, of course, this may be a desirable characteristic; (3) service temperatures are limited to only a few hundred degrees because of the softening of thermoplastic polymers or degradation of thermosetting polymers and elastomers; (4) some polymers degrade when subjected to sunlight and other forms of radiation; and (5) plastics exhibit viscoelastic properties (Section 3.5), which can be a distinct limitation in load bearing applications. In this chapter we examine the technology of polymeric materials. The first section is devoted to an introductory discussion of polymer science and technology. Subsequent sections survey the three basic categories of polymers: thermoplastics, thermosets, and elastomers. 8.1 FUNDAMENTALS OF POLYMER SCIENCE AND TECHNOLOGY Polymers are synthesized by joining many small molecules together to form very large molecules, called macromolecules, that possess a chain-like structure. The small units, called monomers, are generally simple unsaturated organic molecules such as ethylene (C2H4). The atoms in these molecules are held together by covalent bonds; and when joined to form the polymer, the same covalent bonding holds the links of the chain together. Thus, each large molecule is characterized by strong primary bonding. Synthesis of the polyethylene molecule is depicted in Figure 8.1. As we have described its structure here, polyethylene is a linear polymer; its mers form one long chain. A mass of polymer material consists of many macromolecules; the analogy of a bowl of just-cooked spaghetti (without sauce) is sometimes used to visualize the relationship of the individual molecules to the bulk material. Entanglement among the long strands helps E1C08 11/10/2009 156 13:19:37 Page 156 Chapter 8/Polymers FIGURE 8.1 Synthesis of polyethylene from ethylene monomers: (1) n ethylene monomers yields (2a) polyethylene of chain length n; (2b) concise notation for depicting the polymer structure of chain length n. n H H H H H H H H C C C C C C C C H H H H H H H H (1) n H H C C H H n (2b) (2a) to hold the mass together, but atomic bonding is more significant. The bonding between macromolecules in the mass is due to van der Waals and other secondary bonding types. Thus, the aggregate polymer material is held together by forces that are substantially weaker than the primary bonds holding the molecules together. This explains why plastics in general are not nearly as stiff and strong as metals or ceramics. When a thermoplastic polymer is heated, it softens. The heat energy causes the macromolecules to become thermally agitated, exciting them to move relative to each other within the polymer mass (here, the wet spaghetti analogy loses its appeal). The material begins to behave like a viscous liquid, viscosity decreasing (fluidity increasing) with rising temperature. Let us expand on these opening remarks, tracing how polymers are synthesized and examining the characteristics of the materials that result from the synthesis. 8.1.1 POLYMERIZATION As a chemical process, the synthesis of polymers can occur by either of two methods: (1) addition polymerization and (2) step polymerization. Production of a given polymer is generally associated with one method or the other. Addition Polymerization In this process, exemplified by polyethylene, the double bonds between carbon atoms in the ethylene monomers are induced to open so that they join with other monomer molecules. The connections occur on both ends of the expanding macromolecule, developing long chains of repeating mers. Because of the way the molecules are formed, the process is also known as chain polymerization. It is initiated using a chemical catalyst (called an initiator) to open the carbon double bond in some of the monomers. These monomers, which are now highly reactive because of their unpaired electrons, then capture other monomers to begin forming chains that are reactive. The chains propagate by capturing still other monomers, one at a time, until large molecules have been produced and the reaction is terminated. The process proceeds as indicated in Figure 8.2. The entire polymerization reaction takes only seconds for any given macromolecule. However, in the industrial process, it may take many minutes or even hours to complete the polymerization of a given batch, since all of the chain reactions do not occur simultaneously in the mixture. FIGURE 8.2 Model of addition (chain) polymerization: (1) initiation, (2) rapid addition of monomers, and (3) resulting longchain polymer molecule with n mers at termination of reaction. Monomers Mers Initiation (1) (2) (3) E1C08 11/10/2009 13:19:37 Page 157 Section 8.1/Fundamentals of Polymer Science and Technology Polymer Monomer Polypropylene Polyvinyl chloride Polystyrene Repeating mer H H H H C C C C H CH3 H CH3 H H H H C C C C H Cl H Cl H H H H C C C C H C6H5 Polytetrafluoroethylene (Teflon) FIGURE 8.3 Some typical polymers formed by addition (chain) polymerization. Polyisoprene (natural rubber) F F F F C C C C F F F F H H C C H H C6H5 H H H C C C CH3 H H C 157 Chemical formula (C3H6)n n (C2H3Cl)n n (C8H8)n n (C2F4)n n H C (C5H8)n C CH3 H n Other polymers typically formed by addition polymerization are presented in Figure 8.3, along with the starting monomer and the repeating mer. Note that the chemical formula for the monomer is the same as that of the mer in the polymer. This is a characteristic of this method of polymerization. Note also that many of the common polymers involve substitution of some alternative atom or molecule in place of one of the H atoms in polyethylene. Polypropylene, polyvinylchloride, and polystyrene are examples of this substitution. Polytetrafluoroethylene replaces all four H atoms in the structure with atoms of fluorine (F). Most addition polymers are thermoplastics. The exception in Figure 8.3 is polyisoprene, the polymer of natural rubber. Although formed by addition polymerization, it is an elastomer. Step Polymerization In this form of polymerization, two reacting monomers are brought together to form a new molecule of the desired compound. In most (but not all) step polymerization processes, a byproduct of the reaction is also produced. The byproduct is typically water, which condenses; hence, the term condensation polymerization is often used for processes that yield the condensate. As the reaction continues, more molecules of the reactants combine with the molecules first synthesized to form polymers of length n ¼ 2, then polymers of length n ¼ 3, and so on. Polymers of increasing n are created in a slow, stepwise fashion. In addition to this gradual elongation of the molecules, intermediate polymers of length n1 and n2 also combine to form molecules of length n ¼ n1 + n2, so that two types of reactions are proceeding simultaneously once the process is under way, as illustrated in Figure 8.4. Accordingly, at any point in the process, the batch contains polymers of various lengths. Only after sufficient time has elapsed are molecules of adequate length formed. E1C08 11/10/2009 158 13:19:37 Page 158 Chapter 8/Polymers (n1 + n2)-mer n1-mer Monomer n-mer (n + 1)-mer n2-mer (1) (2) (1) (a) (2) (b) FIGURE 8.4 Model of step polymerization showing the two types of reactions occurring: (a) n-mer attaching a single monomer to form a (n + 1) -mer; and (b) n1-mer combining with n2-mer to form a (n1 + n2) -mer. Sequence is shown by (1) and (2). It should be noted that water is not always the byproduct of the reaction; for example, ammonia (NH3) is another simple compound produced in some reactions. Nevertheless, the term condensation polymerization is still used. It should also be noted that although most step polymerization processes involve condensation of a byproduct, some do not. Examples of commercial polymers produced by step (condensation) polymerization are given in Figure 8.5. Both thermoplastic and thermosetting polymers are synthesized by this method; nylon-6,6 and polycarbonate are TP polymers, while phenol formaldehyde and urea formaldehyde are TS polymers. Degree of Polymerization and Molecular Weight A macromolecule produced by polymerization consists of n repeating mers. Since molecules in a given batch of polymerized Repeating unit Polymer Nylon-6, 6 H H O H O H C N C C C N H6 H4 Chemical formula Condensate [(CH2)6 (CONH)2 (CH2)4]n H2O (C3H6 (C6H4)2CO3)n HCl [(C6H4)CH2OH]n H2O n CH3 Polycarbonate [ (C6H4) C (C6H4) O C O ]n O CH3 H Phenol formaldehyde [ C6H4 C ]n H OH NH Urea formaldehyde [ C NH H O C ]n (CO(NH)2 CH2)n H2O H FIGURE 8.5 Some typical polymers formed by step (condensation) polymerization (simplified expression of structure and formula; ends of polymer chain are not shown). E1C08 11/10/2009 13:19:37 Page 159 Section 8.1/Fundamentals of Polymer Science and Technology 159 TABLE 8.1 Typical values of degree of polymerization and molecular weight for selected thermoplastic polymers. Polymer Degree of Polymerization (n) Molecular Weight Polyethylene Polystyrene Polyvinylchloride Nylon Polycarbonate 10,000 3,000 1,500 120 200 300,000 300,000 100,000 15,000 40,000 Compiled from [7]. material vary in length, n for the batch is an average; its statistical distribution is normal. The mean value of n is called the degree of polymerization (DP) for the batch. The degree of polymerization affects the properties of the polymer: higher DP increases mechanical strength but also increases viscosity in the fluid state, which makes processing more difficult. The molecular weight (MW) of a polymer is the sum of the molecular weights of the mers in the molecule; it is n times the molecular weight of each repeating unit. Since n varies for different molecules in a batch, the molecule weight must be interpreted as an average. Typical values of DP and MW for selected polymers are presented in Table 8.1. 8.1.2 POLYMER STRUCTURES AND COPOLYMERS There are structural differences among polymer molecules, even molecules of the same polymer. In this section we examine three aspects of molecular structure: (1) stereoregularity, (2) branching and cross-linking, and (3) copolymers. Stereoregularity Stereoregularity is concerned with the spatial arrangement of the atoms and groups of atoms in the repeating units of the polymer molecule. An important aspect of stereoregularity is the way the atom groups are located along the chain for a polymer that has one of the H atoms in its mers replaced by some other atom or atom group. Polypropylene is an example; it is similar to polyethylene except that CH3 is substituted for one of the four H atoms in the mer. Three tactic arrangements are possible, illustrated in Figure 8.6: (a) isotactic, in which the odd atom groups are all on the same side; (b) syndiotactic, in which the atom groups alternate on opposite sides; and (c) atactic, in which the groups are randomly along either side. The tactic structure is important in determining the properties of the polymer. It also influences the tendency of a polymer to crystallize (Section 8.1.3). Continuing with H CH3 H CH3 H CH3 H CH3 H CH3 H H H CH3 H H C C C C C C H H H H C C C C C C C C C C H H H H H H H CH3 H H H CH3 (a) FIGURE 8.6 Possible arrangement of atom groups in polypropylene: (a) isotactic, (b) syndiotactic, and (c) atactic. (b) H H H H H CH3 H H C C C C C C C C H CH3 H CH3 H H H CH3 (c) E1C08 11/10/2009 160 13:19:37 Page 160 Chapter 8/Polymers (a) (b) (c) (d) FIGURE 8.7 Various structures of polymer molecules: (a) linear, characteristic of thermoplastics; (b) branched; (c) loosely cross-linked as in an elastomer; and (d) tightly cross-linked or networked structure as in a thermoset. our polypropylene example, this polymer can be synthesized in any of the three tactic structures. In its isotactic form, it is strong and melts at 175 C (347 F); the syndiotactic structure is also strong, but melts at 131 C (268 F); but atactic polypropylene is soft and melts at around 75 C (167 F) and has little commercial use [6], [9]. Linear, Branched, and Cross-Linked Polymers We have described the polymerization process as yielding macromolecules of a chain-like structure, called a linear polymer. This is the characteristic structure of a thermoplastic polymer. Other structures are possible, as portrayed in Figure 8.7. One possibility is for side branches to form along the chain, resulting in the branched polymer shown in Figure 8.7(b). In polyethylene, this occurs because hydrogen atoms are replaced by carbon atoms at random points along the chain, initiating the growth of a branch chain at each location. For certain polymers, primary bonding occurs between branches and other molecules at certain connection points to form cross-linked polymers as pictured in Figure 8.7(c) and (d). Cross-linking occurs because a certain proportion of the monomers used to form the polymer are capable of bonding to adjacent monomers on more than two sides, thus allowing branches from other molecules to attach. Lightly cross-linked structures are characteristic of elastomers. When the polymer is highly cross-linked we refer to it as having a network structure, as in (d); in effect, the entire mass is one gigantic macromolecule. Thermosetting plastics take this structure after curing. The presence of branching and cross-linking in polymers has a significant effect on properties. It is the basis of the difference between the three categories of polymers: TP, TS, and E. Thermoplastic polymers always possess linear or branched structures, or a mixture of the two. Branching increases entanglement among the molecules, usually making the polymer stronger in the solid state and more viscous at a given temperature in the plastic or liquid state. Thermosetting plastics and elastomers are cross-linked polymers. Cross-linking causes the polymer to become chemically set; the reaction cannot be reversed. The effect E1C08 11/10/2009 13:19:37 Page 161 Section 8.1/Fundamentals of Polymer Science and Technology (a) FIGURE 8.8 Various structures of copolymers: (a) alternating, (b) random, (c) block, and (d) graft. 161 (b) (c) (d) is to permanently change the structure of the polymer; upon heating, it degrades or burns rather than melts. Thermosets possess a high degree of cross-linking, while elastomers possess a low degree of cross-linking. Thermosets are hard and brittle, while elastomers are elastic and resilient. Copolymers Polyethylene is a homopolymer; so are polypropylene, polystyrene, and many other common plastics; their molecules consist of repeating mers that are all the same type. Copolymers are polymers whose molecules are made of repeating units of two different types. An example is the copolymer synthesized from ethylene and propylene to produce a copolymer with elastomeric properties. The ethylene-propylene copolymer can be represented as follows:   (C2 H4 )n (C3 H6 )m  where n and m range between 10 and 20, and the proportions of the two constituents are around 50% each. We find in Section 8.4.3 that the combination of polyethylene and polypropylene with small amounts of diene is an important synthetic rubber. Copolymers can possess different arrangements of their constituent mers. The possibilities are shown in Figure 8.8: (a) alternating copolymer, in which the mers repeat every other place; (b) random, in which the mers are in random order, the frequency depending on the relative proportions of the starting monomers; (c) block, in which mers of the same type tend to group themselves into long segments along the chain; and (d) graft, in which mers of one type are attached as branches to a main backbone of mers of the other type. The ethylene–propylene diene rubber, mentioned previously, is a block type. Synthesis of copolymers is analogous to alloying of metals to form solid solutions. As with metallic alloys, differences in the ingredients and structure of copolymers can have a substantial effect on properties. An example is the polyethylene–polypropylene mixture we have been discussing. Each of these polymers alone is fairly stiff; yet a 50–50 mixture forms a copolymer of random structure that is rubbery. It is also possible to synthesize ternary polymers, or terpolymers, which consist of mers of three different types. An example is the plastic ABS (acrylonitrile–butadiene– styrene—no wonder they call it ABS). 8.1.3 CRYSTALLINITY Both amorphous and crystalline structures are possible with polymers, although the tendency to crystallize is much less than for metals or nonglass ceramics. Not all polymers can form crystals. For those that can, the degree of crystallinity (the proportion of E1C08 11/10/2009 162 13:19:38 Page 162 Chapter 8/Polymers TABLE 8.2 Comparison of low-density polyethylene and high-density polyethylene. Polyethylene Type Degree of crystallinity Specific gravity Modulus of elasticity Melting temperature Low Density High Density 55% 0.92 140 MPa (20,305 lb/in2) 115 C (239 F) 92% 0.96 700 MPa (101,530 lb/in2) 135 C (275 F) Compiled from [6]. Values given are typical. crystallized material in the mass) is always less than 100%. As crystallinity is increased in a polymer, so are (1) density, (2) stiffness, strength, and toughness, and (3) heat resistance. In addition, (4) if the polymer is transparent in the amorphous state, it becomes opaque when partially crystallized. Many polymers are transparent, but only in the amorphous (glassy) state. Some of these effects can be illustrated by the differences between low-density and high-density polyethylene, presented in Table 8.2. The underlying reason for the property differences between these materials is the degree of crystallinity. Linear polymers consist of long molecules with thousands of repeated mers. Crystallization in these polymers involves the folding back and forth of the long chains upon themselves to achieve a very regular arrangement of the mers, as pictured in Figure 8.9(a). The crystallized regions are called crystallites. Owing to the tremendous length of a single molecule (on an atomic scale), it may participate in more than one crystallite. Also, more than one molecule may be combined in a single crystal region. The crystallites take the form of lamellae, as pictured in Figure 8.9(b), that are randomly mixed in with the amorphous material. Thus, a polymer that crystallizes is a two-phase system—crystallites interspersed throughout an amorphous matrix. A number of factors determine the capacity and/or tendency of a polymer to form crystalline regions within the material. The factors can be summarized as follows: (1) as a general rule, only linear polymers can form crystals; (2) stereoregularity of the molecule is critical [15]: isotactic polymers always form crystals; syndiotactic polymers sometimes form FIGURE 8.9 Crystallized regions in a polymer: (a) long molecules forming crystals randomly mixed in with the amorphous material; and (b) folded chain lamella, the typical form of a crystallized region. E1C08 11/10/2009 13:19:38 Page 163 Section 8.1/Fundamentals of Polymer Science and Technology 163 crystals; atactic polymers never form crystals; (3) copolymers, due to their molecular irregularity, rarely form crystals; (4) slower cooling promotes crystal formation and growth, as it does in metals and ceramics; (5) mechanical deformation, as in the stretching of a heated thermoplastic, tends to align the structure and increase crystallization; and (6) plasticizers (chemicals added to a polymer to soften it) reduce the degree of crystallinity. 8.1.4 THERMAL BEHAVIOR OF POLYMERS The thermal behavior of polymers with crystalline structures is different from that of amorphous polymers (Section 2.4). The effect of structure can be observed on a plot of specific volume (reciprocal of density) as a function of temperature, as plotted in Figure 8.10. A highly crystalline polymer has a melting point Tm at which its volume undergoes an abrupt change. Also, at temperatures above Tm, the thermal expansion of the molten material is greater than for the solid material below Tm. An amorphous polymer does not undergo the same abrupt changes at Tm. As it is cooled from the liquid, its coefficient of thermal expansion continues to decline along the same trajectory as when it was molten, and it becomes increasingly viscous with decreasing temperature. During cooling below Tm, the polymer changes from liquid to rubbery. As temperature continues to drop, a point is finally reached at which the thermal expansion of the amorphous polymer suddenly becomes lower. This is the glass-transition temperature, Tg (Section 3.5), seen as the change in slope. Below Tg, the material is hard and brittle. A partially crystallized polymer lies between these two extremes, as indicated in Figure 8.10. It is an average of the amorphous and crystalline states, the average depending on the degree of crystallinity. Above Tm it exhibits the viscous characteristics of a liquid; between Tm and Tg it has viscoelastic properties; and below Tg it has the conventional elastic properties of a solid. What we have described in this section applies to thermoplastic materials, which can move up and down the curve of Figure 8.10 multiple times. The manner in which they are heated and cooled may change the path that is followed. For example, fast cooling rates may inhibit crystal formation and increase the glass-transition temperature. Thermosets and elastomers cooled from the liquid state behave like an amorphous polymer until cross-linking occurs. Their molecular structure restricts the formation of crystals. And once their molecules are cross-linked, they cannot be reheated to the molten state. FIGURE 8.10 Behavior of polymers as a function of temperature. E1C08 11/10/2009 164 13:19:38 Page 164 Chapter 8/Polymers 8.1.5 ADDITIVES The properties of a polymer can often be beneficially changed by combining them with additives. Additives either alter the molecular structure of the polymer or add a second phase to the plastic, in effect transforming a polymer into a composite material. Additives can be classified by function as (1) fillers, (2) plasticizers, (3) colorants, (4) lubricants, (5) flame retardants, (6) cross-linking agents, (7) ultraviolet light absorbers, and (8) antioxidants. Filler Fillers are solid materials added to a polymer usually in particulate or fibrous form to alter its mechanical properties or to simply reduce material cost. Other reasons for using fillers are to improve dimensional and thermal stability. Examples of fillers used in polymers include cellulosic fibers and powders (e.g., cotton fibers and wood flour, respectively); powders of silica (SiO2), calcium carbonate (CaCO3), and clay (hydrous aluminum silicate); and fibers of glass, metal, carbon, or other polymers. Fillers that improve mechanical properties are called reinforcing agents, and composites thus created are referred to as reinforced plastics; they have higher stiffness, strength, hardness, and toughness than the original polymer. Fibers provide the greatest strengthening effect. Plasticizers Plasticizers are chemicals added to a polymer to make it softer and more flexible, and to improve its flow characteristics during forming. The plasticizer works by reducing the glass transition temperature to below room temperature. Whereas the polymer is hard and brittle below Tg, it is soft and tough above it. Addition of a plasticizer1 to polyvinylchloride (PVC) is a good example; depending on the proportion of plasticizer in the mix, PVC can be obtained in a range of properties, from rigid and brittle to flexible and rubbery. Colorants An advantage of many polymers over metals or ceramics is that the material itself can be obtained in most any color. This eliminates the need for secondary coating operations. Colorants for polymers are of two types: pigments and dies. Pigments are finely powdered materials that are insoluble in and must be uniformly distributed throughout the polymer in very low concentrations, usually less than 1%. They often add opacity as well as color to the plastic. Dies are chemicals, usually supplied in liquid form, that are generally soluble in the polymer. They are normally used to color transparent plastics such as styrene and acrylics. Other Additives Lubricants are sometimes added to the polymer to reduce friction and promote flow at the mold interface. Lubricants are also helpful in releasing the part from the mold in injection molding. Mold-release agents, sprayed onto the mold surface, are often used for the same purpose. Nearly all polymers burn if the required heat and oxygen are supplied. Some polymers are more combustible than others. Flame retardants are chemicals added to polymers to reduce flammability by any or a combination of the following mechanisms: (1) interfering with flame propagation, (2) producing large amounts of incombustible gases, and/or (3) increasing the combustion temperature of the material. The chemicals may also function to (4) reduce the emission of noxious or toxic gases generated during combustion. We should include among the additives those that cause cross-linking to occur in thermosetting polymers and elastomers. The term cross-linking agent refers to a variety of ingredients that cause a cross-linking reaction or act as a catalyst to promote such a reaction. Important commercial examples are (1) sulfur in vulcanization of natural rubber, (2) formaldehyde for phenolics to form phenolic thermosetting plastics, and (3) peroxides for polyesters. 1 The common plasticizer in PVC is dioctyl phthalate, a phthalate ester. E1C08 11/10/2009 13:19:38 Page 165 Section 8.2/Thermoplastic Polymers 165 Many polymers are susceptible to degradation by ultraviolet light (e.g., from sunlight) and oxidation. The degradation manifests itself as the breaking of links in the long chain molecules. Polyethylene, for example, is vulnerable to both types of degradation, which lead to a loss of mechanical strength. Ultraviolet light absorbers and antioxidants are additives that reduce the susceptibility of the polymer to these forms of attack. 8.2 THERMOPLASTIC POLYMERS In this section, we discuss the properties of the thermoplastic polymer group and then survey its important members. 8.2.1 PROPERTIES OF THERMOPLASTIC POLYMERS The defining property of a thermoplastic polymer is that it can be heated from a solid state to a viscous liquid state and then cooled back down to solid, and that this heating and cooling cycle can be applied multiple times without degrading the polymer. The reason for this property is that TP polymers consist of linear (and/or branched) macromolecules that do not cross-link when heated. By contrast, thermosets and elastomers undergo a chemical change when heated, which cross-links their molecules and permanently sets these polymers. In truth, thermoplastics do deteriorate chemically with repeated heating and cooling. In plastic molding, a distinction is made between new or virgin material, and plastic that has been previously molded (e.g., sprues, defective parts) and therefore has experienced thermal cycling. For some applications, only virgin material is acceptable. Thermoplastic polymers also degrade gradually when subjected to continuous elevated temperatures below Tm. This long-term effect is called thermal aging and involves slow chemical deterioration. Some TP polymers are more susceptible to thermal aging than others, and for a given material the rate of deterioration depends on temperature. Mechanical Properties In our discussion of mechanical properties in Chapter 3, we compared polymers to metals and ceramics. The typical thermoplastic at room temperature is characterized by the following: (1) much lower stiffness, the modulus of elasticity being two (in some cases, three) orders of magnitude lower than metals and ceramics; (2) lower tensile strength, about 10% of the metals; (3) much lower hardness; and (4) greater ductility on average, but there is a tremendous range of values, from 1% elongation for polystyrene to 500% or more for polypropylene. Mechanical properties of thermoplastics depend on temperature. The functional relationships must be discussed in the context of amorphous and crystalline structures. Amorphous thermoplastics are rigid and glass-like below their glass transition temperature Tg and flexible or rubber-like just above it. As temperature increases above Tg, the polymer becomes increasingly soft, finally becoming a viscous fluid (it never becomes a thin liquid due to its high molecular weight). The effect on mechanical behavior can be portrayed as in Figure 8.11, in which mechanical behavior is defined as deformation resistance. This is analogous to modulus of elasticity but it allows us to observe the effect of temperature on the amorphous polymer as it transitions from solid to liquid. Below Tg, the material is elastic and strong. At Tg, a rather sudden drop in deformation resistance is observed as the material transforms into its rubbery phase; its behavior is viscoelastic in this region. As temperature increases, it gradually becomes more fluid-like. A theoretical thermoplastic with 100% crystallinity would have a distinct melting point Tm at which it transforms from solid to liquid, but would show no perceptible Tg point. Of course, real polymers have less than 100% crystallinity. For partially crystallized polymers, the resistance to deformation is characterized by the curve that lies between E1C08 11/10/2009 166 13:19:38 Page 166 Chapter 8/Polymers FIGURE 8.11 Relationship of mechanical properties, portrayed as deformation resistance, as a function of temperature for an amorphous thermoplastic, a 100% crystalline (theoretical) thermoplastic, and a partially crystallized thermoplastic. the two extremes, its position determined by the relative proportions of the two phases. The partially crystallized polymer exhibits features of both amorphous and fully crystallized plastics. Below Tg, it is elastic with deformation resistance sloping downward with rising temperatures. Above Tg, the amorphous portions of the polymer soften, while the crystalline portions remain intact. The bulk material exhibits properties that are generally viscoelastic. As Tm is reached, the crystals now melt, giving the polymer a liquid consistency; resistance to deformation is now due to the fluid’s viscous properties. The degree to which the polymer assumes liquid characteristics at and above Tm depends on molecular weight and degree of polymerization. Higher DP and MW reduce flow of the polymer, making it more difficult to process by molding and similar shaping methods. This is a dilemma faced by those who select these materials because higher MW and DP mean higher strength. Physical Properties Physical properties of materials are discussed in Chapter 4. In general, thermoplastic polymers have the following characteristics: (1) lower densities than metals or ceramics—typical specific gravities for polymers are around 1.2, for ceramics around 2.5, and for metals around 7.0; (2) much higher coefficient of thermal expansion— roughly 5 times the value for metals and 10 times the value for ceramics; (3) much lower melting temperatures; (4) specific heats that are 2 to 4 times those of metals and ceramics; (5) thermal conductivities that are about three orders of magnitude lower than those of metals; and (6) insulating electrical properties. 8.2.2 IMPORTANT COMMERCIAL THERMOPLASTICS Thermoplastic products include molded and extruded items, fibers, films, sheets, packaging materials, paints, and varnishes. The starting raw materials for these products are normally supplied to the fabricator in the form of powders or pellets in bags, drums, or larger loads by truck or rail car. The most important TP polymers are discussed in alphabetical order in this section. For each plastic, Table 8.3 lists the chemical formula and selected properties. Approximate market share is given relative to all plastics (thermoplastic and thermosetting). Acetals Acetal is the popular name given to polyoxymethylene, an engineering polymer prepared from formaldehyde (CH2O) with high stiffness, strength, toughness, and wear resistance. In addition, it has a high melting point, low moisture absorption, and is insoluble E1C08 11/10/2009 13:19:38 Page 167 Section 8.2/Thermoplastic Polymers TABLE 8.3 167 Important commercial thermoplastic polymers: (a) acetal. Polymer: Symbol: Polymerization method: Degree of crystallinity: Modulus of elasticity: Tensile strength: Polyoxymethylene, also known as polyacetal (OCH2)n POM Elongation: Step (condensation) Specific gravity: 75% typical Glass transition temperature: Melting temperature: 3500 MPa (507,630 lb/in2) 70 MPa (10,150 lb/in2) Approximate market share: 25%–75% 1.42 80 C (112 F) 180 C (356 F) Much less than 1% Table 8.3 is compiled from [2], [4], [6], [7], [9], [16], and other sources. TABLE 8.3 (continued): (b) acrylics (thermoplastic). Representative polymer: Symbol: Polymerization method: Degree of crystallinity: Modulus of elasticity: Tensile strength: Polymethylmethacrylate (C5H8O2)n PMMA Addition None (amorphous) 2800 MPa (406,110 lb/in2) 55 MPa (7975 lb/in2) Elongation: Specific gravity: Glass transition temperature: Melting temperature: Approximate market share: 5 1.2 105 C (221 F) 200 C (392 F) About 1% in common solventsat ambienttemperatures.Because ofthiscombination ofproperties,acetal resins are competitive with certain metals (e.g., brass and zinc) in automotive components such as door handles, pump housings, and similar parts; appliance hardware; and machinery components. Acrylics The acrylics are polymers derived from acrylic acid (C3H4O2) and compounds originating from it. The most important thermoplastic in the acrylics group is polymethylmethacrylate (PMMA) or Plexiglas (Rohm & Haas’s trade name for PMMA). Data on PMMA are listed in Table 8.3(b). It is an amorphous linear polymer. Its outstanding property is excellent transparency, which makes it competitive with glass in optical applications. Examples include automotive tail-light lenses, optical instruments, and aircraft windows. Its limitation when compared with glass is a much lower scratch resistance. Other uses of PMMA include floor waxes and emulsion latex paints. Another important use of acrylics is in fibers for textiles; polyacrylonitrile (PAN) is an example that goes by the more familiar trade names Orlon (DuPont) and Acrilan (Monsanto). Acrylonitrile–Butadiene–Styrene ABS is called an engineering plastic due to its excellent combination of mechanical properties, some of which are listed in Table 8.3(c). ABS is a twophase terpolymer, one phase being the hard copolymer styrene–acrylonitrile, while the other phase is styrene-butadiene copolymer that is rubbery. The name of the plastic is derived from the three starting monomers, which may be mixed in various proportions. Typical applications include components for automotive, appliances, business machines; and pipes and fittings. Cellulosics Cellulose (C6H10O5) is a carbohydrate polymer commonly occurring in nature. Wood and cotton fibers, the chief industrial sources of cellulose, contain about 50% and 95% TABLE 8.3 (continued): (c) acrylonitrile–butadiene–styrene. Polymer: Symbol: Polymerization method: Degree of crystallinity: Modulus of elasticity: Terpolymer of acrylonitrile (C3H3N), butadiene (C4H6), and styrene (C8H8) ABS Tensile strength: 50 MPa (7250 lb/in2) Addition Elongation: 10%–30% None (amorphous) Specific gravity: 1.06 2100 MPa (304,580 lb/in2) Approximate market share: About 3% E1C08 11/10/2009 168 13:19:38 Page 168 Chapter 8/Polymers TABLE 8.3 (continued): (d) cellulosics. Representative polymer: Symbol: Polymerization method: Degree of crystallinity: Modulus of elasticity: Tensile strength: Cellulose acetate (C6H9O5–COCH3)n CA Step (condensation) Amorphous 2800 MPa (406,110 lb/in2) 30 MPa (4350 lb/in2) Elongation: Specific gravity: Glass transition temperature: Melting temperature: Approximate market share: 10%–50% 1.3 105 C (221 F) 306 C (583 F) Less than 1% of the polymer, respectively. When cellulose is dissolved and reprecipitated during chemical processing, the resulting polymer is called regenerated cellulose. When this is produced as a fiber for apparel it is known as rayon (of course, cotton itself is a widely used fiber for apparel). When it is produced as a thin film, it is cellophane, a common packaging material. Cellulose itself cannot be used as a thermoplastic because it decomposes before melting when its temperature is increased. However, it can be combined with various compounds to form several plastics of commercial importance; examples are cellulose acetate (CA) and cellulose acetate–butyrate (CAB). CA, data for which are given in Table 8.3(d), is produced in the form of sheets (for wrapping), film (for photography), and molded parts. CAB is a better molding material than CA and has greater impact strength, lower moisture absorption, and better compatibility with plasticizers. The cellulosic thermoplastics share about 1% of the market. Fluoropolymers Polytetrafluorethylene (PTFE), commonly known as Teflon, accounts for about 85% of the family of polymers called fluoropolymers, in which F atoms replace H atoms in the hydrocarbon chain. PTFE is extremely resistant to chemical and environmental attack, is unaffected by water, good heat resistance, and very low coefficient of friction. These latter two properties have promoted its use in nonstick household cookware. Other applications that rely on the same property include nonlubricating bearings and similar components. PTFE also finds applications in chemical equipment and food processing. Polyamides An important polymer family that forms characteristic amide linkages (CONH) during polymerization is the polyamides (PA). The most important members of the PA familyarenylons, ofwhichthetwoprincipal grades arenylon-6and nylon-6,6(the numbers are codes thatindicatethenumberofcarbonatoms inthemonomer).Thedatagivenin Table8.3(f) are for nylon-6,6, which was developed at DuPont in the 1930s. Properties of nylon-6, developed in Germany are similar. Nylon is strong, highly elastic, tough, abrasion resistant, and self-lubricating. It retains good mechanical properties at temperatures up to about 125 C (257 F). One shortcoming is that it absorbs water with an accompanying degradation in properties. The majority of applications of nylon (about 90%) are in fibers for carpets, apparel, and tire cord. The remainder (10%) are in engineering components; nylon is commonly a good substitute for metals in bearings, gears, and similar parts where strength and low friction are needed. A second group of polyamides is the aramids (aromatic polyamides) of which Kevlar (DuPont trade name) is gaining in importance as a fiber in reinforced plastics. The reason for the interest in Kevlar is that its strength is the same as steel at 20% of the weight. TABLE 8.3 (continued): (e) fluoropolymers. Representative polymer: Symbol: Polymerization method: Degree of crystallinity: Modulus of elasticity: Tensile strength: Polytetrafluorethylene (C2F4)n PTFE Addition About 95% crystalline 425 MPa (61,640 lb/in2) 20 MPa (2900 lb/in2) Elongation: Specific gravity: Glass transition temperature: Melting temperature: Approximate market share: 100%–300% 2.2 127 C (260 F) 327 C (620 F) Less than 1% E1C08 11/10/2009 13:19:39 Page 169 Section 8.2/Thermoplastic Polymers TABLE 8.3 169 (continued): (f) polyamides. Representative polymer: Symbol: Polymerization method: Degree of crystallinity: Modulus of elasticity: Tensile strength: Nylon-6,6 ((CH2)6(CONH) 2(CH2)4)n PA-6,6 Elongation: Step (condensation) Specific gravity: Highly crystalline Glass transition temperature: Melting temperature: 700 MPa (101,500 lb/in2) 70 MPa (10,150 lb/in2) Approximate market share: 300% 1.14 50 C (122 F) 260 C (500 F) 1% for all polyamides Polycarbonate Polycarbonate (PC) is noted for its generally excellent mechanical properties, which include high toughness and good creep resistance. It is one of the best thermoplastics for heat resistance—it can be used to temperatures around 125 C (257 F). In addition, it is transparent and fire resistant. Applications include molded machinery parts, housings for business machines, pump impellers, safety helmets, and compact disks (e.g., audio, video, and computer). It is also widely used in glazing (window and windshield) applications. Polyesters The polyesters form a family of polymers made up of the characteristic ester linkages (CO–O). They can be either thermoplastic or thermosetting, depending on whether cross-linking occurs. Of the thermoplastic polyesters, a representative example is polyethylene terephthalate (PET), data for which are compiled in the table. It can be either amorphous or partially crystallized (up to about 30%), depending on how it is cooled after shaping. Fast cooling favors the amorphous state, which is highly transparent. Significant applications include blow-molded beverage containers, photographic films, and magnetic recording tape. In addition, PET fibers are widely used in apparel. Polyester fibers have low moisture absorption and good deformation recovery, both of which make them ideal for ‘‘wash and wear’’ garments that resist wrinkling. The PET fibers are almost always blended with cotton or wool. Familiar trade names for polyester fibers include Dacron (DuPont), Fortrel (Celanese), and Kodel (Eastman Kodak). Polyethylene Polyethylene (PE) was first synthesized in the 1930s, and today it accounts for the largest volume of all plastics. The features that make PE attractive as an engineering material are low cost, chemical inertness, and easy processing. Polyethylene is available in TABLE 8.3 (continued): (g) polycarbonate. Polymer: Symbol: Polymerization method: Degree of crystallinity: Modulus of elasticity: Tensile strength: TABLE 8.3 Polycarbonate (C3H6(C6H4)2CO3)n PC Step (condensation) Amorphous 2500 MPa (362,590 lb/in2) 65 MPa (9425 lb/in2) Elongation: Specific gravity: Glass transition temperature: Melting temperature: Approximate market share: 110% 1.2 150 C (302 F) 230 C (446 F) Less than 1% (continued): (h) polyesters (thermoplastic). Representative polymer: Symbol: Polymerization method: Degree of crystallinity: Modulus of elasticity: Tensile strength: Polyethylene terephthalate (C2H4–C8H4O4)n PET Elongation: Step (condensation) Specific gravity: Amorphous to 30% crystalline Glass transition temperature: 2300 MPa (333,590 lb/in2) Melting temperature: Approximate market share: 55 MPa (7975 lb/in2) 200% 1.3 70 C (158 F) 265 C (509 F) About 2% E1C08 11/10/2009 170 13:19:39 Page 170 Chapter 8/Polymers TABLE 8.3 (continued): (i) polyethylene. Polyethylene: Symbol: Polymerization method: Degree of crystallinity: Modulus of elasticity: Tensile strength: Elongation: Specific gravity: Glass transition temperature: Melting temperature: Approximate market share: (C2H4)n (low density) LDPE Addition 55% typical 140 MPa (20,305 lb/in2) 15 MPa (2175 lb/in2) 100%–500% 0.92 100 C (148 F) 115 C (239 F) About 20% (C2H4)n (high density) HDPE Addition 92% typical 700 MPa (101,500 lb/in2) 30 MPa (4350 lb/in2) 20%–100% 0.96 115 C (175 F) 135 C (275 F) About 15% several grades, the most common of which are low-density polyethylene (LDPE) and highdensity polyethylene (HDPE). The low-density grade is a highly branched polymer with lower crystallinity and density. Applications include squeezable bottles, frozen food bags, sheets, film, and wire insulation. HDPE has a more linear structure, with higher crystallinity and density. These differences make HDPE stiffer and stronger and give it a higher melting temperature. HDPE is used to produce bottles, pipes, and housewares. Both grades can be processed by most polymer shaping methods (Chapter 13). Properties for the two grades are given in Table 8.3(i). Polypropylene Polypropylene (PP) has become a major plastic, especially for injection molding, since its introduction in the late 1950s. PP can be synthesized in isotactic, syndiotactic, or atactic structures, the first of these being the most important and for which the characteristics are given in the table. It is the lightest of the plastics, and its strength-to-weight ratio is high. PP is frequently compared with HDPE because its cost and many of its properties are similar. However, the high melting point of polypropylene allows certain applications that preclude use of polyethylene—for example, components that must be sterilized. Other applications are injection molded parts for automotive and houseware, and fiber products for carpeting. A special application suited to polypropylene is one-piece hinges that can be subjected to a high number of flexing cycles without failure. Polystyrene There are several polymers, copolymers, and terpolymers based on the monomer styrene (C8H8), of which polystyrene (PS) is used in the highest volume. It is a linear homopolymer with amorphous structure that is generally noted for its brittleness. PS is transparent, easily colored, and readily molded, but degrades at elevated temperatures and dissolves in various solvents. Because of its brittleness, some PS grades contain 5% to 15% rubber and the term high-impact polystyrene (HIPS) is used for these types. They have higher toughness, but transparency and tensile strength are reduced. In addition to injection molding applications (e.g., molded toys, housewares), polystyrene also finds uses in packaging in the form of PS foams. TABLE 8.3 (continued): (j) polypropylene. Polymer: Symbol: Polymerization method: Degree of crystallinity: Modulus of elasticity: Tensile strength: a Polypropylene (C3H6)n PP Addition High, varies with processing 1400 MPa (203,050 lb/in2) 35 MPa (5075 lb/in2) Elongation depends on additives. Elongation: Specific gravity: Glass transition temperature: Melting temperature: Approximate market share: 10%–500%a 0.90 20 C (4 F) 176 C (348 F) About 13% E1C08 11/10/2009 13:19:39 Page 171 Section 8.3/Thermosetting Polymers TABLE 8.3 (continued): (k) polystyrene. Polymer: Symbol: Polymerization method: Degree of crystallinity: Modulus of elasticity: Tensile strength: TABLE 8.3 Polystyrene (C8H8)n PS Addition None (amorphous) 3200 MPa (464,120 lb/in2) 50 MPa (7250 lb/in2) Elongation: Specific gravity: Glass transition temperature: Melting temperature: Approximate market share: 1% 1.05 100 C (212 F) 240 C (464 F) About 10% (continued): (l) polyvinylchloride. Polymer: Symbol: Polymerization method: Degree of crystallinity: Modulus of elasticity: Tensile strength: b 171 Polyvinylchloride (C2H3Cl)n PVC Addition None (amorphous structure) 2800 MPa (406,110 lb/in2)a 40 MPa (5800 lb/in2) Elongation: Specific gravity: Glass transition temperature: Melting temperature: Approximate market share: 2% with no plasticizer 1.40 81 C (178 F)b 212 C (414 F) About 16% With no plasticizer. Polyvinylchloride Polyvinylchloride (PVC) is a widely used plastic whose properties can be varied by combining additives with the polymer. In particular, plasticizers are used to achieve thermoplastics ranging from rigid PVC (no plasticizers) to flexible PVC (high proportions of plasticizer). The range of properties makes PVC a versatile polymer, with applications that include rigid pipe (used in construction, water and sewer systems, irrigation), fittings, wire and cable insulation, film, sheets, food packaging, flooring, and toys. PVC by itself is relatively unstable to heat and light, and stabilizers must be added to improve its resistance to these environmental conditions. Care must be taken in the production and handling of the vinyl chloride monomer used to polymerize PVC, due to its carcinogenic nature. 8.3 THERMOSETTING POLYMERS Thermosetting (TS) polymers are distinguished by their highly cross-linked structure. In effect, the formed part (e.g., the pot handle or electrical switch cover) becomes one large macromolecule. Thermosets are always amorphous and exhibit no glass transition temperature. In this section, we examine the general characteristics of the TS plastics and identify the important materials in this category. 8.3.1 GENERAL PROPERTIES AND CHARACTERISTICS Owing to differences in chemistry and molecular structure, properties of thermosetting plastics are different from those of thermoplastics. In general, thermosets are (1) more rigid—modulus of elasticity is 2 to 3 times greater; (2) brittle—they possess virtually no ductility; (3) less soluble in common solvents; (4) capable of higher service temperatures; and (5) not capable of being remelted—instead they degrade or burn. The differences in properties of the TS plastics are attributable to cross-linking, which forms a thermally stable, three-dimensional, covalently bonded structure within the molecule. Cross-linking is accomplished in three ways [7]: 1. Temperature-activated systems—In the most common systems, the changes are caused by heat supplied during the part-shaping operation (e.g., molding). The starting E1C08 11/10/2009 172 13:19:39 Page 172 Chapter 8/Polymers material is a linear polymer in granular form supplied by the chemical plant. As heat is added, the material softens for molding; continued heating results in cross-linking of the polymer. The term thermosetting is most aptly applied to these polymers. 2. Catalyst-activated systems—Cross-linking in these systems occurs when small amounts of a catalyst are added to the polymer, which is in liquid form. Without the catalyst, the polymer remains stable; once combined with the catalyst, it changes into solid form. 3. Mixing-activated systems—Most epoxies are examples of these systems. The mixing of two chemicals results in a reaction that forms a cross-linked solid polymer. Elevated temperatures are sometimes used to accelerate the reactions. The chemical reactions associated with cross-linking are called curing or setting. Curing is done at the fabrication plants that shape the parts rather than the chemical plants that supply the starting materials to the fabricator. 8.3.2 IMPORTANT THERMOSETTING POLYMERS Thermosetting plastics are not as widely used as the thermoplastics, perhaps because of the added processing complications involved in curing the TS polymers. The largest volume thermosets are phenolic resins, whose annual volume is about 6% of the total plastics market. This is significantly less than polyethylene, the leading thermoplastic, whose volume is about 35% of the total. Technical data for these materials are given in Table 8.4. Market share data refer to total plastics (TP plus TS). Amino Resins Amino plastics, characterized by the amino group (NH2), consist of two thermosetting polymers, urea-formaldehyde and melamine-formaldehyde, which are produced by the reaction of formaldehyde (CH2O) with either urea (CO(NH2)2) or melamine (C3H6N6), respectively. In commercial importance, the amino resins rank just below the other formaldehyde resin, phenol-formaldehyde, discussed below. Urea–formaldehyde is competitive with the phenols in certain applications, particularly as a plywood and particle-board adhesive. The resins are also used as a molding compound. It is slightly more expensive than the phenol material. Melamine–formaldehyde plastic is water resistant and is used for dishware and as a coating in laminated table and counter tops (Formica, trade name of Cyanamid Co.). When used as molding materials, amino plastics usually contain significant proportions of fillers, such as cellulose. Epoxies Epoxy resins are based on a chemical group called the epoxides. The simplest formulation of epoxide is ethylene oxide (C2H3O). Epichlorohydrin (C3H5OCl) is a much more widely used epoxide for producing epoxy resins. Uncured, epoxides have a low degree of polymerization. To increase molecular weight and to cross-link the epoxide, a curing agent TABLE 8.4 Important commercial thermosetting polymers: (a) amino resins. Representative polymer: Melamine-formaldehyde Monomers: Melamine (C3H6N6) and formaldehyde (CH2O) Polymerization method: Step (condensation) Modulus of elasticity: 9000 MPa (1,305,000 lb/in2) Tensile strength: 50 MPa (7250 lb/in2) Elongation: Less than 1% Specific gravity: 1.5 Approximate market share: About 4% for ureaformaldehyde and melamine-formaldehyde. Table 8.4 is compiled from [2], [4], [6], [7], [9], [16], and other sources. E1C08 11/10/2009 13:19:39 Page 173 Section 8.3/Thermosetting Polymers TABLE 8.4 (continued): (b) epoxy. Example chemistry: Polymerization method: Modulus of elasticity: Tensile strength: TABLE 8.4 173 Epichlorohydrin (C3H5OCl) plus curing agent such as triethylamine (C6H5–CH2N–(CH3)2) Condensation 7000 MPa (1,015,000 lb/in2) 70 MPa (10,150 lb/in2) Elongation: Specific gravity: Approximate market share: 0% 1.1 About 1% (continued): (c) phenol formaldehyde. Monomer ingredients: Polymerization method: Modulus of elasticity: Tensile strength: Phenol (C6H5OH) and formaldehyde (CH2O) Step (condensation) Elongation: Specific gravity: 7000 MPa (1,015,000 lb/in2) 70 MPa (10,150 lb/in2) Approximate market share: Less than 1% 1.4 6% must be used. Possible curing agents include polyamines and acid anhydrides. Cured epoxies are noted for strength, adhesion, and heat and chemical resistance. Applications include surface coatings, industrial flooring, glass fiber-reinforced composites, and adhesives. Insulating properties of epoxy thermosets make them useful in various electronic applications, such as encapsulation of integrated circuits and lamination of printed circuit boards. Phenolics Phenol (C6H5OH) is an acidic compound that can be reacted with aldehydes (dehydrogenated alcohols), formaldehyde (CH2O) being the most reactive. Phenolformaldehyde is the most important of the phenolic polymers; it was first commercialized around 1900 under the trade name Bakelite. It is almost always combined with fillers such as wood flour, cellulose fibers, and minerals when used as a molding material. It is brittle, possesses good thermal, chemical, and dimensional stability. Its capacity to accept colorants is limited—it is available only in dark colors. Molded products constitute only about 10% of total phenolics use. Other applications include adhesives for plywood, printed circuit boards, counter tops, and bonding material for brake linings and abrasive wheels. Polyesters Polyesters, which contain the characteristic ester linkages (CO–O), can be thermosetting as well as thermoplastic (Section 8.2). Thermosetting polyesters are used largely in reinforced plastics (composites) to fabricate large items such as pipes, tanks, boat hulls, auto body parts, and construction panels. They can also be used in various molding processes to produce smaller parts. Synthesis of the starting polymer involves reaction of an acid or anhydride such as maleic anhydride (C4H2O3) with a glycol such as ethylene glycol (C2H6O2). This produces an unsaturated polyester of relatively low molecular weight (MW ¼ 1000 to 3000). This ingredient is mixed with a monomer capable of polymerizing and crosslinking with the polyester. Styrene (C8H8) is commonly used for this purpose, in proportions of 30% to 50%. A third component, called an inhibitor, is added to prevent premature crosslinking. This mixture forms the polyester resin system that is supplied to the fabricator. Polyesters are cured either by heat (temperature-activated systems), or by means of a catalyst TABLE 8.4 (continued): (d) unsaturated polyester. Example chemistry: Polymerization method: Modulus of elasticity: Tensile strength: Maleic anhydride (C4H2O3) and ethylene glycol (C2H6O2) plus styrene (C8H8) Step (condensation) Elongation: Specific gravity: 7000 MPa (1,015,000 lb/in2) 30 MPa (4350 lb/in2) Approximate market share: 0% 1.1 3% E1C08 11/10/2009 174 13:19:40 Page 174 Chapter 8/Polymers TABLE 8.4 (continued): (e) polyimides. Starting monomers: Polymerization method: Modulus of elasticity: Tensile strength: Pyromellitic dianhydride (C6H2(C2O3)2), 4,40 -oxydianiline (O(C6H4NH2)2) Condensation Elongation: 5% 3200 MPa (464,120 lb/in2) Specific gravity: 1.43 80 MPa (11,600 lb/in2) Approximate market share: Less than 1% added to the polyester resin (catalyst-activated systems). Curing is done at the time of fabrication (molding or other forming process) and results in cross-linking of the polymer. An important class of polyesters are the alkyd resins (the name derived by abbreviating and combining the words alcohol and acid and changing a few letters). They are used primarily as bases for paints, varnishes, and lacquers. Alkyd molding compounds are also available, but their applications are limited. Polyimides These plastics are available as both thermoplastics and thermosets, but the TS types are more important commercially. They are available under brand names such as Kapton (Dupont) and Kaptrex (Professional Plastics) in several forms including tapes, films, coatings, and molding resins. TS polyimides (PI) are noted for chemical resistance, high tensile strength and stiffness, and stability at elevated temperatures. They are called hightemperature polymers due to their excellent heat resistance. Applications that exploit these properties include insulating films, molded parts used in elevated temperature service, flexible cables in laptop computers, medical tubing, and fibers for protective clothing. Polyurethanes This includes a large family of polymers, all characterized by the urethane group (NHCOO) in their structure. The chemistry of the polyurethanes is complex, and there are many chemical varieties in the family. The characteristic feature is the reaction of a polyol, whose molecules contain hydroxyl (OH) groups, such as butylene ether glycol (C4H10O2); and an isocyanate, such as diphenylmethane diisocyanate (C15H10O2N2). Through variations in chemistry, cross-linking, and processing, polyurethanes can be thermoplastic, thermosetting, or elastomeric materials, the latter two being the most important commercially. The largest application of polyurethane is in foams. These can range between elastomeric and rigid, the latter being more highly cross-linked. Rigid foams are used as a filler material in hollow construction panels and refrigerator walls. In these types of applications, the material provides excellent thermal insulation, adds rigidity to the structure, and does not absorb water in significant amounts. Many paints, varnishes, and similar coating materials are based on urethane systems. We discuss polyurethane elastomers in Section 8.4. Silicones Silicones are inorganic and semi-inorganic polymers, distinguished by the presence of the repeating siloxane link (–Si–O–) in their molecular structure. A typical formulation combines the methyl radical (CH3) with (SiO) in various proportions to obtain TABLE 8.4 (continued): (f) polyurethane. Polymer: Polymerization method: Modulus of elasticity: Tensile strength: a Polyurethane is formed by the reaction of a polyol and an isocyanate. Chemistry varies significantly Step (condensation) Elongation: Depends on cross-linking Depends on chemistry Specific gravity: 1.2 and processing Approximate market share: About 4%, including 30 MPa (4350 lb/in2)a elastomers Typical for highly cross-linked polyurethane. E1C08 11/10/2009 13:19:40 Page 175 Section 8.4/Elastomers TABLE 8.4 175 (continued): (g) silicone thermosetting resins. Example chemistry: Polymerization method: Tensile strength: ((CH3)6 –SiO)n Step (condensation), usually 30 MPa (4350 lb/in2) Elongation: Specific gravity: Approximate market share: 0% 1.65 Less than 1% the repeating unit –((CH3)m–SiO)–, where m establishes the proportionality. By variations in composition and processing, polysiloxanes can be produced in three forms: (1) fluids, (2) elastomers, and (3) thermosetting resins. Fluids (1) are low molecular weight polymers used for lubricants, polishes, waxes, and other liquids—not really polymers in the sense of this chapter, but important commercial products nevertheless. Silicone elastomers (2), covered in Section 8.4, and thermosetting silicones (3), treated here, are cross-linked. When highly crosslinked, polysiloxanes form rigid resin systems used for paints, varnishes, and other coatings; and laminates such as printed circuit boards. They are also used as molding materials for electrical parts. Curing is accomplished by heating or by allowing the solvents containing the polymers to evaporate. Silicones are noted for their good heat resistance and water repellence, but their mechanical strength is not as great as other cross-linked polymers. Data in Table 8.4(g) are for a typical silicone thermosetting polymer. 8.4 ELASTOMERS Elastomers are polymers capable of large elastic deformation when subjected to relatively low stresses. Some elastomers can withstand extensions of 500% or more and still return to their original shape. The more popular term for elastomer is, of course, rubber. We can divide rubbers into two categories: (1) natural rubber, derived from certain biological plants; and (2) synthetic elastomers, produced by polymerization processes similar to those used for thermoplastic and thermosetting polymers. Before discussing natural and synthetic rubbers, let us consider the general characteristics of elastomers. 8.4.1 CHARACTERISTICS OF ELASTOMERS Elastomers consist of long-chain molecules that are cross-linked. They owe their impressive elastic properties to the combination of two features: (1) the long molecules are tightly kinked when unstretched, and (2) the degree of cross-linking is substantially below that of the thermosets. These features are illustrated in the model of Figure 8.12(a), which shows a tightly kinked cross-linked molecule under no stress. When the material is stretched, the molecules are forced to uncoil and straighten as shown in Figure 8.12(b). The molecules’ natural resistance to uncoiling provides the initial elastic modulus of the aggregate material. As further strain is experienced, the covalent bonds FIGURE 8.12 Model of long elastomer molecules, with low degree of cross-linking: (a) unstretched, and (b) under tensile stress. E1C08 11/10/2009 176 13:19:40 Page 176 Chapter 8/Polymers FIGURE 8.13 Increase in stiffness as a function of strain for three grades of rubber: natural rubber, vulcanized rubber, and hard rubber. of the cross-linked molecules begin to play an increasing role in the modulus, and the stiffness increases as illustrated in Figure 8.13. With greater cross-linking, the elastomer becomes stiffer and its modulus of elasticity is more linear. These characteristics are shown in the figure by the stress–strain curves for three grades of rubber: natural crude rubber, whose cross-linking is very low; cured (vulcanized) rubber with low-to-medium cross-linking; and hard rubber (ebonite), whose high degree of cross-linking transforms it into a thermosetting plastic. For apolymer to exhibit elastomeric properties, it must beamorphous in theunstretched condition, and its temperature must be above Tg. If below the glass transition temperature, the material is hard and brittle; above Tg the polymer is in the ‘‘rubbery’’ state. Any amorphous thermoplastic polymer will exhibit elastomeric properties above Tg for ashort time,becauseits linear molecules are always coiled to some extent, thus allowing for elastic extension. It is the absence of cross-linking in TP polymers that prevents them from being truly elastic; instead they exhibit viscoelastic behavior. Curing is required to effect cross-linking in most of the common elastomers today. The term for curing used in the context of natural rubber (and certain synthetic rubbers) is vulcanization, which involves the formation of chemical cross-links between the polymer chains. Typical cross-linking in rubber is 1 to 10 links per 100 carbon atoms in the linear polymer chain, depending on the degree of stiffness desired in the material. This is considerably less than the degree of cross-linking in thermosets. An alternative method of curing involves the use of starting chemicals that react when mixed (sometimes requiring a catalyst or heat) to form elastomers with relatively infrequent cross-links between molecules. These synthetic rubbers are known as reactive system elastomers. Certain polymers that cure by this means, such as urethanes and silicones, can be classified as either thermosets or elastomers, depending on the degree of cross-linking achieved during the reaction. A relatively new class of elastomers, called thermoplastic elastomers, possesses elastomeric properties that result from the mixture of two phases, both thermoplastic. One is above its Tg at room temperature while the other is below its Tg. Thus, we have a polymer that includes soft rubbery regions intermixed with hard particles that act as crosslinks. The composite material is elastic in its mechanical behavior, although not as extensible as most other elastomers. Because both phases are thermoplastic, the aggregate material can be heated above its Tm for forming, using processes that are generally more economical than those used for rubber. We discuss the elastomers in the following two sections. The first deals with natural rubber and how it is vulcanized to create a useful commercial material; the second examines the synthetic rubbers. E1C08 11/10/2009 13:19:40 Page 177 Section 8.4/Elastomers 177 8.4.2 NATURAL RUBBER Natural rubber (NR) consists primarily of polyisoprene, a high-molecular-weight polymer of isoprene (C5H8). It is derived from latex, a milky substance produced by various plants, the most important of which is the rubber tree (Hevea brasiliensis) that grows in tropical climates (Historical Note 8.2). Latex is a water emulsion of polyisoprene (about one-third by weight), plus various other ingredients. Rubber is extracted from the latex by various methods (e.g., coagulation, drying, and spraying) that remove the water. Historical Note 8.2 T Natural rubber he first use of natural rubber seems to have been in the form of rubber balls used for sport by the natives of Central and South America at least 500 hundred years ago. Columbus noted this during his second voyage to the New World in 1493–1496. The balls were made from the dried gum of a rubber tree. The first white men in South America called the tree caoutchouc, which was their way of pronouncing the Indian name for it. The name rubber came from the English chemist Joseph Priestley, who discovered (around 1770) that gum rubber would ‘‘rub’’ away pencil marks. Early rubber goods were less than satisfactory; they melted in summer heat and hardened in winter cold. One of those in the business of making and selling rubber goods was American Charles Goodyear. Recognizing the deficiencies of the natural material, he experimented with ways to improve its properties and discovered that rubber could be cured by heating it with sulfur. This was in 1839, and the process, later called vulcanization, was patented by him in 1844. Vulcanization and the emerging demand for rubber products led to tremendous growth in rubber production and the industry that supported it. In 1876, Henry Wickham collected thousands of rubber tree seeds from the Brazilian jungle and planted them in England; the sprouts were later transplanted to Ceylon and Malaya (then British colonies) to form rubber plantations. Soon, other countries in the region followed the British example. Southeast Asia became the base of the rubber industry. In 1888, a British veterinary surgeon named John Dunlop patented pneumatic tires for bicycles. By the twentieth century, the motorcar industry was developing in the United States and Europe. Together, the automobile and rubber industries grew to occupy positions of unimagined importance. Natural crude rubber (without vulcanization) is sticky in hot weather, but stiff and brittle in cold weather. To form an elastomer with useful properties, natural rubber must be vulcanized. Traditionally, vulcanization has been accomplished by mixing small amounts of sulfur and other chemicals with the crude rubber and heating. The chemical effect of vulcanization is cross-linking; the mechanical result is increased strength and stiffness, yet maintenance of extensibility. The dramatic change in properties caused by vulcanization can be seen in the stress–strain curves of Figure 8.13. Sulfur alone can cause cross-linking, but the process is slow, taking hours to complete. Other chemicals are added to sulfur during vulcanization to accelerate the process and serve other beneficial functions. Also, rubber can be vulcanized using chemicals other than sulfur. Today, curing times have been reduced significantly compared to the original sulfur curing of years ago. As an engineering material, vulcanized rubber is noted among elastomers for its high tensile strength, tear strength, resilience (capacity to recover shape after deformation), and resistance to wear and fatigue. Its weaknesses are that it degrades when subjected to heat, sunlight, oxygen, ozone, and oil. Some of these limitations can be reduced through the use of additives. Typical properties and other data for vulcanized natural rubber are listed in Table 8.5. Market share is relative to total annual rubber volume, natural plus synthetic. Rubber volume is about 15% of total polymer market. E1C08 11/10/2009 178 13:19:40 Page 178 Chapter 8/Polymers TABLE 8.5 Characteristics and typical properties of vulcanized rubber. Polymer: Symbol: Modulus of elasticity: Tensile strength: Elongation: Polyisoprene (C5H8)n NR 18 MPa (2610 lb/in2) at 300% elongation 25 MPa (3625 lb/in2) 700% at failure Specific gravity: High temperature limit: Low temperature limit: Approximate market share: 0.93 80 C (176 F) 50 C (58 F) 22% Compiled from [2], [6], [9], and other sources. The largest single market for natural rubber is automotive tires. In tires, carbon black is an important additive; it reinforces the rubber, serving to increase tensile strength and resistance to tearing and abrasion. Other products made of rubber include shoe soles, bushings, seals, and shock-absorbing components. In each case, the rubber is compounded to achieve the specific properties required in the application. Besides carbon black, other additives used in rubber and some of the synthetic elastomers include clay, kaolin, silica, talc, and calcium carbonate, as well as chemicals that accelerate and promote vulcanization. 8.4.3 SYNTHETIC RUBBERS Today, the tonnage of synthetic rubbers is more than three times that of natural rubber. Development of these synthetic materials was motivated largely by the world wars when NR was difficult to obtain (Historical Note 8.3). The most important of the synthetics is styrene– butadiene rubber (SBR), a copolymer of butadiene (C4H6) and styrene (C8H8). As with most other polymers, the predominant raw material for the synthetic rubbers is petroleum. Only the synthetic rubbers of greatest commercial importance are discussed here. Technical data are presented in Table 8.6. Market share data are for total volume of natural and synthetic Historical Note 8.3 I Synthetic rubbers n 1826, Faraday recognized the formula of natural rubber to be C5H8. Subsequent attempts at reproducing this molecule over many years were generally unsuccessful. Regrettably, it was the world wars that created the necessity which became the mother of invention for synthetic rubber. In World War I, the Germans, denied access to natural rubber, developed a methyl-based substitute. This material was not very successful, but it marks the first large-scale production of synthetic rubber. After World War I, the price of natural rubber was so low that many attempts at fabricating synthetics were abandoned. However, the Germans, perhaps anticipating a future conflict, renewed their development efforts. The firm I.G. Farben developed two synthetic rubbers, starting in the early 1930s, called Buna-S and Buna-N. Buna is derived from butadiene (C4H6), which has become the critical ingredient in many modern synthetic rubbers, and Na, the symbol for sodium, used to accelerate or catalyze the polymerization process (Natrium is the German word for sodium). The symbol S in Buna-S stands for styrene. Buna-S is the copolymer we know today as styrene–butadiene rubber, or SBR. The N in Buna-N stands for acryloNitrile, and the synthetic rubber is called nitrile rubber in current usage. Other efforts included the work at the DuPont Company in the United States, which led to the development of polychloroprene, first marketed in 1932 under the name Duprene, later changed to Neoprene, its current name. During World War II, the Japanese cut off the supply of natural rubber from Southeast Asia to the United States. Production of Buna-S synthetic rubber was begun on a large scale in America. The federal government preferred to use the name GR-S (Government RubberStyrene) rather than Buna-S (the German name). By 1944, the United States was outproducing Germany in SBR 10-to-1. Since the early 1960s, worldwide production of synthetic rubbers has exceeded that of natural rubbers. E1C08 11/10/2009 13:19:40 Page 179 Section 8.4/Elastomers TABLE 8.6 179 Characteristics and typical properties of synthetic rubbers: (a) butadiene rubber. Polymer: Symbol: Tensile strength: Elongation: Polybutadiene (C4H6)n BR 15 MPa (2175 lb/in2) 500% at failure Specific gravity: High temperature limit: Low temperature limit: Approx. market share: 0.93 100 C (212 F) 50 C (58 F) 12% Table 8.6 is compiled from [2], [4], [6], [9], [11], and other sources. rubbers. About 10% of total volume of rubber production is reclaimed; thus, total tonnages in Tables 8.5 and 8.6 do not sum to 100%. Butadiene Rubber Polybutadiene (BR) is important mainly in combination with other rubbers. It is compounded with natural rubber and with styrene (styrene–butadiene rubber is discussed later) in the production of automotive tires. Without compounding, the tear resistance, tensile strength, and ease of processing of polybutadiene are less than desirable. Butyl Rubber Butyl rubber is a copolymer of polyisobutylene (98%–99%) and polyisoprene (1%–2%). It can be vulcanized to provide a rubber with very low air permeability, which has led to applications in inflatable products such as inner tubes, liners in tubeless tires, and sporting goods. Chloroprene Rubber Polychloroprene was one of the first synthetic rubbers to be developed (early 1930s). Commonly known today as Neoprene, it is an important specialpurpose rubber. It crystallizes when strained to provide good mechanical properties. Chloroprene rubber (CR) is more resistant to oils, weather, ozone, heat, and flame (chlorine makes this rubber self-extinguishing) than NR, but somewhat more expensive. Its applications include fuel hoses (and other automotive parts), conveyor belts, and gaskets, but not tires. Ethylene–Propylene Rubber Polymerization of ethylene and propylene with small proportions (3%–8%) of a diene monomer produces the terpolymer ethylene–propylene–diene (EPDM), a useful synthetic rubber. Applications are for parts mostly in the automotive industry other than tires. Other uses are wire and cable insulation. TABLE 8.6 (continued): (b) butyl rubber. Polymer: Symbol: Modulus of elasticity: Tensile strength: Elongation: TABLE 8.6 Copolymer of isobutylene (C4H8)n and isoprene (C5H8)n PIB Specific gravity: High temperature limit: 7 MPa (1015 lb/in2) at 300% elongation 20 MPa (2900 lb/in2) Low temperature limit: 700% Approximate market share: 0.92 110 C (230 F) 50 C (58 F) About 3% (continued): (c) chloroprene rubber (neoprene). Polymer: Symbol: Modulus of elasticity: Tensile strength: Elongation: Polychloroprene (C4H5Cl)n CR 7 MPa (1015 lb/in2) at 300% elongation 25 MPa (3625 lb/in2) 500% at failure Specific gravity: High temperature limit: Low temperature limit: Approximate market share: 1.23 120 C (248 F) 20 C (4 F) 2% E1C08 11/10/2009 180 13:19:40 Page 180 Chapter 8/Polymers TABLE 8.6 (continued): (d) ethylene–propylene–diene rubber. Representative polymer: Symbol: Tensile strength: Elongation: TABLE 8.6 Terpolymer of ethylene (C2H4), propylene (C3H6), and a diene monomer (3%–8%) for cross-linking EPDM Specific gravity: 0.86 15 MPa (2175 lb/in2) High temperature limit: 150 C (302 F) 300% at failure Low temperature limit: 50 C (58 F) Approximate market share: 5% (continued): (e) isoprene rubber (synthetic). Polymer: Symbol: Modulus of elasticity: Tensile strength: Elongation: Polyisoprene (C5H8)n IR 17 MPa (2465 lb/in2) at 300% elongation 25 MPa (3625 lb/in2) 500% at failure Specific gravity: High temperature limit: Low temperature limit: Approximate market share: 0.93 80 C (176 F) 50 C (58 F) 2% Isoprene Rubber Isoprene can be polymerized to synthesize a chemical equivalent of natural rubber. Synthetic (unvulcanized) polyisoprene is softer and more easily molded than raw natural rubber. Applications of the synthetic material are similar to those of its natural counterpart, car tires being the largest single market. It is also used for footwear, conveyor belts, and caulking compound. Cost per unit weight is about 35% higher than for NR. Nitrile Rubber This is a vulcanizable copolymer of butadiene (50%–75%) and acrylonitrile (25%–50%). Its more technical name is butadiene-acrylonitrile rubber. It has good strength and resistance to abrasion, oil, gasoline, and water. These properties make it ideal for applications such as gasoline hoses and seals, and also for footwear. Polyurethanes Thermosetting polyurethanes (Section 8.3.2) with minimum cross-linking are elastomers, most commonly produced as flexible foams. In this form, they are widely used as cushion materials for furniture and automobile seats. Unfoamed polyurethane can TABLE 8.6 (continued): (f) nitrile rubber. Polymer: Symbol: Modulus of elasticity: Tensile strength: Elongation: TABLE 8.6 Copolymer of butadiene (C4H6) and acrylonitrile (C3H3N) NBR Specific gravity: High temperature limit: 10 MPa (1450 lb/in2) at 300% elongation Low temperature limit: 30 MPa (4350 lb/in2) 500% at failure Approximate market share: 1.00 (without fillers) 120 C (248 F) 50 C (58 F) 2% (continued): (g) polyurethane. Polymer: Symbol: Modulus of elasticity: Tensile strength: Elongation: Polyurethane (chemistry varies) PUR 10 MPa (1450 lb/in2) at 300% elongation 60 MPa (8700 lb/in2) 700% at failure Specific gravity: High temperature limit: Low temperature limit: Approximate market share: 1.25 100 C (212 F) 50 C (–58 F) Listed under thermosets, Table 8.4(e) E1C08 11/10/2009 13:19:41 Page 181 Section 8.4/Elastomers TABLE 8.6 181 (continued): (h) silicone rubber. Representative polymer: Symbol: Tensile strength: Elongation: Polydimethylsiloxane (SiO(CH3)2)n VMQ 10 MPa (1450 lb/in2) 700% at failure Specific gravity: High temperature limit: Low temperature limit: Approximate market share: 0.98 230 C (446 F) 50 C (58 F) Less than 1% be molded into products ranging from shoe soles to car bumpers, with cross-linking adjusted to achieve the desired properties for the application. With no cross-linking, the material is a thermoplastic elastomer that can be injection molded. As an elastomer or thermoset, reaction injection molding and other shaping methods are used. Silicones Like the polyurethanes, silicones can be elastomeric or thermosetting, depending on the degree of cross-linking. Silicone elastomers are noted for the wide temperature range over which they can be used. Their resistance to oils is poor. The silicones possess various chemistries, the most common being polydimethylsiloxane (Table 8.6(h)). To obtain acceptable mechanical properties, silicone elastomers must be reinforced, usually with fine silica powders. Owing to their high cost, they are considered special-purpose rubbers for applications such as gaskets, seals, wire and cable insulation, prosthetic devices, and bases for caulking materials. Styrene–Butadiene Rubber SBR is a random copolymer of styrene (about 25%) and butadiene (about 75%). It was originally developed in Germany as Buna-S rubber before World War II. Today, it is the largest tonnage elastomer, totaling about 40% of all rubbers produced (natural rubber is second in tonnage). Its attractive features are low cost, resistance to abrasion, and better uniformity than NR. When reinforced with carbon black and vulcanized, its characteristics and applications are very similar to those of natural rubber. Cost is also similar. A close comparison of properties reveals that most of its mechanical properties except wear resistance are inferior to NR, but its resistance to heat aging, ozone, weather, and oils is superior. Applications include automotive tires, footwear, and wire and cable insulation. A material chemically related to SBR is styrene– butadiene–styrene block copolymer, a thermoplastic elastomer discussed below. Thermoplastic Elastomers As previously described, a thermoplastic elastomer (TPE) is a thermoplastic that behaves like an elastomer. It constitutes a family of polymers that is a fast-growing segment of the elastomer market. TPEs derive their elastomeric properties not from chemical cross-links, but from physical connections between soft and hard phases that make up the material. Thermoplastic elastomers include styrene– butadiene–styrene (SBS), a block copolymer as opposed to styrene–butadiene rubber (SBR) which is a random copolymer (Section 8.1.2); thermoplastic polyurethanes; TABLE 8.6 (continued): (i) styrene–butadiene rubber. Polymer: Symbol: Modulus of elasticity: Tensile strength: Copolymer of styrene (C8H8) and butadiene (C4H6) SBR Elongation: Specific gravity: 17 MPa (2465 lb/in2) at 300% elongation 20 MPa (2900 lb/in2) reinforced High temperature limit: Low temperature limit: Approximate market share: 700% at failure 0.94 110 C (230 F) 50 C (58 F) Slightly less than 30% E1C08 11/10/2009 182 13:19:41 Page 182 Chapter 8/Polymers TABLE 8.6 (continued): (j) thermoplastic elastomers (TPE). Representative polymer: Symbol: Tensile strength: Elongation: Styrene–butadiene–styrene block copolymer SBS (also YSBR) Specific gravity: 14 MPa (2030 lb/in2) High temperature limit: 400% Low temperature limit: Approximate market share: 1.0 65 C (149 F) 50 C (58 F) 12% thermoplastic polyester copolymers; and other copolymers and polymer blends. Table 8.6 (j) gives data on SBS. The chemistry and structure of these materials are generally complex, involving two materials that are incompatible so that they form distinct phases whose room temperature properties are different. Owing to their thermoplasticity, the TPEs cannot match conventional cross-linked elastomers in elevated temperature strength and creep resistance. Typical applications include footwear, rubber bands, extruded tubing, wire coating, and molded parts for automotive and other uses in which elastomeric properties are required. TPEs are not suitable for tires. 8.5 POLYMER RECYCLING AND BIODEGRADABILITY It is estimated that since the 1950s, 1 billion tons of plastic have been discarded as garbage.2 This plastic trash could be around for centuries, because the primary bonds that make plastics so durable also make them resistant to degradation by the environmental and biological processes of nature. In this section, we consider two polymer topics related to environmental concerns: (1) recycling of polymer products and (2) biodegradable plastics. 8.5.1 POLYMER RECYCLING Approximately 200 million tons of plastic products are made annually throughout the world, more than one-eighth of which are produced in the United States.3 Only about 6% of the U.S. tonnage is recycled as plastic waste; the rest either remains in products and/or ends up in garbage landfills. Recycling means recovering the discarded plastic items and reprocessing them into new products, in some cases products that are quite different from the original discarded items. In general the recycling of plastics is more difficult that recycling of glass and metal products. There are several reasons for this: (1) compared to plastic parts, many recycled metal items are much larger and heavier (e.g., structural steel from buildings and bridges, steel car body frames), so the economics of recycling are more favorable for recycling metals; most plastic items are lightweight; (2) compared to plastics, which come in a variety of chemical compositions that do not mix well, glass products are all based on silicon dioxide; and (3) many plastic products contain fillers, dyes, and other additives that cannot be readily separated from the polymer itself. Of course, a common problem in all recycling efforts is the fluctuation in prices of recycled materials. To cope with the problem of mixing different types of plastics and to promote recycling of plastics, the Plastic Identification Code (PIC) was developed by the Society 2 en.wikipedia.org/wiki/Plastic. According to the Society of Plastics Engineers, as reported in en.wikipedia.org/wiki/Biodegradable_ plastic. 3 E1C08 11/10/2009 13:19:41 Page 183 Section 8.5/Polymer Recycling and Biodegradability 183 of the Plastics Industry. The code is a symbol consisting of a triangle formed by three bent arrows enclosing a number. It is printed or molded on the plastic item. The number identifies the plastic for recycling purposes. The seven plastics (all thermoplastics) used in the PIC recycling program are (1) polyethylene terephthalate, used in 2-liter beverage containers; (2) high-density polyethylene, used in milk jugs and shopping bags; (3) polyvinyl chloride, used in juice bottles and PVC pipes; (4) low-density polyethylene, used in squeezable bottles and flexible container lids; (5) polypropylene, used in yogurt and margarine containers; (6) polystyrene, used in egg cartons, disposable plates, cups, and utensils, and as foamed packing materials; and (7) other, such as polycarbonate or ABS. The PIC facilitates the separation of items made from the different types of plastics for reprocessing. Nevertheless, sorting the plastics is a labor-intensive activity. Once separated, the thermoplastic items can be readily reprocessed into new products by remelting. This is not the case with thermosets and rubbers because of the cross-linking in these polymers. Thus, these materials must be recycled and reprocessed by different means. Recycled thermosets are typically ground up into particulate matter and used as fillers, for example, in molded plastic parts. Most recycled rubber comes from used tires. While some of these tires are retreaded, others are ground up into granules in forms such as chunks and nuggets that can be used for landscape mulch, playgrounds, and similar purposes. 8.5.2 BIODEGRADABLE POLYMERS Another approach that addresses the environmental concerns about plastics involves the development of biodegradable plastics, which are defined as plastics that are decomposed by the actions of microorganisms occurring in nature, such as bacteria and fungi. Conventional plastic products usually consist of a combination of a petroleumbased polymer and a filler (Section 8.1.5). In effect, the material is a polymer-matrix composite (Section 9.4). The purpose of the filler is to improve mechanical properties and/or reduce material cost. In many cases, neither the polymer nor the filler are biodegradable. Distinguished from these non-biodegradable plastics are two forms of biodegradable plastics: (1) partially degradable and (2) completely degradable. Partially biodegradable plastics consist of a conventional polymer and a natural filler. The polymer matrix is petroleum-based, which is non-biodegradable, but the natural filler can be consumed by microorganisms (e.g., in a landfill), thus converting the polymer into a sponge-like structure and possibly leading to its degradation over time. The plastics of greatest interest from an environmental viewpoint are the completely biodegradable plastics (aka bioplastics) consisting of a polymer and filler that are both derived from natural and renewable sources. Various agricultural products are used as the raw materials for biodegradable plastics. A common polymeric starting material is starch, which is a major component in corn, wheat, rice, and potatoes. It consists of the two polymers amylose and amylopectin. Starch can be used to synthesize several thermoplastic materials that are processable by conventional plastic shaping methods, such as extrusion and injection molding (Chapter 13). Another starting point for biodegradable plastics involves fermentation of either corn starch or sugar cane to produce lactic acid, which can be polymerized to form polylactide, another thermoplastic material. A common filler used in bioplastics is cellulose, often in the form of reinforcing fibers in the polymer-matrix composite. Cellulose is grown as flax or hemp. It is inexpensive and possesses good mechanical strength. Applications of biodegradable plastics are inhibited by the fact that these materials are more expensive than petroleum-based polymers. That may change in the future due to technological advances and economies of scale. Biopolymers are most attractive in situations where degradability is a higher priority than cost savings. At the top of the list are packaging materials that are quickly discarded as waste in E1C08 11/10/2009 184 13:19:41 Page 184 Chapter 8/Polymers landfills. It is estimated that approximately 40% of all plastics are used in packaging, mostly for food products [12]. Thus, biodegradable plastics are being used increasingly as substitutes for conventional plastics in packaging applications. Other applications include disposable food service items, coatings for paper and cardboard, waste bags, and mulches for agricultural crops. Medical applications include sutures, catheter bags, and sanitary laundry bags in hospitals. 8.6 GUIDE TO THE PROCESSING OF POLYMERS Polymers are nearly always shaped in a heated, highly plastic consistency. Common operations are extrusion and molding. The molding of thermosets is generally more complicated because they require curing (cross-linking). Thermoplastics are easier to mold, and a greater variety of molding operations are available to process them (Chapter 13). Although plastics readily lend themselves to net shape processing, machining is sometimes required (Chapter 22); and plastic parts can be assembled into products by permanent joining techniques such as welding (Chapter 29), adhesive bonding (Section 31.3), or mechanical assembly (Chapter 32). Rubber processing has a longer history than plastics, and the industries associated with these polymer materials have traditionally been separated, even though their processing is similar in many ways. We cover rubber processing technology in Chapter 14. REFERENCES [1] Alliger, G., and Sjothum, I. J. (eds.). Vulcanization of Elastomers. Krieger Publishing Company, New York, 1978. [2] Billmeyer, F. W., Jr. Textbook of Polymer Science, 3rd ed. John Wiley & Sons, Inc., New York, 1984. [3] Blow, C. M., and Hepburn, C. Rubber Technology and Manufacture, 2nd ed. Butterworth Scientific, London, 1982. [4] Brandrup, J., and Immergut, E. E. (eds.). Polymer Handbook, 4th ed. John Wiley & Sons, Inc., New York, 2004. [5] Brydson, J. A. Plastics Materials, 4th ed. Butterworths & Co., Ltd., London, 1999. [6] Chanda, M., and Roy, S. K. Plastics Technology Handbook, 4th ed. CRC Taylor & Francis, Boca Raton, Florida, 2006. [7] Charrier, J-M. Polymeric Materials and Processing. Oxford University Press, New York, 1991. [8] Engineering Materials Handbook, Vol. 2, Engineering Plastics. ASM International, Materials Park, Ohio, 2000. [9] Flinn, R. A., and Trojan, P. K. Engineering Materials and Their Applications, 5th ed. John Wiley & Sons, Inc., New York, 1995. [10] Hall, C. Polymer Materials, 2nd ed. John Wiley & Sons, New York, 1989. [11] Hofmann, W. Rubber Technology Handbook. Hanser Publishers, Munich, Germany, 1988. [12] Kolybaba, M., Tabil, L. G., Panigrahi, S., Crerar, W. J., Powell, T., and Wang, B. ‘‘Biodegradable Polymers: Past Present, and Future,’’ Paper Number RRV03-0007, American Society of Agricultural Engineers, October 2003. [13] Margolis, J. M. Engineering Plastics Handbook. McGraw-Hill, New York, 2006. [14] Mark, J. E., and Erman, B. (eds.). Science and Technology of Rubber, 3rd ed. Academic Press, Orlando, Florida, 2005. [15] McCrum, N. G., Buckley, C. P., and Bucknall, C. B. Principles of Polymer Engineering, 2nd ed. Oxford University Press, Oxford, UK, 1997. [16] Modern Plastics Encyclopedia. Modern Plastics, McGraw-Hill, Inc., New York, 1990. [17] Reisinger, T. J. G. ‘‘Polymers of Tomorrow,’’ Advanced Materials & Processes, March 2004, pp. 43–45. [18] Rudin, A. The Elements of Polymer Science and Engineering, 2nd ed. Academic Press, Inc., Orlando, Florida, 1998. [19] Seymour, R. B., and Carraher, C. E. Seymour/ Carraher’s Polymer Chemistry, 5th ed. Marcel Dekker, Inc., New York, 2000. [20] Seymour, R. B. Engineering Polymer Sourcebook. McGraw-Hill Book Company, New York, 1990. [21] Wikipedia. ‘‘Plastic recycling.’’ Available at: http://en. wikipedia.org/wiki/Plastic_recycling. ‘‘Biodegradable plastic.’’ Available at: http://en.wikipedia.org/wiki/ E1C08 11/10/2009 13:19:41 Page 185 Multiple Choice Quiz Biodegradable_plastic. ‘‘Plastic.’’ Available at: http:// en.wikipedia.org/wiki/Plastic. [22] Green Plastics. Available at: http://www.greenplastics.com/reference. 185 [23] Young, R. J., and Lovell, P. Introduction to Polymers, 3rd ed. CRC Taylor and Francis, Boca Raton, Florida, 2008. REVIEW QUESTIONS 8.1. What is a polymer? 8.2. What are the three basic categories of polymers? 8.3. How do the properties of polymers compare with those of metals? 8.4. What does the degree of polymerization indicate? 8.5. What is cross-linking in a polymer, and what is its significance? 8.6. What is a copolymer? 8.7. Copolymers can possess four different arrangements of their constituent mers. Name and briefly describe the four arrangements. 8.8. What is a terpolymer? 8.9. How are a polymer’s properties affected when it takes on a crystalline structure? 8.10. Does any polymer ever become 100% crystalline? 8.11. What are some of the factors that influence a polymer’s tendency to crystallize? 8.12. Why are fillers added to a polymer? 8.13. What is a plasticizer? 8.14. In addition to fillers and plasticizers, what are some other additives used with polymers? 8.15. Describe the difference in mechanical properties as a function of temperature between a highly crystalline thermoplastic and an amorphous thermoplastic. 8.16. What is unique about the polymer cellulose? 8.17. The nylons are members of which polymer group? 8.18. What is the chemical formula of ethylene, the monomer for polyethylene? 8.19. What is the basic difference between low-density and high-density polyethylene? 8.20. How do the properties of thermosetting polymers differ from those of thermoplastics? 8.21. Cross-linking (curing) of thermosetting plastics is accomplished by one of three ways. Name the three ways. 8.22. Elastomers and thermosetting polymers are both cross-linked. Why are their properties so different? 8.23. What happens to an elastomer when it is below its glass transition temperature? 8.24. What is the primary polymer ingredient in natural rubber? 8.25. How do thermoplastic elastomers differ from conventional rubbers? MULTIPLE CHOICE QUIZ There are 20 correct answers in the following multiple choice questions (some questions have multiple answers that are correct). To attain a perfect score on the quiz, all correct answers must be given. Each correct answer is worth 1 point. Each omitted answer or wrong answer reduces the score by 1 point, and each additional answer beyond the correct number of answers reduces the score by 1 point. Percentage score on the quiz is based on the total number of correct answers. 8.1. Of the three polymer types, which one is the most important commercially: (a) thermoplastics, (b) thermosets, or (c) elastomers? 8.2. Which one of the three polymer types is not normally considered to be a plastic: (a) thermoplastics, (b) thermosets, or (c) elastomers? 8.3. Which one of the three polymer types does not involve cross-linking: (a) thermoplastics, (b) thermosets, or (c) elastomers? 8.4. As the degree of crystallinity in a given polymer increases, the polymer becomes denser and stiffer, and its melting temperature decreases: (a) true or (b) false? 8.5. Which one of the following is the chemical formula for the repeating unit in polyethylene: (a) CH2, (b) C2H4, (c) C3H6, (d) C5H8, or (e) C8H8? 8.6. Degree of polymerization is which one of the following: (a) average number of mers in the molecule chain; (b) proportion of the monomer that has been polymerized; (c) sum of the molecule weights of the mers in the molecule; or (d) none of the above? 8.7. A branched molecular structure is stronger in the solid state and more viscous in the molten state than a linear structure for the same polymer: (a) true or (b) false? 8.8. A copolymer is a mixture of the macromolecules of two different homopolymers: (a) true or (b) false? 8.9. As the temperature of a polymer increases, its density (a) increases, (b) decreases, or (c) remains fairly constant? 8.10. Which of the following plastics has the highest market share: (a) phenolics, (b) polyethylene, E1C08 11/10/2009 186 8.11. 8.12. 8.13. 8.14. 13:19:41 Page 186 Chapter 8/Polymers (c) polypropylene, (d) polystyrene, or (e) polyvinylchloride? Which of the following polymers are normally thermoplastic (four best answers): (a) acrylics, (b) cellulose acetate, (c) nylon, (d) phenolics, (e) polychloroprene, (f) polyesters, (g) polyethylene, (h) polyisoprene, and (i) polyurethane? Polystyrene (without plasticizers) is amorphous, transparent, and brittle: (a) true or (b) false? The fiber rayon used in textiles is based on which one of the following polymers: (a) cellulose, (b) nylon, (c) polyester, (d) polyethylene, or (e) polypropylene? The basic difference between low-density polyethylene and high-density polyethylene is that the latter has a much higher degree of crystallinity: (a) true or (b) false? 8.15. Among the thermosetting polymers, the most widely used commercially is which one of the following: (a) epoxies, (b) phenolics, (c) silicones, or (d) urethanes? 8.16. The chemical formula for polyisoprene in natural rubber is which of the following: (a) CH2, (b) C2H4, (c) C3H6, (d) C5H8, or (e) C8H8? 8.17. The leading commercial synthetic rubber is which one of the following: (a) butyl rubber, (b) isoprene rubber, (c) polybutadiene, (d) polyurethane, (e) styrene-butadiene rubber, or (f) thermoplastic elastomers? E1C09 11/11/2009 14:30:30 9 Page 187 COMPOSITE MATERIALS Chapter Contents 9.1 Technology and Classification of Composite Materials 9.1.1 Components in a Composite Material 9.1.2 The Reinforcing Phase 9.1.3 Properties of Composite Materials 9.1.4 Other Composite Structures 9.2 Metal Matrix Composites 9.2.1 Cermets 9.2.2 Fiber-Reinforced Metal Matrix Composites 9.3 Ceramic Matrix Composites 9.4 Polymer Matrix Composites 9.4.1 Fiber-Reinforced Polymers 9.4.2 Other Polymer Matrix Composites 9.5 Guide to Processing Composite Materials In addition to metals, ceramics, and polymers, a fourth material category can be distinguished: composites. A composite material is a material system composed of two or more physically distinct phases whose combination produces aggregate properties that are different from those of its constituents. In certain respects, composites are the most interesting of the engineering materials because their structure is more complex than the other three types. The technological and commercial interest in composite materials derives from the fact that their properties are not just different from their components but are often far superior. Some of the possibilities include: å Composites can be designed that are very strong and stiff, yet very light in weight, giving them strengthto-weight and stiffness-to-weight ratios several times greater than steel or aluminum. These properties are highly desirable in applications ranging from commercial aircraft to sports equipment. å Fatigue properties are generally better than for the common engineering metals. Toughness is often greater, too. å Composites can be designed that do not corrode like steel; this is important in automotive and other applications. å With composite materials, it is possible to achieve combinations of properties not attainable with metals, ceramics, or polymers alone. å Better appearance and control of surface smoothness are possible with certain composite materials. Along with the advantages, there are disadvantages and limitations associated with composite materials. These include: (1) properties of many important composites are anisotropic, which means the properties differ depending on the direction in which they are measured; (2) many of the polymer-based composites are subject to attack by chemicals or 187 E1C09 11/11/2009 188 14:30:30 Page 188 Chapter 9/Composite Materials solvents, just as the polymers themselves are susceptible to attack; (3) composite materials are generally expensive, although prices may drop as volume increases; and (4) certain of the manufacturing methods for shaping composite materials are slow and costly. We have already encountered several composite materials in our coverage of the three other material types. Examples include cemented carbides (tungsten carbide with cobalt binder), plastic molding compounds that contain fillers (e.g., cellulose fibers, wood flour), and rubber mixed with carbon black. We did not always identify these materials as composites; however, technically, they fit the above definition. It could even be argued that a two-phase metal alloy (e.g., Fe þ Fe3C) is a composite material, although it is not classified as such. Perhaps the most important composite material of all is wood. In our presentation of composite materials, we first examine their technology and classification. There are many different materials and structures that can be used to form composites; we survey the various categories, devoting the most time to fiber-reinforced plastics, which are commercially the most important type. In the final section, we provide a guide to the manufacturing processes for composites. 9.1 TECHNOLOGY AND CLASSIFICATION OF COMPOSITE MATERIALS As noted in our definition, a composite material consists of two or more distinct phases. The term phase indicates a homogeneous material, such as a metal or ceramic in which all of the grains have the same crystal structure, or a polymer with no fillers. By combining the phases, using methods yet to be described, a new material is created with aggregate performance exceeding that of its parts. The effect is synergistic. Composite materials can be classified in various ways. One possible classification distinguishes between (1) traditional and (2) synthetic composites. Traditional composites are those that occur in nature or have been produced by civilizations for many years. Wood is a naturally occurring composite material, while concrete (Portland cement plus sand or gravel) and asphalt mixed with gravel are traditional composites used in construction. Synthetic composites are modern material systems normally associated with the manufacturing industries, in which the components are first produced separately and then combined in a controlled way to achieve the desired structure, properties, and part geometry. These synthetic materials are the composites normally thought of in the context of engineered products. Our attention in this chapter is focused on these materials. 9.1.1 COMPONENTS IN A COMPOSITE MATERIAL In the simplest manifestation of our definition, a composite material consists of two phases: a primary phase and a secondary phase. The primary phase forms the matrix within which the secondary phase is imbedded. The imbedded phase is sometimes referred to as a reinforcing agent (or similar term), because it usually serves to strengthen the composite. The reinforcing phase may be in the form of fibers, particles, or various other geometries, as we shall see. The phases are generally insoluble in each other, but strong adhesion must exist at their interface(s). The matrix phase can be any of three basic material types: polymers, metals, or ceramics. The secondary phase may also be one of the three basic materials, or it may be an element such as carbon or boron. Possible combinations in a two-component composite material can be organized as a 3  4 chart, as in Table 9.1. We see that certain combinations are not feasible, such as a polymer in a ceramic matrix. We also see that the possibilities E1C09 11/11/2009 14:30:30 Page 189 Section 9.1/Technology and Classification of Composite Materials TABLE 9.1 189 Possible combinations of two-component composite materials. Primary Phase (matrix) Secondary phase (reinforcement) Metal Ceramic Polymer Elements (C, B) Metal Powder metal parts infiltrated with a second metal Cermetsa Fiber-reinforced metals Powder metal parts impregnated with polymer Fiber-reinforced metals Ceramic NA SiC whiskerreinforced Al2O3 NA NA Polymer Plastic molding compounds Steelbelted radial tires Plastic molding compounds Fiberglass-reinforced plastic Plastic molding compounds Kevlar-reinforced epoxy Rubber with carbon black B or C fiber-reinforced plastic NA ¼ Not applicable currently. a Cermets include cemented carbides. include two-phase structures consisting of components of the same material type, such as fibers of Kevlar (polymer) in a plastic (polymer) matrix. In other composites the imbedded material is an element such as carbon or boron. The classification system for composite materials used in this book is based on the matrix phase. We list the classes here and discuss them in Sections 9.2 through 9.4: 1. Metal Matrix Composites (MMCs) include mixtures of ceramics and metals, such as cemented carbides and other cermets, as well as aluminum or magnesium reinforced by strong, high stiffness fibers. 2. Ceramic Matrix Composites (CMCs) are the least common category. Aluminum oxide and silicon carbide are materials that can be imbedded with fibers for improved properties, especially in high temperature applications. 3. Polymer Matrix Composites (PMCs). Thermosetting resins are the most widely used polymers in PMCs. Epoxy and polyester are commonly mixed with fiber reinforcement, and phenolic is mixed with powders. Thermoplastic molding compounds are often reinforced, usually with powders (Section 8.1.5). The classification can be applied to traditional composites as well as synthetics.Concrete is a ceramic matrix composite, while asphalt and wood are polymer matrix composites. The matrix material serves several functions in the composite. First, it provides the bulk form of the part or product made of the composite material. Second, it holds the imbedded phase in place, usually enclosing and often concealing it. Third, when a load is applied, the matrix shares the load with the secondary phase, in some cases deforming so that the stress is essentially born by the reinforcing agent. 9.1.2 THE REINFORCING PHASE It is important to understand that the role played by the secondary phase is to reinforce the primary phase. The imbedded phase is most commonly one of the shapes illustrated in Figure 9.1: fibers, particles, or flakes. In addition, the secondary phase can take the form of an infiltrated phase in a skeletal or porous matrix. Fibers Fibers are filaments of reinforcing material, generally circular in cross-section, although alternative shapes are sometimes used (e.g., tubular, rectangular, hexagonal). 11/11/2009 190 14:30:30 Page 190 Chapter 9/Composite Materials FIGURE 9.1 Possible physical shapes of imbedded phases in composite materials: (a) fiber, (b) particle, and (c) flake. Diameters range from less than 0.0025 mm (0.0001 in) to about 0.13 mm (0.005 in), depending on material. Fiber reinforcement provides the greatest opportunity for strength enhancement of composite structures. In fiber-reinforced composites, the fiber is often considered to be the principal constituent since it bears the major share of the load. Fibers are of interest as reinforcing agents because the filament form of most materials is significantly stronger than the bulk form. The effect of fiber diameter on tensile strength can be seen in Figure 9.2. As diameter is reduced, the material becomes oriented in the direction of the fiber axis and the probability of defects in the structure decreases significantly. As a result, tensile strength increases dramatically. Fibers used in composites can be either continuous or discontinuous. Continuous fibers are very long; in theory, they offer a continuous path by which a load can be carried by the composite part. In reality, this is difficult to achieve due to variations in the fibrous material and processing. Discontinuous fibers (chopped sections of continuous fibers) are short lengths (L/D  100). An important type of discontinuous fiber are whiskers—hair-like single crystals with diameters down to about 0.001 mm (0.00004 in) and very high strength. Fiber orientation is another factor in composite parts. We can distinguish three cases, illustrated in Figure 9.3: (a) one-dimensional reinforcement, in which maximum strength and stiffness are obtained in the direction of the fiber; (b) planar reinforcement, Fiber diameter, in. 0.0003 0.0004 0.0005 400 FIGURE 9.2 Relationship between tensile strength and diameter for a carbon fiber. (Source: [1]). Other filament materials show similar relationships. FIGURE 9.3 Fiber orientation in composite materials: (a) onedimensional, continuous fibers; (b) planar, continuous fibers in the form of a woven fabric; and (c) random, discontinuous fibers. 2500 300 2000 1500 200 0.006 0.008 0.010 0.012 Fiber diameter, mm Tensile strength, 1000 lb/in.2 3000 Tensile strength, MPa E1C09 E1C09 11/11/2009 14:30:31 Page 191 Section 9.1/Technology and Classification of Composite Materials 191 TABLE 9.2 Typical properties of fiber materials used as reinforcement in composites. Diameter Tensile Strength Elastic Modulus Fiber Material mm milsa MPa lb/in2 GPa lb/in2 Metal: Steel Metal: Tungsten Ceramic: Al2O3 Ceramic: SiC Ceramic: E-glass Ceramic: S-glass Polymer: Kevlar Element: Carbon Element: Boron 0.13 0.013 0.02 0.13 0.01 0.01 0.013 0.01 0.14 5.0 0.5 0.8 5.0 0.4 0.4 0.5 0.4 5.5 1000 4000 1900 3275 3450 4480 3450 2750 3100 150,000 580,000 275,000 475,000 500,000 650,000 500,000 400,000 450,000 206 407 380 400 73 86 130 240 393 30  106 59  106 55  106 58  106 10  106 12  106 19  106 35  106 57  106 a 1 mil ¼ 0.001 in. Compiled from [3], [7], [11], and other sources. Note that strength depends on fiber diameter (Figure 9.2); the properties in this table must be interpreted accordingly. in some cases in the form of a two-dimensional woven fabric; and (c) random or threedimensional in which the composite material tends to possess isotropic properties. Various materials are used as fibers in fiber-reinforced composites: metals, ceramics, polymers, carbon, and boron. The most important commercial use of fibers is in polymer composites. However, use of fiber-reinforced metals and ceramics is growing. Following is a survey of the important types of fiber materials, with properties listed in Table 9.2: å Glass—The most widely used fiber in polymers, the term fiberglass is applied to denote glass fiber-reinforced plastic (GFRP). The two common glass fibers are Eglass and S-glass (compositions listed in Table 7.4). E-glass is strong and low cost, but its modulus is less than other fibers. S-glass is stiffer, and its tensile strength is one of the highest of all fiber materials; however, it is more expensive than E-glass. å Carbon—Carbon (Section 7.5.1) can be made into high-modulus fibers. Besides stiffness, other attractive properties include low-density and low-thermal expansion. C-fibers are generally a combination of graphite and amorphous carbon. å Boron—Boron (Section 7.5.3) has a very high elastic modulus, but its high cost limits applications to aerospace components in which this property (and others) are critical. å Kevlar 49—This is the most important polymer fiber; it is a highly crystalline aramid, a member of the polyamide family (Section 8.2.2). Its specific gravity is low, giving it one of the highest strength-to-weight ratios of all fibers. å Ceramics—Silicon carbide (SiC) and aluminum oxide (Al2O3) are the main fiber materials among ceramics. Both have high elastic moduli and can be used to strengthen low-density, low-modulus metals such as aluminum and magnesium. å Metal—Steel filaments, both continuous and discontinuous, are used as reinforcing fibers in plastics. Other metals are currently less common as reinforcing fibers. Particles and Flakes A second common shape of the imbedded phase is particulate, ranging in size from microscopic to macroscopic. Particles are an important material form for metals and ceramics; we discuss the characterization and production of engineering powders in Chapters 16 and 17. The distribution of particles in the composite matrix is random, and therefore strength and other properties of the composite material are usually isotropic. The strengthening mechanism depends on particle size. The microscopic size is represented by very fine powders E1C09 11/11/2009 192 14:30:31 Page 192 Chapter 9/Composite Materials (around 1 mm) distributed in the matrix in concentrations of 15% or less. The presence of these powders results in dispersion-hardening of the matrix, in which dislocation movement in the matrix material is restricted by the microscopic particles. In effect, the matrix itself is strengthened, and no significant portion of the applied load is carried by the particles. As particle size increases to the macroscopic range, and the proportion of imbedded material increases to 25% and more, the strengthening mechanism changes. In this case, the applied load is shared between the matrix and the imbedded phase. Strengthening occurs due to the load-carrying ability of the particles and the bonding of particles in the matrix. This form of composite strengthening occurs in cemented carbides, in which tungsten carbide is held in a cobalt binder. The proportion of tungsten carbide (WC) in the cobalt (Co) matrix is typically 80% or more. Flakes are basically two-dimensional particles—small flat platelets. Two examples of this shape are the minerals mica (silicate of K and Al) and talc (Mg3Si4O10(OH)2), used as reinforcing agents in plastics. They are generally lower cost materials than polymers, and they add strength and stiffness to plastic molding compounds. Platelet sizes are usually in the range 0.01– to 1 mm (0.0004–0.040 in) across the flake, with a thickness of 0.001– to 0.005 mm (0.00004–0.00020 in). Infiltrated Phase The fourth form of imbedded phase occurs when the matrix has the form of a porous skeleton (like a sponge), and the second phase is simply a filler. In this case, the imbedded phase assumes the shape of the pores in the matrix. Metallic fillers are sometimes used to infiltrate the open porous structure of parts made by powder metallurgy techniques (Section 16.3.4), in effect creating a composite material. Oil-impregnated sintered PM components, such as bearings and gears, might be considered another example of this category. The Interface There is always an interface between constituent phases in a composite material. For the composite to operate effectively, the phases must bond where they join. In some cases, there is a direct bonding between the two ingredients, as suggested by Figure 9.4 FIGURE 9.4 Interfaces and interphases between phases in a composite material: (a) direct bonding between primary and secondary phases; (b) addition of a third ingredient to bond the primary and secondary phases and form an interphase; and (c) formation of an interphase by solution of the primary and secondary phases at their boundary. E1C09 11/11/2009 14:30:31 Page 193 Section 9.1/Technology and Classification of Composite Materials 193 (a). In other cases, a third ingredient is added to promote bonding of the two primary phases. Called an interphase, this third ingredient can be thought of as an adhesive. An important example is the coating of glass fibers to achieve adhesion with thermosetting resin in fiberglass-reinforced plastics. As illustrated in Figure 9.4(b), this case results in two interfaces, one on either boundary of the interphase. Finally, a third form of interface occurs when the two primary components are not completely insoluble in each other; in this case, the interphase is formed consisting of a solution of the phases, as in Figure 9.4(c). An example occurs in cemented carbides (Section 9.2.1); at the high sintering temperatures used on these materials, some solubility results at the boundaries to create an interphase. 9.1.3 PROPERTIES OF COMPOSITE MATERIALS In the selection of a composite material, an optimum combination of properties is usually being sought, rather than one particular property. For example, the fuselage and wings of an aircraft must be lightweight as well as strong, stiff, and tough. Finding a monolithic material that satisfies these requirements is difficult. Several fiber-reinforced polymers possess this combination of properties. Another example is rubber. Natural rubber is a relatively weak material. In the early 1900s, it was discovered that by adding significant amounts of carbon black (almost pure carbon) to natural rubber, its strength is increased dramatically. The two ingredients interact to provide a composite material that is significantly stronger than either one alone. Rubber, of course, must also be vulcanized to achieve full strength. Rubber itself is a useful additive in polystyrene. One of the distinctive and disadvantageous properties of polystyrene is its brittleness. Although most other polymers have considerable ductility, polystyrene has virtually none. Rubber (natural or synthetic) can be added in modest amounts (5%–15%) to produce high-impact polystyrene, which has much superior toughness and impact strength. Properties of a composite material are determined by three factors: (1) the materials used as component phases in the composite, (2) the geometric shapes of the constituents and resulting structure of the composite system, and (3) the manner in which the phases interact with one another. Rule of Mixtures The properties of a composite material are a function of the starting materials. Certain properties of a composite material can be computed by means of a rule of mixtures, which involves calculating a weighted average of the constituent material properties. Density is an example of this averaging rule. The mass of a composite material is the sum of the masses of the matrix and reinforcing phases: mc ¼ mm þ mr ð9:1Þ where m ¼ mass, kg (lb); and the subscripts c, m, and r indicate composite, matrix, and reinforcing phases, respectively. Similarly, the volume of the composite is the sum of its constituents: Vc ¼ Vm þ Vr þ Vv ð9:2Þ where V ¼ volume, cm3 (in3). Vv is the volume of any voids in the composite (e.g., pores). The density of the composite is the mass divided by the volume. rc ¼ mc mm þ mr ¼ Vc Vc ð9:3Þ E1C09 11/11/2009 194 14:30:31 Page 194 Chapter 9/Composite Materials Because the masses of the matrix and reinforcing phase are their respective densities multiplied by their volumes, mm ¼ rm V m and mr ¼ rr V r we can substitute these terms into Eq. (9.3) and conclude that rc ¼ f m rm þ f r rr ð9:4Þ where f m ¼ V m =V c and f r ¼ V r =V c are simply the volume fractions of the matrix and reinforcing phases. Fiber-Reinforced Composites Determining mechanical properties of composites from constituent properties is usually more involved. The rule of mixtures can sometimes be used to estimate the modulus of elasticity of a fiber-reinforced composite made of continuous fibers where Ec is measured in the longitudinal direction. The situation is depicted in Figure 9.5(a); we assume that the fiber material is much stiffer than the matrix and that the bonding between the two phases is secure. Under this model, the modulus of the composite can be predicted as follows: Ec ¼ f m Em þ f r Er ð9:5Þ where Ec, Em, and Er are the elastic moduli of the composite and its constituents, MPa (lb/ in2); and fm and fr are again the volume fractions of the matrix and reinforcing phase. The effect of Eq. (9.5) is seen in Figure 9.5(b). Perpendicular to the longitudinal direction, the fibers contribute little to the overall stiffness except for their filling effect. The composite modulus can be estimated in this FIGURE 9.5 (a) Model of a fiber-reinforced composite material showing direction in which elastic modulus is being estimated by the rule of mixtures. (b) Stress–strain relationships for the composite material and its constituents. The fiber is stiff but brittle, while the matrix (commonly a polymer) is soft but ductile. The composite’s modulus is a weighted average of its components’ moduli. But when the reinforcing fibers fail, the composite does likewise. 14:30:31 Page 195 195 FIGURE 9.6 Variation in elastic modulus and tensile strength as a function of direction of measurement relative to longitudinal axis of carbon fiber-reinforced epoxy composite. (Source: [7]). 30 200 25 150 20 15 100 10 50 Ec 0 30 60 Fiber angle, degrees 5 Tensile strength, MPa 35 Ec Elastic modulus, lb/in.2 250 106 Section 9.1/Technology and Classification of Composite Materials TS 80 600 60 400 40 200 0 90 20 30 60 Fiber angle, degrees Tensile strength, ksi 11/11/2009 Elastic modulus, GPa E1C09 90 direction using the following: E0c ¼ Em Er f m Er þ f r Em ð9:6Þ where Ec0 ¼ elastic modulus perpendicular to the fiber direction, MPa (lb/in2). Our two equations for Ec demonstrate the significant anisotropy of fiber-reinforced composites. This directional effect can be seen in Figure 9.6 for a fiber-reinforced polymer composite, in which both elastic modulus and tensile strength are measured relative to fiber direction. Fibers illustrate the importance of geometric shape. Most materials have tensile strengths several times greater in a fibrous form than in bulk. However, applications of fibers are limited by surface flaws, buckling when subjected to compression, and the inconvenience of the filament geometry when a solid component is needed. By imbedding the fibers in a polymer matrix, a composite material is obtained that avoids the problems of fibers but utilizes their strengths. The matrix provides the bulk shape to protect the fiber surfaces and resist buckling; and the fibers lend their high strength to the composite. When a load is applied, the low-strength matrix deforms and distributes the stress to the high-strength fibers, which then carry the load. If individual fibers break, the load is redistributed through the matrix to other fibers. 9.1.4 OTHER COMPOSITE STRUCTURES Our model of a composite material described above is one in which a reinforcing phase is imbedded in a matrix phase, the combination having properties that are superior in certain respects to either of the constituents alone. However, composites can take alternative forms that do not fit this model, some of which are of considerable commercial and technological importance. A laminar composite structure consists of two or more layers bonded together to form an integral piece, as in Figure 9.7(a). The layers are usually thick enough that this composite can be readily identified—not always the case with other composites. The layers are often of different materials, but not necessarily. Plywood is such an example; the layers are of the same wood, but the grains are oriented differently to increase overall strength of the laminated piece. A laminar composite often uses different materials in its layers to gain the advantage of combining the particular properties of each. In some cases, the layers themselves may be composite materials. We have mentioned that wood is a composite material; therefore, plywood is a laminar composite structure in which the layers themselves are composite materials. A list of examples of laminar composites is compiled in Table 9.3. E1C09 11/11/2009 196 14:30:32 Page 196 Chapter 9/Composite Materials Foam material FIGURE 9.7 Laminar composite structures: (a) conventional laminar structure; (b) sandwich structure using a foam core, and (c) honeycomb sandwich structure. TABLE 9.3 (a) (b) Honey comb (c) Examples of laminar composite structures. Laminar Composite Description (reference in text if applicable) Automotive tires A tire consists of multiple layers bonded together; the layers are composite materials (rubber reinforced with carbon black), and the plies consist of rubber-impregnated fabrics (Chapter 14). A lightweight honeycomb structure is bonded on either face to thin sheets, as in Figure 9.7(c). Multilayered fiber-reinforced plastic panels are used for aircraft, automobile body panels, and boat hulls (Chapter 15). Alternating sheets of wood are bonded together at different orientations for improved strength. Layers of copper and reinforced plastic are used for electrical conductivity and insulation, respectively (Section 36.2). Skis are laminar composite structures consisting of multiple layers of metals, particle board, and phenolic plastic. Two layers of glass on either side of a sheet of tough plastic (Section 12.3.1). Honeycomb sandwich Fiber-reinforced polymers Plywood Printed circuit boards Snow skis Windshield glass The sandwich structure is sometimes distinguished as a special case of the laminar composite structure. It consists of a relatively thick core of low-density material bonded on both faces to thin sheets of a different material. The low-density core may be a foamed material, as in Figure 9.7(b), or a honeycomb, as in (c). The reason for using a sandwich structure is to obtain a material with high strength-to-weight and stiffness-to-weight ratios. 9.2 METAL MATRIX COMPOSITES Metal matrix composites (MMCs) consist of a metal matrix reinforced by a second phase. Common reinforcing phases include (1) particles of ceramic and (2) fibers of various materials, including other metals, ceramics, carbon, and boron. MMCs of the first type are commonly called cermets. 9.2.1 CERMETS A cermet1 is a composite material in which a ceramic is contained in a metallic matrix. The ceramic often dominates the mixture, sometimes ranging up to 96% by volume. 1 The word ‘‘cermet’’ was first used in the English language around 1948. E1C09 11/11/2009 14:30:32 Page 197 Section 9.2/Metal Matrix Composites 197 FIGURE 9.8 Photomicrograph (1500x) of cemented carbide with 85% WC and 15% Co. (Photo courtesy of Kennametal Inc.) Bonding can be enhanced by slight solubility between phases at the elevated temperatures used in processing these composites. Cermets can be subdivided into (1) cemented carbides and (2) oxide-based cermets. Cemented Carbides Cemented carbides are composed of one or more carbide compounds bonded in a metallic matrix. The term cermet is not used for all of these materials, even though it is technically correct. The common cemented carbides are based on tungsten carbide (WC), titanium carbide (TiC), and chromium carbide (Cr3C2). Tantalum carbide (TaC) and others are also used but less commonly. The principal metallic binders are cobalt and nickel. We have previously discussed the carbide ceramics (Section 7.3.2); they constitute the principal ingredient in cemented carbides, typically ranging in content from 80% to 95% of total weight. Cemented carbide parts are produced by particulate processing techniques (Section 17.3). Cobalt is the binder used for WC (see Figure 9.8), and nickel is a common binder for TiC and Cr3C2. Even though the binder constitutes only about 5% to 15%, its effect on mechanical properties is significant in the composite material. Using WC–Co as an example, as the percentage of Co is increased, hardness is decreased and transverse rupture strength (TRS) is increased, as shown in Figure 9.9. TRS correlates with toughness of the WC–Co composite. Cutting tools are the most common application of cemented carbides based on tungsten carbide. Other applications of WC–Co cemented carbides include wire drawing dies, rock-drilling bits and other mining tools, dies for powder metallurgy, indenters for hardness testers, and other applications where hardness and wear resistance are critical requirements. Titanium carbide cermets are used principally for high temperature applications. Nickel is the preferred binder; its oxidation resistance at high temperatures is superior to that of cobalt. Applications include gas-turbine nozzle vanes, valve seats, thermocouple protection tubes, torch tips, and hot-working spinning tools [11]. TiC–Ni is also used as a cutting tool material in machining operations. 198 14:30:32 Page 198 Chapter 9/Composite Materials 94 FIGURE 9.9 Typical plot of hardness and transverse rupture strength as a function of cobalt content. 93 92 91 90 89 Transverse rupture strength, MPa 11/11/2009 Hardness, HRA E1C09 2800 Transverse rupture strength 2450 2100 1750 1400 Hardness 1050 0 3 6 9 12 15 Cobalt content, % Compared with WC–Co cemented carbides, nickel-bonded chromium carbides are more brittle, but have excellent chemical stability and corrosion resistance. This combination, together with good wear resistance, makes it suitable for applications such as gage blocks, valve liners, spray nozzles, and bearing seal rings [11]. Oxide-based Cermets Most of these composites utilize Al2O3 as the particulate phase; MgO is another oxide sometimes used. A common metal matrix is chromium, although other metals can also be used as binders. Relative proportions of the two phases vary significantly, with the possibility for the metal binder to be the major ingredient. Applications include cutting tools, mechanical seals, and thermocouple shields. 9.2.2 FIBER-REINFORCED METAL MATRIX COMPOSITES These MMCs are of interest because they combine the high tensile strength and modulus of elasticity of a fiber with metals of low density, thus achieving good strength-to-weight and modulus-to-weight ratios in the resulting composite material. Typical metals used as the low-density matrix are aluminum, magnesium, and titanium. Some of the important fiber materials used in the composite include Al2O3, boron, carbon, and SiC. Properties of fiber-reinforced MMCs are anisotropic, as expected. Maximum tensile strength in the preferred direction is obtained by using continuous fibers bonded strongly to the matrix metal. Elastic modulus and tensile strength of the composite material increase with increasing fiber volume. MMCs with fiber reinforcement have good high-temperature strength properties; and they are good electrical and thermal conductors. Applications have largely been components in aircraft and turbine machinery, where these properties can be exploited. 9.3 CERAMIC MATRIX COMPOSITES Ceramics have certain attractive properties: high stiffness, hardness, hot hardness, and compressive strength; and relatively low density. Ceramics also have several faults: low toughness and bulk tensile strength, and susceptibility to thermal cracking. Ceramic matrix composites (CMCs) represent an attempt to retain the desirable properties of ceramics E1C09 11/11/2009 14:30:33 Page 199 Section 9.4/Polymer Matrix Composites 199 FIGURE 9.10 Highly magnified electron microscopy photograph (3000x) showing fracture surface of SiC whisker reinforced ceramic (Al2O3) used as cutting tool material. (Courtesy of Greenleaf Corporation, Saegertown, Pennsylvania.) while compensating for their weaknesses. CMCs consist of a ceramic primary phase imbedded with a secondary phase. To date, most development work has focused on the use of fibers as the secondary phase. Success has been elusive. Technical difficulties include thermal and chemical compatibility of the constituents in CMCs during processing. Also, as with any ceramic material, limitations on part geometry must be considered. Ceramic materials used as matrices include alumina (Al2O3), boron carbide (B4C), boron nitride (BN), silicon carbide (SiC), silicon nitride (Si3N4), titanium carbide (TiC), and several types of glass [10]. Some of these materials are still in the development stage as CMC matrices. Fiber materials in CMCs include carbon, SiC, and Al2O3. The reinforcing phase in current CMC technology consists of either short fibers, such as whiskers, or long fibers. Products with short fibers have been successfully fabricated using particulate processing methods (Chapter 17), the fibers being treated as a form of powder in these materials. Although there are performance advantages in using long fibers as reinforcement in ceramic matrix composites, development of economical processing techniques for these materials has been difficult. One promising commercial application of CMCs is in metal-cutting tools as a competitor of cemented carbides, as illustrated in Figure 9.10. The composite tool material has whiskers of SiC in a matrix of Al2O3. Other potential applications are in elevated temperatures and environments that are chemically corrosive to other materials. 9.4 POLYMER MATRIX COMPOSITES A polymer matrix composite (PMC) consists of a polymer primary phase in which a secondary phase is imbedded in the form of fibers, particles, or flakes. Commercially, PMCs are the most important of the three classes of synthetic composites. They include most plastic molding compounds, rubber reinforced with carbon black, and fiberreinforced polymers (FRPs). Of the three, FRPs are most closely identified with the term composite. If one mentions ‘‘composite material’’ to a design engineer, FRP is usually the composite that comes to mind. Our video clip on composite materials and manufacturing provides an overview of fiber-reinforced polymer composites. VIDEO CLIP View the segment titled Composite Materials and Manufacturing. E1C09 11/11/2009 200 14:30:33 Page 200 Chapter 9/Composite Materials 9.4.1 FIBER-REINFORCED POLYMERS A fiber-reinforced polymer is a composite material consisting of a polymer matrix imbedded with high-strength fibers. The polymer matrix is usually a thermosetting plastic such as unsaturated polyester or epoxy, but thermoplastic polymers, such as nylons (polyamides), polycarbonate, polystyrene, and polyvinylchloride, are also used. In addition, elastomers are also reinforced by fibers for rubber products such as tires and conveyor belts. Fibers in PMCs come in various forms: discontinuous (chopped), continuous, or woven as a fabric. Principal fiber materials in FRPs are glass, carbon, and Kevlar 49. Less common fibers include boron, SiC, and Al2O3, and steel. Glass (in particular E-glass) is the most common fiber material in today’s FRPs; its use to reinforce plastics dates from around 1920. The term advanced composites is sometimes used in connection with FRPs developed since the late 1960s that use boron, carbon, or Kevlar, as the reinforcing fibers [13]. Epoxy is the common matrix polymer. These composites generally have high fiber content (>50% by volume) and possess high strength and modulus of elasticity. When two or more fiber materials are combined in the FRP composite, it is called a hybrid composite. Advantages cited for hybrids over conventional or advanced FRPs include balanced strength and stiffness, improved toughness and impact resistance, and reduced weight [11]. Advanced and hybrid composites are used in aerospace applications. The most widely used form of the FRP itself is a laminar structure, made by stacking and bonding thin layers of fiber and polymer until the desired thickness is obtained. By varying the fiber orientation among the layers, a specified level of anisotropy in properties can be achieved in the laminate. This method is used to form parts of thin cross section, such as aircraft wing and fuselage sections, automobile and truck body panels, and boat hulls. Properties There are a number of attractive features that distinguish fiber-reinforced plastics as engineering materials. Most notable are (1) high strength-to-weight ratio, (2) high modulus-to-weight ratio, and (3) low specific gravity. A typical FRP weighs only about onefifth as much as steel; yet strength and modulus are comparable in the fiber direction. Table 9.4 compares these properties for several FRPs, steels, and an aluminum alloy. Properties listed in Table 9.4 depend on the proportion of fibers in the composite. Both tensile strength and elastic modulus increase as the fiber content is increased, by Eq. (9.5). Other properties and characteristics of fiber-reinforced plastics include (4) good fatigue strength; (5) good corrosion resistance, although polymers are soluble in various chemicals; (6) low thermal expansion for many FRPs, leading to good dimensional stability; and (7) significant anisotropy in TABLE 9.4 Comparison of typical properties of fiber-reinforced plastics and representative metal alloys. Material Low-C steel Alloy steel (heat treated) Aluminum alloy (heat treated) FRP: fiberglass in polyester FRP: Carbon in epoxyb FRP: Carbon in epoxyc FRP: Kevlar in epoxy matrix a Specific Gravity (SG) 7.87 7.87 2.70 1.50 1.55 1.65 1.40 Tensile Strength (TS) MPa 345 3450 415 205 1500 1200 1380 lb/in2 50,000 500,000 60,000 30,000 220,000 175,000 200,000 Elastic Modulus (E) Indexa GPa lb/in2 207 207 69 69 140 214 76 30  10 30  106 10  106 10  106 20  106 31  106 11  106 6 TS/SG E/SG 1.0 10.0 3.5 3.1 22.3 16.7 22.5 1.0 1.0 1.0 1.7 3.4 4.9 2.1 Indices are relative tensile strength-to-weight (TS/SG) and elastic modulus-to-weight (E/SG) ratios compared to low-C steel as the base (index ¼ 1.0 for the base). b High tensile-strength carbon fibers used in FRP. c High modulus carbon fibers used in FRP. Compiled from [3], [7], and other sources. Properties are measured in the fiber direction. E1C09 11/11/2009 14:30:33 Page 201 Section 9.5/Guide to Processing Composite Materials 201 properties. With regard to this last feature, the mechanical properties of the FRPs given in Table 9.4 are in the direction of the fiber. As previously noted, their values are significantly less when measured in a different direction. Applications During the last three decades there has been a steady growth in the application of fiber-reinforced polymers in products requiring high strength and low weight, often as substitutions for metals. The aerospace industry is one of the biggest users of advanced composites. Designers are continually striving to reduce aircraft weight to increase fuel efficiency and payload capacity. Applications of advanced composites in both military and commercial aircraft have increased steadily. Much of the structural weight of today’s airplanes and helicopters consists of FRPs. The new Boeing 787 Dreamliner features 50% (by weight) composite (carbon fiber-reinforced plastic). That’s about 80% of the volume of the aircaft. Composites are used for the fuselage, wings, tail, doors, and interior. By comparison, Boeing’s 777 has only about 12% composites (by weight). The automotive industry is another important user of FRPs. The most obvious applications are FRP body panels for cars and truck cabs. A notable example is the Chevrolet Corvette that has been produced with FRP bodies for decades. Less apparent applications are in certain chassis and engine parts. Automotive applications differ from those in aerospace in two significant respects. First, the requirement for high strength-toweight ratio is less demanding than for aircraft. Car and truck applications can use conventional fiberglass reinforced plastics rather than advanced composites. Second, production quantities are much higher in automotive applications, requiring more economical methods of fabrication. Continued use of low-carbon sheet steel in automobiles in the face of FRP’s advantages is evidence of the low cost and processability of steel. FRPs have been widely adopted for sports and recreational equipment. Fiberglass reinforced plastic has been used for boat hulls since the 1940s. Fishing rods were another early application. Today, FRPs are represented in a wide assortment of sports products, including tennis rackets, golf club shafts, football helmets, bows and arrows, skis, and bicycle wheels. 9.4.2 OTHER POLYMER MATRIX COMPOSITES In addition to FRPs, other PMCs contain particles, flakes, and short fibers. Ingredients of the secondary phase are called fillers when used in polymer molding compounds (Section 8.1.5). Fillers divide into two categories: (1) reinforcements and (2) extenders. Reinforcing fillers serve to strengthen or otherwise improve mechanical properties of the polymer. Common examples include: wood flour and powdered mica in phenolic and amino resins to increase strength, abrasion resistance, and dimensional stability; and carbon black in rubber to improve strength, wear, and tear resistance. Extenders simply increase the bulk and reduce the cost-per-unit weight of the polymer, but have little or no effect on mechanical properties. Extenders may be formulated to improve molding characteristics of the resin. Foamed polymers (Section 13.11) are a form of composite in which gas bubbles are imbedded in a polymer matrix. Styrofoam and polyurethane foam are the most common examples. The combination of near-zero density of the gas and relatively low density of the matrix makes these materials extremely light weight. The gas mixture also lends very low thermal conductivity for applications in which heat insulation is required. 9.5 GUIDE TO PROCESSING COMPOSITE MATERIALS Composite materials are formed into shapes by many different processing technologies. The two phases are typically produced separately before being combined into the E1C09 11/11/2009 202 14:30:33 Page 202 Chapter 9/Composite Materials composite part geometry. The matrix phases are generally processed by the technologies described in Chapters 6, 7, and 8 for metals, ceramics, and polymers. Processing methods for the imbedded phase depend on geometry. Fiber production is described in Section 12.2.3 for glass and Section 13.4 for polymers. Fiber production methods for carbon, boron, and other materials are summarized in Table 15.1. Powder production for metals is described in Section 16.2 and for ceramics in Section 17.1.1. Processing techniques to fabricate MMC and CMC components, are similar to those used for powdered metals and ceramics (Chapters 16 and 17). We deal with the processing of cermets specifically in Section 17.3. Molding processes are commonly performed on PMCs, both particle and chopped fiber types. Molding processes for these composites are the same as those used for polymers (Chapter 13). Other more specialized processes for polymer matrix composites, fiber-reinforced polymers in particular, are described in Chapter 15. Many laminated composite and honeycomb structures are assembled by adhesive bonding (Section 31.3). REFERENCES [1] Chawla, K. K. Composite Materials: Science and Engineering, 3rd ed. Springer-Verlag, New York, 2008. [2] Delmonte, J. Metal-Polymer Composites. Van Nostrand Reinhold, New York, 1990. [3] Engineering Materials Handbook, Vol. 1, Composites. ASM International, Metals Park, Ohio, 1987. [4] Flinn, R. A., and Trojan, P. K. Engineering Materials and Their Applications, 5th ed. John Wiley & Sons, New York, 1995. [5] Greenleaf Corporation. WG-300—Whisker Reinforced Ceramic/Ceramic Composites [marketing literature]. Saegertown, Pennsylvania, YEAR??. [6] Hunt, W. H., Jr., and Herling, S. R. ‘‘Aluminum Metal-Matrix Composites,’’ Advanced Materials & Processes, February 2004, pp. 39–42. [7] Mallick, P. K. Fiber-Reinforced Composites: Materials, Manufacturing, and Designs, 3rd ed. CRC Taylor & Francis, Boca Raton, Florida, 2007. [8] McCrum, N. G., Buckley, C. P., and Bucknall, C. B. Principles of Polymer Engineering, 2nd ed. Oxford University Press, Oxford, UK, 1997. [9] Morton-Jones, D. H. Polymer Processing. Chapman and Hall, London, 1989. [10] Naslain, R., and Harris, B. (eds.). Ceramic Matrix Composites. Elsevier Applied Science, London and New York, 1990. [11] Schwartz, M. M. Composite Materials Handbook, 2nd ed. McGraw-Hill Book Company, New York, 1992. [12] Tadmor, Z., and Gogos, C. G. Principles of Polymer Processing. Wiley-Interscience, Hoboken, New Jersey, 2006. [13] Wick, C., and Veilleux R. F. (eds.). Tool and Manufacturing Engineers Handbook, 4th ed, Volume III—, Materials, Finishing, and Coating, Chapter 8. Society of Manufacturing Engineers, Dearborn, Michigan, 1985. [14] Wikipedia. ‘‘Boeing 787.’’ Available at: wikipedia. org/wiki/Boeing_787. [15] Zweben, C., Hahn, H. T., and Chou, T-W. Delaware Composites Design Encyclopedia, Vol. 1, Mechanical Behavior and Properties of Composite Materials. Technomic Publishing, Lancaster, Pennsylvania, 1989. REVIEW QUESTIONS 9.1. What is a composite material? 9.2. Identify some of the characteristic properties of composite materials. 9.3. What does the term anisotropic mean? 9.4. How are traditional composites distinguished from synthetic composites? 9.5. Name the three basic categories of composite materials. 9.6. What are the common forms of the reinforcing phase in composite materials? 9.7. What is a whisker? 9.8. What are the two forms of sandwich structure among laminar composite structures? Briefly describe each. 9.9. Give some examples of commercial products which are laminar composite structures. E1C09 11/11/2009 14:30:34 Page 203 Problems 9.10. What are the three general factors that determine the properties of a composite material? 9.11. What is the rule of mixtures? 9.12. What is a cermet? 9.13. Cemented carbides are what class of composites? 9.14. What are some of the weaknesses of ceramics that might be corrected in fiber-reinforced ceramic matrix composites? 203 9.15. What is the most common fiber material in fiberreinforced plastics? 9.16. What does the term advanced composites mean? 9.17. What is a hybrid composite? 9.18. Identify some of the important properties of fiberreinforced plastic composite materials. 9.19. Name some of the important applications of FRPs. 9.20. What is meant by the term interface in the context of composite materials? MULTIPLE CHOICE QUIZ There are 19 correct answers in the following multiple-choice questions (some questions have multiple answers that are correct). To attain a perfect score on the quiz, all correct answers must be given. Each correct answer is worth 1 point. Each omitted answer or wrong answer reduces the score by 1 point, and each additional answer beyond the correct number of answers reduces the score by 1 point. Percentage score on the quiz is based on the total number of correct answers. 9.1. Anisotropic means which one of the following: (a) composite materials with composition consisting of more than two materials, (b) properties are the same in every direction, (c) properties vary depending on the direction in which they are measured, or (d) strength and other properties are a function of curing temperature? 9.2. The reinforcing phase is the matrix within which the secondary phase is imbedded: (a) true or (b) false? 9.3. Which one of the following reinforcing geometries offers the greatest potential for strength and stiffness improvement in the resulting composite material: (a) fibers, (b) flakes, (c) particles, or (d) infiltrated phase? 9.4. Wood is which one of the following composite types: (a) CMC, (b) MMC, or (c) PMC? 9.5. Which of the following materials are used as fibers in fiber-reinforced plastics (four best answers): (a) aluminum oxide, (b) boron, (c) cast iron, (d) E-glass, (e) epoxy, (f) Kevlar 49, (g) polyester, and (h) silicon? 9.6. Which of the following metals are used as the matrix material in fiber-reinforced MMCs (two best answers): (a) aluminum, (b) copper, (c) iron, (d) magnesium, and (e) zinc? 9.7. Which of the following metals are used as the matrix metals in nearly all WC cemented carbides and TiC cermets (two correct answers): (a) aluminum, (b) chromium, (c) cobalt, (d) lead, (e) nickel, (f) tungsten, and (g) tungsten carbide? 9.8. Ceramic matrix composites are designed to overcome which of the following weaknesses of ceramics (two best answers): (a) compressive strength, (b) hardness, (c) hot hardness, (d) modulus of elasticity, (e) tensile strength, and (f) toughness? 9.9. Which one of the following polymer types are most commonly used in polymer matrix composites: (a) elastomers, (b) thermoplastics, or (c) thermosets? 9.10. Which of the following materials are not composites (two correct answers): (a) cemented carbide, (b) phenolic molding compound, (c) plywood, (d) Portland cement, (e) rubber in automobile tires, (f) wood, and (g) 1020 steel? 9.11. In the Boeing 787 Dreamliner, what percentage of the aircraft consist of composite materials (two correct answers): (a) 12% by volume, (b) 20% by volume, (c) 50% by volume, (d) 80% by volume, (e) 12% by weight, (f) 20% by weight, (g) 50% by weight, and (h) 80% by weight? PROBLEMS 9.1. A fiberglass composite is composed of a matrix of vinyl ester and reinforcing fibers of E-glass. The volume fraction of E-glass is 35%. The remainder is vinyl ester. The density of the vinyl ester is 0.882 g/ cm3, and its modulus of elasticity is 3.60 GPa. The density of E-glass is 2.60 g/cm3, and its modulus of elasticity is 76.0 GPa. A section of composite 1.00 cm  50.00 cm  200.00 cm is fabricated with the E-glass fibers running longitudinal along the 200-cm direction. Assume there are no voids in the composite. Determine the (a) mass of vinyl ester in the section, (b) mass of E-glass fibers in the section, and (c) the density of the composite. E1C09 11/11/2009 204 14:30:34 Page 204 Chapter 9/Composite Materials 9.2. For problem 9.1, determine the modulus of elasticity in (a) the longitudinal direction of the glass fibers and (b) the perpendicular direction to the glass fibers. 9.3. A composite sample of carbon reinforced epoxy has dimensions of 12 in  12 in  0.25 in and mass of 1.8 lb. The carbon fibers have a modulus of elasticity of 50(106) lb/in2 and a density of 0.069 lb/in3. The epoxy matrix has modulus of elasticity of 0.61(106) lb/in2 and a density of 0.042 lb/in3. What is the volume fraction of (a) the carbon fibers and (b) the epoxy matrix in the sample? Assume there are no voids in the sample. 9.4. In problem 9.3, what is the predicted value for the modulus of elasticity (a) in the longitudinal direction and (b) the perpendicular to the carbon fibers? 9.5. A composite has a matrix of polyester with Kevlar49 fibers. The volume fractions of polyester and Kevlar are 60% and 40%, respectively. The Kevlar fibers have a modulus of elasticity of 60 GPa in the longitudinal direction and 3 GPa in the transverse direction. The polyester matrix has a modulus of elasticity of 5.6 GPa in both directions. (a) Determine the modulus of elasticity for the composite in the longitudinal direction. (b) Determine the modulus of elasticity in the transverse direction. E1C10 11/11/2009 14:39:16 Page 205 Part III Solidification Processes 10 FUNDAMENTALS OF METAL CASTING Chapter Contents 10.1 Overview of Casting Technology 10.1.1 Casting Processes 10.1.2 Sand-Casting Molds 10.2 Heating and Pouring 10.2.1 Heating the Metal 10.2.2 Pouring the Molten Metal 10.2.3 Engineering Analysis of Pouring 10.2.4 Fluidity 10.3 Solidification and Cooling 10.3.1 Solidification of Metals 10.3.2 Solidification Time 10.3.3 Shrinkage 10.3.4 Directional Solidification 10.3.5 Riser Design In this part of the book, we consider those manufacturing processes in which the starting work material is either a liquid or is in a highly plastic condition, and a part is created through solidification of the material. Casting and molding processes dominate this category of shaping operations. With reference to Figure 10.1, the solidification processes can be classified according to the engineering material that is processed: (1) metals, (2) ceramics, specifically glasses,1 and (3) polymers and polymer matrix composites (PMCs). Casting of metals is covered in this and the following chapter. Glassworking is covered in Chapter 12, and polymer and PMC processing is treated in Chapters 13, 14, and 15. Casting is a process in which molten metal flows by gravity or other force into a mold where it solidifies in the shape of the mold cavity. The term casting is also applied to the part that is made by this process. It is one of the oldest shaping processes, dating back 6000 years (Historical Note 10.1). The principle of casting seems simple: melt the metal, pour it into a mold, and let it cool and solidify; yet there are many factors and variables that must be considered in order to accomplish a successful casting operation. Casting includes both the casting of ingots and the casting of shapes. The term ingot is usually associated with the primary metals industries; it describes a large casting that is simple in shape and intended for subsequent reshaping by 1 Among the ceramics, only glass is processed by solidification; traditional and new ceramics are shaped using particulate processes (Chapter 17). 205 E1C10 11/11/2009 206 14:39:16 Page 206 Chapter 10/Fundamentals of Metal Casting Expendable-mold casting Sand casting Other casting processes Casting of metals Permanent-mold casting Solidification processes Glassworking Extrusion and related processes Injection molding Processing of polymers and PMCs Other molding processes Special processes for PMCs FIGURE 10.1 Historical Note 10.1 Classification of solidification processes. Origins of casting C asting of metals can be traced back to around 4000 BCE. Gold was the first metal to be discovered and used by the early civilizations; it was malleable and could be readily hammered into shape at room temperature. There seemed to be no need for other ways to shape gold. It was the subsequent discovery of copper that gave rise to the need for casting. Although copper could be forged to shape, the process was more difficult (due to strain hardening) and limited to relatively simple forms. Historians believe that hundreds of years elapsed before the process of casting copper was first performed, probably by accident during the reduction of copper ore in preparation for hammering the metal into some useful form. Thus, through serendipity, the art of casting was born. It is likely that the discovery occurred in Mesopotamia, and the ‘‘technology’’ quickly spread throughout the rest of the ancient world. It was an innovation of significant importance in the history of mankind. Shapes much more intricate could be formed by casting than by hammering. More sophisticated tools and weapons could be fabricated. More detailed implements and ornaments could be fashioned. Fine gold jewelry could be made more beautiful and valuable than by previous methods. Alloys were first used for casting when it was discovered that mixtures of copper and tin (the alloy thus formed was bronze) yielded much better castings than copper alone. Casting permitted the creation of wealth to those nations that could perform it best. Egypt ruled the Western civilized world during the Bronze Age (nearly 2000 years) largely due to its ability to perform the casting process. Religion provided an important influence during the Dark Ages (circa 400 to 1400) for perpetuating the foundryman’s skills. Construction of cathedrals and churches required the casting of bells that were used in these structures. Indeed, the time and effort needed to cast the large bronze bells of the period helped to move the casting process from the realm of art toward the regimen of technology. Advances in melting and moldmaking techniques were made. Pit molding, in which the molds were formed in a deep pit located in front of the furnace to simplify the pouring process, was improved as a casting procedure. In addition, the bellfounder learned the relationships between the tone of the bell, which was the important measure of product quality, and its size, shape, thickness, and metal composition. Another important product associated with the development of casting was the cannon. Chronologically, it followed the bell, and therefore many E1C10 11/11/2009 14:39:16 Page 207 Section 10.1/Overview of Casting Technology of the casting techniques developed for bellfounding were applied to cannon making. The first cast cannon was made in Ghent, Belgium, in the year 1313—by a religious monk, of all people. It was made of bronze, and the bore was formed by means of a core during casting. Because of the rough bore surface created by the casting 207 process, these early guns were not accurate and had to be fired at relatively close range to be effective. It was soon realized that accuracy and range could be improved if the bore were made smooth by machining the surface. Quite appropriately, this machining process was called boring (Section 22.1.5). processes such as rolling or forging. Ingot casting was discussed in Chapter 6. Shape casting involves the production of more complex geometries that are much closer to the final desired shape of the part or product. It is with the casting of shapes rather than ingots that this chapter and the next are concerned. A variety of shape casting methods are available, thus making it one of the most versatile of all manufacturing processes. Among its capabilities and advantages are the following: å Casting can be used to create complex part geometries, including both external and internal shapes. å Some casting processes are capable of producing parts to net shape. No further manufacturing operations are required to achieve the required geometry and dimensions of the parts. Other casting processes are near net shape, for which some additional shape processing is required (usually machining) in order to achieve accurate dimensions and details. å Casting can be used to produce very large parts. Castings weighing more than 100 tons have been made. å The casting process can be performed on any metal that can be heated to the liquid state. å Some casting methods are quite suited to mass production. There are also disadvantages associated with casting—different disadvantages for different casting methods. These include limitations on mechanical properties, porosity, poor dimensional accuracy and surface finish for some casting processes, safety hazards to humans when processing hot molten metals, and environmental problems. Parts made by casting processes range in size from small components weighing only a few ounces up to very large products weighing tons. The list of parts includes dental crowns, jewelry, statues, wood-burning stoves, engine blocks and heads for automotive vehicles, machine frames, railway wheels, frying pans, pipes, and pump housings. All varieties of metals can be cast, ferrous and nonferrous. Casting can also be used on other materials such as polymers and ceramics; however, the details are sufficiently different that we postpone discussion of the casting processes for these materials until later chapters. This chapter and the next deal exclusively with metal casting. Here we discuss the fundamentals that apply to virtually all casting operations. In the following chapter, the individual casting processes are described, along with some of the product design issues that must be considered when making parts out of castings. 10.1 OVERVIEW OF CASTING TECHNOLOGY As a production process, casting is usually carried out in a foundry. A foundry is a factory equipped for making molds, melting and handling metal in molten form, performing the casting process, and cleaning the finished casting. The workers who perform the casting operations in these factories are called foundrymen. E1C10 11/11/2009 208 14:39:16 Page 208 Chapter 10/Fundamentals of Metal Casting 10.1.1 CASTING PROCESSES Discussion of casting logically begins with the mold. The mold contains a cavity whose geometry determines the shape of the cast part. The actual size and shape of the cavity must be slightly oversized to allow for shrinkage that occurs in the metal during solidification and cooling. Different metals undergo different amounts of shrinkage, so the mold cavity must be designed for the particular metal to be cast if dimensional accuracy is critical. Molds are made of a variety of materials, including sand, plaster, ceramic, and metal. The various casting processes are often classified according to these different types of molds. To accomplish a casting operation, the metal is first heated to a temperature high enough to completely transform it into a liquid state. It is then poured, or otherwise directed, into the cavity of the mold. In an open mold, Figure 10.2(a), the liquid metal is simply poured until it fills the open cavity. In a closed mold, Figure 10.2(b), a passageway, called the gating system, is provided to permit the molten metal to flow from outside the mold into the cavity. The closed mold is by far the more important category in production casting operations. As soon as the molten metal is in the mold, it begins to cool. When the temperature drops sufficiently (e.g., to the freezing point for a pure metal), solidification begins. Solidification involves a change of phase of the metal. Time is required to complete the phase change, and considerable heat is given up in the process. It is during this step in the process that the metal assumes the solid shape of the mold cavity and many of the properties and characteristics of the casting are established. Once the casting has cooled sufficiently, it is removed from the mold. Depending on the casting method and metal used, further processing may be required. This may include trimming the excess metal from the actual cast part, cleaning the surface, inspecting the product, and heat treatment to enhance properties. In addition, machining (Chapter 22) may be required to achieve closer tolerances on certain part features and to remove the cast surface. Casting processes divide into two broad categories, according to type of mold used: expendable-mold casting and permanent-mold casting. An expendable mold means that the mold in which the molten metal solidifies must be destroyed in order to remove the casting. These molds are made out of sand, plaster, or similar materials, whose form is maintained by using binders of various kinds. Sand casting is the most prominent example of the expendable-mold processes. In sand casting, the liquid metal is poured into a mold FIGURE 10.2 Two forms of mold: (a) open mold, simply a container in the shape of the desired part; and (b) closed mold, in which the mold geometry is more complex and requires a gating system (passageway) leading into the cavity. E1C10 11/11/2009 14:39:16 Page 209 Section 10.1/Overview of Casting Technology 209 made of sand. After the metal hardens, the mold must be sacrificed in order to recover the casting. A permanent mold is one that can be used over and over to produce many castings. It is made of metal (or, less commonly, a ceramic refractory material) that can withstand the high temperatures of the casting operation. In permanent-mold casting, the mold consists of two (or more) sections that can be opened to permit removal of the finished part. Die casting is the most familiar process in this group. More intricate casting geometries are generally possible with the expendable-mold processes. Part shapes in the permanent-mold processes are limited by the need to open the mold. On the other hand, some of the permanent mold processes have certain economic advantages in high production operations. We discuss the expendable-mold and permanent-mold casting processes in Chapter 11. 10.1.2 SAND-CASTING MOLDS Sand casting is by far the most important casting process. A sand-casting mold will be used to describe the basic features of a mold. Many of these features and terms are common to the molds used in other casting processes. Figure 10.2(b) shows the cross-sectional view of a typical sand-casting mold, indicating some of the terminology. The mold consists of two halves: cope and drag. The cope is the upper half of the mold, and the drag is the bottom half. These two mold parts are contained in a box, called a flask, which is also divided into two halves, one for the cope and the other for the drag. The two halves of the mold separate at the parting line. In sand casting (and other expendable-mold processes) the mold cavity is formed by means of a pattern, which is made of wood, metal, plastic, or other material and has the shape of the part to be cast. The cavity is formed by packing sand around the pattern, about half each in the cope and drag, so that when the pattern is removed, the remaining void has the desired shape of the cast part. The pattern is usually made oversized to allow for shrinkage of the metal as it solidifies and cools. The sand for the mold is moist and contains a binder to maintain its shape. The cavity in the mold provides the external surfaces of the cast part. In addition, a casting may have internal surfaces. These surfaces are determined by means of a core, a form placed inside the mold cavity to define the interior geometry of the part. In sand casting, cores are generally made of sand, although other materials can be used, such as metals, plaster, and ceramics. The gating system in a casting mold is the channel, or network of channels, by which molten metal flows into the cavity from outside the mold. As shown in the figure, the gating system typically consists of a downsprue (also called simply the sprue), through which the metal enters a runner that leads into the main cavity. At the top of the downsprue, a pouring cup is often used to minimize splash and turbulence as the metal flows into the downsprue. It is shown in our diagram as a simple cone-shaped funnel. Some pouring cups are designed in the shape of a bowl, with an open channel leading to the downsprue. In addition to the gating system, any casting in which shrinkage is significant requires a riser connected to the main cavity. The riser is a reservoir in the mold that serves as a source of liquid metal for the casting to compensate for shrinkage during solidification. The riser must be designed to freeze after the main casting in order to satisfy its function. As the metal flows into the mold, the air that previously occupied the cavity, as well as hot gases formed by reactions of the molten metal, must be evacuated so that the metal will completely fill the empty space. In sand casting, for example, the natural porosity of the sand mold permits the air and gases to escape through the walls of the cavity. In permanentmetal molds, small vent holes are drilled into the mold or machined into the parting line to permit removal of air and gases. E1C10 11/11/2009 210 14:39:17 Page 210 Chapter 10/Fundamentals of Metal Casting 10.2 HEATING AND POURING To perform a casting operation, the metal must be heated to a temperature somewhat above its melting point and then poured into the mold cavity to solidify. In this section, we consider several aspects of these two steps in casting. 10.2.1 HEATING THE METAL Heating furnaces of various kinds (Section 11.4.1) are used to heat the metal to a molten temperature sufficient for casting. The heat energy required is the sum of (1) the heat to raise the temperature to the melting point, (2) the heat of fusion to convert it from solid to liquid, and (3) the heat to raise the molten metal to the desired temperature for pouring. This can be expressed:    ð10:1Þ H ¼ rV Cs ðT m  T o Þ þ H f þ Cl T p  T m where H ¼ total heat required to raise the temperature of the metal to the pouring  temperature, J (Btu); r ¼ density; g=cm3 lbm/in3 ; Cs ¼ weight specific heat for the solid metal, J/g-C (Btu/lbm-F); Tm ¼ melting temperature of the metal,  C ( F); To ¼ starting temperature—usually ambient,  C ( F); Hf ¼ heat of fusion, J/g (Btu/lbm); Cl ¼ weight specific heat of the liquid metal, J/g-C (Btu/lbm-F); Tp ¼ pouring temperature,  C ( F); and V ¼ volume of metal being heated, cm3 (in3). Example 10.1 Heating Metal for Casting One cubic meter of a certain eutectic alloy is heated in a crucible from room temperature to 100 C above its melting point for casting. The alloy’s density ¼ 7.5 g/cm3, melting point ¼ 800 C, specific heat ¼ 0.33 J/g C in the solid state and 0.29 J/g C in the liquid state; and heat of fusion ¼ 160 J/g. How much heat energy must be added to accomplish the heating, assuming no losses? Solution: We assume ambient temperature in the foundry ¼ 25 C and that the density of the liquid and solid states of the metal are the same. Noting that one m3 ¼ 106 cm3, and substituting the property values into Eq. (10.1), we have     n H ¼ ð7:5Þ 106 f0:33ð800  25Þ þ 160 þ 0:29ð100Þg ¼ 3335 106 J The above equation is of conceptual value, but its computational value is limited, notwithstanding our example calculation. Use of Eq. (10.1) is complicated by the following factors: (1) Specific heat and other thermal properties of a solid metal vary with temperature, especially if the metal undergoes a change of phase during heating. (2) A metal’s specific heat may be different in the solid and liquid states. (3) Most casting metals are alloys, and most alloys melt over a temperature range between a solidus and liquidus rather than at a single melting point; thus, the heat of fusion cannot be applied so simply as indicated above. (4) The property values required in the equation for a particular alloy are not readily available in most cases. (5) There are significant heat losses to the environment during heating. 10.2.2 POURING THE MOLTEN METAL Pouring Temperature pouring rate turbulence After heating, the metal is ready for pouring. Introduction of molten metal into the mold, including its flow through the gating system and into the cavity, is a critical step in the casting process. For this step to be successful, the metal must flow into all regions of the mold before solidifying. Factors affecting the pouring operation include pouring temperature, pouring rate, and turbulence. E1C10 11/11/2009 14:39:17 Page 211 Section 10.2/Heating and Pouring 211 The pouring temperature is the temperature of the molten metal as it is introduced into the mold. What is important here is the difference between the temperature at pouring and the temperature at which freezing begins (the melting point for a pure metal or the liquidus temperature for an alloy). This temperature difference is sometimes referred to as the superheat. This term is also used for the amount of heat that must be removed from the molten metal between pouring and when solidification commences [7]. Pouring rate refers to the volumetric rate at which the molten metal is poured into the mold. If the rate is too slow, the metal will chill and freeze before filling the cavity. If the pouring rate is excessive, turbulence can become a serious problem. Turbulence in fluid flow is characterized by erratic variations in the magnitude and direction of the velocity throughout the fluid. The flow is agitated and irregular rather than smooth and streamlined, as in laminar flow. Turbulent flow should be avoided during pouring for several reasons. It tends to accelerate the formation of metal oxides that can become entrapped during solidification, thus degrading the quality of the casting. Turbulence also aggravates mold erosion, the gradual wearing away of the mold surfaces due to impact of the flowing molten metal. The densities of most molten metals are much higher than water and other fluids we normally deal with. These molten metals are also much more chemically reactive than at room temperature. Consequently, the wear caused by the flow of these metals in the mold is significant, especially under turbulent conditions. Erosion is especially serious when it occurs in the main cavity because the geometry of the cast part is affected. 10.2.3 ENGINEERING ANALYSIS OF POURING There are several relationships that govern the flow of liquid metal through the gating system and into the mold. An important relationship is Bernoulli’s theorem, which states that the sum of the energies (head, pressure, kinetic, and friction) at any two points in a flowing liquid are equal. This can be written in the following form: h1 þ p1 v2 p2 v2 þ 1 þ F 1 ¼ h2 þ þ 2 þ F2 r 2g r 2g ð10:2Þ v21 v2 ¼ h2 þ 2 2g 2g ð10:3Þ   where h ¼ head, cm (in), p ¼ pressure on the liquid, N=cm2 lb/in2 ; r ¼ density; g/cm3 (lbm/in3 ); v ¼ flow velocity; cm/s ðin/secÞ; g ¼ gravitational acceleration constant, 981 cm/s/s (32.2  12 ¼ 386 in/sec/sec); and F ¼ head losses due to friction, cm (in). Subscripts 1 and 2 indicate any two locations in the liquid flow. Bernoulli’s equation can be simplified in several ways. If we ignore friction losses (to be sure, friction will affect the liquid flow through a sand mold), and assume that the system remains at atmospheric pressure throughout, then the equation can be reduced to h1 þ This can be used to determine the velocity of the molten metal at the base of the sprue. Let us define point 1 at the top of the sprue and point 2 at its base. If point 2 is used as the reference plane, then the head at that point is zero (h2 ¼ 0) and h1 is the height (length) of the sprue. When the metal is poured into the pouring cup and overflows down the sprue, its initial velocity at the top is zero (v1 ¼ 0). Hence, Eq. (10.3) further simplifies to h1 ¼ v22 2g which can be solved for the flow velocity: v¼ pffiffiffiffiffiffiffiffi 2gh ð10:4Þ E1C10 11/11/2009 212 14:39:17 Page 212 Chapter 10/Fundamentals of Metal Casting where v ¼ the velocity of the liquid metal at the base of the sprue, cm/s (in/sec); g ¼ 981 cm/s/s (386 in/sec/sec); and h ¼ the height of the sprue, cm (in). Another relationship of importance during pouring is the continuity law, which states that the volume rate of flow remains constant throughout the liquid. The volume flow rate is equal to the velocity multiplied by the cross-sectional area of the flowing liquid. The continuity law can be expressed: Q ¼ v1 A1 ¼ v2 A2 ð10:5Þ where Q ¼ volumetric flow rate, cm3/s (in3/sec); v ¼ velocity as before; A ¼ crosssectional area of the liquid, cm2 (in2); and the subscripts refer to any two points in the flow system. Thus, an increase in area results in a decrease in velocity, and vice versa. Equations (10.4) and (10.5) indicate that the sprue should be tapered. As the metal accelerates during its descent into the sprue opening, the cross-sectional area of the channel must be reduced; otherwise, as the velocity of the flowing metal increases toward the base of the sprue, air can be aspirated into the liquid and conducted into the mold cavity. To prevent this condition, the sprue is designed with a taper, so that the volume flow rate vA is the same at the top and bottom of the sprue. Assuming that the runner from the sprue base to the mold cavity is horizontal (and therefore the head h is the same as at the sprue base), then the volume rate of flow through the gate and into the mold cavity remains equal to vA at the base. Accordingly, we can estimate the time required to fill a mold cavity of volume V as T MF ¼ V Q ð10:6Þ where TMF ¼ mold filling time, s (sec); V ¼ volume of mold cavity, cm3 (in3); and Q ¼ volume flow rate, as before. The mold filling time computed by Eq. (10.6) must be considered a minimum time. This is because the analysis ignores friction losses and possible constriction of flow in the gating system; thus, the mold filling time will be longer than what is given by Eq. (10.6). Example 10.2 Pouring Calculations A mold sprue is 20 cm long, and the cross-sectional area at its base is 2.5 cm2. The sprue feeds a horizontal runner leading into a mold cavity whose volume is 1560 cm3. Determine: (a) velocity of the molten metal at the base of the sprue, (b) volume rate of flow, and (c) time to fill the mold. Solution: (a) The velocity of the flowing metal at the base of the sprue is given by Eq. (10.4): v¼ pffiffiffiffiffiffiffiffiffiffiffiffiffiffiffiffiffiffiffiffiffiffi 2(981)(20) ¼ 198:1 cm=s (b) The volumetric flow rate is   Q ¼ 2:5 cm2 ð198:1 cm=sÞ ¼ 495 cm2 =s (c) Time required to fill a mold cavity of 100 in3 at this flow rate is T MF ¼ 1560=495 ¼ 3:2s n 10.2.4 FLUIDITY The molten metal flow characteristics are often described by the term fluidity, a measure of the capability of a metal to flow into and fill the mold before freezing. Fluidity is the inverse of viscosity (Section 3.4); as viscosity increases, fluidity decreases. Standard E1C10 11/11/2009 14:39:17 Page 213 Section 10.3/Solidification and Cooling 213 FIGURE 10.3 Spiral mold test for fluidity, in which fluidity is measured as the length of the spiral channel that is filled by the molten metal prior to solidification. testing methods are available to assess fluidity, including the spiral mold test shown in Figure 10.3, in which fluidity is indicated by the length of the solidified metal in the spiral channel. A longer cast spiral means greater fluidity of the molten metal. Factors affecting fluidity include pouring temperature relative to melting point, metal composition, viscosity of the liquid metal, and heat transfer to the surroundings. A higher pouring temperature relative to the freezing point of the metal increases the time it remains in the liquid state, allowing it to flow further before freezing. This tends to aggravate certain casting problems such as oxide formation, gas porosity, and penetration of liquid metal into the interstitial spaces between the grains of sand forming the mold. This last problem causes the surface of the casting to contain imbedded sand particles, thus making it rougher and more abrasive than normal. Composition also affects fluidity, particularly with respect to the metal’s solidification mechanism. The best fluidity is obtained by metals that freeze at a constant temperature (e.g., pure metals and eutectic alloys). When solidification occurs over a temperature range (most alloys are in this category), the partially solidified portion interferes with the flow of the liquid portion, thereby reducing fluidity. In addition to the freezing mechanism, metal composition also determines heat of fusion—the amount of heat required to solidify the metal from the liquid state. A higher heat of fusion tends to increase the measured fluidity in casting. 10.3 SOLIDIFICATION AND COOLING After pouring into the mold, the molten metal cools and solidifies. In this section we examine the physical mechanism of solidification that occurs during casting. Issues associated with solidification include the time for a metal to freeze, shrinkage, directional solidification, and riser design. 10.3.1 SOLIDIFICATION OF METALS Solidification involves the transformation of the molten metal back into the solid state. The solidification process differs depending on whether the metal is a pure element or an alloy. Pure Metals A pure metal solidifies at a constant temperature equal to its freezing point, which is the same as its melting point. The melting points of pure metals are well known and documented (Table 4.1). The process occurs over time as shown in the plot of Figure 10.4, called a cooling curve. The actual freezing takes time, called the local solidification time in casting, during which the metal’s latent heat of fusion is released into the surrounding mold. The total solidification time is the time taken between pouring and complete solidification. E1C10 11/11/2009 214 14:39:17 Page 214 Chapter 10/Fundamentals of Metal Casting FIGURE 10.4 Cooling curve for a pure metal during casting. After the casting has completely solidified, cooling continues at a rate indicated by the downward slope of the cooling curve. Because of the chilling action of the mold wall, a thin skin of solid metal is initially formed at the interface immediately after pouring. Thickness of the skin increases to form a shell around the molten metal as solidification progresses inward toward the center of the cavity. The rate at which freezing proceeds depends on heat transfer into the mold, as well as the thermal properties of the metal. It is of interest to examine the metallic grain formation and growth during this solidification process. The metal which forms the initial skin has been rapidly cooled by the extraction of heat through the mold wall. This cooling action causes the grains in the skin to be fine and randomly oriented. As cooling continues, further grain formation and growth occur in a direction away from the heat transfer. Since the heat transfer is through the skin and mold wall, the grains grow inwardly as needles or spines of solid metal. As these spines enlarge, lateral branches form, and as these branches grow, further branches form at right angles to the first branches. This type of grain growth is referred to as dendritic growth, and it occurs not only in the freezing of pure metals but alloys as well. These treelike structures are gradually filled-in during freezing, as additional metal is continually deposited onto the dendrites until complete solidification has occurred. The grains resulting from this dendritic growth take on a preferred orientation, tending to be coarse, columnar grains aligned toward the center of the casting. The resulting grain formation is illustrated in Figure 10.5. Most Alloys Most alloys freeze over a temperature range rather than at a single temperature. The exact range depends on the alloy system and the particular composition. FIGURE 10.5 Characteristic grain structure in a casting of a pure metal, showing randomly oriented grains of small size near the mold wall, and large columnar grains oriented toward the center of the casting. E1C10 11/11/2009 14:39:17 Page 215 Section 10.3/Solidification and Cooling 215 FIGURE 10.6 (a) Phase diagram for a copper– nickel alloy system and (b) associated cooling curve for a 50%Ni–50%Cu composition during casting. Solidification of an alloy can be explained with reference to Figure 10.6, which shows the phase diagram for a particular alloy system (Section 6.1.2) and the cooling curve for a given composition. As temperature drops, freezing begins at the temperature indicated by the liquidus and is completed when the solidus is reached. The start of freezing is similar to that of the pure metal. A thin skin is formed at the mold wall due to the large temperature gradient at this surface. Freezing then progresses as before through the formation of dendrites that grow away from the walls. However, owing to the temperature spread between the liquidus and solidus, the nature of the dendritic growth is such that an advancing zone is formed in which both liquid and solid metal coexist. The solid portions are the dendrite structures that have formed sufficiently to trap small islands of liquid metal in the matrix. This solid–liquid region has a soft consistency that has motivated its name as the mushy zone. Depending on the conditions of freezing, the mushy zone can be relatively narrow, or it can exist throughout most of the casting. The latter condition is promoted by factors such as slow heat transfer out of the hot metal and a wide difference between liquidus and solidus temperatures. Gradually, the liquid islands in the dendrite matrix solidify as the temperature of the casting drops to the solidus for the given alloy composition. Another factor complicating solidification of alloys is that the composition of the dendrites as they start to form favors the metal with the higher melting point. As freezing continues and the dendrites grow, there develops an imbalance in composition between the metal that has solidified and the remaining molten metal. This composition imbalance is finally manifested in the completed casting in the form of segregation of the elements. The segregation is of two types, microscopic and macroscopic. At the microscopic level, the chemical composition varies throughout each individual grain. This is due to the fact that the beginning spine of each dendrite has a higher proportion of one of the elements in the alloy. As the dendrite grows in its local vicinity, it must expand using the remaining liquid metal that has been partially depleted of the first component. Finally, the last metal to freeze in each grain is that which has been trapped by the branches of the dendrite, and its composition is even further out of balance. Thus, we have a variation in chemical composition within single grains of the casting. At the macroscopic level, the chemical composition varies throughout the entire casting. Since the regions of the casting that freeze first (at the outside near the mold walls) are richer in one component than the other, the remaining molten alloy is deprived of that component by the time freezing occurs at the interior. Thus, there is a general segregation E1C10 11/11/2009 216 14:39:17 Page 216 Chapter 10/Fundamentals of Metal Casting FIGURE 10.7 Characteristic grain structure in an alloy casting, showing segregation of alloying components in the center of casting. through the cross-section of the casting, sometimes called ingot segregation, as illustrated in Figure 10.7. Eutectic Alloys Eutectic alloys constitute an exception to the general process by which alloys solidify. A eutectic alloy is a particular composition in an alloy system for which the solidus and liquidus are at the same temperature. Hence, solidification occurs at a constant temperature rather than over a temperature range, as described above. The effect can be seen in the phase diagram of the lead–tin system shown in Figure 6.3. Pure lead has a melting point of 327 C (621 F), while pure tin melts at 232 C (450 F). Although most lead–tin alloys exhibit the typical solidus–liquidus temperature range, the particular composition of 61.9% tin and 38.1% lead has a melting (freezing) point of 183 C (362 F). This composition is the eutectic composition of the lead–tin alloy system, and 183 C is its eutectic temperature. Lead–tin alloys are not commonly used in casting, but Pb–Sn compositions near the eutectic are used for electrical soldering, where the low melting point is an advantage. Examples of eutectic alloys encountered in casting include aluminum–silicon (11.6% Si) and cast iron (4.3% C). 10.3.2 SOLIDIFICATION TIME Whether the casting is pure metal or alloy, solidification takes time. The total solidification time is the time required for the casting to solidify after pouring. This time is dependent on the size and shape of the casting by an empirical relationship known as Chvorinov’s rule, which states:  n V T TS ¼ Cm ð10:7Þ A where TTS ¼ total solidification time, min; V ¼ volume of the casting, cm3 (in3); A ¼ surface area of the casting, cm2 (in2); n is an exponent usually taken to have a value ¼ 2; and Cm is the mold constant. Given that n ¼ 2, the units of Cm are min/cm2 (min/in2), and its value depends on the particular conditions of the casting operation, including mold material (e.g., specific heat, thermal conductivity), thermal properties of the cast metal (e.g., heat of fusion, specific heat, thermal conductivity), and pouring temperature relative to the melting point of the metal. The value of Cm for a given casting operation can be based on experimental data from previous operations carried out using the same mold material, metal, and pouring temperature, even though the shape of the part may be quite different. Chvorinov’s rule indicates that a casting with a higher volume-to-surface area ratio will cool and solidify more slowly than one with a lower ratio. This principle is put to good use in designing the riser in a mold. To perform its function of feeding molten metal to the main cavity, the metal in the riser must remain in the liquid phase longer than the casting. In other words, the TTS for the riser must exceed the TTS for the main casting. Since the mold conditions for both riser and casting are the same, their mold constants will be equal. By E1C10 11/11/2009 14:39:17 Page 217 Section 10.3/Solidification and Cooling 217 designing the riser to have a larger volume-to-area ratio, we can be fairly sure that the main casting solidifies first and that the effects of shrinkage are minimized. Before considering how the riser might be designed using Chvorinov’s rule, let us consider the topic of shrinkage, which is the reason why risers are needed. 10.3.3 SHRINKAGE Our discussion of solidification has neglected the impact of shrinkage that occurs during cooling and freezing. Shrinkage occurs in three steps: (1) liquid contraction during cooling prior to solidification; (2) contraction during the phase change from liquid to solid, called solidification shrinkage; and (3) thermal contraction of the solidified casting during cooling to room temperature. The three steps can be explained with reference to a cylindrical casting made in an open mold, as shown in Figure 10.8. The molten metal immediately after pouring is shown in part (0) of the series. Contraction of the liquid metal during cooling from pouring temperature to freezing temperature causes the height of the liquid to be reduced from its starting level as in (1) of the figure. The amount of this liquid contraction is usually around 0.5%. Solidification shrinkage, seen in part (2), has two effects. First, contraction causes a further reduction in the height of the casting. Second, the amount of liquid metal available to feed the top center portion of the casting becomes restricted. This is usually the last region to freeze, and the absence of metal creates a void in the casting at this location. This shrinkage cavity is called a pipe by foundrymen. Once FIGURE 10.8 Shrinkage of a cylindrical casting during solidification and cooling: (0) starting level of molten metal immediately after pouring; (1) reduction in level caused by liquid contraction during cooling; (2) reduction in height and formation of shrinkage cavity caused by solidification shrinkage; and (3) further reduction in height and diameter due to thermal contraction during cooling of the solid metal. For clarity, dimensional reductions are exaggerated in our sketches. E1C10 11/11/2009 218 14:39:17 Page 218 Chapter 10/Fundamentals of Metal Casting TABLE 10.1 Typical linear shrinkage values for different casting metals due to solid thermal contraction. Linear Metal Aluminum alloys Brass, yellow Cast iron, gray Cast iron, white shrinkage 1.3% 1.3%–1.6% 0.8%–1.3% 2.1% Linear Metal Magnesium Magnesium alloy Nickel Steel, carbon shrinkage 2.1% 1.6% 2.1% 1.6%–2.1% Linear Metal Steel, chrome Tin Zinc shrinkage 2.1% 2.1% 2.6% Compiled from [10]. solidified, the casting experiences further contraction in height and diameter while cooling, as in (3). This shrinkage is determined by the solid metal’s coefficient of thermal expansion, which in this case is applied in reverse to determine contraction. Solidification shrinkage occurs in nearly all metals because the solid phase has a higher density than the liquid phase. The phase transformation that accompanies solidification causes a reduction in the volume per unit weight of metal. The exception is cast iron containing high carbon content, whose solidification during the final stages of freezing is complicated by a period of graphitization, which results in expansion that tends to counteract the volumetric decrease associated with the phase change [7]. Compensation for solidification shrinkage is achieved in several ways depending on the casting operation. In sand casting, liquid metal is supplied to the cavity by means of risers (Section 10.3.5). In die casting (Section 11.3.3), the molten metal is applied under pressure. Pattern-makers account for thermal contraction by making the mold cavities oversized. The amount by which the mold must be made larger relative to the final casting size is called the pattern shrinkage allowance. Although the shrinkage is volumetric, the dimensions of the casting are expressed linearly, so the allowances must be applied accordingly. Special ‘‘shrink rules’’ with slightly elongated scales are used to make the patterns and molds larger than the desired casting by the appropriate amount. Table 10.1 lists typical values of linear shrinkage for various cast metals; these values can be used to determine shrink rule scales. 10.3.4 DIRECTIONAL SOLIDIFICATION In order to minimize the damaging effects of shrinkage, it is desirable for the regions of the castingmostdistantfromtheliquid metal supplytofreezefirstandforsolidificationto progress from these remote regions toward the riser(s). In this way, molten metal will continually be available from the risers to prevent shrinkage voids during freezing. The term directional solidification is used to describe thisaspect of the freezing process and the methods by which it iscontrolled.ThedesireddirectionalsolidificationisachievedbyobservingChvorinov’srulein the design of the casting itself, its orientation within the mold, and the design of the riser system that feeds it. For example, by locating sections of the casting with lower V/A ratios away from theriser,freezingwill occur firstintheseregionsandthesupplyofliquidmetal fortherestofthe casting will remain open until these bulkier sections solidify. Another way to encourage directional solidification is to use chills—internal or external heat sinks that cause rapid freezing in certain regions of the casting. Internal chills are small metal parts placed inside the cavity before pouring so that the molten metal will solidify first around these parts. The internal chill should have a chemical composition similar to the metal being poured, most readily achieved by making the chill out of the same metal as the casting itself. External chills are metal inserts in the walls of the mold cavity that can remove heat from the molten metal more rapidly than the surrounding sand in order to promote solidification. They are often used effectively in sections of the casting that are difficult to E1C10 11/11/2009 14:39:18 Page 219 Section 10.3/Solidification and Cooling 219 FIGURE 10.9 (a) External chill to encourage rapid freezing of the molten metal in a thin section of the casting; and (b) the likely result if the external chill were not used. feed with liquid metal, thus encouraging rapid freezing in these sections while the connection to liquid metal is still open. Figure 10.9 illustrates a possible application of external chills and the likely result in the casting if the chill were not used. As important as it is to initiate freezing in the appropriate regions of the cavity, it is also important to avoid premature solidification in sections of the mold nearest the riser. Of particular concern is the passageway between the riser and the main cavity. This connection must be designed in such a way that it does not freeze before the casting, which would isolate the casting from the molten metal in the riser. Although it is generally desirable to minimize the volume in the connection (to reduce wasted metal), the cross-sectional area must be sufficient to delay the onset of freezing. This goal is usually aided by making the passageway short in length, so that it absorbs heat from the molten metal in the riser and the casting. 10.3.5 RISER DESIGN As described earlier, a riser, Figure 10.2(b), is used in a sand-casting mold to feed liquid metal to the casting during freezing in order to compensate for solidification shrinkage. To function, the riser must remain molten until after the casting solidifies. Chvorinov’s rule can be used to compute the size of a riser that will satisfy this requirement. The following example illustrates the calculation. Example 10.3 Riser Design Using Chvorinov’s Rule A cylindrical riser must be designed for a sand-casting mold. The casting itself is a steel rectangular plate with dimensions 7:5 cm  12:5 cm  2:0 cm. Previous observations have indicated that the total solidification time (TTS) for this casting ¼ 1.6 min. The cylinder for the riser will have a diameter-to-height ratio ¼ 1.0. Determine the dimensions of the riser so that its TTS ¼ 2.0 min. Solution: First determine the V/A ratio for the plate. Its volume V ¼ 7:5  12:5  2:0 ¼ 187:5 cm3 , and its surface area A ¼ 2ð7:5  12:5 þ 7:5  2:0 þ 12:5  2:0Þ ¼ 267:5 cm2 . Given that TTS ¼ 1.6 min, we can determine the mold constant Cm from Eq. (10.7), using a value of n ¼ 2 in the equation. Cm ¼ T TS (V=A)2 ¼ 1:6 (187:5=267:5)2 ¼ 3:26 min=cm2 Next we must design the riser so that its total solidification time is 2.0 min, using the same value of mold constant. The volume of the riser is given by V¼ p D2 h 4 E1C10 11/11/2009 220 14:39:18 Page 220 Chapter 10/Fundamentals of Metal Casting and the surface area is given by A ¼ p Dh þ 2p D2 4 Since we are using a D/H ratio ¼ 1.0, then D ¼ H. Substituting D for H in the volume and area formulas, we get V ¼ pD3 =4 and A ¼ p D2 þ 2p D2 =4 ¼ 1:5p D2 Thus the V=A ratio ¼ D=6. Using this ratio in Chvorinov’s equation, we have  2 D ¼ 0:09056 D2 T TS ¼ 2:0 ¼ 3:26 6 D2 ¼ 2:0=0:09056 ¼ 22:086 cm2 D ¼ 4:7 cm Since H ¼ D; then H ¼ 4:7 cm also. The riser represents waste metal that will be separated from the cast part and remelted to make subsequent castings. It is desirable for the volume of metal in the riser to be a minimum. Since the geometry of the riser is normally selected to maximize the V/A ratio, this tends to reduce the riser volume as much as possible. Note that the volume of the riser in our example problem is V ¼ pð4:7Þ3 =4 ¼ 81:5 cm3, only 44% of the volume of the plate (casting), even though its total solidification time is 25% longer. Risers can be designed in different forms. The design shown in Figure 10.2(b) is a side riser. It is attached to the side of the casting by means of a small channel. A top riser is one that is connected to the top surface of the casting. Risers can be open or blind. An open riser is exposed to the outside at the top surface of the cope. This has the disadvantage of allowing more heat to escape, promoting faster solidification. A blind n riser is entirely enclosed within the mold, as in Figure 10.1(b). REFERENCES [1] Amstead, B. H., Ostwald, P. F., and Begeman, M. L. Manufacturing Processes. John Wiley & Sons, Inc., New York, 1987. [2] Beeley, P. R. Foundry Technology. ButterworthsHeinemann, Oxford, UK, 2001. [3] Black, J, and Kohser, R. DeGarmo’s Materials and Processes in Manufacturing, 10th ed. John Wiley & Sons, Inc., Hoboken, New Jersey, 2008. [4] Datsko, J. Material Properties and Manufacturing Processes. John Wiley & Sons, Inc., New York, 1966. [5] Edwards, L., and Endean, M. Manufacturing with Materials. Open University, Milton Keynes, and Butterworth Scientific Ltd., London, 1990. [6] Flinn, R. A. Fundamentals of Metal Casting. American Foundrymen’s Society, Inc., Des Plaines, Illinois, 1987. [7] Heine, R. W., Loper, Jr., C. R., and Rosenthal, C. Principles of Metal Casting, 2nd ed. McGraw-Hill Book Co., New York, 1967. [8] Kotzin, E. L. (ed.). Metalcasting and Molding Processes. American Foundrymen’s Society, Inc., Des Plaines, Illinois, 1981. [9] Lessiter, M. J., and K. Kirgin. ‘‘Trends in the Casting Industry,’’ Advanced Materials & Processes, January 2002, pp. 42–43. [10] Metals Handbook, Vol. 15, Casting. ASM International, Materials Park, Ohio, 2008. [11] Mikelonis, P. J. (ed.). Foundry Technology. American Society for Metals, Metals Park, Ohio, 1982. E1C10 11/11/2009 14:39:18 Page 221 Multiple Choice Quiz [12] Niebel, B. W., Draper, A. B., Wysk, R. A. Modern Manufacturing Process Engineering. McGraw-Hill Book Co., New York, 1989. [13] Simpson, B. L. History of the Metalcasting Industry. American Foundrymen’s Society, Inc., Des Plaines, Illinois, 1997. 221 [14] Taylor, H. F., Flemings, M. C., and Wulff, J. Foundry Engineering, 2nd ed. American Foundrymen’s Society, Inc., Des Plaines, Illinois, 1987. [15] Wick, C., Benedict, J. T., and Veilleux, R. F. Tool and Manufacturing Engineers Handbook, 4th ed., Vol. II, Forming. Society of Manufacturing Engineers, Dearborn, Michigan, 1984. REVIEW QUESTIONS 10.1. Identify some of the important advantages of shape-casting processes. 10.2. What are some of the limitations and disadvantages of casting? 10.3. What is a factory that performs casting operations usually called? 10.4. What is the difference between an open mold and a closed mold? 10.5. Name the two basic mold types that distinguish casting processes. 10.6. Which casting process is the most important commercially? 10.7. What is the difference between a pattern and a core in sand molding? 10.8. What is meant by the term superheat? 10.9. Why should turbulent flow of molten metal into the mold be avoided? 10.10. What is the continuity law as it applies to the flow of molten metal in casting? 10.11. What are some of the factors that affect the fluidity of a molten metal during pouring into a mold cavity? 10.12. What does heat of fusion mean in casting? 10.13. How does solidification of alloys differ from solidification of pure metals? 10.14. What is a eutectic alloy? 10.15. What is the relationship known as Chvorinov’s rule in casting? 10.16. Identify the three sources of contraction in a metal casting after pouring. 10.17. What is a chill in casting? MULTIPLE CHOICE QUIZ There are 15 correct answers in the following multiple choice questions (some questions have multiple answers that are correct). To attain a perfect score on the quiz, all correct answers must be given. Each correct answer is worth 1 point. Each omitted answer or wrong answer reduces the score by 1 point, and each additional answer beyond the correct number of answers reduces the score by 1 point. Percentage score on the quiz is based on the total number of correct answers. 10.1. Sand casting is which of the following types: (a) expendable mold or (b) permanent mold? 10.2. The upper half of a sand-casting mold is called which of the following: (a) cope or (b) drag? 10.3. In casting, a flask is which one of the following: (a) beverage bottle for foundrymen, (b) box which holds the cope and drag, (c) container for holding liquid metal, or (d) metal which extrudes between the mold halves? 10.4. In foundry work, a runner is which one of the following: (a) channel in the mold leading from the downsprue to the main mold cavity, (b) foundryman who moves the molten metal to the mold, or (c) vertical channel into which molten metal is poured into the mold? 10.5. Turbulence during pouring of the molten metal is undesirable for which of the following reasons (two best answers): (a) it causes discoloration of the mold surfaces, (b) it dissolves the binder used to hold together the sand mold, (c) it increases erosion of the mold surfaces, (d) it increases the formation of metallic oxides that can become entrapped during solidification, (e) it increases the mold filling time, and (f) it increases total solidification time? 10.6. Total solidification time is defined as which one of the following: (a) time between pouring and complete solidification, (b) time between pouring and cooling to room temperature, (c) time between solidification and cooling to room temperature, or (d) time to give up the heat of fusion? 10.7. During solidification of an alloy when a mixture of solid and liquid metals is present, the solid-liquid mixture is referred to as which one of the following: (a) eutectic composition, (b) ingot segregation, (c) liquidus, (d) mushy zone, or (e) solidus? E1C10 11/11/2009 222 14:39:18 Page 222 Chapter 10/Fundamentals of Metal Casting 10.8. Chvorinov’s rule states that total solidification time is proportional to which one of the following quantities: (a) (A/V)n, (b) Hf, (c) Tm, (d) V, (e) V/A, or (f) (V/A)2; where A ¼ surface area of casting, Hf ¼ heat of fusion, Tm ¼ melting temperature, and V ¼ volume of casting? 10.9. Ariserincastingisdescribedbywhichofthefollowing (three correct answers): (a) an insert in the casting that inhibits buoyancy of the core,(b) gating system in which the sprue feeds directly into the cavity, (c) metal that is not part of the casting, (d) source of molten metal to feed the casting and compensate for shrinkage during solidification, and (e) waste metal that is usually recycled? 10.10. In a sand-casting mold, the V/A ratio of the riser should be (a) equal to, (b) greater than, or (c) smaller than the V/A ratio of the casting itself? 10.11. Which of the following riser types are completely enclosed within the sand mold and connected to the main cavity by a channel to feed the molten metal (two correct answers): (a) blind riser, (b) open riser, (c) side riser, and (d) top riser? PROBLEMS Heating and Pouring 10.1. A disk 40 cm in diameter and 5 cm thick is to be cast of pure aluminum in an open-mold casting operation. The melting temperature of aluminum ¼ 660 C, and the pouring temperature will be 800 C. Assume that the amount of aluminum heated will be 5% more than what is needed to fill the mold cavity. Compute the amount of heat that must be added to the metal to heat it to the pouring temperature, starting from a room temperature of 25 C. The heat of fusion of aluminum ¼ 389.3 J/g. Other properties can be obtained from Tables 4.1 and 4.2 in the text. Assume the specific heat has the same value for solid and molten aluminum. 10.2. A sufficient amount of pure copper is to be heated for casting a large plate in an open mold. The plate has dimensions: length ¼ 20 in, width ¼ 10 in, and thickness ¼ 3 in. Compute the amount of heat that must be added to the metal to heat it to a temperature of 2150 F for pouring. Assume that the amount of metal heated will be 10% more than what is needed to fill the mold cavity. Properties of the metal are: density ¼ 0.324 lbm/in3, melting point ¼ 1981 F, specific heat of the metal ¼ 0.093 Btu/lbm-F in the solid state and 0.090 Btu/lbm-F in the liquid state, and heat of fusion ¼ 80 Btu/lbm. 10.3. The downsprue leading into the runner of a certain mold has a length ¼ 175 mm. The cross-sectional area at the base of the sprue is 400 mm2. The mold cavity has a volume ¼ 0.001 m3. Determine (a) the velocity of the molten metal flowing through the base of the downsprue, (b) the volume rate of flow, and (c) the time required to fill the mold cavity. 10.4. A mold has a downsprue of length ¼ 6.0 in. The cross-sectional area at the bottom of the sprue is 0.5 in2. The sprue leads into a horizontal runner which feeds the mold cavity, whose volume ¼ 10.5. 10.6. 10.7. 10.8. 75 in3. Determine (a) the velocity of the molten metal flowing through the base of the downsprue, (b) the volume rate of flow, and (c) the time required to fill the mold cavity. The flow rate of liquid metal into the downsprue of a mold ¼ 1 L/s. The cross-sectional area at the top of the sprue ¼ 800 mm2, and its length ¼ 175 mm. What area should be used at the base of the sprue to avoid aspiration of the molten metal? The volume rate of flow of molten metal into the downsprue from the pouring cup is 50 in3/sec. At the top where the pouring cup leads into the downsprue, the cross-sectional area ¼ 1.0 in2. Determine what the area should be at the bottom of the sprue if its length ¼ 8.0 in. It is desired to maintain a constant flow rate, top and bottom, in order to avoid aspiration of the liquid metal. Molten metal can be poured into the pouring cup of a sand mold at a steady rate of 1000 cm3/s. The molten metal overflows the pouring cup and flows into the downsprue. The cross-section of the sprue is round, with a diameter at the top ¼ 3.4 cm. If the sprue is 25 cm long, determine the proper diameter at its base so as to maintain the same volume flow rate. During pouring into a sand mold, the molten metal can be poured into the downsprue at a constant flow rate during the time it takes to fill the mold. At the end of pouring the sprue is filled and there is negligible metal in the pouring cup. The downsprue is 6.0 in long. Its cross-sectional area at the top ¼ 0.8 in2 and at the base ¼ 0.6 in2. The cross-sectional area of the runner leading from the sprue also ¼ 0.6 in2, and it is 8.0 in long before leading into the mold cavity, whose volume ¼ 65 in3. The volume of the riser located along the runner near the mold cavity ¼ 25 in3. It takes a total of 3.0 E1C10 11/11/2009 14:39:18 Page 223 Problems sec to fill the entire mold (including cavity, riser, runner, and sprue. This is more than the theoretical time required, indicating a loss of velocity due to friction in the sprue and runner. Find (a) the theoretical velocity and flow rate at the base of the 223 downsprue; (b) the total volume of the mold; (c) the actual velocity and flow rate at the base of the sprue; and (d) the loss of head in the gating system due to friction. Shrinkage 10.9. Determine the shrink rule to be used by pattern makers for white cast iron. Using the shrinkage value in Table 10.1, express your answer in terms of decimal fraction inches of elongation per foot of length compared to a standard 1-foot scale. 10.10. Determine the shrink rule to be used by mold makers for die casting of zinc. Using the shrinkage value in Table 10.1, express your answer in terms of decimal mm of elongation per 300 mm of length compared to a standard 300-mm scale. 10.11. A flat plate is to be cast in an open mold whose bottom has a square shape that is 200 mm  200 mm. The mold is 40 mm deep. A total of 1,000,000 mm3 of molten aluminum is poured into the mold. Solidification shrinkage is known to be 6.0%. Table 10.1 lists the linear shrinkage due to thermal contraction after solidification to be 1.3%. If the availability of molten metal in the mold allows the square shape of the cast plate to maintain its 200 mm  200 mm dimensions until solidification is completed, determine the final dimensions of the plate. Solidification Time and Riser Design 10.12. In the casting of steel under certain mold conditions, the mold constant in Chvorinov’s rule is known to be 4.0 min/cm2, based on previous experience. The casting is a flat plate whose length ¼ 30 cm, width ¼ 10 cm, and thickness ¼ 20 mm. Determine how long it will take for the casting to solidify. 10.13. Solve for total solidification time in the previous problem only using an exponent value of 1.9 in Chvorinov’s rule instead of 2.0. What adjustment must be made in the units of the mold constant? 10.14. A disk-shaped part is to be cast out of aluminum. The diameter of the disk ¼ 500 mm and its thickness ¼ 20 mm. If the mold constant ¼ 2.0 s/mm2 in Chvorinov’s rule, how long will it take the casting to solidify? 10.15. In casting experiments performed using a certain alloy and type of sand mold, it took 155 s for a cube-shaped casting to solidify. The cube was 50 mm on a side. (a) Determine the value of the mold constant in Chvorinov’s rule. (b) If the same alloy and mold type were used, find the total solidification time for a cylindrical casting in which the diameter ¼ 30 mm and length ¼ 50 mm. 10.16. A steel casting has a cylindrical geometry with 4.0 in diameter and weighs 20 lb. This casting takes 6.0 min to completely solidify. Another cylindricalshaped casting with the same diameter-to-length ratio weighs 12 lb. This casting is made of the same steel, and the same conditions of mold and pouring were used. Determine: (a) the mold constant in Chvorinov’s rule, (b) the dimensions, and (c) the total solidification time of the lighter casting. The density of steel ¼ 490 lb/ft3. 10.17. The total solidification times of three casting shapes are to be compared: (1) a sphere with diameter ¼ 10 cm, (2) a cylinder with diameter and length both ¼ 10 cm, and (3) a cube with each side ¼ 10 cm. The same casting alloy is used in the three cases. (a) Determine the relative solidification times for each geometry. (b) Based on the results of part (a), which geometric element would make the best riser? (c) If the mold constant ¼ 3.5 min/cm2 in Chvorinov’s rule, compute the total solidification time for each casting. 10.18. The total solidification times of three casting shapes are to be compared: (1) a sphere, (2) a cylinder, in which the length-to-diameter ratio ¼ 1.0, and (3) a cube. For all three geometries, the volume ¼ 1000 cm3. The same casting alloy is used in the three cases. (a) Determine the relative solidification times for each geometry. (b) Based on the results of part (a), which geometric element would make the best riser? (c) If the mold constant ¼ 3.5 min/ cm2 in Chvorinov’s rule, compute the total solidification time for each casting. 10.19. A cylindrical riser is to be used for a sand-casting mold. For a given cylinder volume, determine the diameter-to-length ratio that will maximize the time to solidify. 10.20. A riser in the shape of a sphere is to be designed for a sand casting mold. The casting is a rectangular plate, with length ¼ 200 mm, width ¼ 100 mm, and thickness ¼ 18 mm. If the total solidification time of the casting itself is known to be 3.5 min, determine the diameter of the riser so that it will take 25% longer for the riser to solidify. E1C10 11/11/2009 224 14:39:19 Page 224 Chapter 10/Fundamentals of Metal Casting 10.21. A cylindrical riser is to be designed for a sand casting mold. The length of the cylinder is to be 1.25 times its diameter. The casting is a square plate, each side ¼ 10 in and thickness ¼ 0.75 in. If the metal is cast iron, and the mold constant ¼ 16.0 min/in2 in Chvorinov’s rule, determine the dimensions of the riser so that it will take 30% longer for the riser to solidify. FIGURE P10.22 Casting geometry for Problem 10.22 (units are in inches). 10.22. A cylindrical riser with diameter-to-length ratio ¼ 1.0 is to be designed for a sand casting mold. The casting geometry is illustrated in Figure P10.22, in which the units are inches. If the mold constant in Chvorinov’s rule ¼ 19.5 min/in2, determine the dimensions of the riser so that the riser will take 0.5 min longer to freeze than the casting itself. E1C11 11/10/2009 15:0:0 Page 225 11 METAL CASTING PROCESSES Chapter Contents 11.1 Sand Casting 11.1.1 Patterns and Cores 11.1.2 Molds and Mold Making 11.1.3 The Casting Operation 11.2 Other 11.2.1 11.2.2 11.2.3 11.2.4 11.2.5 Expendable-Mold Casting Processes Shell Molding Vacuum Molding Expanded Polystyrene Process Investment Casting Plaster-Mold and Ceramic-Mold Casting 11.3 Permanent-Mold Casting Processes 11.3.1 The Basic Permanent-Mold Process 11.3.2 Variations of Permanent-Mold Casting 11.3.3 Die Casting 11.3.4 Squeeze Casting and Semisolid Metal Casting 11.3.5 Centrifugal Casting 11.4 Foundry Practice 11.4.1 Furnaces 11.4.2 Pouring, Cleaning, and Heat Treatment 11.5 Casting Quality 11.6 Metals for Casting 11.7 Product Design Considerations Metal casting processes divide into two categories, based on mold type: (1) expendable mold and (2) permanent mold. In expendable mold casting operations, the mold is sacrificed in order to remove the cast part. Since a new mold is required for each new casting, production rates in expendable-mold processes are often limited by the time required to make the mold rather than the time to make the casting itself. However, for certain part geometries, sand molds can be produced and castings made at rates of 400 parts per hour and higher. In permanent-mold casting processes, the mold is fabricated out of metal (or other durable material) and can be used many times to make many castings. Accordingly, these processes possess a natural advantage in terms of higher production rates. Our discussion of casting processes in this chapter is organized as follows: (1) sand casting, (2) other expendable-mold casting processes, and (3) permanent-mold casting processes. The chapter also includes casting equipment and practices used in foundries. Another section deals with inspection and quality issues. Product design guidelines are presented in the final section. 11.1 SAND CASTING Sand casting is the most widely used casting process, accounting for a significant majority of the total tonnage cast. Nearly all casting alloys can be sand cast; indeed, it is one of the few processes that can be used for metals with high melting temperatures, such as steels, nickels, and titaniums. Its versatility permits the casting of parts ranging in size from small to very large (Figure 11.1) and in production quantities from one to millions. Sand casting, also known as sand-mold casting, consists of pouring molten metal into a sand mold, allowing the metal to solidify, and then breaking up the mold to remove the casting. The casting must then be cleaned and 225 E1C11 11/10/2009 226 15:0:0 Page 226 Chapter 11/Metal Casting Processes FIGURE 11.1 A large sand casting weighing more than 680 kg (1500 lb) for an air compressor frame. (Courtesy of Elkhart Foundry, photo by Paragon Inc., Elkhart, Indiana.) inspected, and heat treatment is sometimes required to improve metallurgical properties. The cavity in the sand mold is formed by packing sand around a pattern (an approximate duplicate of the part to be cast), and then removing the pattern by separating the mold into two halves. The mold also contains the gating and riser system. In addition, if the casting is to have internal surfaces (e.g., hollow parts or parts with holes), a core must be included in the mold. Since the mold is sacrificed to remove the FIGURE 11.2 Steps in the production sequence in sand casting. The steps include not only the casting operation but also pattern making and mold making. E1C11 11/10/2009 15:0:0 Page 227 Section 11.1/Sand Casting 227 casting, a new sand mold must be made for each part that is produced. From this brief description, sand casting is seen to include not only the casting operation itself, but also the fabrication of the pattern and the making of the mold. The production sequence is outlined in Figure 11.2 Our video clip on casting contains a segment on sand casting. VIDEO CLIP Casting: View the segment titled Sand-Mold Casting. 11.1.1 PATTERNS AND CORES Sand casting requires a pattern—a full-sized model of the part, enlarged to account for shrinkage and machining allowances in the final casting. Materials used to make patterns include wood, plastics, and metals. Wood is a common pattern material because it is easily shaped. Its disadvantages are that it tends to warp, and it is abraded by the sand being compacted around it, thus limiting the number of times it can be reused. Metal patterns are more expensive to make, but they last much longer. Plastics represent a compromise between wood and metal. Selection of the appropriate pattern material depends to a large extent on the total quantity of castings to be made. There are various types of patterns, as illustrated in Figure 11.3. The simplest is made of one piece, called a solid pattern—same geometry as the casting, adjusted in size for shrinkage and machining. Although it is the easiest pattern to fabricate, it is not the easiest to use in making the sand mold. Determining the location of the parting line between the two halves of the mold for a solid pattern can be a problem, and incorporating the gating system and sprue into the mold is left to the judgment and skill of the foundry worker. Consequently, solid patterns are generally limited to very low production quantities. Split patterns consist of two pieces, dividing the part along a plane coinciding with the parting line of the mold. Split patterns are appropriate for complex part geometries and moderate production quantities. The parting line of the mold is predetermined by the two pattern halves, rather than by operator judgment. For higher production quantities, match-plate patterns or cope-and-drag patterns are used. In match-plate patterns, the two pieces of the split pattern are attached to opposite sides of a wood or metal plate. Holes in the plate allow the top and bottom (cope FIGURE 11.3 Types of patterns used in sand casting: (a) solid pattern, (b) split pattern, (c) match-plate pattern, and (d) cope-and-drag pattern. E1C11 11/10/2009 228 15:0:0 Page 228 Chapter 11/Metal Casting Processes FIGURE 11.4 (a) Core held in place in the mold cavity by chaplets, (b) possible chaplet design, and (c) casting with internal cavity. and drag) sections of the mold to be aligned accurately. Cope-and-drag patterns are similar to match-plate patterns except that split pattern halves are attached to separate plates, so that the cope and drag sections of the mold can be fabricated independently, instead of using the same tooling for both. Part (d) of the figure includes the gating and riser system in the cope-and-drag patterns. Patterns define the external shape of the cast part. If the casting is to have internal surfaces, a core is required. A core is a full-scale model of the interior surfaces of the part. It is inserted into the mold cavity prior to pouring, so that the molten metal will flow and solidify between the mold cavity and the core to form the casting’s external and internal surfaces. The core is usually made of sand, compacted into the desired shape. As with the pattern, the actual size of the core must include allowances for shrinkage and machining. Depending on the geometry of the part, the core may or may not require supports to hold it in position in the mold cavity during pouring. These supports, called chaplets, are made of a metal with a higher melting temperature than the casting metal. For example, steel chaplets would be used for cast iron castings. On pouring and solidification, the chaplets become bonded into the casting. A possible arrangement of a core in a mold using chaplets is sketched in Figure 11.4. The portion of the chaplet protruding from the casting is subsequently cut off. 11.1.2 MOLDS AND MOLD MAKING Foundry sands are silica (SiO2) or silica mixed with other minerals. The sand should possess good refractory properties—capacity to stand up under high temperatures without melting or otherwise degrading. Other important features of the sand include grain size, distribution of grain size in the mixture, and shape of the individual grains (Section 16.1). Small grain size provides a better surface finish on the cast part, but large grain size is more permeable (to allow escape of gases during pouring). Molds made from grains of irregular shape tend to be stronger than molds of round grains because of interlocking, yet interlocking tends to restrict permeability. In making the mold, the grains of sand are held together by a mixture of water and bonding clay. A typical mixture (by volume) is 90% sand, 3% water, and 7% clay. Other bonding agents can be used in place of clay, including organic resins (e.g., phenolic resins) and inorganic binders (e.g., sodium silicate and phosphate). Besides sand and binder, additives are sometimes combined with the mixture to enhance properties such as strength and/or permeability of the mold. E1C11 11/10/2009 15:0:0 Page 229 Section 11.1/Sand Casting 229 To form the mold cavity, the traditional method is to compact the molding sand around the pattern for both cope and drag in a container called a flask. The packing process is performed by various methods. The simplest is hand ramming, accomplished manually by a foundry worker. In addition, various machines have been developed to mechanize the packing procedure. These machines operate by any of several mechanisms, including (1) squeezing the sand around the pattern by pneumatic pressure; (2) a jolting action in which the sand, contained in the flask with the pattern, is dropped repeatedly in order to pack it into place; and (3) a slinging action, in which the sand grains are impacted against the pattern at high speed. An alternative to traditional flasks for each sand mold is flaskless molding, which refers to the use of one master flask in a mechanized system of mold production. Each sand mold is produced using the same master flask. Mold production rates up to 600 per hour are claimed for this more automated method [8]. Severalindicatorsare usedtodetermine the quality of the sandmold [7]: (1) strength— the mold’s ability to maintain its shape and resist erosion caused by the flow of molten metal; it depends on grain shape, adhesive qualities of the binder, and other factors; (2) permeability—capacity of the mold to allow hot air and gases from the casting operation to pass through the voids in the sand; (3) thermal stability—ability of the sand at the surface of the mold cavity to resist cracking and buckling upon contact with the molten metal; (4) collapsibility—ability of the mold to give way and allow the casting to shrink without cracking the casting; it also refers to the ability to remove the sand from the casting during cleaning; and (5) reusability—can the sand from the broken mold be reused to make other molds? These measures are sometimes incompatible; for example, a mold with greater strength is less collapsible. Sand molds are often classified as green-sand, dry-sand, or skin-dried molds. Greensand molds are made of a mixture of sand, clay, and water, the word green referring to the fact that the mold contains moisture at the time of pouring. Green-sand molds possess sufficient strength for most applications, good collapsibility, good permeability, good reusability, and are the least expensive of the molds. They are the most widely used mold type, but they are not without problems. Moisture in the sand can cause defects in some castings, depending on the metal and geometry of the part. A dry-sand mold is made using organic binders rather than clay, and the mold is baked in a large oven at temperatures ranging from 200 C to 320 C (392 F to 608 F) [8]. Oven baking strengthens the mold and hardens the cavity surface. A drysand mold provides better dimensional control in the cast product, compared to green-sand molding. However, dry-sand molding is more expensive, and production rate is reduced because of drying time. Applications are generally limited to medium and large castings in low to medium production rates. In a skin-dried mold, the advantages of a dry-sand mold is partially achieved by drying the surface of a green-sand mold to a depth of 10 to 25 mm (0.4–1 in) at the mold cavity surface, using torches, heating lamps, or other means. Special bonding materials must be added to the sand mixture to strengthen the cavity surface. The preceding mold classifications refer to the use of conventional binders consisting of either clay-and-water or ones that require heating to cure. In addition to these classifications, chemically bonded molds have been developed that are not based on either of these traditional binder ingredients. Some of the binder materials used in these ‘‘no-bake’’ systems include furan resins (consisting of furfural alcohol, urea, and formaldehyde), phenolics, and alkyd oils. No-bake molds are growing in popularity due to their good dimensional control in high production applications. 11.1.3 THE CASTING OPERATION After the core is positioned (if one is used) and the two halves of the mold are clamped together, then casting is performed. Casting consists of pouring, solidification, and cooling E1C11 11/10/2009 230 15:0:0 Page 230 Chapter 11/Metal Casting Processes TABLE 11.1 Densities of selected casting alloys. Density Metal g/cm Aluminum (99% ¼ pure) Aluminum-silicon alloy Aluminum-copper (92% Al) Brassa 2.70 2.65 2.81 8.62 3 lb/in Density 3 0.098 0.096 0.102 0.313 Metal g/cm3 lb/in3 Cast iron, graya Copper (99% ¼ pure) Lead (pure) Steel 7.16 8.73 11.30 7.82 0.260 0.317 0.410 0.284 Source: [7]. Density depends on composition of alloy; value given is typical. a of the cast part (Sections 10.2 and 10.3). The gating and riser system in the mold must be designed to deliver liquid metal into the cavity and provide for a sufficient reservoir of molten metal during solidification shrinkage. Air and gases must be allowed to escape. One of the hazards during pouring is that the buoyancy of the molten metal will displace the core. Buoyancy results from the weight of molten metal being displaced by the core, according to Archimedes’ principle. The force tending to lift the core is equal to the weight of the displaced liquid less the weight of the core itself. Expressing the situation in equation form, Fb ¼ Wm  Wc ð11:1Þ where Fb ¼ buoyancy force, N (lb); Wm ¼ weight of molten metal displaced, N (lb); and % Wc ¼ weight of the core, N (lb). Weights are determined as the volume of the core multiplied by the respective densities of the core material (typically sand) and the metal being cast. The density of a sand core is approximately 1.6 g/cm3 (0.058 lb/in3). Densities of several common casting alloys are given in Table 11.1. Example 11.1 Buoyancy in Sand Casting A sand core has a volume ¼ 1875 cm3 and is located inside a sand mold cavity. Determine the buoyancy force tending to lift the core during pouring of molten lead into the mold. Solution: Density of the sand core is 1.6 g/cm3. Weight of the core is 1875(1.6) ¼ 3000 g ¼ 3.0 kg. Density of lead, based on Table 11.1, is 11.3 g/cm3. The weight of lead displaced by the core is 1875(11.3) ¼ 21,188 g ¼ 21.19 kg. The difference ¼ 21.19  3.0 ¼ 18.19 kg. Given that 1 kg ¼ 9.81 N, the buoyancy force is therefore Fb ¼ 9.81(18.19) ¼ 178.4 N. Following solidification and cooling, the sand mold is broken away from the casting to retrieve the part. The part is then cleaned—gating and riser system are separated, and n sand is removed. The casting is then inspected (Section 11.5). 11.2 OTHER EXPENDABLE-MOLD CASTING PROCESSES As versatile as sand casting is, there are other casting processes that have been developed to meet special needs. The differences between these methods are in the composition of the mold material, or the manner in which the mold is made, or in the way the pattern is made. 11.2.1 SHELL MOLDING Shell molding is a casting process in which the mold is a thin shell (typically 9 mm or 3/8 in) made of sand held together by a thermosetting resin binder. Developed in Germany during the early 1940s, the process is described and illustrated in Figure 11.5. E1C11 11/10/2009 15:0:0 Page 231 Section 11.2/Other Expendable-Mold Casting Processes 231 FIGURE 11.5 Steps in shell molding: (1) a match-plate or cope-and-drag metal pattern is heated and placed over a box containing sand mixed with thermosetting resin; (2) box is inverted so that sand and resin fall onto the hot pattern, causing a layer of the mixture to partially cure on the surface to form a hard shell; (3) box is repositioned so that loose, uncured particles drop away; (4) sand shell is heated in oven for several minutes to complete curing; (5) shell mold is stripped from the pattern; (6) two halves of the shell mold are assembled, supported by sand or metal shot in a box, and pouring is accomplished. The finished casting with sprue removed is shown in (7). There are many advantages to the shell-molding process. The surface of the shellmold cavity is smoother than a conventional green-sand mold, and this smoothness permits easier flow of molten metal during pouring and better surface finish on the final casting. Finishes of 2.5 mm (100 m-in) can be obtained. Good dimensional accuracy is also achieved, with tolerances of 0.25 mm (0.010 in) possible on small-to-medium-sized parts. The good finish and accuracy often precludes the need for further machining. Collapsibility of the mold is generally sufficient to avoid tearing and cracking of the casting. Disadvantages of shell molding include a more expensive metal pattern than the corresponding pattern for green-sand molding. This makes shell molding difficult to justify for small quantities of parts. Shell molding can be mechanized for mass production and is very economical for large quantities. It seems particularly suited to steel castings of less than 20 lb. Examples of parts made using shell molding include gears, valve bodies, bushings, and camshafts. 11.2.2 VACUUM MOLDING Vacuum molding, also called the V-process, was developed in Japan around 1970. It uses a sand mold held together by vacuum pressure rather than by a chemical binder. Accordingly, the term vacuum in this process refers to the making of the mold rather than the casting operation itself. The steps of the process are explained in Figure 11.6. Because no binders are used, the sand is readily recovered in vacuum molding. Also, the sand does not require extensive mechanical reconditioning normally done when E1C11 11/10/2009 232 15:0:1 Page 232 Chapter 11/Metal Casting Processes FIGURE 11.6 Steps in vacuum molding: (1) a thin sheet of preheated plastic is drawn over a match-plate or cope-and-drag pattern by vacuum—the pattern has small vent holes to facilitate vacuum forming; (2) a specially designed flask is placed over the pattern plate and filled with sand, and a sprue and pouring cup are formed in the sand; (3) another thin plastic sheet is placed over the flask, and a vacuum is drawn that causes the sand grains to be held together, forming a rigid mold; (4) the vacuum on the mold pattern is released to permit the pattern to be stripped from the mold; (5) this mold is assembled with its matching half to form the cope and drag, and with vacuum maintained on both halves, pouring is accomplished. The plastic sheet quickly burns away on contacting the molten metal. After solidification, nearly all of the sand can be recovered for reuse. binders are used in the molding sand. Since no water is mixed with the sand, moisturerelated defects are absent from the product. Disadvantages of the V-process are that it is relatively slow and not readily adaptable to mechanization. 11.2.3 EXPANDED POLYSTYRENE PROCESS The expanded polystyrene casting process uses a mold of sand packed around a polystyrene foam pattern that vaporizes when the molten metal is poured into the mold. The process and variations of it are known by other names, including lost-foam process, lostpattern process, evaporative-foam process, and full-mold process (the last being a trade name). The foam pattern includes the sprue, risers, and gating system, and it may also contain internal cores (if needed), thus eliminating the need for a separate core in the mold. Also, since the foam pattern itself becomes the cavity in the mold, considerations of draft and parting lines can be ignored. The mold does not have to be opened into cope and drag sections. The sequence in this casting process is illustrated and described in Figure 11.7. Various methods for making the pattern can be used, depending on the quantities of castings to be produced. For one-of-a-kind castings, the foam is manually cut from large strips and assembled to form the pattern. For large production runs, an E1C11 11/10/2009 15:0:1 Page 233 Section 11.2/Other Expendable-Mold Casting Processes 233 FIGURE 11.7 Expanded polystyrene casting process: (1) pattern of polystyrene is coated with refractory compound; (2) foam pattern is placed in mold box, and sand is compacted around the pattern; and (3) molten metal is poured into the portion of the pattern that forms the pouring cup and sprue. As the metal enters the mold, the polystyrene foam is vaporized ahead of the advancing liquid, thus allowing the resulting mold cavity to be filled. automated molding operation can be set up to mold the patterns prior to making the molds for casting. The pattern is normally coated with a refractory compound to provide a smoother surface on the pattern and to improve its high temperature resistance. Molding sands usually include bonding agents. However, dry sand is used in certain processes in this group, which aids recovery and reuse. The video clip on casting features a segment titled Evaporative-Foam Casting. VIDEO CLIP Casting: View the segment titled Evaporative-Foam Casting. A significant advantage for this process is that the pattern need not be removed from the mold. This simplifies and expedites mold making. In a conventional green-sand mold, two halves are required with proper parting lines, draft allowances must be provided in the mold design, cores must be inserted, and the gating and riser system must be added. With the expanded polystyrene process, these steps are built into the pattern itself. A new pattern is needed for every casting, so the economics of the expanded polystyrene casting process depend largely on the cost of producing the patterns. The process has been applied to mass produce castings for automobiles engines. Automated production systems are installed to mold the polystyrene foam patterns for these applications. 11.2.4 INVESTMENT CASTING In investment casting, a pattern made of wax is coated with a refractory material to make the mold, after which the wax is melted away prior to pouring the molten metal. The term investment comes from one of the less familiar definitions of the word invest, which is ‘‘to cover completely,’’ this referring to the coating of the refractory material around the wax pattern. It is a precision casting process, because it is capable of making castings of high accuracy and intricate detail. The process dates back to ancient Egypt (Historical Note 11.1) and is also known as the lost-wax process, because the wax pattern is lost from the mold prior to casting. E1C11 11/10/2009 234 15:0:1 Page 234 Chapter 11/Metal Casting Processes Historical Note 11.1 T Investment casting he lost wax casting process was developed by the ancient Egyptians some 3500 years ago. Although written records do not identify when the invention occurred or the artisan responsible, historians speculate that the process resulted from the close association between pottery and molding in early times. It was the potter who crafted the molds that were used for casting. The idea for the lost wax process must have originated with a potter who was familiar with the casting process. As he was working one day on a ceramic piece—perhaps an ornate vase or bowl—it occurred to him that the article might be more attractive and durable if made of metal. So he fashioned a core in the general shape of the piece, but smaller than the desired final dimensions, and coated it with wax to establish the size. The wax proved to be an easy material to form, and intricate designs and shapes could be created by the craftsman. On the wax surface, he carefully plastered several layers of clay and devised a means of holding the resulting components together. He then baked the mold in a kiln, so that the clay hardened and the wax melted and drained out to form a cavity. At last, he poured molten bronze into the cavity and, after the casting had solidified and cooled, broke away the mold to recover the part. Considering the education and experience of this early pottery maker and the tools he had to work with, development of the lost wax casting process demonstrated great innovation and insight. ‘‘No other process can be named by archeologists so crowded with deduction, engineering ability and ingenuity’’ [14]. FIGURE 11.8 Steps in investment casting: (1) wax patterns are produced; (2) several patterns are attached to a sprue to form a pattern tree; (3) the pattern tree is coated with a thin layer of refractory material; (4) the full mold is formed by covering the coated tree with sufficient refractory material to make it rigid; (5) the mold is held in an inverted position and heated to melt the wax and permit it to drip out of the cavity; (6) the mold is preheated to a high temperature, which ensures that all contaminants are eliminated from the mold; it also permits the liquid metal to flow more easily into the detailed cavity; the molten metal is poured; it solidifies; and (7) the mold is broken away from the finished casting. Parts are separated from the sprue. E1C11 11/10/2009 15:0:1 Page 235 Section 11.2/Other Expendable-Mold Casting Processes 235 Steps in investment casting are described in Figure 11.8. Since the wax pattern is melted off after the refractory mold is made, a separate pattern must be made for every casting. Pattern production is usually accomplished by a molding operation—pouring or injecting the hot wax into a master die that has been designed with proper allowances for shrinkage of both wax and subsequent metal casting. In cases where the part geometry is complicated, several separate wax pieces must be joined to make the pattern. In highproduction operations, several patterns are attached to a sprue, also made of wax, to form a pattern tree; this is the geometry that will be cast out of metal. The video clip on casting contains a segment on investment casting. VIDEO CLIP Casting: View the segment titled Investment Casting. Coating with refractory (step 3) is usually accomplished by dipping the pattern tree into a slurry of very fine grained silica or other refractory (almost in powder form) mixed with plaster to bond the mold into shape. The small grain size of the refractory material provides a smooth surface and captures the intricate details of the wax pattern. The final mold (step 4) is accomplished by repeatedly dipping the tree into the refractory slurry or by gently packing the refractory around the tree in a container. The mold is allowed to air dry for about 8 hours to harden the binder. Advantages of investment casting include: (1) parts of great complexity and intricacy can be cast; (2) close dimensional control—tolerances of 0.075 mm (0.003 in) are possible; (3) good surface finish is possible; (4) the wax can usually be recovered for reuse; and (5) additional machining is not normally required—this is a net shape process. Because many steps are involved in this casting operation, it is a relatively expensive process. Investment castings are normally small in size, although parts with complex geometries weighing up to 75 lb have been successfully cast. All types of metals, including steels, stainless steels, and other high temperature alloys, can be investment cast. Examples of parts include complex machinery parts, blades, and other components for turbine engines, jewelry, and dental fixtures. Shown in Figure 11.9 is a part illustrating the intricate features possible with investment casting. 11.2.5 PLASTER-MOLD AND CERAMIC-MOLD CASTING Plaster-mold casting is similar to sand casting except that the mold is made of plaster of Paris (gypsum, CaSO4–2H2O) instead of sand. Additives such as talc and silica flour are mixed with the plaster to control contraction and setting time, reduce cracking, and increase strength. To make the mold, the plaster mixture combined with water is poured over a plastic or metal pattern in a flask and allowed to set. Wood patterns are generally unsatisfactory due to the extended contact with water in the plaster. The fluid consistency permits the plaster mixture to readily flow around the pattern, capturing its details and surface finish. Thus, the cast product in plaster molding is noted for these attributes. Curing of the plaster mold is one of the disadvantages of this process, at least in high production. The mold must set for about 20 minutes before the pattern is stripped. The mold is then baked for several hours to remove moisture. Even with the baking, not all of the moisture content is removed from the plaster. The dilemma faced by foundrymen is that mold strength is lost when the plaster becomes too dehydrated, and yet moisture content can cause casting defects in the product. A balance must be achieved between these undesirable alternatives. Another disadvantage with the plaster mold is that it is not permeable, thus limiting escape of gases from the mold cavity. This problem can be solved in E1C11 11/10/2009 236 15:0:1 Page 236 Chapter 11/Metal Casting Processes FIGURE 11.9 A one-piece compressor stator with 108 separate airfoils made by investment casting. (Courtesy of Howmet Corp.). a number of ways: (1) evacuating air from the mold cavity before pouring; (2) aerating the plaster slurry prior to mold making so that the resulting hard plaster contains finely dispersed voids; and (3) using a special mold composition and treatment known as the Antioch process. This process involves using about 50% sand mixed with the plaster, heating the mold in an autoclave (an oven that uses superheated steam under pressure), and then drying. The resulting mold has considerably greater permeability than a conventional plaster mold. Plaster molds cannot withstand the same high temperatures as sand molds. They are therefore limited to the casting of lower-melting-point alloys, such as aluminum, magnesium, and some copper-base alloys. Applications include metal molds for plastic and rubber molding, pump and turbine impellers, and other parts of relatively intricate geometry. Casting sizes range from about 20 g (less than 1 oz) to more than 100 kg (more than 220 lb). Parts weighing less than about 10 kg (22 lb) are most common. Advantages of plaster molding for these applications are good surface finish and dimensional accuracy and the capability to make thin cross-sections in the casting. Ceramic-mold casting is similar to plaster-mold casting, except that the mold is made of refractory ceramic materials that can withstand higher temperatures than plaster. Thus, ceramic molding can be used to cast steels, cast irons, and other hightemperature alloys. Its applications (relatively intricate parts) are similar to those of plaster-mold casting except for the metals cast. Its advantages (good accuracy and finish) are also similar. E1C11 11/10/2009 15:0:1 Page 237 Section 11.3/Permanent-Mold Casting Processes 11.3 237 PERMANENT-MOLD CASTING PROCESSES The economic disadvantage of any of the expendable-mold processes is that a new mold is required for every casting. In permanent-mold casting, the mold is reused many times. In this section, we treat permanent-mold casting as the basic process in the group of casting processes that all use reusable metal molds. Other members of the group include die casting and centrifugal casting. 11.3.1 THE BASIC PERMANENT-MOLD PROCESS Permanent-mold casting uses a metal mold constructed of two sections that are designed for easy, precise opening and closing. These molds are commonly made of steel or cast iron. The cavity, with gating system included, is machined into the two halves to provide accurate dimensions and good surface finish. Metals commonly cast in permanent molds include aluminum, magnesium, copper-base alloys, and cast iron. However, cast iron requires a high pouring temperature, 1250 C to 1500 C (2282 F–2732 F), which takes a heavy toll on mold life. The very high pouring temperatures of steel make permanent molds unsuitable for this metal, unless the mold is made of refractory material. Cores can be used in permanent molds to form interior surfaces in the cast product. The cores can be made of metal, but either their shape must allow for removal from the casting or they must be mechanically collapsible to permit removal. If withdrawal of a metal core would be difficult or impossible, sand cores can be used, in which case the casting process is often referred to as semipermanent-mold casting. Steps in the basic permanent-mold casting process are described in Figure 11.10. In preparation for casting, the mold is first preheated and one or more coatings are sprayed on the cavity. Preheating facilitates metal flow through the gating system and into the cavity. The coatings aid heat dissipation and lubricate the mold surfaces for easier separation of the cast product. After pouring, as soon as the metal solidifies, the mold is opened and the casting is removed. Unlike expendable molds, permanent molds do not collapse, so the mold must be opened before appreciable cooling contraction occurs in order to prevent cracks from developing in the casting. Advantages of permanent-mold casting include good surface finish and close dimensional control, as previously indicated. In addition, more rapid solidification caused by the metal mold results in a finer grain structure, so stronger castings are produced. The process is generally limited to metals of lower melting points. Other limitations include simple part geometries compared to sand casting (because of the need to open the mold), and the expense of the mold. Because mold cost is substantial, the process is best suited to high-volume production and can be automated accordingly. Typical parts include automotive pistons, pump bodies, and certain castings for aircraft and missiles. 11.3.2 VARIATIONS OF PERMANENT-MOLD CASTING Several casting processes are quite similar to the basic permanent-mold method. These include slush casting, low-pressure casting, and vacuum permanent-mold casting. Slush Casting Slush casting is a permanent-mold process in which a hollow casting is formed by inverting the mold after partial freezing at the surface to drain out the liquid metal in the center. Solidification begins at the mold walls because they are relatively cool, and it progresses over time toward the middle of the casting (Section 10.3.1). Thickness of E1C11 11/10/2009 238 15:0:1 Page 238 Chapter 11/Metal Casting Processes FIGURE 11.10 Steps in permanent-mold casting: (1) mold is preheated and coated; (2) cores (if used) are inserted, and mold is closed; (3) molten metal is poured into the mold; and (4) mold is opened. Finished part is shown in (5). the shell is controlled by the length of time allowed before draining. Slush casting is used to make statues, lamp pedestals, and toys out of low-melting-point metals such as zinc and tin. In these items, the exterior appearance is important, but the strength and interior geometry of the casting are minor considerations. Low-Pressure Casting In the basic permanent-mold casting process and in slush casting, the flow of metal into the mold cavity is caused by gravity. In low-pressure casting, the liquid metal is forced into the cavity under low pressure—approximately 0.1 MPa (14.5 lb/in2)—from beneath so that the flow is upward, as illustrated in Figure 11.11. The advantage of this approach over traditional pouring is that clean molten metal from the center of the ladle is introduced into the mold, rather than metal that has been exposed to air. Gas porosity and oxidation defects are thereby minimized, and mechanical properties are improved. Vacuum Permanent-Mold Casting Not to be confused with vacuum molding (Section 11.2.2), this process is a variation of low-pressure casting in which a vacuum is used to draw the molten metal into the mold cavity. The general configuration of the vacuum permanentmold casting process is similar to the low-pressure casting operation. The difference is that reduced air pressure from the vacuum in the mold is used to draw the liquid metal into the E1C11 11/10/2009 15:0:1 Page 239 Section 11.3/Permanent-Mold Casting Processes FIGURE 11.11 Low-pressure casting. The diagram shows how air pressure is used to force the molten metal in the ladle upward into the mold cavity. Pressure is maintained until the casting has solidified. 239 e cavity, rather than forcing it by positive air pressure from below. There are several benefits of the vacuum technique relative to low-pressure casting: air porosity and related defects are reduced, and greater strength is given to the cast product. 11.3.3 DIE CASTING Die casting is a permanent-mold casting process in which the molten metal is injected into the mold cavity under high pressure. Typical pressures are 7 to 350 MPa (1015–50,763 lb/ in2). The pressure is maintained during solidification, after which the mold is opened and the part is removed. Molds in this casting operation are called dies; hence the name die casting. The use of high pressure to force the metal into the die cavity is the most notable feature that distinguishes this process from others in the permanent-mold category. The reader can see the various forms of this process in the video clip on die casting. VIDEO CLIP Die Casting. This clip contains two segments: (1) die casting machines and (2) die casting tooling. Die casting operations are carried out in special die casting machines (Historical Note 11.2), which are designed to hold and accurately close the two halves of the mold, and keep them closed while the liquid metal is forced into the cavity. The general configuration is shown in Figure 11.12. There are two main types of die casting machines: (1) hot-chamber and (2) cold-chamber, differentiated by how the molten metal is injected into the cavity. In hot-chamber machines, the metal is melted in a container attached to the machine, and a piston is used to inject the liquid metal under high pressure into the die. Typical injection pressures are 7 to 35 MPa (1015–5076 lb/in2). The casting cycle is summarized in Figure 11.13. Production rates up to 500 parts per hour are not uncommon. Hot-chamber die casting imposes a special hardship on the injection system because much of it is submerged in the molten metal. The process is therefore limited in its applications to lowmelting-point metals that do not chemically attack the plunger and other mechanical components. The metals include zinc, tin, lead, and sometimes magnesium. E1C11 11/10/2009 240 15:0:1 Page 240 Chapter 11/Metal Casting Processes Historical Note 11.2 T Die casting machines he modern die casting machine has its origins in the printing industry and the need in the mid to late 1800s to satisfy an increasingly literate population with a growing appetite for reading. The linotype, invented and developed by O. Mergenthaler in the late 1800s, is a machine that produces printing type. It is a casting machine because it casts a line of type characters out of lead to be used in preparing printing plates. The name linotype derives from the fact that the machine produces a line of type characters during each cycle of operation. FIGURE 11.12 General configuration of a (coldchamber) die casting machine. FIGURE 11.13 Cycle in hotchamber casting: (1) with die closed and plunger withdrawn, molten metal flows into the chamber; (2) plunger forces metal in chamber to flow into die, maintaining pressure during cooling and solidification; and (3) plunger is withdrawn, die is opened, and solidified part is ejected. Finished part is shown in (4). The machine was first used successfully on a commercial basis in New York City by The Tribune in 1886. The linotype proved the feasibility of mechanized casting machines. The first die casting machine was patented by H. Doehler in 1905 (this machine is displayed in the Smithsonian Institute in Washington, DC). In 1907, E. Wagner developed the first die casting machine to utilize the hot-chamber design. It was first used during World War I to cast parts for binoculars and gas masks. E1C11 11/10/2009 15:0:1 Page 241 Section 11.3/Permanent-Mold Casting Processes 241 In cold-chamber die casting machines, molten metal is poured into an unheated chamber from an external melting container, and a piston is used to inject the metal under high pressure into the die cavity. Injection pressures used in these machines are typically 14 to 140 MPa (2031–20,305 lb/in2). The production cycle is explained in Figure 11.14. Compared to hot-chamber machines, cycle rates are not usually as fast because of the need to ladle the liquid metal into the chamber from an external source. Nevertheless, this casting process is a high production operation. Cold-chamber machines are typically used for casting aluminum, brass, and magnesium alloys. Low-melting-point alloys (zinc, tin, lead) can also be cast on cold-chamber machines, but the advantages of the hot-chamber process usually favor its use on these metals. Molds used in die casting operations are usually made of tool steel, mold steel, or maraging steel. Tungsten and molybdenum with good refractory qualities are also being used, especially in attempts to die cast steel and cast iron. Dies can be single-cavity or multiple-cavity. Single-cavity dies are shown in Figures 11.13 and 11.14. Ejector pins are required to remove the part from the die when it opens, as in our diagrams. These pins push the part away from the mold surface so that it can be removed. Lubricants must also be sprayed into the cavities to prevent sticking. Because the die materials have no natural porosity and the molten metal rapidly flows into the die during injection, venting holes and passageways must be built into the dies at the parting line to evacuate the air and gases in the cavity. The vents are quite small; yet they fill with metal during injection. This metal must later be trimmed from the part. Also, formation of flash is common in die casting, in which the liquid metal under high pressure squeezes into the small space between the die halves at the parting line or into the clearances around the cores and ejector pins. This flash must be trimmed from the casting, along with the sprue and gating system. Advantages of die casting include (1) high production rates possible; (2) economical for large production quantities; (3) close tolerances possible, on the order of 0.076 mm (0.003 FIGURE 11.14 Cycle in cold-chamber casting: (1) with die closed and ram withdrawn, molten metal is poured into the chamber; (2) ram forces metal to flow into die, maintaining pressure during cooling and solidification; and (3) ram is withdrawn, die is opened, and part is ejected. (Gating system is simplified.) E1C11 11/10/2009 242 15:0:1 Page 242 Chapter 11/Metal Casting Processes in) for small parts; (4) good surface finish; (5) thin sections are possible, down to about 0.5 mm (0.020 in); and (6) rapid cooling provides small grain size and good strength to the casting. The limitation of this process, in addition to the metals cast, is the shape restriction. The part geometry must allow for removal from the die cavity. 11.3.4 SQUEEZE CASTING AND SEMISOLID METAL CASTING These are two processes that are often associated with die casting. Squeeze casting is a combination ofcastingandforging(Section 19.3)inwhich amolten metalispoured intoa preheated lower die, and the upper die is closed to create the mold cavity after solidification begins. This differs from the usualpermanent-mold casting processin which the die halves are closed prior to pouring or injection. Owing to the hybrid nature of the process, it is also known as liquid–metal forging. The pressure applied by the upper die in squeeze casting causes the metal to completely fill the cavity, resulting in good surface finish and low shrinkage. The required pressures are significantly less than in forging of a solid metal billet and much finer surface detail can be imparted by the die than in forging. Squeeze casting can be used for both ferrous and non-ferrous alloys, but aluminum and magnesium alloys are the most common due to their lower melting temperatures. Automotive parts are a common application. Semi-solid metal casting is a family of net-shape and near net-shape processes performed on metal alloys at temperatures between the liquidus and solidus (Section 10.3.1). Thus the alloy is a mixture of solid and molten metals during casting, like a slurry; it is in the mushy state. In order to flow properly, the mixture must consist of solid metal globules in a liquid rather than the more typical dendritic solid shapes that form during freezing of a molten metal. This is achieved by forcefully stirring the slurry to prevent dendrite formation and instead encourage the spherical shapes, which in turn reduces the viscosity of the work metal. Advantages of semisolid metal casting include the following [16]: (1) complex part geometries, (2) thin walls in parts, (3) close tolerances, (4) zero or low porosity, resulting in high strength of the casting. There are several forms of semisolid metal casting. When applied to aluminum, the terms thixocasting and rheocasting are used. The prefix in thixocasting is derived from the word thixotropy, which refers to the decrease in viscosity of some fluid-like materials when agitated. The prefix in rheocasting comes from rheology, the science that relates deformation and flow of materials. In thixocasting, the starting work material is a precast billet that has a nondendritic microstructure; this is heated into the semisolid temperature range and injected into a mold cavity using die casting equipment. In rheocasting, a semisolid slurry is injected into the mold cavity by a die casting machine, very much like conventional die casting. The difference is that the starting metal in rheocasting is at a temperature between the solidus and liquidus rather than above the liquidus. And the mushy mixture is agitated to prevent dendrite formation. When applied to magnesium, the term is thixomolding, which utilizes equipment similar to an injection-molding machine (Section 13.6.3). Magnesium alloy granules are fed into a barrel and propelled forward by a rotating screw as they are heated into the semisolid temperature range. The required globular form of the solid phase is accomplished by the mixing action of the rotating screw. The slurry is then injected into the mold cavity by a linear forward movement of the screw. 11.3.5 CENTRIFUGAL CASTING Centrifugal casting refers to several casting methods in which the mold is rotated at high speed so that centrifugal force distributes the molten metal to the outer regions of the die cavity. The group includes (1) true centrifugal casting, (2) semicentrifugal casting, and (3) centrifuge casting. E1C11 11/10/2009 15:0:1 Page 243 Section 11.3/Permanent-Mold Casting Processes 243 FIGURE 11.15 Setup for true centrifugal casting. True Centrifugal Casting In true centrifugal casting, molten metal is poured into a rotating mold to produce a tubular part. Examples of parts made by this process include pipes, tubes, bushings, and rings. One possible setup is illustrated in Figure 11.15. Molten metal is poured into a horizontal rotating mold at one end. In some operations, mold rotation commences after pouring has occurred rather than beforehand. The high-speed rotation results in centrifugal forces that cause the metal to take the shape of the mold cavity. Thus, the outside shape of the casting can be round, octagonal, hexagonal, and so on. However, the inside shape of the casting is (theoretically) perfectly round, due to the radially symmetric forces at work. Orientation of the axis of mold rotation can be either horizontal or vertical, the former being more common. Let us consider how fast the mold must rotate in horizontal centrifugal casting for the process to work successfully. Centrifugal force is defined by this physics equation: F¼ mv2 R ð11:2Þ where F ¼ force, N (lb); m ¼ mass, kg (lbm); v ¼ velocity, m/s (ft/sec); and R ¼ inside radius of the mold, m (ft). The force of gravity is its weight W ¼ mg, where W is given in kg (lb), and g ¼ acceleration of gravity, 9.8 m/s2 (32.2 ft/sec2). The so-called G-factor GF is the ratio of centrifugal force divided by weight: GF ¼ mv2 mv2 v2 ¼ ¼ R Rmg Rg ð11:3Þ Velocity v can be expressed as 2pRN=60 ¼ pRN=30, where N ¼ rotational speed, rev/min. Substituting this expression into Eq. (11.3), we obtain  R pN 30 GF ¼ g 2 ð11:4Þ Rearranging this to solve for rotational speed N, and using diameter D rather than radius in the resulting equation, we have 30 N¼ p rffiffiffiffiffiffiffiffiffiffiffiffiffi 2gGF D ð11:5Þ where D ¼ inside diameter of the mold, m (ft). If the G-factor is too low in centrifugal casting, the liquid metal will not remain forced against the mold wall during the upper half of the circular path but will ‘‘rain’’ inside the cavity. Slipping occurs between the molten metal and the mold wall, which means that the rotational speed of the metal is less than that of the mold. On an empirical basis, values of GF ¼ 60 to 80 are found to be appropriate for horizontal centrifugal casting [2], although this depends to some extent on the metal being cast. E1C11 11/10/2009 244 15:0:2 Page 244 Chapter 11/Metal Casting Processes Example 11.2 Rotation Speed in True Centrifugal Casting A true centrifugal casting operation is to be performed horizontally to make copper tube sections with OD ¼ 25 cm and ID ¼ 22.5 cm. What rotational speed is required if a Gfactor of 65 is used to cast the tubing? Solution: The inside diameterof the mold D ¼ OD ofthe casting ¼25 cm ¼0.25 m. Wecan compute the required rotational speed from Eq. (11.5) as follows: 30 N¼ p rffiffiffiffiffiffiffiffiffiffiffiffiffiffiffiffiffiffiffiffiffi 2(9:8)(26) ¼ 61:7 rev/min: 0:25 n In vertical centrifugal casting, the effect of gravity acting on the liquid metal causes the casting wall to be thicker at the base than at the top. The inside profile of the casting wall takes on a parabolic shape. The difference in inside radius between top and bottom is related to speed of rotation as follows: 30 N¼ p sffiffiffiffiffiffiffiffiffiffiffiffiffiffiffiffiffi 2gL R2t  R2b ð11:6Þ where L ¼ vertical length of the casting, m (ft); Rt ¼ inside radius at the top of the casting, m (ft); and Rb ¼ inside radius at the bottom of the casting, m (ft). Equation (11.6) can be used to determine the required rotational speed for vertical centrifugal casting, given specifications on the inside radii at top and bottom. One can see from the formula that for Rt to equal Rb, the speed of rotation N would have to be infinite, which is impossible of course. As a practical matter, part lengths made by vertical centrifugal casting are usually no more than about twice their diameters. This is quite satisfactory for bushings and other parts that have large diameters relatively to their lengths, especially if machining will be used to accurately size the inside diameter. Castings made by true centrifugal casting are characterized by high density, especially in the outer regions of the part where F is greatest. Solidification shrinkage at the exterior of the cast tube is not a factor, because the centrifugal force continually reallocates molten metal toward the mold wall during freezing. Any impurities in the casting tend to be on the inner wall and can be removed by machining if necessary. Semicentrifugal Casting In this method, centrifugal force is used to produce solid castings, as in Figure 11.16, rather than tubular parts. The rotation speed in semicentrifugal casting is usually set so that G-factors of around 15 are obtained [2], and the molds are FIGURE 11.16 casting. Semicentrifugal E1C11 11/10/2009 15:0:2 Page 245 Section 11.4/Foundry Practice 245 FIGURE 11.17 (a) Centrifuge casting—centrifugal force causes metal to flow to the mold cavities away from the axis of rotation; and (b) the casting. designed with risers at the center to supply feed metal. Density of metal in the final casting is greater in the outer sections than at the center of rotation. The process is often used on parts in which the center of the casting is machined away, thus eliminating the portion of the casting where the quality is lowest. Wheels and pulleys are examples of castings that can be made by this process. Expendable molds are often used in semicentrifugal casting, as suggested by our illustration of the process. Centrifuge Casting In centrifuge casting, Figure 11.17, the mold is designed with part cavities located away from the axis of rotation, so that the molten metal poured into the mold is distributed to these cavities by centrifugal force. The process is used for smaller parts, and radial symmetry of the part is not a requirement as it is for the other two centrifugal casting methods. 11.4 FOUNDRY PRACTICE In all casting processes, the metal must be heated to the molten state to be poured or otherwise forced into the mold. Heating and melting are accomplished in a furnace. This section covers the various types of furnaces used in foundries and the pouring practices for delivering the molten metal from furnace to mold. 11.4.1 FURNACES The types of furnaces most commonly used in foundries are (1) cupolas, (2) direct fuel-fired furnaces, (3) crucible furnaces, (4) electric-arc furnaces, and (5) induction furnaces. Selection of the most appropriate furnace type depends on factors such as the casting alloy; its melting and pouring temperatures; capacity requirements of the furnace; costs of investment, operation, and maintenance; and environmental pollution considerations. Cupolas A cupola is a vertical cylindrical furnace equipped with a tapping spout near its base. Cupolas are used only for melting cast irons, and although other furnaces are also used, the largest tonnage of cast iron is melted in cupolas. General construction and operating features of the cupola are illustrated in Figure 11.18. It consists of a large shell E1C11 11/10/2009 246 15:0:2 Page 246 Chapter 11/Metal Casting Processes FIGURE 11.18 Cupola used for melting cast iron. Furnace shown is typical for a small foundry and omits details of emissions control system required in a modern cupola. of steel plate lined with refractory. The ‘‘charge,’’ consisting of iron, coke, flux, and possible alloying elements, is loaded through a charging door located less than halfway up the height of the cupola. The iron is usually a mixture of pig iron and scrap (including risers, runners, and sprues left over from previous castings). Coke is the fuel used to heat the furnace. Forced air is introduced through openings near the bottom of the shell for combustion of the coke. The flux is a basic compound such as limestone that reacts with coke ash and other impurities to form slag. The slag serves to cover the melt, protecting it from reaction with the environment inside the cupola and reducing heat loss. As the mixture is heated and melting of the iron occurs, the furnace is periodically tapped to provide liquid metal for the pour. Direct Fuel-Fired Furnaces A direct fuel-fired furnace contains a small open-hearth, in which the metal charge is heated by fuel burners located on the side of the furnace. The roof of the furnace assists the heating action by reflecting the flame down against the charge. Typical fuel is natural gas, and the combustion products exit the furnace through a stack. At the bottom of the hearth is a tap hole to release the molten metal. Direct fuelfired furnaces are generally used in casting for melting nonferrous metals such as copperbase alloys and aluminum. Crucible Furnaces These furnaces melt the metal without direct contact with a burning fuel mixture. For this reason, they are sometimes called indirect fuel-fired furnaces. Three types of crucible furnaces are used in foundries: (a) lift-out type, (b) stationary, and E1C11 11/10/2009 15:0:2 Page 247 Section 11.4/Foundry Practice FIGURE 11.19 247 Three types of crucible furnaces: (a) lift-out crucible, (b) stationary pot, and (c) tilting-pot furnace. (c) tilting, illustrated in Figure 11.19. They all utilize a container (the crucible) made out of a suitable refractory material (e.g., a clay–graphite mixture) or high-temperature steel alloy to hold the charge. In the lift-out crucible furnace, the crucible is placed in a furnace and heated sufficiently to melt the metal charge. Oil, gas, or powdered coal are typical fuels for these furnaces. When the metal is melted, the crucible is lifted out of the furnace and used as a pouring ladle. The other two types, sometimes referred to as pot furnaces, have the heating furnace and container as one integral unit. In the stationary pot furnace, the furnace is stationary and the molten metal is ladled out of the container. In the tiltingpot furnace, the entire assembly can be tilted for pouring. Crucible furnaces are used for nonferrous metals such as bronze, brass, and alloys of zinc and aluminum. Furnace capacities are generally limited to several hundred pounds. Electric-Arc Furnaces In this furnace type, the charge is melted by heat generated from an electric arc. Various configurations are available, with two or three electrodes (see Figure 6.9). Power consumption is high, but electric-arc furnaces can be designed for high melting capacity (23,000–45,000 kg/hr or 25–50 tons/hr), and they are used primarily for casting steel. Induction Furnaces An induction furnace uses alternating current passing through a coil to develop a magnetic field in the metal, and the resulting induced current causes rapid heating and melting of the metal. Features of an induction furnace for foundry operations are illustrated in Figure 11.20. The electromagnetic force field causes a mixing FIGURE 11.20 Induction furnace. E1C11 11/10/2009 248 15:0:2 Page 248 Chapter 11/Metal Casting Processes action to occur in the liquid metal. Also, since the metal does not come in direct contact with the heating elements, the environment in which melting takes place can be closely controlled. All of this results in molten metals of high quality and purity, and induction furnaces are used for nearly any casting alloy when these requirements are important. Melting steel, cast iron, and aluminum alloys are common applications in foundry work. 11.4.2 POURING, CLEANING, AND HEAT TREATMENT Moving the molten metal from the melting furnace to the mold is sometimes done using crucibles. More often, the transfer is accomplished by ladles of various kinds. These ladles receive the metal from the furnace and allow for convenient pouring into the molds. Two common ladles are illustrated in Figure 11.21, one for handling large volumes of molten metal using an overhead crane, and the other a ‘‘two-man ladle’’ for manually moving and pouring smaller amounts. One of the problems in pouring is that oxidized molten metal can be introduced into the mold. Metal oxides reduce product quality, perhaps rendering the casting defective, so measures are taken to minimize the entry of these oxides into the mold during pouring. Filters are sometimes used to catch the oxides and other impurities as the metal is poured from the spout, and fluxes are used to cover the molten metal to retard oxidation. In addition, ladles have been devised to pour the liquid metal from the bottom, since the top surface is where the oxides accumulate. After the casting has solidified and been removed from the mold, a number of additional steps are usually required. These operations include (1) trimming, (2) removing the core, (3) surface cleaning, (4) inspection, (5) repair, if required, and (6) heat treatment. Steps (1) through (5) are collectively referred to in foundry work as ‘‘cleaning.’’ The extent to which these additional operations are required varies with casting processes and metals. When required, they are usually labor intensive and costly. Trimming involves removal of sprues, runners, risers, parting-line flash, fins, chaplets, and any other excess metal from the cast part. In the case of brittle casting alloys and when the cross sections are relatively small, these appendages on the casting can be broken off. Otherwise, hammering, shearing, hack-sawing, band-sawing, abrasive wheel cutting, or various torch cutting methods are used. If cores have been used to cast the part, they must be removed. Most cores are chemically bonded or oil-bonded sand, and they often fall out of the casting as the binder deteriorates. In some cases, they are removed by shaking the casting, either manually or mechanically. In rare instances, cores are removed by chemically dissolving the bonding agent used in the sand core. Solid cores must be hammered or pressed out. FIGURE 11.21 Two common types of ladles: (a) crane ladle and (b) two-man ladle. E1C11 11/10/2009 15:0:2 Page 249 Section 11.5/Casting Quality 249 Surface cleaning is most important in the case of sand casting. In many of the other casting methods, especially the permanent-mold processes, this step can be avoided. Surface cleaning involves removal of sand from the surface of the casting and otherwise enhancing the appearance of the surface. Methods used to clean the surface include tumbling, air-blasting with coarse sand grit or metal shot, wire brushing, buffing, and chemical pickling (Chapter 28). Defects are possible in casting, and inspection is needed to detect their presence. We consider these quality issues in the following section. Castings are often heat treated to enhance their properties, either for subsequent processing operations such as machining or to bring out the desired properties for application of the part. 11.5 CASTING QUALITY There are numerous opportunities for things to go wrong in a casting operation, resulting in quality defects in the cast product. In this section, we compile a list of the common defects that occur in casting, and we indicate the inspection procedures to detect them. Casting Defects Some defects are common to any and all casting processes. These defects are illustrated in Figure 11.22 and briefly described in the following: (a) Misruns, which are castings that solidify before completely filling the mold cavity. Typical causes include (1) fluidity of the molten metal is insufficient, (2) pouring temperature is too low, (3) pouring is done too slowly, and/or (4) cross-section of the mold cavity is too thin. (b) Cold Shuts, which occur when two portions of the metal flow together but there is a lack of fusion between them due to premature freezing. Its causes are similar to those of a misrun. (c) Cold shots, which result from splattering during pouring, causing the formation of solid globules of metal that become entrapped in the casting. Pouring procedures and gating system designs that avoid splattering can prevent this defect. FIGURE 11.22 Some common defects in castings: (a) misrun, (b) cold shut, (c) cold shot, (d) shrinkage cavity, (e) microporosity, and (f) hot tearing. E1C11 11/10/2009 250 15:0:3 Page 250 Chapter 11/Metal Casting Processes (d) Shrinkage cavity is a depression in the surface or an internal void in the casting, caused by solidification shrinkage that restricts the amount of molten metal available in the last region to freeze. It often occurs near the top of the casting, in which case it is referred to as a ‘‘pipe.’’ See Figure 10.8(3). The problem can often be solved by proper riser design. (e) Microporosity consists of a network of small voids distributed throughout the casting caused by localized solidification shrinkage of the final molten metal in the dendritic structure. The defect is usually associated with alloys, because of the protracted manner in which freezing occurs in these metals. (f) Hot tearing, also called hot cracking, occurs when the casting is restrained from contraction by an unyielding mold during the final stages of solidification or early stages of cooling after solidification. The defect is manifested as a separation of the metal (hence, the terms tearing and cracking) at a point of high tensile stress caused by the metal’s inability to shrink naturally. In sand casting and other expendable-mold processes, it is prevented by compounding the mold to be collapsible. In permanent-mold processes, hot tearing is reduced by removing the part from the mold immediately after solidification. Some defects are related to the use of sand molds, and therefore they occur only in sand castings. To a lesser degree, other expendable-mold processes are also susceptible to these problems. Defects found primarily in sand castings are shown in Figure 11.23 and described here: (a) Sand blow is a defect consisting of a balloon-shaped gas cavity caused by release of mold gases during pouring. It occurs at or below the casting surface near the top of the casting. Low permeability, poor venting, and high moisture content of the sand mold are the usual causes. (b) Pinholes, also caused by release of gases during pouring, consist of many small gas cavities formed at or slightly below the surface of the casting. (c) Sand wash, which is an irregularity in the surface of the casting that results from erosion of the sand mold during pouring, and the contour of the erosion is formed in the surface of the final cast part. FIGURE 11.23 Common defects in sand castings: (a) sand blow, (b) pin holes, (c) sand wash, (d) scabs, (e) penetration, (f) mold shift, (g) core shift, and (h) mold crack. E1C11 11/10/2009 15:0:3 Page 251 Section 11.6/Metals for Casting 251 (d) Scabs are rough areas on the surface of the casting due to encrustations of sand and metal. It is caused by portions of the mold surface flaking off during solidification and becoming imbedded in the casting surface. (e) Penetration refers to a surface defect that occurs when the fluidity of the liquid metal is high, and it penetrates into the sand mold or sand core. Upon freezing, the casting surface consists of a mixture of sand grains and metal. Harder packing of the sand mold helps to alleviate this condition. (f) Mold shift refers to a defect caused by a sidewise displacement of the mold cope relative to the drag, the result of which is a step in the cast product at the parting line. (g) Core shift is similar to mold shift, but it is the core that is displaced, and the displacement is usually vertical. Core shift and mold shift are caused by buoyancy of the molten metal (Section 11.1.3). (h) Mold crack occurs when mold strength is insufficient, and a crack develops, into which liquid metal can seep to form a ‘‘fin’’ on the final casting. Inspection Methods Foundry inspection procedures include (1) visual inspection to detect obvious defects such as misruns, cold shuts, and severe surface flaws; (2) dimensional measurements to ensure that tolerances have been met; and (3) metallurgical, chemical, physical, and other tests concerned with the inherent quality of the cast metal [7]. Tests in category (3) include: (a) pressure testing—to locate leaks in the casting; (b) radiographic methods, magnetic particle tests, the use of fluorescent penetrants, and supersonic testing—to detect either surface or internal defects in the casting; and (c) mechanical testing to determine properties such as tensile strength and hardness. If defects are discovered but are not too serious, it is often possible to save the casting by welding, grinding, or other salvage methods to which the customer has agreed. 11.6 METALS FOR CASTING Most commercial castings are made of alloys rather than pure metals. Alloys are generally easier to cast, and properties of the resulting product are better. Casting alloys can be classified as ferrous or nonferrous. The ferrous category is subdivided into cast iron and cast steel. Ferrous Casting Alloys: Cast Iron Cast iron is the most important of all casting alloys (Historical Note 11.3). The tonnage of cast iron castings is several times that of all other metals combined. There are several types of cast iron: (1) gray cast iron, (2) nodular iron, (3) white cast iron, (4) malleable iron, and (5) alloy cast irons (Section 6.2.4). Typical pouring temperatures for cast iron are around 1400 C (2552 F), depending on composition. Ferrous Casting Alloys: Steel The mechanical properties of steel make it an attractive engineering material (Section 6.2.3), and the capability to create complex geometries makes casting an appealing process. However, great difficulties are faced by the foundry specializing in steel. First, the melting point of steel is considerably higher than for most other metals that are commonly cast. The solidification range for low carbon steels (Figure 6.4) begins at just under 1540 C (2804 F). This means that the pouring temperature required for steel is very high—about 1650 C (3002 F). At these high temperatures, steel is chemically very reactive. It readily oxidizes, so special procedures must be used during melting and pouring to isolate the molten metal from air. Also, molten steel has relatively poor fluidity, and this limits the design of thin sections in components cast out of steel. E1C11 11/10/2009 252 15:0:3 Page 252 Chapter 11/Metal Casting Processes Historical Note 11.3 I Early cast iron products n the early centuries of casting, bronze and brass were preferred over cast iron as foundry metals. Iron was more difficult to cast, due to its higher melting temperatures and lack of knowledge about its metallurgy. Also, there was little demand for cast iron products. This all changed starting in the sixteenth and seventeenth centuries. The art of sand-casting iron entered Europe from China, where iron was cast in sand molds more than 2500 years ago. In 1550 the first cannons were cast from iron in Europe. Cannon balls for these guns were made of cast iron starting around 1568. Guns and their projectiles created a large demand for cast iron. But these items were for military rather than civilian use. Two cast iron products that became significant to the general public in the sixteenth and seventeenth centuries were the cast iron stove and cast iron water pipe. As unspectacular a product as it may seem today, the cast iron stove brought comfort, health, and improved living conditions to many people in Europe and America. During the 1700s, the manufacture of cast iron stoves was one of the largest and most profitable industries on these two continents. The commercial success of stove making was due to the large demand for the product and the art and technology of casting iron that had been developed to produce it. Cast iron water pipe was another product that spurred the growth of the iron casting industry. Until the advent of cast iron pipes, a variety of methods had been tried to supply water directly to homes and shops, including hollow wooden pipes (which quickly rotted), lead pipes (too expensive), and open trenches (susceptible to pollution). Development of the iron casting process provided the capability to fabricate water pipe sections at relatively low cost. Cast iron water pipes were used in France starting in 1664, and later in other parts of Europe. By the early 1800s, cast iron pipe lines were being widely installed in England for water and gas delivery. The first significant water pipe installation in the United States was in Philadelphia in 1817, using pipe imported from England. Several characteristics of steel castings make it worth the effort to solve these problems. Tensile strength is higher than for most other casting metals, ranging upward from about 410 MPa (59,465 lb/in2) [9]. Steel castings have better toughness than most other casting alloys. The properties of steel castings are isotropic; strength is virtually the same in all directions. By contrast, mechanically formed parts (e.g., rolling, forging) exhibit directionality in their properties. Depending on the requirements of the product, isotropic behavior of the material may be desirable. Another advantage of steel castings is ease of welding. They can be readily welded without significant loss of strength, to repair the casting, or to fabricate structures with other steel components. Nonferrous Casting Alloys Nonferrous casting metals include alloys of aluminum, magnesium, copper, tin, zinc, nickel, and titanium (Section 6.3). Aluminum alloys are generally considered to be very castable. The melting point of pure aluminum is 660 C (1112 F), so pouring temperatures for aluminum casting alloys are low compared to cast iron and steel. Their properties make them attractive for castings: light weight, wide range of strength properties attainable through heat treatment, and ease of machining. Magnesium alloys are the lightest of all casting metals. Other properties include corrosion resistance, as well as high strength-to-weight and stiffness-to-weight ratios. Copper alloys include bronze, brass, and aluminum bronze. Properties that make them attractive include corrosion resistance, attractive appearance, and good bearing qualities. The high cost of copper is a limitation on the use of its alloys. Applications include pipe fittings, marine propeller blades, pump components, and ornamental jewelry. Tin has the lowest melting point of the casting metals. Tin-based alloys are generally easy to cast. They have good corrosion resistant but poor mechanical strength, which limits their applications to pewter mugs and similar products not requiring high strength. Zinc alloys are commonly used in die casting. Zinc has a low melting point and good fluidity, making it highly castable. Its major weakness is low creep strength, so its castings cannot be subjected to prolonged high stresses. E1C11 11/10/2009 15:0:3 Page 253 Section 11.7/Product Design Considerations 253 Nickel alloys have good hot strength and corrosion resistance, which make them suited to high-temperature applications such as jet engine and rocket components, heat shields, and similar components. Nickel alloys also have a high melting point and are not easy to cast. Titanium alloys for casting are corrosion resistant and possess high strengthto-weight ratios. However, titanium has a high melting point, low fluidity, and a propensity to oxidize at high temperatures. These properties make it and its alloys difficult to cast. 11.7 PRODUCT DESIGN CONSIDERATIONS If casting is selected by the product designer as the primary manufacturing process for a particular component, then certain guidelines should be followed to facilitate production of the part and avoid many of the defects enumerated in Section 11.5. Some of the important guidelines and considerations for casting are presented here. å Geometric simplicity. Although casting is a process that can be used to produce complex part geometries, simplifying the part design will improve its castability. Avoiding unnecessary complexities simplifies mold making, reduces the need for cores, and improves the strength of the casting. å Corners. Sharp corners and angles should be avoided, because they are sources of stress concentrations and may cause hot tearing and cracks in the casting. Generous fillets should be designed on inside corners, and sharp edges should be blended. å Section thicknesses. Section thicknesses should be uniform in order to avoid shrinkage cavities. Thicker sections create hot spots in the casting, because greater volume requires more time for solidification and cooling. These are likely locations of shrinkage cavities. Figure 11.24 illustrates the problem and offers some possible solutions. å Draft. Part sections that project into the mold should have a draft or taper, as defined in Figure 11.25. In expendable-mold casting, the purpose of this draft is to facilitate removal of the pattern from the mold. In permanent-mold casting, its purpose is to aid in removal of the part from the mold. Similar tapers should be allowed if solid cores are used in the casting process. The required draft need only be about 1 for sand casting and 2 to 3 for permanent-mold processes. å Use of cores. Minor changes in part design can reduce the need for coring, as shown in Figure 11.25. å Dimensional tolerances. There are significant differences in the dimensional accuracies that can be achieved in castings, depending on which process is used. Table 11.2 provides a compilation of typical part tolerances for various casting processes and metals. å Surface finish. Typical surface roughness achieved in sand casting is around 6 mm (250 m-in). Similarly poor finishes are obtained in shell molding, while plaster-mold and investment casting produce much better roughness values: 0.75 mm (30 m-in). FIGURE 11.24 (a) Thick section at intersection can result in a shrinkage cavity. Remedies include (b) redesign to reduce thickness and (c) use of a core. E1C11 11/10/2009 254 15:0:3 Page 254 Chapter 11/Metal Casting Processes FIGURE 11.25 Design change to eliminate the need for using a core: (a) original design and (b) redesign. TABLE 11.2 Typical dimensional tolerances for various casting processes and metals. Casting Process Sand casting Aluminuma Cast iron Copper alloys Steel Shell molding Aluminuma Cast iron Copper alloys Steel Plaster mold Part Size Tolerance mm in Small Small Large Small Small Large 0.5 1.0 1.5 0.4 1.3 2.0 0.020 0.040 0.060 0.015 0.050 0.080 Small Small Small Small Small Large 0.25 0.5 0.4 0.8 0.12 0.4 0.010 0.020 0.015 0.030 0.005 0.015 Casting Process Part Size Tolerance mm in Permanent mold Aluminuma Cast iron Copper alloys Steel Small Small Small Small 0.25 0.8 0.4 0.5 0.010 0.030 0.015 0.020 Die casting Aluminuma Copper alloys Small Small 0.12 0.12 0.005 0.005 Investment a Aluminum Cast iron Copper alloys Steel Small Small Small Small 0.12 0.25 0.12 0.25  0.005 0.010 0.005 0.010 Compiled from [7], [15], and other sources. Values for aluminum also apply to magnesium. a Among the permanent-mold processes, die casting is noted for good surface finishes at around 1 mm (40 m-in). å Machining allowances. Tolerances achievable in many casting processes are insufficient to meet functional needs in many applications. Sand casting is the most prominent example of this deficiency. In these cases, portions of the casting must be machined to the required dimensions. Almost all sand castings must be machined to some extent in order for the part to be made functional. Therefore, additional material, called the machining allowance, is left on the casting for machining those surfaces where necessary. Typical machining allowances for sand castings range between 1.5 mm and 3 mm (0.06 in and 0.12 in). REFERENCES [1] Amstead, B. H., Ostwald, P. F., and Begeman, M. L. Manufacturing Processes. John Wiley & Sons, Inc., New York, 1987. [2] Beeley, P. R. Foundry Technology. Newnes-Butterworths, London, 1972. [3] Black, J, and Kohser, R. DeGarmo’s Materials and Processes in Manufacturing, 10th ed. John Wiley & Sons, Inc., Hoboken, New Jersey, 2008. [4] Datsko, J. Material Properties and Manufacturing Processes. John Wiley & Sons, Inc., New York, 1966. E1C11 11/10/2009 15:0:4 Page 255 Multiple Choice Quiz [5] Decker, R. F., D. M. Walukas, S. E. LeBeau, R. E. Vining, and N. D. Prewitt.‘‘Advances in Semi-Solid Molding,’’ Advanced Materials & Processes, April 2004, pp 41–42. [6] Flinn,R.A.FundamentalsofMetalCasting.American Foundrymen’s Society, Inc., Des Plaines, Illinois, 1987. [7] Heine, R. W., Loper, Jr., C. R., and Rosenthal, C. Principles of Metal Casting, 2nd ed. McGraw-Hill Book Co., New York, 1967. [8] Kotzin, E. L. Metalcasting & Molding Processes. American Foundrymen’s Society, Inc., Des Plaines, Illinois, 1981. [9] Metals Handbook, Vol. 15, Casting. ASM International, Materials Park, Ohio, 2008. [10] Mikelonis, P. J. (ed.). Foundry Technology. American Society for Metals, Metals Park, Ohio, 1982. 255 [11] Mueller, B. ‘‘Investment Casting Trends,’’ Advanced Materials & Processes, March 2005, pp. 30–32. [12] Niebel, B. W., Draper, A. B., Wysk, R. A. Modern Manufacturing Process Engineering. McGraw-Hill Book Co., New York, 1989. [13] Perry, M. C. ‘‘Investment Casting,’’ Advanced Materials & Processes, June 2008, pp. 31–33. [14] Simpson, B. L. History of the Metalcasting Industry. American Foundrymen’s Society, Inc., Des Plaines, Illinois, 1997. [15] Wick, C., Benedict, J. T., and Veilleux, R. F. Tool and Manufacturing Engineers Handbook, 4th ed. Vol. II, Forming, Ch. 16. Society of Manufacturing Engineers, Dearborn, Michigan, 1984. [16] Wikipedia. ‘‘Semi-solid metal casting.’’ Available at: http://en.wikipedia.org/wiki/Semisolid_metal_casting. REVIEW QUESTIONS 11.1. Name the two basic categories of casting processes. 11.2. There are various types of patterns used in sand casting. What is the difference between a split pattern and a match-plate pattern? 11.3. What is a chaplet? 11.4. What properties determine the quality of a sand mold for sand casting? 11.5. What is the Antioch process? 11.6. What is the difference between vacuum permanent-mold casting and vacuum molding? 11.7. What are the most common metals used in die casting? 11.8. Which die casting machines usually have a higher production rate, cold-chamber or hot-chamber, and why? 11.9. What is flash in die casting? 11.10. What is the difference between true centrifugal casting and semicentrifugal casting? 11.11. What is a cupola? 11.12. What are some of the operations required in sand casting after the casting is removed from the mold? 11.13. What are some of the general defects encountered in casting processes? Name and briefly describe three. 11.14. (Video) What is the composition of green sand in the green-sand molding process? 11.15. (Video) What are the advantages and disadvantages of sand casting over investment casting? 11.16. (Video) Explain the difference between horizontal and vertical die casting machines. Which is more popular? 11.17. (Video) Why are aluminum and copper alloys unsuitable for use in hot-chamber die casting? 11.18. (Video) According to the die casting video, what materials are most common for die casting dies? MULTIPLE CHOICE QUIZ There are 27 correct answers in the following multiple choice questions (some questions have multiple answers that are correct). To attain a perfect score on the quiz, all correct answers must be given. Each correct answer is worth 1 point. Each omitted answer or wrong answer reduces the score by 1 point, and each additional answer beyond the correct number of answers reduces the score by 1 point. Percentage score on the quiz is based on the total number of correct answers. 11.1. Which one of the following casting processes is the most widely used: (a) centrifugal casting, (b) die casting, (c) investment casting, (d) sand casting, or (e) shell casting? 11.2. In sand casting, the volumetric size of the pattern is (a) bigger than, (b) same size as, or (c) smaller than the cast part? 11.3. Silica sand has which one of the following compositions: (a) Al2O3, (b) SiO, (c) SiO2, or (d) SiSO4? 11.4. For which one of the following reasons is a green mold named: (a) green is the color of the mold, (b) moisture is contained in the mold, (c) mold is cured, or (d) mold is dry? E1C11 11/10/2009 256 15:0:4 Page 256 Chapter 11/Metal Casting Processes 11.5. Given that Wm ¼ weight of the molten metal displaced by a core and Wc ¼ weight of the core, the buoyancy force is which one of the following: (a) downward force ¼ Wm þ Wc, (b) downward force ¼ Wm  Wc, (c) upward force ¼ Wm þ Wc, or (d) upward force ¼ Wm  Wc? 11.6. Which of the following casting processes are expendable-mold operations (four correct answers): (a) centrifugal casting, (b) die casting, (c) investment casting, (d) low pressure casting, (e) sand casting, (f) shell molding, (g) slush casting, and (h) vacuum molding? 11.7. Shell molding is best described by which one of the following: (a) casting operation in which the molten metal has been poured out after a thin shell has been solidified in the mold, (b) casting process in which the mold is a thin shell of sand bonded by a thermosetting resin, (c) sand casting operation in which the pattern is a shell rather than a solid form, or (d) casting operation used to make artificial sea shells? 11.8. Investment casting is also known by which one of the following names: (a) fast-payback molding, (b) full-mold process, (c) lost-foam process, (d) lost-pattern process, or (e) lost-wax process? 11.9. In plaster-mold casting, the mold is made of which one of the following materials: (a) Al2O3, (b) CaSO4-H2O, (c) SiC, or (d) SiO2? 11.10. Which of the following qualifies as a precisioncasting process (two correct answers): (a) ingot 11.11. 11.12. 11.13. 11.14. 11.15. 11.16. casting, (b) investment casting, (c) plaster-mold casting, (d) sand casting, and (e) shell molding? Which of the following casting processes are permanent-mold operations (three correct answers): (a) centrifugal casting, (b) die casting, (c) expanded polystyrene process, (d) sand casting, (e) shell molding, (f) slush casting, and (g) vacuum molding. Which of the following metals would typically be used in die casting (three best answers): (a) aluminum, (b) cast iron, (c) steel, (d) tin, (e) tungsten, and (f) zinc? Which of the following are advantages of die casting over sand casting (four best answers): (a) better surface finish, (b) closer tolerances, (c) higher melting temperature metals, (d) higher production rates, (e) larger parts can be cast, and (f) mold can be reused? Cupolas are furnaces used to melt which of the following metals (one best answer): (a) aluminum, (b) cast iron, (c) steel, or (d) zinc? A misrun is which one of the following defects in casting: (a) globules of metal becoming entrapped in the casting, (b) metal is not properly poured into the downsprue, (c) metal solidifies before filling the cavity, (d) microporosity, and (e) ‘‘pipe’’ formation? Which one of the following casting metals is most important commercially: (a) aluminum and its alloys, (b) bronze, (c) cast iron, (d) cast steel, or (e) zinc alloys? PROBLEMS Buoyancy Force 11.1. An 92% aluminum-8% copper alloy casting is made in a sand mold using a sand core that weighs 20 kg. Determine the buoyancy force in Newtons tending to lift the core during pouring. 11.2. A sand core located inside a mold cavity has a volume of 157.0 in3. It is used in the casting of a cast iron pump housing. Determine the buoyancy force that will tend to lift the core during pouring. 11.3. Caplets are used to support a sand core inside a sand mold cavity. The design of the caplets and the manner in which they are placed in the mold cavity surface allows each caplet to sustain a force of 10 lb. Several caplets are located beneath the core to support it before pouring; and several other caplets are placed above the core to resist the buoyancy force during pouring. If the volume of the core ¼ 325 in3, and the metal poured is brass, determine the minimum number of caplets that should be placed (a) beneath the core, and (b) above the core. 11.4. A sand core used to form the internal surfaces of a steel casting experiences a buoyancy force of 23 kg. The volume of the mold cavity forming the outside surface of the casting ¼ 5000 cm3. What is the weight of the final casting? Ignore considerations of shrinkage. Centrifugal Casting 11.5. A horizontal true centrifugal casting operation will be used to make copper tubing. The lengths will be 1.5 m with outside diameter ¼ 15.0 cm, and inside diameter ¼ 12.5 cm. If the rotational speed of the pipe ¼ 1000 rev/min, determine the Gfactor. E1C11 11/10/2009 15:0:4 Page 257 Problems 11.6. A true centrifugal casting operation is to be performed in a horizontal configuration to make cast iron pipe sections. The sections will have a length ¼ 42.0 in, outside diameter ¼ 8.0 in, and wall thickness ¼ 0.50 in. If the rotational speed of the pipe ¼ 500 rev/min, determine the G-factor. Is the operation likely to be successful? 11.7. A horizontal true centrifugal casting process is used to make brass bushings with the following dimensions: length ¼ 10 cm, outside diameter ¼ 15 cm, and inside diameter ¼ 12 cm. (a) Determine the required rotational speed in order to obtain a G-factor of 70. (b) When operating at this speed, what is the centrifugal force per square meter (Pa) imposed by the molten metal on the inside wall of the mold? 11.8. True centrifugal casting is performed horizontally to make large diameter copper tube sections. The tubes have a length ¼ 1.0 m, diameter ¼ 0.25 m, and wall thickness ¼ 15 mm. (a) If the rotational speed of the pipe ¼ 700 rev/min, determine the G-factor on the molten metal. (b) Is the rotational speed sufficient to avoid ‘‘rain?’’ (c) What volume of molten metal must be poured into the mold to make the casting if solidification shrinkage and contraction after solidification are considered? Solidification shrinkage for copper ¼ 4.5%, and solid thermal contraction ¼ 7.5%. 11.9. If a true centrifugal casting operation were to be performed in a space station circling the Earth, how would weightlessness affect the process? 11.10. A horizontal true centrifugal casting process is used to make aluminum rings with the following dimensions: length ¼ 5 cm, outside diameter ¼ 65 cm, and inside diameter ¼ 60 cm. (a) Determine the rotational 11.11. 11.12. 11.13. 11.14. 11.15. 257 speed that will provide a G-factor ¼ 60. (b) Suppose that the ring were made out of steel instead of aluminum. If the rotational speed computed in part (a) were used in the steel casting operation, determine the G-factor and (c) centrifugal force per square meter (Pa) on the mold wall. (d) Would this rotational speed result in a successful operation? For the steel ring of preceding Problem 11.10(b), determine the volume of molten metal that must be poured into the mold, given that the liquid shrinkage is 0.5%, solidification shrinkage ¼ 3%, and solid contraction after freezing ¼ 7.2%. A horizontal, true centrifugal casting process is used to make lead pipe for chemical plants. The pipe has length ¼ 0.5 m, outside diameter ¼ 70 mm, and wall thickness ¼ 6.0 mm. Determine the rotational speed that will provide a G-factor ¼ 60. A vertical, true centrifugal casting process is used to make tube sections with length ¼ 10.0 in and outside diameter ¼ 6.0 in. The inside diameter of the tube ¼ 5.5 in at the top and 5.0 in at the bottom. At what speed must the tube be rotated during the operation in order to achieve these specifications? A vertical, true centrifugal casting process is used to produce bushings that are 200 mm long and 200 mm in outside diameter. If the rotational speed during solidification is 500 rev/min, determine the inside diameter at the top of the bushing if the inside diameter at the bottom is 150 mm. A vertical, true centrifugal casting process is used to cast brass tubing that is 15.0 in long and whose outside diameter ¼ 8.0 in. If the speed of rotation during solidification is 1000 rev/min, determine the inside diameters at the top and bottom of the tubing if the total weight of the final casting ¼ 75.0 lbs. Defects and Design Considerations 11.16. The housing for a certain machinery product is made of two components, both aluminum castings. The larger component has the shape of a dish sink, and the second component is a flat cover that is attached to the first component to create an enclosed space for the machinery parts. Sand casting is used to produce the two castings, both of which are plagued by defects in the form of misruns and cold shuts. The foreman complains that the parts are too thin, and that is the reason for the defects. However, it is known that the same components are cast successfully in other foundries. What other explanation can be given for the defects? 11.17. A large, steel sand casting shows the characteristic signs of penetration defect: a surface consisting of a mixture of sand and metal. (a) What steps can be taken to correct the defect? (b) What other possible defects might result from taking each of these steps? E1C12 11/10/2009 15:3:49 12 Page 258 GLASSWORKING Chapter Contents 12.1 Raw Materials Preparation and Melting 12.2 Shaping Processes in Glassworking 12.2.1 Shaping of Piece Ware 12.2.2 Shaping of Flat and Tubular Glass 12.2.3 Forming of Glass Fibers 12.3 Heat Treatment and Finishing 12.3.1 Heat Treatment 12.3.2 Finishing 12.4 Product Design Considerations Glass products are commercially manufactured in an almost unlimited variety of shapes. Many are produced in very large quantities, such as light bulbs, beverage bottles, and window glass. Others, such as giant telescope lenses, are made individually. Glass is one of three basic types of ceramics (Chapter 7). It is distinguished by its noncrystalline (vitreous) structure, whereas the other ceramic materials have a crystalline structure. The methods by which glass is shaped into useful products are quite different from those used for the other types. In glassworking, the principal starting material is silica (SiO2); this is usually combined with other oxide ceramics, which form glasses. The starting material is heated to transform it from a hard solid into a viscous liquid; it is then shaped into the desired geometry while in this highly plastic or fluid condition. When cooled and hard, the material remains in the glassy state rather than crystallizing. The typical manufacturing sequence in glassworking consists of the steps pictured in Figure 12.1. Shaping is accomplished by various processes, including casting, pressing-and-blowing (to produce bottles and other containers), and rolling (to make plate glass). A finishing step is required for certain products. 12.1 RAW MATERIALS PREPARATION AND MELTING The main component in nearly all glasses is silica, the primary source of which is natural quartz in sand. The sand must be washed and classified. Washing removes impurities such as clay and certain minerals that would cause undesirable coloring of the glass. Classifying the sand means grouping the grains according to size. The most desirable particle size for glassmaking is in the range of 0.1 to 0.6 mm (0.004 to 258 E1C12 11/10/2009 15:3:49 Page 259 Section 12.2/Shaping Processes in Glassworking 259 FIGURE 12.1 The typical process sequence in glassworking: (1) preparation of raw materials and melting, (2) shaping, and (3) heat treatment. 0.025 in) [3]. The various other components, such as soda ash (source of Na2O), limestone (source of CaO), aluminum oxide, potash (source of K2O), and other minerals are added in the proper proportions to achieve the desired composition. The mixing is usually done in batches, in amounts that are compatible with the capacities of available melting furnaces. Recycled glass is usually added to the mixture in modern practice. In addition to preserving the environment, recycled glass facilitates melting. Depending on the amount of waste glass available and the specifications of the final composition, the proportion of recycled glass may be up to 100%. The batch of starting materials to be melted is referred to as a charge, and the procedure of loading it into the melting furnace is called charging the furnace. Glassmelting furnaces can be divided into the following types [3]: (1) pot furnaces—ceramic pots of limited capacity in which melting occurs by heating the walls of the pot; (2) day tanks—larger capacity vessels for batch production in which heating is done by burning fuels above the charge; (3) continuous tank furnaces—long tank furnaces in which raw materials are fed in one end, and melted as they move to the other end where molten glass is drawn out for high production; and (4) electric furnaces of various designs for a wide range of production rates. Glass melting is generally carried out at temperatures around 1500 C to 1600 C (2700 F to 2900 F). The melting cycle for a typical charge takes 24 to 48 hours. This is the time required for all of the sand grains to become a clear liquid and the molten glass to be refined and cooled to the appropriate temperature for working. Molten glass is a viscous liquid, the viscosity being inversely related to temperature. Because the shaping operation immediately follows the melting cycle, the temperature at which the glass is tapped from the furnace depends on the viscosity required for the subsequent process. 12.2 SHAPING PROCESSES IN GLASSWORKING The major categories of glass products were identified in Section 7.4.2 as window glass, containers, light bulbs, laboratory glassware, glass fibers, and optical glass. Despite the variety represented by this list, the shaping processes to fabricate these products can be grouped into only three categories: (1) discrete processes for piece ware, which includes bottles, light bulbs, and other individual items; (2) continuous processes for making flat glass (sheet and plate glass for windows) and tubing (for laboratory ware and fluorescent lights); and (3) fiber-making processes to produce fibers for insulation, fiberglass composite materials, and fiber optics. 12.2.1 SHAPING OF PIECE WARE The ancient methods of hand-working glass, such as glass blowing, were briefly described in Historical Note 7.3. Handicraft methods are still employed today for making glassware items of high value in small quantities. Most of the processes discussed in this section are E1C12 11/10/2009 260 15:3:49 Page 260 Chapter 12/Glassworking FIGURE 12.2 Spinning of funnel-shaped glass parts: (1) gob of glass dropped into mold; and (2) rotation of mold to cause spreading of molten glass on mold surface. highly mechanized technologies for producing discrete pieces such as jars, bottles, and light bulbs in high quantities. Spinning Glass spinning is similar to centrifugal casting of metals, and is also known by that name in glassworking. It is used to produce funnel-shaped components such as the back sections of cathode ray tubes for televisions and computer monitors. The setup is pictured in Figure 12.2. A gob of molten glass is dropped into a conical mold made of steel. The mold is rotated so that centrifugal force causes the glass to flow upward and spread itself on the mold surface. The faceplate (i.e., the front viewing screen) is later assembled to the funnel using a sealing glass of low melting point. Pressing This is a widely used process for mass producing glass pieces such as dishes, bake ware, headlight lenses, TV tube faceplates, and similar items that are relatively flat. The process is illustrated and described in Figure 12.3. The large quantities of most pressed products justify a high level of automation in this production sequence. Blowing Several shaping sequences include blowing as one or more of the steps. Instead of a manual operation, blowing is performed on highly automated equipment. The two sequences we describe here are the press-and-blow and blow-and-blow methods. As the name indicates, the press-and-blow method is a pressing operation followed by a blowing operation, as portrayed in Figure 12.4. The process is suited to the production of wide-mouth containers. A split mold is used in the blowing operation for part removal. The blow-and-blow method is used to produce smaller-mouthed bottles. The sequence is similar to the preceding, except that two (or more) blowing operations FIGURE 12.3 Pressing of a flat glass piece: (1) a gob of glass fed into mold from the furnace; (2) pressing into shape by plunger; and (3) plunger is retracted and the finished product is removed. Symbols v and F indicate motion (v ¼ velocity) and applied force, respectively. E1C12 11/10/2009 15:3:49 Page 261 Section 12.2/Shaping Processes in Glassworking 261 FIGURE 12.4 Press-and-blow forming sequence: (1) molten gob is fed into mold cavity; (2) pressing to form a parison; (3) the partially formed parison, held in a neck ring, is transferred to the blow mold; and (4) blown into final shape. Symbols v and F indicate motion (v ¼ velocity) and applied force, respectively. are used rather than pressing and blowing. There are variations to the process, depending on the geometry of the product, with one possible sequence shown in Figure 12.5. Reheating is sometimes required between blowing steps. Duplicate and triplicate molds are sometimes used along with matching gob feeders to increase production rates. Pressand-blow and blow-and-blow methods are used to make jars, beverage bottles, incandescent light bulb enclosures, and similar geometries. FIGURE 12.5 Blow-and-blow forming sequence: (1) gob is fed into inverted mold cavity; (2) mold is covered; (3) first blowing step; (4) partially formed piece is reoriented and transferred to second blow mold; and (5) blown to final shape. E1C12 11/10/2009 262 15:3:49 Page 262 Chapter 12/Glassworking Casting If the molten glass is sufficiently fluid, it can be poured into a mold. Relatively massive objects, such as astronomical lenses and mirrors, are made by this method. These pieces must be cooled very slowly to avoid internal stresses and possible cracking owing to temperature gradients that would otherwise be set up in the glass. After cooling and solidifying, the piece must be finished by lapping and polishing. Casting is not much used in glassworking except for these kinds of special jobs. Not only is cooling and cracking a problem, but also molten glass is relatively viscous at normal working temperatures, and does not flow through small orifices or into small sections as well as molten metals or heated thermoplastics. Smaller lenses are usually made by pressing, discussed in the preceding. 12.2.2 SHAPING OF FLAT AND TUBULAR GLASS Here we describe two methods for making plate glass and one method for producing tube stock. They are continuous processes, in which long sections of flat window glass or glass tubing are made and later cut into appropriate sizes and lengths. They are modern technologies in contrast to the ancient method described in Historical Note 12.1. Historical Note 12.1 G Ancient methods of making flat glass [7] lass windows have been used in buildings for many centuries. The oldest process for making flat window glass was by manual glass blowing. The procedure consisted of the following: (1) a glass globe was blown on a blowpipe; (2) a portion of the globe was made to stick to the end of a ‘‘punty,’’ a metal rod used by glassblowers, and then detached from the blowpipe; and (3) after reheating the glass, the punty was rotated with sufficient speed for centrifugal force to shape the open globe into a flat disk. The disk, whose maximum possible size was only about 1 m (3 ft), was later cut into small panes for windows. At the center of the disk, where the glass was attached to the punty during the third step in the process, a lump would tend to form that had the appearance of a crown. The name ‘‘crown glass’’ was derived from this resemblance. Lenses for spectacles were ground from glass made by this method. Today, the name crown glass is still used for certain types of optical and ophthalmic glass, even though the ancient method has been replaced by modern production technology. Rolling of Flat Plate Flat plate glass can be produced by rolling, as illustrated in Figure 12.6. The starting glass, in a suitably plastic condition from the furnace, is squeezed through opposing rolls whose separation determines the thickness of the sheet. The rolling operation is usually set up so that the flat glass is moved directly into an annealing furnace. The rolled glass sheet must later be ground and polished for parallelism and smoothness. FIGURE 12.6 flat glass. Rolling of E1C12 11/10/2009 15:3:49 Page 263 Section 12.2/Shaping Processes in Glassworking 263 FIGURE 12.7 The float process for producing sheet glass. Float Process This process was developed in the late 1950s. Its advantage over other methods such as rolling is that it obtains smooth surfaces that need no subsequent finishing. In the float process, illustrated in Figure 12.7, the glass flows directly from its melting furnace onto the surface of a molten tin bath. The highly fluid glass spreads evenly across the molten tin surface, achieving a uniform thickness and smoothness. After moving into a cooler region of the bath, the glass hardens and travels through an annealing furnace, after which it is cut to size. Drawing of Glass Tubes Glass tubing is manufactured by a drawing process known as the Danner process, illustrated in Figure 12.8. Molten glass flows around a rotating hollow mandrel through which air is blown while the glass is being drawn. The air temperature and its volumetric flow rate, as well as the drawing velocity, determine the diameter and wall thickness of the tubular cross section. During hardening, the glass tube is supported by a series of rollers extending about 30 m (100 ft) beyond the mandrel. The continuous tubing is then cut into standard lengths. Tubular glass products include laboratory glassware, fluorescent light tubes, and thermometers. 12.2.3 FORMING OF GLASS FIBERS Glass fibers are used in applications ranging from insulation wool to fiber optics communications lines (Section 7.4.2). Glass fiber products can be divided into two categories [6]: (1) fibrous glass for thermal insulation, acoustical insulation, and air filtration, in which the fibers are in a random, wool-like condition; and (2) long, continuous filaments suitable for fiber-reinforced plastics, yarns and fabrics, and fiber optics. Different production methods are used for the two categories; we describe two methods in the following, representing each of the product categories, respectively. FIGURE 12.8 Drawing of glass tubes by the Danner process. Symbols v and F indicate motion (v ¼ velocity) and applied force, respectively. E1C12 11/10/2009 264 15:3:49 Page 264 Chapter 12/Glassworking FIGURE 12.9 glass fibers. Drawing of continuous Centrifugal Spraying In a typical process for making glass wool, molten glass flows into a rotating bowl with many small orifices around its periphery. Centrifugal force causes the glass to flow through the holes to become a fibrous mass suitable for thermal and acoustical insulation. Drawing of Continuous Filaments In this process, illustrated in Figure 12.9, continuous glass fibers of small diameter (the lower size limit is around 0.0025 mm [0.0001 in]) are produced by drawing strands of molten glass through small orifices in a heated plate made of a platinum alloy. The plate may have several hundred holes, each making one fiber. The individual fibers are collected into a strand by reeling them onto a spool. Before spooling, the fibers are coated with various chemicals to lubricate and protect them. Drawing speeds of around 50 m/s (10,000 ft/min) or more are not unusual. 12.3 HEAT TREATMENT AND FINISHING Heat treatment of the glass product is the third step in the glassworking sequence. For some products, additional finishing operations are performed. 12.3.1 HEAT TREATMENT We discussed glass-ceramics in Section 7.4.3 This unique material is made by a special heat treatment that transforms most of the vitreous state into a polycrystalline ceramic. Other heat treatments performed on glass cause changes that are less dramatic technologically but perhaps more important commercially; examples include annealing and tempering. E1C12 11/10/2009 15:3:50 Page 265 Section 12.3/Heat Treatment and Finishing 265 Annealing Glass products usually have undesirable internal stresses after forming, which reduce their strength. Annealing is done to relieve these stresses; the treatment therefore has the same function in glassworking as it does in metalworking. Annealing involves heating the glass to an elevated temperature and holding it for a certain period to eliminate stresses and temperature gradients, then slowly cooling the glass to suppress stress formation, followed by more rapid cooling to room temperature. Common annealing temperatures are around 500 C (900 F). The length of time the product is held at the temperature, as well as the heating and cooling rates during the cycle, depend on thickness of the glass, the usual rule being that the required annealing time varies with the square of thickness. Annealing in modern glass factories is performed in tunnel-like furnaces, called lehrs, in which the products flow slowly through the hot chamber on conveyors. Burners are located only at the front end of the chamber, so that the glass experiences the required heating and cooling cycle. Tempered Glass and Related Products A beneficial internal stress pattern can be developed in glass products by a heat treatment known as tempering, and the resulting material is called tempered glass. As in the treatment of hardened steel, tempering increases the toughness of glass. The process involves heating the glass to a temperature somewhat above its annealing temperature and into the plastic range, followed by quenching of the surfaces, usually with air jets. When the surfaces cool, they contract and harden while the interior is still plastic and compliant. As the internal glass slowly cools, it contracts, thus putting the hard surfaces in compression. Like other ceramics, glass is much stronger when subjected to compressive stresses than tensile stresses. Accordingly, tempered glass is much more resistant to scratching and breaking because of the compressive stresses on its surfaces. Applications include windows for tall buildings, all-glass doors, safety glasses, and other products requiring toughened glass. When tempered glass fails, it does so by shattering into numerous small fragments that are less likely to cut someone than conventional (annealed) window glass. Interestingly, automobile windshields are not made of tempered glass, because of the danger posed to the driver by this fragmentation. Instead, conventional glass is used; however, it is fabricated by sandwiching two pieces of glass on either side of a tough polymer sheet. Should this laminated glass fracture, the glass splinters are retained by the polymer sheet and the windshield remains relatively transparent. 12.3.2 FINISHING Finishing operations are sometimes required for glassware products. These secondary operations include grinding, polishing, and cutting. When glass sheets are produced by drawing and rolling, the opposite sides are not necessarily parallel, and the surfaces contain defects and scratch marks caused by the use of hard tooling on soft glass. The glass sheets must be ground and polished for most commercial applications. In pressing and blowing operations when split dies are used, polishing is often required to remove the seam marks from the container product. In continuous glassworking processes, such as plate and tube production, the continuous sections must be cut into smaller pieces. This is accomplished by first scoring the glass with a glass-cutting wheel or cutting diamond and then breaking the section along the score line. Cutting is generally done as the glass exits the annealing lehr. Decorative and surface processes are performed on certain glassware products. These processes include mechanical cutting and polishing operations; sandblasting; chemical etching (with hydrofluoric acid, often in combination with other chemicals); and coating (for example, coating of plate glass with aluminum or silver to produce mirrors). E1C12 11/10/2009 266 15:3:50 Page 266 Chapter 12/Glassworking 12.4 PRODUCT DESIGN CONSIDERATIONS Glass possesses special properties that make it desirable in certain applications. The following design recommendations are compiled from Bralla [1] and other sources. å Glass is transparent and has certain optical properties that are unusual if not unique among engineering materials. For applications requiring transparency, light transmittance, magnification, and similar optical properties, glass is likely to be the material of choice. Certain polymers are transparent and may be competitive, depending on design requirements. å Glass is several times stronger in compression than in tension; components should be designed so that they are subjected to compressive stresses, not tensile stresses. å Ceramics, including glass, are brittle. Glass parts should not be used in applications that involve impact loading or high stresses, which might cause fracture. å Certain glass compositions have very low thermal expansion coefficients and are therefore tolerant of thermal shock. These glasses can be selected for applications in which this characteristic is important. å Outside edges and corners on glass parts should have large radii or chamfers; likewise, inside corners should have large radii. Both outside and inside corners are potential points of stress concentration. å Unlike parts made of traditional and new ceramics, threads may be included in the design of glass parts; they are technically feasible with the press-and-blow shaping processes. However, the threads should be coarse. REFERENCES [1] Bralla, J. G. (editor). Design for Manufacturability Handbook. 2nd ed. McGraw-Hill, New York, 1998. [2] Flinn, R. A., and Trojan, P. K. Engineering Materials and Their Applications, 5th ed. John Wiley & Sons, New York, 1995. [3] Hlavac, J. The Technology of Glass and Ceramics. Elsevier Scientific Publishing, New York, 1983. [4] McColm, I. J. Ceramic Science for Materials Technologists. Chapman and Hall, New York, 1983. [5] McLellan, G., and Shand, E. B. Glass Engineering Handbook. 3rd ed. McGraw-Hill, New York, 1984. [6] Mohr, J. G., and Rowe, W. P. Fiber Glass. Krieger, New York, 1990. [7] Scholes, S. R., and Greene, C. H. Modern Glass Practice. 7th ed. TechBooks, Marietta, Georgia, 1993. REVIEW QUESTIONS 12.1. Glass is classified as a ceramic material; yet glass is different from the traditional and new ceramics. What is the difference? 12.2. What is the predominant chemical compound in almost all glass products? 12.3. What are the three basic steps in the glassworking sequence? 12.4. Melting furnaces for glassworking can be divided into four types. Name three of the four types. 12.5. Describe the spinning process in glassworking. 12.6. What is the main difference between the press-andblow and the blow-and-blow shaping processes in glassworking? 12.7. There are several ways of shaping plate or sheet glass. Name and briefly describe one of them. 12.8. Describe the Danner process. 12.9. Two processes for forming glass fibers are discussed in the text. Name and briefly describe one of them. 12.10. What is the purpose of annealing in glassworking? 12.11. Describe how a piece of glass is heat treated to produce tempered glass. 12.12. Describe the type of material that is commonly used to make windshields for automobiles. 12.13. What are some of the design recommendations for glass parts? E1C12 11/10/2009 15:3:50 Page 267 Multiple Choice Quiz 267 MULTIPLE CHOICE QUIZ There are 10 correct answers in the following multiple choice questions (some questions have multiple answers that are correct). To attain a perfect score on the quiz, all correct answers must be given. Each correct answer is worth 1 point. Each omitted answer or wrong answer reduces the score by 1 point, and each additional answer beyond the correct number of answers reduces the score by 1 point. Percentage score on the quiz is based on the total number of correct answers. 12.1. Which one of the following terms refers to the glassy state of a material: (a) crystalline, (b) devitrified, (c) polycrystalline, (d) vitiated, or (e) vitreous? 12.2. Besides helping to preserve the environment, the use of recycled glass as an ingredient of the starting material in glassmaking serves what other useful purpose (one answer): (a) adds coloring variations to the glass for aesthetic value, (b) makes the glass easier to melt, (c) makes the glass stronger, or (d) reduces odors in the plant? 12.3. The charge in glassworking is which one of the following: (a) the duration of the melting cycle, (b) the electric energy required to melt the glass, (c) the name given to the melting furnace, or (d) the starting materials in melting? 12.4. Typical glass melting temperatures are in which of the following ranges: (a) 400 C to 500 C, (b) 900 C to 1000 C, (c) 1500 C to 1600 C, or (d) 2000 C to 2200 C? 12.5. Casting is a glassworking process used for (a) high production, (b) low production, or (c) medium production? 12.6. Which one of the following processes or processing steps is not applicable in glassworking: (a) annealing, (b) pressing, (c) quenching, (d) sintering, and (e) spinning? 12.7. The press-and-blow process is best suited to the production of (narrow-necked) beverage bottles, whereas the blow-and-blow process is more appropriate for producing (wide-mouthed) jars: (a) true, or (b) false? 12.8. Which one of the following processes is used to produce glass tubing: (a) Danner process, (b) pressing, (c) rolling, or (d) spinning? 12.9. If a glass part with a wall thickness of 5 mm (0.20 in) takes 10 minutes to anneal, how much time would a glass part of similar geometry but with a wall thickness of 7.5 mm (0.30 in) take to anneal (choose the one closest answer): (a) 10 minutes, (b) 15 minutes, (c) 20 minutes, or (d) 30 minutes? 12.10. A lehr is which of the following: (a) a lion’s den, (b) a melting furnace, (c) a sintering furnace, (d) an annealing furnace, or (e) none of the above? E1C13 11/02/2009 15:30:26 13 Page 268 SHAPING PROCESSES FOR PLASTICS Chapter Contents 13.1 Properties of Polymer Melts 13.2 Extrusion 13.2.1 Process and Equipment 13.2.2 Analysis of Extrusion 13.2.3 Die Configurations and Extruded Products 13.2.4 Defects in Extrusion 13.3 13.4 13.5 13.6 Production of Sheet and Film Fiber and Filament Production (Spinning) Coating Processes Injection Molding 13.6.1 Process and Equipment 13.6.2 The Mold 13.6.3 Injection Molding Machines 13.6.4 Shrinkage and Defects in Injection Molding 13.6.5 Other Injection Molding Processes 13.7 Compression and Transfer Molding 13.7.1 Compression Molding 13.7.2 Transfer Molding 13.8 Blow Molding and Rotational Molding 13.8.1 Blow Molding 13.8.2 Rotational Molding 13.9 13.10 13.11 13.12 268 Thermoforming Casting Polymer Foam Processing and Forming Product Design Considerations Plastics can be shaped into a wide variety of products, such as molded parts, extruded sections, films and sheets, insulation coatings on electrical wires, and fibers for textiles. In addition, plastics are often the principal ingredient in other materials, such as paints and varnishes; adhesives; and various polymer matrix composites. In this chapter we consider the technologies by which these products are shaped, postponing paints and varnishes, adhesives, and composites until later chapters. Many plastic-shaping processes can be adapted to rubbers (Chapter 14) and polymer matrix composites (Chapter 15). The commercial and technological importance of these shapingprocessesderivesfrom the growing importanceofthe materials being processed. Applications of plastics have increased at a much faster rate than either metals or ceramics during the last 50 years. Indeed, many parts previously made of metals are today being made of plastics and plastic composites. The same is true of glass; plastic containers have been largelysubstitutedforglassbottlesandjarsinproductpackaging. The total volume of polymers (plastics and rubbers) now exceeds that of metals. We can identify several reasons why the plastic-shaping processes are important: å The variety of shaping processes, and the ease with which polymers can be processed, allows an almost unlimited variety of part geometries to be formed. å Many plastic parts are formed by molding, which is a net shape process. Further shaping is generally not needed. å Although heating is usually required to form plastics, less energy is required than for metals because the processing temperatures are much lower. å Because lower temperatures are used in processing, handling of the product is simplified during production. Because many plastic processing methods are one-step operations (e.g., molding), the amount of product handling required is substantially reduced compared with metals. E1C13 11/02/2009 15:30:26 Page 269 Section 13.1/Properties of Polymer Melts 269 å Finishing by painting or plating is not required (except in unusual circumstances) for plastics. As discussed in Chapter 8, the two types of plastics are thermoplastics and thermosets. The difference is that thermosets undergo a curing process during heating and shaping, which causes a permanent chemical change (cross-linking) in their molecular structure. Once they have been cured, they cannot be melted through reheating. By contrast, thermoplastics do not cure, and their chemical structure remains basically unchanged upon reheating even though they transform from solid to fluid. Of the two types, thermoplastics are by far the more important type commercially, comprising more than 80% of the total plastics tonnage. Plastic-shaping processes can be classified as follows according to the resulting product geometry: (1) continuous extruded products with constant cross section other than sheets, films, and filaments; (2) continuous sheets and films; (3) continuous filaments (fibers); (4) molded parts that are mostly solid; (5) hollow molded parts with relatively thin walls; (6) discrete parts made of formed sheets and films; (7) castings; and (8) foamed products. This chapter examines each of these categories. The most important processes commercially are those associated with thermoplastics; the two processes of greatest significance are extrusion and injection molding. A brief history of plastic-shaping processes is presented in Historical Note 13.1. Coverage of the plastic-shaping processes begins by examining the properties of polymer melts, because nearly all of the thermoplastic shaping processes share the common step of heating the plastic so that it flows. Historical Note 13.1 E Plastic shaping processes quipment for shaping plastics evolved largely from rubber processing technology. Noteworthy among the early contributors was Edwin Chaffee, an American who developed a two-roll steam-heated mill for mixing additives into rubber around 1835 (Section 14.1.3). He was also responsible for a similar device called a calender, which consists of a series of heated rolls for coating rubber onto cloth (Section 13.3). Both machines are still used today for plastics as well as rubbers. The first extruders, dating from around 1845 in the United Kingdom, were ram-driven machines for extruding rubber and coating rubber onto electrical wire. The trouble with ram-type extruders is that they operate in an intermittent fashion. An extruder that could operate continuously, especially for wire and cable coating, was highly desirable. Although several individuals worked with varying degrees of success on a screw-type extruder (Section 13.2.1), Mathew Gray in the United Kingdom is 13.1 credited with the invention; his patent is dated 1879. As thermoplastics were subsequently developed, these screw extruders, originally designed for rubber, were adapted. An extrusion machine specifically designed for thermoplastics was introduced in 1935. Injection molding machines for plastics were adaptations of equipment designed for metal die casting (Historical Note 11.2). Around 1872, John Hyatt, an important figure in the development of plastics (Historical Note 8.1), patented a molding machine specifically for plastics. It was a plunger-type machine (Section 13.6.3). The injection molding machine in its modern form was introduced in 1921, with semiautomatic controls added in 1937. Ram-type machines were the standard in the plastic molding industry for many decades, until the superiority of the reciprocating screw machine, developed by William Willert in the United States in 1952, became obvious. PROPERTIES OF POLYMER MELTS To shape a thermoplastic polymer it must be heated so that it softens to the consistency of a liquid. In this form, it is called a polymer melt. Polymer melts exhibit several unique properties and characteristics, considered in this section. E1C13 11/02/2009 270 15:30:27 Page 270 Chapter 13/Shaping Processes for Plastics FIGURE 13.1 Viscosity relationships for Newtonian fluid and typical polymer melt. Viscosity Because of its high molecular weight, a polymer melt is a thick fluid with high viscosity. As we defined the term in Section 3.4, viscosity is a fluid property that relates the shear stress experienced during flow of the fluid to the rate of shear. Viscosity is important in polymer processing because most of the shaping methods involve flow of the polymer melt through small channels or die openings. The flow rates are often large, thus leading to high rates of shear; and the shear stresses increase with shear rate, so that significant pressures are required to accomplish the processes. Figure 13.1 shows viscosity as a function of shear rate for two types of fluids. For a Newtonian fluid (which includes most simple fluids such as water and oil), viscosity is a constant at a given temperature; it does not change with shear rate. The relationship between shear stress and shear strain is proportional, with viscosity as the constant of proportionality: t ¼ h g_ or h ¼ tg_ ð13:1Þ where t ¼ shear stress, Pa (lb/in2); h ¼ coefficient of shear viscosity, Ns/m2, or Pa-s (lb-sec/ in2); and g_ ¼ shear rate, 1/s (1/sec). However, for a polymer melt, viscosity decreases with shear rate, indicating that the fluid becomes thinner at higher rates of shear. This behavior is called pseudoplasticity and can be modeled to a reasonable approximation by the expression t ¼ kðg_ Þn ð13:2Þ where k ¼ a constant corresponding to the viscosity coefficient and n ¼ flow behavior index. For n ¼ 1, the equation reduces to the previous Eq. (13.1) for a Newtonian fluid, and k becomes h. For a polymer melt, values of n are less than 1. In addition to the effect of shear rate (fluid flow rate), viscosity of a polymer melt is also affected by temperature. Like most fluids, the value decreases with increasing temperature. This is shown in Figure 13.2 for several common polymers at a shear rate of 103 s1, which is approximately the same as the rates encountered in injection molding and high speed extrusion. Thus we see that the viscosity of a polymer melt decreases with increasing values of shear rate and temperature. Equation (13.2) can be applied, except that k depends on temperature as shown in Figure 13.2. Viscoelasticity Another property possessed by polymer melts is viscoelasticity. We discussed this property in the context of solid polymers in Section 3.5. However, liquid polymers exhibit it also. A good example is die swell in extrusion, in which the hot plastic expands when exiting the die opening. The phenomenon, illustrated in Figure 13.3, can be explained by noting that the polymer was contained in a much larger cross section before entering the narrow die channel. In effect, the extruded material ‘‘remembers’’ its former shape and attempts to return to it after leaving the die orifice. More technically, the compressive E1C13 11/02/2009 15:30:27 Page 271 Section 13.2/Extrusion 271 FIGURE 13.2 Viscosity as a function of temperatures for selected polymers at a shear rate of 103 s-1. (Data compiled from [12].) FIGURE 13.3 Die swell, a manifestation of viscoelasticity in polymer melts, as depicted here on exiting an extrusion die. stresses acting on the material as it enters the small die opening do not relax immediately. When the material subsequently exits the orifice and the restriction is removed, the unrelaxed stresses cause the cross section to expand. Die swell can be most easily measured for a circular cross section by means of the swell ratio, defined as Dx ð13:3Þ rs ¼ Dd where rs ¼ swell ratio; Dx ¼ diameter of the extruded cross section, mm (in); and Dd ¼ diameter of the die orifice, mm (in). The amount of die swell depends on the time the polymer melt spends in the die channel. Increasing the time in the channel, by means of a longer channel, reduces die swell. 13.2 EXTRUSION Extrusion is one of the fundamental shaping processes, for metals and ceramics as well as polymers. Extrusion is a compression process in which material is forced to flow through a die orifice to provide long continuous product whose cross-sectional shape is determined by E1C13 11/02/2009 272 15:30:27 Page 272 Chapter 13/Shaping Processes for Plastics the shape of the orifice. As a polymer shaping process, it is widely used for thermoplastics and elastomers (but rarely for thermosets) to mass produce items such as tubing, pipes, hose, structural shapes (such as window and door molding), sheet and film, continuous filaments, and coated electrical wire and cable. For these types of products, extrusion is carried out as a continuous process; the extrudate (extruded product) is subsequently cut into desired lengths. This section covers the basic extrusion process, and several subsequent sections examine processes based on extrusion. 13.2.1 PROCESS AND EQUIPMENT In polymer extrusion, feedstock in pellet or powder form is fed into an extrusion barrel where it is heated and melted and forced to flow through a die opening by means of a rotating screw, as illustrated in Figure 13.4. The two main components of the extruder are the barrel and the screw. The die is not a component of the extruder; it is a special tool that must be fabricated for the particular profile to be produced. The internal diameter of the extruder barrel typically ranges from 25 to 150 mm (1.0 to 6.0 in). The barrel is long relative to its diameter, with L=D ratios usually between 10 and 30. The L=D ratio is reduced in Figure 13.4 for clarity of drawing. The higher ratios are used for thermoplastic materials, whereas lower L=D values are for elastomers. A hopper containing the feedstock is located at the end of the barrel opposite the die. The pellets are fed by gravity onto the rotating screw whose turning moves the material along the barrel. Electric heaters are used to initially melt the solid pellets; subsequent mixing and mechanical working of the material generate additional heat, which maintains the melt. In some cases, enough heat is supplied through the mixing and shearing action that external heating is not required. Indeed, in some cases the barrel must be externally cooled to prevent overheating of the polymer. The material is conveyed through the barrel toward the die opening by the action of the extruder screw, which rotates at about 60 rev/min. The screw serves several functions and is divided into sections that correspond to these functions. The sections and functions are the (1) feed section, in which the stock is moved from the hopper port and preheated; (2) compression section, where the polymer is transformed into liquid consistency, air entrapped amongst the pellets is extracted from the melt, and the material is compressed; and (3) metering section, in which the melt is homogenized and sufficient pressure is developed to pump it through the die opening. FIGURE 13.4 Components and features of a (single-screw) extruder for plastics and elastomers. E1C13 11/02/2009 15:30:27 Page 273 Section 13.2/Extrusion Barrel 273 Pitch p Screw A Direction of melt flow D dc Channel FIGURE 13.5 Details of an extruder screw inside the barrel. wc wf Flight The operation of the screw is determined by its geometry and speed of rotation. Typical extruder screw geometry is depicted in Figure 13.5. The screw consists of spiraled ‘‘flights’’ (threads) with channels between them through which the polymer melt is moved. The channel has a width wc and depth dc. As the screw rotates, the flights push the material forward through the channel from the hopper end of the barrel toward the die. Although not discernible in the diagram, the flight diameter is smaller than the barrel diameter D by a very small clearance—around 0.05 mm (0.002 in). The function of the clearance is to limit leakage of the melt backward to the trailing channel. The flight land has a width wf and is made of hardened steel to resist wear as it turns and rubs against the inside of the barrel. The screw has a pitch whose value is usually close to the diameter D. The flight angle A is the helix angle of the screw and can be determined from the relation tan A ¼ p pD ð13:4Þ where p ¼ pitch of the screw1. The increase in pressure applied to the polymer melt in the three sections of the barrel is determined largely by the channel depth dc. In Figure 13.4, dc is relatively large in the feed section to allow large amounts of granular polymer to be admitted into the barrel. In the compression section, dc is gradually reduced, thus applying increased pressure on the polymer as it melts. In the metering section, dc is small and pressure reaches a maximum as flow is restrained by the screen pack and backer plate. The three sections of the screw are shown as being about equal in length in Figure 13.4; this is appropriate for a polymer that melts gradually, such as low-density polyethylene. For other polymers, the optimal section lengths are different. For crystalline polymers such as nylon, melting occurs rather abruptly at a specific melting point; therefore, a short compression section is appropriate. Amorphous polymers such as polyvinylchloride melt more slowly than LDPE, and the compression zone for these materials must take almost the entire length of the screw. Although the optimal screw design for each material type is different, it is common practice to use general-purpose screws. These designs represent a compromise among the different materials, and they avoid the need to make frequent screw changes, which result in costly equipment downtime. 1 Unfortunately, p is the natural symbol to use for two variables in this chapter. It represents the screw pitch here and in several other chapters. We use the same symbol p for pressure later in the chapter. E1C13 11/02/2009 274 15:30:27 Page 274 Chapter 13/Shaping Processes for Plastics Progress of the polymer along the barrel leads ultimately to the die zone. Before reaching the die, the melt passes through a screen pack—a series of wire meshes supported by a stiff plate (called a breaker plate) containing small axial holes. The screen pack assembly functions to (1) filter contaminants and hard lumps from the melt; (2) build pressure in the metering section; and (3) straighten the flow of the polymer melt and remove its ‘‘memory’’ of the circular motion imposed by the screw. This last function is concerned with the polymer’s viscoelastic property; if the flow were left unstraightened, the polymer would play back its history of turning inside the extrusion chamber, tending to twist and distort the extrudate. 13.2.2 ANALYSIS OF EXTRUSION In this section, we develop mathematical models to describe, in a simplified way, several aspects of polymer extrusion. Melt Flow in the Extruder As the screw rotates inside the barrel, the polymer melt is forced to move forward toward the die; the system operates much like an Archimedian screw. The principal transport mechanism is drag flow, resulting from friction between the viscous liquid and two opposing surfaces moving relative to each other: (1) the stationary barrel and (2) the channel of the turning screw. The arrangement can be likened to the fluid flow that occurs between a stationary plate and a moving plate separated by a viscous liquid, as illustrated in Figure 3.17. Given that the moving plate has a velocity v, it can be reasoned that the average velocity of the fluid is v=2, resulting in a volume flow rate of Qd ¼ 0:5 v d w ð13:5Þ where Qd ¼ volume drag flow rate, m /s (in /sec.); v ¼ velocity of the moving plate, m/s (in/sec.); d ¼ distance separating the two plates, m (in); and w ¼ the width of the plates perpendicular to velocity direction, m (in). These parameters can be compared with those in the channel defined by the rotating extrusion screw and the stationary barrel surface. 3 and 3 v ¼ pDN cos A ð13:6Þ d ¼ dc ð13:7Þ  w ¼ wc ¼ pD tan A  wf cos A ð13:8Þ where D ¼ screw flight diameter, m (in); N ¼ screw rotational speed, rev/s; dc ¼ screw channel depth, m (in); wc ¼ screw channel width, m (in); A ¼ flight angle; and wf ¼ flight land width, m (in). If we assume that the flight land width is negligibly small, then the last of these equations reduces to ð13:9Þ wc ¼ pD tan A cos A ¼ pD sin A Substituting Eqs. (13.6), (13.7), and (13.9) into Eq. (13.5), and using several trigonometric identities, we get Qd ¼ 0:5 p2 D2 N dc sin A cos A ð13:10Þ If no forces were present to resist the forward motion of the fluid, this equation would provide a reasonable description of the melt flow rate inside the extruder. However, compressing the polymer melt through the downstream die creates a back E1C13 11/02/2009 15:30:27 Page 275 Section 13.2/Extrusion 275 FIGURE 13.6 Typical pressure gradient in an extruder; dashed line indicates a straight line approximation to facilitate computations. pressure in the barrel that reduces the material moved by drag flow in Eq. (13.10). This flow reduction, called the back pressure flow, depends on the screw dimensions, viscosity of the polymer melt, and pressure gradient along the barrel. These dependencies can be summarized in this equation [12]:   pDd3c sin2 A dp Qb ¼ ð13:11Þ 12h dl where Qb ¼ back pressure flow, m3/s (in3/sec); h ¼ viscosity, N-s/m2 (lb-sec/in2); dp=dl ¼ the pressure gradient, MPa/m (lb/in2/in); and the other terms were previously defined. The actual pressure gradient in the barrel is a function of the shape of the screw over its length; a typical pressure profile is given in Figure 13.6. If we assume as an approximation that the profile is a straight line, indicated by the dashed line in the figure, then the pressure gradient becomes a constant p=L, and the previous equation reduces to Qb ¼ ppDd3c sin2 A 12hL ð13:12Þ where p ¼ head pressure in the barrel, MPa (lb/in2); and L ¼ length of the barrel, m (in). Recall that this back pressure flow is really not an actual flow by itself; it is a reduction in the drag flow. Thus, we can compute the magnitude of the melt flow in an extruder as the difference between the drag flow and back pressure flow: Qx ¼ Qd  Qb Qx ¼ 0:5 p2 D2 N dc sin A cos A  ppDd3c sin2 A 12hL ð13:13Þ where Qx ¼ the resulting flow rate of polymer melt in the extruder. Equation (13.13) assumes that there is minimal leak flow through the clearance between flights and barrel. Leak flow of melt will be small compared with drag and back pressure flow except in badly worn extruders. Equation (13.13) contains many parameters, which can be divided into two types: (1) design parameters, and (2) operating parameters. The design parameters are those that define the geometry of the screw and barrel: diameter D, length L, channel depth dc, and helix angle A. For a given extruder operation, these factors cannot be changed during the process. The operating parameters are those that can be changed during the process to affect output flow; they include rotational speed N, head pressure p, and melt viscosity h. Of course, melt viscosity is controllable only to the extent to which temperature and shear rate can be manipulated to affect this property. Let us see how the parameters play out their roles in the following example. E1C13 11/02/2009 276 15:30:27 Page 276 Chapter 13/Shaping Processes for Plastics Example 13.1 Extrusion Flow Rates An extruder barrel has a diameter D ¼ 75 mm. The screw rotates at N ¼ 1 rev/s. Channel depth dc ¼ 6.0 mm and flight angle A ¼ 20 . Head pressure at the end of the barrel p ¼ 7.0  106 Pa, length of the barrel L ¼ 1.9 m, and viscosity of the polymer melt is assumed to be h ¼ 100 Pa-s. Determine the volume flow rate of the plastic in the barrel Qx. Solution: Using Eq. (13.13) we can compute the drag flow and opposing back pressure flow in the barrel. 2   Qd ¼ 0:5 p2 75  103 ð1:0Þ 6  103 ðsin 20Þðcos 20Þ ¼ 53; 525 109 m3 /s   3   p 7  106 75  103 6  103 ðsin 20Þ2 ¼ 18:276 106 ¼ 18; 276 109 m3 /s Qb ¼ 12ð100Þð1:9Þ   Qx ¼ Qd  Qb ¼ ð53; 525  18; 276Þ 109 ¼ 35;249 109 m3 /s n Extruder and Die Characteristics If back pressure is zero, so that melt flow is unrestrained in the extruder, then the flow would equal drag flow Qd given by Eq. (13.10). Given the design and operating parameters (D, A, N, etc.), this is the maximum possible flow capacity of the extruder. Denote it as Qmax: Qmax ¼ 0:5p2 D2 N dc sin A cos A ð13:14Þ On the other hand, if back pressure were so great as to cause zero flow, then back pressure flow would equal drag flow; that is Qx ¼ Qd  Qb ¼ 0; so Qd ¼ Qb Using the expressions for Qd and Qb in Eq. (13.13), we can solve for p to determine what this maximum head pressure pmax would have to be to cause no flow in the extruder: pmax ¼ 6pDNLh cot A d2c ð13:15Þ The two values Qmax and pmax are points along the axes of a diagram known as the extruder characteristic (or screw characteristic), as in Figure 13.7. It defines the relationship between head pressure and flow rate in an extrusion machine with given operating parameters. With a die in the machine and the extrusion process underway, the actual values of Qx and p will lie somewhere between the extreme values, the location determined by the characteristics of the die. Flow rate through the die depends on the size and shape of the opening and the pressure applied to force the melt through it. This can be expressed as Qx ¼ K s p FIGURE 13.7 Extruder characteristic (also called the screw characteristic) and die characteristic. The extruder operating point is at intersection of the two lines. ð13:16Þ E1C13 11/02/2009 15:30:28 Page 277 277 Section 13.2/Extrusion where Qx ¼ flow rate, m3/s (in3/sec.); p ¼ head pressure, Pa (lb/in2); and Ks ¼ shape factor for the die, m5/Ns (in5/lb-sec). For a circular die opening of a given channel length, the shape factor can be computed [12] as Ks ¼ pD4d 128hLd ð13:17Þ where Dd ¼ die opening diameter, m (in) h ¼ melt viscosity, N-s/m2 (lb-sec/in2); and Ld ¼ die opening length, m (in). For shapes other than round, the die shape factor is less than for a round of the same cross-sectional area, meaning that greater pressure is required to achieve the same flow rate. The relationship between Qx and p in Eq. (13.16) is called the die characteristic. In Figure 13.7, this is drawn as a straight line that intersects with the previous extruder characteristic. The intersection point identifies the values of Qx and p that are known as the operating point for the extrusion process. Example 13.2 Extruder and Die Characteristics Consider the extruder from Example 13.1, in which D ¼ 75 mm, L ¼ 1.9 m, N ¼ 1 rev/s, dc ¼ 6 mm, and A ¼ 20 . The plastic melt has a shear viscosity h ¼ 100 Pa-s. Determine (a) Qmax and pmax, (b) shape factor Ks for a circular die opening in which Dd ¼ 6.5 mm and Ld ¼ 20 mm, and (c) values of Qx and p at the operating point. Solution: (a) Qmax is given by Eq. (13.14). 2  Qmax ¼ 0:5p2 D2 Ndc sin A cos A ¼ 0:5 p2 75  103 ð1:0Þ 6  103 ðsin 20Þðcos 20Þ  ¼ 53;525 109 m3 /s pmax is given by Eq. (13.15). pmax  6pDNLh cot A 6p 75  103 ð1:9Þð1:0Þð100Þ cot 20 ¼ ¼ ¼ 20;499;874 Pa 2 d2c 6  103 These two values define the intersection with the ordinate and abscissa for the extruder characteristic. (b) The shape factor for a circular die opening with Dd ¼ 6.5 mm and Ld ¼ 20 mm can be determined from Eq. (13.17). Ks ¼ p 6:5  103 4 3 128ð100Þ 20  10   ¼ 21:9 1012 m5 /Ns This shape factor defines the slope of the die characteristic. (c) The operating point is defined by the values of Qx and p at which the screw characteristic intersects with the die characteristic. The screw characteristic can be expressed as the equation of the straight line between Qmax and pmax, which is Qx ¼ Qmax  ðQmax =pmax Þp      ¼ 53; 525 109  53; 525 109 =20; 499; 874 p ¼ 53; 525 109  2:611 1012 p ð13:18Þ The die characteristic is given by Eq. (13.16) using the value of Ks computed in part (b).  Qx ¼ 21:9 1012 p E1C13 11/02/2009 278 15:30:28 Page 278 Chapter 13/Shaping Processes for Plastics Setting the two equations equal, we have    53; 525 109  2:611 1012 p ¼ 21:9 1012 p  p ¼ 2:184 106 Pa Solving for Qx using one of the starting equations, we obtain     Qx ¼ 53:525 106  2:611 1012 ð2:184Þ 106 ¼ 47:822 106 m3 /s Checking this with the other equation for verification,    Qx ¼ 21:9 1012 ð2:184Þ 106 ¼ 47:82 106 m3 /s n 13.2.3 DIE CONFIGURATIONS AND EXTRUDED PRODUCTS The shape of the die orifice determines the cross-sectional shape of the extrudate. We can enumerate the common die profiles and corresponding extruded shapes as follows: (1) solid profiles; (2) hollow profiles, such as tubes; (3) wire and cable coating; (4) sheet and film; and (5) filaments. The first three categories are covered in the present section. Methods for producing sheet and film are examined in Section 13.3; and filament production is discussed in Section 13.4. These latter shapes sometimes involve forming processes other than extrusion. Solid Profiles Solid profiles include regular shapes such as rounds and squares and irregular cross sections such as structural shapes, door and window moldings, automobile trim, and house siding. The side view cross section of a die for these solid shapes is illustrated in Figure 13.8. Just beyond the end of the screw and before the die, the polymer melt passes through the screen pack and breaker plate to straighten the flow lines. Then it flows into a (usually) converging die entrance, the shape designed to maintain laminar flow and avoid dead spots in the corners that would otherwise be present near the orifice. The melt then flows through the die opening itself. When the material exits the die, it is still soft. Polymers with high melt viscosities are the best candidates for extrusion, because they hold shape better during cooling. Cooling is accomplished by air blowing, water spray, or passing the extrudate through a water trough. FIGURE 13.8 (a) Side view cross section of an extrusion die for solid regular shapes, such as round stock; (b) front view of die, with profile of extrudate. Die swell is evident in both views. (Some die construction details are simplified or omitted for clarity.) E1C13 11/02/2009 15:30:28 Page 279 Section 13.2/Extrusion 279 (b) FIGURE 13.9 (a) Die cross section showing required orifice profile to obtain (b) a square extruded profile. (a) To compensate for die swell, the die opening is made long enough to remove some of the memory in the polymer melt. In addition, the extrudate is often drawn (stretched) to offset expansion from die swell. For shapes other than round, the die opening is designed with a cross section that is slightly different from the desired profile, so that the effect of die swell is to provide shape correction. This correction is illustrated in Figure 13.9 for a square cross section. Because different polymers exhibit varying degrees of die swell, the shape of the die profile depends on the material to be extruded. Considerable skill and judgment are required by the die designer for complex cross sections. Hollow Profiles Extrusion of hollow profiles, such as tubes, pipes, hoses, and other cross sections containing holes, requires a mandrel to form the hollow shape. A typical die configuration is shown in Figure 13.10. The mandrel is held in place using a spider, seen in Section A-A of the figure. The polymer melt flows around the legs supporting the mandrel to reunite into a monolithic tube wall. The mandrel often includes an air channel through which air is blown to maintain the hollow form of the extrudate during FIGURE 13.10 Side view cross section of extrusion die for shaping hollow cross sections such as tubes and pipes; Section A-A is a front view cross section showing how the mandrel is held in place; Section B-B shows the tubular cross section just prior to exiting the die; die swell causes an enlargement of the diameter. (Some die construction details are simplified.) E1C13 11/02/2009 280 15:30:28 Page 280 Chapter 13/Shaping Processes for Plastics FIGURE 13.11 Side view cross section of die for coating of electrical wire by extrusion. (Some die construction details are simplified.) hardening. Pipes and tubes are cooled using open water troughs or by pulling the soft extrudate through a water-filled tank with sizing sleeves that limit the OD of the tube while air pressure is maintained on the inside. Wire and Cable Coating The coating of wire and cable for insulation is one of the most important polymer extrusion processes. As shown in Figure 13.11 for wire coating, the polymer melt is applied to the bare wire as it is pulled at high speed through a die. A slight vacuum is drawn between the wire and the polymer to promote adhesion of the coating. The taught wire provides rigidity during cooling, which is usually aided by passing the coated wire through a water trough. The product is wound onto large spools at speeds of up to 50 m/s (10,000 ft/min). 13.2.4 DEFECTS IN EXTRUSION A number of defects can afflict extruded products. One of the worst is melt fracture, in which the stresses acting on the melt immediately before and during its flow through the die are so high as to cause failure, manifested in the form of a highly irregular surface on the extrudate. As suggested by Figure 13.12, melt fracture can be caused by a sharp reduction at the die entrance, causing turbulent flow that breaks up the melt. This contrasts with the streamlined, laminar flow in the gradually converging die in Figure 13.8. FIGURE 13.12 Melt fracture, caused by turbulent flow of the melt through a sharply reduced die entrance. E1C13 11/02/2009 15:30:29 Page 281 Section 13.3/Production of Sheet and Film 281 FIGURE 13.13 (a) Velocity profile of the melt as it flows through the die opening, which can lead to defects called sharkskin and (b) bambooing. A more common defect in extrusion is sharkskin, in which the surface of the product becomes roughened upon exiting the die. As the melt flows through the die opening, friction at the interface results in a velocity profile across the cross section, Figure 13.13. Tensile stresses develop at the surface as this material is stretched to keep up with the faster moving center core. These stresses cause minor ruptures that roughen the surface. If the velocity gradient becomes extreme, prominent marks occur on the surface, giving it the appearance of a bamboo pole; hence, the name bambooing for this more severe defect. 13.3 PRODUCTION OF SHEET AND FILM Thermoplastic sheet and film are produced by a number of processes, most important of which are two methods based on extrusion. The term sheet refers to stock with a thickness ranging from 0.5 mm (0.020 in) to about 12.5 mm (0.5 in) and used for products such as flat window glazing and stock for thermoforming (Section 13.9). Film refers to thicknesses below 0.5 mm (0.020 in). Thin films are used for packaging (product wrapping material, grocery bags, and garbage bags); thicker film applications include covers and liners (pool covers and liners for irrigation ditches). All of the processes covered in this section are continuous, high-production operations. More than half of the films produced today are polyethylene, mostly lowdensity PE. The principal other materials are polypropylene, polyvinylchloride, and regenerated cellulose (cellophane). These are all thermoplastic polymers. Slit-Die Extrusion of Sheet and Film Sheet and film of various thicknesses are produced by conventional extrusion, using a narrow slit as the die opening. The slit may be up to 3 m (10 ft) wide and as narrow as around 0.4 mm (0.015 in). One possible die configuration is illustrated in Figure 13.14. The die includes a manifold that spreads the polymer melt laterally before it flows through the slit (die orifice). One of the difficulties in this extrusion method is uniformity of thickness throughout the width of the stock. This is caused by the drastic shape change experienced by the polymer melt during its flow through the die and also to temperature and pressure variations in the die. Usually, the edges of the film must be trimmed because of thickening at the edges. To achieve high production rates, an efficient method of cooling and collecting the film must be integrated with the extrusion process. This is usually done by immediately directing the extrudate into a quenching bath of water or onto chill rolls, as shown in Figure 13.15. The chill roll method seems to be the more important commercially. Contact with the cold rolls quickly quenches and solidifies the extrudate; in effect, the extruder serves as a feeding device for the chill rolls that actually form the film. The process is noted for very high production speeds—5 m/s (1000 ft/min). In addition, close tolerances on film thickness can be achieved. Owing to the cooling method used in this process, it is known as chill-roll extrusion. E1C13 11/02/2009 282 15:30:29 Page 282 Chapter 13/Shaping Processes for Plastics FIGURE 13.14 One of several die configurations for extruding sheet and film. Blown-Film Extrusion Process This is the other widely used process for making thin polyethylene film for packaging. It is a complex process, combining extrusion and blowing to produce a tube of thin film; it is best explained with reference to the diagram in Figure 13.16. The process begins with the extrusion of a tube that is immediately drawn upward while still molten and simultaneously expanded in size by air inflated into it through the die mandrel. A ‘‘frost line’’ marks the position along the upward moving bubble where solidification of the polymer occurs. Air pressure in the bubble must be kept constant to maintain uniform film thickness and tube diameter. The air is contained in the tube by pinch rolls that squeeze the tube back together after it has cooled. Guide rolls and collapsing rolls are also used to restrain the blown tube and direct it into the pinch rolls. The flat tube is then collected onto a windup reel. The effect of air inflation is to stretch the film in both directions as it cools from the molten state. This results in isotropic strength properties, which is an advantage over other processes in which the material is stretched primarily in one direction. Other advantages include the ease with which extrusion rate and air pressure can be changed to control stock width and gage. Comparing this process with slit-die extrusion, the blownfilm method produces stronger film (so that a thinner film can be used to package a product), but thickness control and production rates are lower. The final blown film can FIGURE 13.15 Use of (a) water quenching bath or (b) chill rolls to achieve fast solidification of the molten film after extrusion. E1C13 11/02/2009 15:30:29 Page 283 Section 13.3/Production of Sheet and Film 283 FIGURE 13.16 Blownfilm process for high production of thin tubular film. be left in tubular form (e.g., for garbage bags), or it can be subsequently cut at the edges to provide two parallel thin films. Calendering Calendering is a process for producing sheet and film stock out of rubber (Section 14.1.4) or rubbery thermoplastics such as plasticized PVC. In the process, the initial feedstock is passed through a series of rolls to work the material and reduce its thickness to the desired gage. A typical setup is illustrated in Figure 13.17. The equipment is expensive, but production rate is high; speeds approaching 2.5 m/s (500 ft/min) are possible. Close control is required over roll temperatures, pressures, and rotational speed. The process is noted for its good surface finish and high gage accuracy in the film. Plastic products made by the calendering process include PVC floor covering, shower curtains, vinyl table cloths, pool liners, and inflatable boats and toys. FIGURE 13.17 calendering. A typical roll configuration in E1C13 11/02/2009 284 15:30:29 Page 284 Chapter 13/Shaping Processes for Plastics 13.4 FIBER AND FILAMENT PRODUCTION (SPINNING) The most important application of polymer fibers and filaments is in textiles. Their use as reinforcing materials in plastics (composites) is a growing application, but still small compared with textiles. A fiber can be defined as a long, thin strand of material whose length is at least 100 times its cross-sectional dimension. A filament is a fiber of continuous length. Fibers can be natural or synthetic. Synthetic fibers constitute about 75% of the total fiber market today, polyester being the most important, followed by nylon, acrylics, and rayon. Natural fibers are about 25% of the total produced, with cotton by far the most important staple (wool production is significantly less than cotton). The term spinning is a holdover from the methods used to draw and twist natural fibers into yarn or thread. In the production of synthetic fibers, the term refers to the process of extruding a polymer melt or solution through a spinneret (a die with multiple small holes) to make filaments that are then drawn and wound onto a bobbin. There are three principal variations in the spinning of synthetic fibers, depending on the polymer being processed: (1) melt spinning, (2) dry spinning, and (3) wet spinning. Melt spinning is used when the starting polymer can best be processed by heating to the molten state and pumping through the spinneret, much in the manner of conventional extrusion. A typical spinneret is 6 mm (0.25 in) thick and contains approximately 50 holes of diameter 0.25 mm (0.010 in); the holes are countersunk, so that the resulting bore has an L/D ratio of only 5/1 or less. The filaments that emanate from the die are drawn and simultaneously air cooled before being collected together and spooled onto the bobbin, as shown in Figure 13.18. Significant extension and thinning of the filaments occur while the polymer is still molten, so that the final diameter wound onto the bobbin may be only 1/10 FIGURE 13.18 Melt spinning of continuous filaments. E1C13 11/02/2009 15:30:29 Page 285 Section 13.5/Coating Processes 285 of the extruded size. Melt spinning is used for polyesters and nylons; because these are the most important synthetic fibers, melt spinning is the most important of the three processes for synthetic fibers. In dry spinning, the starting polymer is in solution and the solvent can be separated by evaporation. The extrudate is pulled through a heated chamber that removes the solvent; otherwise the sequence is similar to the previous. Fibers of cellulose acetate and acrylic are produced by this process. In wet spinning, the polymer is also in solution— only the solvent is nonvolatile. To separate the polymer, the extrudate must be passed through a liquid chemical that coagulates or precipitates the polymer into coherent strands that are then collected onto bobbins. This method is used to produce rayon (regenerated cellulose fibers). Filaments produced by any of the three processes are usually subjected to further cold drawing to align the crystal structure along the direction of the filament axis. Extensions of 2 to 8 are typical [13]. This has the effect of significantly increasing the tensile strength of the fibers. Drawing is accomplished by pulling the thread between two spools, where the winding spool is driven at a faster speed than the unwinding spool. 13.5 COATING PROCESSES Plastic (or rubber) coating involves application of a layer of the given polymer onto a substrate material. Three categories are distinguished [6]: (1) wire and cable coating; (2) planar coating, which involves the coating of a flat film; and (3) contour coating—the coating of a three-dimensional object. We have already examined wire and cable coating (Section 13.2.3); it is basically an extrusion process. The other two categories are surveyed in the following paragraphs. In addition, there is the technology of applying paints, varnishes, lacquers, and other similar coatings (Section 28.6). Planar coating is used to coat fabrics, paper, cardboard, and metal foil; these items are major products for some plastics. The important polymers include polyethylene and polypropylene, with lesser applications for nylon, PVC, and polyester. In most cases, the coating is only 0.01 to 0.05 mm (0.0005–0.002 in) thick. The two major planar coating techniques are illustrated in Figure 13.19. In the roll method, the polymer coating material is squeezed against the substrate by means of opposing rolls. In the doctor blade method, a sharp knife edge controls the amount of polymer melt that is coated onto the FIGURE 13.19 Planar coating processes: (a) roll method, and (b) doctor-blade method. E1C13 11/02/2009 286 15:30:29 Page 286 Chapter 13/Shaping Processes for Plastics substrate. In both cases, the coating material is supplied either by a slit-die extrusion process or by calendering. Contour coating of three-dimensional objects can be accomplished by dipping or spraying. Dipping involves submersion of the object into a suitable bath of polymer melt or solution, followed by cooling or drying. Spraying (such as spray painting) is an alternative method for applying a polymer coating to a solid object. 13.6 INJECTION MOLDING Injection molding is a process in which a polymer is heated to a highly plastic state and forced to flow under high pressure into a mold cavity, where it solidifies. The molded part, called a molding, is then removed from the cavity. The process produces discrete components that are almost always net shape. The production cycle time is typically in the range of 10 to 30 sec, although cycles of 1 min or longer are not uncommon for large parts. Also, the mold may contain more than one cavity, so that multiple moldings are produced each cycle. Many aspects of injection molding are illustrated in the video clip. VIDEO CLIP Plastic Injection Molding. This clip contains three segments: (1) plastic materials and molding, (2) injection molding machines, and (3) injection molds. Complex and intricate shapes are possible with injection molding. The challenge in these cases is to fabricate a mold whose cavity is the same geometry as the part and that also allows for part removal. Part size can range from about 50 g (2 oz) up to about 25 kg (more than 50 lb), the upper limit represented by components such as refrigerator doors and automobile bumpers. The mold determines the part shape and size and is the special tooling in injection molding. For large, complex parts, the mold can cost hundreds of thousands of dollars. For small parts, the mold can be built to contain multiple cavities, also making the mold expensive. Thus, injection molding is economical only for large production quantities. Injection molding is the most widely used molding process for thermoplastics. Some thermosets and elastomers are injection molded, with modifications in equipment and operating parameters to allow for cross-linking of these materials. We discuss these and other variations of injection molding in Section 13.6.6. 13.6.1 PROCESS AND EQUIPMENT Equipment for injection molding evolved from metal die casting (Historical Note 13.1). A large injection molding machine is shown in Figure 13.20. As illustrated in the schematic in Figure 13.21, an injection molding machine consists of two principal components: (1) the plastic injection unit and (2) the mold clamping unit. The injection unit is much like an extruder. It consists of a barrel that is fed from one end by a hopper containing a supply of plastic pellets. Inside the barrel is a screw whose operation surpasses that of an extruder screw in the following respect: in addition to turning for mixing and heating the polymer, it also acts as a ram that rapidly moves forward to inject molten plastic into the mold. A nonreturn valve mounted near the tip of the screw prevents the melt from flowing backward along the screw threads. Later in the molding cycle the ram retracts to its former position. Because of its dual action, it is called a reciprocating screw, a name that also identifies the machine type. Older injection E1C13 11/02/2009 15:30:29 Page 287 Section 13.6/Injection Molding 287 FIGURE 13.20 A large (3000-ton capacity) injection molding machine. (Courtesy of Cincinnati Milacron.) molding machines used a simple ram (without screw flights), but the superiority of the reciprocating screw design has led to its widespread adoption in today’s molding plants. To summarize, the functions of the injection unit are to melt and homogenize the polymer, and then inject it into the mold cavity. The clamping unit is concerned with the operation of the mold. Its functions are to (1) hold the two halves of the mold in proper alignment with each other; (2) keep the mold closed during injection by applying a clamping force sufficient to resist the injection force; and (3) open and close the mold at the appropriate times in the molding cycle. The clamping unit consists of two platens, a fixed platen and a moveable platen, and a mechanism for translating the latter. The mechanism is basically a power press that is operated by hydraulic piston or mechanical toggle devices of various types. Clamping forces of several thousand tons are available on large machines. The cycle for injection molding of a thermoplastic polymer proceeds in the following sequence, illustrated in Figure 13.22. Let us pick up the action with the mold open and the machine ready to start a new molding: (1) The mold is closed and clamped. (2) A shot of melt, which has been brought to the right temperature and viscosity by heating and the mechanical working of the screw, is injected under high pressure into the mold cavity. The plastic cools and begins to solidify when it encounters the cold surface of the mold. Ram pressure is maintained to pack additional melt into the cavity to FIGURE 13.21 simplified). Diagram of an injection molding machine, reciprocating screw type (some mechanical details are E1C13 11/02/2009 288 15:30:29 Page 288 Chapter 13/Shaping Processes for Plastics FIGURE 13.22 Typical molding cycle: (1) mold is closed, (2) melt is injected into cavity, (3) screw is retracted, and (4) mold opens, and part is ejected. compensate for contraction during cooling. (3) The screw is rotated and retracted with the nonreturn valve open to permit fresh polymer to flow into the forward portion of the barrel. Meanwhile, the polymer in the mold has completely solidified. (4) The mold is opened, and the part is ejected and removed. 13.6.2 THE MOLD The mold is the special tool in injection molding; it is custom designed and fabricated for the given part to be produced. When the production run for that part is finished, the mold is replaced with a new mold for the next part. In this section we examine several types of mold for injection molding. Two-Plate Mold The conventional two-plate mold, illustrated in Figure 13.23, consists of two halves fastened to the two platens of the molding machine’s clamping unit. When the clamping unit is opened, the two mold halves open, as shown in (b). The most obvious feature of the mold is the cavity, which is usually formed by removing metal from the mating surfaces of the two halves. Molds can contain a single cavity or multiple cavities to produce more than one part in a single shot. The figure shows a mold with two cavities. The parting surfaces (or parting line in a cross-sectional view of the mold) are where the mold opens to remove the part(s). In addition to the cavity, other features of the mold serve indispensable functions during the molding cycle. A mold must have a distribution channel through which the polymer melt flows from the nozzle of the injection barrel into the mold cavity. The distribution channel consists of (1) a sprue, which leads from the nozzle into the mold; (2) runners, which lead from the sprue to the cavity (or cavities); and (3) gates that constrict E1C13 11/02/2009 15:30:29 Page 289 Section 13.6/Injection Molding 289 FIGURE 13.23 Details of a two-plate mold for thermoplastic injection molding: (a) closed and (b) open. Mold has two cavities to produce two cup-shaped parts (cross section shown) with each injection shot. the flow of plastic into the cavity. The constriction increases the shear rate, thereby reducing the viscosity of the polymer melt. There are one or more gates for each cavity in the mold. An ejection system is needed to eject the molded part from the cavity at the end of the molding cycle. Ejector pins built into the moving half of the mold usually accomplish this function. The cavity is divided between the two mold halves in such a way that the natural shrinkage of the molding causes the part to stick to the moving half. When the mold opens, the ejector pins push the part out of the mold cavity. A cooling system is required for the mold. This consists of an external pump connected to passageways in the mold, through which water is circulated to remove heat from the hot plastic. Air must be evacuated from the mold cavity as the polymer rushes in. Much of the air passes through the small ejector pin clearances in the mold. In addition, narrow air vents are often machined into the parting surface; only about 0.03 mm (0.001 in) deep and 12 to 25 mm (0.5 to 1.0 in) wide, these channels permit air to escape to the outside but are too small for the viscous polymer melt to flow through. To summarize, a mold consists of (1) one or more cavities that determine part geometry, (2) distribution channels through which the polymer melt flows to the cavities, (3) an ejection system for part removal, (4) a cooling system, and (5) vents to permit evacuation of air from the cavities. Other Mold Types The two-plate mold is the most common mold in injection molding. An alternative is a three-plate mold, shown in Figure 13.24, for the same part geometry as before. There are advantages to this mold design. First, the flow of molten plastic is through a gate located at the base of the cup-shaped part, rather than at the side. This allows more even distribution of melt into the sides of the cup. In the side gate design in the two-plate mold of Figure 13.23, the plastic must flow around the core and join on the opposite side, possibly creating a weakness at the weld line. Second, the three-plate mold allows more automatic operation of the molding machine. As the mold opens, it divides into three plates with two openings between them. This action separates the runner from the parts, which drop by gravity into containers beneath the mold. E1C13 11/02/2009 290 15:30:29 Page 290 Chapter 13/Shaping Processes for Plastics FIGURE 13.24 Three-plate mold: (a) closed, and (b) open. The sprue and runner in a conventional two- or three-plate mold represent waste material. In many instances they can be ground and reused; however, in some cases the product must be made of ‘‘virgin’’ plastic (plastic that has not been previously molded). The hot-runner mold eliminates the solidification of the sprue and runner by locating heaters around the corresponding runner channels. Although the plastic in the mold cavity solidifies, the material in the sprue and runner channels remains molten, ready to be injected into the cavity in the next cycle. 13.6.3 INJECTION MOLDING MACHINES Injection molding machines differ in both injection unit and clamping unit. This section discusses the important types of machines available today. The name of the injection molding machine is generally based on the type of injection unit used. Injection Units Two types of injection units are widely used today. The reciprocatingscrew machine (Section 13.6.1, Figures 13.21 and 13.22) is the most common. This design uses the same barrel for melting and injection of plastic. The alternative unit involves the use of separate barrels for plasticizing and injecting the polymer, as shown in Figure 13.25 (a). This type is called a screw-preplasticizer machine or two-stage machine. Plastic pellets are fed from a hopper into the first stage, which uses a screw to drive the polymer forward and melt it. This barrel feeds a second barrel, which uses a plunger to inject the melt into the mold. Older machines used one plunger-driven barrel to melt and inject the plastic. These machines are referred to as plunger-type injection molding machines (Figure 13.25(b)). Clamping Units Clamping designs are of three types [11]: toggle, hydraulic, and hydromechanical. Toggle clamps include various designs, one of which is illustrated in Figure 13.26(a). An actuator moves the crosshead forward, extending the toggle links to push the moving platen toward a closed position. At the beginning of the movement, mechanical advantage is low and speed is high; but near the end of the stroke, the reverse is true. Thus, toggle clamps provide both high speed and high force at different points in the cycle when they are desirable. They are actuated either by hydraulic cylinders or ball screws driven by electric motors. Toggle-clamp units seem most suited to relatively low tonnage machines. E1C13 11/02/2009 15:30:30 Page 291 Section 13.6/Injection Molding 291 FIGURE 13.25 Two alternative injection systems to the reciprocating screw shown in Figure 13.21: (a) screw preplasticizer, and (b) plunger type. Hydraulic clamps, shown in Figure 13.26(b), are used on higher-tonnage injection molding machines, typically in the range 1300 to 8900 kN (150 to 1000 tons). These units are also more flexible than toggle clamps in terms of setting the tonnage at given positions during the stroke. Hydromechanical clamps are designed for large tonnages, usually above 8900 kN (1000 tons). They operate by (1) using hydraulic cylinders to rapidly move the mold toward closing position, (2) locking the position by mechanical means, and (3) using highpressure hydraulic cylinders to finally close the mold and build tonnage. FIGURE 13.26 Two clamping designs: (a) one possible toggle clamp design: (1) open and (2) closed; and (b) hydraulic clamping: (1) open, and (2) closed. Tie rods used to guide moving platens not shown. E1C13 11/02/2009 292 15:30:30 Page 292 Chapter 13/Shaping Processes for Plastics 13.6.4 SHRINKAGE AND DEFECTS IN INJECTION MOLDING Polymers have high thermal expansion coefficients, and significant shrinkage can occur during cooling of the plastic in the mold. Contraction of crystalline plastics tends to be greater than for amorphous polymers. Shrinkage is usually expressed as the reduction in linear size that occurs during cooling to room temperature from the molding temperature for the given polymer. Appropriate units are therefore mm/mm (in/in) of the dimension under consideration. Typical values for selected polymers are given in Table 13.1. Fillers in the plastic tend to reduce shrinkage. In commercial molding practice, shrinkage values for the specific molding compound should be obtained from the producer before making the mold. To compensate for shrinkage, the dimensions of the mold cavity must be made larger than the specified part dimensions. The following formula can be used [14]: Dc ¼ Dp þ Dp S þ Dp S2 ð13:19Þ where Dc ¼ dimension of cavity, mm (in); Dp ¼ molded part dimension, mm (in), and S ¼ shrinkage values obtained from Table 13.1. The third term on the right-hand side corrects for shrinkage that occurs in the shrinkage. Example 13.3 Shrinkage in Injection Molding The nominal length of a part made of polyethylene is to be 80 mm. Determine the corresponding dimension of the mold cavity that will compensate for shrinkage. Solution: From Table 13.1, the shrinkage for polyethylene is S ¼ 0.025. Using Eq. (13.19), the mold cavity diameter should be: Dc ¼ 80:0 þ 80:0ð0:025Þ þ 80:0ð0:025Þ2 ¼ 80:0 þ 2:0 þ 0:05 ¼ 82:05 mm n Because of differences in shrinkage among plastics, mold dimensions must be determined for the particular polymer to be molded. The same mold will produce different part sizes for different polymer types. Values in Table 13.1 represent a gross simplification of the shrinkage issue. In reality, shrinkage is affected by a number of factors, any of which can alter the amount of contraction experienced by a given polymer. The most important factors are injection pressure, compaction time, molding temperature, and part thickness. As injection pressure is increased, forcing more material into the mold cavity, shrinkage is reduced. Increasing compaction time has a similar effect, assuming the polymer in the gate does not solidify and seal off the cavity; maintaining pressure forces more material into the cavity while shrinkage is taking place. Net shrinkage is thereby reduced. Molding temperature refers to the temperature of the polymer in the cylinder immediately before injection. One might expect that a higher polymer temperature would increase shrinkage, on the reasoning that the difference between molding and TABLE 13.1 Typical values of shrinkage for moldings of selected thermoplastics. Plastic ABS Nylon-6,6 Polycarbonate Compiled from [14]. Shrinkage, mm/mm (in/in) 0.006 0.020 0.007 Plastic Polyethylene Polystyrene PVC Shrinkage, mm/mm (in/in) 0.025 0.004 0.005 E1C13 11/02/2009 15:30:30 Page 293 Section 13.6/Injection Molding 293 room temperatures is greater. However, shrinkage is actually lower at higher molding temperatures. The explanation is that higher temperatures significantly lower the viscosity of the polymer melt, allowing more material to be packed into the mold; the effect is the same as higher injection pressures. Thus, the effect on viscosity more than compensates for the larger temperature difference. Finally, thicker parts show greater shrinkage. A molding solidifies from the outside; the polymer in contact with the mold surface forms a skin that grows toward the center of the part. At some point during solidification, the gate solidifies, isolating the material in the cavity from the runner system and compaction pressure. When this happens, the molten polymer inside the skin accounts for most of the remaining shrinkage that occurs in the part. A thicker part section experiences greater shrinkage because it contains a higher proportion of molten material. In addition to the shrinkage issue, other things can also go wrong. Here are some of the common defects in injection molded parts: å Short shots. As in casting, a short shot is a molding that has solidified before completely filling the cavity. The defect can be corrected by increasing temperature and/or pressure. The defect may also result from use of a machine with insufficient shot capacity, in which case a larger machine is needed. å Flashing. Flashing occurs when the polymer melt is squeezed into the parting surface between mold plates; it can also occur around ejection pins. The defect is usually caused by (1) vents and clearances in the mold that are too large; (2) injection pressure too high compared with clamping force; (3) melt temperature too high; or (4) excessive shot size. å Sink marks and voids. These are defects usually related to thick molded sections. A sink mark occurs when the outer surface on the molding solidifies, but contraction of the internal material causes the skin to be depressed below its intended profile. A void is caused by the same basic phenomenon; however, the surface material retains its form and the shrinkage manifests itself as an internal void because of high tensile stresses on the still-molten polymer. These defects can be addressed by increasing the packing pressure after injection. A better solution is to design the part to have uniform section thicknesses and use thinner sections. å Weld lines. Weld lines occur when polymer melt flows around a core or other convex detail in the mold cavity and meets from opposite directions; the boundary thus formed is called a weld line, and it may have mechanical properties that are inferior to those in the rest of the part. Higher melt temperatures, higher injection pressures, alternative gating locations on the part, and better venting are ways of dealing with this defect. 13.6.5 OTHER INJECTION MOLDING PROCESSES The vast majority of injection molding applications involve thermoplastics. Several variants of the process are described in this section. Thermoplastic Foam Injection Molding Plastic foams have a variety of applications, and we discuss these materials and their processing in Section 13.11. One of the processes, sometimes called structural foam molding, is appropriate to discuss here because it is injection molding. It involves the molding of thermoplastic parts that possess a dense outer skin surrounding a lightweight foam center. Such parts have high stiffness-toweight ratios suitable for structural applications. A structural foam part can be produced either by introducing a gas into the molten plastic in the injection unit or by mixing a gas-producing ingredient with the starting pellets. During injection, an insufficient amount of melt is forced into the mold cavity, where it E1C13 11/02/2009 294 15:30:30 Page 294 Chapter 13/Shaping Processes for Plastics expands (foams) to fill the mold. The foam cells in contact with the cold mold surface collapse to form a dense skin, while the material in the core retains its cellular structure. Items made of structural foam include electronic cases, business machine housings, furniture components, and washing machine tanks. Advantages cited for structural foam molding include lower injection pressures and clamping forces, and thus the capability to produce large components, as suggested by the preceding list. A disadvantage of the process is that the resulting part surfaces tend to be rough, with occasional voids. If good surface finish is needed for the application, then additional processing is required, such as sanding, painting, and adhesion of a veneer. Multi-Injection Molding Processes Unusual effects can be achieved by multiple injection of different polymers to mold a part. The polymers are injected either simultaneously or sequentially, and there may be more than one mold cavity involved. Several processes fall under this heading, all characterized by two or more injection units—thus, the equipment for these processes is expensive. Sandwich molding involves injection of two separate polymers—one is the outer skin of the part and the other is the inner core, which is typically a polymer foam. A specially designed nozzle controls the flow sequence of the two polymers into the mold. The sequence is designed so that the core polymer is completely surrounded by the skin material inside the mold cavity. The final structure is similar to that of a structural foam molding. However, the molding possesses a smooth surface, thus overcoming one of the major shortcomings of the previous process. In addition, it consists of two distinct plastics, each with its own characteristics suited to the application. Another multi-injection molding process involves sequential injection of two polymers into a two-position mold. With the mold in the first position, the first polymer is injected into the cavity. Then the mold opens to the second position, and the second melt is injected into the enlarged cavity. The resulting part consists of two integrally connected plastics. Bi-injection molding is used to combine plastics of two different colors (e.g., automobile tail light covers) or to achieve different properties in different sections of the same part. Injection Molding of Thermosets Injection molding is used for thermosetting (TS) plastics, with certain modifications in equipment and operating procedure to allow for cross-linking. The machines for thermoset injection molding are similar to those used for thermoplastics. They use a reciprocating-screw injection unit, but the barrel length is shorter to avoid premature curing and solidification of the TS polymer. For the same reason, temperatures in the barrel are kept at relatively low levels, usually 50 C to 125 C (120 F to 260 F), depending on the polymer. The plastic, usually in the form of pellets or granules, is fed into the barrel through a hopper. Plasticizing occurs by the action of the rotating screw as the material is moved forward toward the nozzle. When sufficient melt has accumulated ahead of the screw, it is injected into a mold that is heated to 150 C to 230 C (300 F to 450 F), where cross-linking occurs to harden the plastic. The mold is then opened, and the part is ejected and removed. Molding cycle times typically range from 20 sec to 2 min, depending on polymer type and part size. Curing is the most time-consuming step in the cycle. In many cases, the part can be removed from the mold before curing is completed, so that final hardening occurs because of retained heat within a minute or two after removal. An alternative approach is to use a multiple-mold machine, in which two or more molds are attached to an indexing head served by a single injection unit. The principal thermosets for injection molding are phenolics, unsaturated polyesters, melamines, epoxies, and urea-formaldehyde. Elastomers are also injected molded (Section 14.1.4). More than 50% of the phenolic moldings currently produced in the United States are made by this process [11], representing a shift away from compression and transfer E1C13 11/02/2009 15:30:30 Page 295 Section 13.7/Compression and Transfer Molding 295 FIGURE 13.27 Reaction injection molding (RIM) system, shown immediately after ingredients A and B have been pumped into the mixing head prior to injection into the mold cavity (some details of processing equipment omitted). molding, the traditional processes used for thermosets (Section 13.7). Most of the TS molding materials contain large proportions of fillers (up to 70% by weight), including glass fibers, clay, wood fibers, and carbon black. In effect, these are composite materials that are being injected molded. Reaction Injection Molding Reaction injection molding (RIM) involves the mixing of two highly reactive liquid ingredients and immediately injecting the mixture into a mold cavity, where chemical reactions leading to solidification occur. The two ingredients form the components used in catalyst-activated or mixing-activated thermoset systems (Section 8.3.1). Urethanes, epoxies, and urea-formaldehyde are examples of these systems. RIM was developed with polyurethane to produce large automotive components such as bumpers, spoilers, and fenders. These kinds of parts still constitute the major application of the process. RIM-molded polyurethane parts typically possess a foam internal structure surrounded by a dense outer skin. As shown in Figure 13.27, liquid ingredients are pumped in precisely measured amounts from separate holding tanks into a mixing head. The ingredients are rapidly mixed and then injected into the mold cavity at relatively low pressure where polymerization and curing occur. A typical cycle time is around 2 min. For relatively large cavities the molds for RIM are much less costly than corresponding molds for conventional injection molding. This is because of the low clamping forces required in RIM and the opportunity to use lightweight components in the molds. Other advantages of RIM include (1) low energy is required in the process; (2) equipment costs are less than injection molding; (3) a variety of chemical systems are available that enable specific properties to be obtained in the molded product; and (4) the production equipment is reliable, and the chemical systems and machine relationships are well understood [17]. 13.7 COMPRESSION AND TRANSFER MOLDING Discussed in this section are two molding techniques widely used for thermosetting polymers and elastomers. For thermoplastics, these techniques cannot match the efficiency of injection molding, except for very special applications. E1C13 11/02/2009 296 15:30:30 Page 296 Chapter 13/Shaping Processes for Plastics 13.7.1 COMPRESSION MOLDING Compression molding is an old and widely used molding process for thermosetting plastics. Its applications also include rubber tires and various polymer matrix composite parts. The process, illustrated in Figure 13.28 for a TS plastic, consists of (1) loading a precise amount of molding compound, called the charge, into the bottom half of a heated mold; (2) bringing the mold halves together to compress the charge, forcing it to flow and conform to the shape of the cavity; (3) heating the charge by means of the hot mold to polymerize and cure the material into a solidified part; and (4) opening the mold halves and removing the part from the cavity. The initial charge of molding compound can be any of several forms, including powders or pellets, liquid, or preform. The amount of polymer must be precisely controlled to obtain repeatable consistency in the molded product. It has become common practice to preheat the charge before its placement into the mold; this softens the polymer and shortens the production cycle time. Preheating methods include infrared heaters, convection heating in an oven, and use of a heated rotating screw in a barrel. The latter technique (borrowed from injection molding) is also used to meter the amount of the charge. Compression molding presses are oriented vertically and contain two platens to which the mold halves are fastened. The presses involve either of two types of actuation: (1) upstroke of the bottom platen or (2) downstroke of the top platen, the former being the more common machine configuration. They are generally powered by a hydraulic cylinder that can be designed to provide clamping capacities up to several hundred tons. Molds for compression molding are generally simpler than their injection mold counterparts. There is no sprue and runner system in a compression mold, and the process itself is generally limited to simpler part geometries because of the lower flow capabilities of the starting thermosetting materials. However, provision must be made for heating the mold, usually accomplished by electric resistance heating, steam, or hot oil circulation. Compression molds can be classified as hand molds, used for trial runs; semiautomatic, in which the press follows a programmed cycle but the operator manually loads and unloads the press; and automatic, which operate under a fully automatic press cycle (including automatic loading and unloading). FIGURE 13.28 Compression molding for thermosetting plastics: (1) charge is loaded; (2) and (3) charge is compressed and cured; and (4) part is ejected and removed (some details omitted). E1C13 11/02/2009 15:30:31 Page 297 Section 13.7/Compression and Transfer Molding 297 Materials for compression molding include phenolics, melamine, urea-formaldehyde, epoxies, urethanes, and elastomers. Typical moldings include electric plugs and sockets, pot handles, and dinnerware plates. Advantages of compression molding in these applications include (1) molds that are simpler and less expensive, (2) less scrap, and (3) low residual stresses in the molded parts. A typical disadvantage is longer cycle times and therefore lower production rates than injection molding. 13.7.2 TRANSFER MOLDING In this process, a thermosetting charge is loaded into a chamber immediately ahead of the mold cavity, where it is heated; pressure is then applied to force the softened polymer to flow into the heated mold where curing occurs. There are two variants of the process, illustrated in Figure 13.29: (a) pot transfer molding, in which the charge is injected from a FIGURE 13.29 (a) Pot transfer molding, and (b) plunger transfer molding. Cycle in both processes is: (1) charge is loaded into pot, (2) softened polymer is pressed into mold cavity and cured, and (3) part is ejected. E1C13 11/02/2009 298 15:30:31 Page 298 Chapter 13/Shaping Processes for Plastics ‘‘pot’’ through a vertical sprue channel into the cavity; and (b) plunger transfer molding, in which the charge is injected by means of a plunger from a heated well through lateral channels into the mold cavity. In both cases, scrap is produced each cycle in the form of the leftover material in the base of the well and lateral channels, called the cull. In addition, the sprue in pot transfer is scrap material. Because the polymers are thermosetting, the scrap cannot be recovered. Transfer molding is closely related to compression molding, because it is used on the same polymer types (thermosets and elastomers). One can also see similarities to injection molding, in the way the charge is preheated in a separate chamber and then injected into the mold. Transfer molding is capable of molding part shapes that are more intricate than compression molding but not as intricate as injection molding. Transfer molding also lends itself to molding with inserts, in which a metal or ceramic insert is placed into the cavity before injection, and the heated plastic bonds to the insert during molding. 13.8 BLOW MOLDING AND ROTATIONAL MOLDING Both of these processes are used to make hollow, seamless parts out of thermoplastic polymers. Rotational molding can also be used for thermosets. Parts range in size from small plastic bottles of only 5 mL (0.15 oz) to large storage drums of 38,000-L (10,000-gal) capacity. Although the two processes compete in certain cases, generally they have found their own niches. Blow molding is more suited to the mass production of small disposable containers, whereas rotational molding favors large, hollow shapes. 13.8.1 BLOW MOLDING Blow molding is a molding process in which air pressure is used to inflate soft plastic inside a mold cavity. It is an important industrial process for making one-piece hollow plastic parts with thin walls, such as bottles and similar containers. Because many of these items are used for consumer beverages for mass markets, production is typically organized for very high quantities. The technology is borrowed from the glass industry (Section 12.2.1) with which plastics compete in the disposable and recyclable bottle market. Blow molding is accomplished in two steps: (1) fabrication of a starting tube of molten plastic, called a parison (same as in glass-blowing); and (2) inflation of the tube to the desired final shape. Forming the parison is accomplished by either extrusion or injection molding. The video clip on plastic blow molding illustrates the two categories. VIDEO CLIP Plastic Blow Molding. This clip contains three segments: (1) blow molding materials and processes, (2) extrusion blow molding, and (3) injection blow molding. Extrusion Blow Molding This form of blow molding consists of the cycle illustrated in Figure 13.30. In most cases, the process is organized as a very high production operation for making plastic bottles. The sequence is automated and often integrated with downstream operations such as bottle filling and labeling. It is usually a requirement that the blown container be rigid, and rigidity depends on wall thickness among other factors. We can relate wall thickness of the blown container to the starting extruded parison [12], assuming a cylindrical shape for the final product. The effect of die swell on the parison is shown in Figure 13.31. The mean diameter of the tube as E1C13 11/02/2009 15:30:31 Page 299 Section 13.8/Blow Molding and Rotational Molding 299 FIGURE 13.30 Extrusion blow molding: (1) extrusion of parison; (2) parison is pinched at the top and sealed at the bottom around a metal blow pin as the two halves of the mold come together; (3) the tube is inflated so that it takes the shape of the mold cavity; and (4) mold is opened to remove the solidified part. it exits the die is determined by the mean die diameter Dd. Die swell causes expansion to a mean parison diameter Dp. At the same time, wall thickness swells from td to tp. The swell ratio of the parison diameter and wall thickness is given by Dp tp rs ¼ ¼ ð13:20Þ Dd td When the parison is inflated to the blow mold diameter Dm, there is a corresponding reduction in wall thickness to tm. Assuming constant volume of cross section, we have ð13:21Þ pDp tp ¼ pDm tm Solving for tm, we obtain tm ¼ FIGURE 13.31 (1) Dimensions of extrusion die, showing parisonafter die swell; and (2) final blow-molded container in extrusion blow molding. Dp tp Dm E1C13 11/02/2009 300 15:30:31 Page 300 Chapter 13/Shaping Processes for Plastics FIGURE 13.32 Injection blow molding: (1) parison is injected molded around a blowing rod; (2) injection mold is opened and parison is transferred to a blow mold; (3) soft polymer is inflated to conform to the blow mold; and (4) blow mold is opened, and blown product is removed. Substituting Eq. (13.20) into this equation, we get tm ¼ r2s td Dd Dm ð13:22Þ The amount of die swell in the initial extrusion process can be measured by direct observation; and the dimensions of the die are known. Thus, we can determine the wall thickness on the blow-molded container. Injection Blow Molding In this process, the starting parison is injection molded rather than extruded. A simplified sequence is outlined in Figure 13.32. Compared to its extrusion-based competitor, injection blow molding usually has the following advantages: (1) higher production rate, (2) greater accuracy in the final dimensions, (3) lower scrap rates, and (4) less wasteful of material. On the other hand, larger containers can be produced with extrusion blow molding because the mold in injection molding is so expensive for large parisons. Also, extrusion blow molding is technically more feasible and economical for double-layer bottles used for storing certain medicines, personal care products, and various chemical compounds.2 In a variation of injection blow molding, called stretch blow molding (Figure 13.33), the blowing rod extends downward into the injection molded parison during step 2, thus stretching the soft plastic and creating a more favorable stressing of the polymer than conventional injection blow molding or extrusion blow molding. The resulting structure is more rigid, with higher transparency and better impact resistance. The most widely used material for stretch blow molding is polyethylene terephthalate (PET), a polyester that has very low permeability and is strengthened by the stretch-blow-molding process. The combination of properties makes it ideal as a container for carbonated beverages (e.g., 2-L soda bottles). Materials and Products Blow molding is limited to thermoplastics. Polyethylene is the polymer most commonly used for blow molding—in particular, high density and high molecular weight polyethylene (HDPE and HMWPE). In comparing their properties with those of low density PE given the requirement for stiffness in the final product, it is more 2 The author is indebted to Tom Walko, plant manager at one of Graham Packaging Company’s blow molding plants for providing the preceding comparisons between extrusion and injection blow molding. E1C13 11/02/2009 15:30:32 Page 301 Section 13.8/Blow Molding and Rotational Molding FIGURE 13.33 301 Stretch blow molding: (1) injection molding of parison, (2) stretching, and (3) blowing. economical to use these more expensive materials because the container walls can be made thinner. Other blow moldings are made of polypropylene (PP), polyvinylchloride (PVC), and polyethylene terephthalate. Disposable containers for packaging liquid consumer goods constitute the major share of products made by blow molding; but they are not the only products. Other items include large shipping drums (55-gal) for liquids and powders, large storage tanks (2000gal), automotive gasoline tanks, toys, and hulls for sail boards and small boats. In the latter case, two boat hulls are made in a single blow molding and subsequently cut into two open hulls. 13.8.2 ROTATIONAL MOLDING Rotational molding uses gravity inside a rotating mold to achieve a hollow form. Also called rotomolding, it is an alternative to blow molding for making large, hollow shapes. It is used principally for thermoplastic polymers, but applications for thermosets and elastomers are becoming more common. Rotomolding tends to favor more complex external geometries, larger parts, and lower production quantities than blow molding. The process consists of the following steps: (1) A predetermined amount of polymer powder is loaded into the cavity of a split mold. (2) The mold is then heated and simultaneously rotated on two perpendicular axes, so that the powder impinges on all internal surfaces of the mold, gradually forming a fused layer of uniform thickness. (3) While still rotating, the mold is cooled so that the plastic skin solidifies. (4) The mold is opened, and the part is unloaded. Rotational speeds used in the process are relatively slow. It is gravity, not centrifugal force, that causes uniform coating of the mold surfaces. Molds in rotational molding are simple and inexpensive compared with injection molding or blow molding, but the production cycle is much longer, lasting perhaps 10 min or more. To balance these advantages and disadvantages in production, rotational molding is often performed on a multicavity indexing machine, such as the three-station machine shown in Figure 13.34. The machine is designed so that three molds are indexed in sequence through three workstations. Thus, all three molds are working simultaneously. The first workstation is an unload–load station in which the finished part is unloaded from the mold, and the powder for the next part is loaded into the cavity. The second station consists of a heating chamber where hot-air convection heats the mold while it is simultaneously rotated. Temperatures inside the chamber are around 375 C E1C13 11/02/2009 302 15:30:32 Page 302 Chapter 13/Shaping Processes for Plastics FIGURE 13.34 Rotational molding cycle performed on a three-station indexing machine: (1) unload–load station; (2) heat and rotate mold; (3) cool the mold. (700 F), depending on the polymer and the item being molded. The third station cools the mold, using forced cold air or water spray, to cool and solidify the plastic molding inside. A fascinating variety of articles are made by rotational molding. The list includes hollow toys such as hobby horses and playing balls; boat and canoe hulls, sandboxes, small swimming pools; buoys and other flotation devices; truck body parts, automotive dashboards, fuel tanks; luggage pieces, furniture, garbage cans; fashion mannequins; large industrial barrels, containers, and storage tanks; portable outhouses, and septic tanks. The most popular molding material is polyethylene, especially HDPE. Other plastics include polypropylene, ABS, and high-impact polystyrene. 13.9 THERMOFORMING Thermoforming is a process in which a flat thermoplastic sheet is heated and deformed into the desired shape. The process is widely used in packaging of consumer products and fabricating large items such as bathtubs, contoured skylights, and internal door liners for refrigerators. Thermoforming consists of two main steps: heating and forming. Heating is usually accomplished by radiant electric heaters, located on one or both sides of the starting plastic sheetatadistanceofroughly125mm(5in).Duration oftheheatingcycleneededtosufficiently soften thesheetdepends on thepolymer—its thickness and color.Methods bywhichformingis accomplished can be classified into three basic categories: (1) vacuum thermoforming, (2) pressure thermoforming, and (3) mechanical thermoforming. In our discussion of these methods, we describe the forming of sheet stock, but in the packaging industry most thermoforming operations are performed on thin films. Vacuum Thermoforming This was the first thermoforming process (simply called vacuum forming when it was developed in the 1950s). Negative pressure is used to draw a preheated sheet into a mold cavity. The process is explained in Figure 13.35 in its most E1C13 11/02/2009 15:30:33 Page 303 Section 13.9/Thermoforming 303 FIGURE 13.35 Vacuum thermoforming: (1) a flat plastic sheet is softened by heating; (2) the softenedsheet is placed over a concave mold cavity; (3) a vacuum draws the sheet into the cavity; and (4) the plastic hardens on contact with the cold mold surface, and the part is removed and subsequently trimmed from the web. basic form. The holes for drawing the vacuum in the mold are on the order of 0.8 mm (0.031 in) in diameter, so their effect on the plastic surface is minor. Pressure Thermoforming An alternative to vacuum forming involves positive pressure to force the heated plastic into the mold cavity. This is called pressure thermoforming or blow forming; its advantage over vacuum forming is that higher pressures can be developed because the latter is limited to a theoretical maximum of 1 atm. Blow-forming pressures of 3 to 4 atm are common. The process sequence is similar to the previous, the difference being that the sheet is pressurized from above into the mold cavity. Vent holes are provided in the mold to exhaust the trapped air. The forming portion of the sequence (steps 2 and 3) is illustrated in Figure 13.36. At this point it is useful to distinguish between negative and positive molds. The molds shown in Figures 13.35 and 13.36 are negative molds because they have concave cavities. A positive mold has a convex shape. Both types are used in thermoforming. In the case of the positive mold, the heated sheet is draped over the convex form and negative or positive pressure is used to force the plastic against the mold surface. A positive mold is shown in Figure 13.37 for vacuum thermoforming. The difference between positive and negative molds may seem unimportant, because the part shapes are the same in the diagrams. However, if the part is drawn into the negative mold, then its exterior surface will have the exact surface contour of the mold cavity. The inside surface will be an approximation of the contour and will possess a finish corresponding to that of the starting sheet. By contrast, if the sheet is draped over a positive mold, then its interior surface will be identical to that of the convex mold; and its outside surface will follow approximately. Depending on the requirements of the product, this distinction might be important. E1C13 11/02/2009 304 15:30:33 Page 304 Chapter 13/Shaping Processes for Plastics FIGURE 13.36 Pressure thermoforming. The sequence is similar to the previous figure, the difference being: (2) sheet is placed over a mold cavity; and (3) positive pressure forces the sheet into the cavity. Another difference is in the thinning of the plastic sheet, one of the problems in thermoforming. Unless the contour of the mold is very shallow, there will be significant thinning of the sheet as it is stretched to conform to the mold contour. Positive and negative molds produce a different pattern of thinning in a given part. Consider the tubshaped part in our figures. In the positive mold, as the sheet is draped over the convex form, the portion making contact with the top surface (corresponding to the base of the tub) solidifies quickly and experiences virtually no stretching. This results in a thick base but significant thinning in the walls of the tub. By contrast, a negative mold results in a more even distribution of stretching and thinning in the sheet before contact is made with the cold surface. A way to improve the thinning distribution with a positive mold is to prestretch the sheet before draping it over the convex form. As shown in Figure 13.38, the heated plastic sheet is stretched uniformly by vacuum pressure into a spherical shape before drawing it over the mold. The first step depicted in frame (1) of Figure 13.38 can be used alone as a method to produce globe-shaped parts such as skylight windows and transparent domes. In the process, closely controlled air pressure is applied to inflate the soft sheet. The pressure is maintained until the blown shape has solidified. FIGURE 13.37 Use of a positive mold in vacuum thermoforming: (1) the heated plastic sheet is positioned above the convex mold and (2) the clamp is lowered into position, draping the sheet over the mold as a vacuum forces the sheet against the mold surface. E1C13 11/02/2009 15:30:33 Page 305 Section 13.9/Thermoforming 305 FIGURE 13.38 Prestretching the sheet in (1) prior to draping and vacuuming it over a positive mold in (2). Mechanical Thermoforming The third method, called mechanical thermoforming, uses matching positive and negative molds that are brought together against the heated plastic sheet, forcing it to assume their shape. In pure mechanical forming, air pressure is not used at all. The process is illustrated in Figure 13.39. Its advantages are better dimensional control and the opportunity for surface detailing on both sides of the part. The disadvantage is that two mold halves are required; therefore, the molds for the other two methods are less costly. Applications Thermoforming is a secondary shaping process, the primary process being that which produces the sheet or film (Section 13.3). Only thermoplastics can be thermoformed, because extruded sheets of thermosetting or elastomeric polymers have already been cross-linked and cannot be softened by reheating. Common thermoforming plastics are polystyrene, cellulose acetate and cellulose acetate butyrate, ABS, PVC, acrylic (polymethylmethacrylate), polyethylene, and polypropylene. FIGURE 13.39 Mechanical thermoforming: (1) heated sheet placed above a negative mold, and (2) mold is closed to shape the sheet. E1C13 11/02/2009 306 15:30:33 Page 306 Chapter 13/Shaping Processes for Plastics Mass production thermoforming operations are performed in the packaging industry. The starting sheet or film is rapidly fed through a heating chamber and then mechanically formed into the desired shape. The operations are often designed to produce multiple parts with each stroke of the press using molds with multiple cavities. In some cases, the extrusion machine that produces the sheet or film is located directly upstream from the thermoforming process, thereby eliminating the need to reheat the plastic. For best efficiency, the filling process to put the consumable food item into the container is placed immediately downstream from thermoforming. Thin film packaging items that are mass produced by thermoforming include blister packs and skin packs. They offer an attractive way to display certain commodity products such as cosmetics, toiletries, small tools, and fasteners (nails, screws, etc.). Thermoforming applications include large parts that can be produced from thicker sheet stock. Examples include covers for business machines, boat hulls, shower stalls, diffusers for lights, advertising displays and signs, bathtubs, and certain toys. Contoured skylights and internal door liners for refrigerators are made, respectively, out of acrylic (because of its transparency) and ABS (because of its ease in forming and resistance to oils and fats found in refrigerators). 13.10 CASTING In polymer shaping, casting involves pouring of a liquid resin into a mold, using gravity to fill the cavity, and allowing the polymer to harden. Both thermoplastics and thermosets are cast. Examples of the former include acrylics, polystyrene, polyamides (nylons), and vinyls (PVC). Conversion of the liquid resin into a hardened thermoplastic can be accomplished in several ways, which include (1) heating the thermoplastic resin to a highly fluid state so that it readily pours and fills the mold cavity, and then permitting it to cool and solidify in the mold; (2) using a low-molecular-weight prepolymer (or monomer) and polymerizing it in the mold to form a high-molecular-weight thermoplastic; and (3) pouring a plastisol (a liquid suspension of fine particles of a thermoplastic resin such as PVC in a plasticizer) into a heated mold so that it gels and solidifies. Thermosetting polymers shaped by casting include polyurethane, unsaturated polyesters, phenolics, and epoxies. The process involves pouring the liquid ingredients that form the thermoset into a mold so that polymerization and cross-linking occur. Heat and/or catalysts may be required depending on the resin system. The reactions must be sufficiently slow to allow mold pouring to be completed. Fast-reacting thermosetting systems, such as certain polyurethane systems, require alternative shaping processes like reaction injection molding (Section 13.6.5). Advantages of casting over alternative processes such as injection molding include: (1) the mold is simpler and less costly, (2) the cast item is relatively free of residual stresses and viscoelastic memory, and (3) the process is suited to low production quantities. Focusing on advantage (2), acrylic sheets (Plexiglas, Lucite) are generally cast between two pieces of highly polished plate glass. The casting process permits a high degree of flatness and desirable optical qualities to be achieved in the clear plastic sheets. Such flatness and clarity cannot be obtained by flat sheet extrusion. A disadvantage in some applications is significant shrinkage of the cast part during solidification. For example, acrylic sheets undergo a volumetric contraction of about 20% when cast. This is much more than in injection molding, in which high pressures are used to pack the mold cavity to reduce shrinkage. Slush casting is an alternative to conventional casting, borrowed from metal casting technology. In slush casting, a liquid plastisol is poured into the cavity of a heated split mold, so that a skin forms at the surface of the mold. After a duration that depends on the E1C13 11/02/2009 15:30:34 Page 307 Section 13.11/Polymer Foam Processing and Forming 307 desired thickness of the skin, the excess liquid is poured out of the mold; the mold is then opened for part removal. The process is also referred to as shell casting [6]. An important application of casting in electronics is encapsulation, in which items such as transformers, coils, connectors, and other electrical components are encased in plastic by casting. 13.11 POLYMER FOAM PROCESSING AND FORMING A polymer foam is a polymer-and-gas mixture, which gives the material a porous or cellular structure. Other terms used for polymer foams include cellular polymer, blown polymer, and expanded polymer. The most common polymer foams are polystyrene (Styrofoam) and polyurethane. Other polymers used to make foams include natural rubber (‘‘foamed rubber’’) and polyvinylchloride (PVC). The characteristic properties of a foamed polymer include (1) low density, (2) high strength per unit weight, (3) good thermal insulation, and (4) good energy absorbing qualities. The elasticity of the base polymer determines the corresponding property of the foam. Polymer foams can be classified [6] as (1) elastomeric, in which the matrix polymer is a rubber, capable of large elastic deformation; (2) flexible, in which the matrix is a highly plasticized polymer such as soft PVC; and (3) rigid, in which the polymer is a stiff thermoplastic such as polystyrene or a thermosetting plastic such as a phenolic. Depending on chemical formulation and degree of cross-linking, polyurethanes can range over all three categories. The characteristic properties of polymer foams, and the ability to control their elastic behavior through selection of the base polymer, make these materials highly suitable for certain types of applications, including hot beverage cups, heat insulating structural materials and cores for structural panels, packaging materials, cushion materials for furniture and bedding, padding for automobile dashboards, and products requiring buoyancy. Common gases used in polymer foams are air, nitrogen, and carbon dioxide. The proportion of gas can range up to 90% or more. The gas is introduced into the polymer by several methods, called foaming processes. These include (1) mixing a liquid resin with air by mechanical agitation, then hardening the polymer by means of heat or chemical reaction; (2) mixing a physical blowing agent with the polymer—a gas such as nitrogen (N2) or pentane (C5H12), which can be dissolved in the polymer melt under pressure, so that the gas comes out of solution and expands when the pressure is subsequently reduced; and (3) mixing the polymer with chemical compounds, called chemical blowing agents, that decompose at elevated temperatures to liberate gases such as CO2 or N2 within the melt. The way the gas is distributed throughout the polymer matrix distinguishes two basic foam structures, illustrated in Figure 13.40: (a) closed cell, in which the gas pores are roughly spherical and completely separated from each other by the polymer matrix; and (b) open cell, in which the pores are interconnected to some extent, allowing passage of a fluid through the foam. A closed cell structure makes a satisfactory life jacket; an open cell structure would become waterlogged. Other attributes that characterize the structure include the relative proportions of polymer and gas (already mentioned) and the cell density (number of cells per unit volume), which is inversely related to the size of the individual air cells in the foam. There are many shaping processes for polymer foam products. Because the two most important foams are polystyrene and polyurethane, this discussion is limited to shaping processes for these two materials. Because polystyrene is a thermoplastic and polyurethane can be either a thermoset or an elastomer (it can also be a thermoplastic but E1C13 11/02/2009 308 15:30:34 Page 308 Chapter 13/Shaping Processes for Plastics FIGURE 13.40 Two polymer foam structures: (a) closed cell, and (b) open cell. is less important in this form), the processes covered here for these materials are representative of those used for other polymer foams. Polystyrene foams are shaped by extrusion and molding. In extrusion, a physical or chemical blowing agent is fed into the polymer melt near the die end of the extruder barrel; thus, the extrudate consists of the expanded polymer. Large sheets and boards are made in this way and are subsequently cut to size for heat insulation panels and sections. Several moldingprocessesare available for polystyrene foam. We previouslydiscussed structuralfoammoldingandsandwichmolding(Section13.6.5).Amorewidelyusedprocess is expandable foam molding, in which the molding material usually consists of prefoamed polystyrene beads. The prefoamed beads are produced from pellets of solid polystyrene that have been impregnated with a physical blowing agent. Prefoaming is performed in a large tank by applying steam heat to partially expand the pellets, simultaneously agitating them to prevent fusion. Then, in the molding process, the prefoamed beads are fed into a mold cavity, where they are further expanded and fused together to form the molded product. Hot beverage cups of polystyrene foam are produced in this way. In some processes, the prefoaming step is omitted, and the impregnated beads are fed directly into the mold cavity, where they are heated, expanded, and fused. In other operations, the expandable foam is first formedintoaflatsheetbythe blown-filmextrusionprocess (Section13.3) andthenshapedby thermoforming (Section 13.9) into packaging containers such as egg cartons. Polyurethane foam products are made in a one-step process in which the two liquid ingredients (polyol and isocyanate) are mixed and immediately fed into a mold or other form, so that the polymer is synthesized and the part geometry is created at the same time. Shaping processes for polyurethane foam can be divided into two basic types [11]: spraying and pouring. Spraying involves use of a spray gun into which the two ingredients are continuously fed, mixed, and then sprayed onto a target surface. The reactions leading to polymerization and foaming occur after application on the surface. This method is used to apply rigid insulating foams onto construction panels, railway cars, and similar large items. Pouring involves dispensing the ingredients from a mixing head into an open or closed mold in which the reactions occur. An open mold can be a container with the required contour (e. g., for an automobile seat cushion) or a long channel that is slowly moved past the pouring spout to make long, continuous sections of foam. The closed mold is a completely enclosed cavity into which a certain amount of the mixture is dispensed. Expansion of the reactants completely fills the cavity to shape the part. For fast-reacting polyurethanes, the mixture must be rapidly injected into the mold cavity using reaction injection molding (Section 13.66). The degree of cross-linking, controlled by the starting ingredients, determines the relative stiffness of the resulting foam. 13.12 PRODUCT DESIGN CONSIDERATIONS Plastics are an important design material, but the designer must be aware of their limitations. This section lists some design guidelines for plastic components, beginning with those that apply in general, and then ones applicable to extrusion and molding (injection molding, compression molding, and transfer molding). E1C13 11/02/2009 15:30:34 Page 309 Section 13.12/Product Design Considerations 309 Several general guidelines apply, irrespective of the shaping process. They are mostly limitations of plastic materials that must be considered by the designer. å Strength and stiffness. Plastics are not as strong or stiff as metals. They should not be used in applications in which high stresses will be encountered. Creep resistance is also a limitation. Strength properties vary significantly among plastics, and strengthto-weight ratios for some plastics are competitive with metals in certain applications. å Impact resistance. The capacity of plastics to absorb impact is generally good; plastics compare favorably with most metals. å Service temperatures of plastics are limited relative to engineering metals and ceramics. å Thermal expansion is greater for plastics than metals; so dimensional changes owing to temperature variations are much more significant than for metals. å Many types of plastics are subject to degradation from sunlight and certain other forms of radiation. Also, some plastics degrade in oxygen and ozone atmospheres. Finally, plastics are soluble in many common solvents. On the positive side, plastics are resistant to conventional corrosion mechanisms that afflict many metals. The weaknesses of specific plastics must be taken into account by the designer. Extrusion is one of the most widely used plastic shaping processes. Several design recommendations are presented here for conventional extrusion (compiledmostly from [3]). å Wall thickness. Uniform wall thickness is desirable in an extruded cross section. Variations in wall thickness result in nonuniform plastic flow and uneven cooling that tend to warp the extrudate. å Hollow sections. Hollow sections complicate die design and plastic flow. It is desirable to use extruded cross sections that are not hollow yet satisfy functional requirements. å Corners. Sharp corners, inside and outside, should be avoided in the cross section, because they result in uneven flow during processing and stress concentrations in the final product. The following guidelines apply to injection molding (the most popular molding process), compression molding, and transfer molding (compiled from Bralla [3], McCrum [10], and other sources). å Economic production quantities. Each molded part requires a unique mold, and the mold for any of these processes can be costly, particularly for injection molding. Minimum production quantities for injection molding are usually around 10,000 pieces; for compression molding, minimum quantities are around 1000 parts, because of the simpler mold designs involved. Transfer molding lies between the other two. å Part complexity. Although more complex part geometries mean more costly molds, it may nevertheless be economical to design a complex molding if the alternative involves many individual components assembled together. An advantage of plastic molding is that it allows multiple functional features to be combined into one part. å Wall thickness. Thick cross sections are generally undesirable; they are wasteful of material, more likely to cause warping caused by shrinkage, and take longer to harden. Reinforcing ribs can be used in molded plastic parts to achieve increased stiffness without excessive wall thickness. The ribs should be made thinner than the walls they reinforce, to minimize sink marks on the outside wall. å Corner radii and fillets. Sharp corners, both external and internal, are undesirable in molded parts; they interrupt smooth flow of the melt, tend to create surface defects, and cause stress concentrations in the finished part. E1C13 11/02/2009 310 15:30:34 Page 310 Chapter 13/Shaping Processes for Plastics TABLE 13.2 Typical tolerances on molded parts for selected plastics. Tolerances for:a Plastic Thermoplastic: ABS Polyethylene Polystyrene 50-mm Dimension 10-mm Hole 0.2 mm (0.007 in) 0.3 mm (0.010 in) 0.15 mm (0.006 in) 0.08 mm (0.003 in) 0.13 mm (0.005 in) 0.1 mm (0.004 in) Tolerances for:a 50-mm Dimension 10-mm Hole 0.15 mm (0.006 in) 0.2 mm (0.008 in) 0.05 mm (0.002 in) 0.08 mm (0.003 in) Plastic Thermosetting: Epoxies Phenolics Values represent typical commercial molding practice. Compiled from [3], [7], [14], and [19]. For smaller sizes, tolerances can be reduced. For larger sizes, more generous tolerances are required. a å Holes. Holes are quite feasible in plastic moldings, but they complicate mold design and part removal. They also cause interruptions in melt flow. å Draft. A molded part should be designed with a draft on its sides to facilitate removal from the mold. This is especially important on the inside wall of a cupshaped part because the molded plastic contracts against the positive mold shape. The recommended draft for thermosets is around 1/2 to 1 ; for thermoplastics it usually ranges between 1/8 and 1/2 . Suppliers of plastic molding compounds provide recommended draft values for their products. å Tolerances. Tolerances specify the allowable manufacturing variations for a part. Although shrinkage is predictable under closely controlled conditions, generous tolerances are desirable for injection moldings because of variations in process parameters that affect shrinkage and diversity of part geometries encountered. Table 13.2 lists typical tolerances for molded part dimensions of selected plastics. REFERENCES [1] Baird, D. G., and Collias, D. I. Polymer Processing Principles and Design, John Wiley & Sons, New York, 1998. [2] Billmeyer, Fred, W., Jr. Textbook of Polymer Science, 3rd ed. John Wiley & Sons, New York, 1984. [3] Bralla, J. G.(editor in chief). Design for Manufacturability Handbook, 2nd ed. McGraw-Hill Book Company, New York, 1998. [4] Briston, J. H. Plastic Films, 3rd ed. Longman Group U.K., Essex, England, 1989. [5] Chanda, M., and Roy, S. K. Plastics Technology Handbook, Marcel Dekker, New York, 1998. [6] Charrier, J-M. Polymeric Materials and Processing, Oxford University Press, New York, 1991. [7] Engineering Materials Handbook, Vol. 2, Engineering Plastics, ASM International, Metals Park, Ohio, 1988. [8] Hall, C. Polymer Materials, 2nd ed. John Wiley & Sons. New York, 1989. [9] Hensen, F. (ed.). Plastic Extrusion Technology, Hanser Publishers, Munich, FRG, 1988. (Distributed in United States by Oxford University Press, New York.) [10] McCrum, N. G., Buckley, C. P., and Bucknall, C. B. Principles of Polymer Engineering, 2nd ed., Oxford University Press, Oxford, UK, 1997. [11] Modern Plastics Encyclopedia, Modern Plastics, McGraw-Hill, Hightstown, New Jersey, 1991. [12] Morton-Jones, D. H. Polymer Processing, Chapman and Hall, London, UK, 1989. [13] Pearson, J. R. A. Mechanics of Polymer Processing, Elsevier Applied Science Publishers, London, 1985. [14] Rubin, I. I. Injection Molding: Theory and Practice, John Wiley & Sons, New York, 1973. [15] Rudin, A. The Elements of Polymer Science and Engineering, 2nd ed., Academic Press, Orlando, Florida, 1999. [16] Strong, A. B. Plastics: Materials and Processing, 3rd ed. Pearson Educational, Upper Saddle River, New Jersey, 2006. [17] Sweeney, F. M. Reaction Injection Molding Machinery and Processes, Marcel Dekker, Inc., New York, 1987. E1C13 11/02/2009 15:30:34 Page 311 Multiple Choice Quiz [18] Tadmor, Z., and Gogos, C. G. Principles of Polymer Processing, John Wiley & Sons, New York, 1979. 311 [19] Wick, C., Benedict, J. T., and Veilleux, R. F. Tool and Manufacturing Engineers Handbook, 4th ed., Vol. II: Forming. Society of Manufacturing Engineers, Dearborn, Michigan, 1984, Chapter 18. REVIEW QUESTIONS 13.1. What are some of the reasons why plastic shaping processes are important? 13.2. Identify the main categories of plastics shaping processes, as classified by the resulting product geometry. 13.3. Viscosity is an important property of a polymer melt in plastics shaping processes. Upon what parameters does viscosity depend? 13.4. How does the viscosity of a polymer melt differ from most fluids that are Newtonian. 13.5. What does viscoelasticity mean, when applied to a polymer melt? 13.6. Define die swell in extrusion. 13.7. Briefly describe the plastic extrusion process. 13.8. The barrel and screw of an extruder are generally divided into three sections; identify the sections. 13.9. What are the functions of the screen pack and breaker plate at the die end of the extruder barrel? 13.10. What are the various forms of extruded shapes and corresponding dies? 13.11. What is the distinction between plastic sheet and film? 13.12. What is the blown-film process for producing film stock? 13.13. Describe the calendering process. 13.14. Polymer fibers and filaments are used in several applications; what is the most important application commercially? 13.15. Technically, what is the difference between a fiber and a filament? 13.16. Among the synthetic fiber materials, which are the most important? 13.17. Briefly describe the injection molding process. 13.18. An injection-molding machine is divided into two principal components. Name them. 13.19. What are the two basic types of clamping units? 13.20. What is the function of gates in injection molds? 13.21. What are the advantages of a three-plate mold over a two-plate mold in injection molding? 13.22. Discuss some of the defects that can occur in plastic injection molding. 13.23. Describe structural-foam molding. 13.24. What are the significant differences in the equipment and operating procedures between injection molding of thermoplastics and injection molding of thermosets? 13.25. What is reaction injection molding? 13.26. What kinds of products are produced by blow molding? 13.27. What is the form of the starting material in thermoforming? 13.28. What is the difference between a positive mold and a negative mold in thermoforming? 13.29. Why are the molds generally more costly in mechanical thermoforming than in pressure or vacuum thermoforming? 13.30. What are the processes by which polymer foams are produced? 13.31. What are some of the general considerations that product designers must keep in mind when designing components out of plastics? 13.32. (Video) According to the injection molding videos, what are the four primary elements that influence the injection molding process? 13.33. (Video) According to the injection molding video, name the four types of mold design most common in industry. 13.34. (Video) According to the injection molding video, what is the most common type of injection molding machine used in industry? 13.35. (Video) According to the blow molding video, what materials are used in blow molding? Name three. 13.36. (Video) List the four most common blow-molding processes according to the video on blow molding. 13.37. (Video) List the stages of extrusion blow molding according to the video. 13.38. (Video) Name the four types of finishing operations performed on plastics, according to the plastics finishing video. 13.39. (Video) What are the different processes that can be used to apply decorations to plastic parts according to the plastics finishing video? MULTIPLE CHOICE QUIZ There are 29 correct answers in the following multiple choice questions (some questions have multiple answers that are correct). To attain a perfect score on the quiz, all correct answers must be given. Each correct answer is worth 1 point. Each E1C13 11/02/2009 312 15:30:34 Page 312 Chapter 13/Shaping Processes for Plastics omitted answer or wrong answer reduces the score by 1 point, and each additional answer beyond the correct number of answers reduces the score by 1 point. Percentage score on the quiz is based on the total number of correct answers. 13.1. The forward movement of polymer melt in an extruder barrel is resisted by drag flow, which is caused by the resistance to flow through the die orifice: (a) true or (b) false? 13.2. Which of the following are sections of a conventional extruder barrel for thermoplastics (three best answers): (a) compression section, (b) die section, (c) feed section, (d) heating section, (e) metering section, and (f) shaping section? 13.3. Which of the following processes are associated with the production of plastic sheet and film (three correct answers): (a) blown-film extrusion process, (b) calendering, (c) chill-roll extrusion, (d) doctor blade method, (e) spinning, (f) thermoforming, and (g) transfer molding? 13.4. The principal components of an injection molding machine are which two of the following: (a) clamping unit, (b) hopper, (c) injection unit, (d) mold, and (e) part ejection unit? 13.5. The parting line in injection molding is which one of the following: (a) the lines formed where polymer melt meets after flowing around a core in the mold, (b) the narrow gate sections where the parts are separated from the runner, (c) where the clamping unit is joined to the injection unit in the molding machine, or (d) where the two mold halves come together? 13.6. The function of the ejection system is which one of the following: (a) move polymer melt into the mold cavity, (b) open the mold halves after the cavity is filled, (c) remove the molded parts from the runner system after molding, or (d) separate the part from the cavity after molding? 13.7. A three-plate mold offers which of the following advantages when compared to a two-plate mold (two best answers): (a) automatic separation of parts from runners, (b) gating is usually at the base of the part to reduce weld lines, (c) sprue does not solidify, and (d) stronger molded parts? 13.8. Which of the following defects or problems is associated with injection molding (three correct 13.9. 13.10. 13.11. 13.12. 13.13. 13.14. 13.15. 13.16. answers): (a) bambooing, (b) die swell, (c) drag flow, (d) flash, (e) melt fracture, (f) short shots, or (g) sink marks? In rotational molding, centrifugal force is used to force the polymer melt against the surfaces of the mold cavity where solidification occurs: (a) true or (b) false? Use of a parison is associated with which one of the following plastic shaping processes: (a) bi-injection molding, (b) blow molding, (c) compression molding, (d) pressure thermoforming, or (e) sandwich molding? A thermoforming mold with a convex form is called which one of the following: (a) a die, (b) a negative mold, (c) a positive mold, or (d) a three-plate mold? The term encapsulation refers to which one of the following plastics shaping processes: (a) casting, (b) compression molding, (c) extrusion of hollow forms, (d) injection molding in which a metal insert is encased in the molded part, or (e) vacuum thermoforming using a positive mold? The two most common polymer foams are which of the following: (a) polyacetal, (b) polyethylene, (c) polystyrene, (d) polyurethane, and (e) polyvinylchloride? In which of the following properties do plastic parts often compare favorably with metals (two best answers): (a) impact resistance, (b) resistance to ultraviolet radiation, (c) stiffness, (d) strength, (e) strength-to-weight ratio, and (f) temperature resistance? Which of the following processes are generally limited to thermoplastic polymers (two best answers): (a) blow molding, (b) compression molding, (c) reaction injection molding, (d) thermoforming, (e) transfer molding, and (f) wire coating? Which of the following processes would be applicable to produce hulls for small boats (three best answers): (a) blow molding, (b) compression molding, (c) injection molding, (d) rotational molding, and (e) vacuum thermoforming? PROBLEMS Extrusion 13.1. The diameter of an extruder barrel is 65 mm and its length ¼ 1.75 m. The screw rotates at 55 rev/min. The screw channel depth ¼ 5.0 mm, and the flight angle ¼ 18 . The head pressure at the die end of the barrel is 5.0  106 Pa. The viscosity of the polymer melt is given as 100 Pa-s. Find the volume flow rate of the plastic in the barrel. 13.2. An extruder has a diameter of 5.0 in and a length to diameter ratio of 26. The barrel heats the polypropylene melt to 450 F, which provides a melt E1C13 11/02/2009 15:30:35 Page 313 Problems 13.3. 13.4. 13.5. 13.6. 13.7. 13.8. 13.9. viscosity of 0.0025 lb-s/in2. The pitch of the screw is 4.2 in and the channel depth is 0.15 in. In operation the screw rotates at 50 rev/min and a head pressure of 450 lb/in2 is generated. What is the volume flow rate of polypropylene from the die at the end of the barrel? An extruder barrel has a diameter of 110 mm and a length of 3.0 m. The screw channel depth ¼ 7.0 mm, and its pitch ¼ 95 mm. The viscosity of the polymer melt is 105 Pa-s, and the head pressure in the barrel is 4.0 MPa. What rotational speed of the screw is required to achieve a volumetric flow rate of 90 cm3/s? An extruder has a barrel diameter of 2.5 in and a length of 6.0 ft. The screw has a channel depth of 0.25 in, a flight angle of 20 , and rotates at 55 rev/min. The material being extruded is polypropylene. At the present settings, the volumetric flow rate of the polymer melt is 1.50 in3/sec and the head pressure is 500 lb/in2. (a) Under these operating characteristics, what is the viscosity of the polypropylene? (b) Using Figure 13.2, approximate the temperature in  F of the polypropylene. An extruder has diameter ¼ 80 mm and length ¼ 2.0 m. Its screw has a channel depth ¼ 5 mm, flight angle ¼ 18 degrees, and it rotates at 1 rev/sec. The plastic melt has a shear viscosity ¼ 150 Pa-s. Determine the extruder characteristic by computing Qmax and pmax and then finding the equation of the straight line between them. Determine the helix angle A such that the screw pitch p is equal to the screw diameter D. This is called the ‘‘square’’ angle in plastics extrusion - the angle that provides a flight advance equal to one diameter for each rotation of the screw. An extruder barrel has a diameter of 2.5 in. The screw rotates at 60 rev/min; its channel depth ¼ 0.20 in, and its flight angle ¼ 17.5 . The head pressure at the die end of the barrel is 800 lb/in2 and the length of the barrel is 50 in. The viscosity of the polymer melt is 122  104 lb-sec/in2. Determine the volume flow rate of the plastic in the barrel. An extruder barrel has a diameter of 4.0 in and an L/D ratio of 28. The screw channel depth ¼ 0.25 in, and its pitch ¼ 4.8 in. It rotates at 60 rev/min. The viscosity of the polymer melt is 100  104 lb-sec/ in2. What head pressure is required to obtain a volume flow rate ¼ 150 in3/min? An extrusion operation produces continuous tubing with outside diameter ¼ 2.0 in and inside diameter ¼ 1.7 in. The extruder barrel has a diameter ¼ 4.0 in and length ¼ 10 ft. The screw rotates at 50 rev/min; it has a channel depth ¼ 0.25 in and flight angle ¼ 16 . The head pressure has a value of 13.10. 13.11. 13.12. 13.13. 13.14. 13.15. 313 350 lb/in2 and the viscosity of the polymer melt is 80  104 lb-sec/in2. Under these conditions, what is the production rate in length of tube/min, assuming the extrudate is pulled at a rate that eliminates the effect of die swell (i.e., the tubing has the same OD and ID as the die profile)? Continuous tubing is produced in a plastic extrusion operation through a die orifice whose outside diameter ¼ 2.0 in and inside diameter ¼ 1.5 in. The extruder barrel diameter ¼ 5.0 in and length ¼ 12 ft. The screw rotates at 50 rev/min; it has a channel depth ¼ 0.30 in and flight angle ¼ 16 . The head pressure has a value of 350 lb/in2 and the viscosity of the polymer melt is 90  104 lb-sec/in2. Under these conditions, what is the production rate in length of tube/min, given that the die swell ratio is 1.25. An extruder has barrel diameter and length of 100 mm and 2.8 m, respectively. The screw rotational speed ¼ 50 rev/min, channel depth ¼ 7.5 mm, and flight angle ¼ 17 . The plastic melt has a shear viscosity ¼ 175 Pa-s. Determine: (a) the extruder characteristic, (b) the shape factor Ks for a circular die opening with diameter ¼ 3.0 mm and length ¼ 12.0 mm, and (c) the operating point (Q and p). For Problem 01, assume the material is acrylic. (a) Using Figure 13.2, determine the temperature of the polymer melt. (b) If the temperature is lowered 20 C, estimate the resulting viscosity of the polymer melt. (Hint: the y-axis of Figure 13.2 is a log scale, not linear). Consider an extruder in which the barrel diameter ¼ 4.5 in and length ¼ 11 ft. The extruder screw rotates at 60 rev/min; it has channel depth ¼ 0.35 in and flight angle ¼ 20 . The plastic melt has a shear viscosity ¼ 125  104 lb-sec/in2. Determine: (a) Qmax and pmax; (b) the shape factor Ks for a circular die opening in which Dd ¼ 0.312 in and Ld ¼ 0.75 in; and (c) the values of Q and p at the operating point. An extruder has a barrel diameter ¼ 5.0 in and length ¼ 12 ft. The extruder screw rotates at 50 rev/ min; it has channel depth ¼ 0.30 in and flight angle ¼ 17.7 . The plastic melt has a shear viscosity ¼ 100  104 lb-sec/in2. Find: (a) the extruder characteristic, (b) the values of Q and p at the operating point, given that the die characteristic is Qx ¼ 0.00150 p. Given the data in Problem 13.14, except that the flight angle of the extruder screw is a variable instead of a constant 17.7 . Use a spreadsheet calculator to determine the value of the flight angle that maximizes the volumetric flow rate Qx. Explore values of flight angle between 10 and 20 . Determine the optimum value to the nearest tenth of a degree. E1C13 11/02/2009 314 15:30:35 Page 314 Chapter 13/Shaping Processes for Plastics A T-shaped cross section is extruded at a rate of 0.11 lb/sec. The density of water is 62.5 lb/ft3. (a) Find the equation for the extruder characteristic. (b) Find the operating point (Q and p), and (c) the die characteristic that is indicated by the operating point. 13.16. An extruder has a barrel diameter of 3.5 in and a length of 5.0 ft. It has a screw channel depth of 0.16 in and a flight angle of 22 . The extruder screw rotates at 75 rev/min. The polymer melt has a shear viscosity ¼ 65  104 lb-sec/in2 at the operating temperature of 525 F. The specific gravity of the polymer is 1.2 and its tensile strength is 8000 lb/in2. Injection Molding 13.17. Compute the percentage volumetric contraction of a polyethylene molded part, based on the value of shrinkage given in Table 13.1. 13.18. The specified dimension ¼ 225.00 mm for a certain injection molded part made of ABS. Compute the corresponding dimension to which the mold cavity should be machined, using the value of shrinkage given in Table 13.1. 13.19. The part dimension for a certain injection molded part made of polycarbonate is specified as 3.75 in. Compute the corresponding dimension to which the mold cavity should be machined, using the value of shrinkage given in Table 13.1. 13.20. The foreman in the injection molding department says that a polyethylene part produced in one of the operations has greater shrinkage than the calculations indicate it should have. The important dimension of the part is specified as 112.5  0.25 mm. However, the actual molded part measures 112.02 mm. (a) As a first step, the corresponding mold cavity dimension should be checked. Compute the correct value of the mold dimension, given that the shrinkage value for polyethylene is 0.025 (from Table 13.1). (b) What adjustments in process parameters could be made to reduce the amount of shrinkage? 13.21. An injection molded polyethylene part has a dimension of 2.500 in. A new material, polycarbonate, is used in the same mold. What is the expected corresponding dimension of the polycarbonate molding? Other Molding Operations and Thermoforming 13.22. The extrusion die for a polyethylene parison used in blow molding has a mean diameter of 18.0 mm. The size of the ring opening in the die is 2.0 mm. The mean diameter of the parison is observed to swell to a size of 21.5 mm after exiting the die orifice. If the diameter of the blow molded container is to be 150 mm, determine (a) the corresponding wall thickness of the container and (b) the wall thickness of the parison. 13.23. A parison is extruded from a die with outside diameter ¼ 11.5 mm and inside diameter ¼ 7.5 mm. The observed die swell is 1.25. The parison is used to blow mold a beverage container whose outside diameter ¼ 112 mm (a standard size 2-L soda bottle). (a) What is the corresponding wall thickness of the container? (b) Obtain an empty 2-L plastic soda bottle and (carefully) cut it across the diameter. Using a micrometer, measure the wall thickness to compare with your answer in (a). 13.24. A blow-molding operation is used to produce a bottle with a diameter of 2.250 in and a wall thickness of 0.045 in. The parison has a thickness of 0.290 in. The observed die swell ratio is 1.30. (a) What is the required diameter of the parison? (b) What is the diameter of the die? 13.25. An extrusion operation is used to produce a parison whose mean diameter ¼ 27 mm. The inside and outside diameters of the die that produced the parison are 18 mm and 22 mm, respectively. If the minimum wall thickness of the blow-molded container is to be 0.40 mm, what is the maximum possible diameter of the blow mold? 13.26. A rotational molding operation is to be used to mold a hollow playing ball out of polypropylene. The ball will be 1.25 ft in diameter and its wall thickness should be 3/32 in. What weight of PP powder should be loaded into the mold to meet these specifications? The specific gravity of the PP grade is 0.90, and the density of water is 62.4 lb/ft3. 13.27. The problem in a certain thermoforming operation is that there is too much thinning in the walls of the large cup-shaped part. The operation is conventional pressure thermoforming using a positive mold, and the plastic is an ABS sheet with an initial thickness of 3.2 mm. (a) Why is thinning occurring in the walls of the cup? (b) What changes could be made in the operation to correct the problem? E1C14 11/10/2009 13:21:53 Page 315 14 RUBBERPROCESSING TECHNOLOGY Chapter Contents 14.1 Rubber Processing and Shaping 14.1.1 Production of Rubber 14.1.2 Compounding 14.1.3 Mixing 14.1.4 Shaping and Related Processes 14.1.5 Vulcanization 14.2 Manufacture of Tires and Other Rubber Products 14.2.1 Tires 14.2.2 Other Rubber Products 14.2.3 Processing of Thermoplastic Elastomers 14.3 Product Design Considerations Many of the shaping processes used for plastics (Chapter 13) are also applicable to rubbers. However, rubber-processing technology is different in certain respects, and the rubber industry is largely separate from the plastics industry. The rubber industry and goods made of rubber are dominated by one product: tires. Tires are used in large numbers for automobiles, trucks, aircraft, and bicycles. Although pneumatic tires date from the late 1880s, rubber technology can be traced to the discovery in 1839 of vulcanization (Historical Note 8.2), the process by which raw natural rubber is transformed into a usable material through cross–linking of the polymer molecules. During its first century, the rubber industry was concerned only with the processing of natural rubber. Around World War II, synthetic rubbers were developed (Historical Note 8.3); today they account for the majority of rubber production. 14.1 RUBBER PROCESSING AND SHAPING Production of rubber goods can be divided into two basic steps: (1) production of the rubber itself, and (2) processing of the rubber into finished goods. Production of rubber differs, depending on whether it is natural or synthetic. The difference results from the source of the raw materials. Natural rubber (NR) is produced as an agricultural crop, whereas most synthetic rubbers are made from petroleum. Production of rubber is followed by processing into final products; this consists of (1) compounding, (2) mixing, (3) shaping, and (4) vulcanizing. Processing techniques for natural and synthetic rubbers are virtually the same, differences being in the chemicals used to effect vulcanization (cross–linking). This sequence does not apply to thermoplastic elastomers, whose shaping techniques are the same as for other thermoplastic polymers. 315 E1C14 11/10/2009 316 13:21:53 Page 316 Chapter 14/Rubber-Processing Technology There are several distinct industries involved in the production and processing of rubber. Production of raw natural rubber might be classified as farming because latex, the starting ingredient for natural rubber, is grown on large plantations located in tropical climates. By contrast, synthetic rubbers are produced by the petrochemical industry. Finally, the processing of these materials into tires, shoe soles, and other rubber products occurs at processor (fabricator) plants. The processors are commonly known as the rubber industry. Some of the great names in this industry include Goodyear, B. F. Goodrich, and Michelin. The importance of the tire is reflected in these names. 14.1.1 PRODUCTION OF RUBBER In this section we briefly survey the production of rubber before it goes to the processor. Our coverage distinguishes natural rubber and synthetic rubber. Natural Rubber Natural rubber is tapped from rubber trees (Hevea brasiliensis) as latex. The trees are grown on plantations in Southeast Asia and other parts of the world. Latex is a colloidal dispersion of solid particles of the polymer polyisoprene (Section 8.4.2) in water. Polyisoprene is the chemical substance that comprises rubber, and its content in the emulsion is about 30%. The latex is collected in large tanks, thus blending the yield of many trees together. The preferred method of recovering rubber from the latex involves coagulation. The latex is first diluted with water to about half its natural concentration. An acid such as formic acid (HCOOH) or acetic acid (CH3COOH) is added to cause the latex to coagulate after about 12 hours. The coagulum, now in the form of soft solid slabs, is then squeezed through a series of rolls that drive out most of the water and reduce the thickness to about 3 mm (1/8 in). The final rolls have grooves that impart a criss-cross pattern to the resulting sheets. The sheets are then draped over wooden frames and dried in smokehouses. The hot smoke contains creosote, which prevents mildew and oxidation of the rubber. Several days are normally required to complete the drying process. The resulting rubber, now in a form called ribbed smoked sheet, is folded into large bales for shipment to the processor. This raw rubber has a characteristic dark brown color. In some cases, the sheets are dried in hot air rather than smokehouses, and the term air-dried sheet is applied; this is considered to be a better grade of rubber. A still better grade, called pale crepe rubber, involves two coagulation steps; the first removes undesirable components of the latex, then the resulting coagulum is subjected to a more involved washing and mechanical working procedure, followed by warm air drying. The color of pale crepe rubber approaches a light tan. Synthetic Rubber The various types of synthetic rubber were identified in Section 8.4.3. Most synthetics are produced from petroleum by the same polymerization techniques used to synthesize other polymers (Section 8.1.1). However, unlike thermoplastic and thermosetting polymers, which are normally supplied to the fabricator as pellets or liquid resins, synthetic rubbers are supplied to rubber processors in the form of large bales. The industry has developed a long tradition of handling natural rubber in these unit loads. 14.1.2 COMPOUNDING Rubber is always compounded with additives. It is through compounding that the specific rubber is designed to satisfy the given application in terms of properties, cost, and processability. Compounding adds chemicals for vulcanization. Sulfur has traditionally E1C14 11/10/2009 13:21:53 Page 317 Section 14.1/Rubber Processing and Shaping 317 been used for this purpose. The vulcanization process and the chemicals used to accomplish it are discussed in Section 14.1.5. Additives include fillers that act either to enhance the rubber’s mechanical properties (reinforcing fillers) or to extend the rubber to reduce cost (nonreinforcing fillers). The single most important reinforcing filler in rubber is carbon black, a colloidal form of carbon, black in color, obtained from the thermal decomposition of hydrocarbons (soot). Its effect is to increase tensile strength and resistance to abrasion and tearing of the final rubber product. Carbon black also provides protection from ultraviolet radiation. These enhancements are especially important in tires. Most rubber parts are black in color because of their carbon black content. Although carbon black is the most important filler, others are also used. They include china clays—hydrous aluminum silicates (Al2Si2O5(OH)4), which provide less reinforcing than carbon black but are used when the black color is not acceptable; calcium carbonate (CaCO3), which is a nonreinforcing filler; and silica (SiO2), which can serve reinforcing or nonreinforcing functions, depending on particle size; and other polymers, such as styrene, PVC, and phenolics. Reclaimed (recycled) rubber is also added as a filler in some rubber products, but usually not in proportions exceeding 10%. Other additives compounded with the rubber include antioxidants, to retard aging by oxidation; fatigue- and ozone-protective chemicals; coloring pigments; plasticizers and softening oils; blowing agents in the production of foamed rubber; and mold-release compounds. Many products require filament reinforcement to reduce extensibility but retain the other desirable properties of rubber. Tires and conveyor belts are notableexamples. Filaments used for this purpose include cellulose, nylon, and polyester. Fiberglass and steel are also used as reinforcements (e.g., steel-belted radial tires). These continuous fiber materials must be added as part of the shaping process; they are not mixed with the other additives. 14.1.3 MIXING The additives must be thoroughly mixed with the base rubber to achieve uniform dispersion of the ingredients. Uncured rubbers possess high viscosity. Mechanical working experienced by the rubber can increase its temperature up to 150 C (300 F). If vulcanizing agents were present from the start of mixing, premature vulcanization would result—the rubber processor’s nightmare [7]. Accordingly, a two-stage mixing process is usually employed. In the first stage, carbon black and other nonvulcanizing additives are combined with the raw rubber. The term masterbatch is used for this first-stage mixture. After thorough mixing has been accomplished, and time for cooling has been allowed, the second stage is carried out in which the vulcanizing agents are added. Equipment for mixing includes the two-roll mill and internal mixers such as the Banbury mixer (Figure 14.1). The two-roll mill consists of two parallel rolls, supported in a frame so they can be brought together to obtain a desired ‘‘nip’’ (gap size), and driven to rotate at the same or slightly different speeds. An internal mixer has two rotors encased in a jacket, as in Figure 14.1(b) for the Banbury-type internal mixer. The rotors have blades and rotate in opposite directions at different speeds, causing a complex flow pattern in the contained mixture. 14.1.4 SHAPING AND RELATED PROCESSES Shaping processes for rubber products can be divided into four basic categories: (1) extrusion, (2) calendering, (3) coating, and (4) molding and casting. Most of these E1C14 11/10/2009 318 13:21:53 Page 318 Chapter 14/Rubber-Processing Technology FIGURE 14.1 Mixers used in rubber processing: (a) two-roll mill and (b) Banbury-type internal mixer. These machines can also be used for mastication of natural rubber. processes are discussed in the previous chapter. We will examine the special issues that arise when they are applied to rubber. Some products require several basic processes plus assembly work in their manufacture, for example, tires. Extrusion Extrusion of polymers is discussed in the preceding chapter. Screw extruders are generally used for extrusion of rubber. As with extrusion of thermosetting plastics, the L/D ratio of the extruder barrels is less than for thermoplastics, typically in the range 10 to 15, to reduce the risk of premature cross–linking. Die swell occurs in rubber extrudates, because the polymer is in a highly plastic condition and exhibits the memory property. It has not yet been vulcanized. Calendering This process involves passing rubber stock through a series of gaps of decreasing size made by a stand of rotating rolls (Section 13.3). The rubber process must be operated at lower temperatures than for thermoplastic polymers, to avoid premature vulcanization. Also, equipment used in the rubber industry is of heavier construction than that used for thermoplastics, because rubber is more viscous and harder to form. The output of the process is a rubber sheet of thickness determined by the final roll gap; again, swelling occurs in the sheet, causing its thickness to be slightly greater than the gap size. Calendering can also be used to coat or impregnate textile fabrics to produce rubberized fabrics. There are problems in producing thick sheet by either extrusion or calendering. Thickness control is difficult in the former process, and air entrapment occurs in the latter. These problems are largely solved when extrusion and calendering are combined in the roller die process (Figure 14.2). The extruder die is a slit that feeds the calender rolls. Coating Coating or impregnating fabrics with rubber is an important process in the rubber industry. These composite materials are used in automobile tires, conveyor belts, inflatable rafts, and waterproof cloth for tarpaulins, tents, and rain coats. The coating of rubber onto substrate fabrics includes a variety of processes. We have previously indicated E1C14 11/10/2009 13:21:53 Page 319 Section 14.1/Rubber Processing and Shaping 319 FIGURE 14.2 Roller die process rubber extrusion followed by rolling. that calendering is one of the coating methods. Figure 14.3 illustrates one possible way in which the fabric is fed into the calendering rolls to obtain a reinforced rubber sheet. Alternatives to calendering include skimming, dipping, and spraying. In the skimming process, a thick solution of rubber compound in an organic solvent is applied to the fabric as it is unreeled from a supply spool. The coated fabric passes under a doctor blade that skims the solvent to the proper thickness, and then moves into a steam chamber where the solvent is driven off by heat. As its name suggests, dipping involves temporary immersion of the fabric into a highly fluid solution of rubber, followed by drying. Likewise, in spraying, a spray gun is used to apply the rubber solution. Molding and Casting Molded articles include shoe soles and heels, gaskets and seals, suction cups, and bottle stops. Many foamed rubber parts are produced by molding. In addition, molding is an important process in tire production. Principal molding processes for rubber are (1) compression molding, (2) transfer molding, and (3) injection molding. Compression molding is the most important technique because of its use in tire manufacture. Curing (vulcanizing) is accomplished in the mold in all three processes, this representing a departure from the shaping methods already discussed, which require a separate vulcanizing step. With injection molding of rubber, there are risks of premature curing similar to those faced in the same process when applied to thermosetting plastics. Advantages of injection molding over traditional methods for producing rubber parts include better dimensional control, less scrap, and shorter cycle times. In addition to its use in the molding of conventional rubbers, injection molding is also applied for thermoplastic elastomers. Because of high mold costs, large production quantities are required to justify injection molding. A form of casting, called dip casting, is used for producing rubber gloves and overshoes. It involves submersion of a positive mold in a liquid polymer (or a heated form into plastisol) for a certain duration (the process may involve repeated dippings) to form the desired thickness. The coating is then stripped from the form and cured to cross–link the rubber. FIGURE 14.3 Coating of fabric with rubber using a calendering process. E1C14 11/10/2009 320 13:21:53 Page 320 Chapter 14/Rubber-Processing Technology Long-chain rubber molecules FIGURE 14.4 Effect of vulcanization on the rubber molecules: (1) raw rubber; (2) vulcanized (cross–linked) rubber. Variations of (2) include (a) soft rubber, low degree of cross–linking; and (b) hard rubber, high degree of cross–linking. (a) Crosslinks (b) (1) (2) 14.1.5 VULCANIZATION Vulcanization is the treatment that accomplishes cross–linking of elastomer molecules, so that the rubber becomes stiffer and stronger but retains extensibility. It is a critical step in the rubber processing sequence. On a submicroscopic scale, the process can be pictured as in Figure 14.4, in which the long-chain molecules of the rubber become joined at certain tie points, the effect of which is to reduce the ability of the elastomer to flow. A typical soft rubber has one or two cross–links per thousand units (mers). As the number of cross–links increases, the polymer becomes stiffer and behaves more like a thermosetting plastic (hard rubber). Vulcanization, as it was first invented by Goodyear, involved the use of sulfur (about 8 parts by weight of S mixed with 100 parts of natural rubber) at a temperature of 140 C (280 F) for about 5 hours. No other chemicals were included in the process. Vulcanization with sulfur alone is no longer used as a commercial treatment today, because of the long curing times. Various other chemicals, including zinc oxide (ZnO) and stearic acid (C18H36O2), are combined with smaller doses of sulfur to accelerate and strengthen the treatment. The resulting cure time is 15 to 20 minutes for a typical passenger car tire. In addition, various nonsulfur vulcanizing treatments have been developed. In rubber-molding processes, vulcanization is accomplished in the mold by maintaining the mold temperature at the proper level for curing. In the other forming processes, vulcanization is performed after the part has been shaped. The treatments generally divide between batch processes and continuous processes. Batch methods include the use of an autoclave, a steam-heated pressure vessel; and gas curing, in which a heated inert gas such as nitrogen cures the rubber. Many of the basic processes make a continuous product, and if the output is not cut into discrete pieces, continuous vulcanization is appropriate. Continuous methods include high-pressure steam, suited to the curing of rubber coated wire and cable; hot-air tunnel, for cellular extrusions and carpet underlays [3]; and continuous drum cure, in which continuous rubber sheets (e.g., belts and flooring materials) pass through one or more heated rolls to effect vulcanization. 14.2 MANUFACTURE OF TIRES AND OTHER RUBBER PRODUCTS Tires are the principal product of the rubber industry, accounting for about three fourths of total tonnage. Other important products include footwear, hose, conveyor belts, seals, shock-absorbing components, foamed rubber products, and sports equipment. E1C14 11/10/2009 13:21:53 Page 321 Section 14.2/Manufacture of Tires and Other Rubber Products 321 14.2.1 TIRES Pneumatic tires are critical components of the vehicles on which they are used. They are used on automobiles, trucks, buses, farm tractors, earth-moving equipment, military vehicles, bicycles, motorcycles, and aircraft. Tires support the weight of the vehicle and the passengers and cargo on board; they transmit the motor torque to propel the vehicle (except on aircraft); and they absorb vibrations and shock to provide a comfortable ride. Tire Construction and Production Sequence A tire is an assembly of many parts, whose manufacture is unexpectedly complex. A passenger car tire consists of about 50 individual pieces; a large earthmover tire may have as many as 175. To begin with, there are three basic tire constructions: (a) diagonal ply, (b) belted bias, and (c) radial ply, pictured in Figure 14.5. In all three cases, the internal structure of the tire, known as the carcass, consists of multiple layers of rubber-coated cords, called plies. The cords are strands of various materials such as nylon, polyester, fiberglass, and steel, which provide inextensibility to reinforce the rubber in the carcass. The diagonal ply tire has the cords running diagonally, but in perpendicular directions in adjacent layers. A typical diagonal ply tire may have four plies. The belted bias tire is constructed of diagonal plies with opposite bias but adds several more layers around the outside periphery of the carcass. These belts increase the stiffness of the tire in the tread area and limit its diametric expansion during inflation. The cords in the belt also run diagonally, as indicated in the sketch. A radial tire has plies running radially rather than diagonally; it also uses belts around the periphery for support. A steel-belted radial is a tire in which the circumferential FIGURE 14.5 Three principal tire constructions: (a) diagonal ply, (b) belted bias, and (c) radial ply. E1C14 11/10/2009 322 13:21:54 Page 322 Chapter 14/Rubber-Processing Technology belts have cords made of steel. The radial construction provides a more flexible sidewall, which tends to reduce stress on the belts and treads as they continually deform on contact with the flat road surface during rotation. This effect is accompanied by greater tread life, improved cornering and driving stability, and a better ride at high speeds. In each construction, the carcass is covered by solid rubber that reaches a maximum thickness in the tread area. The carcass is also lined on the inside with a rubber coating. For tires with inner tubes, the inner liner is a thin coating applied to the innermost ply during its fabrication. For tubeless tires, the inner liner must have low permeability because it holds the air pressure; it is generally a laminated rubber. Tire production can be summarized in three steps: (1) preforming of components, (2) building the carcass and adding rubber strips to form the sidewalls and treads, and (3) molding and curing the components into one integral piece. The descriptions of these steps that follow are typical; there are variations in processing depending on construction, tire size, and type of vehicle on which the tire will be used. Preforming of Components As Figure 14.5 shows, the carcass consists of a number of separate components, most of which are rubber or reinforced rubber. These, as well as the sidewall and tread rubber, are produced by continuous processes and then pre-cut to size and shape for subsequent assembly. The components, labeled in Figure 14.5, and the preforming processes to fabricate them are: å Bead coil. Continuous steel wire is rubber-coated, cut, coiled, and the ends joined. å Plies. Continuous fabric (textile, nylon, fiber glass, steel) is rubber coated in a calendering process and pre-cut to size and shape. å Inner lining. For tube tires, the inner liner is calendered onto the innermost ply. For tubeless tires, the liner is calendered as a two-layered laminate. å Belts. Continuous fabric is rubber coated (similar to plies), but cut at different angles for better reinforcement; then made into a multi-ply belt. å Tread. Extruded as continuous strip; then cut and preassembled to belts. å Sidewall. Extruded as continuous strip; then cut to size and shape. Building the Carcass The carcass is traditionally assembled using a machine known as a building drum, whose main element is a cylindrical arbor that rotates. Pre-cut strips that form the carcass are built up around this arbor in a step-by-step procedure. The layered plies that form the cross section of the tire are anchored on opposite sides of the rim by two bead coils. The bead coils consist of multiple strands of high-strength steel wire. Their function is to provide a rigid support when the finished tire is mounted on the wheel rim. Other components are combined with the plies and bead coils. These include various wrappings and filler pieces to give the tire the proper strength, heat resistance, air retention, and fitting to the wheel rim. After these parts are placed around the arbor and the proper number of plies have been added, the belts are applied. This is followed by the outside rubber that will become the sidewall and tread.1 At this point in the process, the treads are rubber strips of uniform cross section—the tread design is added later in molding. The building drum is collapsible, so that the unfinished tire can be removed when finished. The form of the tire at this stage is roughly tubular, as portrayed in Figure 14.6. Molding and Curing Tire molds are usually two-piece construction (split molds) and contain the tread pattern to be impressed on the tire. The mold is bolted into a press, one half attached to the upper platen (the lid) and the bottom half fastened to the lower 1 Technically, the tread and sidewall are not usually considered to be components of the carcass. E1C14 11/10/2009 13:21:54 Page 323 Section 14.2/Manufacture of Tires and Other Rubber Products 323 FIGURE 14.6 Tire just before removal from building drum, before molding and curing. platen (the base). The uncured tire is placed over an expandable diaphragm and inserted between the mold halves, as in Figure 14.7. The press is then closed and the diaphragm expanded, so that the soft rubber is pressed against the cavity of the mold. This causes the tread pattern to be imparted to the rubber. At the same time, the rubber is heated, both from the outside by the mold and from the inside by the diaphragm. Circulating hot water or steam under pressure are used to heat the diaphragm. The duration of this curing step depends on the thickness of the tire wall. A typical passenger tire can be cured in about 15 minutes. Bicycle tires cure in about 4 minutes, whereas tires for large earth-moving equipment take several hours to cure. After curing is completed, the tire is cooled and removed from the press. 14.2.2 OTHER RUBBER PRODUCTS Most other rubber products are made by less complex processes. Rubber belts are widely used in conveyors and mechanical power transmission systems. As with tires, rubber is an ideal material for these products, but the belt must have flexibility but little or no extensibility to function. Accordingly, it is reinforced with fibers, commonly polyester or nylon. Fabrics of these polymers are usually coated in calendering operations, assembled together to obtain the required number of plies and thickness, and subsequently vulcanized by continuous or batch heating processes. Rubber hose can be either plain or reinforced. Plain hose is extruded tubing. Reinforced tube consists of an inner tube, a reinforcing layer (sometimes called the carcass), and a cover. The internal tubing is extruded of a rubber that has been FIGURE 14.7 Tire molding (tire is shown in cross-sectional view): (1) the uncured tire is placed over expandable diaphragm; (2) the mold is closed and the diaphragm is expanded to force uncured rubber against mold cavity, impressing tread pattern into rubber; mold and diaphragm are heated to cure rubber. E1C14 11/10/2009 324 13:21:54 Page 324 Chapter 14/Rubber-Processing Technology compounded for the particular substance that will flow through it. The reinforcement layer is applied to the tube in the form of a fabric, or by spiraling, knitting, braiding, or other application method. The outer layer is compounded to resist environmental conditions. It is applied by extrusion, using rollers, or other techniques. Footwear components include soles, heels, rubber overshoes, and certain upper parts. Various rubbers are used to make footwear components (Section 8.4). Molded parts are produced by injection molding, compression molding, and certain special molding techniques developed by the shoe industry; the rubbers include both solid and foamed varieties. In some cases, for low volume production, manual methods are used to cut rubber from flat stock. Rubber is widely used in sports equipment and supplies, including ping pong paddle surfaces, golf club grips, football bladders, and sports balls of various kinds. Tennis balls, for example, are made in significant numbers. Production of these sports products relies on the various shaping processes discussed in Section 14.1.4, as well as special techniques that have been developed for particular items. 14.2.3 PROCESSING OF THERMOPLASTIC ELASTOMERS A thermoplastic elastomer (TPE) is a thermoplastic polymer that possesses the properties of a rubber (Section 8.4.3); the term thermoplastic rubber is also used. TPEs can be processed like thermoplastics, but their applications are those of an elastomer. The most common shaping processes are injection molding and extrusion, which are generally more economical and faster than the traditional processes used for rubbers that must be vulcanized. Molded products include shoe soles, athletic footwear, and automotive components such as fender extensions and corner panels (but not tires—TPEs have been found to be unsatisfactory for that application). Extruded items include insulation coating for electrical wire, tubing for medical applications, conveyor belts, sheet and film stock. Other shaping techniques for TPEs include blow molding and thermoforming (Sections 13.8 and 13.9); these processes cannot be used for vulcanized rubbers. 14.3 PRODUCT DESIGN CONSIDERATIONS Many of the same guidelines used for plastics apply to rubber products. There are differences, owing to the elastomeric properties of rubber. The following are compiled largely from Bralla [4]; they apply to conventional soft rubber, not hard rubber. å Economic production quantities. Rubber parts produced by compression molding (the traditional process) can often be produced in quantities of a thousand or less. The mold cost is relatively low compared with other molding methods. Injection molding, as with plastic parts, requires higher production quantities to justify the more expensive mold. å Draft. Draft is usually unnecessary for rubber molded parts. The flexibility of the material allows it to deform for removal from the mold. Shallow undercuts, although undesirable, are possible with rubber-molded parts for the same reason. The low stiffness and high elasticity of the material permits removal from the mold. å Holes. Holes are difficult to cut into the rubber after initial forming, due the flexibility of the material. It is generally desirable to mold holes into the rubber during the primary shaping process. E1C14 11/10/2009 13:21:54 Page 325 Multiple Choice Quiz 325 å Screw threads. Screw threads are generally not incorporated into molded rubber parts; the elastic deformability of rubber makes it difficult to assemble parts using the threads, and stripping is a problem once inserted. REFERENCES [1] Alliger, G., and Sjothun, I. J. (eds.). Vulcanization of Elastomers. Krieger Publishing, New York, 1978. [2] Billmeyer, Fred, W., Jr. Textbook of Polymer Science. 3rd ed. John Wiley & Sons, New York, 1984. [3] Blow, C. M., and Hepburn, C. Rubber Technology and Manufacture. 2nd ed. Butterworth-Heinemann, London, 1982. [4] Bralla, J. G. (ed.). Design for Manufacturability Handbook. 2nd ed. McGraw-Hill, New York, 1999. [5] Hofmann, W. Rubber Technology Handbook. HanserGardner Publications, Cincinnati, Ohio, 1989. [6] Mark, J. E., and Erman, B. (eds.), Science and Technology of Rubber, 3rd ed. Academic Press, Orlando, Florida, 2005. [7] Morton-Jones, D. H. Polymer Processing. Chapman and Hall, London, 1989. REVIEW QUESTIONS 14.1. How is the rubber industry organized? 14.2. How is raw rubber recovered from the latex that is tapped from a rubber tree? 14.3. What is the sequence of processing steps required to produce finished rubber goods? 14.4. What are some of the additives that are combined with rubber during compounding? 14.5. Name the four basic categories of processes used to shape rubber. 14.6. What does vulcanization do to the rubber? 14.7. Name the three basic tire constructions and briefly identify the differences in their construction. 14.8. What are the three basic steps in the manufacture of a pneumatic tire? 14.9. What is the purpose of the bead coil in a pneumatic tire? 14.10. What is a TPE? 14.11. Many of the design guidelines that are applicable to plastics are also applicable to rubber. However, the extreme flexibility of rubber results in certain differences. What are some examples of these differences? MULTIPLE CHOICE QUIZ There are 10 correct answers in the following multiple choice questions (some questions have multiple answers that are correct). To attain a perfect score on the quiz, all correct answers must be given. Each correct answer is worth 1 point. Each omitted answer or wrong answer reduces the score by 1 point, and each additional answer beyond the correct number of answers reduces the score by 1 point. Percentage score on the quiz is based on the total number of correct answers. 14.1. The most important rubber product is which one of the following: (a) footwear, (b) conveyor belts, (c) pneumatic tires, or (d) tennis balls? 14.2. The chemical name of the ingredient recovered from the latex of the rubber tree is which one of the following: (a) polybutadiene, (b) polyisobutylene, (c) polyisoprene, or (d) polystyrene? 14.3. Of the following rubber additives, which one would rank as the single most important: (a) antioxidants, (b) carbon black, (c) clays and other hydrous aluminum silicates, (d) plasticizers and softening oils, or (e) reclaimed rubber? 14.4. Which one of the following molding processes is the most important in the production of products made of conventional rubber: (a) compression molding, (b) injection molding, (c) thermoforming, or (d) transfer molding? 14.5. Which of the following ingredients do not contribute to the vulcanizing process (two correct E1C14 11/10/2009 326 13:21:54 Page 326 Chapter 14/Rubber-Processing Technology answers): (a) calcium carbonate, (b) carbon black, (c) stearic acid, (d) sulfur, and (e) zinc oxide? 14.6. How many minutes are required to cure (vulcanize) a modern passenger car tire: (a) 5, (b) 15, (c) 25, or (d) 45? 14.7. When is the tread pattern imprinted onto the circumference of the tire: (a) during preforming, (b) while building the carcass, (c) during molding, or (d) during curing? 14.8. Which of the following are not normally used in the processing of thermoplastic elastomers (two correct answers): (a) blow molding, (b) compression molding, (c) extrusion, (d) injection molding, or (e) vulcanization? E1C15 11/11/2009 14:42:13 Page 327 15 SHAPING PROCESSES FOR POLYMER MATRIX COMPOSITES Chapter Contents 15.1 Starting Materials for PMCs 15.1.1 Polymer Matrix 15.1.2 Reinforcing Agent 15.1.3 Combining Matrix and Reinforcement 15.2 Open Mold Processes 15.2.1 Hand Lay-Up 15.2.2 Spray-Up 15.2.3 Automated Tape-Laying Machines 15.2.4 Curing 15.3 Closed 15.3.1 15.3.2 15.3.3 Mold Processes Compression Molding PMC Processes Transfer Molding PMC Processes Injection Molding PMC Processes 15.4 Filament Winding 15.5 Pultrusion Processes 15.5.1 Pultrusion 15.5.2 Pulforming 15.6 Other PMC Shaping Processes In this chapter we consider manufacturing processes by which polymer matrix composites are shaped into useful components and products. A polymer matrix composite (PMC) is a composite material consisting of a polymer embedded with a reinforcing phase such as fibers or powders. The technological and commercial importance of the processes in this chapter derives from the growing use of this class of material, especially fiber-reinforced polymers (FRPs). In popular usage, PMC generally refers to fiberreinforced polymers. FRP composites can be designed with very high strength-to-weight and stiffness-to-weight ratios. These features make them attractive in aircraft, cars, trucks, boats, and sports equipment. Some of the shaping processes described in this chapter are slow and labor intensive. In general, techniques for shaping composites are less efficient than manufacturing processes for other materials. There are two reasons for this: (1) composite materials are more complex than other materials, consisting as they do of two or more phases and the need to orient the reinforcing phase in the case of fiberreinforced plastics; and (2) processing technologies for composites have not been the object of improvement and refinement over as many years as processes for other materials. The variety of shaping methods for fiber-reinforced polymers is sometimes bewildering to students on first reading. Let us provide a road map for the reader entering this new territory. FRP composite shaping processes can be divided into five categories, as organized in Figure 15.1: (1) open mold processes, (2) closed mold processes, (3) filament winding, (4) pultrusion processes, and (5) other. Open mold processes include some of the original manual procedures for laying resins and fibers onto forms. Closed mold processes are much the same as those used in 327 E1C15 11/11/2009 328 14:42:13 Page 328 Chapter 15/Shaping Processes for Polymer Matrix Composites Hand lay-up Open mold processes Processes for continuous-fiber PMCs Closed mold processes Filament winding Automated tape laying Compression molding Resin transfer molding Pultrusion processes FRP shaping processes Other Tube rolling Open mold processes Spray-up Compression molding Processes for short-fiber PMCs FIGURE 15.1 Classification of manufacturing processes for fiber-reinforced polymer composites. Closed mold processes Transfer molding Injection molding Centrifugal casting Other Continuous laminating plastic molding; the reader will recognize the names—compression molding, transfer molding, and injection molding—although the names are sometimes changed and modifications are sometimes made for PMCs. In filament winding, continuous filaments that have been dipped in liquid resin are wrapped around a rotating mandrel; when the resin cures, a rigid, hollow, generally cylindrical shape is created. Pultrusion is a shaping process for producing long, straight sections of constant cross section; it is similar to extrusion, but adapted to include continuous fiber reinforcement. The ‘‘other’’ category includes several operations that do not fit into the previous categories. Some of these processes are used to shape composites with continuous fibers, whereas others are used for short fiber PMCs. Figure 15.1 provides an overview of the processes in each division. Let us begin our coverage by exploring how the individual phases in a PMC are produced and how these phases are combined into the starting materials for shaping. For a good overview of the PMC processes, the reader should view the video clip titled Composite Materials and Manufacturing. VIDEO CLIP Composite Materials and Manufacturing. This clip contains three segments: (1) composite materials, (2) composites manufacturing processes, and (3) composites overview. Segment (2) is especially relevant to this chapter. E1C15 11/11/2009 14:42:13 Page 329 Section 15.1/Starting Materials for PMCS 15.1 329 STARTING MATERIALS FOR PMCS In a PMC, the starting materials are a polymer and a reinforcing phase. They are processed separately before becoming phases in the composite. This section considers how these materials are produced before being combined, and then how they are combined to make the composite part. 15.1.1 POLYMER MATRIX All three of the basic polymer types—thermoplastics, thermosets, and elastomers—are used as matrices in PMCs. Thermosetting (TS) polymers are the most common matrix materials. The principal TS polymers are phenolics, unsaturated polyesters, and epoxies. Phenolics are associated with the use of particulate reinforcing phases, whereas polyesters and epoxies are more closely associated with FRPs. Thermoplastic (TP) polymers are also used in PMCs, and in fact, most molding compounds are composite materials that include fillers and/or reinforcing agents. Most elastomers are composite materials because nearly all rubbers are reinforced with carbon black. Shaping processes for rubbers are covered in Chapter 14. In this chapter, coverage is limited to the processing of PMCs that use TS and TP polymers as the matrix. Many of the polymer shaping processes discussed in Chapter 13 are applicable to polymer matrix composites. However, combining the polymer with the reinforcing agent sometimes complicates the operations. 15.1.2 REINFORCING AGENT The reinforcing phase can be any of several geometries and materials. The geometries include fibers, particles, and flakes, and the materials are ceramics, metals, other polymers, or elements such as carbon or boron. The role of the reinforcing phase and some of its technical features are discussed in Section 9.1.2. Fibers Common fiber materials in FRPs are glass, carbon, and the polymer Kevlar. Fibers of these materials are produced by various techniques, some of which we have covered in other chapters. Glass fibers are produced by drawing through small orifices (Section 12.2.3). For carbon, a series of heating treatments is performed to convert a precursor filament containing a carbon compound into a more pure carbon form. The precursor can be any of several substances, including polyacrylonitrile (PAN), pitch (a black carbon resin formed in the distillation of coal tar, wood tar, petroleum, etc.), or rayon (cellulose). Kevlar fibers are produced by extrusion combined with drawing through small orifices in a spinneret (Section 13.4). Starting as continuous filaments, the fibers are combined with the polymer matrix in any of several forms, depending on the properties desired in the material and the processing method to be used to shape the composite. In some fabrication processes, the filaments are continuous, whereas in others they are chopped into short lengths. In the continuous form, individual filaments are usually available as rovings. A roving is a collection of untwisted (parallel) continuous strands; this is a convenient form for handling and processing. Rovings typically contain from 12 to 120 individual strands. By contrast, a yarn is a twisted collection of filaments. Continuous rovings are used in several PMC processes, including filament winding and pultrusion. The most familiar form of continuous fiber is a cloth—a fabric of woven yarns. Very similar to a cloth, but distinguished here, is a woven roving, a fabric consisting of untwisted filaments rather than yarns. Woven rovings can be produced with unequal E1C15 11/11/2009 330 14:42:13 Page 330 Chapter 15/Shaping Processes for Polymer Matrix Composites numbers of strands in the two directions so that they possess greater strength in one direction than the other. Such unidirectional woven rovings are often preferred in laminated FRP composites. Fibers can also be prepared in the form of a mat—a felt consisting of randomly oriented short fibers held loosely together with a binder, sometimes in a carrier fabric. Mats are commercially available as blankets of various weights, thicknesses, and widths. Mats can be cut and shaped for use as preforms in some of the closed mold processes. During molding, the resin impregnates the preform and then cures, thus yielding a fiberreinforced molding. Particles and Flakes Particles and flakes are really in the same class. Flakes are particles whose length and width are large relative to thickness. We discuss these and other issues on characterization of engineering powders in Section 16.1. Production methods for metal powders are discussed in Section 16.2, and techniques for producing ceramic powders are discussed in Section 17.1.1. 15.1.3 COMBINING MATRIX AND REINFORCEMENT Incorporation of the reinforcing agent into the polymer matrix either occurs during the shaping process or beforehand. In the first case, the starting materials arrive at the fabricating operation as separate entities and are combined into the composite during shaping. Examples of this case are filament winding and pultrusion. The starting reinforcement in these processes consists of continuous fibers. In the second case, the two component materials are combined into some preliminary form that is convenient for use in the shaping process. Nearly all of the thermoplastics and thermosets used in the plastic shaping processes are really polymers combined with fillers (Section 8.1.5). The fillers are either short fibers or particulate (including flakes). Of greatest interest in this chapter are the starting forms used in processes designed for FRP composites. We might think of the starting forms as prefabricated composites that arrive ready for use at the shaping process. These forms are molding compounds and prepregs. Molding Compounds Molding compounds are similar to those used in plastic molding. They are designed for use in molding operations, and so they must be capable of flowing. Most molding compounds for composite processing are thermosetting polymers. Accordingly, they have not been cured before shape processing. Curing is done during and/or after final shaping. FRP composite molding compounds consist of the resin matrix with short, randomly dispersed fibers. They come in several forms. Sheet molding compound (SMC) is a combination of TS polymer resin, fillers and other additives, and chopped glass fibers (randomly oriented) all rolled into a sheet of typical thickness ¼ 6.5 mm (0.250 in). The most common resin is unsaturated polyester; fillers are usually mineral powders such as talc, silica, limestone; and the glass fibers are typically 12 to 75 mm (0.5 to 3.0 in) long and account for about 30% of the SMC by volume. SMCs are very convenient for handling and cutting to proper size as molding charges. Sheet molding compounds are generally produced between thin layers of polyethylene to limit evaporation of volatiles from the thermosetting resin. The protective coating also improves surface finish on subsequent molded parts. The process for fabricating continuous SMC sheets is depicted in Figure 15.2. Bulk molding compound (BMC) consists of similar ingredients as those in SMC, but the compounded polymer is in billet form rather than sheet. The fibers in BMC are shorter, typically 2 to 12 mm (0.1 to 0.5 in), because greater fluidity is required in the molding operations for which these materials are designed. Billet diameter is usually E1C15 11/11/2009 14:42:13 Page 331 Section 15.2/Open Mold Processes 331 FIGURE 15.2 Process for producing sheet molding compound (SMC). 25 to 50 mm (1 to 2 in). The process for producing BMC is similar to that for SMC, except extrusion is used to obtain the final billet form. BMC is also known as dough molding compound (DMC), because of its dough-like consistency. Other FRP molding compounds include thick molding compound (TMC), similar to SMC but thicker—up to 50 mm (2 in); and pelletized molding compounds—basically conventional plastic molding compounds containing short fibers. Prepregs Another prefabricated form for FRP shaping operations is prepreg, which consists of fibers impregnated with partially cured thermosetting resins to facilitate shape processing. Completion of curing must be accomplished during and/or after shaping. Prepregs are available in the form of tapes or cross-plied sheets or fabrics. The advantage of prepregs is that they are fabricated with continuous filaments rather than chopped random fibers, thus increasing strength and modulus of the final product. Prepreg tapes and sheets are associated with advanced composites (reinforced with boron, carbon/graphite, and Kevlar) as well as fiberglass. 15.2 OPEN MOLD PROCESSES The distinguishing feature of this family of FRP shaping processes is its use of a single positive or negative mold surface (Figure 15.3) to produce laminated FRP structures. Other names for open mold processes include contact lamination and contact molding. The starting materials (resins, fibers, mats, and woven rovings) are applied to the mold in layers, building up to the desired thickness. This is followed by curing and part removal. Common resins are unsaturated polyesters and epoxies, using fiberglass as the reinforcement. The moldings are usually large (e.g., boat hulls). The advantage of using an open mold is that the mold costs much less than if two matching molds were used. The disadvantage is that only the part surface in contact with the mold surface is finished; the FIGURE 15.3 Types of open mold: (a) positive and (b) negative. E1C15 11/11/2009 332 14:42:13 Page 332 Chapter 15/Shaping Processes for Polymer Matrix Composites other side is rough. For the best possible part surface on the finished side, the mold itself must be very smooth. There are several important open mold FRP processes. The differences are in the methods of applying the laminations to the mold, alternative curing techniques, and other variations. In this section we describe the family of open mold processes for shaping fiberreinforced plastics: (1) hand lay-up, (2) spray-up, (3) automated tape-laying machines, and (4) bag molding. We treat hand lay-up as the base process and the others as modifications and refinements. 15.2.1 HAND LAY-UP Hand lay-up is the oldest open mold method for FRP laminates, dating to the 1940s when it was first used to fabricate boat hulls. It is also the most labor-intensive method. As the name suggests, hand lay-up is a shaping method in which successive layers of resin and reinforcement are manually applied to an open mold to build the laminated FRP composite structure. The basic procedure consists of five steps, illustrated in Figure 15.4. The finished molding must usually be trimmed with a power saw to size the outside edges. In general, these same five steps are required for all of the open mold processes; the differences between methods occur in steps 3 and 4. In step 3, each layer of fiber reinforcement is dry when placed onto the mold. The liquid (uncured) resin is then applied by pouring, brushing, or spraying. Impregnation of resin into the fiber mat or fabric is accomplished by hand rolling. This approach is referred to as wet lay-up. An alternative approach is to use prepregs, in which the impregnated layers of fiber reinforcement are first prepared outside the mold and then laid onto the mold surface. Advantages cited for the prepregs include closer control over fiber–resin mixture and more efficient methods of adding the laminations [11]. Molds for open mold contact laminating can be made of plaster, metal, glass fiberreinforced plastic, or other materials. Selection of material depends on economics, surface quality, and other technical factors. For prototype fabrication, in which only one part is produced, plaster molds are usually adequate. For medium quantities, the mold can be FIGURE 15.4 Hand lay-up procedure: (1) mold is cleaned and treated with a mold release agent; (2) a thin gel coat (resin, possibly pigmented to color) is applied, which will become the outside surface of the molding; (3) when the gel coat has partially set, successive layers of resin and fiber are applied, the fiber being in the form of mat or cloth; each layer is rolled to fully impregnate the fiber with resin and remove air bubbles; (4) the part is cured; and (5) the fully hardened part is removed from the mold. E1C15 11/11/2009 14:42:13 Page 333 Section 15.2/Open Mold Processes 333 made of fiberglass-reinforced plastic. High production generally requires metal molds. Aluminum, steel, and nickel are used, sometimes with surface hardening on the mold face to resist wear. An advantage of metal, in addition to durability, is its high thermal conductivity that can be used to implement a heat-curing system, or simply to dissipate heat from the laminate while it cures at room temperature. Products suited to hand lay-up are generally large in size but low in production quantity. In addition to boat hulls, other applications include swimming pools, large container tanks, stage props, radomes, and other formed sheets. Automotive parts have also been made, but the method is not economical for high production. The largest moldings ever made by this process were ship hulls for the British Royal Navy: 85 m (280 ft) long [2]. 15.2.2 SPRAY-UP This represents an attempt to mechanize the application of resin-fiber layers and reduce the time for lay-up. It is an alternative for step 3 in the hand lay-up procedure. In the spray-up method, liquid resin and chopped fibers are sprayed onto an open mold to build successive FRP laminations, as in Figure 15.5. The spray gun is equipped with a chopper mechanism that feeds in continuous filament rovings and cuts them into fibers of length 25 to 75 mm (1 to 3 in) that are added to the resin stream as it exits the nozzle. The mixing action results in random orientation of the fibers in the layer—unlike hand lay-up, in which the filaments can be oriented if desired. Another difference is that the fiber content in spray-up is limited to about 35% (compared with a maximum of around 65% in hand lay-up). This is a shortcoming of the spraying and mixing process. Spraying can be accomplished manually using a portable spray gun or by an automated machine in which the path of the spray gun is preprogrammed and computer controlled. The automated procedure is advantageous for labor efficiency and environmental protection. Some of the volatile emissions from the liquid resins are hazardous, and the path-controlled machines can operate in sealed-off areas without humans present. However, rolling is generally required for each layer, as in hand lay-up. Products made by the spray-up method include boat hulls, bathtubs, shower stalls, automobile and truck body parts, recreational vehicle components, furniture, large structural panels, and containers. Movie and stage props are sometimes made by this method. Because products made by spray-up have randomly oriented short fibers, they are not as strong as those made by lay-up, in which the fibers are continuous and directed. FIGURE 15.5 method. Spray-up E1C15 11/11/2009 334 14:42:13 Page 334 Chapter 15/Shaping Processes for Polymer Matrix Composites 15.2.3 AUTOMATED TAPE-LAYING MACHINES This is another attempt to automate and accelerate step 3 in the lay-up procedure. Automated tape-laying machines operate by dispensing a prepreg tape onto an open mold following a programmed path. The typical machine consists of an overhead gantry, to which is attached the dispensing head, as shown in Figure 15.6. The gantry permits x-y-z travel of the head, for positioning and following a defined continuous path. The head itself has several rotational axes, plus a shearing device to cut the tape at the end of each path. Prepreg tape widths are commonly 75 mm (3 in), although 300 mm (12 in) widths have been reported [10]; thickness is around 0.13 mm (0.005 in). The tape is stored on the machine in rolls, which are unwound and deposited along the defined path. Each lamination is placed by following a series of back-and-forth passes across the mold surface until the parallel rows of tape complete the layer. Much of the work to develop automated tape-laying machines has been pioneered by the aircraft industry, which is eager to save labor costs and at the same time achieve the highest possible quality and uniformity in its manufactured components. The disadvantage of this and other computer numerically controlled machines is that it must be programmed, and programming takes time. 15.2.4 CURING Curing (step 4) is required of all thermosetting resins used in FRP laminated composites. Curing accomplishes cross-linking of the polymer, transforming it from its liquid or highly plastic condition into a hardened product. There are three principal process parameters in curing: time, temperature, and pressure. Curing normally occurs at room temperature for the TS resins used in hand lay-up and spray-up procedures. Moldings made by these processes are often large (e.g., boat hulls), and heating would be difficult for such parts. In some cases, days are required before room temperature curing is sufficiently complete to remove the part. If feasible, heat is added to speed the curing reaction. FIGURE 15.6 Automated tape-laying machine. (Courtesy of Cincinnati Milacron.) E1C15 11/11/2009 14:42:13 Page 335 Section 15.3/Closed Mold Processes 335 Heating is accomplished by several means. Oven curing provides heat at closely controlled temperatures; some curing ovens are equipped to draw a partial vacuum. Infrared heating can be used in applications in which it is impractical or inconvenient to place the molding in an oven. Curing in an autoclave provides control over both temperature and pressure. An autoclave is an enclosed chamber equipped to apply heat and/or pressure at controlled levels. In FRP composites processing, it is usually a large horizontal cylinder with doors at either end. The term autoclave molding is sometimes used to refer to the curing of a prepreg laminate in an autoclave. This procedure is used extensively in the aerospace industry to produce advanced composite components of very high quality. 15.3 CLOSED MOLD PROCESSES These molding operations are performed in molds consisting of two sections that open and close during each molding cycle. The name matched die molding is used for some of these processes. One might think that a closed mold is about twice the cost of a comparable open mold. However, tooling cost is even greater owing to the more complex equipment required in these processes. Despite their higher cost, advantages of a closed mold are (1) good finish on all part surfaces, (2) higher production rates, (3) closer control over tolerances, and (4) more complex three-dimensional shapes are possible. We divide the closed mold processes into three classes based on their counterparts in conventional plastic molding, even though the terminology is often different when polymer matrix composites are molded: (1) compression molding, (2) transfer molding, and (3) injection molding. 15.3.1 COMPRESSION MOLDING PMC PROCESSES In compression molding of conventional molding compounds (Section 13.7.1), a charge is placed in the lower mold section, and the sections are brought together under pressure, causing the charge to take the shape of the cavity. The mold halves are heated to cure the thermosetting polymer. When the molding is sufficiently cured, the mold is opened and the part is removed. There are several shaping processes for PMCs based on compression molding; the differences are mostly in the form of the starting materials. The flow of the resin, fibers, and other ingredients during the process is a critical factor in compression molding of FRP composites. SMC, TMC, and BMC Molding Several of the FRP molding compounds, namely sheet molding compound (SMC), bulk molding compound (BMC), and thick molding compound (TMC), can be cut to proper size and used as the starting charge in compression molding. Refrigeration is often required to store these materials prior to shape processing. The names of the molding processes are based on the starting molding compound (i.e., SMC molding is when the starting charge is precut sheet molding compound; BMC molding uses BMC cut to size as the charge; and so on). Preform Molding Another form of compression molding, called preform molding [11], involves placement of a precut mat into the lower mold section along with a polymer resin charge (e.g., pellets or sheet). The materials are then pressed between heated mold halves, causing the resin to flow and impregnate the fiber mat to produce a fiber reinforced molding. Variations of the process use either thermoplastic or thermosetting polymers. E1C15 11/11/2009 336 14:42:13 Page 336 Chapter 15/Shaping Processes for Polymer Matrix Composites FIGURE 15.7 Elastic reservoir molding: (1) foam is placed into mold between two fiber layers; (2) mold is closed, releasing resin from foam into fiber layers. Elastic Reservoir Molding The starting charge in elastic reservoir molding (ERM) is a sandwich consisting of a center of polymer foam between two dry fiber layers. The foam core is commonly open-cell polyurethane, impregnated with liquid resin such as epoxy or polyester, and the dry fiber layers can be cloth, woven roving, or other starting fibrous form. As depicted in Figure 15.7, the sandwich is placed in the lower mold section and pressed at moderate pressure—around 0.7 MPa (100 lb/in2). As the core is compressed, it releases the resin to wet the dry surface layers. Curing produces a lightweight part consisting of a low-density core and thin FRP skins. 15.3.2 TRANSFER MOLDING PMC PROCESSES In conventional transfer molding (Section 13.7.2), a charge of thermosetting resin is placed in a pot or chamber, heated, and squeezed by ram action into one or more mold cavities. The mold is heated to cure the resin. The name of the process derives from the fact that the fluid polymer is transferred from the pot into the mold. It can be used to mold TS resins in which the fillers include short fibers to produce an FRP composite part. Another form of transfer molding for PMCs is called resin transfer molding (RTM) [4], [11]; it refers to a closed mold process in which a preform mat is placed in the lower mold section, the mold is closed, and a thermosetting resin (e.g., polyester resin) is transferred into the cavity under moderate pressure to impregnate the preform. To confuse matters, RTM is sometimes called resin injection molding [4], [13]. (The distinction between transfer molding and injection molding is blurry anyway, as the reader may have noted in Chapter 13.) RTM has been used to manufacture such products as bathtubs, swimming pool shells, bench and chair seats, and hulls for small boats. Several enhancements of the basic RTM process have been developed [5]. One enhancement, called advanced RTM, uses high-strength polymers such as epoxy resins and continuous fiber reinforcement instead of mats. Applications include aerospace components, missile fins, and snow skis. Two additional processes are thermal expansion resin transfer molding and ultimately reinforced thermoset resin injection. Thermal expansion resin transfer molding (TERTM) is a patented process of TERTM, Inc. that consists of the following steps [5]: (1) A rigid polymer foam (e.g., polyurethane) is shaped into a preform. (2) The preform is enclosed in a fabric reinforcement and placed in a closed mold. (3) A thermosetting resin (e.g., epoxy) is injected into the mold to impregnate the fabric and surround the foam. (4) The mold is heated to expand the foam, fill the mold cavity, and cure the resin. Ultimately, reinforced thermoset resin injection (URTRI) is similar to TERTM except that the starting foam core is cast epoxy embedded with miniature hollow glass spheres. E1C15 11/11/2009 14:42:13 Page 337 Section 15.4/Filament Winding 337 15.3.3 INJECTION MOLDING PMC PROCESSES Injection molding is noted for low-cost production of plastic parts in large quantities. Although it is most closely associated with thermoplastics, the process can also be adapted to thermosets (Section 13.6.5). Conventional Injection Molding In PMC shape processing, injection molding is used for both TP- and TS-type FRPs. In the TP category, virtually all thermoplastic polymers can be reinforced with fibers. Chopped fibers must be used; if continuous fibers were used, they would be reduced anyway by the action of the rotating screw in the barrel. During injection from the chamber into the mold cavity, the fibers tend to become aligned during their journey through the nozzle. Designers can sometimes exploit this feature to optimize directional properties through part design, location of gates, and cavity orientation relative to the gate [8]. Whereas TP molding compounds are heated and then injected into a cold mold, TS polymers are injected into a heated mold for curing. Control of the process with thermosets is trickier because of the risk of premature cross-linking in the injection chamber. Subject to the same risk, injection molding can be applied to fiber-reinforced TS plastics in the form of pelletized molding compound and dough molding compound. Reinforced Reaction Injection Molding Some thermosets cure by chemical reaction rather than heat; these resins can be molded by reaction injection molding (Section 13.6.5). In RIM, two reactive ingredients are mixed and immediately injected into a mold cavity where curing and solidification of the chemicals occur rapidly. A closely related process includes reinforcing fibers, typically glass, in the mixture. In this case, the process is called reinforced reaction injection molding (RRIM). Its advantages are similar to those in RIM, with the added benefit of fiber reinforcement. RRIM is used extensively in auto body and truck cab applications for bumpers, fenders, and other body parts. 15.4 FILAMENT WINDING Filament winding is a process in which resin-impregnated continuous fibers are wrapped around a rotating mandrel that has the internal shape of the desired FRP product. The resin is subsequently cured and the mandrel removed. Hollow axisymmetric components (usually circular in cross section) are produced, as well as some irregular shapes. The most common form of the process is depicted in Figure 15.8. A band of fiber rovings is pulled through a resin bath immediately before being wound in a helical pattern onto a cylindrical mandrel. Continuation of the winding pattern finally completes a surface layer of one filament thickness on the mandrel. The operation is repeated to form additional layers, each having a criss-cross pattern with the previous, until the desired part thickness has been obtained. There are several methods by which the fibers can be impregnated with resin: (1) wet winding, in which the filament is pulled through the liquid resin just before winding, as in the figure; (2) prepreg winding (also called dry winding), in which filaments preimpregnated with partially cured resin are wrapped around a heated mandrel; and (3) postimpregnation, in which filaments are wound onto a mandrel and then impregnated with resin by brushing or other technique. Two basic winding patterns are used in filament winding: (a) helical and (b) polar (Figure 15.9). In helical winding, the filament band is applied in a spiral pattern around the mandrel, at a helix angle u. If the band is wrapped with a helix angle approaching 90 , so that the winding advance is one bandwidth per revolution (and the filaments form nearly E1C15 11/11/2009 338 14:42:13 Page 338 Chapter 15/Shaping Processes for Polymer Matrix Composites Pulleys Carriage Drive box Continuous roving Resin bath Rotating mandrel FIGURE 15.8 winding. Filament circular rings around the mandrel), this is referred to as a hoop winding; it is a special case of helical winding. In polar winding, the filament is wrapped around the long axis of the mandrel, as in Figure 15.9(b); after each longitudinal revolution, the mandrel is indexed (partially rotated) by one bandwidth, so that a hollow enclosed shape is gradually created. Hoop and polar patterns can be combined in successive windings of the mandrel to produce adjacent layers with filament directions that are approximately perpendicular; this is called a bi-axial winding [2]. Filament winding machines have motion capabilities similar to those of an engine lathe (Section 22.2.3). The typical machine has a drive motor to rotate the mandrel and a powered feed mechanism to move the carriage. Relative motion between mandrel and carriage must be controlled to accomplish a given winding pattern. In helical winding, the relationship between helix angle and the machine parameters can be expressed as follows: tan u ¼ vc pDN ð15:1Þ where u ¼ helix angle of the windings on the mandrel, as in Figure 15.9(a); vc ¼ speed at which the carriage traverses in the axial direction, m/s (in/sec); D ¼ diameter of the mandrel, m (in); and N ¼ rotational speed, 1/s (rev/sec). Various types of control are available in filament winding machines. Modern equipment uses computer numerical control (CNC, Section 38.3), in which mandrel rotation and carriage speed are independently controlled to permit greater adjustment and flexibility in the relative motions. CNC is especially useful in helical winding of contoured shapes, as in Figure 15.10. As indicated in Eq. (15.1), the ratio vc/DN must remain fixed to maintain a constant helix angle u. Thus, either vc and/or N must be adjusted on-line to compensate for changes in D. The mandrel is the special tooling that determines the geometry of the filamentwound part. For part removal, mandrels must be capable of collapsing after winding and curing. Various designs are possible, including inflatable/deflatable mandrels, collapsible metal mandrels, and mandrels made of soluble salts or plasters. FIGURE 15.9 Two basic winding patterns in filament winding: (a) helical and (b) polar. θ (a) (b) E1C15 11/11/2009 14:42:13 Page 339 Section 15.5/Pultrusion Processes 339 FIGURE 15.10 Filament winding machine. (Courtesy of Cincinnati Milacron.) Applications of filament winding are often classified as aerospace or commercial [10], the engineering requirements being more demanding in the first category. Aerospace applications include rocket-motor cases, missile bodies, radomes, helicopter blades, and airplane tail sections and stabilizers. These components are made of advanced composites and hybrid composites (Section 9.4.1), with epoxy resins being most common and reinforced with fibers of carbon, boron, Kevlar, and glass. Commercial applications include storage tanks, reinforced pipes and tubing, drive shafts, wind-turbine blades, and lightning rods; these are made of conventional FRPs. Polymers include polyester, epoxy, and phenolic resins; glass is the common reinforcing fiber. 15.5 PULTRUSION PROCESSES The basic pultrusion process was developed around 1950 for making fishing rods of glass fiber–reinforced polymer (GFRP). The process is similar to extrusion (hence the similarity in name), but it involves pulling of the workpiece (so the prefix ‘‘pul-’’ is used in place of ‘‘ex-’’). Like extrusion, pultrusion produces continuous, straight sections of constant cross section. A related process, called pulforming, can be used to make parts that are curved and may have variations in cross section throughout their lengths. 15.5.1 PULTRUSION Pultrusion is a process in which continuous fiber rovings are dipped into a resin bath and pulled through a shaping die where the impregnated resin cures. The setup is sketched in Figure15.11,whichshowsthecuredproductbeingcutintolong,straightsections.Thesections are reinforced throughout their length by continuous fibers. Like extrusion, the pieces have a constant cross section, whose profile is determined by the shape of the die opening. The process consists of five steps (identified in the sketch) performed in a continuous sequence [2]: (1) filament feeding, in which the fibers are unreeled from E1C15 11/11/2009 340 14:42:14 Page 340 Chapter 15/Shaping Processes for Polymer Matrix Composites FIGURE 15.11 Pultrusion process (see text for interpretation of sequence numbers). a creel (shelves with skewers that hold filament bobbins); (2) resin impregnation, in which the fibers are dipped in the uncured liquid resin; (3) pre-die forming—the collection of filaments is gradually shaped into the approximate cross section desired; (4) shaping and curing, in which the impregnated fibers are pulled through the heated die whose length is 1 to 1.5 m (3 to 5 ft) and whose inside surfaces are highly polished; and (5) pulling and cutting—pullers are used to draw the cured length through the die, after which it is cut by a cut-off wheel with SiC or diamond grits. Common resins used in pultrusion are unsaturated polyesters, epoxies, and silicones, all thermosetting polymers. There are difficulties in processing with epoxy polymers because of sticking on the die surface. Thermoplastics have also been studied for possible applications [2]. E-glass is by far the most widely used reinforcing material; proportions range from 30% to 70%. Modulus of elasticity and tensile strength increase with reinforcement content. Products made by pultrusion include solid rods, tubing, long and flat sheets, structural sections (such as channels, angled and flanged beams), tool handles for high-voltage work, and third-rail covers for subways. 15.5.2 PULFORMING The pultrusion process is limited to straight sections of constant cross section. There is also a need for long parts with continuous fiber reinforcement that are curved rather than straight and whose cross sections may vary throughout the length. The pulforming process is suited to these less-regular shapes. Pulforming can be defined as pultrusion with additional steps to form the length into a semicircular contour and alter the cross section at one or more locations along the length. A sketch of the equipment is illustrated in Figure 15.12. After exiting the shaping die, the continuous workpiece is fed into a rotating table with negative molds positioned around its periphery. The work is forced into the mold cavities by a die shoe, which squeezes the cross section at various locations and forms the curvature in the length. The diameter of the table determines the radius of the part. As the work leaves the die table, it is cut to length to provide discrete parts. Resins and fibers similar to those for pultrusion are used in pulforming. An important application of the process is production of automobile leaf springs. E1C15 11/11/2009 14:42:14 Page 341 Section 15.6/Other PMC Shaping Processes 341 FIGURE 15.12 Pulforming process (not shown in the sketch is the cut-off of the pulformed part). 15.6 OTHER PMC SHAPING PROCESSES Additional PMC shaping processes worth noting include centrifugal casting, tube rolling, continuous laminating, and cutting. In addition, many of the traditional thermoplastic shaping processes are applicable to (short-fiber) FRPs based on TP polymers; these include blow molding, thermoforming, and extrusion. Centrifugal Casting This process is ideal for cylindrical products such as pipes and tanks. The process is the same as its counterpart in metal casting (Section 11.3.5). Chopped fibers combined with liquid resin are poured into a fast-rotating cylindrical mold. Centrifugal force presses the ingredients against the mold wall, where curing takes place. The resulting inner surfaces are quite smooth. Part shrinkage or use of split molds permits part removal. Tube Rolling FRP tubes can be fabricated from prepreg sheets by a rolling technique [7], shown in Figure 15.13. Such tubes are used in bicycle frames and space trusses. In the process, a precut prepreg sheet is wrapped around a cylindrical mandrel several times to obtain a tube wall of multiple sheet thicknesses. The rolled sheets are then encased in a heat-shrinking sleeve and oven cured. As the sleeve contracts, entrapped gases are squeezed out the ends of the tube. When curing is complete, the mandrel is removed to yield a rolled FRP tube. The operation is simple, and tooling cost is low. There are variations in the process, such as using different wrapping methods or using a steel mold to enclose the rolled prepreg tube for better dimensional control. FIGURE 15.13 Tube rolling, showing (a) one possible means of wrapping FRP prepregs around a mandrel, and (b) the completed tube after curing and removal of mandrel. E1C15 11/11/2009 342 14:42:14 Page 342 Chapter 15/Shaping Processes for Polymer Matrix Composites Continuous Laminating Fiber-reinforced plastic panels, sometimes translucent and/or corrugated, are used in construction. The process to produce them consists of (1) impregnating layers of glass fiber mat or woven fabric by dipping in liquid resin or by passing beneath a doctor blade; (2) gathering between cover films (cellophane, polyester, or other polymer); and (3) compacting between squeeze rolls and curing. Corrugation (4) is added by formed rollers or mold shoes. Cutting Methods FRP laminated composites must be cut in both uncured and cured states. Uncured materials (prepregs, preforms, SMCs, and other starting forms) must be cut to size for lay-up, molding, and so on. Typical cutting tools include knives, scissors, power shears, and steel-rule blanking dies. Also used are nontraditional cutting methods, such as laser beam cutting and water jet cutting (Chapter 26). Cured FRPs are hard, tough, abrasive, and difficult to cut; but cutting is necessary in many FRP shaping processes to trim excess material, cut holes and outlines, and for other purposes. For fiberglass-reinforced plastics, cemented carbide cutting tools and high-speed steel saw blades must be used. For some advanced composites (e.g., boronepoxy), diamond cutting tools obtain best results. Water jet cutting is also used with good success on cured FRPs; this process reduces the dust and noise problems associated with conventional sawing methods. REFERENCES [1] ASM Handbook, Vol. 21: Composites, ASM International, Materials Park, Ohio, 2001. [2] Bader, M. G., Smith, W., Isham, A. B., Rolston, J. A., and Metzner, A. B. Delaware Composites Design Encyclopedia. Vol. 3. Processing and Fabrication Technology. Technomic Publishing Co., Lancaster, Pennsylvania, 1990. [3] Chawla, K. K. Composite Materials: Science and Engineering, 3rd ed., Springer-Verlag, New York, 2008. [4] Charrier, J-M. Polymeric Materials and Processing. Oxford University Press, New York, 1991. [5] Coulter, J. P.‘‘Resin Impregnation During the Manufacture of Composite Materials,’’ PhD Dissertation. University of Delaware, 1988. [6] Engineering Materials Handbook. Vol. 1. Composites. ASM International, Metals Park, Ohio, 1987. [7] Mallick, P. K. Fiber-Reinforced Composites: Materials, Manufacturing, and Design. 2nd ed. Marcel Dekker, New York, 1993. [8] McCrum, N. G., Buckley, C. P., and Bucknall, C. B. Principles of Polymer Engineering. Oxford University Press, Inc., Oxford, U.K., 1988. [9] Morton-Jones, D. H. Polymer Processing. Chapman and Hall, London, 1989. [10] Schwartz, M. M. Composite Materials Handbook. 2nd ed. McGraw-Hill, New York, 1992. [11] Strong, A. B. Fundamentals of Composites Manufacturing: Materials, Methods, and Applications. 2nd ed. Society of Manufacturing Engineers, Dearborn, Michigan, 2007. [12] Wick, C., Benedict, J. T., and Veilleux, R. F. (eds.). Tool and Manufacturing Engineers Handbook. 4th ed. Vol. II. Forming, 1984. [13] Wick, C., and Veilleux, R. F. (eds.). Tool and Manufacturing Engineers Handbook. 4th ed. Vol. III. Materials, Finishing, and Coating, 1985. REVIEW QUESTIONS 15.1. What are the principal polymers used in fiberreinforced polymers? 15.2. What is the difference between a roving and a yarn? 15.3. In the context of fiber reinforcement, what is a mat? 15.4. Why are particles and flakes members of the same basic class of reinforcing material? 15.5. What is sheet molding compound (SMC)? 15.6. Howisaprepregdifferentfromamoldingcompound? 15.7. Why are laminated FRP products made by the spray-up method not as strong as similar products made by hand lay-up? E1C15 11/11/2009 14:42:14 Page 343 Multiple Choice Quiz 15.8. What is the difference between the wet lay-up approach and the prepreg approach in hand lay-up? 15.9. What is an autoclave? 15.10. What are some of the advantages of the closed mold processes for PMCs relative to open mold processes? 15.11. Identify some of the different forms of polymer matrix composite molding compounds. 15.12. What is preform molding? 15.13. Describe reinforced reaction injection molding (RRIM). 15.14. What is filament winding? 15.15. Describe the pultrusion process. 15.16. How does pulforming differ from pultrusion? 343 15.17. With what kinds of products is tube rolling associated? 15.18. How are FRPs cut? 15.19. (Video) According to the video on composites, list the primary purpose of the matrix and the reinforcement in a composite. 15.20. (Video) List the primary methods of fiber reinforced thermoset polymer composite production according to the composite video. 15.21. (Video) What are the advantages and disadvantages of using prepreg material for lay-up of composites according to the composite video? MULTIPLE CHOICE QUIZ There are 14 correct answers in the following multiple choice questions (some questions have multiple answers that are correct). To attain a perfect score on the quiz, all correct answers must be given. Each correct answer is worth 1 point. Each omitted answer or wrong answer reduces the score by 1 point, and each additional answer beyond the correct number of answers reduces the score by 1 point. Percentage score on the quiz is based on the total number of correct answers. 15.1. Which one of the following is the most common polymer type in fiber-reinforced polymer composites: (a) elastomers, (b) thermoplastics, or (c) thermosets? 15.2. Most rubber products are properly classified into which of the following categories (three best answers): (a) elastomer reinforced with carbon black, (b) fiber-reinforced composite, (c) particle-reinforced composite, (d) polymer matrix composite, (e) pure elastomer, and (f) pure polymer? 15.3. Other names for open mold processes include which of the following (two best answers): (a) compression molding, (b) contact lamination, (c) contact molding, (d) filament winding, (e) matched die molding, (f) preform molding, and (g) pultrusion? 15.4. Hand lay-up is classified in which of the following general categories of PMC shaping processes (two best answers): (a) closed mold process, (b) compression molding, (c) contact molding, (d) filament winding, or (e) open mold process? 15.5. A positive mold with a smooth surface will produce a good finish on which surface of the laminated product in the hand lay-up method:(a) inside surface or (b) outside surface? 15.6. A molding operation that uses sheet-molding compound (SMC) is a form of which one of the following: (a) compression molding, (b) contact molding, (c) injection molding, (d) open mold processing, (e) pultrusion, or (f) transfer molding? 15.7. Filament winding involves the use of which one of the following fiber reinforcements: (a) continuous filaments, (b) fabrics, (c) mats, (d) prepregs, (e) short fibers, or (f) woven rovings? 15.8. In filament winding, when the continuous filament is wound around the cylindrical mandrel at a helix angle close to 90 , it is called which of the following (one best answer): (a) bi-axial winding, (b) helical winding, (c) hoop winding, (d) perpendicular winding, (e) polar winding, or (f) radial winding? 15.9. Pultrusion is most similar to which one of the following plastic shaping processes: (a) blow-molding, (b) extrusion, (c) injection molding, or (d) thermoforming? 15.10. Water jet cutting is one of several ways of cutting or trimming uncured or cured FRPs; in the case of cured FRPs, the process is noted for its reduction of dust and noise: (a) true or (b) false? E1C16 11/10/2009 16:37:8 Page 344 Part IV Particulate Processing of Metals and Ceramics 16 POWDER METALLURGY Chapter Contents 16.1 Characterization of Engineering Powders 16.1.1 Geometric Features 16.1.2 Other Features 16.2 Production of Metallic Powders 16.2.1 Atomization 16.2.2 Other Production Methods 16.3 Conventional Pressing and Sintering 16.3.1 Blending and Mixing of the Powders 16.3.2 Compaction 16.3.3 Sintering 16.3.4 Secondary Operations 16.3.5 Heat Treatment and Finishing 16.4 Alternative Pressing and Sintering Techniques 16.4.1 Isostatic Pressing 16.4.2 Powder Injection Molding 16.4.3 Powder Rolling, Extrusion, and Forging 16.4.4 Combined Pressing and Sintering 16.4.5 Liquid Phase Sintering 16.5 Materials and Products for Powder Metallurgy 16.6 Design Considerations in Powder Metallurgy 344 This part of the book is concerned with the processing of metals and ceramics that are in the form of powders—very small particulate solids. In the case of traditional ceramics, the powders are produced by crushing and grinding common materials that are found in nature, such as silicate minerals (clay) and quartz. In the case of metals and the new ceramics, the powders are produced by a variety of industrial processes. We cover the powder-making processes as well as the methods used to shape products out of powders in two chapters: Chapter 16 on powder metallurgy and Chapter 17 on particulate processing of ceramics and cermets. Powder metallurgy (PM) is a metal processing technology in which parts are produced from metallic powders. In the usual PM production sequence, the powders are compressed into the desired shape and then heated to cause bonding of the particles into a hard, rigid mass. Compression, called pressing, is accomplished in a press-type machine using tools designed specifically for the part to be manufactured. The tooling, which typically consists of a die and one or more punches, can be expensive, and PM is therefore most appropriate for medium and high production. The heating treatment, called sintering, is performed at a temperature below the melting point of the metal. The video clip titled Powder Metallurgy illustrates PM E1C16 11/10/2009 16:37:8 Page 345 Chapter 16 /Powder Metallurgy 345 production technology. Considerations that make powder metallurgy an important commercial technology include: å PM parts can be mass produced to net shape or near net shape, eliminating or reducing the need for subsequent processing. å The PM process itself involves very little waste of material; about 97% of the starting powders are converted to product. This compares favorably with casting processes in which sprues, runners, and risers are wasted material in the production cycle. å Owing to the nature of the starting material in PM, parts having a specified level of porosity can be made. This feature lends itself to the production of porous metal parts such as filters and oil-impregnated bearings and gears. å Certain metals that are difficult to fabricate by other methods can be shaped by powder metallurgy. Tungsten is an example; tungsten filaments used in incandescent lamp bulbs are made using PM technology. å Certain metal alloy combinations and cermets can be formed by PM that cannot be produced by other methods. å PM compares favorably with most casting processes in terms of dimensional control of the product. Tolerances of 0.13 mm (0.005 in) are held routinely. å PM production methods can be automated for economical production. VIDEO CLIP Powder Metallurgy. This clip contains two segments: (1) powder metal parts production and (2) PM overview. There are limitations and disadvantages associated with PM processing. These include the following: (1) tooling and equipment costs are high, (2) metallic powders are expensive, and (3) there are difficulties with storing and handling metal powders (such as degradation of the metal over time, and fire hazards with particular metals). Also, (4) there are limitations on part geometry because metal powders do not readily flow laterally in the die during pressing, and allowances must be provided for ejection of the part from the die after pressing. In addition, (5) variations in material density throughout the part may be a problem in PM, especially for complex part geometries. Although parts as large as 22 kg (50 lb) can be produced, most PM components are less than 2.2 kg (5 lb). A collection of typical PM parts is shown in Figure 16.1. The largest tonnage of metals for PM are alloys of iron, steel, and aluminum. Other PM metals include copper, nickel, and refractory metals such as molybdenum and tungsten. Metallic carbides such as tungsten carbide are often included within the scope of powder metallurgy; however, because these materials are ceramics, we defer their consideration until the next chapter. The development of the modern field of powder metallurgy dates back to the 1800s (Historical Note 16.1). The scope of the modern technology includes not only parts production, but also preparation of the starting powders. Success in powder metallurgy depends to a large degree on the characteristics of the starting powders; we discuss this topic in Section 16.1. Later sections describe powder production, pressing, and sintering. There is a close correlation between PM technology and aspects of ceramics processing (Chapter 17). In ceramics (except glass), the starting material is also powder, so the methods for characterizing the powders are closely related to those in PM. Several of the shape-forming methods are similar, also. E1C16 11/10/2009 346 16:37:8 Page 346 Chapter 16/Powder Metallurgy FIGURE 16.1 A collection of powder metallurgy parts. (Courtesy of Dorst America, Inc.) Historical Note 16.1 P Powder metallurgy owders of metals such as gold and copper, as well as some of the metallic oxides, have been used for decorative purposes since ancient times. The uses included decorations on pottery, bases for paints, and in cosmetics. It is believed that the Egyptians used PM to make tools as far back as 3000 BCE. The modern field of powder metallurgy dates to the early nineteenth century, when there was a strong interest in the metal platinum. Around 1815, Englishman William Wollaston developed a technique for preparing platinum powders, compacting them under high pressure, and baking (sintering) them at red heat. The Wollaston process marks the beginning of powder metallurgy as it is practiced today. U.S. patents were issued in 1870 to S. Gwynn that relate to PM self-lubricating bearings. He used a mixture of 99% powdered tin and 1% petroleum, mixing, heating, and finally subjecting the mixture to extreme pressures to form it into the desired shape inside a mold cavity. By the early 1900s, the incandescent lamp had become an important commercial product. A variety of filament materials had been tried, including carbon, zirconium, vanadium, and osmium; but it was concluded that tungsten was the best filament material. The problem was that tungsten was difficult to process because of its high melting point and unique properties. In 1908, William Coolidge developed a procedure that made production of tungsten incandescent lamp filaments feasible. In his process, fine powders of tungsten oxide (WO3) were reduced to metallic powders, pressed into compacts, presintered, hot-forged into rounds, sintered, and finally drawn into filament wire. The Coolidge process is still used today to make filaments for incandescent light bulbs. In the 1920s, cemented carbide tools (WC–Co) were being fabricated by PM techniques (Historical Note 7.2). Self-lubricating bearings were produced in large quantities starting in the 1930s. Powder metal gears and other components were mass produced in the 1960s and 1970s, especially in the automotive industry; and in the 1980s, PM parts for aircraft turbine engines were developed. E1C16 11/10/2009 16:37:8 Page 347 Section 16.1/Characterization of Engineering Powders 16.1 347 CHARACTERIZATION OF ENGINEERING POWDERS A powder can be defined as a finely divided particulate solid. In this section we characterize metallic powders. However, most of the discussion applies to ceramic powders as well. 16.1.1 GEOMETRIC FEATURES The geometry of the individual powders can be defined by the following attributes: (1) particle size and distribution, (2) particle shape and internal structure, and (3) surface area. Particle Size and Distribution Particle size refers to the dimensions of the individual powders. If the particle shape is spherical, a single dimension is adequate. For other shapes, two or more dimensions are needed. There are various methods available to obtain particle size data. The most common method uses screens of different mesh sizes. The term mesh count is used to refer to the number of openings per linear inch of screen. Higher mesh count indicates smaller particle size. A mesh count of 200 means there are 200 openings per linear inch. Because the mesh is square, the count is the same in both directions, and the total number of openings per square inch is 2002 ¼ 40,000. Particles are sorted by passing them through a series of screens of progressively smaller mesh size. The powders are placed on a screen of a certain mesh count and vibrated so that particles small enough to fit through the openings pass through to the next screen below. The second screen empties into a third, and so forth, so that the particles are sorted according to size. A certain powder size might be called size 230 through 200, indicating that the powders have passed through the 200 mesh, but not 230. To make the specification easier, we simply say that the particle size is 200. The procedure of separating the powders by size is called classification. The openings in the screen are less than the reciprocal of the mesh count because of the thickness of the wire in the screen, as illustrated in Figure 16.2. Assuming that the limiting dimension of the particle is equal to the screen opening, we have 1 PS ¼ ð16:1Þ  tw MC where PS ¼ particle size, in; MC ¼ mesh count, openings per linear inch; and tw ¼ wire thickness of screen mesh, in. The figure shows how smaller particles would pass through the openings, whereas larger powders would not. Variations occur in the powder sizes sorted by screening owing to differences in particle shapes, the range of sizes between mesh count steps, and variations in screen openings within a given mesh count. Also, the screening method has a FIGURE 16.2 Screen mesh for sorting particle sizes. E1C16 11/10/2009 348 16:37:8 Page 348 Chapter 16/Powder Metallurgy FIGURE 16.3 Several of the possible (ideal) particle shapes in powder metallurgy. practical upper limit of MC ¼ 400 (approximately), because of both the difficulty in making such fine screens and agglomeration of the small powders. Other methods to measure particle size include microscopy and X-ray techniques. Typical particle sizes used in conventional powder metallurgy (press and sinter) range between 25 and 300 mm (0.001 and 0.012 in).1 The high end of this range corresponds to a mesh count of around 65. The low end of the range is too small to be measured by the mesh count method. Particle Shape and Internal Structure Metal powder shapes can be cataloged into various types, several of which are illustrated in Figure 16.3. There will be a variation in the particle shapes in a collection of powders, just as the particle size will vary. A simple and useful measure of shape is the aspect ratio—the ratio of maximum dimension to minimum dimension for a given particle. The aspect ratio for a spherical particle is 1.0, but for an acicular grain the ratio might be 2 to 4. Microscopic techniques are required to determine shape characteristics. Any volume of loose powders will contain pores between the particles. These are called open pores because they are external to the individual particles. Open pores are spaces into which a fluid such as water, oil, or a molten metal can penetrate. In addition, there are closed pores—internal voids in the structure of an individual particle. The existence of these internal pores is usually minimal, and their effect when they do exist is minor, but they can influence density measurements, as we shall see later. Surface Area Assuming that the particle shape is a perfect sphere, its area A and volume V are given by A ¼ pD2 ð16:2Þ 3 pD ð16:3Þ 6 where D ¼ diameter of the spherical particle, mm (in). The area-to-volume ratio A/V for a sphere is then given by V¼ A 6 ¼ V D ð16:4Þ In general, the area-to-volume ratio can be expressed for any particle shape—spherical or nonspherical—as follows: A Ks ¼ D V or Ks ¼ AD V ð16:5Þ 1 These values are provided by Prof. Wojciech Misiolek, my colleague in Lehigh’s Department of Materials Science and Engineering. Powder metallurgy is one of his research areas. E1C16 11/10/2009 16:37:8 Page 349 Section 16.1/Characterization of Engineering Powders 349 where Ks ¼ shape factor; D in the general case ¼ the diameter of a sphere of equivalent volume as the nonspherical particle, mm (in). Thus, Ks ¼ 6.0 for a sphere. For particle shapes other than spherical, Ks > 6. We can infer the following from these equations. Smaller particle size and higher shape factor (Ks) mean higher surface area for the same total weight of metal powders. This means greater area for surface oxidation to occur. Small powder size also leads to more agglomeration of the particles, which is a problem in automatic feeding of the powders. The reason for using smaller particle sizes is that they provide more uniform shrinkage and better mechanical properties in the final PM product. 16.1.2 OTHER FEATURES Other features of engineering powders include interparticle friction, flow characteristics, packing, density, porosity, chemistry, and surface films. Interparticle Friction and Flow Characteristics Friction between particles affects the ability of a powder to flow readily and pack tightly. A common measure of interparticle friction is the angle of repose, which is the angle formed by a pile of powders as they are poured from a narrow funnel, as in Figure 16.4. Larger angles indicate greater friction between particles. Smaller particle sizes generally show greater friction and steeper angles. Spherical shapes result in the lowest interpartical friction; as shape deviates more from spherical, friction between particles tends to increase. Flow characteristics are important in die filling and pressing. Automatic die filling depends on easy and consistent flow of the powders. In pressing, resistance to flow increases density variations in the compacted part; these density gradients are generally undesirable. A common measure of flow is the time required for a certain amount of powder (by weight) to flow through a standard-sized funnel. Smaller flow times indicate easier flow and lower interparticle friction. To reduce interparticle friction and facilitate flow during pressing, lubricants are often added to the powders in small amounts. Packing, Density, and Porosity Packing characteristics depend on two density measures. First, true density is the density of the true volume of the material. This is the density when the powders are melted into a solid mass, values of which are given in Table 4.1. Second, bulk density is the density of the powders in the loose state after pouring, which includes the effect of pores between particles. Because of the pores, bulk density is less than true density. The packing factor is the bulk density divided by the true density. Typical values for loose powders range between 0.5 and 0.7. The packing factor depends on particle shape and FIGURE 16.4 Interparticle friction as indicated by the angle of repose of a pile of powders poured from a narrow funnel. Larger angles indicate greater interparticle friction. E1C16 11/10/2009 350 16:37:8 Page 350 Chapter 16/Powder Metallurgy the distribution of particle sizes. If powders of various sizes are present, the smaller powders will fit into the interstices of the larger ones that would otherwise be taken up by air, thus resulting in a higher packing factor. Packing can also be increased by vibrating the powders, causing them to settle more tightly. Finally, we should note that external pressure, as applied during compaction, greatly increases packing of powders through rearrangement and deformation of the particles. Porosity represents an alternative way of considering the packing characteristics of a powder. Porosity is defined as the ratio of the volume of the pores (empty spaces) in the powder to the bulk volume. In principle Porosity þ Packing factor ¼ 1:0 ð16:6Þ The issue is complicated by the possible existence of closed pores in some of the particles. If these internal pore volumes are included in the above porosity, then the equation is exact. Chemistry and Surface Films Characterization of the powder would not be complete without an identification of its chemistry. Metallic powders are classified as either elemental, consisting of a pure metal, or pre-alloyed, wherein each particle is an alloy. We discuss these classes and the metals commonly used in PM more thoroughly in Section 16.5.1. Surface films are a problem in powder metallurgy because of the large area per unit weight of metal when dealing with powders. The possible films include oxides, silica, adsorbed organic materials, and moisture [6]. Generally, these films must be removed before shape processing. 16.2 PRODUCTION OF METALLIC POWDERS In general, producers of metallic powders are not the same companies as those that make PM parts. The powder producers are the suppliers; the plants that manufacture components out of powder metals are the customers. It is therefore appropriate to separate the discussion of powder production (this section) from the processes used to make PM products (later sections). Virtually any metal can be made into powder form. There are three principal methods by which metallic powders are commercially produced, each of which involves energy input to increase the surface area of the metal. The methods are (1) atomization, (2) chemical, and (3) electrolytic [13]. In addition, mechanical methods are occasionally used to reduce powder sizes; however, these methods are much more commonly associated with ceramic powder production and we treat them in the next chapter. 16.2.1 ATOMIZATION This method involves the conversion of molten metal into a spray of droplets that solidify into powders. It is the most versatile and popular method for producing metal powders today, applicable to almost all metals, alloys as well as pure metals. There are multiple ways of creating the molten metal spray, several of which are illustrated in Figure 16.5. Two of the methods shown are based on gas atomization, in which a high velocity gas stream (air or inert gas) is utilized to atomize the liquid metal. In Figure 16.5(a), the gas flows through an expansion nozzle, siphoning molten metal from the melt below and spraying it into a container. The droplets solidify into powder form. In a closely related method shown in Figure 16.5(b), molten metal flows by gravity through a nozzle and is immediately atomized E1C16 11/10/2009 16:37:8 Page 351 Section 16.2/Production of Metallic Powders 351 FIGURE 16.5 Several atomization methods for producing metallic powders: (a) and (b) two gas atomization methods; (c) water atomization; and (d) centrifugal atomization by the rotating disk method. by air jets. The resulting metal powders, which tend to be spherical, are collected in a chamber below. The approach shown in Figure 16.5(c) is similar to (b), except that a high-velocity water stream is used instead of air. This is known as water atomization and is the most common of the atomization methods, particularly suited to metals that melt below 1600 C (2900 F). Cooling is more rapid, and the resulting powder shape is irregular rather than spherical. The disadvantage of using water is oxidation on the particle surface. A recent innovation involves the use of synthetic oil rather than water to reduce oxidation. In both air and water atomization processes, particle size is controlled largely by the velocity of the fluid stream; particle size is inversely related to velocity. Several methods are based on centrifugal atomization. In one approach, the rotating disk method shown in Figure 16.5(d), the liquid metal stream pours onto a rapidly rotating disk that sprays the metal in all directions to produce powders. 16.2.2 OTHER PRODUCTION METHODS Other metal powder production methods include various chemical reduction processes, precipitation methods, and electrolysis. Chemical reduction includes a variety of chemical reactions by which metallic compounds are reduced to elemental metal powders. A common process involves liberation of metals from their oxides by use of reducing agents such as hydrogen or carbon monoxide. The reducing agent is made to combine with the oxygen in the compound to free the E1C16 11/10/2009 352 16:37:9 Page 352 Chapter 16/Powder Metallurgy FIGURE 16.6 Iron powders produced by decomposition of iron pentacarbonyl; particle sizes range from about 0.25 to 3.0 mm (10–125 m-in). (Photo courtesy of GAF Chemicals Corporation, Advanced Materials Division.) metallic element. This approach is used to produce powders of iron, tungsten, and copper. Another chemical process for iron powders involves the decomposition of iron pentacarbonyl (Fe(Co)5) to produce spherical particles of high purity. Powders produced by this method are illustrated in the photomicrograph of Figure 16.6. Other chemical processes include precipitation of metallic elements from salts dissolved in water. Powders of copper, nickel, and cobalt can be produced by this approach. In electrolysis, an electrolytic cell is set up in which the source of the desired metal is the anode. The anode is slowly dissolved under an applied voltage, transported through the electrolyte, and deposited on the cathode. The deposit is removed, washed, and dried to yield a metallic powder of very high purity. The technique is used for producing powders of beryllium, copper, iron, silver, tantalum, and titanium. 16.3 CONVENTIONAL PRESSING AND SINTERING After the metallic powders have been produced, the conventional PM sequence consists of three steps: (1) blending and mixing of the powders; (2) compaction, in which the powders are pressed into the desired part shape; and (3) sintering, which involves heating to a temperature below the melting point to cause solid-state bonding of the particles and strengthening of the part. The three steps, sometimes referred to as primary operations in PM, are portrayed in Figure 16.7. In addition, secondary operations are sometimes performed to improve dimensional accuracy, increase density, and for other reasons. 16.3.1 BLENDING AND MIXING OF THE POWDERS To achieve successful results in compaction and sintering, the metallic powders must be thoroughly homogenized beforehand. The terms blending and mixing are both used in this context. Blending refers to when powders of the same chemical composition but possibly different particle sizes are intermingled. Different particle sizes are often blended to reduce porosity. Mixing refers to powders of different chemistries being combined. An advantage of PM technology is the opportunity to mix various metals into alloys that would be difficult or impossible to produce by other means. The distinction between blending and mixing is not always precise in industrial practice. Blending and mixing are accomplished by mechanical means. Four alternatives are illustrated in Figure 16.8: (a) rotation in a drum; (b) rotation in a double-cone container; E1C16 11/10/2009 16:37:9 Page 353 Section 16.3/Conventional Pressing and Sintering 353 FIGURE 16.7 The conventional powder metallurgy production sequence: (1) blending, (2) compacting, and (3) sintering; (a) shows the condition of the particles, whereas (b) shows the operation and/ or workpart during the sequence. (c) agitation in a screw mixer; and (d) stirring in a blade mixer. There is more science to these devices than one would suspect. Best results seem to occur when the container is between 20% and 40% full. The containers are usually designed with internal baffles or other ways of preventing free-fall during blending of powders of different sizes, because variations in settling rates between sizes result in segregation—just the opposite of what is wanted in blending. Vibration of the powder is undesirable, because it also causes segregation. Other ingredients are usually added to the metallic powders during the blending and/ or mixing step. These additives include (1) lubricants, such as stearates of zinc and aluminum, in small amounts to reduce friction between particles and at the die wall during compaction; (2) binders, which are required in some cases to achieve adequate strength in the pressed but unsintered parts; and (3) deflocculants, which inhibit agglomeration of powders for better flow characteristics during subsequent processing. FIGURE 16.8 Several blending and mixing devices: (a) rotating drum, (b) rotating double-cone, (c) screw mixer, and (d) blade mixer. E1C16 11/10/2009 354 16:37:9 Page 354 Chapter 16/Powder Metallurgy FIGURE 16.9 Pressing, the conventional method of compacting metal powders in PM: (1) filling the die cavity with powder, done by automatic feed in production, (2) initial, and (3) final positions of upper and lower punches during compaction, and (4) ejection of part. 16.3.2 COMPACTION In compaction, high pressure is applied to the powders to form them into the required shape. The conventional compaction method is pressing, in which opposing punches squeeze the powders contained in a die. The steps in the pressing cycle are shown in Figure 16.9. The workpart after pressing is called a green compact, the word green meaning not yet fully processed. As a result of pressing, the density of the part, called the green density, is much greater than the starting bulk density. The green strength of the part when pressed is adequate for handling but far less than that achieved after sintering. The applied pressure in compaction results initially in repacking of the powders into a more efficient arrangement, eliminating ‘‘bridges’’ formed during filling, reducing pore space, and increasing the number of contacting points between particles. As pressure increases, the particles are plastically deformed, causing interparticle contact area to increase and additional particles to make contact. This is accompanied by a further reduction in pore volume. The progression is illustrated in three views in Figure 16.10 FIGURE 16.10 (a) Effect of applied pressure during compaction: (1) initial loose powders after filling, (2) repacking, and (3) deformation of particles; and (b) density of the powders as a function of pressure. The sequence here corresponds to steps 1, 2, and 3 in Figure 16.9. E1C16 11/10/2009 16:37:9 Page 355 Section 16.3/Conventional Pressing and Sintering 355 FIGURE 16.11 A 450-kN (50-ton) hydraulic press for compaction of powder metallurgy components. (Photo courtesy of Dorst America, Inc.) for starting particles of spherical shape. Also shown is the associated density represented by the three views as a function of applied pressure. Presses used in conventional PM compaction are mechanical, hydraulic, or a combination of the two. A 450 kN (50 ton) hydraulic unit is shown in Figure 16.11. Because of differences in part complexity and associated pressing requirements, presses can be distinguished as (1) pressing from one direction, referred to as single-action presses; or (2) pressing from two directions, any of several types including opposed ram, double-action, and multiple action. Current available press technology can provide up to 10 separate action controls to produce parts of significant geometric complexity. We examine part complexity and other design issues in Section 16.6. The capacity of a press for PM production is generally given in tons or kN or MN. The required force for pressing depends on the projected area of the PM part (area in the horizontal plane for a vertical press) multiplied by the pressure needed to compact the given metal powders. Reducing this to equation form F ¼ Ap pc ð16:7Þ where F ¼ required force, N (lb); Ap ¼ projected area of the part, mm2 (in2); and pc ¼ compaction pressure required for the given powder material, MPa (lb/in2). Compaction pressures typically range from 70 MPa (10,000 lb/in2) for aluminum powders to 700 MPa (100,000 lb/in2) for iron and steel powders. 16.3.3 SINTERING After pressing, the green compact lacks strength and hardness; it is easily crumbled under low stresses. Sintering is a heat treatment operation performed on the compact to bond E1C16 11/10/2009 356 16:37:9 Page 356 Chapter 16/Powder Metallurgy FIGURE 16.12 Sintering on a microscopic scale: (1) particle bonding is initiated at contact points; (2) contact points grow into ‘‘necks’’; (3) the pores between particles are reduced in size; and (4) grain boundaries develop between particles in place of the necked regions. its metallic particles, thereby increasing strength and hardness. The treatment is usually carried out at temperatures between 0.7 and 0.9 of the metal’s melting point (absolute scale). The terms solid-state sintering or solid-phase sintering are sometimes used for this conventional sintering because the metal remains unmelted at these treatment temperatures. It is generally agreed among researchers that the primary driving force for sintering is reduction of surface energy [6], [16]. The green compact consists of many distinct particles, each with its own individual surface, and so the total surface area contained in the compact is very high. Under the influence of heat, the surface area is reduced through the formation and growth of bonds between the particles, with associated reduction in surface energy. The finer the initial powder size, the higher the total surface area, and the greater the driving force behind the process. The series of sketches in Figure 16.12 shows on a microscopic scale the changes that occur during sintering of metallic powders. Sintering involves mass transport to create the necks and transform them into grain boundaries. The principal mechanism by which this occurs is diffusion; other possible mechanisms include plastic flow. Shrinkage occurs during sintering as a result of pore size reduction. This depends to a large extent on the density of the green compact, which depends on the pressure during compaction. Shrinkage is generally predictable when processing conditions are closely controlled. Because PM applications usually involve medium-to-high production, most sintering furnaces are designed with mechanized flow-through capability for the workparts. The heat treatment consists of three steps, accomplished in three chambers in these continuous furnaces: (1) preheat, in which lubricants and binders are burned off; (2) sinter; and (3) cool down. The treatment is illustrated in Figure 16.13. Typical sintering temperatures and times are given for selected metals in Table 16.1. In modern sintering practice, the atmosphere in the furnace is controlled. The purposes of a controlled atmosphere include (1) protection from oxidation, (2) providing a reducing atmosphere to remove existing oxides, (3) providing a carburizing atmosphere, and (4) assisting in removing lubricants and binders used in pressing. Common sintering furnace atmospheres are inert gas, nitrogen-based, dissociated ammonia, hydrogen, and natural gas [6]. Vacuum atmospheres are used for certain metals, such as stainless steel and tungsten. 16.3.4 SECONDARY OPERATIONS PM secondary operations include densification, sizing, impregnation, infiltration, heat treatment, and finishing. E1C16 11/10/2009 16:37:10 Page 357 Section 16.3/Conventional Pressing and Sintering 357 FIGURE 16.13 (a) Typical heat treatment cycle in sintering; and (b) schematic cross section of a continuous sintering furnace. Densification and Sizing A number of secondary operations are performed to increase density, improve accuracy, or accomplish additional shaping of the sintered part. Repressing is a pressing operation in which the part is squeezed in a closed die to increase density and improve physical properties. Sizing is the pressing of a sintered part to improve dimensional accuracy. Coining is a pressworking operation on a sintered part to press details into its surface. Some PM parts require machining after sintering. Machining is rarely done to size the part, but rather to create geometric features that cannot be achieved by pressing, such as internal and external threads, side holes, and other details. Impregnation and Infiltration Porosity is a unique and inherent characteristic of powder metallurgy technology. It can be exploited to create special products by filling the available pore space with oils, polymers, or metals that have lower melting temperatures than the base powder metal. Impregnation is the term used when oil or other fluid is permeated into the pores of a sintered PM part. The most common products of this process are oil-impregnated bearings, TABLE 16.1 Typical sintering temperatures and times for selected powder metals. Sintering Temperatures Metal  Brass Bronze Copper Iron Stainless steel Tungsten 850 820 850 1100 1200 2300 C Compiled from [10] and [17].  F Typical Time 1600 1500 1600 2000 2200 4200 25 min 15 min 25 min 30 min 45 min 480 min E1C16 11/10/2009 358 16:37:10 Page 358 Chapter 16/Powder Metallurgy gears, and similar machinery components. Self-lubricating bearings, usually made of bronze or iron with 10% to 30% oil by volume, are widely used in the automotive industry. The treatment is accomplished by immersing the sintered parts in a bath of hot oil. An alternative application of impregnation involves PM parts that must be made pressure tight or impervious to fluids. In this case, the parts are impregnated with various types of polymer resins that seep into the pore spaces in liquid form and then solidify. In some cases, resin impregnation is used to facilitate subsequent processing, for example, to permit the use of processing solutions (such as plating chemicals) that would otherwise soak into the pores and degrade the product, or to improve machinability of the PM workpart. Infiltration is an operation in which the pores of the PM part are filled with a molten metal. The melting point of the filler metal must be below that of the PM part. The process involves heating the filler metal in contact with the sintered component so that capillary action draws the filler into the pores. The resulting structure is relatively nonporous, and the infiltrated part has a more uniform density, as well as improved toughness and strength. An application of the process is copper infiltration of iron PM parts. 16.3.5 HEAT TREATMENT AND FINISHING Powder metal components can be heat treated (Chapter 27) and finished (electroplated or painted, Chapter 28) by most of the same processes used on parts produced by casting and other metalworking processes. Special care must be exercised in heat treatment because of porosity; for example, salt baths are not used for heating PM parts. Plating and coating operations are applied to sintered parts for appearance purposes and corrosion resistance. Again, precautions must be taken to avoid entrapment of chemical solutions in the pores; impregnation and infiltration are frequently used for this purpose. Common platings for PM parts include copper, nickel, chromium, zinc, and cadmium. 16.4 ALTERNATIVE PRESSING AND SINTERING TECHNIQUES The conventional press and sinter sequence is the most widely used shaping technology in powder metallurgy. Additional methods for processing PM parts are discussed in this section. 16.4.1 ISOSTATIC PRESSING A feature of conventional pressing is that pressure is applied uniaxially. This imposes limitations on part geometry, because metallic powders do not readily flow in directions perpendicular to the applied pressure. Uniaxial pressing also leads to density variations in the compact after pressing. In isostatic pressing, pressure is applied from all directions against the powders that are contained in a flexible mold; hydraulic pressure is used to achieve compaction. Isostatic pressing takes two alternative forms: (1) cold isostatic pressing and (2) hot isostatic pressing. Cold isostatic pressing (CIP) involves compaction performed at room temperature. The mold, made of rubber or other elastomer material, is oversized to compensate for shrinkage. Water or oil is used to provide the hydrostatic pressure against the mold inside the chamber. Figure 16.14 illustrates the processing sequence in cold isostatic pressing. Advantages of CIP include more uniform density, less expensive tooling, and greater applicability to shorter production runs. Good dimensional accuracy is difficult to E1C16 11/10/2009 16:37:10 Page 359 Section 16.4/Alternative Pressing and Sintering Techniques 359 FIGURE 16.14 Cold isostatic pressing: (1) powders are placed in the flexible mold; (2) hydrostatic pressure is applied against the mold to compact the powders; and (3) pressure is reduced and the part is removed. achieve in isostatic pressing because of the flexible mold. Consequently, subsequent finish shaping operations are often required to obtain the required dimensions, either before or after sintering. Hot isostatic pressing (HIP) is carried out at high temperatures and pressures, using a gas such as argon or helium as the compression medium. The mold in which the powders are contained is made of sheet metal to withstand the high temperatures. HIP accomplishes pressing and sintering in one step. Despite this apparent advantage, it is a relatively expensive process and its applications seem to be concentrated in the aerospace industry. PM parts made by HIP are characterized by high density (porosity near zero), thorough interparticle bonding, and good mechanical strength. 16.4.2 POWDER INJECTION MOLDING Injection molding is closely associated with the plastics industry (Section 13.6). The same basic process can be applied to form parts of metal or ceramic powders, the difference being that the starting polymer contains a high content of particulate matter, typically from 50% to 85% by volume. When used in powder metallurgy, the term metal injection molding (MIM) is used. The more general process is powder injection molding (PIM), which includes both metal and ceramic powders. The steps in MIM proceed as follows [7]: (1) Metallic powders are mixed with an appropriate binder. (2) Granular pellets are formed from the mixture. (3) The pellets are heated to molding temperature, injected into a mold cavity, and the part is cooled and removed from the mold. (4) The part is processed to remove the binder using any of several thermal or solvent techniques. (5) The part is sintered. (6) Secondary operations are performed as appropriate. The binder in powder injection molding acts as a carrier for the particles. Its functions are to provide proper flow characteristics during molding and hold the powders in the molded shape until sintering. The five basic types of binders in PIM are: (1) thermosetting polymers, such as phenolics; (2) thermoplastic polymers, such as polyethylene; (3) water; (4) gels; and (5) inorganic materials [7]. Polymers are the most frequently used. Powder injection molding is suited to part geometries similar to those in plastic injection molding. It is not cost competitive for simple axisymmetric parts, because the E1C16 11/10/2009 360 16:37:10 Page 360 Chapter 16/Powder Metallurgy FIGURE 16.15 Powder rolling: (1) powders are fed through compaction rolls to form a green strip; (2) sintering; (3) cold rolling; and (4) resintering. conventional press-and-sinter process is quite adequate for these cases. PIM seems most economical for small, complex parts of high value. Dimensional accuracy is limited by the shrinkage that accompanies densification during sintering. 16.4.3 POWDER ROLLING, EXTRUSION, AND FORGING Rolling, extrusion, and forging are familiar bulk metal forming processes (Chapter 19). We describe them here in the context of powder metallurgy. Powder Rolling Powders can be compressed in a rolling mill operation to form metal strip stock. The process is usually set up to run continuously or semicontinuously, as shown in Figure 16.15. The metallic powders are compacted between rolls into a green strip that is fed directly into a sintering furnace. It is then cold rolled and resintered. Powder Extrusion Extrusion is one of the basic manufacturing processes (Section 1.3.1). In PM extrusion, the starting powders can be in different forms. In the most popular method, powders are placed in a vacuum-tight sheet metal can, heated, and extruded with the container. In another variation, billets are preformed by a conventional press and sinter process, and then the billet is hot extruded. These methods achieve a high degree of densification in the PM product. Powder Forging Forging is an important metal forming process (Section 1.3.1). In powder forging, the starting workpart is a powder metallurgy part preformed to proper size by pressing and sintering. Advantages of this approach are: (1) densification of the PM part, (2) lower tooling costs and fewer forging ‘‘hits’’ (and therefore higher production rate) because the starting workpart is preformed, and (3) reduced material waste. 16.4.4 COMBINED PRESSING AND SINTERING Hot isostatic pressing (Section 16.4.1) accomplishes compaction and sintering in one step. Other techniques that combine the two steps are hot pressing and spark sintering. Hot Pressing The setup in uniaxial hot pressing is very similar to conventional PM pressing, except that heat is applied during compaction. The resulting product is generally E1C16 11/10/2009 16:37:10 Page 361 Section 16.5/Materials and Products for Powder Metallurgy 361 dense, strong, hard, and dimensionally accurate. Despite these advantages, the process presents certain technical problems that limit its adoption. Principal among these are (1) selecting a suitable mold material that can withstand the high sintering temperatures; (2) longer production cycle required to accomplish sintering; and (3) heating and maintaining atmospheric control in the process [2]. Hot pressing has found some application in the production of sintered carbide products using graphite molds. Spark Sintering An alternative approach that combines pressing and sintering but overcomes some of the problems in hot pressing is spark sintering. The process consists of two basic steps [2], [17]: (1) powder or a green compacted preform is placed in a die; and (2) upper and lower punches, which also serve as electrodes, compress the part and simultaneously apply a high-energy electrical current that burns off surface contaminants and sinters the powders, forming a dense, solid part in about 15 seconds. The process has been applied to a variety of metals. 16.4.5 LIQUID PHASE SINTERING Conventional sintering (Section 16.3.3) is solid-state sintering; the metal is sintered at a temperature below its melting point. In systems involving a mixture of two powder metals, in which there is a difference in melting temperature between the metals, an alternative type of sintering is used, called liquid phase sintering. In this process, the two powders are initially mixed, and then heated to a temperature that is high enough to melt the lowermelting-point metal but not the other. The melted metal thoroughly wets the solid particles, creating a dense structure with strong bonding between the metals upon solidification. Depending on the metals involved, prolonged heating may result in alloying of the metals by gradually dissolving the solid particles into the liquid melt and/or diffusion of the liquid metal into the solid. In either case, the resulting product is fully densified (no pores) and strong. Examples of systems that involve liquid phase sintering include Fe–Cu, W–Cu, and Cu–Co [6]. 16.5 MATERIALS AND PRODUCTS FOR POWDER METALLURGY The raw materials for PM processing are more expensive than for other metalworking because of the additional energy required to reduce the metal to powder form. Accordingly, PM is competitive only in a certain range of applications. In this section we identify the materials and products that seem most suited to powder metallurgy. Powder Metallurgy Materials From a chemistry standpoint, metal powders can be classified as either elemental or pre-alloyed. Elemental powders consist of a pure metal and are used in applications in which high purity is important. For example, pure iron might be used where its magnetic properties are important. The most common elemental powders are those of iron, aluminum, and copper. Elemental powders are also mixed with other metal powders to produce special alloys that are difficult to formulate using conventional processing methods. Tool steels are an example; PM permits blending of ingredients that is difficult or impossible by traditional alloying techniques. Using mixtures of elemental powders to form an alloy provides a processing benefit, even where special alloys are not involved. Because the powders are pure metals, they are not as strong as pre-alloyed metals. Therefore, they deform more readily during pressing, so that density and green strength are higher than with pre-alloyed compacts. E1C16 11/10/2009 362 16:37:11 Page 362 Chapter 16/Powder Metallurgy In pre-alloyed powders, each particle is an alloy composed of the desired chemical composition. Pre-alloyed powders are used for alloys that cannot be formulated by mixing elemental powders; stainless steel is an important example. The most common pre-alloyed powders are certain copper alloys, stainless steel, and high-speed steel. The commonly used elemental and pre-alloyed powdered metals, in approximate order of tonnage usage, are: (1) iron, by far the most widely used PM metal, frequently mixed with graphite to make steel parts, (2) aluminum, (3) copper and its alloys, (4) nickel, (5) stainless steel, (6) high-speed steel, and (7) other PM materials such as tungsten, molybdenum, titanium, tin, and precious metals. Powder Metallurgy Products A substantial advantage offered by PM technology is that parts can be made to near net shape or net shape; they require little or no additional shaping after PM processing. Some of the components commonly manufactured by powder metallurgy are gears, bearings, sprockets, fasteners, electrical contacts, cutting tools, and various machinery parts. When produced in large quantities, metal gears and bearings are particularly well suited to PM for two reasons: (1) the geometry is defined principally in two dimensions, so the part has a top surface of a certain shape, but there are no features along the sides; and (2) there is a need for porosity in the material to serve as a reservoir for lubricant. More complex parts with true three-dimensional geometries are also feasible in powder metallurgy, by adding secondary operations such as machining to complete the shape of the pressed and sintered part, and by observing certain design guidelines such as those outlined in the following section. 16.6 DESIGN CONSIDERATIONS IN POWDER METALLURGY Use of PM techniques is generally suited to a certain class of production situations and part designs. In this section we attempt to define the characteristics of this class of applications for which powder metallurgy is most appropriate. We first present a classification system for PM parts, and then offer some guidelines on component design. The Metal Powder Industries Federation (MPIF) defines four classes of powder metallurgy part designs, by level of difficulty in conventional pressing. The system is useful because it indicates some of the limitations on shape that can be achieved with conventional PM processing. The four part classes are illustrated in Figure 16.16. FIGURE 16.16 Four classes of PM parts—side view shown; cross section is circular: (a) Class I—simple thin shapes that can be pressed from one direction; (b) Class II—simple but thicker shapes that require pressing from two directions; (c) Class III—two levels of thickness, pressed from two directions; and (d) Class IV—multiple levels of thickness, pressed from two directions, with separate controls for each level to achieve proper densification throughout the compact. E1C16 11/10/2009 16:37:11 Page 363 Section 16.6/Design Considerations in Powder Metallurgy 363 The MPIF classification system provides some guidance concerning part geometries that are suited to conventional PM pressing techniques. Additional advice is offered in the following design guidelines, compiled from [3], [13], and [17]. å Economics of PM processing usually require large part quantities to justify the cost of equipment and special tooling required. Minimum quantities of 10,000 units are suggested [17], although exceptions exist. å Powder metallurgy is unique in its capability to fabricate parts with a controlled level of porosity. Porosities up to 50% are possible. å PM can be used to make parts out of unusual metals and alloys—materials that would be difficult if not impossible to fabricate by other means. å The geometry of the part must permit ejection from the die after pressing; this generally means that the part must have vertical or near-vertical sides, although steps in the part are permissible as suggested by the MPIF classification system (Figure 16.16). Design features such as undercuts and holes on the part sides, as shown in Figure 16.17, must be avoided. Vertical undercuts and holes, as in Figure 16.18, are permissible because they do not interfere with ejection. Vertical holes can be of crosssectional shapes other than round (e.g., squares, keyways) without significant increases in tooling or processing difficulty. å Screw threads cannot be fabricated by PM pressing; if required, they must be machined into the PM component after sintering. å Chamfers and corner radii are possible by PM pressing, as shown in Figure 16.19. Problems are encountered in punch rigidity when angles are too acute. FIGURE 16.17 Part features to be avoided in PM: (a) side holes and (b) side undercuts. Part ejection is impossible. FIGURE 16.18 Permissible part features in PM: (a) vertical hole, blind and through, (b) vertical stepped hole, and (c) undercut in vertical direction. These features allow part ejection. E1C16 11/10/2009 364 16:37:11 Page 364 Chapter 16/Powder Metallurgy FIGURE 16.19 Chamfers and corner radii are accomplished but certain rules should be observed: (a) avoid acute chamfer angles; (b) larger angles are preferred for punch rigidity; (c) small inside radius is desirable; (d) full outside corner radius is difficult because punch is fragile at corner’s edge; (e) outside corner problem can be solved by combining radius and chamfer. FIGURE 16.20 Minimum recommended wall thickness (a) between holes or (b) between a hole and an outside wall should be 1.5 mm (0.060 in). å Wall thickness should be a minimum of 1.5 mm (0.060 in) between holes or a hole and the outside part wall, as indicated in Figure 16.20. Minimum recommended hole diameter is 1.5 mm (0.060 in). REFERENCES [1] ASM Handbook, Vol. 7: Powder Metal Technologies and Applications, ASM International, Materials Park, Ohio, 1998. [2] Amstead, B. H., Ostwald, P. F., and Begeman, M. L. Manufacturing Processes. 8th ed. John Wiley & Sons, New York, 1987. [3] Bralla, J. G. (ed.). Design for Manufacturability Handbook. 2nd ed. McGraw-Hill, New York, 1998. [4] Bulger, M.‘‘Metal Injection Molding,’’ Advanced Materials & Processes. March 2005, pp. 39–40. [5] Dixon, R. H. T., and Clayton, A. Powder Metallurgy for Engineers. The Machinery Publishing Co. Ltd., Brighton, U.K., 1971. [6] German, R. M. Powder Metallurgy Science. 2nd ed. Metal Powder Industries Federation, Princeton, New Jersey, 1994. E1C16 11/10/2009 16:37:11 Page 365 Multiple Choice Quiz [7] German, R. M. Powder Injection Molding. Metal Powder Industries Federation, Princeton, New Jersey, 1990. [8] German, R. M. A-Z of Powder Metallurgy, Elsevier Science, Amsterdam, Netherlands, 2006. [9] Johnson, P. K.‘‘P/M Industry Trends in 2005,’’ Advanced Materials & Processes, March 2005, pp. 25–28. [10] Metals Handbook. 9th ed. Vol. 7. Powder Metallurgy. American Society for Metals, Metals Park, Ohio, 1984. [11] Pease, L. F. ‘‘A Quick Tour of Powder Metallurgy,’’ Advanced Materials & Processes, March 2005, pp. 36–38. [12] Pease, L. F., and West, W. G. Fundamentals of Powder Metallurgy, Metal Powder Industries Federation, Princeton, New Jersey, 2002. 365 [13] Powder Metallurgy Design Handbook. Metal Powder Industries Federation, Princeton, New Jersey, 1989. [14] Schey, J. A. Introduction to Manufacturing Processes. 3rd ed. McGraw-Hill, New York, 1999. [15] Smythe, J. ‘‘Superalloy Powders: An Amazing History,’’ Advanced Materials & Processes, November 2008, pp. 52–55. [16] Waldron, M. B., and Daniell, B. L. Sintering. Heyden, London, 1978. [17] Wick, C., Benedict, J. T., and Veilleux, R. F. (eds.). Tool and Manufacturing Engineers Handbook. 4th ed. Vol. II, Forming. Society of Manufacturing Engineers, Dearborn, Michigan, 1984. REVIEW QUESTIONS 16.1. Name some of the reasons for the commercial importance of powder metallurgy technology. 16.2. What are some of the disadvantages of PM methods? 16.3. In the screening of powders for sizing, what is meant by the term mesh count? 16.4. What is the difference between open pores and closed pores in metallic powders? 16.5. What is meant by the term aspect ratio for a metallic particle? 16.6. How would one measure the angle of repose for a given amount of metallic powder? 16.7. Define bulk density and true density for metallic powders. 16.8. What are the principal methods used to produce metallic powders? 16.9. What are the three basic steps in the conventional powder metallurgy shaping process? 16.10. What is the technical difference between mixing and blending in powder metallurgy? 16.11. What are some of the ingredients usually added to the metallic powders during blending and/or mixing? 16.12. What is meant by the term green compact? 16.13. Describe what happens to the individual particles during compaction. 16.14. What are the three steps in the sintering cycle in PM? 16.15. What are some of the reasons why a controlled atmosphere furnace is desirable in sintering? 16.16. What are the advantages of infiltration in PM? 16.17. What is the difference between powder injection molding and metal injection molding? 16.18. How is isostatic pressing distinguished from conventional pressing and sintering in PM? 16.19. Describe liquid phase sintering. 16.20. What are the two basic classes of metal powders as far as chemistry is concerned? 16.21. Why is PM technology so well suited to the production of gears and bearings? 16.22. (Video) List the most common methods for forming the pressed parts in powder metallurgy according to the powder metallurgy video. 16.23. (Video) List the types of environments that can be present during the sintering process according to the powder metallurgy video. MULTIPLE CHOICE QUIZ There are 19 correct answers in the following multiple choice questions (some questions have multiple answers that are correct). To attain a perfect score on the quiz, all correct answers must be given. Each correct answer is worth 1 point. Each omitted answer or wrong answer reduces the score by 1 point, and each additional answer beyond the correct number of answers reduces the score by 1 point. Percentage score on the quiz is based on the total number of correct answers. E1C16 11/10/2009 366 16:37:12 Page 366 Chapter 16/Powder Metallurgy 16.1. The particle size that can pass through a screen is obtained by taking the reciprocal of the mesh count of the screen: (a) true or (b) false? 16.2. For a given weight of metallic powders, the total surface area of the powders is increased by which of the following (two best answers): (a) larger particle size, (b) smaller particle size, (c) higher shape factor, and (d) smaller shape factor? 16.3. As particle size increases, interparticle friction (a) decreases, (b) increases, or (c) remains the same? 16.4. Which of the following powder shapes would tend to have the lowest interparticle friction: (a) acicular, (b) cubic, (c) flakey, (d) spherical, and (e) rounded? 16.5. Which of the following statements is correct in the context of metallic powders (three correct answers): (a) porosity þ packing factor ¼ 1.0, (b) packing factor ¼ 1/porosity, (c) packing factor ¼ 1.0 – porosity, (d) packing factor ¼ – porosity, (e) packing factor ¼ bulk density/true density? 16.6. Which of the following most closely typifies the sintering temperatures in PM? (a) 0.5 Tm, (b) 0.8 Tm, (c) Tm, where Tm ¼ melting temperature of the metal? 16.7. Repressing refers to a pressworking operation used to compress a sintered part in a closed die to achieve closer sizing and better surface finish: (a) true or (b) false? 16.8. Impregnation refers to which of the following (two best answers): (a) filling the pores of the PM part with a molten metal, (b) putting polymers into the pores of a PM part, (c) soaking oil by capillary action into the pores of a PM part, and (d) something that should not happen in a factory? 16.9. In cold isostatic pressing, the mold is most typically made of which one of the following: (a) rubber, (b) sheetmetal, (c) textile, (d) thermosetting polymer, or (e) tool steel? 16.10. Which of the following processes combines pressing and sintering of the metal powders (three best answers): (a) hot isostatic pressing, (b) hot pressing, (c) metal injection molding, (d) pressing and sintering, and (e) spark sintering? 16.11. Which of the following design features would be difficult or impossible to achieve by conventional pressing and sintering (three best answers): (a) outside rounded corners, (b) side holes, (c) threaded holes, (d) vertical stepped holes, and (e) vertical wall thickness of 1/8 inch (3 mm)? PROBLEMS Characterization of Engineering Powders 16.1. Ascreenwith325meshcounthaswireswithadiameter of 0.001377 in. Determine (a) the maximum particle size that will pass through the wire mesh and (b) the proportion of open space in the screen. 16.2. A screen with 10 mesh count has wires with a diameter of 0.0213 in. Determine (a) the maximum particle size that will pass through the wire mesh and (b) the proportion of open space in the screen. 16.3. What is the aspect ratio of a cubic particle shape? 16.4. Determine the shape factor for metallic particles of the following ideal shapes: (a) sphere, (b) cubic, (c) cylindrical with length-to-diameter ratio of 1:1, (d) cylindrical with length-to-diameter ratio of 2:1, and (e) a disk-shaped flake whose thickness-to-diameter ratio is 1:10. 16.5. A pile of iron powder weighs 2 lb. The particles are spherical in shape and all have the same diameter of 0.002 in. (a) Determine the total surface area of all the particles in the pile. (b) If the packing factor ¼ 0.6, determine the volume taken by the pile. Note: the density of iron ¼ 0.284 lb/in3. 16.6. Solve Problem 16.5, except that the diameter of the particles is 0.004 in. Assume the same packing factor. 16.7. Suppose in Problem 16.5 that the average particle diameter ¼ 0.002 in; however, the sizes vary, forming a statistical distribution as follows: 25% of the particles by weight are 0.001 in, 50% are 0.002 in, and 25% are 0.003 in. Given this distribution, what is the total surface area of all the particles in the pile? 16.8. A solid cube of copper with each side ¼ 1.0 ft is converted into metallic powders of spherical shape by gas atomization. What is the percentage increase in total surface area if the diameter of each particle is 0.004 in (assume that all particles are the same size)? 16.9. A solid cube of aluminum with each side ¼ 1.0 m is converted into metallic powders of spherical shape by gas atomization. How much total surface area is added by the process if the diameter of each particle is 100 microns (assume that all particles are the same size)? E1C16 11/10/2009 16:37:12 Page 367 Problems 16.10. Given a large volume of metallic powders, all of which are perfectly spherical and having the same 367 exact diameter, what is the maximum possible packing factor that the powders can take? Compaction and Design Considerations 16.11. In a certain pressing operation, the metallic powder fed into the open die has a packing factor of 0.5. The pressing operation reduces the powders to two thirds of their starting volume. In the subsequent sintering operation, shrinkage amounts to 10% on a volume basis. Given that these are the only factors that affect the structure of the finished part, determine its final porosity. 16.12. A bearing of simple geometry is to be pressed out of bronze powders, using a compacting pressure of 207 MPa. The outside diameter ¼ 44 mm, the inside diameter ¼ 22 mm, and the length of the bearing ¼ 25 mm. What is the required press tonnage to perform this operation? 16.13. The part shown in Figure P16.13 is to be pressed of iron powders using a compaction pressure of 75,000 lb/in2. Dimensions are inches. Determine (a) the most appropriate pressing direction, (b) the required press tonnage to perform this operation, and (c) the final weight of the part if the porosity is 10%. Assume shrinkage during sintering can be neglected. FIGURE P16.13 Part for Problem 16.13 (dimensions in inches). 16.14. For each of the four part drawings in Figure P16.14, indicate which PM class the parts belong to, whether the part must be pressed from one or two directions, and how many levels of press control will be required? Dimensions are mm. + + + + 38.0 62.5 12.5 38.0 (a) FIGURE P16.14 56.0 56.0 22.0 12.5 12.5 45.0 0.875 12.5 12.5 12.5 100 11.0 47.5 40.5 (b) (c) Parts for Problem 16.14 (dimensions in mm). 40.5 (d) E1C17 11/09/2009 11:8:28 17 Page 368 PROCESSING OF CERAMICS AND CERMETS Chapter Contents 17.1 Processing of Traditional Ceramics 17.1.1 Preparation of the Raw Material 17.1.2 Shaping Processes 17.1.3 Drying 17.1.4 Firing (Sintering) 17.2 Processing of New Ceramics 17.2.1 Preparation of Starting Materials 17.2.2 Shaping 17.2.3 Sintering 17.2.4 Finishing 17.3 Processing of Cermets 17.3.1 Cemented Carbides 17.3.2 Other Cermets and Ceramic Matrix Composites 17.4 Product Design Considerations Ceramic materials divide into three categories (Chapter 7): (1) traditional ceramics, (2) new ceramics, and (3) glasses. The processing of glass involves solidification primarily and is covered in Chapter 12. In the present chapter we consider the particulate processing methods used for traditional and new ceramics. We also consider the processing of metal matrix composites and ceramic matrix composites. Traditional ceramics are made from minerals occurring in nature. They include pottery, porcelain, bricks, and cement. New ceramics are made from synthetically produced raw materials and cover a wide spectrum of products such as cutting tools, artificial bones, nuclear fuels, and substrates for electronic circuits. The starting material for all of these items is powder. In the case of the traditional ceramics, the powders are usually mixed with water to temporarily bind the particles together and achieve the proper consistency for shaping. For new ceramics, other substances are used as binders during shaping. After shaping, the green parts are sintered. This is often called firing in ceramics, but the function is the same as in powder metallurgy: to effect a solid-state reaction that bonds the material into a hard solid mass. The processing methods discussed in this chapter are commercially and technologically important because virtually all ceramic products are formed by these methods (except, of course, glass products). The manufacturing sequence is similar for traditional and new ceramics because the form of the starting material is the same: powder. However, the processing methods for the two categories are sufficiently different that we discuss them separately. 17.1 PROCESSING OF TRADITIONAL CERAMICS In this section we describe the production technology used to make traditional ceramic products such as pottery, stoneware and other dinnerware, bricks, tile, and ceramic 368 E1C17 11/09/2009 11:8:35 Page 369 Section 17.1/Processing of Traditional Ceramics 369 FIGURE 17.1 Usual steps in traditional ceramics processing: (1) preparation of raw materials, (2) shaping, (3) drying, and (4) firing. Part (a) shows the workpart during the sequence, whereas (b) shows the condition of the powders. refractories. Bonded grinding wheels are also produced by the same basic methods. What these products have in common is that their raw materials consist primarily of silicate ceramics—clays. The processing sequence for most of the traditional ceramics consists of the steps depicted in Figure 17.1. 17.1.1 PREPARATION OF THE RAW MATERIAL The shaping processes for traditional ceramics require that the starting material be in the form of a plastic paste. This paste is made of fine ceramic powders mixed with water, and its consistency determines the ease of forming the material and the quality of the final product. The raw ceramic material usually occurs in nature as rocky lumps, and reduction to powder is the purpose of the preparation step in ceramics processing. Techniques for reducing particle size in ceramics processing involve mechanical energy in various forms, such as impact, compression, and attrition. The term comminution is used for these techniques, which are most effective on brittle materials, including cement, metallic ores, and brittle metals. Two general categories of comminution operations are distinguished: crushing and grinding. Crushing refers to the reduction of large lumps from the mine to smaller sizes for subsequent further reduction. Several stages may be required (e.g., primary crushing, secondary crushing), the reduction ratio in each stage being in the range 3 to 6. Crushing of minerals is accomplished by compression against rigid surfaces or by impact against surfaces in a rigid constrained motion [1]. Figure 17.2 shows several types of equipment used to perform crushing: (a) jaw crushers, in which a large jaw toggles back and forth to crush lumps against a hard, rigid surface; (b) gyratory crushers, which use a gyrating cone to compress lumps against a rigid surface; (c) roll crushers, in which the ceramic lumps are squeezed between rotating rolls; and (d) hammer mills, which use rotating hammers impacting the material to break up the lumps. Grinding, in the context here, refers to the operation of reducing the small pieces produced by crushing into a fine powder. Grinding is accomplished by abrasion and impact of the crushed mineral by the free motion of unconnected hard media such as balls, pebbles, E1C17 11/09/2009 370 11:8:35 Page 370 Chapter 17/Processing of Ceramics and Cermets FIGURE 17.2 Crushing operations: (a) jaw crusher, (b) gyratory crusher, (c) roll crusher, and (d) hammer mill. or rods [1]. Examples of grinding include (a) ball mill, (b) roller mill, and (c) impact grinding, illustrated in Figure 17.3. In a ball mill, hard spheres mixed with the stock to be comminuted are tumbled inside a rotating cylindrical container. The rotation causes the balls and stock to be carried up the container wall, and then pulled back down by gravity to accomplish a grinding action by a combination of impact and attrition. These operations are often carried out with water added to the mixture, so that the ceramic is in the form of a slurry. In a roller mill, stock is compressed against a flat horizontal grinding table by rollers riding over the table surface. Although not clearly shown in the sketch, the pressure of the grinding rollers against the table is regulated by mechanical springs or hydraulic-pneumatic means. In impact grinding, which seems to be less frequently used, particles of stock are thrown against a hard flat surface, either in a high velocity air stream or a high-speed slurry. The impact fractures the pieces into smaller particles. The plastic paste required for shaping consists of ceramic powders and water. Clay is usually the main ingredient in the paste because it has ideal forming characteristics. The more E1C17 11/09/2009 11:8:35 Page 371 Section 17.1/Processing of Traditional Ceramics FIGURE 17.3 grinding. 371 Mechanical methods of producing ceramic powders: (a) ball mill, (b) roller mill, and (c) impact water there is in the mixture, the more plastic and easily formed is the clay paste. However, when the formed part is later dried and fired, shrinkage occurs that can lead to cracking in the product.To addressthisproblem,otherceramicrawmaterialsthatdo notshrink on dryingand firing are usually added to the paste, often in significant amounts. Also, other components can be included to serve special functions. Thus, the ingredients of the ceramic paste can be divided into the following three categories [3]: (1) clay, which provides the consistency and plasticity required for shaping; (2) nonplastic raw materials, such as alumina and silica, which do not shrink in drying and firing but unfortunately reduce plasticity in the mixture during forming; and (3) other ingredients, such as fluxes that melt (vitrify) during firing and promote sintering of the ceramic material, and wetting agents that improve mixing of ingredients. These ingredients must be thoroughly mixed, either wet or dry. The ball mill often serves this purpose in addition to its grinding function. Also, the proper amounts of powder and water in the paste must be attained, so water must be added or removed, depending on the prior condition of the paste and its desired final consistency. 17.1.2 SHAPING PROCESSES The optimum proportions of powder and water depend on the shaping process used. Some shaping processes require high fluidity; others act on a composition that contains very low water content. At about 50% water by volume, the mixture is a slurry that flows like a liquid. As the water content is reduced, increased pressure is required on the paste to produce a similar flow. Thus, the shaping processes can be divided according to the consistency of the mixture: (1) slip casting, in which the mixture is a slurry with 25% to 40% water; (2) plastic-forming methods that shape the clay in a plastic condition at 15% to 25% water; (3) semi-dry pressing, in which the clay is moist (10% to 15% water) but has low plasticity; and (4) dry pressing, in which the clay is basically dry, containing less than 5% water. Dry clay has no plasticity. The four categories are represented in the chart of Figure 17.4, which compares the categories with the condition of the clay used as starting material. Each category includes several different shaping processes. Slip Casting In slip casting, a suspension of ceramic powders in water, called a slip, is poured into a porous plaster of paris (CaSO4–2H2O) mold so that water from the mix is E1C17 11/09/2009 372 11:8:36 Page 372 Chapter 17/Processing of Ceramics and Cermets FIGURE 17.4 Four categories of shaping processes used for traditional ceramics, compared with water content and pressure required to form the clay. gradually absorbed into the plaster to form a firm layer of clay at the mold surface. The composition of the slip is typically 25% to 40% water, the remainder being clay often mixed with other ingredients. It must be sufficiently fluid to flow into the crevices of the mold cavity, yet lower water content is desirable for faster production rates. Slip casting has two principal variations: drain casting and solid casting. In drain casting, which is the traditional process, the mold is inverted to drain excess slip after the semi-solid layer has been formed, thus leaving a hollow part in the mold; the mold is then opened and the part removed. The sequence, which is very similar to slush casting of metals, is illustrated in Figure 17.5. It is used to make tea pots, vases, art objects, and other hollow-ware products. In solid casting, used to produce solid products, adequate time is allowed for the entire body to become firm. The mold must be periodically resupplied with additional slip to account for shrinkage because of absorbed water. Plastic Forming Thiscategoryincludesavariety ofmethods,both manual and mechanized. They all require the starting mixture to have a plastic consistency, which is generally achieved with 15% to 25% water. Manual methods generally make use of clay at the upper end of the range because it provides a material that is more easily formed; however, this is accompanied by greater shrinkage in drying. Mechanized methods generally employ a mixture with lower water content so that the starting clay is stiffer. Although manual forming methods date back thousands of years, they are still used today by skilled artisans, either in production or for artworks. Hand modeling involves the creation of the ceramic product by manipulating the mass of plastic clay FIGURE 17.5 Sequence of steps in drain casting, a form of slip casting: (1) slip is poured into mold cavity; (2) water is absorbed into plaster mold to form a firm layer; (3) excess slip is poured out; and (4) part is removed from mold and trimmed. E1C17 11/09/2009 11:8:36 Page 373 Section 17.1/Processing of Traditional Ceramics 373 FIGURE 17.6 Sequence in jiggering: (1) wet clay slug is placed on a convex mold; (2) batting; and (3) a jigger tool imparts the final product shape. Symbols v and F indicate motion (v ¼ velocity) and applied force, respectively. into the desired geometry. In addition to art pieces, patterns for plaster molds in slip casting are often made this way. Hand molding is a similar method, only a mold or form is used to define portions of the geometry. Hand throwing on a potter’s wheel is another refinement of the handicraft methods. The potter’s wheel is a round table that rotates on a vertical spindle, powered either by motor or foot-operated treadle. Ceramic products of circular cross section can be formed on the rotating table by throwing and shaping the clay, sometimes using a mold to provide the internal shape. Strictly speaking, use of a motor-driven potter’s wheel is a mechanized method. However, most mechanized clay-forming methods are characterized by much less manual participation than the hand-throwing method described above. These more mechanized methods include jiggering, plastic pressing, and extrusion. Jiggering is an extension of the potter’s wheel methods, in which hand throwing is replaced by mechanized techniques. It is used to produce large numbers of identical items such as houseware plates and bowls. Although there are variations in the tools and methods used, reflecting different levels of automation and refinements to the basic process, a typical sequence is as follows, depicted in Figure 17.6: (1) a wet clay slug is placed on a convex mold; (2) a forming tool is pressed into the slug to provide the initial rough shape—the operation is called batting and the workpiece thus created is called a bat; and (3) a heated jigger tool is used to impart the final contoured shape to the product by pressing the profile into the surface during rotation of the workpart. The reason for heating the tool is to produce steam from the wet clay that prevents sticking. Closely related to jiggering is jolleying, in which the basic mold shape is concave rather than convex [8]. In both of these processes, a rolling tool is sometimes used in place of the nonrotating jigger (or jolley) tool; this rolls the clay into shape, avoiding the need to first bat the slug. Plastic pressing is a forming process in which a plastic clay slug is pressed between upper and lower molds, contained in metal rings. The molds are made of a porous material such as gypsum, so that when a vacuum is drawn on the backs of the mold halves, moisture is removed from the clay. The mold sections are then opened, using positive air pressure to prevent sticking of the part in the mold. Plastic pressing achieves a higher production rate than jiggering and is not limited to radially symmetric parts. Extrusion is used in ceramics processing to produce long sections of uniform cross section, which are then cut to required piece length. The extrusion equipment utilizes a screw-type action to assist in mixing the clay and pushing the plastic material through the die opening. This production sequence is widely used to make hollow bricks, shaped tiles, drain pipes, tubes, and insulators. It is also used to make the starting clay slugs for other ceramics processing methods such as jiggering and plastic pressing. Semi-dry Pressing In semi-dry pressing, the proportion of water in the starting clay is typically 10% to 15%. This results in low plasticity, precluding the use of plastic forming methods that require very plastic clay. Semi-dry pressing uses high pressure to overcome E1C17 11/09/2009 374 11:8:36 Page 374 Chapter 17/Processing of Ceramics and Cermets FIGURE 17.7 Semi-dry pressing: (1) depositing moist powder into die cavity, (2) pressing, and (3) opening the die sections and ejection. Symbols v and F indicate motion (v ¼ velocity) and applied force, respectively. the material’s low plasticity and force it to flow into a die cavity, as depicted in Figure 17.7. Flash is often formed from excess clay being squeezed between the die sections. Dry Pressing The main distinction between semi-dry and dry pressing is the moisture content of the starting mix. The moisture content of the starting clay in dry pressing is typically below 5%. Binders are usually added to the dry powder mix to provide sufficient strength in the pressed part for subsequent handling. Lubricants are also added to prevent die sticking during pressing and ejection. Because dry clay has no plasticity and is very abrasive, there are differences in die design and operating procedures, compared with semidry pressing. The dies must be made of hardened tool steel or cemented tungsten carbide to reduce wear. Because dry clay will not flow during pressing, the geometry of the part must be relatively simple, and the amount and distribution of starting powder in the die cavity must be right. No flash is formed in dry pressing, and no drying shrinkage occurs, so drying time is eliminated and good accuracy can be achieved in the dimensions of the final product. The process sequence in dry pressing is similar to semi-dry pressing. Typical products include bathroom tile, electrical insulators, and refractory brick. 17.1.3 DRYING Water plays an important role in most of the traditional ceramics shaping processes. Thereafter, it serves no purpose and must be removed from the body of the clay piece before firing. Shrinkage is a problem during this step in the processing sequence because water contributes volume to the piece, and when it is removed, the volume is reduced. The effect can be seen in Figure 17.8. As water is initially added to dry clay, it simply replaces the air in the pores between ceramic grains, and there is no volumetric change. Increasing the water content above a certain point causes the grains to become separated and the volume to grow, resulting in wet clay that has plasticity and formability. As more water is added, the mixture eventually becomes a liquid suspension of clay particles in water. The reverse of this process occurs in drying. As water is removed from the wet clay, the volume of the piece shrinks. The drying process occurs in two stages, as depicted in Figure 17.9. In the first stage, the rate of drying is rapid and constant, as water is evaporated E1C17 11/09/2009 11:8:36 Page 375 Section 17.1/Processing of Traditional Ceramics 375 FIGURE 17.8 Volume of clay as a function of water content. Relationship shown here is typical; it varies for different clay compositions. from the surface of the clay into the surrounding air and water from the interior migrates by capillary action toward the surface to replace it. It is during this stage that shrinkage occurs, with the associated risk of warping and cracking owing to variations in drying in different sections of the piece. In the second stage of drying, the moisture content has been reduced to where the ceramic grains are in contact, and little or no further shrinkage occurs. The drying process slows, and this is seen in the decreasing rate in the plot. In production, drying is usually accomplished in drying chambers in which temperature and humidity are controlled to achieve the proper drying schedule. Care must be taken so that water is not removed too rapidly, lest large moisture gradients be set up in the piece, making it more prone to crack. Heating is usually by a combination of convection and radiation, using infrared sources. Typical drying times range between a quarter of an hour for thin sections to several days for very thick sections. 17.1.4 FIRING (SINTERING) After shaping but before firing, the ceramic piece is said to be green (the same term as in powder metallurgy), meaning not fully processed or treated. The green piece lacks hardness and strength; it must be fired to fix the part shape and achieve hardness and strength in the finished ware. Firing is the heat treatment process that sinters the ceramic material; it is performed in a furnace called a kiln. In sintering, bonds are developed between the ceramic grains, and this is accompanied by densification and reduction of porosity. Therefore, shrinkage occurs in the polycrystalline material in addition to the shrinkage that has already occurred in drying. Sintering in ceramics is basically the same FIGURE 17.9 Typical drying rate curve and associated volume reduction (drying shrinkage) for a ceramic body in drying. Drying rate in the second stage of drying is depicted here as a straight line (constant rate decrease as a function of water content); the function is variously shown as concave or convex in the literature [3], [8]. E1C17 11/09/2009 376 11:8:36 Page 376 Chapter 17/Processing of Ceramics and Cermets mechanism as in powder metallurgy. In the firing of traditional ceramics, certain chemical reactions between the components in the mixture may also take place, and a glassy phase also forms among the crystals that acts as a binder. Both of these phenomena depend on the chemical composition of the ceramic material and the firing temperatures used. Unglazed ceramic ware is fired only once; glazed products are fired twice. Glazing refers to the application of a ceramic surface coating to make the piece more impervious to water and to enhance its appearance (Section 7.2.2). The usual processing sequence with glazed ware is (1) fire the ware once before glazing to harden the body of the piece, (2) apply the glaze, and (3) fire the piece a second time to harden the glaze. 17.2 PROCESSING OF NEW CERAMICS Most of the traditional ceramics are based on clay, which possesses a unique capacity to be plastic when mixed with water but hard when dried and fired. Clay consists of various formulations of hydrous aluminum silicate, usually mixed with other ceramic materials, to form a rather complex chemistry. New ceramics (Section 7.3) are based on simpler chemical compounds, such as oxides, carbides, and nitrides. These materials do not possess the plasticity and formabilityoftraditionalclaywhenmixed withwater.Accordingly,otheringredientsmust be combined with the ceramic powders to achieve plasticity and other desirable properties during forming, so that conventional shaping methods can be used. The new ceramics are generallydesignedforapplicationsthatrequirehigherstrength,hardness,andotherproperties not found in the traditional ceramic materials. These requirements have motivated the introduction of several new processingtechniques not previouslyused for traditional ceramics. The manufacturing sequence for the new ceramics can be summarized in the following steps: (1) preparation of starting materials, (2) shaping, (3) sintering, and (4) finishing. Although the sequence is nearly the same as for the traditional ceramics, the details are often quite different, as we shall see in the following. 17.2.1 PREPARATION OF STARTING MATERIALS Because the strength specified for these materials is usually much greater than for traditional ceramics, the starting powders must be more homogeneous in size and composition, and particle size must be smaller (strength of the resulting ceramic product is inversely related to grain size). All of this means that greater control of the starting powders is required. Powder preparation includes mechanical and chemical methods. The mechanical methods consist of the same ball mill grinding operations used for traditional ceramics. The trouble with these methods is that the ceramic particles become contaminated from the materials used in the balls and walls of the mill. This compromises the purity of the ceramic powders and results in microscopic flaws that reduce the strength of the final product. Two chemical methods are used to achieve greater homogeneity in the powders of new ceramics: freeze drying and precipitation from solution. In freeze drying, salts of the appropriate starting chemistry are dissolved in water and the solution is sprayed to form small droplets, which are rapidly frozen. The water is then removed from the droplets in a vacuum chamber, and the resulting freeze-dried salt is decomposed by heating to form the ceramic powders. Freeze drying is not applicable to all ceramics, because in some cases a suitable water-soluble salt cannot be identified as the starting material. Precipitation from solution is another preparation method used for new ceramics. In the typical process, the desired ceramic compound is dissolved from the starting mineral, thus permitting impurities to be filtered out. An intermediate compound is then precipitated from solution, which is converted into the desired compound by heating. An E1C17 11/09/2009 11:8:37 Page 377 Section 17.2/Processing of New Ceramics 377 example of the precipitation method is the Bayer process for producing high purity alumina (also used in the production of aluminum). In this process, aluminum oxide is dissolved from the mineral bauxite so that iron compounds and other impurities can be removed. Then, aluminum hydroxide (Al(OH)3) is precipitated from solution and reduced to Al2O3 by heating. Further preparation of the powders includes classification by size and mixing before shaping. Very fine powders are required for new ceramics applications, and so the grains must be separated and classified according to size. Thorough mixing of the particles, especially when different ceramic powders are combined, is required to avoid segregation. Various additives are often combined with the starting powders, usually in small amounts. The additives include (1) plasticizers to improve plasticity and workability; (2) binders to bond the ceramic particles into a solid mass in the final product, (3) wetting agents for better mixing; (4) deflocculants, which help to prevent clumping and premature bonding of the powders; and (5) lubricants, to reduce friction between ceramic grains during forming and to reduce sticking during mold release. 17.2.2 SHAPING Many of the shaping processes for new ceramics are borrowed from powder metallurgy (PM) and traditional ceramics. The press and sinter methods discussed in Section 16.3 have been adapted to the new ceramic materials. And some of the traditional ceramics-forming techniques (Section 17.1.2) are used to shape the new ceramics, including slip casting, extrusion, and dry pressing. The following processes are not normally associated with the forming of traditional ceramics, although several are associated with PM. Hot Pressing Hot pressing is similar to dry pressing (Section 17.1.2), except that the process is carried out at elevated temperatures, so that sintering of the product is accomplished simultaneously with pressing. This eliminates the need for a separate firing step in the sequence. Higher densities and finer grain size are obtained, but die life is reduced by the hot abrasive particles against the die surfaces. Isostatic Pressing Isostatic pressing of ceramics is the same process used in powder metallurgy (Section 16.4.1). It uses hydrostatic pressure to compact the ceramic powders from all directions, thus avoiding the problem of nonuniform density in the final product that is often observed in the traditional uniaxial pressing method. Doctor-Blade Process This process is used for making thin sheets of ceramic. One common application of the sheets is in the electronics industry as a substrate material for integrated circuits. The process is diagrammed in Figure 17.10. A ceramic slurry is introduced onto a moving carrier film such as cellophane. Thickness of the ceramic on the carrier is determined by a wiper, called a doctor-blade. As the slurry moves down the line, it is dried FIGURE 17.10 The doctor-blade process, used to fabricate thin ceramic sheets. Symbol v indicates motion (v ¼ velocity). E1C17 11/09/2009 378 11:8:37 Page 378 Chapter 17/Processing of Ceramics and Cermets into a flexible green ceramic tape. At the end of the line, a take-up spool reels in the tape for later processing. In its green condition, the tape can be cut or otherwise shaped before firing. Powder Injection Molding Powder injection molding (PIM) is the same as the PM process (Section 16.4.2), except that the powders are ceramic rather than metallic. Ceramic particles are mixed with a thermoplastic polymer that acts as a carrier and provides the proper flow characteristics at molding temperatures. The mix is then heated and injected into a mold cavity. Upon cooling, which hardens the polymer, the mold is opened and the part is removed. Because the temperatures needed to plasticize the carrier are much lower than those required for sintering the ceramic, the piece is green after molding. Before sintering, the plastic binder must be removed. This is called debinding, which is usually accomplished by a combination of thermal and solvent treatments. Applications of ceramic PIM are currently inhibited by difficulties in debinding and sintering. Burning off the polymer is relatively slow, and its removal significantly weakens the green strength of the molded part. Warping and cracking often occur during sintering. Further, ceramic products made by powder injection molding are especially vulnerable to microstructural flaws that limit their strength. 17.2.3 SINTERING Because the plasticity needed to shape the new ceramics is not normally based on a water mixture, the drying step so commonly required to remove water from the traditional green ceramics can be omitted in the processing of most new ceramic products. The sintering step, however, is still very much required to obtain maximum possible strength and hardness. The functions of sintering are the same as before, to: (1) bond individual grains into a solid mass, (2) increase density, and (3) reduce or eliminate porosity. Temperatures around 80% to 90% of the melting temperature of the material are commonly used in sintering ceramics. Sintering mechanisms differ somewhat between the new ceramics, which are based predominantly on a single chemical compound (e.g., Al2O3), and the clay-based ceramics, which usually consist of several compounds having different melting points. In the case of the new ceramics, the sintering mechanism is mass diffusion across the contacting particle surfaces, probably accompanied by some plastic flow. This mechanism causes the centers of the particles to move closer together, resulting in densification of the final material. In the sintering of traditional ceramics, this mechanism is complicated by the melting of some constituents and the formation of a glassy phase that acts as a binder between the grains. 17.2.4 FINISHING Parts made of new ceramics sometimes require finishing. In general, these operations have one or more of the following purposes, to: (1) increase dimensional accuracy, (2) improve surface finish, and (3) make minor changes in part geometry. Finishing operations usually involve grinding and other abrasive processes (Chapter 25). Diamond abrasives must be used to cut the hardened ceramic materials. 17.3 PROCESSING OF CERMETS Many metal matrix composites (MMCs) and ceramic matrix composites (CMCs) are processed by particulate processing methods. The most prominent examples are cemented carbides and other cermets. E1C17 11/09/2009 11:8:37 Page 379 Section 17.3/Processing of Cermets 379 17.3.1 CEMENTED CARBIDES The cemented carbides are a family of composite materials consisting of carbide ceramic particles embedded in a metallic binder. They are classified as metal matrix composites because the metallic binder is the matrix that holds the bulk material together; however, the carbide particles constitute the largest proportion of the composite material, normally ranging between 80% and 96% by volume. Cemented carbides are technically classified as cermets, although they are often distinguished from the other materials in this class. The most important cemented carbide is tungsten carbide in a cobalt binder (WC– Co). Generally included within this category are certain mixtures of WC, TiC, and TaC in a Co matrix, in which tungsten carbide is the major component. Other cemented carbides include titanium carbide in nickel (TiC–Ni) and chromium carbide in nickel (Cr3C2–Ni). These composites are discussed in Section 9.2.1, and the carbide ingredients are described in Section 7.3.2. In our present discussion we are concerned with the particulate processing of cemented carbide. To provide a strong and pore-free part, the carbide powders must be sintered with a metal binder. Cobalt works best with WC, whereas nickel is better with TiC and Cr3C2. The usual proportion of binder metal is from around 4% up to 20%. Powders of carbide and binder metal are thoroughly mixed wet in a ball mill (or other suitable mixing machine) to form a homogeneous sludge. Milling also serves to refine particle size. The sludge is then dried in a vacuum or controlled atmosphere to prevent oxidation in preparation for compaction. Compaction Various methods are used to shape the powder mix into a green compact of the desired geometry. The most common process is cold pressing, described earlier and used for high production of cemented carbide parts such as cutting tool inserts. The dies used in cold pressing must be made oversized to account for shrinkage during sintering. Linear shrinkage can be 20% or more. For high production, the dies themselves are made with WC–Co liners to reduce wear, because of the abrasive nature of carbide particles. For smaller quantities, large flat sections are sometimes pressed and then cut into smaller pieces of the specified size. Other compaction methods used for cemented carbide products include isostatic pressing and hot pressing for large pieces, such as draw dies and ball mill balls; and extrusion, for long sections of circular, rectangular, or other cross section. Each of these processes has been described previously, either in this or the preceding chapter. Sintering Although it is possible to sinter WC and TiC without a binder metal, the resulting material is somewhat less than 100% of true density. Use of a binder yields a structure that is virtually free of porosity. Sintering of WC–Co involves liquid phase sintering (Section 16.4.5). The process can be explained with reference to the binary phase diagram for these constituents in Figure 17.11. The typical composition range for commercial cemented carbide products is identified in the diagram. The usual sintering temperatures for WC–Co are in the range 1370 C to 1425 C (2500 C to 2600 F), which is below cobalt’s melting point of 1495 C (2716 F). Thus, the pure binder metal does not melt at the sintering temperature. However, as the phase diagram shows, WC dissolves in Co in the solid state. During the heat treatment, WC is gradually dissolved into the gamma phase, and its melting point is reduced so that melting finally occurs. As the liquid phase forms, it flows and wets the WC particles, further dissolving the solid. The presence of the molten metal also serves to remove gases from the internal regions of the compact. These mechanisms combine to effect a rearrangement of the remaining WC particles into a closer packing, which results in significant densification and shrinkage of the WC–Co mass. Later, during cooling in the sintering cycle, the dissolved carbide is precipitated and deposited onto the existing crystals to form a coherent WC skeleton, throughout which the Co binder is embedded. 11/09/2009 380 11:8:38 Page 380 Chapter 17/Processing of Ceramics and Cermets Typical composition range of cemented carbide products Liquid 1800 3200 1600 1400 + liquid 1320∞C (2408∞F) 2400 1200 WC + 1000 FIGURE 17.11 WC–Co phase diagram. (Source: [7]). 2800 Temperature, ∞F WC + liquid Temperature, ∞C E1C17 2000 1600 0 WC 25 75 50 Weight percent cobalt 100 Co Secondary Operations Subsequent processing is usually required after sintering to achieve adequate dimensional control of cemented carbide parts. Grinding with a diamond abrasive wheel is the most common secondary operation performed for this purpose. Other processes used to shape the hard cemented carbides include electric discharge machining and ultrasonic machining, two nontraditional material removal processes discussed in Chapter 26. 17.3.2 OTHER CERMETS AND CERAMIC MATRIX COMPOSITES In addition to cemented carbides, other cermets are based on oxide ceramics such as Al2O3 and MgO. Chromium is a common metal binder used in these composite materials. The ceramic-to-metal proportions cover a wider range than those of the cemented carbides; in some cases, the metal is the major ingredient. These cermets are formed into useful products by the same basic shaping methods used for cemented carbides. The current technology of ceramic matrix composites (Section 9.3) includes ceramic materials (e.g., Al2O3, BN, Si3N4, and glass) reinforced by fibers of carbon, SiC, or Al2O3. If the fibers are whiskers (fibers consisting of single crystals), these CMCs can be processed by particulate methods used for new ceramics (Section 17.2). 17.4 PRODUCT DESIGN CONSIDERATIONS Ceramic materials have special properties that make them attractive to designers if the application is right. The following design recommendations, compiled from Bralla [2] and other sources, apply to both new and traditional ceramic materials, although designers are more likely to find opportunities for new ceramics in engineered products. In general, the same guidelines apply to cemented carbides. å Ceramic materials are several times stronger in compression than in tension; accordingly, ceramic components should be designed to be subjected to compressive stresses, not tensile stresses. å Ceramics are brittle and possess almost no ductility. Ceramic parts should not be used in applications that involve impact loading or high stresses that might cause fracture. E1C17 11/09/2009 11:8:38 Page 381 Review Questions 381 å Although many of the ceramic shaping processes allow complex geometries to be formed, it is desirable to keep shapes simple for both economic and technical reasons. Deep holes, channels, and undercuts should be avoided, as should large cantilevered projections. å Outside edges and corners should have radii or chamfers; likewise, inside corners should have radii. This guideline is, of course, violated in cutting tool applications, in which the cutting edge must be sharp to function. The cutting edge is often fabricated with a very small radius or chamfer to protect it from microscopic chipping, which could lead to failure. å Part shrinkage in drying and firing (for traditional ceramics) and sintering (for new ceramics) may be significant and must be taken into account by the designer in dimensioning and tolerancing. This is mostly a problem for manufacturing engineers, who must determine appropriate size allowances so that the final dimensions will be within the tolerances specified. å Screw threads in ceramic parts should be avoided. They are difficult to fabricate and do not have adequate strength in service after fabrication. REFERENCES [1] Bhowmick, A. K. Bradley Pulverizer Company, Allentown, Pennsylvania, personal communication, February 1992. [2] Bralla, J. G. (editor-in-chief). Design for Manufacturability Handbook. 2nd ed. McGraw-Hill, New York, 1999. [3] Hlavac, J. The Technology of Glass and Ceramics. Elsevier Scientific Publishing, New York, 1983. [4] Kingery, W. D., Bowen, H. K., and Uhlmann, D. R. Introduction to Ceramics. 2nd ed. John Wiley & Sons, New York, 1995. [5] Rahaman, M. N. Ceramic Processing. CRC Taylor & Francis, Boca Raton, Florida, 2007. [6] Richerson, D. W. Modern Ceramic Engineering: Properties, Processing, and Use in Design, 3rd ed. CRC Taylor & Francis, Boca Raton, Flotida, 2006. [7] Schwarzkopf, P., and Kieffer, R. Cemented Carbides. Macmillan, New York, 1960. [8] Singer, F., and Singer, S. S. Industrial Ceramics. Chemical Publishing Company, New York, 1963. [9] Somiya, S. (ed.). Advanced Technical Ceramics. Academic Press, San Diego, California, 1989. REVIEW QUESTIONS 17.1. What is the difference between the traditional ceramics and the new ceramics, as far as raw materials are concerned? 17.2. List the basic steps in the traditional ceramics processing sequence. 17.3. What is the technical difference between crushing and grinding in the preparation of traditional ceramic raw materials? 17.4. Describe the slip casting process in traditional ceramics processing. 17.5. List and briefly describe some of the plastic forming methods used to shape traditional ceramic products. 17.6. What is the process of jiggering? 17.7. What is the difference between dry pressing and semi-dry pressing of traditional ceramic parts? 17.8. What happens to a ceramic material when it is sintered? 17.9. What is the name given to the furnace used to fire ceramic ware? 17.10. What is glazing in traditional ceramics processing? 17.11. Why is the drying step, so important in the processing of traditional ceramics, usually not required in processing of new ceramics? 17.12. Why is raw material preparation more important in the processing of new ceramics than for traditional ceramics? 17.13. What is the freeze drying process used to make certain new ceramic powders? 17.14. Describe the doctor-blade process. 17.15. Liquid phase sintering is used for WC–Co compacts, even though the sintering temperatures are below the meltingpointsofeitherWCorCo.Howisthispossible? 17.16. What are some design recommendations for ceramic parts? E1C17 11/09/2009 382 11:8:42 Page 382 Chapter 17/Processing of Ceramics and Cermets MULTIPLE CHOICE QUIZ There are 16 correct answers in the following multiple choice questions (some questions have multiple answers that are correct). To attain a perfect score on the quiz, all correct answers must be given. Each correct answer is worth 1 point. Each omitted answer or wrong answer reduces the score by 1 point, and each additional answer beyond the correct number of answers reduces the score by 1 point. Percentage score on the quiz is based on the total number of correct answers. 17.1. The following equipment is used for crushing and grinding of minerals in the preparation of traditional ceramics raw materials. Which of the pieces listed is used for grinding (two correct answers): (a) ball mill, (b) hammer mill, (c) jaw crusher, (d) roll crusher, and (e) roller mill? 17.2. Which one of the following compounds becomes a plastic and formable material when mixed with suitable proportions of water: (a) aluminum oxide, (b) hydrogen oxide, (c) hydrous aluminum silicate, or (d) silicon dioxide? 17.3. At which one of the following water contents does clay become a suitably plastic material for the traditional ceramics plastic forming processes: (a) 5%, (b) 10%, (c) 20%, or (d) 40%? 17.4. Which of the following processes are not plastic forming methods used in the shaping of traditional ceramics (three correct answers): (a) dry pressing, (b) extrusion, (c) jangling, (d) jiggering, (e) jolleying, (f) slip casting, and (g) spinning? 17.5. The term green piece in ceramics refers to a part that has been shaped but not yet fired: (a) true or (b) false? 17.6. In the final product made of a polycrystalline new ceramic material, strength increases with grain size: (a) true or (b) false? 17.7. Which one of the following processes for the new ceramic materials accomplishes shaping and sintering simultaneously: (a) doctor-blade process, (b) freeze drying, (c) hot pressing, (d) injection molding, or (e) isostatic pressing? 17.8. Which of the following are the purposes of finishing operations used for parts made of the new ceramics (two best answers): (a) apply a surface coating, (b) electroplate the surface, (c) improve surface finish, (d) increase dimensional accuracy, and (e) work harden the surface? 17.9. Which of the following terms describes what a cemented carbide is (one best answer): (a) ceramic, (b) cermet, (c) composite, (d) metal, (e) new ceramic, or (f) traditional ceramic? 17.10. Which of the following geometric features should be avoided if possible in the design of structural components made of new ceramics (three best answers): (a) deep holes, (b) rounded inside corners, (c) rounded outside corners, (d) sharp edges, (e) thick sections, and (f) threads? E1C18 11/10/2009 15:7:28 Page 383 Part V Metal Forming and Sheet Metalworking 18 FUNDAMENTALS OF METAL FORMING Chapter Contents 18.1 18.2 18.3 18.4 18.5 Overview of Metal Forming Material Behavior in Metal Forming Temperature in Metal Forming Strain Rate Sensitivity Friction and Lubrication in Metal Forming Metal forming includes a large group of manufacturing processes in which plastic deformation is used to change the shape of metal workpieces. Deformation results from the use of a tool, usually called a die in metal forming, which applies stresses that exceed the yield strength of the metal. The metal therefore deforms to take a shape determined by the geometry of the die. Metal forming dominates the class of shaping operations identified in Chapter 1 as the deformation processes (Figure 1.4). Stresses applied to plastically deform the metal are usually compressive. However, some forming processes stretch the metal, while others bend the metal, and still others apply shear stresses to the metal. To be successfully formed, a metal must possess certain properties. Desirable properties include low yield strength and high ductility. These properties are affected by temperature. Ductility is increased and yield strength is reduced when work temperature is raised. The effect of temperature gives rise to distinctions between cold working, warm working, and hot working. Strain rate and friction are additional factors that affect performance in metal forming. We examine all of these issues in this chapter, but first let us provide an overview of the metal forming processes. 18.1 OVERVIEW OF METAL FORMING Metal forming processes can be classified into two basic categories: bulk deformation processes and sheet metalworking 383 E1C18 11/10/2009 384 15:7:29 Page 384 Chapter 18/Fundamentals of Metal Forming Rolling processes Bulk deformation Forging processes Extrusion processes Wire and bar drawing Metal forming Bending operations Sheet metalworking Deep or cup drawing Shearing processes FIGURE 18.1 Classification of metal forming operations. Miscellaneous processes processes. These two categories are covered in detail in Chapters 19 and 20, respectively. Each category includes several major classes of shaping operations, as indicated in Figure 18.1. Bulk Deformation Processes Bulk deformation processes are generally characterized by significant deformations and massive shape changes, and the surface area-to-volume of the work is relatively small. The term bulk describes the workparts that have this low areato-volume ratio. Starting work shapes for these processes include cylindrical billets and rectangular bars. Figure 18.2 illustrates the following basic operations in bulk deformation: FIGURE 18.2 Basic bulk deformation processes: (a) rolling, (b) forging, (c) extrusion, and (d) drawing. Relative motion in the operations is indicated by v; forces are indicated by F. E1C18 11/10/2009 15:7:29 Page 385 Section 18.1/Overview of Metal Forming 385 å Rolling. This is a compressive deformation process in which the thickness of a slab or plate is reduced by two opposing cylindrical tools called rolls. The rolls rotate so as to draw the work into the gap between them and squeeze it. å Forging. In forging, a workpiece is compressed between two opposing dies, so that the die shapes are imparted to the work. Forging is traditionally a hot working process, but many types of forging are performed cold. å Extrusion. This is a compression process in which the work metal is forced to flow through a die opening, thereby taking the shape of the opening as its own cross section. å Drawing. In this forming process, the diameter of a round wire or bar is reduced by pulling it through a die opening. Sheet Metalworking Sheet metalworking processes are forming and cutting operations performed on metal sheets, strips, and coils. The surface area-to-volume ratio of the starting metal is high; thus, this ratio is a useful means to distinguish bulk deformation from sheet metal processes. Pressworking is the term often applied to sheet metal operations because the machines used to perform these operations are presses (presses of various types are also used in other manufacturing processes). A part produced in a sheet metal operation is often called a stamping. Sheet metal operations are always performed as cold working processes and are usually accomplished using a set of tools called a punch and die. The punch is the positive portion and the die is the negative portion of the tool set. The basic sheet metal operations are sketched in Figure 18.3 and are defined as follows: å Bending. Bending involves straining of a metal sheet or plate to take an angle along a (usually) straight axis. FIGURE 18.3 Basic sheet metalworking operations: (a) bending, (b) drawing, and (c) shearing: (1) as punch first contacts sheet, and (2) after cutting. Force and relative motion in these operations are indicated by F and v. E1C18 11/10/2009 386 15:7:29 Page 386 Chapter 18/Fundamentals of Metal Forming å Drawing. In sheet metalworking, drawing refers to the forming of a flat metal sheet into a hollow or concave shape, such as a cup, by stretching the metal. A blankholder is used to hold down the blank while the punch pushes into the sheet metal, as shown in Figure 18.3(b). To distinguish this operation from bar and wire drawing, the terms cup drawing or deep drawing are often used. å Shearing. This process seems somewhat out-of-place in a list of deformation processes, because it involves cutting rather than forming. A shearing operation cuts the work using a punch and die, as in Figure 18.3(c). Although it is not a forming process, it is included here because it is a necessary and very common operation in sheet metalworking. The miscellaneous processes within the sheet metalworking classification in Figure 18.1 include a variety of related shaping processes that do not use punch and die tooling. Examples of these processes are stretch forming, roll bending, spinning, and bending of tube stock. 18.2 MATERIAL BEHAVIOR IN METAL FORMING Considerable insight about the behavior of metals during forming can be obtained from the stress–strain curve. The typical stress–strain curve for most metals is divided into an elastic region and a plastic region (Section 3.1.1). In metal forming, the plastic region is of primary interest because the material is plastically and permanently deformed in these processes. The typical stress–strain relationship for a metal exhibits elasticity below the yield point and strain hardening above it. Figures 3.4 and 3.5 indicate this behavior in linear and logarithmic axes. In the plastic region, the metal’s behavior is expressed by the flow curve: s ¼ Ken where K ¼ the strength coefficient, MPa (lb/in2); and n is the strain-hardening exponent. The stress s and strain e in the flow curve are true stress and true strain. The flow curve is generally valid as a relationship that defines a metal’s plastic behavior in cold working. Typical values of K and n for different metals at room temperature are listed in Table 3.4. Flow Stress The flow curve describes the stress–strain relationship in the region in which metal forming takes place. It indicates the flow stress of the metal—the strength property that determines forces and power required to accomplish a particular forming operation. For most metals at room temperature, the stress–strain plot of Figure 3.5 indicates that as the metal is deformed, its strength increases due to strain hardening. The stress required to continue deformation must be increased to match this increase in strength. Flow stress is defined as the instantaneous value of stress required to continue deforming the material— to keep the metal ‘‘flowing.’’ It is the yield strength of the metal as a function of strain, which can be expressed: Y f ¼ Ken ð18:1Þ where Yf ¼ flow stress, MPa (lb/in2). In the individual forming operations discussed in the following two chapters, the instantaneous flow stress can be used to analyze the process as it is occurring. For example, in certain forging operations, the instantaneous force during compression can be determined from the flow stress value. Maximum force can be calculated based on the flow stress that results from the final strain at the end of the forging stroke. In other cases, the analysis is based on the average stresses and strains that occur during deformation rather than instantaneous values. Extrusion represents this case, Figure 18.2(c). As the billet is reduced in cross section to pass through the extrusion E1C18 11/10/2009 15:7:29 Page 387 Section 18.3/Temperature in Metal Forming 387 FIGURE 18.4 Stress–strain curve indicating location of average flow stress Y f in relation to yield strength Y and final flow stress Yf. die opening, the metal gradually strain hardens to reach a maximum value. Rather than determine a sequence of instantaneous stress–strain values during the reduction, which would be not only difficult but also of limited interest, it is more useful to analyze the process based on the average flow stress during deformation. Average Flow Stress The average flow stress (also called the mean flow stress) is the average value of stress over the stress–strain curve from the beginning of strain to the final (maximum) value that occurs during deformation. The value is illustrated in the stress– strain plot of Figure 18.4. The average flow stress is determined by integrating the flow curve equation, Eq. (18.1), between zero and the final strain value defining the range of interest. This yields the equation: Yf ¼ Ken 1þn ð18:2Þ where Y f ¼ average flow stress, MPa (lb/in2); and e ¼ maximum strain value during the deformation process. We make extensive use of the average flow stress in our study of the bulk deformation processes in the following chapter. Given values of K and n for the work material, a method of computing final strain will be developed for each process. Based on this strain, Eq. (18.2) can be used to determine the average flow stress to which the metal is subjected during the operation. 18.3 TEMPERATURE IN METAL FORMING The flow curve is a valid representation of stress–strain behavior of a metal during plastic deformation, particularly for cold working operations. For any metal, the values of K and n depend on temperature. Strength and strain hardening are both reduced at higher temperatures. These property changes are important because they result in lower forces and power during forming. In addition, ductility is increased at higher temperatures, which allows greater plastic deformation of the work metal. We can distinguish three temperature ranges that are used in metal forming: cold, warm, and hot working. Cold Working Cold working (also known as cold forming) is metal forming performed at room temperature or slightly above. Significant advantages of cold forming compared E1C18 11/10/2009 388 15:7:29 Page 388 Chapter 18/Fundamentals of Metal Forming to hot working are (1) greater accuracy, meaning closer tolerances can be achieved; (2) better surface finish; (3) higher strength and hardness of the part due to strain hardening; (4) grain flow during deformation provides the opportunity for desirable directional properties to be obtained in the resulting product; and (5) no heating of the work is required, which saves on furnace and fuel costs and permits higher production rates. Owing to this combination of advantages, many cold forming processes have become important mass-production operations. They provide close tolerances and good surfaces, minimizing the amount of machining required so that these operations can be classified as net shape or near net shape processes (Section 1.3.1). There are certain disadvantages or limitations associated with cold forming operations: (1) higher forces and power are required to perform the operation; (2) care must be taken to ensure that the surfaces of the starting workpiece are free of scale and dirt; and (3) ductility and strain hardening of the work metal limit the amount of forming that can be done to the part. In some operations, the metal must be annealed (Section 27.1) in order to allow further deformation to be accomplished. In other cases, the metal is simply not ductile enough to be cold worked. To overcome the strain-hardening problem and reduce force and power requirements, many forming operations are performed at elevated temperatures. There are two elevated temperature ranges involved, giving rise to the terms warm working and hot working. Warm Working Because plastic deformation properties are normally enhanced by increasing workpiece temperature, forming operations are sometimes performed at temperatures somewhat above room temperature but below the recrystallization temperature. The term warm working is applied to this second temperature range. The dividing line between cold working and warm working is often expressed in terms of the melting point for the metal. The dividing line is usually taken to be 0.3 Tm, where Tm is the melting point (absolute temperature) for the particular metal. The lower strength and strain hardening at the intermediate temperatures, as well as higher ductility, provide warm working with the following advantages over cold working: (1) lower forces and power, (2) more intricate work geometries possible, and (3) need for annealing may be reduced or eliminated. Hot Working Hot working (also called hot forming) involves deformation at temperatures above the recrystallization temperature (Section 3.3). The recrystallization temperature for a given metal is about one-half of its melting point on the absolute scale. In practice, hot working is usually carried out at temperatures somewhat above 0.5Tm. The work metal continues to soften as temperature is increased beyond 0.5Tm, thus enhancing the advantage of hot working above this level. However, the deformation process itself generates heat, which increases work temperatures in localized regions of the part. This can cause melting in these regions, which is highly undesirable. Also, scale on the work surface is accelerated at higher temperatures. Accordingly, hot working temperatures are usually maintained within the range 0.5Tm to 0.75Tm. The most significant advantage of hot working is the capability to produce substantial plastic deformation of the metal—far more than is possible with cold working or warm working. The principal reason for this is that the flow curve of the hot-worked metal has a strength coefficient that is substantially less than at room temperature, the strain-hardening exponent is zero (at least theoretically), and the ductility of the metal is significantly increased. All of this results in the following advantages relative to cold working: (1) the shape of the workpart can be significantly altered, (2) lower forces and power are required to deform the metal, (3) metals that usually fracture in cold working can be hot formed, (4) strength properties are generally isotropic because of the absence of the oriented grain structure typically created in cold working, and (5) no strengthening of the part occurs from work hardening. This last advantage may seem inconsistent, since strengthening of E1C18 11/10/2009 15:7:29 Page 389 Section 18.4/Strain Rate Sensitivity 389 the metal is often considered an advantage for cold working. However, there are applications in which it is undesirable for the metal to be work hardened because it reduces ductility, for example, if the part is to be subsequently processed by cold forming. Disadvantages of hot working include (1) lower dimensional accuracy, (2) higher total energy required (due to the thermal energy to heat the workpiece), (3) work surface oxidation (scale), (4) poorer surface finish, and (5) shorter tool life. Recrystallization of the metal in hot working involves atomic diffusion, which is a time-dependent process. Metal forming operations are often performed at high speeds that do not allow sufficient time for complete recrystallization of the grain structure during the deformation cycle itself. However, because of the high temperatures, recrystallization eventually does occur. It may occur immediately following the forming process or later, as the workpiece cools. Even though recrystallization may occur after the actual deformation, its eventual occurrence, and the substantial softening of the metal at high temperatures, are the features that distinguish hot working from warm working or cold working. Isothermal Forming Certain metals, such as highly alloyed steels, many titanium alloys, and high-temperature nickel alloys, possess good hot hardness, a property that makes them useful for high-temperature service. However, this very property that makes them attractive in these applications also makes them difficult to form with conventional methods. The problem is that when these metals are heated to their hot working temperatures and then come in contact with the relatively cold forming tools, heat is quickly transferred away from the part surfaces, thus raising the strength in these regions. The variations in temperature and strength in different regions of the workpiece cause irregular flow patterns in the metal during deformation, leading to high residual stresses and possible surface cracking. Isothermal forming refers to forming operations that are carried out in such a way as to eliminate surface cooling and the resulting thermal gradients in the workpart. It is accomplished by preheating the tools that come in contact with the part to the same temperature as the work metal. This weakens the tools and reduces tool life, but it avoids the problems described above when these difficult metals are formed by conventional methods. In some cases, isothermal forming represents the only way in which these work materials can be formed. The procedure is most closely associated with forging, and we discuss isothermal forging in the following chapter. 18.4 STRAIN RATE SENSITIVITY Theoretically, a metal in hot working behaves like a perfectly plastic material, with strainhardening exponent n ¼ 0. This means that the metal should continue to flow under the same level of flow stress, once that stress level is reached. However, there is an additional phenomenon that characterizes the behavior of metals during deformation, especially at the elevated temperatures of hot working. That phenomenon is strain rate sensitivity. Let us begin our discussion of this topic by defining strain rate. The rate at which the metal is strained in a forming process is directly related to the speed of deformation, v. In many forming operations, deformation speed is equal to the velocity of the ram or other moving element of the equipment. It is most easily visualized in a tensile test as the velocity of the testing machine head relative to its fixed base. Given the deformation speed, strain rate is defined: v ð18:3Þ e_ ¼ h where e_ ¼ true strain rate, m/s/m (in/sec/in), or simply s–1; and h ¼ instantaneous height of the workpiece being deformed, m (in). If deformation speed v is constant during the operation, strain rate will change as h changes. In most practical forming operations, E1C18 11/10/2009 390 15:7:29 Page 390 Chapter 18/Fundamentals of Metal Forming FIGURE 18.5 (a) Effect of strain rate on flow stress at an elevated work temperature. (b) Same relationship plotted on log–log coordinates. valuation of strain rate is complicated by the geometry of the workpart and variations in strain rate in different regions of the part. Strain rate can reach 1000 s–1 or more for some metal forming processes such as high-speed rolling and forging. We have already observed that the flow stress of a metal is a function of temperature. At the temperatures of hot working, flow stress depends on strain rate. The effect of strain rate on strength properties is known as strain rate sensitivity. The effect can be seen in Figure 18.5. As strain rate is increased, resistance to deformation increases. This usually plots approximately as a straight line on a log–log graph, thus leading to the relationship: Y f ¼ Ce_ m ð18:4Þ where C is the strength constant (similar but not equal to the strength coefficient in the flow curve equation), and m is the strain rate sensitivity exponent. The value of C is determined at a strain rate of 1.0, and m is the slope of the curve in Figure 18.5(b). FIGURE 18.6 Effect of temperature on flow stress for a typical metal. The constant C in Eq. (18.4), indicated by the intersection of each plot with the vertical dashed line at strain rate ¼ 1.0, decreases, and m (slope of each plot) increases with increasing temperature. E1C18 11/10/2009 15:7:29 Page 391 Section 18.5/Friction and Lubrication in Metal Forming 391 TABLE 18.1 Typical values of temperature, strain-rate sensitivity, and coefficient of friction in cold, warm, and hot working. Temperature Strain-Rate Coefficient Category Range Sensitivity Exponent of Friction Cold working Warm working Hot working 0.3Tm 0.3Tm–0.5Tm 0.5Tm–0.75Tm 0.000  m  0.05 0.05  m  0.1 0.05  m  0.4 0.1 0.2 0.4–0.5 The effect of temperature on the parameters of Eq. (18.4) is pronounced. Increasing temperature decreases the value of C (consistent with its effect on K in the flow curve equation) and increases the value of m. The general result can be seen in Figure 18.6. At room temperature, the effect of strain rate is almost negligible, indicating that the flow curve is a good representation of the material behavior. As temperature is increased, strain rate plays a more important role in determining flow stress, as indicated by the steeper slopes of the strain rate relationships. This is important in hot working because deformation resistance of the material increases so dramatically as strain rate is increased. To give a sense of the effect, typical values of m for the three temperature ranges of metal working are given in Table 18.1. Thus we see that even in cold working, strain rate can have an effect, if small, on flow stress. In hot working, the effect can be significant. A more complete expression for flow stress as a function of both strain and strain rate would be the following: Y f ¼ Aen e_ m ð18:5Þ where A ¼ a strength coefficient, combining the effects of the previous K and C values. Of course, A, n, and m would all be functions of temperature, and the enormous task of testing and compiling the values of these parameters for different metals and various temperatures would be forbidding. In our coverage of the various bulk deformation processes in Chapter 19, many of which are performed hot, we neglect the effect of strain rate in analyzing forces and power. For cold working and warm working, and for hot working operations at relatively low deformation speeds, this neglect represents a reasonable assumption. 18.5 FRICTION AND LUBRICATION IN METAL FORMING Friction in metal forming arises because of the close contact between the tool and work surfaces and the high pressures that drive the surfaces together in these operations. In most metal forming processes, friction is undesirable for the following reasons: (1) metal flow in the work is retarded, causing residual stresses and sometimes defects in the product; (2) forces and power to perform the operation are increased, and (3) tool wear can lead to loss of dimensional accuracy, resulting in defective parts and requiring replacement of the tooling. Since tools in metal forming are generally expensive, tool wear is a major concern. Friction and tool wear are more severe in hot working because of the much harsher environment. Friction in metal forming is different from that encountered in most mechanical systems, such as gear trains, shafts and bearings, and other components involving relative motion between surfaces. These other cases are generally characterized by low contact pressures, low to moderate temperatures, and ample lubrication to minimize metal-tometal contact. By contrast, the metal forming environment features high pressures between a hardened tool and a soft workpart, plastic deformation of the softer material, and high E1C18 11/10/2009 392 15:7:30 Page 392 Chapter 18/Fundamentals of Metal Forming temperatures (at least in hot working). These conditions can result in relatively high coefficients of friction in metal working, even in the presence of lubricants. Typical values of coefficient of friction for the three categories of metal forming are listed in Table 18.1. If the coefficient of friction becomes large enough, a condition known as sticking occurs. Sticking in metalworking (also called sticking friction) is the tendency for the two surfaces in relative motion to adhere to each other rather than slide. It means that the friction stress between the surfaces exceeds the shear flow stress of the work metal, thus causing the metal to deform by a shear process beneath the surface rather than slip at the surface. Sticking occurs in metal forming operations and is a prominent problem in rolling; we discuss it in that context in the following chapter. Metalworking lubricants are applied to the tool–work interface in many forming operations to reduce the harmful effects of friction. Benefits include reduced sticking, forces, power, and tool wear; and better surface finish on the product. Lubricants also serve other functions, such as removing heat from the tooling. Considerations in choosing an appropriate metalworking lubricant include (1) type of forming process (rolling, forging, sheet metal drawing, and so on), (2) whether used in hot working or cold working, (3) work material, (4) chemical reactivity with the tool and work metals (it is generally desirable for the lubricant to adhere to the surfaces to be most effective in reducing friction), (5) ease of application, (6) toxicity, (7) flammability, and (8) cost. Lubricants used for cold working operations include [4], [7] mineral oils, fats and fatty oils, water-based emulsions, soaps, and other coatings. Hot working is sometimes performed dry for certain operations and materials (e.g., hot rolling of steel and extrusion of aluminum). When lubricants are used in hot working, they include mineral oils, graphite, and glass. Molten glass becomes an effective lubricant for hot extrusion of steel alloys. Graphite contained in water or mineral oil is a common lubricant for hot forging of various work materials. More detailed treatments of lubricants in metalworking are found in references [7] and [9]. REFERENCES [1] Altan, T., Oh, S.-I., and Gegel, H. L. Metal Forming: Fundamentals and Applications. ASM International, Materials Park, Ohio, 1983. [2] Cook, N. H. Manufacturing Analysis. AddisonWesley Publishing Company, Inc., Reading, Massachusetts, 1966. [3] Hosford, W. F., and Caddell, R. M. Metal Forming: Mechanics and Metallurgy, 3rd ed. Cambridge University Press, Cambridge, UK, 2007. [4] Lange, K. Handbook of Metal Forming. Society of Manufacturing Engineers, Dearborn, Michigan, 2006. [5] Lenard, J. G. Metal Forming Science and Practice, Elsevier Science, Amsterdam, The Netherlands, 2002. [6] Mielnik, E. M. Metalworking Science and Engineering. McGraw-Hill, Inc., New York, 1991. [7] Nachtman, E. S., and Kalpakjian, S. Lubricants and Lubrication in Metalworking Operations. Marcel Dekker, Inc., New York, 1985. [8] Wagoner, R. H., and Chenot, J.-L. Fundamentals of Metal Forming. John Wiley & Sons, Inc., New York, 1997. [9] Wick, C., et al. (eds.). Tool and Manufacturing Engineers Handbook, 4th ed. Vol. II, Forming. Society of Manufacturing Engineers, Dearborn, Michigan, 1984. REVIEW QUESTIONS 18.1. What are the differences between bulk deformation processes and sheet metal processes? 18.2. Extrusion is a fundamental shaping process. Describe it. 18.3. Why is the term pressworking often used for sheet metal processes? 18.4. What is the difference between deep drawing and bar drawing? E1C18 11/10/2009 15:7:30 Page 393 Problems 18.5. Indicate the mathematical equation for the flow curve. 18.6. How does increasing temperature affect the parameters in the flow curve equation? 18.7. Indicate some of the advantages of cold working relative to warm and hot working. 393 18.8. What is isothermal forming? 18.9. Describe the effect of strain rate in metal forming. 18.10. Why is friction generally undesirable in metal forming operations? 18.11. What is sticking friction in metalworking? MULTIPLE CHOICE QUIZ There are 13 correct answers in the following multiple choice questions (some questions have multiple answers that are correct). To attain a perfect score on the quiz, all correct answers must be given. Each correct answer is worth 1 point. Each omitted answer or wrong answer reduces the score by 1 point, and each additional answer beyond the correct number of answers reduces the score by 1 point. Percentage score on the quiz is based on the total number of correct answers. 18.1. Which of the following are bulk deformation processes (three correct answers): (a) bending, (b) deep drawing, (c) extrusion, (d) forging, (e) rolling, and (f) shearing? 18.2. Which of the following is typical of the starting work geometry in sheet metal processes: (a) high volume-to-area ratio or (b) low volume-to-area ratio? 18.3. The flow curve expresses the behavior of a metal in which of the following regions of the stress– strain curve: (a) elastic region or (b) plastic region? 18.4. The average flow stress is the flow stress multiplied by which of the following factors: (a) n, (b) (1 þ n), (c) 1/n, or (d) 1/(1 þ n), where n is the strainhardening exponent? 18.5. Hot working of metals refers to which one of the following temperature regions relative to the melting point of the given metal on an absolute temperature scale: (a) room temperature, (b) 0.2Tm, (c) 0.4Tm, or (d) 0.6Tm? 18.6. Which of the following are advantages and characteristics of hot working relative to cold working (four correct answers): (a) fracture of workpart is less likely, (b) friction is reduced, (c) increased strength properties, (d) isotropic mechanical properties, (e) less overall energy is required, (f) lower deformation forces is required, (g) more significant shape changes are possible, and (h) strain-rate sensitivity is reduced? 18.7. Increasing strain rate tends to have which one of the following effects on flow stress during hot forming of metal: (a) decreases flow stress, (b) has no effect, or (c) increases flow stress? 18.8. The coefficient of friction between the part and the tool in cold working tends to be (a) higher, (b) lower, or (c) no different relative to its value in hot working? PROBLEMS Flow Curve in Forming 18.1. The strength coefficient ¼ 550 MPa and strainhardening exponent ¼ 0.22 for a certain metal. During a forming operation, the final true strain that the metal experiences ¼ 0.85. Determine the flow stress at this strain and the average flow stress that the metal experienced during the operation. 18.2. A metal has a flow curve with parameters: strength coefficient ¼ 850 MPa and strain-hardening exponent ¼ 0.30. A tensile specimen of the metal with gage length ¼ 100 mm is stretched to a length ¼ 157 mm. Determine the flow stress at the new length and the average flow stress that the metal has been subjected to during the deformation. 18.3. A particular metal has a flow curve with parameters: strength coefficient ¼ 35,000 lb/in2 and strain-hardening exponent ¼ 0.26. A tensile specimen of the metal with gage length ¼ 2.0 in is stretched to a length ¼ 3.3 in. Determine the flow stress at this new length and the average flow stress that the metal has been subjected to during deformation. 18.4. The strength coefficient and strain-hardening exponent of a certain test metal are 40,000 lb/in2 and 0.19, respectively. A cylindrical specimen of the metal with starting diameter ¼ 2.5 in and length ¼ 3.0 in is compressed to a length of 1.5 in. Determine the flow stress at this compressed length and E1C18 11/10/2009 394 18.5. 18.6. 18.7. 18.8. 15:7:30 Page 394 Chapter 18/Fundamentals of Metal Forming the average flow stress that the metal has experienced during deformation. Derive the equation for average flow stress, Eq. (18.2) in the text. For a certain metal, the strength coefficient ¼ 700 MPa and strain-hardening exponent ¼ 0.27. Determine the average flow stress that the metal experiences if it is subjected to a stress that is equal to its strength coefficient K. Determine the value of the strain-hardening exponent for a metal that will cause the average flow stress to be 3/4 of the final flow stress after deformation. The strength coefficient ¼ 35,000 lb/in2 and strainhardening exponent ¼ 0.40 for a metal used in a forming operation in which the workpart is reduced in cross-sectional area by stretching. If the average flow stress on the part is 20,000 lb/in2, determine the amount of reduction in cross-sectional area experienced by the part. 18.9. In a tensile test, two pairs of values of stress and strain were measured for the specimen metal after it had yielded: (1) true stress ¼ 217 MPa and true strain ¼ 0.35, and (2) true stress ¼ 259 MPa and true strain ¼ 0.68. Based on these data points, determine the strength coefficient and strain-hardening exponent. 18.10. The following stress and strain values were measured in the plastic region during a tensile test carried out on a new experimental metal: (1) true stress ¼ 43,608 lb/in2 and true strain ¼ 0.27 in/in, and (2) true stress ¼ 52,048 lb/in2 and true strain ¼ 0.85 in/in. Based on these data points, determine the strength coefficient and strain-hardening exponent. Strain Rate 18.11. The gage length of a tensile test specimen ¼ 150 mm. It is subjected to a tensile test in which the grips holding the end of the test specimen are moved with a relative velocity ¼ 0.1 m/s. Construct a plot of the strain rate as a function of length as the specimen is pulled to a length ¼ 200 mm. 18.12. A specimen with 6.0 in starting gage length is subjected to a tensile test in which the grips holding the end of the test specimen are moved with a relative velocity ¼ 1.0 in/sec. Construct a plot of the strain rate as a function of length as the specimen is pulled to a length ¼ 8.0 in. 18.13. A workpart with starting height h ¼ 100 mm is compressed to a final height of 50 mm. During the deformation, the relative speed of the platens compressing the part ¼ 200 mm/s. Determine the strain rate at (a) h ¼ 100 mm, (b) h ¼ 75 mm, and (c) h ¼ 51 mm. 18.14. A hot working operation is carried out at various speeds. The strength constant ¼ 30,000 lb/in2 and the strain-rate sensitivity exponent ¼ 0.15. Determine the flow stress if the strain rate is (a) 0.01/sec (b) 1.0/sec, (c) 100/sec. 18.15. A tensile test is performed to determine the parameters strength constant C and strain-rate sensitivity exponent m in Eq. (18.4) for a certain metal. The temperature at which the test is performed ¼ 500 C. At a strain rate ¼ 12/s, the stress is measured at 160 MPa; and at a strain rate ¼ 250/s, the stress ¼ 300 MPa. (a) Determine C and m. (b) If the temperature were 600 C, what changes would you expect in the values of C and m? 18.16. A tensile test is carried out to determine the strength constant C and strain-rate sensitivity exponent m for a certain metal at 1000 F. At a strain rate ¼ 10/sec, the stress is measured at 23,000 lb/in2; and at a strain rate ¼ 300/sec, the stress ¼ 45,000 lb/in2. (a) Determine C and m. (b) If the temperature were 900 F, what changes would you expect in the values of C and m? E1C19 11/11/2009 16:35:33 Page 395 19 BULK DEFORMATION PROCESSES IN METAL WORKING Chapter Contents 19.1 Rolling 19.1.1 Flat Rolling and Its Analysis 19.1.2 Shape Rolling 19.1.3 Rolling Mills 19.2 Other Deformation Processes Related to Rolling 19.3 Forging 19.3.1 Open-Die Forging 19.3.2 Impression-Die Forging 19.3.3 Flashless Forging 19.3.4 Forging Hammers, Presses, and Dies 19.4 Other Deformation Processes Related to Forging 19.5 Extrusion 19.5.1 Types of Extrusion 19.5.2 Analysis of Extrusion 19.5.3 Extrusion Dies and Presses 19.5.4 Other Extrusion Processes 19.5.5 Defects in Extruded Products 19.6 Wire and Bar Drawing 19.6.1 Analysis of Drawing 19.6.2 Drawing Practice 19.6.3 Tube Drawing The deformation processes described in this chapter accomplish significant shape change in metal parts whose initial form is bulk rather than sheet. The starting forms include cylindrical bars and billets, rectangular billets and slabs, and similar elementary geometries. The bulk deformation processes refine the starting shapes, sometimes improving mechanical properties, and always adding commercial value. Deformation processes work by stressing the metal sufficiently to cause it to plastically flow into the desired shape. Bulk deformation processes are performed as cold, warm, and hot working operations. Cold and warm working is appropriate when the shape change is less severe, and there is a need to improve mechanical properties and achieve good finish on the part. Hot working is generally required when massive deformation of large workparts is involved. The commercial and technological importance of bulk deformation processes derives from the following: å When performed as hot working operations, they can achieve significant change in the shape of the workpart. å When performed as cold working operations, they can be used not only to shape the product, but also to increase its strength through strain hardening. å These processes produce little or no waste as a byproduct of the operation. Some bulk deformation operations are near net shape or net shape processes; they achieve final product geometry with little or no subsequent machining. The bulk deformation processes covered in this chapter are (1) rolling, (2) forging, (3) extrusion, and (4) wire and bar drawing. The chapter also documents the variations and related operations of the four basic processes that have been developed over the years. 395 E1C19 11/11/2009 396 16:35:33 Page 396 Chapter 19/Bulk Deformation Processes in Metal Working 19.1 ROLLING Rolling is a deformation process in which the thickness of the work is reduced by compressive forces exerted by two opposing rolls. The rolls rotate as illustrated in Figure 19.1 to pull and simultaneously squeeze the work between them. The basic process shown in our figure is flat rolling, used to reduce the thickness of a rectangular cross section. A closely related process is shape rolling, in which a square cross section is formed into a shape such as an I-beam. Most rolling processes are very capital intensive, requiring massive pieces of equipment, called rolling mills, to perform them. The high investment cost requires the mills to be used for production in large quantities of standard items such as sheets and plates. Most rolling is carried out by hot working, called hot rolling, owing to the large amount of deformation required. Hot-rolled metal is generally free of residual stresses, and its properties are isotropic. Disadvantages of hot rolling are that the product cannot be held to close tolerances, and the surface has a characteristic oxide scale. Steelmaking provides the most common application of rolling mill operations (Historical Note 19.1). Let us follow the sequence of steps in a steel rolling mill to illustrate the variety of products made. Similar steps occur in other basic metal industries. The work starts out as a cast steel ingot that has just solidified. While it is still hot, the ingot is placed in a furnace where it remains for many hours until it has reached a uniform temperature throughout, so that the metal will flow consistently during rolling. For steel, the desired temperature for rolling is around 1200 C (2200 F). The heating operation is called soaking, and the furnaces in which it is carried out are called soaking pits. From soaking, the ingot is moved to the rolling mill, where it is rolled into one of three intermediate shapes called blooms, billets, or slabs. A bloom has a square cross section 150 mm  150 mm (6 in  6 in) or larger. A slab is rolled from an ingot or a bloom and has a rectangular cross section of width 250 mm (10 in) or more and thickness 40 mm (1.5 in) or more. A billet is rolled from a bloom and is square with dimensions 40 mm (1.5 in) on a side or larger. These intermediate shapes are subsequently rolled into final product shapes. Blooms are rolled into structural shapes and rails for railroad tracks. Billets are rolled into bars and rods. These shapes are the raw materials for machining, wire drawing, forging, and other metalworking processes. Slabs are rolled into plates, sheets, and strips. Hot-rolled plates are used in shipbuilding, bridges, boilers, welded structures for various heavy machines, tubes and pipes, and many other products. Figure 19.2 shows some of these rolled steel products. Further flattening of hot-rolled plates and sheets is often accomplished by cold rolling, in order to prepare them for subsequent sheet metal operations (Chapter 20). Cold rolling strengthens the metal and permits a tighter tolerance on thickness. In addition, the surface of the cold-rolled sheet is absent of scale and generally superior to the corresponding hot-rolled product. These characteristics make cold-rolled sheets, strips, and coils ideal for stampings, exterior panels, and other parts of products ranging from automobiles to appliances and office furniture. FIGURE 19.1 The rolling process (specifically, flat rolling). E1C19 11/11/2009 16:35:34 Page 397 Section 19.1/Rolling Historical Note 19.1 397 Rolling R olling of gold and silver by manual methods dates from the fourteenth century. Leonardo da Vinci designed one of the first rolling mills in 1480, but it is doubtful that his design was ever built. By around 1600, cold rolling of lead and tin was accomplished on manually operated rolling mills. By around 1700, hot rolling of iron was being done in Belgium, England, France, Germany, and Sweden. These mills were used to roll iron bars into sheets. Prior to this time, the only rolls in steelmaking were slitting mills—pairs of opposing rolls with collars (cutting disks) used to slit iron and steel into narrow strips for making nails and similar products. Slitting mills were not intended to reduce thickness. Modern rolling practice dates from 1783 when a patent was issued in England for using grooved rolls to produce iron bars. The Industrial Revolution created a tremendous demand for iron and steel, stimulating developments in rolling. The first mill for rolling railway rails was started in 1820 in England. The first I-beams were rolled in France in 1849. In addition, the size and capacity of flat rolling mills increased dramatically during this period. Rolling is a process that requires a very large power source. Water wheels were used to power rolling mills until the eighteenth century. Steam engines increased the capacity of these rolling mills until soon after 1900 when electric motors replaced steam. 19.1.1 FLAT ROLLING AND ITS ANALYSIS Flat rolling is illustrated in Figures 19.1 and 19.3. It involves the rolling of slabs, strips, sheets, and plates—workparts of rectangular cross section in which the width is greater than the thickness. In flat rolling, the work is squeezed between two rolls so that its thickness is reduced by an amount called the draft: d ¼ to  tf ð19:1Þ where d ¼ draft, mm (in); to ¼ starting thickness, mm (in); and tf ¼ final thickness, mm (in). Draft is sometimes expressed as a fraction of the starting stock thickness, called the FIGURE 19.2 Some of the steel products made in a rolling mill. E1C19 11/11/2009 398 16:35:34 Page 398 Chapter 19/Bulk Deformation Processes in Metal Working reduction: r¼ d to ð19:2Þ where r ¼ reduction. When a series of rolling operations are used, reduction is taken as the sum of the drafts divided by the original thickness. In addition to thickness reduction, rolling usually increases work width. This is called spreading, and it tends to be most pronounced with low width-to-thickness ratios and low coefficients of friction. Conservation of matter is preserved, so the volume of metal exiting the rolls equals the volume entering t o wo Lo ¼ t f wf Lf ð19:3Þ where wo and wf are the before and after work widths, mm (in); and Lo and Lf are the before and after work lengths, mm (in). Similarly, before and after volume rates of material flow must be the same, so the before and after velocities can be related: to wo vo ¼ tf wf vf ð19:4Þ where vo and vf are the entering and exiting velocities of the work. The rolls contact the work along an arc defined by the angle u. Each roll has radius R, and its rotational speed gives it a surface velocity vr. This velocity is greater than the entering speed of the work vo and less than its exiting speed vf. Since the metal flow is continuous, there is a gradual change in velocity of the work between the rolls. However, there is one point along the arc where work velocity equals roll velocity. This is called the no-slip point, also known as the neutral point. On either side of this point, slipping and friction occur between roll and work. The amount of slip between the rolls and the work can be measured by means of the forward slip, a term used in rolling that is defined: s¼ vf  vr vr ð19:5Þ where s ¼ forward slip; vf ¼ final (exiting) work velocity, m/s (ft/sec); and vr ¼ roll speed, m/s (ft/sec). The true strain experienced by the work in rolling is based on before and after stock thicknesses. In equation form, e ¼ ln to tf ð19:6Þ The true strain can be used to determine the average flow stress Y f applied to the work material in flat rolling. Recall from the previous chapter, Eq. (18.2), that Yf ¼ Ken 1þn ð19:7Þ The average flow stress is used to compute estimates of force and power in rolling. Friction in rolling occurs with a certain coefficient of friction, and the compression force of the rolls, multiplied by this coefficient of friction, results in a friction force between the rolls and the work. On the entrance side of the no-slip point, friction force is in one direction, and on the other side it is in the opposite direction. However, the two forces are not equal. The friction force on the entrance side is greater, so that the net force pulls the work through the rolls. If this were not the case, rolling would not be possible. There is a limit to the maximum possible draft that can be accomplished in flat rolling with a given coefficient of friction, defined by: dmax ¼ m2 R ð19:8Þ E1C19 11/11/2009 16:35:34 Page 399 Section 19.1/Rolling 399 where dmax ¼ maximum draft, mm (in); m ¼ coefficient of friction; and R ¼ roll radius mm (in). The equation indicates that if friction were zero, draft would be zero, and it would be impossible to accomplish the rolling operation. Coefficient of friction in rolling depends on lubrication, work material, and working temperature. In cold rolling, the value is around 0.1; in warm working, a typical value is around 0.2; and in hot rolling, m is around 0.4 [16]. Hot rolling is often characterized by a condition called sticking, in which the hot work surface adheres to the rolls over the contact arc. This condition often occurs in the rolling of steels and high-temperature alloys. When sticking occurs, the coefficient of friction can be as high as 0.7. The consequence of sticking is that the surface layers of the work are restricted to move at the same speed as the roll speed vr; and below the surface, deformation is more severe in order to allow passage of the piece through the roll gap. Given a coefficient of friction sufficient to perform rolling, roll force F required to maintain separation between the two rolls can be computed by integrating the unit roll pressure (shown as p in Figure 19.3) over the roll-work contact area. This can be expressed: ZL F¼w pd L ð19:9Þ 0 where F ¼ rolling force, N (lb); w ¼ the width of the work being rolled, mm (in); p ¼ roll pressure, MPa (lb/in2); and L ¼ length of contact between rolls and work, mm (in). The integration requires two separate terms, one for either side of the neutral point. Variation in roll pressure along the contact length is significant. A sense of this variation can be obtained from the plot in Figure 19.4. Pressure reaches a maximum at the neutral point, and trails off on either side to the entrance and exit points. As friction increases, maximum pressure increases relative to entrance and exit values. As friction decreases, the neutral point shifts away from the entrance and toward the exit in order to maintain a net pull force in the direction of rolling. Otherwise, with low friction, the work would slip rather than pass between the rolls. An approximation of the results obtained by Eq. (19.9) can be calculated based on the average flow stress experienced by the work material in the roll gap. That is, F ¼ Y f wL FIGURE 19.3 Side view of flat rolling, indicating before and after thicknesses, work velocities, angle of contact with rolls, and other features. ð19:10Þ E1C19 11/11/2009 400 16:35:35 Page 400 Chapter 19/Bulk Deformation Processes in Metal Working FIGURE 19.4 Typical variation in pressure along the contact length in flat rolling. The peak pressure is located at the neutral point. The area beneath the curve, representing the integration in Eq. (19.9), is the roll force F. where Y f ¼ average flow stress from Eq. (19.7), MPa (lb/in2); and the product wL is the roll-work contact area, mm2 (in2). Contact length can be approximated by L¼ qffiffiffiffiffiffiffiffiffiffiffiffiffiffiffiffiffiffiffiffi R(to  tf ) ð19:11Þ The torque in rolling can be estimated by assuming that the roll force is centered on the work as it passes between the rolls, and that it acts with a moment arm of one-half the contact length L. Thus, torque for each roll is T ¼ 0:5 FL ð19:12Þ The power required to drive each roll is the product of torque and angular velocity. Angular velocity is 2pN, where N ¼ rotational speed of the roll. Thus, the power for each roll is 2pNT. Substituting Eq. (19.12) for torque in this expression for power, and doubling the value to account for the fact that a rolling mill consists of two powered rolls, we get the following expression: P ¼ 2pNFL ð19:13Þ where P ¼ power, J/s or W (in-lb/min); N ¼ rotational speed, 1/s (rev/min); F ¼ rolling force, N (lb); and L ¼ contact length, m (in). Example 19.1 Flat Rolling A 300-mm-wide strip 25-mm thick is fed through a rolling mill with two powered rolls each of radius ¼ 250 mm. The work thickness is to be reduced to 22 mm in one pass at a roll speed of 50 rev/min. The work material has a flow curve defined by K ¼ 275 MPa and n ¼ 0.15, and the coefficient of friction between the rolls and the work is assumed to be 0.12. Determine if the friction is sufficient to permit the rolling operation to be accomplished. If so, calculate the roll force, torque, and horsepower. Solution: The draft attempted in this rolling operation is d ¼ 25  22 ¼ 3 mm E1C19 11/11/2009 16:35:35 Page 401 Section 19.1/Rolling 401 From Eq. (19.8), the maximum possible draft for the given coefficient of friction is dmax ¼ (0:12)2 (250) ¼ 3:6 mm Since the maximum allowable draft exceeds the attempted reduction, the rolling operation is feasible. To compute rolling force, we need the contact length L and the average flow stress Y f . The contact length is given by Eq. (19.11): pffiffiffiffiffiffiffiffiffiffiffiffiffiffiffiffiffiffiffiffiffiffiffiffiffiffiffi L ¼ 250(25  22) ¼ 27:4 mm Y f is determined from the true strain: e ¼ ln Yf ¼ 25 ¼ 0:128 22 275(0:128)0:15 ¼ 175:7 MPa 1:15 Rolling force is determined from Eq. (19.10): F ¼ 175:7(300)(27:4) ¼ 1; 444; 786 N Torque required to drive each roll is given by Eq. (19.12): T ¼ 0:5(1; 444; 786)(27; 4)(103 ) ¼ 19; 786 N-m and the power is obtained from Eq. (19.13): P ¼ 2p(50)(1; 444; 786)(27:4)(103 ) ¼ 12; 432; 086 N-m/min ¼ 207; 201 N-m/s(W) For comparison, let us convert this to horsepower (we note that one horsepower ¼ 745.7 W): HP ¼ 207; 201 ¼ 278 hp 745:7 n It can be seen from this example that large forces and power are required in rolling. Inspection of Eqs. (19.10) and (19.13) indicates that force and/or power to roll a strip of a given width and work material can be reduced by any of the following: (1) using hot rolling rather than cold rolling to reduce strength and strain hardening (K and n) of the work material; (2) reducing the draft in each pass; (3) using a smaller roll radius R to reduce force; and (4) using a lower rolling speed N to reduce power. 19.1.2 SHAPE ROLLING In shape rolling, the work is deformed into a contoured cross section. Products made by shape rolling include construction shapes such as I-beams, L-beams, and U-channels; rails for railroad tracks; and round and square bars and rods (see Figure 19.2). The process is accomplished by passing the work through rolls that have the reverse of the desired shape. Most of the principles that apply in flat rolling are also applicable to shape rolling. Shaping rolls are more complicated; and the work, usually starting as a square shape, requires a gradual transformation through several rolls in order to achieve the final cross section. Designing the sequence of intermediate shapes and corresponding rolls is called roll-pass design. Its goal is to achieve uniform deformation throughout the cross section in each reduction. Otherwise, certain portions of the work are reduced more than others, causing greater elongation in these sections. The consequence of nonuniform reduction can E1C19 11/11/2009 402 16:35:35 Page 402 Chapter 19/Bulk Deformation Processes in Metal Working be warping and cracking of the rolled product. Both horizontal and vertical rolls are utilized to achieve consistent reduction of the work material. 19.1.3 ROLLING MILLS Various rolling mill configurations are available to deal with the variety of applications and technical problems in the rolling process. The basic rolling mill consists of two opposing rolls and is referred to as a two-high rolling mill, shown in Figure 19.5(a). The rolls in these mills have diameters in the range of 0.6 to 1.4 m (2.0–4.5 ft). The two-high configuration can be either reversing or nonreversing. In the nonreversing mill, the rolls always rotate in the same direction, and the work always passes through from the same side. The reversing mill allows the direction of roll rotation to be reversed, so that the work can be passed through in either direction. This permits a series of reductions to be made through the same set of rolls, simply by passing through the work from opposite directions multiple times. The disadvantage of the reversing configuration is the significant angular momentum possessed by large rotating rolls and the associated technical problems involved in reversing the direction. Several alternative arrangements are illustrated in Figure 19.5. In the three-high configuration, Figure 19.5(b), there are three rolls in a vertical column, and the direction of rotation of each roll remains unchanged. To achieve a series of reductions, the work can be passed through from either side by raising or lowering the strip after each pass. The equipment in a three-high rolling mill becomes more complicated, because an elevator mechanism is needed to raise and lower the work. As several of the previous equations indicate, advantages are gained in reducing roll diameter. Roll-work contact length is reduced with a lower roll radius, and this leads to lower forces, torque, and power. The four-high rolling mill uses two smaller-diameter rolls to contact the work and two backing rolls behind them, as in Figure 19.5(c). Owing to the high roll forces, these smaller rolls would deflect elastically between their end bearings as the work passes through unless the larger backing rolls were used to support them. Another FIGURE 19.5 Various configurations of rolling mills: (a) 2-high, (b) 3-high, (c) 4-high, (d) cluster mill, and (e) tandem rolling mill. E1C19 11/11/2009 16:35:35 Page 403 Section 19.2/Other Deformation Processes Related to Rolling 403 roll configuration that allows smaller working rolls against the work is the cluster rolling mill (Figure 19.5(d)). To achieve higher throughput rates in standard products, a tandem rolling mill is often used. This configuration consists of a series of rolling stands, as represented in Figure 19.5(e). Although only three stands are shown in our sketch, a typical tandem rolling mill may have eight or ten stands, each making a reduction in thickness or a refinement in shape of the work passing through. With each rolling step, work velocity increases, and the problem of synchronizing the roll speeds at each stand is a significant one. Modern tandem rolling mills are often supplied directly by continuous casting operations (Section 7.2.2). These setups achieve a high degree of integration among the processes required to transform starting raw materials into finished products. Advantages include elimination of soaking pits, reduction in floor space, and shorter manufacturing lead times. These technical advantages translate into economic benefits for a mill that can accomplish continuous casting and rolling. 19.2 OTHER DEFORMATION PROCESSES RELATED TO ROLLING Several other bulk deformation processes use rolls to form the workpart. The operations include thread rolling, ring rolling, gear rolling, and roll piercing. Thread Rolling Thread rolling is used to form threads on cylindrical parts by rolling them between two dies. It is the most important commercial process for mass producing external threaded components (e.g., bolts and screws). The competing process is thread cutting (Section 22.7.1). Most thread rolling operations are performed by cold working in thread rolling machines. These machines are equipped with special dies that determine the size and form of the thread. The dies are of two types: (1) flat dies, which reciprocate relative to each other, as illustrated in Figure 19.6; and (2) round dies, which rotate relative to each other to accomplish the rolling action. Production rates in thread rolling can be high, ranging up to eight parts per second for small bolts and screws. Not only are these rates significantly higher than thread cutting, but there are other advantages over machining as well: (1) better material utilization, (2) stronger threads due to work hardening, (3) smoother surface, and (4) better fatigue resistance due to compressive stresses introduced by rolling. Ring Rolling Ring rolling is a deformation process in which a thick-walled ring of smaller diameter is rolled into a thin-walled ring of larger diameter. The before and after views of the FIGURE 19.6 Thread rolling with flat dies: (1) start of cycle and (2) end of cycle. E1C19 11/11/2009 404 16:35:35 Page 404 Chapter 19/Bulk Deformation Processes in Metal Working FIGURE 19.7 Ring rolling used to reduce the wall thickness and increase the diameter of a ring: (1) start and (2) completion of process. process are illustrated in Figure 19.7. As the thick-walled ring is compressed, the deformed material elongates, causing the diameter of the ring to be enlarged. Ring rolling is usually performed as a hot-working process for large rings and as a cold-working process for smaller rings. Applications of ring rolling include ball and roller bearing races, steel tires for railroad wheels, and rings for pipes, pressure vessels, and rotating machinery. The ring walls are not limited to rectangular cross sections; the process permits rolling of more complex shapes. Thereareseveraladvantagesofringrollingoveralternativemethodsofmakingthesameparts: raw material savings, ideal grain orientation for the application, and strengthening through cold working. Gear Rolling Gear rolling is a cold working process to produce certain gears. The automotive industry is an important user of these products. The setup in gear rolling is similar to thread rolling, except that the deformed features of the cylindrical blank or disk are oriented parallel to its axis (or at an angle in the case of helical gears) rather than spiraled as in thread rolling. Alternative production methods for gears include several machining operations, discussed in Section 22.7.2. Advantages of gear rolling compared to machining are similar to those of thread rolling: higher production rates, better strength and fatigue resistance, and less material waste. Roll Piercing Ring rolling is a specialized hot working process for making seamless thick-walled tubes. It utilizes two opposing rolls, and hence it is grouped with the rolling processes. The process is based on the principle that when a solid cylindrical part is compressed on its circumference, as in Figure 19.8(a), high tensile stresses are FIGURE 19.8 Roll piercing: (a) formation of internal stresses and cavity by compression of cylindrical part; and (b) setup of Mannesmann roll mill for producing seamless tubing. E1C19 11/11/2009 16:35:36 Page 405 Section 19.3/Forging 405 developed at its center. If compression is high enough, an internal crack is formed. In roll piercing, this principle is exploited by the setup shown in Figure 19.8(b). Compressive stresses on a solid cylindrical billet are applied by two rolls, whose axes are oriented at slight angles (6 ) from the axis of the billet, so that their rotation tends to pull the billet through the rolls. A mandrel is used to control the size and finish of the hole created by the action. The terms rotary tube piercing and Mannesmann process are also used for this tube-making operation. 19.3 FORGING Forging is a deformation process in which the work is compressed between two dies, using either impact or gradual pressure to form the part. It is the oldest of the metal forming operations, dating back to perhaps 5000 BCE (Historical Note 19.2). Today, forging is an important industrial process used to make a variety of high-strength components for automotive, aerospace, and other applications. These components include engine crankshafts and connecting rods, gears, aircraft structural components, and jet engine turbine parts. In addition, steel and other basic metals industries use forging to establish the basic form of large components that are subsequently machined to final shape and dimensions. Historical Note 19.2 T Forging he forging process dates from the earliest written records of man, around 7000 years ago. There is evidence that forging was used in ancient Egypt, Greece, Persia, India, China, and Japan to make weapons, jewelry, and a variety of implements. Craftsmen in the art of forging during these times were held in high regard. Engraved stone platens were used as impression dies in the hammering of gold and silver in ancient Crete around 1600 BCE. This evolved into the fabrication of coins by a similar process around 800 BCE. More complicated impression dies were used in Rome around 200 CE. The blacksmith’s trade remained relatively unchanged for many centuries until the drop hammer with guided ram was introduced near the end of the eighteenth century. This development brought forging practice into the Industrial Age. Forging is carried out in many different ways. One way to classify the operations is by working temperature. Most forging operations are performed hot or warm, owing to the significant deformation demanded by the process and the need to reduce strength and increase ductility of the work metal. However, cold forging is also very common for certain products. The advantage of cold forging is the increased strength that results from strain hardening of the component. Either impact or gradual pressure is used in forging. The distinction derives more from the type of equipment used than differences in process technology. A forging machine that applies an impact load is called a forging hammer, while one that applies gradual pressure is called a forging press. Another difference among forging operations is the degree to which the flow of the work metal is constrained by the dies. By this classification, there are three types of forging operations, shown in Figure 19.9: (a) open-die forging, (b) impression-die forging, and (c) flashless forging. In open-die forging, the work is compressed between two flat (or almost flat) dies, thus allowing the metal to flow without constraint in a lateral direction relative to the die surfaces. In impression-die forging, the die surfaces contain a shape or impression that is imparted to the work during compression, thus constraining metal flow to a significant degree. In this type of operation, a portion of the work metal flows beyond the E1C19 11/11/2009 406 16:35:36 Page 406 Chapter 19/Bulk Deformation Processes in Metal Working FIGURE 19.9 Three types of forging operation illustrated by cross-sectional sketches: (a) open-die forging, (b) impression-die forging, and (c) flashless forging. die impression to form flash, as shown in the figure. Flash is excess metal that must be trimmed off later. In flashless forging, the work is completely constrained within the die and no excess flash is produced. The volume of the starting workpiece must be controlled very closely so that it matches the volume of the die cavity. The reader can obtain a good sense of these operations in our video clip on forging. VIDEO CLIP Forging. The three segments on this clip are (1) the forging process, (2) open-die forging, and (3) impression-die forging. 19.3.1 OPEN-DIE FORGING The simplest case of open-die forging involves compression of a workpart of cylindrical cross section between two flat dies, much in the manner of a compression test (Section 3.1.2). This forging operation, known as upsetting or upset forging, reduces the height of the work and increases its diameter. Analysis of Open-Die Forging If open-die forging is carried out under ideal conditions of no friction between work and die surfaces, then homogeneous deformation occurs, and the radial flow of the material is uniform throughout its height, as pictured in Figure 19.10. Under these ideal conditions, the true strain experienced by the work during the process can be determined by e ¼ ln ho h ð19:14Þ E1C19 11/11/2009 16:35:36 Page 407 Section 19.3/Forging 407 FIGURE 19.10 Homogeneous deformation of a cylindrical workpart under ideal conditions in an open-die forging operation: (1) start of process with workpiece at its original length and diameter, (2) partial compression, and (3) final size. where ho ¼ starting height of the work, mm (in); and h ¼ the height at some intermediate point in the process, mm (in). At the end of the compression stroke, h ¼ its final value hf, and the true strain reaches its maximum value. Estimates of force to perform upsetting can be calculated. The force required to continue the compression at any given height h during the process can be obtained by multiplying the corresponding cross-sectional area by the flow stress: F ¼ Yf A ð19:15Þ where F ¼ force, lb (N); A ¼ cross-sectional area of the part, mm2 (in2); and Yf ¼ flow stress corresponding to the strain given by Eq. (19.14), MPa (lb/in2). Area A continuously increases during the operation as height is reduced. Flow stress Yf also increases as a result of work hardening, except when the metal is perfectly plastic (e.g., in hot working). In this case, the strain-hardening exponent n ¼ 0, and flow stress Yf equals the metal’s yield strength Y. Force reaches a maximum value at the end of the forging stroke, when both area and flow stress are at their highest values. An actual upsetting operation does not occur quite as shown in Figure 19.10 because friction opposes the flow of work metal at the die surfaces. This creates the barreling effect shown in Figure 19.11. When performed on a hot workpart with cold dies, the barreling effect is even more pronounced. This results from a higher coefficient of friction typical in hot working and heat transfer at and near the die surfaces, which cools the metal and increases its resistance to deformation. The hotter metal in the middle of the part flows more readily than the cooler metal at the ends. These effects are more significant as the diameter- FIGURE 19.11 Actual deformation of a cylindrical workpart in open-die forging, showing pronounced barreling: (1) start of process, (2) partial deformation, and (3) final shape. E1C19 11/11/2009 408 16:35:36 Page 408 Chapter 19/Bulk Deformation Processes in Metal Working to-height ratio of the workpart increases, due to the greater contact area at the work–die interface. All of these factors cause the actual upsetting force to be greater than what is predicted by Eq. (19.15). As an approximation, we can apply a shape factor to Eq. (19.15) to account for effects of the D/h ratio and friction: F ¼ Kf Y f A ð19:16Þ where F, Yf, and A have the same definitions as in the previous equation; and Kf is the forging shape factor, defined as Kf ¼ 1 þ 0:4 mD h ð19:17Þ where m ¼ coefficient of friction; D ¼ workpart diameter or other dimension representing contact length with die surface, mm (in); and h ¼ workpart height, mm (in). Example 19.2 Open-Die Forging A cylindrical workpiece is subjected to a cold upset forging operation. The starting piece is 75 mm in height and 50 mm in diameter. It is reduced in the operation to a height of 36 mm. The work material has a flow curve defined by K ¼ 350 MPa and n ¼ 0.17. Assume a coefficient of friction of 0.1. Determine the force as the process begins, at intermediate heights of 62 mm, 49 mm, and at the final height of 36 mm. Solution: Workpiece volume V ¼ 75p(502=4) ¼ 147,262 mm3. At the moment contact is made by the upper die, h ¼ 75 mm and the force F ¼ 0. At the start of yielding, h is slightly less than 75 mm, and we assume that strain ¼ 0.002, at which the flow stress is Y f ¼ Ken ¼ 350(0:002)0:17 ¼ 121:7 MPa The diameter is still approximately D ¼ 50 mm and area A ¼ p(502=4) ¼ 1963.5 mm2. For these conditions, the adjustment factor Kf is computed as Kf ¼ 1 þ 0:4(0:1)(50) ¼ 1:027 75 The forging force is F ¼ 1:027(121:7)(1963:5) ¼ 245; 410 MPa At h ¼ 62 mm, e ¼ ln 75 ¼ ln(1:21) ¼ 0:1904 62 Y f ¼ 350(0:1904)17 ¼ 264:0 MPa Assuming constant volume, and neglecting barreling, rffiffiffiffiffiffiffiffiffiffiffiffiffiffiffiffiffiffiffiffi 4ð2375:2Þ 2 A ¼ 147; 262=62 ¼ 2375:2 mm and D ¼ ¼ 55:0 mm p 0:4ð0:1Þð55Þ Kf ¼ 1 þ ¼ 1:035 62 F ¼ 1:035(264)(2375:2) ¼ 649; 303 N Similarly, at h ¼ 49 mm, F ¼ 955,642 N; and at h ¼ 36 mm, F ¼ 1,467,422 N. The loadn stroke curve in Figure 19.12 was developed from the values in this example. Open-Die Forging Practice Open-die hot forging is an important industrial process. Shapes generated by open-die operations are simple; examples include shafts, disks, and 11/11/2009 16:35:36 Page 409 Section 19.3/Forging 409 1500 Forging force (1000 N) E1C19 FIGURE 19.12 Upsetting force as a function of height h and height reduction (ho  h). This plot is sometimes called the load stroke curve. 1000 500 0 75 62 49 36 h (mm) 0 13 26 39 (ho – h) rings. In some applications, the dies have slightly contoured surfaces that help to shape the work. In addition, the work must often be manipulated (e.g., rotating in steps) to effect the desired shape change. Skill of the human operator is a factor in the success of these operations. An example of open-die forging in the steel industry is the shaping of a large square cast ingot into a round cross section. Open-die forging operations produce rough forms, and subsequent operations are required to refine the parts to final geometry and dimensions. An important contribution of open-die hot-forging is that it creates a favorable grain flow and metallurgical structure in the metal. Operations classified as open-die forging or related operations include fullering, edging, and cogging, illustrated in Figure 19.13. Fullering is a forging operation performed to reduce the cross section and redistribute the metal in a workpart in preparation for subsequent shape forging. It is accomplished by dies with convex surfaces. Fullering die cavities are often designed into multi-cavity impression dies, so that the starting bar can be rough formed before final shaping. Edging is similar to fullering, except that the dies have concave surfaces. A cogging operation consists of a sequence of forging compressions along the length of a workpiece to reduce cross section and increase length. It is used in the steel industry to produce blooms and slabs from cast ingots. It is accomplished using open dies with flat or slightly contoured surfaces. The term incremental forging is sometimes used for this process. 19.3.2 IMPRESSION-DIE FORGING Impression-die forging, sometimes called closed-die forging, is performed with dies that contain the inverse of the desired shape of the part. The process is illustrated in a three-step sequence in Figure 19.14. The raw workpiece is shown as a cylindrical part similar to that used in the previous open-die operation. As the die closes to its final position, flash is formed by metal that flows beyond the die cavity and into the small gap between the die plates. Although this flash must be cut away from the part in a subsequent trimming operation, it actually serves an important function during impression-die forging. As the flash begins to form in the E1C19 11/11/2009 410 16:35:36 Page 410 Chapter 19/Bulk Deformation Processes in Metal Working FIGURE 19.13 Several open-die forging operations: (a) fullering, (b) edging, and (c) cogging. die gap, friction resists continued flow of metal into the gap, thus constraining the bulk of the work material to remain in the die cavity. In hot forging, metal flow is further restricted because the thin flash cools quickly against the die plates, thereby increasing its resistance to deformation. Restricting metal flow in the gap causes the compression pressures on the part to increase significantly, thus forcing the material to fill the sometimes intricate details of the die cavity to ensure a high-quality product. FIGURE 19.14 Sequence in impression-die forging: (1) just prior to initial contact with raw workpiece, (2) partial compression, and (3) final die closure, causing flash to form in gap between die plates. E1C19 11/11/2009 16:35:37 Page 411 411 Section 19.3/Forging TABLE 19.1 Typical Kf values for various part shapes in impression-die and flashless forging. Part Shape Kf Part Shape Impression-die forging: Simple shapes with flash Complex shapes with flash Very complex shapes with flash 6.0 8.0 10.0 Kf Flashless forging: Coining (top and bottom surfaces) Complex shapes 6.0 8.0 Several forming steps are often required in impression-die forging to transform the starting blank into the desired final geometry. Separate cavities in the die are needed for each step. The beginning steps are designed to redistribute the metal in the workpart to achieve a uniform deformation and desired metallurgical structure in the subsequent steps. The final steps bring the part to its final geometry. In addition, when drop forging is used, several blows of the hammer may be required for each step. When impression-die drop forging is done manually, as it often is, considerable operator skill is required under adverse conditions to achieve consistent results. Because of flashformation inimpression-dieforging and the more complex partshapes made with these dies, forces in this process are significantly greater and more difficult to analyzethaninopen-dieforging. Relatively simpleformulas anddesign factors areoftenused to estimate forces in impression-die forging. The force formula is the same as previous Eq. (19.16) for open-die forging, but its interpretation is slightly different: F ¼ Kf Y f A ð19:18Þ where F ¼ maximum force in the operation, N (lb); A ¼ projected area of the part including flash, mm2 (in2); Yf ¼ flow stress of the material, MPa (lb/in2); and Kf ¼ forging shape factor. In hot forging, the appropriate value of Yf is the yield strength of the metal at the elevated temperature. In other cases, selecting the proper value of flow stress is difficult because the strain varies throughout the workpiece for complex shapes. Kf in Eq. (19.18) is a factor intended to account for increases in force required to forge part shapes of various complexities. Table 19.1 indicates the range of values of Kf for different part geometries. Obviously, the problem of specifying the proper Kf value for a given workpart limits the accuracy of the force estimate. Eq. (19.18) applies to the maximum force during the operation, since this is the load that will determine the required capacity of the press or hammer used in the operation. The maximum force is reached at the end of the forging stroke, when the projected area is greatest and friction is maximum. Impression-die forging is not capable of close tolerance work, and machining is often required to achieve the accuracies needed. The basic geometry of the part is obtained from the forging process, with machining performed on those portions of the part that require precision finishing (e.g., holes, threads, and surfaces that mate with other components). The advantages of forging, compared to machining the part completely, are higher production rates, conservation of metal, greater strength, and favorable grain orientation of the metal that results from forging. A comparison of the grain flow in forging and machining is illustrated in Figure 19.15. Improvements in the technology of impression-die forging have resulted in the capability to produce forgings with thinner sections, more complex geometries, drastic reductions in draft requirements on the dies, closer tolerances, and the virtual elimination of machining allowances. Forging processes with these features are known as precision forging. Common work metals used for precision forging include aluminum and titanium. A comparison of precision and conventional impression-die forging is presented in Figure 19.16. Note that precision forging in this example does not eliminate flash, although it reduces it. E1C19 11/11/2009 412 16:35:37 Page 412 Chapter 19/Bulk Deformation Processes in Metal Working FIGURE 19.15 Comparison of metal grain flow in a part that is: (a) hot forged with finish machining, and (b) machined complete. Some precision forging operations are accomplished without producing flash. Depending on whether machining is required to finish the part geometry, precision forgings are properly classified as near net shape or net shape processes. 19.3.3 FLASHLESS FORGING As mentioned above, impression-die forging is sometimes called closed-die forging in industry terminology. However, there is a technical distinction between impression-die forging and true closed-die forging. The distinction is that in closed-die forging, the raw workpiece is completely contained within the die cavity during compression, and no flash is formed. The process sequence is illustrated in Figure 19.17. The term flashless forging is appropriate to identify this process. Flashless forging imposes requirements on process control that are more demanding than impression-die forging. Most important is that the work volume must equal the space in the die cavity within a very close tolerance. If the starting blank is too large, excessive pressures may cause damage to the die or press. If the blank is too small, the cavity will not be filled. Because of the special demands made by flashless forging, the process lends itself best to part geometries that are usually simple and symmetrical, and to work materials such as aluminum and magnesium and their alloys. Flashless forging is often classified as a precision forging process [5]. Forces in flashless forging reach values comparable to those in impression-die forging. Estimates of these forces can be computed using the same methods as for impression-die forging: Eq. (19.18) and Table 19.1. Coining is a special application of closed-die forging in which fine details in the die are impressed into the top and bottom surfaces of the workpart. There is little flow of metal in coining, yet the pressures required to reproduce the surface details in the die cavity are high, as indicated by the value of Kf in Table 19.1. A common application of FIGURE 19.16 Cross sections of (a) conventional- and (b) precision forgings. Dashed lines in (a) indicate subsequent machining required to make the conventional forging equivalent in geometry to the precision forging. In both cases, flash extensions must be trimmed. E1C19 11/11/2009 16:35:37 Page 413 Section 19.3/Forging 413 FIGURE 19.17 Flashless forging: (1) just before initial contact with workpiece, (2) partial compression, and (3) final punch and die closure. Symbols v and F indicate motion (v ¼ velocity) and applied force, respectively. coining is, of course, in the minting of coins, shown in Figure 19.18. The process is also used to provide good surface finish and dimensional accuracy on workparts made by other operations. 19.3.4 FORGING HAMMERS, PRESSES, AND DIES Equipment used in forging consists of forging machines, classified as hammers or presses, and forging dies, which are the special tooling used in these machines. In addition, auxiliary equipment is needed, such as furnaces to heat the work, mechanical devices to load and unload the work, and trimming stations to cut away the flash in impression-die forging. Forging Hammers Forging hammers operate by applying an impact loading against the work. The term drop hammer is often used for these machines, owing to the means of delivering impact energy (see Figures 19.19 and 19.20). Drop hammers are most frequently FIGURE 19.18 Coining operation: (1) start of cycle, (2) compression stroke, and (3) ejection of finished part. E1C19 11/11/2009 414 16:35:37 Page 414 Chapter 19/Bulk Deformation Processes in Metal Working FIGURE 19.19 Drop forging hammer, fed by conveyor and heating units at the right of the scene. (Photo courtesy of Chambersburg Engineering Company, Chambersburg, Pennsylvania) used for impression-die forging. The upper portion of the forging die is attached to the ram, and the lower portion is attached to the anvil. In the operation, the work is placed on the lower die, and the ram is lifted and then dropped. When the upper die strikes the work, the impact energy causes the part to assume the form of the die cavity. Several blows of the hammer are often required to achieve the desired change in shape. Drop hammers can be classified as gravity drop hammers and power drop hammers. Gravity drop hammers achieve their energy by the falling weight of a heavy ram. The force of the blow is determined by the height of the drop and the weight of the ram. Power drop hammers accelerate the ram by pressurized air or steam. One of the disadvantages of drop hammers is that a large amount of the impact energy is transmitted through the anvil and into the floor of the building. Head (containing cylinder) Piston rod Frame Ram Anvil FIGURE 19.20 Diagram showing details of a drop hammer for impression-die forging. E1C19 11/11/2009 16:35:37 Page 415 Section 19.3/Forging 415 FIGURE 19.21 Terminology for a conventional impressiondie in forging. Forging Presses Presses apply gradual pressure, rather than sudden impact, to accomplish the forging operation. Forging presses include mechanical presses, hydraulic presses, and screw presses. Mechanical presses operate by means of eccentrics, cranks, or knuckle joints, which convert the rotating motion of a drive motor into the translation motion of the ram. These mechanisms are very similar to those used in stamping presses (Section 20.5.2). Mechanical presses typically achieve very high forces at the bottom of the forging stroke. Hydraulic presses use a hydraulically driven piston to actuate the ram. Screw presses apply force by a screw mechanism that drives the vertical ram. Both screw drive and hydraulic drive operate at relatively low ram speeds and can provide a constant force throughout the stroke. These machines are therefore suitable for forging (and other forming) operations that require a long stroke. Forging Dies Proper die design is important in the success of a forging operation. Parts to be forged must be designed based on knowledge of the principles and limitations of this process. Our purpose here is to describe some of the terminology and guidelines used in the design of forgings and forging dies. Design of open dies is generally straightforward because the dies are relatively simple in shape. Our comments apply to impression dies and closed dies. Figure 19.21 defines some of the terminology in an impression die. We indicate some of the principles and limitations that must be considered in the part design or in the selection of forging as the manufacturing process to make the part in the following discussion of forging die terminology [5]: å Parting line. The parting line is the plane that divides the upper die from the lower die. Called the flash line in impression-die forging, it is the plane where the two die halves meet. Its selection by the designer affects grain flow in the part, required load, and flash formation. å Draft. Draft is the amount of taper on the sides of the part required to remove it from the die. The term also applies to the taper on the sides of the die cavity. Typical draft angles are 3 on aluminum and magnesium parts and 5 to 7 on steel parts. Draft angles on precision forgings are near zero. å Webs and ribs. A web is a thin portion of the forging that is parallel to the parting line, while a rib is a thin portion that is perpendicular to the parting line. These part features cause difficulty in metal flow as they become thinner. å Fillet and corner radii. Fillet and corner radii are illustrated in Figure 19.21. Small radii tend to limit metal flow and increase stresses on die surfaces during forging. å Flash. Flash formation plays a critical role in impression-die forging by causing pressure buildup inside the die to promote filling of the cavity. This pressure buildup E1C19 11/11/2009 416 16:35:37 Page 416 Chapter 19/Bulk Deformation Processes in Metal Working is controlled by designing a flash land and gutter into the die, as pictured in Figure 19.21. The land determines the surface area along which lateral flow of metal occurs, thereby controlling the pressure increase inside the die. The gutter permits excess metal to escape without causing the forging load to reach extreme values. 19.4 OTHER DEFORMATION PROCESSES RELATED TO FORGING In addition to the conventional forging operations discussed in the preceding sections, other metal forming operations are closely associated with forging. Upsetting and Heading Upsetting (also called upset forging) is a deformation operation in which a cylindrical workpart is increased in diameter and reduced in length. This operation was analyzed in our discussion of open-die forging (Section 19.3.1). However, as an industrial operation, it can also be performed as closed-die forging, as seen in Figure 19.22. Upsetting is widely used in the fastener industry to form heads on nails, bolts, and similar hardware products. In these applications, the term heading is often used to denote the operation. Figure 19.23 illustrates a variety of heading applications, indicating various possible die configurations. Owing to these types of applications, more parts are produced FIGURE 19.22 An upset forging operation to form a head on a bolt or similar hardware item. The cycle is as follows: (1) wire stock is fed to the stop; (2) gripping dies close on the stock and the stop is retracted; (3) punch moves forward; and (4) bottoms to form the head. FIGURE 19.23 Examples of heading (upset forging) operations: (a) heading a nail using open dies, (b) round head formed by punch, (c) and (d) heads formed by die, and (e) carriage bolt head formed by punch and die. E1C19 11/11/2009 16:35:38 Page 417 Section 19.4/Other Deformation Processes Related to Forging 417 FIGURE 19.24 Swaging process to reduce solid rod stock; the dies rotate as they hammer the work. In radial forging, the workpiece rotates while the dies remain in a fixed orientation as they hammer the work. by upsetting than by any other forging operation. It is performed as a mass-production operation—cold, warm, or hot—on special upset forging machines, called headers or formers. These machines are usually equipped with horizontal slides, rather than vertical slides as in conventional forging hammers and presses. Long wire or bar stock is fed into the machines, the end of the stock is upset forged, and then the piece is cut to length to make the desired hardware item. For bolts and screws, thread rolling (Section 19.2) is used to form the threads. There are limits on the amount of deformation that can be achieved in upsetting, usually defined as the maximum length of stock to be forged. The maximum length that can be upset in one blow is three times the diameter of the starting stock. Otherwise, the metal bends or buckles instead of compressing properly to fill the cavity. Swaging and Radial Forging Swaging and radial forging are forging processes used to reduce the diameter of a tube or solid rod. Swaging is often performed on the end of a workpiece to create a tapered section. The swaging process, shown in Figure 19.24, is accomplished by means of rotating dies that hammer a workpiece radially inward to taper it as the piece is fed into the dies. Figure 19.25 illustrates some of the shapes and products that are made by swaging. A mandrel is sometimes required to control the shape and size of the internal diameter of tubular parts that are swaged. Radial forging is similar to swaging in its action against the work and is used to create similar part shapes. The difference is that in radial forging the dies do not rotate around the workpiece; instead, the work is rotated as it feeds into the hammering dies. Roll Forging Roll forging is a deformation process used to reduce the cross section of a cylindrical (or rectangular) workpiece by passing it through a set of opposing rolls that have grooves matching the desired shape of the final part. The typical operation is illustrated in Figure 19.26. Roll forging is generally classified as a forging process even though it utilizes rolls. The rolls do not turn continuously in roll forging, but rotate through only a portion of one revolution corresponding to the desired deformation to be accomplished on the part. FIGURE 19.25 Examples of parts made by swaging: (a) reduction of solid stock, (b) tapering a tube, (c) swaging to form a groove on a tube, (d) pointing of a tube, and (e) swaging of neck on a gas cylinder. E1C19 11/11/2009 418 16:35:38 Page 418 Chapter 19/Bulk Deformation Processes in Metal Working FIGURE 19.26 Roll forging. Roll-forged parts are generally stronger and possess favorable grain structure compared to competing processes such as machining that might be used to produce the same part geometry. Orbital Forging In this process, deformation occurs by means of a cone-shaped upper die that is simultaneously rolled and pressed into the workpart. As illustrated in Figure 19.27, the work is supported on a lower die, which has a cavity into which the work is compressed. Because the axis of the cone is inclined, only a small area of the work surface is compressed at any moment. As the upper die revolves, the area under compression also revolves. These operating characteristics of orbital forging result in a substantial reduction in press load required to accomplish deformation of the work. FIGURE 19.27 Orbital forging. At end of deformation cycle, lower die lifts to eject part. E1C19 11/11/2009 16:35:38 Page 419 Section 19.4/Other Deformation Processes Related to Forging 419 FIGURE 19.28 Hubbing: (1) before deformation, and (2) as the process is completed. Note that the excess material formed by the penetration of the hub must be machined away. Hubbing Hubbing is a deformation process in which a hardened steel form is pressed into a soft steel (or other soft metal) block. The process is often used to make mold cavities for plastic molding and die casting, as sketched in Figure 19.28. The hardened steel form, called the hub, is machined to the geometry of the part to be molded. Substantial pressures are required to force the hub into the soft block, and this is usually accomplished by a hydraulic press. Complete formation of the die cavity in the block often requires several steps—hubbing followed by annealing to recover the work metal from strain hardening. When significant amounts of material are deformed in the block, as shown in our figure, the excess must be machined away. The advantage of hubbing in this application is that it is generally easier to machine the positive form than the mating negative cavity. This advantage is multiplied in cases where more than one cavity are made in the die block. Isothermal Forging Isothermal forging is a term applied to a hot-forging operation in which the workpart is maintained at or near its starting elevated temperature during deformation, usually by heating the forging dies to the same elevated temperature. By avoiding chill of the workpiece on contact with the cold die surfaces as in conventional forging, the metal flows more readily and the force required to perform the process is reduced. Isothermal forging is more expensive than conventional forging and is usually reserved for difficult-to-forge metals, such as titanium and superalloys, and for complex part shapes. The process is sometimes carried out in a vacuum to avoid rapid oxidation of the die material. Similar to isothermal forging is hot-die forging, in which the dies are heated to a temperature that is somewhat below that of the work metal. Trimming Trimming is an operation used to remove flash on the workpart in impression-die forging. In most cases, trimming is accomplished by shearing, as in Figure 19.29, in which a punch forces the work through a cutting die, the blades for which have the profile of the desired part. Trimming is usually done while the work is still hot, which means that a separate trimming press is included at each forging hammer or press. In cases where the work might be damaged by the cutting process, trimming may be done by alternative methods, such as grinding or sawing. E1C19 11/11/2009 420 16:35:38 Page 420 Chapter 19/Bulk Deformation Processes in Metal Working FIGURE 19.29 Trimming operation (shearing process) to remove the flash after impression-die forging. 19.5 EXTRUSION Extrusion is a compression process in which the work metal is forced to flow through a die opening to produce a desired cross-sectional shape. The process can be likened to squeezing toothpaste out of a toothpaste tube. Extrusion dates from around 1800 (Historical Note 19.3). There are several advantages of the modern process: (1) a variety of shapes are possible, especially with hot extrusion; (2) grain structure and strength properties are enhanced in cold and warm extrusion; (3) fairly close tolerances are possible, especially in cold extrusion; and (4) in some extrusion operations, little or no wasted material is created. However, a limitation is that the cross section of the extruded part must be uniform throughout its length. Historical Note 19.3 Extrusion E xtrusion as an industrial process was invented around 1800 in England, during the Industrial Revolution when that country was leading the world in technological innovations. The invention consisted of the first hydraulic press for extruding lead pipes. An important step forward was made in Germany around 1890, when the first horizontal extrusion press was built for extruding metals with higher melting points than lead. The feature that made this possible was the use of a dummy block that separated the ram from the work billet. 19.5.1 TYPES OF EXTRUSION Extrusion is carried out in various ways. One important distinction is between direct extrusion and indirect extrusion. Another classification is by working temperature: cold, warm, or hot extrusion. Finally, extrusion is performed as either a continuous process or a discrete process. Direct versus Indirect Extrusion Direct extrusion (also called forward extrusion) is illustrated in Figure 19.30. A metal billet is loaded into a container, and a ram compresses the material, forcing it to flow through one or more openings in a die at the opposite end of the container. As the ram approaches the die, a small portion of the billet remains that cannot be forced through the die opening. This extra portion, called the butt, is separated from the product by cutting it just beyond the exit of the die. E1C19 11/11/2009 16:35:38 Page 421 Section 19.5/Extrusion FIGURE 19.30 extrusion. 421 Direct One of the problems in direct extrusion is the significant friction that exists between the work surface and the walls of the container as the billet is forced to slide toward the die opening. This friction causes a substantial increase in the ram force required in direct extrusion. In hot extrusion, the friction problem is aggravated by the presence of an oxide layer on the surface of the billet. This oxide layer can cause defects in the extruded product. To address these problems, a dummy block is often used between the ram and the work billet. The diameter of the dummy block is slightly smaller than the billet diameter, so that a narrow ring of work metal (mostly the oxide layer) is left in the container, leaving the final product free of oxides. Hollow sections (e.g., tubes) are possible in direct extrusion by the process setup in Figure19.31.Thestartingbilletispreparedwithaholeparalleltoitsaxis.Thisallowspassageofa mandrel that is attached to the dummy block. As the billet is compressed, the material is forced to flow through the clearance between the mandrel and the die opening. The resulting cross section is tubular. Semi-hollow cross-sectional shapes are usually extruded in the same way. The starting billet in direct extrusion is usually round in cross section, but the final shape is determined by the shape of the die opening. Obviously, the largest dimension of the die opening must be smaller than the diameter of the billet. In indirect extrusion, also called backward extrusion and reverse extrusion, Figure 19.32(a), the die is mounted to the ram rather than at the opposite end of the container. As the ram penetrates into the work, the metal is forced to flow through the clearance in a FIGURE 19.31 (a) Direct extrusion to produce a hollow or semi-hollow cross section; (b) hollow and (c) semi-hollow cross sections. E1C19 11/11/2009 422 16:35:38 Page 422 Chapter 19/Bulk Deformation Processes in Metal Working Container Container F, v v, F Hollow ram Ram Final work shape Die (a) FIGURE 19.32 Work billet Final work shape Die Work billet (b) Indirect extrusion to produce (a) a solid cross section and (b) a hollow cross section. direction opposite to the motion of the ram. Since the billet is not forced to move relative to the container, there is no friction at the container walls, and the ram force is therefore lower than in direct extrusion. Limitations of indirect extrusion are imposed by the lower rigidity of the hollow ram and the difficulty in supporting the extruded product as it exits the die. Indirect extrusion can produce hollow (tubular) cross sections, as in Figure 19.32(b). In this method, the ram is pressed into the billet, forcing the material to flow around the ram and take a cup shape. There are practical limitations on the length of the extruded part that can be made by this method. Support of the ram becomes a problem as work length increases. Hot versus Cold Extrusion Extrusion can be performed either hot or cold, depending on work metal and amount of strain to which it is subjected during deformation. Metals that are typically extruded hot include aluminum, copper, magnesium, zinc, tin, and their alloys. These same metals are sometimes extruded cold. Steel alloys are usually extruded hot, although the softer, more ductile grades are sometimes cold extruded (e.g., low carbon steels and stainless steel). Aluminum is probably the most ideal metal for extrusion (hot and cold), and many commercial aluminum products are made by this process (structural shapes, door and window frames, etc.). Hot extrusion involves prior heating of the billet to a temperature above its recrystallization temperature. This reduces strength and increases ductility of the metal, permitting more extreme size reductions and more complex shapes to be achieved in the process. Additional advantages include reduction of ram force, increased ram speed, and reduction of grain flow characteristics in the final product. Cooling of the billet as it contacts the container walls is a problem, and isothermal extrusion is sometimes used to overcome this problem. Lubrication is critical in hot extrusion for certain metals (e.g., steels), and special lubricants have been developed that are effective under the harsh conditions in hot extrusion. Glass is sometimes used as a lubricant in hot extrusion; in addition to reducing friction, it also provides effective thermal insulation between the billet and the extrusion container. Cold extrusion and warm extrusion are generally used to produce discrete parts, often in finished (or near finished) form. The term impact extrusion is used to indicate high-speed cold extrusion, and this method is described in more detail in Section 19.5.4. Some important advantages of cold extrusion include increased strength due to strain hardening, close tolerances, improved surface finish, absence of oxide layers, and high production rates. Cold extrusion at room temperature also eliminates the need for heating the starting billet. Continuous versus Discrete Processing A true continuous process operates in steady state mode for an indefinite period of time. Some extrusion operations approach this ideal E1C19 11/11/2009 16:35:39 Page 423 Section 19.5/Extrusion 423 by producing very long sections in one cycle, but these operations are ultimately limited by the size of the starting billet that can be loaded into the extrusion container. These processes are more accurately described as semi-continuous operations. In nearly all cases, the long section is cut into smaller lengths in a subsequent sawing or shearing operation. In a discrete extrusion operation, a single part is produced in each extrusion cycle. Impact extrusion is an example of the discrete processing case. 19.5.2 ANALYSIS OF EXTRUSION Let us use Figure 19.33 as a reference in discussing some of the parameters in extrusion. The diagram assumes that both billet and extrudate are round in cross section. One important parameter is the extrusion ratio, also called the reduction ratio. The ratio is defined: Ao rx ¼ ð19:19Þ Af where rx ¼ extrusion ratio; Ao ¼ cross-sectional area of the starting billet, mm2 (in2); and Af ¼ final cross-sectional area of the extruded section, mm2 (in2). The ratio applies for both direct and indirect extrusion. The value of rx can be used to determine true strain in extrusion, given that ideal deformation occurs with no friction and no redundant work: Ao e ¼ lnrx ¼ ln ð19:20Þ Af Under the assumption of ideal deformation (no friction and no redundant work), the pressure applied by the ram to compress the billet through the die opening depicted in our figure can be computed as follows: p ¼ Y f ln rx ð19:21Þ where Y f ¼ average flow stress during deformation, MPa (lb/in2). For convenience, we restate Eq. (18.2) from the previous chapter: Yf ¼ Ken 1þn In fact, extrusion is not a frictionless process, and the previous equations grossly underestimate the strain and pressure in an extrusion operation. Friction exists between the die and the work as the billet squeezes down and passes through the die opening. In direct extrusion, friction also exists between the container wall and the billet surface. The effect of friction is to increase the strain experienced by the metal. Thus, the actual pressure is greater than that given by Eq. (19.21), which assumes no friction. FIGURE 19.33 Pressure and other variables in direct extrusion. E1C19 11/11/2009 424 16:35:39 Page 424 Chapter 19/Bulk Deformation Processes in Metal Working Various methods have been suggested to calculate the actual true strain and associated ram pressure in extrusion [1], [3], [6], [11], [12], and [19]. The following empirical equation proposed by Johnson [11] for estimating extrusion strain has gained considerable recognition: ex ¼ a þ b ln rx ð19:22Þ where ex ¼ extrusion strain; and a and b are empirical constants for a given die angle. Typical values of these constants are: a ¼ 0.8 and b ¼ 1.2 to 1.5. Values of a and b tend to increase with increasing die angle. The ram pressure to perform indirect extrusion can be estimated based on Johnson’s extrusion strain formula as follows: p ¼ Y f ex ð19:23aÞ where Y f is calculated based on ideal strain from Eq. (19.20), rather than extrusion strain in Eq. (19.22). In direct extrusion, the effect of friction between the container walls and the billet causes the ram pressure to be greater than for indirect extrusion. We can write the following expression which isolates the friction force in the direct extrusion container: pf pD2o ¼ mpc pDo L 4 where pf ¼ additional pressure required to overcome friction, MPa (lb/in2); pDo2=4 ¼ billet cross-sectional area, mm2 (in2); m ¼ coefficient of friction at the container wall; pc ¼ pressure of the billet against the container wall, MPa (lb/in2); and pDoL ¼ area of the interface between billet and container wall, mm2 (in2). The right-hand side of this equation indicates the billet-container friction force, and the left-hand side gives the additional ram force to overcome that friction. In the worst case, sticking occurs at the container wall so that friction stress equals shear yield strength of the work metal: mps pDo L ¼ Y s pDo L where Ys ¼ shear yield strength, MPa (lb/in2). If we assume that Y s ¼ Y f =2, then pf reduces to the following: pf ¼ Y f 2L Do Based on this reasoning, the following formula can be used to compute ram pressure in direct extrusion:   2L p ¼ Y f ex þ ð19:23bÞ Do where the term 2L/Do accounts for the additional pressure due to friction at the container– billet interface. L is the portion of the billet length remaining to be extruded, and Do is the original diameter of the billet. Note that p is reduced as the remaining billet length decreases during the process. Typical plots of ram pressure as a function of ram stroke for direct and indirect extrusion are presented in Figure 19.34. Eq. (19.23b) probably overestimates ram pressure. With good lubrication, ram pressures would be lower than values calculated by this equation. Ram force in indirect or direct extrusion is simply pressure p from Eqs. (19.23a) or (19.23b), respectively, multiplied by billet area Ao: F ¼ pAo ð19:24Þ E1C19 11/11/2009 16:35:39 Page 425 Section 19.5/Extrusion 425 FIGURE 19.34 Typical plots of ram pressure versus ram stroke (and remaining billet length) for direct and indirect extrusion. The higher values in direct extrusion result from friction at the container wall. The shape of the initial pressure buildup at the beginning of the plot depends on die angle (higher die angles cause steeper pressure buildups). The pressure increase at the end of the stroke is related to formation of the butt. where F ¼ ram force in extrusion, N (lb). Power required to carry out the extrusion operation is simply P ¼ Fv ð19:25Þ where P ¼ power, J/s (in-lb/min); F ¼ ram force, N (lb); and v ¼ ram velocity, m/s (in/min). Example 19.3 Extrusion Pressures A billet 75 mm long and 25 mm in diameter is to be extruded in a direct extrusion operation with extrusion ratio rx ¼ 4.0. The extrudate has a round cross section. The die angle (halfangle) ¼ 90 . The work metal has a strength coefficient ¼ 415 MPa, and strain-hardening exponent ¼ 0.18. Use the Johnson formula with a ¼ 0.8 and b ¼ 1.5 to estimate extrusion strain. Determine the pressure applied to the end of the billet as the ram moves forward. Solution: Let us examine the ram pressure at billet lengths of L ¼ 75 mm (starting value), L ¼ 50 mm, L ¼ 25 mm, and L ¼ 0. We compute the ideal true strain, extrusion strain using Johnson’s formula, and average flow stress: e ¼ ln rx ¼ ln 4:0 ¼ 1:3863 ex ¼ 0:8 þ 1:5(1:3863) ¼ 2:8795 Yf ¼ 415(1:3863)0:18 ¼ 373 MPa 1:18 L ¼ 75 mm: With a die angle of 90 , the billet metal is assumed to be forced through the die opening almost immediately; thus, our calculation assumes that maximum pressure is reached at the billet length of 75 mm. For die angles less than 90 , the pressure would build to a maximum as in Figure 19.34 as the starting billet is squeezed into the cone-shaped portion of the extrusion die. Using Eq. (19.23b),   75 ¼ 3312 MPa p ¼ 373 2:8795 þ 2 25   50 L ¼ 50 mm: p ¼ 373 2:8795 þ 2 ¼ 2566 MPa 25   25 ¼ 1820 MPa L ¼ 25 mm: p ¼ 373 2:8795 þ 2 25 E1C19 11/11/2009 426 16:35:39 Page 426 Chapter 19/Bulk Deformation Processes in Metal Working L ¼ 0: Zero length is a hypothetical value in direct extrusion. In reality, it is impossible to squeeze all of the metal through the die opening. Instead, a portion of the billet (the ‘‘butt’’) remains unextruded and the pressure begins to increase rapidly as L approaches zero. This increase in pressure at the end of the stroke is seen in the plot of ram pressure versus ram stroke in Figure 19.34. Calculated below is the hypothetical minimum value of ram pressure that would result at L ¼ 0.   0 p ¼ 373 2:8795 þ 2 ¼ 1074 MPa 25 This is also the value of ram pressure that would be associated with indirect extrusion n throughout the length of the billet. 19.5.3 EXTRUSION DIES AND PRESSES Important factors in an extrusion die are die angle and orifice shape. Die angle, more precisely die half-angle, is shown as a in Figure 19.35(a). For low angles, surface area of the die is large, leading to increased friction at the die–billet interface. Higher friction results in larger ram force. On the other hand, a large die angle causes more turbulence in the metal flow during reduction, increasing the ram force required. Thus, the effect of die angle on ram force is a U-shaped function, as in Figure 19.35(b). An optimum die angle exists, as suggested by our hypothetical plot. The optimum angle depends on various factors (e.g., work material, billet temperature, and lubrication) and is therefore difficult to determine for a given extrusion job. Die designers rely on rules of thumb and judgment to decide the appropriate angle. Our previous equations for ram pressure, Eqs. (19.23a), apply to a circular die orifice. The shape of the die orifice affects the ram pressure required to perform an extrusion operation. A complex cross section, such as the one shown in Figure 19.36, requires a higher pressure and greater force than a circular shape. The effect of the die orifice shape can be assessed by the die shape factor, defined as the ratio of the pressure required to extrude a cross section of a given shape relative to the extrusion pressure for a round cross section of the same area. We can express the shape factor as follows:  2:25 Cx ð19:26Þ Kx ¼ 0:98 þ 0:02 Cc where Kx ¼ die shape factor in extrusion; Cx ¼ perimeter of the extruded cross section, mm (in); and Cc ¼ perimeter of a circle of the same area as the extruded shape, mm (in). Eq. FIGURE 19.35 (a) Definition of die angle in direct extrusion; (b) effect of die angle on ram force. E1C19 11/11/2009 16:35:39 Page 427 427 Section 19.5/Extrusion FIGURE 19.36 A complex extruded cross section for a heat sink. (Photo courtesy of Aluminum Company of America, Pittsburg, Pennsylvania.) (19.26) is based on empirical data in Altan et al. [1] over a range of Cx/Cc values from 1.0 to about 6.0. The equation may be invalid much beyond the upper limit of this range. As indicated by Eq. (19.26), the shape factor is a function of the perimeter of the extruded cross section divided by the perimeter of a circular cross section of equal area. A circular shape is the simplest shape, with a value of Kx ¼ 1.0. Hollow, thin-walled sections have higher shape factors and are more difficult to extrude. The increase in pressure is not included in our previous pressure equations, Eqs. (19.23a and 19.23b), which apply only to round cross sections. For shapes other than round, the corresponding expression for indirect extrusion is and for direct extrusion, p ¼ K x Y f ex ð19:27aÞ   2L p ¼ K x Y f ex þ Do ð19:27bÞ where p ¼ extrusion pressure, MPa (lb/in2); Kx ¼ shape factor; and the other terms have the same interpretation as before. Values of pressure given by these equations can be used in Eq. (19.24) to determine ram force. Die materials used for hot extrusion include tool and alloy steels. Important properties of these die materials include high wear resistance, high hot hardness, and high thermal conductivity to remove heat from the process. Die materials for cold extrusion include tool steels and cemented carbides. Wear resistance and ability to retain shape under high stress are desirable properties. Carbides are used when high production rates, long die life, and good dimensional control are required. E1C19 11/11/2009 428 16:35:40 Page 428 Chapter 19/Bulk Deformation Processes in Metal Working Extrusion presses are either horizontal or vertical, depending on orientation of the work axis. Horizontal types are more common. Extrusion presses are usually hydraulically driven. This drive is especially suited to semi-continuous production of long sections, as in direct extrusion. Mechanical drives are often used for cold extrusion of individual parts, such as in impact extrusion. 19.5.4 OTHER EXTRUSION PROCESSES Direct and indirect extrusion are the principal methods of extrusion. Various names are given to operations that are special cases of the direct and indirect methods described here. Other extrusion operations are unique. In this section we examine some of these special forms of extrusion and related processes. Impact Extrusion Impact extrusion is performed at higher speeds and shorter strokes than conventional extrusion. It is used to make individual components. As the name suggests, the punch impacts the workpart rather than simply applying pressure to it. Impacting can be carried out as forward extrusion, backward extrusion, or combinations of these. Some representative examples are shown in Figure 19.37. FIGURE 19.37 Several examples of impact extrusion: (a) forward, (b) backward, and (c) combination of forward and backward. E1C19 11/11/2009 16:35:40 Page 429 Section 19.5/Extrusion 429 FIGURE 19.38 Hydrostatic extrusion. Impact extrusion is usually done cold on a variety of metals. Backward impact extrusion is most common. Products made by this process include toothpaste tubes and battery cases. As indicated by these examples, very thin walls are possible on impact extruded parts. The high-speed characteristics of impacting permit large reductions and high production rates, making this an important commercial process. Hydrostatic Extrusion One of the problems in direct extrusion is friction along the billet– container interface. This problem can be addressed by surrounding the billet with fluid inside the container and pressurizing the fluid by the forward motion of the ram, as in Figure 19.38. This way, there is no friction inside the container, and friction at the die opening is reduced. Consequently, ram force is significantly lower than in direct extrusion. The fluid pressure acting on all surfaces of the billet gives the process its name. It can be carried out at room temperature or at elevated temperatures. Special fluids and procedures must be used at elevated temperatures. Hydrostatic extrusion is an adaptation of direct extrusion. Hydrostatic pressure on the work increases the material’s ductility. Accordingly, this process can be used on metals that would be too brittle for conventional extrusion operations. Ductile metals can also be hydrostatically extruded, and high reduction ratios are possible on these materials. One of the disadvantages of the process is the required preparation of the starting work billet. The billet must be formed with a taper at one end to fit snugly into the die entry angle. This establishes a seal to prevent fluid from squirting out the die hole when the container is initially pressurized. 19.5.5 DEFECTS IN EXTRUDED PRODUCTS Owing to the considerable deformation associated with extrusion operations, a number of defects can occur in extruded products. The defects can be classified into the following categories, illustrated in Figure 19.39: FIGURE 19.39 Some common defects in extrusion: (a) centerburst, (b) piping, and (c) surface cracking. E1C19 11/11/2009 430 16:35:40 Page 430 Chapter 19/Bulk Deformation Processes in Metal Working (a) Centerburst. This defect is an internal crack that develops as a result of tensile stresses along the centerline of the workpart during extrusion. Although tensile stresses may seem unlikely in a compression process such as extrusion, they tend to occur under conditions that cause large deformation in the regions of the work away from the central axis. The significant material movement in these outer regions stretches the material along the center of the work. If stresses are great enough, bursting occurs. Conditions that promote centerburst are high die angles, low extrusion ratios, and impurities in the work metal that serve as starting points for crack defects. The difficult aspect of centerburst is its detection. It is an internal defect that is usually not noticeable by visual observation. Other names sometimes used for this defect include arrowhead fracture, center cracking, and chevron cracking. (b) Piping. Piping is a defect associated with direct extrusion. As in Figure 19.39(b), it is the formation of a sink hole in the end of the billet. The use of a dummy block whose diameter is slightly less than that of the billet helps to avoid piping. Other names given to this defect include tailpipe and fishtailing. (c) Surface cracking. This defect results from high workpart temperatures that cause cracks to develop at the surface. They often occur when extrusion speed is too high, leading to high strain rates and associated heat generation. Other factors contributing to surface cracking are high friction and surface chilling of high temperature billets in hot extrusion. 19.6 WIRE AND BAR DRAWING In the context of bulk deformation, drawing is an operation in which the cross section of a bar, rod, or wire is reduced by pulling it through a die opening, as in Figure 19.40. The general features of the process are similar to those of extrusion. The difference is that the work is pulled through the die in drawing, whereas it is pushed through the die in extrusion. Although the presence of tensile stresses is obvious in drawing, compression also plays a significant role because the metal is squeezed down as it passes through the die opening. For this reason, the deformation that occurs in drawing is sometimes referred to as indirect compression. Drawing is a term also used in sheet metalworking (Section 20.3). The term wire and bar drawing is used to distinguish the drawing process discussed here from the sheet metal process of the same name. The basic difference between bar drawing and wire drawing is the stock size that is processed. Bar drawing is the term used for large diameter bar and rod stock, while wire drawing applies to small diameter stock. Wire sizes down to 0.03 mm (0.001 in) are possible in wire drawing. Although the mechanics of the process are the same for the two cases, the methods, equipment, and even the terminology are somewhat different. Bar drawing is generally accomplished as a single-draft operation—the stock is pulled through one die opening. Because the beginning stock has a large diameter, it is in FIGURE 19.40 Drawing of bar, rod, or wire. E1C19 11/11/2009 16:35:40 Page 431 Section 19.6/Wire and Bar Drawing 431 the form of a straight cylindrical piece rather than coiled. This limits the length of the work that can be drawn, necessitating a batch type operation. By contrast, wire is drawn from coils consisting of several hundred (or even several thousand) feet of wire and is passed through a series of draw dies. The number of dies varies typically between 4 and 12. The term continuous drawing is used to describe this type of operation because of the long production runs that are achieved with the wire coils, which can be butt-welded each to the next to make the operation truly continuous. In a drawing operation, the change in size of the work is usually given by the area reduction, defined as follows: r¼ Ao  Af Ao ð19:28Þ where r ¼ area reduction in drawing; Ao ¼ original area of work, mm2 (in2); and Af ¼ final area, mm2 (in2). Area reduction is often expressed as a percentage. In bar drawing, rod drawing, and in drawing of large diameter wire for upsetting and heading operations, the term draft is used to denote the before and after difference in size of the processed work. The draft is simply the difference between original and final stock diameters: d ¼ Do  Df ð19:29Þ where d ¼ draft, mm (in); Do ¼ original diameter of work, mm (in); and Df ¼ final work diameter, mm (in). 19.6.1 ANALYSIS OF DRAWING In this section, we consider the mechanics of wire and bar drawing. How are stresses and forces computed in the process? We also consider how large a reduction is possible in a drawing operation. Mechanics of Drawing If no friction or redundant work occurred in drawing, true strain could be determined as follows: Ao 1 e ¼ ln ¼ ln ð19:30Þ Af 1r where Ao and Af are the original and final cross-sectional areas of the work, as previously defined; and r ¼ drawing reduction as given by Eq. (19.28). The stress that results from this ideal deformation is given by s ¼ Y f e ¼ Y f ln Ao Af ð19:31Þ n Ke where Y f ¼ 1þn ¼ average flow stress based on the value of strain given by Eq. (19.30). Because friction is present in drawing and the work metal experiences inhomogeneous deformation, the actual stress is larger than provided by Eq. (19.31). In addition to the ratio Ao/Af, other variables that influence draw stress are die angle and coefficient of friction at the work–die interface. A number of methods have been proposed for predicting draw stress based on values of these parameters [1], [3], and [19]. We present the equation suggested by Schey [19]:  m  Ao f ln sd ¼ Y f 1 þ Af tan a ð19:32Þ where s d ¼ draw stress, MPa (lb/in2); m ¼ die-work coefficient of friction; a ¼ die angle (half-angle) as defined in Figure 19.40; and f is a factor that accounts for inhomogeneous E1C19 11/11/2009 432 16:35:40 Page 432 Chapter 19/Bulk Deformation Processes in Metal Working deformation which is determined as follows for a round cross section: f ¼ 0:88  0:12 D Lc ð19:33Þ where D ¼ average diameter of work during drawing, mm (in); and Lc ¼ contact length of the work with the draw die in Figure 19.40, mm (in). Values of D and Lc can be determined from the following: D¼ Do þ Df 2 ð19:34aÞ Lc ¼ Do  Df 2 sin a ð19:34bÞ The corresponding draw force is then the area of the drawn cross section multiplied by the draw stress:  m  Ao F ¼ A f s d ¼ Af Y f 1 þ f ln Af tan a ð19:35Þ where F ¼ draw force, N (lb); and the other terms are defined above. The power required in a drawing operation is the draw force multiplied by exit velocity of the work. Example 19.4 Stress and Force in Wire Drawing Wire is drawn through a draw die with entrance angle ¼ 15 . Starting diameter is 2.5 mm and final diameter ¼ 2.0 mm. The coefficient of friction at the work–die interface ¼ 0.07. The metal has a strength coefficient K ¼ 205 MPa and a strain-hardening exponent n ¼ 0.20. Determine the draw stress and draw force in this operation. Solution: The values of D and Lc for Eq. (19.33) can be determined using Eqs. (19.34). D ¼ 2.25 mm and Lc ¼ 0.966 mm. Thus, 2:25 f ¼ 0:88 þ 0:12 ¼ 1:16 0:966 The areas before and after drawing are computed as Ao ¼ 4.91 mm2 and Af ¼ 3.14 mm2. The resulting true strain e ¼ ln(4.91/3.14) ¼ 0.446, and the average flow stress in the operation is computed: Yf ¼ 205(0:446)0:20 ¼ 145:4 MPa 1:20 Draw stress is given by Eq. (19.32):   0:07 s d ¼ (145:4) 1 þ (1:16)(0:446) ¼ 94:1 MPa tan 15 Finally, the draw force is this stress multiplied by the cross-sectional area of the exiting wire: F ¼ 94:1(3:14) ¼ 295:5 N n Maximum Reduction per Pass A question that may occur to the reader is: Why is more than one step required to achieve the desired reduction in wire drawing? Why not take the entire reduction in a single pass through one die, as in extrusion? The answer can be explained as follows. From the preceding equations, it is clear that as the reduction increases, draw stress increases. If the reduction is large enough, draw stress will exceed the yield E1C19 11/11/2009 16:35:40 Page 433 Section 19.6/Wire and Bar Drawing 433 strength of the exiting metal. When that happens, the drawn wire will simply elongate instead of new material being squeezed through the die opening. For wire drawing to be successful, maximum draw stress must be less than the yield strength of the exiting metal. It is a straightforward matter to determine this maximum draw stress and the resulting maximum possible reduction that can be made in one pass, under certain assumptions. Let us assume a perfectly plastic metal (n ¼ 0), no friction, and no redundant work. In this ideal case, the maximum possible draw stress is equal to the yield strength of the work material. Expressing this using the equation for draw stress under conditions of ideal deformation, Eq. (19.31), and setting Y f ¼ Y (because n ¼ 0), s d ¼ Y f ln Ao Ao 1 ¼ Y ln ¼ Y ln ¼Y Af Af 1r This means that ln(Ao=Af) ¼ ln (1=(1  r)) ¼ 1. That is, emax ¼ 1.0. In order for emax to be zero, then Ao=Af ¼ 1=(1  r) must equal the natural logarithm base e. Accordingly, the maximum possible area ratio is Ao ¼ e ¼ 2:7183 Af ð19:36Þ and the maximum possible reduction is rmax ¼ e1 ¼ 0:632 e ð19:37Þ The value given by Eq. (19.37) is often used as the theoretical maximum reduction possible in a single draw, even though it ignores (1) the effects of friction and redundant work, which would reduce the maximum possible value, and (2) strain hardening, which would increase the maximum possible reduction because the exiting wire would be stronger than the starting metal. In practice, draw reductions per pass are quite below the theoretical limit. Reductions of 0.50 for single-draft bar drawing and 0.30 for multiple-draft wire drawing seem to be the upper limits in industrial operations. 19.6.2 DRAWING PRACTICE Drawing is usually performed as a cold working operation. It is most frequently used to produce round cross sections, but squares and other shapes are also drawn. Wire drawing is an important industrial process, providing commercial products such as electrical wire and cable; wire stock for fences, coat hangers, and shopping carts; and rod stock to produce nails, screws, rivets, springs, and other hardware items. Bar drawing is used to produce metal bars for machining, forging, and other processes. Advantages of drawing in these applications include (1) close dimensional control, (2) good surface finish, (3) improved mechanical properties such as strength and hardness, and (4) adaptability to economical batch or mass production. Drawing speeds are as high as 50 m/s (10,000 ft/min) for very fine wire. In the case of bar drawing to provide stock for machining, the operation improves the machinability of the bar (Section 24.1). Drawing Equipment Bar drawing is accomplished on a machine called a draw bench, consisting of an entry table, die stand (which contains the draw die), carriage, and exit rack. The arrangement is shown in Figure 19.41. The carriage is used to pull the stock through the draw die. It is powered by hydraulic cylinders or motor-driven chains. The die stand is often designed to hold more than one die, so that several bars can be pulled simultaneously through their respective dies. E1C19 11/11/2009 434 16:35:41 Page 434 Chapter 19/Bulk Deformation Processes in Metal Working FIGURE 19.41 Hydraulically operated draw bench for drawing metal bars. Wire drawing is done on continuous drawing machines that consist of multiple draw dies, separated by accumulating drums between the dies, as in Figure 19.42. Each drum, called a capstan, is motor driven to provide the proper pull force to draw the wire stock through the upstream die. It also maintains a modest tension on the wire as it proceeds to the next draw die in the series. Each die provides a certain amount of reduction in the wire, so that the desired total reduction is achieved by the series. Depending on the metal to be processed and the total reduction, annealing of the wire is sometimes required between groups of dies in the series. Draw Dies Figure 19.43 identifies the features of a typical draw die. Four regions of the die can be distinguished: (1) entry, (2) approach angle, (3) bearing surface (land), and (4) back relief. The entry region is usually a bell-shaped mouth that does not contact the work. Its purpose is to funnel the lubricant into the die and prevent scoring of work and die surfaces. The approach is where the drawing process occurs. It is cone-shaped with an angle (halfangle) normally ranging from about 6 to 20 . The proper angle varies according to work FIGURE 19.42 Continuous drawing of wire. E1C19 11/11/2009 16:35:41 Page 435 Section 19.6/Wire and Bar Drawing 435 FIGURE 19.43 Draw die for drawing of round rod or wire. material. The bearing surface, or land, determines the size of the final drawn stock. Finally, the back relief is the exit zone. It is provided with a back relief angle (half-angle) of about 30 . Draw dies are made of tool steels or cemented carbides. Dies for high-speed wire drawing operations frequently use inserts made of diamond (both synthetic and natural) for the wear surfaces. Preparation of the Work Prior to drawing, the beginning stock must be properly prepared. This involves three steps: (1) annealing, (2) cleaning, and (3) pointing. The purpose of annealing is to increase the ductility of the stock to accept deformation during drawing. As previously mentioned, annealing is sometimes needed between steps in continuous drawing. Cleaning of the stock is required to prevent damage of the work surface and draw die. It involves removal of surface contaminants (e.g., scale and rust) by means of chemical pickling or shot blasting. In some cases, prelubrication of the work surface is accomplished subsequent to cleaning. Pointing involves the reduction in diameter of the starting end of the stock so that it can be inserted through the draw die to start the process. This is usually accomplished by swaging, rolling, or turning. The pointed end of the stock is then gripped by the carriage jaws or other device to initiate the drawing process. 19.6.3 TUBE DRAWING Drawing can be used to reduce the diameter or wall thickness of seamless tubes and pipes, after the initial tubing has been produced by some other process such as extrusion. Tube drawing can be carried out either with or without a mandrel. The simplest method uses no mandrel and is used for diameter reduction, as in Figure 19.44. The term tube sinking is sometimes applied to this operation. FIGURE 19.44 Tube drawing with no mandrel (tube sinking). E1C19 11/11/2009 436 16:35:41 Page 436 Chapter 19/Bulk Deformation Processes in Metal Working FIGURE 19.45 Tube drawing with mandrels: (a) fixed mandrel, (b) floating plug. The problem with tube drawing in which no mandrel is used, as in Figure 19.44, is that it lacks control over the inside diameter and wall thickness of the tube. This is why mandrels of various types are used, two of which are illustrated in Figure 19.45. The first, Figure 19.45 (a), uses a fixed mandrel attached to a long support bar to establish inside diameter and wall thickness during the operation. Practical limitations on the length of the support bar in this method restrict the length of the tube that can be drawn. The second type, shown in (b), uses a floating plug whose shape is designed so that it finds a ‘‘natural’’ position in the reduction zone of the die. This method removes the limitations on work length present with the fixed mandrel. REFERENCES [1] Altan, T., Oh, S-I., and Gegel, H. L. Metal Forming: Fundamentals and Applications. ASM International, Materials Park, Ohio, 1983. [2] ASM Handbook, Vol. 14A, Metalworking: Bulk Forming. ASM International, Materials Park, Ohio, 2005. [3] Avitzur, B. Metal Forming: Processes and Analysis. Robert E. Krieger Publishing Company, Huntington, New York, 1979. [4] Black, J. T., and Kohser, R. A., DeGarmo’s Materials and Processes in Manufacturing, 10th ed. John Wiley & Sons, Inc., Hoboken, New Jersey, 2008. [5] Byrer, T. G.,et al. (eds.). Forging Handbook. Forging Industry Association, Cleveland, Ohio; and American Society for Metals, Metals Park, Ohio, 1985. [6] Cook, N. H. Manufacturing Analysis. AddisonWesley Publishing Company, Inc., Reading, Massachusetts, 1966. [7] Groover, M. P.‘‘An Experimental Study of the Work Components and Extrusion Strain in the Cold Forward Extrusion of Steel,’’ research report. Bethlehem Steel Corporation, Bethlehem, Pennsylvania, 1966. [8] Harris, J. N. Mechanical Working of Metals. Pergamon Press, Oxford, UK, 1983. [9] Hosford, W. F., and Caddell, R. M. Metal Forming: Mechanics and Metallurgy, 3rd ed. Cambridge University Press, Cambridge, UK, 2007. [10] Jensen, J. E. (ed.). Forging Industry Handbook. Forging Industry Association, Cleveland, Ohio, 1970. [11] Johnson, W.‘‘The Pressure for the Cold Extrusion of Lubricated Rod Through Square Dies of Moderate Reduction at Slow Speeds,’’ Journal of the Institute of Metals, Vol. 85, 1956. [12] Kalpakjian, S. Mechanical Processing of Materials. D. Van Nostrand Company, Inc., Princeton, New Jersey, 1967, Chapter 5. [13] Kalpakjian, S., and SchmidS. R. Manufacturing Processes for Engineering Materials, 6th ed. Pearson Prentice Hall, Upper Saddle River, New Jersey, 2010. [14] Lange, K. Handbook of Metal Forming. Society of Manufacturing Engineers, Dearborn, Michigan, 2006. [15] Laue, K., and Stenger, H. Extrusion: Processes, Machinery, and Tooling. American Society for Metals, Metals Park, Ohio, 1981. E1C19 11/11/2009 16:35:41 Page 437 Multiple Choice Quiz [16] Mielnik, E. M. Metalworking Science and Engineering. McGraw-Hill, Inc., New York, 1991. [17] Roberts, W. L. Hot Rolling of Steel. Marcel Dekker, Inc., New York, 1983. [18] Roberts, W. L. Cold Rolling of Steel. Marcel Dekker, Inc., New York, 1978. 437 [19] Schey, J. A. Introduction to Manufacturing Processes, 3rd ed. McGraw-Hill Book Company, New York, 2000. [20] Wick, C., et al. (eds.). Tool and Manufacturing Engineers Handbook, 4th ed. Vol. II, Forming. Society of Manufacturing Engineers, Dearborn, Michigan, 1984. REVIEW QUESTIONS 19.1. What are the reasons why the bulk deformation processes are important commercially and technologically? 19.2. Name the four basic bulk deformation processes. 19.3. What is rolling in the context of the bulk deformation processes? 19.4. In rolling of steel, what are the differences between a bloom, a slab, and a billet? 19.5. List some of the products produced on a rolling mill. 19.6. What is draft in a rolling operation? 19.7. What is sticking in a hot rolling operation? 19.8. Identify some of the ways in which force in flat rolling can be reduced. 19.9. What is a two-high rolling mill? 19.10. What is a reversing mill in rolling? 19.11. Besides flat rolling and shape rolling, identify some additional bulk forming processes that use rolls to effect the deformation. 19.12. What is forging? 19.13. One way to classify forging operations is by the degree to which the work is constrained in the die. By this classification, name the three basic types. 19.14. Why is flash desirable in impression-die forging? 19.15. What is a trimming operation in the context of impression-die forging? 19.16. What are the two basic types of forging equipment? 19.17. What is isothermal forging? 19.18. What is extrusion? 19.19. Distinguish between direct and indirect extrusion. 19.20. Name some products that are produced by extrusion. 19.21. Why is friction a factor in determining the ram force in direct extrusion but not a factor in indirect extrusion? 19.22. What does the centerburst defect in extrusion have in common with the roll piercing process? 19.23. What is wire drawing and bar drawing? 19.24. Although the workpiece in a wire drawing operation is obviously subjected to tensile stresses, how do compressive stresses also play a role in the process? 19.25. In a wire drawing operation, why must the drawing stress never exceed the yield strength of the work metal? 19.26. (Video) According to the video on forming, what is the primary factor that makes the mechanical performance of forged parts better than cast parts in many situations? 19.27. (Video) List the accessory tools that can be used during open-die forging according to the video on forging. 19.28. (Video) List the performing operations discussed in the forming video. MULTIPLE CHOICE QUIZ There are 27 correct answers in the following multiple choice questions (some questions have multiple answers that are correct). To attain a perfect score on the quiz, all correct answers must be given. Each correct answer is worth 1 point. Each omitted answer or wrong answer reduces the score by 1 point, and each additional answer beyond the correct number of answers reduces the score by 1 point. Percentage score on the quiz is based on the total number of correct answers. 19.1. The starting workpiece in steel hot rolling of plate and sheet stock is which of the following (one best answer): (a) bar stock, (b) billet, (c) bloom, (d) slab, or (e) wire stock? 19.2. The maximum possible draft in a rolling operation depends on which of the following parameters (two correct answers): (a) coefficient of friction between roll and work, (b) roll diameter, (c) roll velocity, (d) stock thickness, (e) strain, and (f) strength coefficient of the work metal? 19.3. Which of the following stress or strength parameters is used in the computation of rolling force (one E1C19 11/11/2009 438 16:35:42 Page 438 Chapter 19/Bulk Deformation Processes in Metal Working 19.4. 19.5. 19.6. 19.7. 19.8. 19.9. best answer): (a) average flow stress, (b) compression strength, (c) final flow stress, (d) tensile strength, or (e) yield strength? Whichofthe followingrollingmilltypesareassociated with relatively small diameter rolls in contact with the work (two correct answers): (a) cluster mill, (b) continuous rolling mill, (c) four-high mill, (d) reversing mill, and (e) three-high configuration? Production of pipes and tubes is associated with which of the following bulk deformation processes (three correct answers): (a) extrusion, (b) hobbing, (c) ring rolling, (d) roll forging, (e) roll piercing, (f) tube sinking, and (g) upsetting? Which of the following stress or strength parameters is used in the computation of the maximum force in a forging operation (one best answer): (a) average flow stress, (b) compression strength, (c) final flow stress, (d) tensile strength, or (e) yield strength? Which of the following operations are closely related to open-die forging (three best answers): (a) cogging, (b) flashless forging, (c) fullering, (d) impression-die forging, (e) Mannesmann process, (f) precision forging, (g) soaking, and (h) upsetting? Flash in impression-die forging serves no useful purpose and is undesirable because it must be trimmed from the part after forming: (a) true or (b) false? Which of the following are classified as forging operations (four correct answers): (a) coining, (b) fullering, (c) impact extrusion, (d) roll piercing, (e) swaging, (f) thread rolling, (g) trimming, and (h) upsetting? 19.10. Which of the following are alternative names for indirect extrusion (two correct answers): (a) backward extrusion, (b) direct extrusion, (c) forward extrusion, (d) impact extrusion, and (e) reverse extrusion? 19.11. The production of tubing is possible in indirect extrusion but not in direct extrusion: (a) true or (b) false? 19.12. Which of the following stress or strength parameters is used in the computation of the force in an extrusion operation (one best answer): (a) average flow stress, (b) compression strength, (c) final flow stress, (d) tensile strength, or (e) yield strength? 19.13. In which of the following extrusion operations is friction a factor in determining the extrusion force (one best answer): (a) direct extrusion or (b) indirect extrusion? 19.14. Theoretically, the maximum reduction possible in a wire drawing operation, under the assumptions of a perfectly plastic metal, no friction, and no redundant work, is which of the following (one answer): (a) zero, (b) 0.63, (c) 1.0, or (d) 2.72? 19.15. Which of the following bulk deformation processes are involved in the production of nails for lumber construction (three best answers): (a) bar and wire drawing, (b) extrusion, (c) flashless forging, (d) impression-die forging, (e) rolling, and (f) upsetting? 19.16. Johnson’s formula is associated with which one of the four bulk deformation processes: (a) bar and wire drawing, (b) extrusion, (c) forging, and (d) rolling? PROBLEMS Rolling 19.1. A 42.0-mm-thick plate made of low carbon steel is to be reduced to 34.0 mm in one pass in a rolling operation. As the thickness is reduced, the plate widens by 4%. The yield strength of the steel plate is 174 MPa and the tensile strength is 290 MPa. The entrance speed of the plate is 15.0 m/min. The roll radius is 325 mm and the rotational speed is 49.0 rev/min. Determine (a) the minimum required coefficient of friction that would make this rolling operation possible, (b) exit velocity of the plate, and (c) forward slip. 19.2. A 2.0-in-thick slab is 10.0 in wide and 12.0 ft long. Thickness is to be reduced in three steps in a hot rolling operation. Each step will reduce the slab to 75% of its previous thickness. It is expected that for this metal and reduction, the slab will widen by 3% in each step. If the entry speed of the slab in the first step is 40 ft/min, and roll speed is the same for the three steps, determine: (a) length and (b) exit velocity of the slab after the final reduction. 19.3. A series of cold rolling operations are to be used to reduce the thickness of a plate from 50 mm down to 25 mm in a reversing two-high mill. Roll diameter ¼ 700 mm and coefficient of friction between rolls and work ¼ 0.15. The specification is that the draft is to be equal on each pass. Determine (a) minimum number of passes required, and (b) draft for each pass? E1C19 11/11/2009 16:35:42 Page 439 Problems 19.4. In the previous problem, suppose that the percent reduction were specified to be equal for each pass, rather than the draft. (a) What is the minimum number of passes required? (b) What is the draft for each pass? 19.5. A continuous hot rolling mill has two stands. Thickness of the starting plate ¼ 25 mm and width ¼ 300 mm. Final thickness is to be 13 mm. Roll radius at each stand ¼ 250 mm. Rotational speed at the first stand ¼ 20 rev/min. Equal drafts of 6 mm are to be taken at each stand. The plate is wide enough relative to its thickness that no increase in width occurs. Under the assumption that the forward slip is equal at each stand, determine (a) speed vr at each stand, and (b) forward slip s. (c) Also, determine the exiting speeds at each rolling stand, if the entering speed at the first stand ¼ 26 m/min. 19.6. A continuous hot rolling mill has eight stands. The dimensions of the starting slab are: thickness ¼ 3.0 in, width ¼ 15.0 in, and length ¼ 10 ft. The final thickness is to be 0.3 in. Roll diameter at each stand ¼ 36 in, and rotational speed at stand number 1 ¼ 30 rev/min. It is observed that the speed of the slab entering stand 1 ¼ 240 ft/min. Assume that no widening of the slab occurs during the rolling sequence. Percent reduction in thickness is to be equal at all stands, and it is assumed that the forward slip will be equal at each stand. Determine (a) percentage reduction at each stand, (b) rotational speed of the rolls at stands 2 through 8, and (c) forward slip. (d) What is the draft at stands 1 and 8? (e) What is the length and exit speed of the final strip exiting stand 8? 19.7. A plate that is 250 mm wide and 25 mm thick is to be reduced in a single pass in a two-high rolling mill to a thickness of 20 mm. The roll has a radius ¼ 500 mm, and its speed ¼ 30 m/min. The work material has a strength coefficient ¼ 240 MPa and a strainhardening exponent ¼ 0.2. Determine (a) roll 19.8. 19.9. 19.10. 19.11. 19.12. 19.13. 439 force, (b) roll torque, and (c) power required to accomplish this operation. Solve Problem 19.7 using a roll radius ¼ 250 mm. Solve Problem 19.7, only assume a cluster mill with working rolls of radius ¼ 50 mm. Compare the results with the previous two problems, and note the important effect of roll radius on force, torque and power. A 4.50-in-thick slab that is 9 in wide and 24 in long is to be reduced in a single pass in a two-high rolling mill to a thickness of 3.87 in. The roll rotates at a speed of 5.50 rev/min and has a radius of 17.0 in. The work material has a strength coefficient ¼ 30,000 lb/in2 and a strain-hardening exponent ¼ 0.15. Determine (a) roll force, (b) roll torque, and (c) power required to accomplish this operation. A single-pass rolling operation reduces a 20 mm thick plate to 18 mm. The starting plate is 200 mm wide. Roll radius ¼ 250 mm and rotational speed ¼ 12 rev/min. The work material has a strength coefficient ¼ 600 MPa and a strength coefficient ¼ 0.22. Determine (a) roll force, (b) roll torque, and (c) power required for this operation. A hot rolling mill has rolls of diameter ¼ 24 in. It can exert a maximum force ¼ 400,000 lb. The mill has a maximum horsepower ¼ 100 hp. It is desired to reduce a 1.5-in thick plate by the maximum possible draft in one pass. The starting plate is 10 in wide. In the heated condition, the work material has a strength coefficient ¼ 20,000 lb/ in2 and a strain-hardening exponent ¼ zero. Determine (a) maximum possible draft, (b) associated true strain, and (c) maximum speed of the rolls for the operation. Solve Problem 19.12 except that the operation is warm rolling and the strain-hardening exponent is 0.18. Assume the strength coefficient remains at 20,000 lb/in2. Forging 19.14. A cylindrical part is warm upset forged in an open die. The initial diameter is 45 mm and the initial height is 40 mm. The height after forging is 25 mm. The coefficient of friction at the die–work interface is 0.20. The yield strength of the work material is 285 MPa, and its flow curve is defined by a strength coefficient of 600 MPa and a strain-hardening exponent of 0.12. Determine the force in the operation (a) just as the yield point is reached (yield at strain ¼ 0.002), (b) at a height of 35 mm, (c) at a height of 30 mm, and (d) at a height of 25 mm. Use of a spreadsheet calculator is recommended. 19.15. A cylindrical workpart with D ¼ 2.5 in and h ¼ 2.5 in is upset forged in an open die to a height ¼ 1.5 in. Coefficient of friction at the die–work interface ¼ 0.10. The work material has a flow curve defined by: K ¼ 40,000 lb/in2 and n ¼ 0.15. Yield strength ¼ 15,750 lb/in2. Determine the instantaneous force in the operation (a) just as the yield point is reached (yield at strain ¼ 0.002), (b) at height h ¼ 2.3 in, (c) h ¼ 2.1 in, (d) h ¼ 1.9 in, (e) h ¼ 1.7 in, and (f) h ¼ 1.5 in. Use of a spreadsheet calculator is recommended. 19.16. A cylindrical workpart has a diameter ¼ 2.5 in and a height ¼ 4.0 in. It is upset forged to a height ¼ 2.75 in. E1C19 11/11/2009 440 16:35:43 Page 440 Chapter 19/Bulk Deformation Processes in Metal Working Coefficient of friction at the die–work interface ¼ 0.10. The work material has a flow curve with strength coefficient ¼ 25,000 lb/in2 and strain-hardening exponent ¼ 0.22. Determine the plot of force vs. work height. Use of a spreadsheet calculator is recommended. 19.17. A cold heading operation is performed to produce the head on a steel nail. The strength coefficient for this steel is 600 MPa, and the strain-hardening exponent is 0.22. Coefficient of friction at the die–work interface is 0.14. The wire stock out of which the nail is made is 5.00 mm in diameter. The head is to have a diameter of 9.5 mm and a thickness of 1.6 mm. The final length of the nail is 120 mm. (a) What length of stock must project out of the die in order to provide sufficient volume of material for this upsetting operation? (b) Compute the maximum force that the punch must apply to form the head in this open-die operation. 19.18. Obtain a large common nail (flat head). Measure the head diameter and thickness, as well as the diameter of the nail shank. (a) What stock length must project out of the die in order to provide sufficient material to produce the nail? (b) Using appropriate values for strength coefficient and strain-hardening exponent for the metal out of which the nail is made (Table 3.4), compute the maximum force in the heading operation to form the head. 19.19. A hot upset forging operation is performed in an open die. The initial size of the workpart is: Do ¼ 25 mm, and ho ¼ 50 mm. The part is upset to a diameter ¼ 50 mm. The work metal at this elevated temperature yields at 85 MPa (n ¼ 0). Coefficient of friction at the die–work interface ¼ 0.40. Determine (a) final height of the part, and (b) maximum force in the operation. 19.20. A hydraulic forging press is capable of exerting a maximum force ¼ 1,000,000 N. A cylindrical workpart is to be cold upset forged. The starting part has diameter ¼ 30 mm and height ¼ 30 mm. The flow curve of the metal is defined by K ¼ 400 MPa and n ¼ 0.2. Determine the maximum reduction in height to which the part can be compressed with this forging press, if the coefficient of friction ¼ 0.1. Use of a spreadsheet calculator is recommended. 19.21. A part is designed to be hot forged in an impression die. The projected area of the part, including flash, is 16 in2. After trimming, the part has a projected area of 10 in2. Part geometry is complex. As heated the work material yields at 10,000 lb/in2, and has no tendency to strain harden. At room temperature, the material yields at 25,000 lb/in2 Determine the maximum force required to perform the forging operation. 19.22. A connecting rod is designed to be hot forged in an impression die. The projected area of the part is 6,500 mm2. The design of the die will cause flash to form during forging, so that the area, including flash, will be 9,000 mm2. The part geometry is considered to be complex. As heated the work material yields at 75 MPa, and has no tendency to strain harden. Determine the maximum force required to perform the operation. Extrusion 19.23. A cylindrical billet that is 100 mm long and 50 mm in diameter is reduced by indirect (backward) extrusion to a 20 mm diameter. The die angle is 90 . The Johnson equation has a ¼ 0.8 and b ¼ 1.4, and the flow curve for the work metal has a strength coefficient of 800 MPa and strain-hardening exponent of 0.13. Determine (a) extrusion ratio, (b) true strain (homogeneous deformation), (c) extrusion strain, (d) ram pressure, and (e) ram force. 19.24. A 3.0-in-long cylindrical billet whose diameter ¼ 1.5 in is reduced by indirect extrusion to a diameter ¼ 0.375 in. Die angle ¼ 90 . In the Johnson equation, a ¼ 0.8 and b ¼ 1.5. In the flow curve for the work metal, K ¼ 75,000 lb/in2 and n ¼ 0.25. Determine (a) extrusion ratio, (b) true strain (homogeneous deformation), (c) extrusion strain, (d) ram pressure, (e) ram force, and (f) power if the ram speed ¼ 20 in/min. 19.25. A billet that is 75 mm long with diameter ¼ 35 mm is direct extruded to a diameter of 20 mm. The extrusion die has a die angle ¼ 75 . For the work metal, K ¼ 600 MPa and n ¼ 0.25. In the Johnson extrusion strain equation, a ¼ 0.8 and b ¼ 1.4. Determine (a) extrusion ratio, (b) true strain (homogeneous deformation), (c) extrusion strain, and (d) ram pressure and force at L ¼ 70, 60, 50, 40, 30, 20, and 10 mm. Use of a spreadsheet calculator is recommended for part (d). 19.26. A 2.0-in-long billet with diameter ¼ 1.25 in is direct extruded to a diameter of 0.50 in. The extrusion die angle ¼ 90 . For the work metal, K ¼ 45,000 lb/in2, and n ¼ 0.20. In the Johnson extrusion strain equation, a ¼ 0.8 and b ¼ 1.5. Determine (a) extrusion ratio, (b) true strain (homogeneous deformation), (c) extrusion strain, and (d) ram pressure at L ¼ 2.0, 1.5, 1.0, 0.5 and 0.0 in. Use of a spreadsheet calculator is recommended for part (d). E1C19 11/11/2009 16:35:43 Page 441 Problems 19.27. A direct extrusion operation is performed on a cylindrical billet with an initial diameter of 2.0 in and an initial length of 4.0 in. The die angle ¼ 60 and orifice diameter is 0.50 in. In the Johnson extrusion strain equation, a ¼ 0.8 and b ¼ 1.5. The operation is carried out hot and the hot metal yields at 13,000 lb/in2 and does not strain harden when hot. (a) What is the extrusion ratio? (b) Determine the ram position at the point when the metal has been compressed into the cone of the die and starts to extrude through the die opening. (c) What is the ram pressure corresponding to this position? (d) Also determine the length of the final part if the ram stops its forward movement at the start of the die cone. 19.28. An indirect extrusion process starts with an aluminum billet with diameter ¼ 2.0 in and length ¼ 3.0 in. Final cross section after extrusion is a square with 1.0 in on a side. The die angle ¼ 90 . The operation is performed cold and the strength coefficient of the metal K ¼ 26,000 lb/in2 and strainhardening exponent n ¼ 0.20. In the Johnson extrusion strain equation, a ¼ 0.8 and b ¼ 1.2. (a) Compute the extrusion ratio, true strain, and extrusion strain. (b) What is the shape factor of the product? (c) If the butt left in the container at the end of the stroke is 0.5 in thick, what is the length of the extruded section? (d) Determine the ram pressure in the process. 19.29. An L-shaped structural section is direct extruded from an aluminum billet in which Lo ¼ 500 mm and Do ¼ 100 mm. Dimensions of the cross section are given in Figure P19.29. Die angle ¼ 90 . Determine (a) extrusion ratio, (b) shape factor, and (c) length of the extruded section if the butt remaining in the container at the end of the ram stroke is 25 mm. 19.30. The flow curve parameters for the aluminum alloy of Problem 19.29 are: K ¼ 240 MPa and n ¼ 0.16. If 441 12 62 12 50 FIGURE P19.29 in mm). Part for Problem 19.29 (dimensions are the die angle in this operation ¼ 90 , and the corresponding Johnson strain equation has constants a ¼ 0.8 and b ¼ 1.5, compute the maximum force required to drive the ram forward at the start of extrusion. 19.31. A cup-shaped part is backward extruded from an aluminum slug that is 50 mm in diameter. The final dimensions of the cup are: OD ¼ 50 mm, ID ¼ 40 mm, height ¼ 100 mm, and thickness of base ¼ 5 mm. Determine (a) extrusion ratio, (b) shape factor, and (c) height of starting slug required to achieve the final dimensions. (d) If the metal has flow curve parameters K ¼ 400 MPa and n ¼ 0.25, and the constants in the Johnson extrusion strain equation are: a ¼ 0.8 and b ¼ 1.5, determine the extrusion force. 19.32. Determine the shape factor for each of the extrusion die orifice shapes in Figure P19.32. FIGURE P19.32 Cross-sectional shapes for Problem 19.32 (dimensions are in mm): (a) rectangular bar, (b) tube, (c) channel, and (d) cooling fins. E1C19 11/11/2009 442 16:35:43 Page 442 Chapter 19/Bulk Deformation Processes in Metal Working 19.33. A direct extrusion operation produces the cross section shown in Figure P19.32(a) from a brass billet whose diameter ¼ 125 mm and length ¼ 350 mm. The flow curve parameters of the brass are K ¼ 700 MPa and n ¼ 0.35. In the Johnson strain equation, a ¼ 0.7 and b ¼ 1.4. Determine (a) the extrusion ratio, (b) the shape factor, (c) the force required to drive the ram forward during extrusion at the point in the process when the billet length remaining in the container ¼ 300 mm, and (d) the length of the extruded section at the end of the operation if the volume of the butt left in the container is 600,000 mm3. 19.34. In a direct extrusion operation the cross section shown in Figure P19.32(b) is produced from a copper billet whose diameter ¼ 100 mm and length ¼ 500 mm. In the flow curve for copper, the strength coefficient ¼ 300 MPa and strain-hardening exponent ¼ 0.50. In the Johnson strain equation, a ¼ 0.8 and b ¼ 1.5. Determine (a) the extrusion ratio, (b) the shape factor, (c) the force required to drive the ram forward during extrusion at the point in the process when the billet length remaining in the container ¼ 450 mm, and (d) the length of the extruded section at the end of the operation if the volume of the butt left in the container is 350,000 mm3. 19.35. A direct extrusion operation produces the cross section shown in Figure P19.32(c) from an aluminum billet whose diameter ¼ 150 mm and length ¼ 500 mm. The flow curve parameters for the aluminum are K ¼ 240 MPa and n ¼ 0.16. In the Johnson strain equation, a ¼ 0.8 and b ¼ 1.2. Determine (a) the extrusion ratio, (b) the shape factor, (c) the force required to drive the ram forward during extrusion at the point in the process when the billet length remaining in the container ¼ 400 mm, and (d) the length of the extruded section at the end of the operation if the volume of the butt left in the container is 600,000 mm3. 19.36. A direct extrusion operation produces the cross section shown in Figure P19.32(d) from an aluminum billet whose diameter ¼ 150 mm and length ¼ 900 mm. The flow curve parameters for the aluminum are K ¼ 240 MPa and n ¼ 0.16. In the Johnson strain equation, a ¼ 0.8 and b ¼ 1.5. Determine (a) the extrusion ratio, (b) the shape factor, (c) the force required to drive the ram forward during extrusion at the point in the process when the billet length remaining in the container ¼ 850 mm, and (d) the length of the extruded section at the end of the operation if the volume of the butt left in the container is 600,000 mm3. Drawing 19.37. A spool of wire has a starting diameter of 2.5 mm. It is drawn through a die with an opening that is to 2.1 mm. The entrance angle of the die is 18 . Coefficient of friction at the work–die interface is 0.08. The work metal has a strength coefficient of 450 MPa and a strain-hardening coefficient of 0.26. The drawing is performed at room temperature. Determine (a) area reduction, (b) draw stress, and (c) draw force required for the operation. 19.38. Rod stock that has an initial diameter of 0.50 in is drawn through a draw die with an entrance angle of 13 . The final diameter of the rod is ¼ 0.375 in. The metal has a strength coefficient of 40,000 lb/in2 and a strain-hardening exponent of 0.20. Coefficient of friction at the work–die interface ¼ 0.1. Determine (a) area reduction, (b) draw force for the operation, and (c) horsepower to perform the operation if the exit velocity of the stock ¼ 2 ft/sec. 19.39. Bar stock of initial diameter ¼ 90 mm is drawn with a draft ¼ 15 mm. The draw die has an entrance angle ¼ 18 , and the coefficient of friction at the work–die interface ¼ 0.08. The metal behaves as a perfectly plastic material with yield stress ¼ 105 MPa. Determine (a) area reduction, (b) draw stress, (c) draw force required for the operation, and (d) power to perform the operation if exit velocity ¼ 1.0 m/min. 19.40. Wire stock of initial diameter ¼ 0.125 in is drawn through two dies each providing a 0.20 area reduction. The starting metal has a strength coefficient ¼ 40,000 lb/in2 and a strain-hardening exponent ¼ 0.15. Each die has an entrance angle of 12 , and the coefficient of friction at the work–die interface is estimated to be 0.10. The motors driving the capstans at the die exits can each deliver 1.50 hp at 90% efficiency. Determine the maximum possible speed of the wire as it exits the second die. E1C20 11/11/2009 16:3:42 Page 443 20 SHEET METALWORKING Chapter Contents 20.1 Cutting Operations 20.1.1 Shearing, Blanking, and Punching 20.1.2 Engineering Analysis of Sheet-Metal Cutting 20.1.3 Other Sheet-Metal-Cutting Operations 20.2 Bending Operations 20.2.1 V-Bending and Edge Bending 20.2.2 Engineering Analysis of Bending 20.2.3 Other Bending and Forming Operations 20.3 Drawing 20.3.1 Mechanics of Drawing 20.3.2 Engineering Analysis of Drawing 20.3.3 Other Drawing Operations 20.3.4 Defects in Drawing 20.4 Other Sheet-Metal-Forming Operations 20.4.1 Operations Performed with Metal Tooling 20.4.2 Rubber Forming Processes 20.5 Dies and Presses for Sheet-Metal Processes 20.5.1 Dies 20.5.2 Presses 20.6 Sheet-Metal Operations Not Performed on Presses 20.6.1 Stretch Forming 20.6.2 Roll Bending and Roll Forming 20.6.3 Spinning 20.6.4 High-Energy-Rate Forming 20.7 Bending of Tube Stock Sheet metalworking includes cutting and forming operations performed on relatively thin sheets of metal. Typical sheet-metal thicknesses are between 0.4 mm (1/64 in) and 6 mm (1/4 in). When thickness exceeds about 6 mm, the stock is usually referred to as plate rather than sheet. The sheet or plate stock used in sheet metalworking is produced by flat rolling (Section 19.1). The most commonly used sheet metal is low carbon steel (0.06%–0.15% C typical). Its low cost and good formability, combined with sufficient strength for most product applications, make it ideal as a starting material. The commercial importance of sheet metalworking is significant. Consider the number of consumer and industrial products that include sheet or plate metal parts: automobile and truck bodies, airplanes, railway cars, locomotives, farm and construction equipment, appliances, office furniture, and more. Although these examples are conspicuous because they have sheet-metal exteriors, many of their internal components are also made of sheet or plate stock. Sheetmetal parts are generally characterized by high strength, good dimensional accuracy, good surface finish, and relatively low cost. For components that must be made in large quantities, economical mass-production operations can be designed to process the parts. Aluminum beverage cans are a prime example. Sheet-metal processing is usually performed at room temperature (cold working). The exceptions are when the stock is thick, the metal is brittle, or the deformation is significant. These are usually cases of warm working rather than hot working. Most sheet-metal operations are performed on machine tools called presses. The term stamping press is used to distinguish these presses from forging and extrusion presses. The tooling that performs sheet metalwork is called a punch-and-die; the term stamping die is also used. The sheet-metal products are called stampings. To facilitate mass production, the sheet metal is often presented to the press as long strips or coils. Various types of 443 E1C20 11/11/2009 444 16:3:42 Page 444 Chapter 20/Sheet Metalworking punch-and-die tooling and stamping presses are described in Section 20.5. Final sections of the chapter cover various operations that do not utilize conventional punch-and-die tooling, and most of them are not performed on stamping presses. Two video clips on our DVD illustrate many of the topics discussed in this chapter. VIDEO CLIP Sheet-Metal Shearing and Bending. This clip has two segments on shearing and bending. VIDEO CLIP Sheet-Metal Stamping Dies and Processes. Two segments are included: (1) sheet metal formability and (2) basic stamping die operations. The three major categories of sheet-metal processes are (1) cutting, (2) bending, and (3) drawing. Cutting is used to separate large sheets into smaller pieces, to cut out part perimeters, and to make holes in parts. Bending and drawing are used to form sheet-metal parts into their required shapes. 20.1 CUTTING OPERATIONS Cutting of sheet metal is accomplished by a shearing action between two sharp cutting edges. The shearing action is depicted in the four stop-action sketches of Figure 20.1, in which the upper cutting edge (the punch) sweeps down past a stationary lower cutting edge (the die). As the punch begins to push into the work, plastic deformation occurs in the surfaces of the sheet. As the punch moves downward, penetration occurs in which the punch compresses the sheet and cuts into the metal. This penetration zone is generally about one-third the thickness of the sheet. As the punch continues to travel into the work, fracture is initiated in the work at the two cutting edges. If the clearance between the punch v v, F v, F v, F Punch Plastic deformation Penetration t c Fracture Die (1) (2) (3) (4) FIGURE 20.1 Shearing of sheet metal between two cutting edges: (1) just before the punch contacts work; (2) punch begins to push into work, causing plastic deformation; (3) punch compresses and penetrates into work causing a smooth cut surface; and (4) fracture is initiated at the opposing cutting edges that separate the sheet. Symbols v and F indicate motion and applied force, respectively, t ¼ stock thickness, c ¼ clearance. E1C20 11/11/2009 16:3:43 Page 445 Section 20.1/Cutting Operations 445 FIGURE 20.2 Characteristic sheared edges of the work. and die is correct, the two fracture lines meet, resulting in a clean separation of the work into two pieces. The sheared edges of the sheet have characteristic features as in Figure 20.2. At the top of the cut surface is a region called the rollover. This corresponds to the depression made by the punch in the work prior to cutting. It is where initial plastic deformation occurred in the work. Just below the rollover is a relatively smooth region called the burnish. This results from penetration of the punch into the work before fracture began. Beneath the burnish is the fractured zone, a relatively rough surface of the cut edge where continued downward movement of the punch caused fracture of the metal. Finally, at the bottom of the edge is a burr, a sharp corner on the edge caused by elongation of the metal during final separation of the two pieces. 20.1.1 SHEARING, BLANKING, AND PUNCHING The three most important operations in pressworking that cut metal by the shearing mechanism just described are shearing, blanking, and punching. Shearing is a sheet-metal cutting operation along a straight line between two cutting edges, as shown in Figure 20.3(a). Shearing is typically used to cut large sheets into smaller sections for subsequent pressworking operations. It is performed on a machine called a power shears, or squaring shears. The upper blade of the power shears is often inclined, as shown in Figure 20.3(b), to reduce the required cutting force. Blanking involves cutting of the sheet metal along a closed outline in a single step to separate the piece from the surrounding stock, as in Figure 20.4(a). The part that is cut out is the desired product in the operation and is called the blank. Punching is similar to blanking except that it produces a hole, and the separated piece is scrap, called the slug. The remaining stock is the desired part. The distinction is illustrated in Figure 20.4(b). FIGURE 20.3 Shearing operation: (a) side view of the shearing operation; (b) front view of power shears equipped with inclined upper cutting blade. Symbol v indicates motion. E1C20 11/11/2009 446 16:3:44 Page 446 Chapter 20/Sheet Metalworking FIGURE 20.4 (a) Blanking and (b) punching. 20.1.2 ENGINEERING ANALYSIS OF SHEET-METAL CUTTING Process parameters in sheet-metal cutting are clearance between punch and die, stock thickness, type of metal and its strength, and length of the cut. Let us define these parameters and some of the relationships among them. Clearance The clearance c in a shearing operation is the distance between the punch and die, as shown in Figure 20.1(a). Typical clearances in conventional pressworking range between 4% and 8% of the sheet-metal thickness t. The effect of improper clearances is illustrated in Figure 20.5. If the clearance is too small, then the fracture lines tend to pass each other, causing a double burnishing and larger cutting forces. If the clearance is too large, the metal becomes pinched between the cutting edges and an excessive burr results. In special operations requiring very straight edges, such as shaving and fine blanking (Section 20.1.3), clearance is only about 1% of stock thickness. The correct clearance depends on sheet-metal type and thickness. The recommended clearance can be calculated by the following formula: c ¼ Ac t ð20:1Þ where c ¼ clearance, mm (in); Ac ¼ clearance allowance; and t ¼ stock thickness, mm (in). The clearance allowance is determined according to type of metal. For convenience, metals are classified into three groups given in Table 20.1, with an associated allowance value for each group. FIGURE 20.5 Effect of clearance: (a) clearance too small causesless-thanoptimal fracture and excessive forces; and (b) clearance too large causes oversized burr. Symbols v and F indicate motion and applied force, respectively. E1C20 11/11/2009 16:3:45 Page 447 Section 20.1/Cutting Operations TABLE 20.1 447 Clearance allowance value for three sheet-metal groups. Metal Group Ac 1100S and 5052S aluminum alloys, all tempers 2024ST and 6061ST aluminum alloys; brass, all tempers; soft coldrolled steel, soft stainless steel Cold-rolled steel, half hard; stainless steel, half-hard and full-hard 0.045 0.060 0.075 Compiled from [3]. These calculated clearance values can be applied to conventional blanking and holepunching operations to determine the proper punch and die sizes. The die opening must always be larger than the punch size (obviously). Whether to add the clearance value to the die size or subtract it from the punch size depends on whether the part being cut out is a blank or a slug, as illustrated in Figure 20.6 for a circular part. Because of the geometry of the sheared edge, the outer dimension of the part cut out of the sheet will be larger than the hole size. Thus, punch and die sizes for a round blank of diameter Db are determined as Blanking punch diameter ¼ Db  2c ð20:2aÞ Blanking die diameter ¼ Db ð20:2bÞ Punch and die sizes for a round hole of diameter Dh are determined as: Hole punch diameter ¼ Dh ð20:3aÞ Hole die diameter ¼ Dh þ 2c ð20:3bÞ In order for the slug or blank to drop through the die, the die opening must have an angular clearance (see Figure 20.7) of 0.25 to 1.5 on each side. Cutting Forces Estimates of cutting force are important because this force determines the size (tonnage) of the press needed. Cutting force F in sheet metalworking can be determined by F ¼ StL ð20:4Þ where S ¼ shear strength of the sheet metal, MPa (lb/in2); t ¼ stock thickness, mm (in), and L ¼ length of the cut edge, mm (in). In blanking, punching, slotting, and similar operations, L is the perimeter length of the blank or hole being cut. The minor effect of clearance in determining the value of L can be neglected. If shear strength is unknown, an FIGURE 20.6 Die size determines blank size Db; punch size determines hole size Dh.; c ¼ clearance. E1C20 11/11/2009 448 16:3:46 Page 448 Chapter 20/Sheet Metalworking FIGURE 20.7 Angular clearance. alternative way of estimating the cutting force is to use the tensile strength: F ¼ 0:7ðTSÞtL ð20:5Þ where TS ¼ ultimate tensile strength MPa (lb/in2). These equations for estimating cutting force assume that the entire cut along the sheared edge length L is made at the same time. In this case the cutting force will be a maximum. It is possible to reduce the maximum force by using an angled cutting edge on the punch or die, as in Figure 20.3(b). The angle (called the shear angle), spreads the cut over time and reduces the force experienced at any one moment. However, the total energy required in the operation is the same, whether it is concentrated into a brief moment or distributed over a longer time period. Example 20.1 Blanking Clearance and Force A round disk of 150-mm diameter is to be blanked from a strip of 3.2-mm, half-hard coldrolled steel whose shear strength ¼ 310 MPa. Determine (a) the appropriate punch and die diameters, and (b) blanking force. Solution: (a) From Table 20.1, the clearance allowance for half-hard cold-rolled steel is Ac ¼ 0.075. Accordingly, c ¼ 0:075ð3:2 mmÞ ¼ 0:24 mm The blank is to have a diameter ¼ 150 mm, and die size determines blank size. Therefore, Die opening diameter ¼ 150:00 mm Punch diameter ¼ 150  2ð0:24Þ ¼ 149:52 mm (b) To determine the blanking force, we assume that the entire perimeter of the part is blanked at one time. The length of the cut edge is L ¼ pDb ¼ 150p ¼ 471:2 mm and the force is F ¼ 310ð471:2Þð3:2Þ ¼ 467; 469 N½ 53 tons n 20.1.3 OTHER SHEET-METAL-CUTTING OPERATIONS In addition to shearing, blanking, and punching, there are several other cutting operations in pressworking. The cutting mechanism in each case involves the same shearing action discussed above. E1C20 11/11/2009 16:3:47 Page 449 Section 20.1/Cutting Operations 449 FIGURE 20.8 (a) Cutoff and (b) parting. Cutoff and Parting Cutoff is a shearing operation in which blanks are separated from a sheet-metal strip by cutting the opposite sides of the part in sequence, as shown in Figure 20.8(a). With each cut, a new part is produced. The features of a cutoff operation that distinguish it from a conventional shearing operation are (1) the cut edges are not necessarily straight, and (2) the blanks can be nested on the strip in such a way that scrap is avoided. Parting involves cutting a sheet-metal strip by a punch with two cutting edges that match the opposite sides of the blank, as shown in Figure 20.8(b). This might be required because the part outline has an irregular shape that precludes perfect nesting of the blanks on the strip. Parting is less efficient than cutoff in the sense that it results in some wasted material. Slotting, Perforating, and Notching Slotting is the term sometimes used for a punching operation that cuts out an elongated or rectangular hole, as pictured in Figure 20.9(a). Perforating involves the simultaneous punching of a pattern of holes in sheet metal, as in Figure 20.9(b). The hole pattern is usually for decorative purposes, or to allow passage of light, gas, or fluid. To obtain the desired outline of a blank, portions of the sheet metal are often removed by notching and seminotching. Notching involves cutting out a portion of metal from the side of the sheet or strip. Seminotching removes a portion of metal from the interior of the sheet. These operations are depicted in Figure 20.9(c). Seminotching might seem to the reader to be the same as a punching or slotting operation. The difference is Slot V Slug (a) FIGURE 20.9 Notching Cutoff line Seminotching (b) Completed blank (c) (a) Slotting, (b) perforating, (c) notching and seminotching. Symbol v indicates motion of strip. E1C20 11/11/2009 450 16:3:49 Page 450 Chapter 20/Sheet Metalworking FIGURE 20.10 (a) Shaving and (b) fine blanking. Symbols: v ¼ motion of punch, Fh ¼ blank holding force. that the metal removed by seminotching creates part of the blank outline, while punching and slotting create holes in the blank. Trimming, Shaving, and Fine Blanking Trimming is a cutting operation performed on a formed part to remove excess metal and establish size. The term has the same basic meaning here as in forging (Section 19.4). A typical example in sheet metalwork is trimming the upper portion of a deep drawn cup to leave the desired dimensions on the cup. Shaving is a shearing operation performed with very small clearance to obtain accurate dimensions and cut edges that are smooth and straight, as pictured in Figure 20.10(a). Shaving is typically performed as a secondary or finishing operation on parts that have been previously cut. Fine blanking is a shearing operation used to blank sheet-metal parts with close tolerances and smooth, straight edges in one step, as illustrated in Figure 20.10(b). At the start of the cycle, a pressure pad with a V-shaped projection applies a holding force Fh against the work adjacent to the punch in order to compress the metal and prevent distortion. The punch then descends with a slower-than-normal velocity and smaller clearances to provide the desired dimensions and cut edges. The process is usually reserved for relatively small stock thicknesses. 20.2 BENDING OPERATIONS Bending in sheet-metal work is defined as the straining of the metal around a straight axis, as in Figure 20.11. During the bending operation, the metal on the inside of the neutral plane is compressed, while the metal on the outside of the neutral plane is stretched. These strain conditions can be seen in Figure 20.11(b). The metal is plastically deformed so that the bend takes a permanent set upon removal of the stresses that caused it. Bending produces little or no change in the thickness of the sheet metal. 20.2.1 V-BENDING AND EDGE BENDING Bending operations are performed using punch and die tooling. The two common bending methods and associated tooling are V-bending, performed with a V-die; and edge bending, performed with a wiping die. These methods are illustrated in Figure 20.12. E1C20 11/11/2009 16:3:49 Page 451 Section 20.2/Bending Operations Metal stretched w Neutral axis plane R FIGURE 20.11 (a) Bending of sheet metal; (b) both compression and tensile elongation of the metal occur in bending. Neutral axis t α′ Bend axis 451 (a) Metal compressed (b) In V-bending, the sheet metal is bent between a V-shaped punch and die. Included angles ranging from very obtuse to very acute can be made with V-dies. V-bending is generally used for low-production operations. It is often performed on a press brake (Section 20.5.2), and the associated V-dies are relatively simple and inexpensive. Edge bending involves cantilever loading of the sheet metal. A pressure pad is used to apply a force Fh to hold the base of the part against the die, while the punch forces the part to yield and bend over the edge of the die. In the setup shown in Figure 20.12(b), edge bending is limited to bends of 90 or less. More complicated wiping dies can be designed for bend angles greater than 90 . Because of the pressure pad, wiping dies are more complicated and costly than V-dies and are generally used for high-production work. 20.2.2 ENGINEERING ANALYSIS OF BENDING Some of the important terms in sheet-metal bending are identified in Figure 20.11. The metal of thickness t is bent through an angle called the bend angle a. This results in a sheet-metal part with an included angle a0, where a + a0 ¼ 180 . The bend radius R is normally specified on the inside of the part, rather than at the neutral axis, and is determined by the radius on the tooling used to perform the operation. The bend is made over the width of the workpiece w. Bend Allowance If the bend radius is small relative to stock thickness, the metal tends to stretch during bending. It is important to be able to estimate the amount of stretching FIGURE 20.12 Two common bending methods: (a) V-bending and (b) edge bending; (1) before and (2) after bending. Symbols: v ¼ motion, F ¼ applied bending force, Fh ¼ blank. E1C20 11/11/2009 452 16:3:50 Page 452 Chapter 20/Sheet Metalworking that occurs, if any, so that the final part length will match the specified dimension. The problem is to determine the length of the neutral axis before bending to account for stretching of the final bent section. This length is called the bend allowance, and it can be estimated as follows: Ab ¼ 2p a ðR þ K ba tÞ 360 ð20:6Þ where Ab ¼ bend allowance, mm (in); a ¼ bend angle, degrees; R ¼ bend radius, mm (in); t ¼ stock thickness, mm (in); and Kba is factor to estimate stretching. The following design values are recommended for Kba [3]: if <2t, Kba ¼ 0.33; and if R  2t, Kba ¼ 0.50. The values of Kba predict that stretching occurs only if bend radius is small relative to sheet thickness. Springback When the bending pressure is removed at the end of the deformation operation, elastic energy remains in the bent part, causing it to recover partially toward its original shape. This elastic recovery is called springback, defined as the increase in included angle of the bent part relative to the included angle of the forming tool after the tool is removed. This is illustrated in Figure 20.13 and is expressed: SB ¼ a0  a0 t a0 t ð20:7Þ where SB ¼ springback; a0 ¼ included angle of the sheet-metal part, degrees; and a0 t ¼ included angle of the bending tool, degrees. Although not as obvious, an increase in the bend radius also occurs due to elastic recovery. The amount of springback increases with modulus of elasticity E and yield strength Y of the work metal. Compensation for springback can be accomplished by several methods. Two common methods are overbending and bottoming. In overbending, the punch angle and radius are fabricated slightly smaller than the specified angle on the final part so that the sheet metal springs back to the desired value. Bottoming involves squeezing the part at the end of the stroke, thus plastically deforming it in the bend region. Bending Force The force required to perform bending depends on the geometry of the punch-and-die and the strength, thickness, and length of the sheet metal. The maximum FIGURE 20.13 Springback in bending shows itself as a decrease in bend angle and an increase in bend radius: (1) during the operation, the work is forced to take the radius Rt and included angle a0 t ¼ determined by the bending tool (punch in V-bending); (2) after the punch is removed, the work springs back to radius R and included angle a0 . Symbol: F ¼ applied bending force. E1C20 11/11/2009 16:3:51 Page 453 453 Section 20.2/Bending Operations FIGURE 20.14 Die opening dimension D: (a) V-die, (b) wiping die. bending force can be estimated by means of the following equation: F¼ Kbf ðTSÞwt2 D ð20:8Þ where F ¼ bending force, N (lb); TS ¼ tensile strength of the sheet metal, MPa (lb/in2); w ¼ width of part in the direction of the bend axis, mm (in); t ¼ stock thickness, mm (in); and D ¼ die opening dimension as defined in Figure 20.14, mm (in). Eq. (20.8) is based on bending of a simple beam in mechanics, and Kbf is a constant that accounts for differences encountered in an actual bending process. Its value depends on type of bending: for V-bending, Kbf ¼ 1.33; and for edge bending, Kbf ¼ 0.33. Example 20.2 Sheet-Metal Bending A sheet-metal blank is to be bent as shown in Figure 20.15. The metal has a modulus of elasticity ¼ 205 (103) MPa, yield strength ¼ 275 MPa, and tensile strength ¼ 450 MPa. Determine (a) the starting blank size and (b) the bending force if a V-die is used with a die opening dimension ¼ 25 mm. Solution: (a) The starting blank ¼ 44.5 mm wide. Its length ¼ 38 þ Ab þ 25 (mm). For the included angle a0 ¼ 120 , the bend angle a ¼ 60 . The value of Kba in Eq. (20.6) ¼ 0.33 since R=t ¼ 4.75=3.2 ¼ 1.48 (less than 2.0). Ab ¼ 2p 60 ð4:75 þ 0:33  3:2Þ ¼ 6:08 mm 360 Length of the blank is therefore 38 + 6.08 + 25 ¼ 69.08 mm. (b) Force is obtained from Eq. (20.8) using Kbf ¼ 1.33. F¼ 1:33ð450Þð44:5Þð3:2Þ2 ¼ 10; 909 N 2:5 n 20.2.3 OTHER BENDING AND FORMING OPERATIONS Some sheet-metal operations involve bending over a curved axis rather than a straight axis, or they have other features that differentiate them from the bending operations described above. w = 44.5 38 R = 4.75 120° (Side view) 25 t = 3.2 FIGURE 20.15 Sheet-metal part of Example 20.220.2 (dimensions in mm). (End view) E1C20 11/11/2009 454 16:3:52 Page 454 Chapter 20/Sheet Metalworking FIGURE 20.16 Flanging: (a) straight flanging, (b) stretch flanging, and (c) shrink flanging. FIGURE 20.17 (a) Hemming, (b) seaming, and (c) curling. Flanging, Hemming, Seaming, and Curling Flanging is a bending operation in which the edge of a sheet-metal part is bent at a 90 angle (usually) to form a rim or flange. It is often used to strengthen or stiffen sheet metal. The flange can be formed over a straight bend axis, as illustrated in Figure 20.16(a), or it can involve some stretching or shrinking of the metal, as in (b) and (c). Hemming involves bending the edge of the sheet over on itself, in more than one bending step. This is often done to eliminate the sharp edge on the piece, to increase stiffness, and to improve appearance. Seaming is a related operation in which two sheet-metal edges are assembled. Hemming and seaming are illustrated in Figure 20.17(a) and (b). Curling, also called beading, forms the edges of the part into a roll or curl, as in Figure 20.17(c). As in hemming, it is done for purposes of safety, strength, and aesthetics. Examples of products in which curling is used include hinges, pots and pans, and pocket-watch cases. These examples show that curling can be performed over straight or curved bend axes. Miscellaneous Bending Operations Various other bending operations are depicted in Figure 20.18 to illustrate the variety of shapes that can be bent. Most of these operations are performed in relatively simple dies similar to V-dies. 20.3 DRAWING Drawing is a sheet-metal-forming operation used to make cup-shaped, box-shaped, or other complex-curved and concave parts. It is performed by placing a piece of sheet metal over a die cavity and then pushing the metal into the opening with a punch, as in Figure 20.19. The blank must usually be held down flat against the die by a blankholder. Common parts made by drawing include beverage cans, ammunition shells, sinks, cooking pots, and automobile body panels. 20.3.1 MECHANICS OF DRAWING Drawing of a cup-shaped part is the basic drawing operation, with dimensions and parameters as pictured in Figure 20.19. A blank of diameter Db is drawn into a die cavity by means of a punch with diameter Dp. The punch and die must have corner radii, given by E1C20 11/11/2009 16:3:54 Page 455 Section 20.3/Drawing FIGURE 20.18 Miscellaneous bending operations: (a) channel bending, (b) U-bending, (c) air bending, (d) offset bending, (e) corrugating, and (f) tube forming. Symbol: F ¼ applied force. FIGURE 20.19 (a) Drawing of a cupshaped part: (1) start of operation before punch contacts work, and (2) near end of stroke; and (b) corresponding workpart: (1) starting blank, and (2) drawn part. Symbols: c ¼ clearance, Db ¼ blank diameter, Dp ¼ punch diameter, Rd ¼ die corner radius, Rp ¼ punch corner radius, F ¼ drawing force, Fh ¼ holding force. 455 E1C20 11/11/2009 456 16:3:56 Page 456 Chapter 20/Sheet Metalworking Rp and Rd. If the punch and die were to have sharp corners (Rp and Rd ¼ 0), a hole-punching operation (and not a very good one) would be accomplished rather than a drawing operation. The sides of the punch and die are separated by a clearance c. This clearance in drawing is about 10% greater than the stock thickness: c ¼ 1:1 t ð20:9Þ The punch applies a downward force F to accomplish the deformation of the metal, and a downward holding force Fh is applied by the blankholder, as shown in the sketch. As the punch proceeds downward toward its final bottom position, the work experiences a complex sequence of stresses and strains as it is gradually formed into the shape defined by the punch and die cavity. The stages in the deformation process are illustrated in Figure 20.20. As the punch first begins to push into the work, the metal is subjected to a bending operation. The sheet is simply bent over the corner of the punch and the corner of the die, as in Figure 20.20(2). The outside perimeter of the blank moves in toward the center in this first stage, but only slightly. As the punch moves further down, a straightening action occurs in the metal that was previously bent over the die radius, as in Figure 20.20(3). The metal at the bottom of the cup, as well as along the punch radius, has been moved downward with the punch, but the metal that was bent over the die radius must now be straightened in order to be pulled into the clearance to form the wall of the cylinder. At the same time, more metal must be added to replace that being used in the cylinder wall. This new metal comes from the FIGURE 20.20 Stages in deformation of the work in deep drawing: (1) punch makes initial contact with work, (2) bending, (3) straightening, (4) friction and compression, and (5) final cup shape showing effects of thinning in the cup walls. Symbols: v ¼ motion of punch, F ¼ punch force, Fh ¼ blankholder force. E1C20 11/11/2009 16:3:56 Page 457 Section 20.3/Drawing 457 outside edge of the blank. The metal in the outer portions of the blank is pulled or drawn toward the die opening to resupply the previously bent and straightened metal now forming the cylinder wall. This type of metal flow through a constricted space gives the drawing process its name. During this stage of the process, friction and compression play important roles in the flange of the blank. In order for the material in the flange to move toward the die opening, friction between the sheet metal and the surfaces of the blankholder and the die must be overcome. Initially, static friction is involved until the metal starts to slide; then, after metal flow begins, dynamic friction governs the process. The magnitude of the holding force applied by the blankholder, as well as the friction conditions at the two interfaces, are determining factors in the success of this aspect of the drawing operation. Lubricants or drawing compounds are generally used to reduce friction forces. In addition to friction, compression is also occurring in the outer edge of the blank. As the metal in this portion of the blank is drawn toward the center, the outer perimeter becomes smaller. Because the volume of metal remains constant, the metal is squeezed and becomes thicker as the perimeter is reduced. This often results in wrinkling of the remaining flange of the blank, especially when thin sheet metal is drawn, or when the blankholder force is too low. It is a condition which cannot be corrected once it has occurred. The friction and compression effects are illustrated in Figure 20.20(4). The holding force applied by the blankholder is now seen to be a critical factor in deep drawing. If it is too small, wrinkling occurs. If it is too large, it prevents the metal from flowing properly toward the die cavity, resulting in stretching and possible tearing of the sheet metal. Determining the proper holding force involves a delicate balance between these opposing factors. Progressive downward motion of the punch results in a continuation of the metal flow caused by drawing and compression. In addition, some thinning of the cylinder wall occurs, as in Figure 20.20(5). The force being applied by the punch is opposed by the metal in the form of deformation and friction in the operation. A portion of the deformation involves stretching and thinning of the metal as it is pulled over the edge of the die opening. Up to 25% thinning of the side wall may occur in a successful drawing operation, mostly near the base of the cup. 20.3.2 ENGINEERING ANALYSIS OF DRAWING It is important to assess the limitations on the amount of drawing that can be accomplished. This is often guided by simple measures that can be readily calculated for a given operation. In addition, drawing force and holding force are important process variables. Finally, the starting blank size must be determined. Measures of Drawing One of the measures of the severity of a deep drawing operation is the drawing ratio DR. This is most easily defined for a cylindrical shape as the ratio of blank diameter Db to punch diameter Dp. In equation form, DR ¼ Db Dp ð20:10Þ The drawing ratio provides an indication, albeit a crude one, of the severity of a given drawing operation. The greater the ratio, the more severe the operation. An approximate upper limit on the drawing ratio is a value of 2.0. The actual limiting value for a given operation depends on punch and die corner radii (Rp and Rd), friction conditions, depth of draw, and characteristics of the sheet metal (e.g., ductility, degree of directionality of strength properties in the metal). E1C20 11/11/2009 458 16:3:57 Page 458 Chapter 20/Sheet Metalworking Another way to characterize a given drawing operation is by the reduction r, where r¼ Db  Dp Db ð20:11Þ It is very closely related to drawing ratio. Consistent with the previous limit on DR (DR  2.0), the value of reduction r should be less than 0.50. A third measure in deep drawing is the thickness-to-diameter ratio t/Db (thickness of the starting blank t divided by the blank diameter Db). Often expressed as a percentage, it is desirable for the t/Db ratio to be greater than 1%. As t/Db decreases, tendency for wrinkling (Section 20.3.4) increases. In cases where these limits on drawing ratio, reduction, and t/Db ratio are exceeded by the design of the drawn part, the blank must be drawn in two or more steps, sometimes with annealing between the steps. Example 20.3 Cup Drawing A drawing operation is used to form a cylindrical cup with inside diameter ¼ 75 mm and height ¼ 50 mm. The starting blank size ¼ 138 mm and the stock thickness ¼ 2.4 mm. Based on these data, is the operation feasible? Solution: To assess feasibility, we determine the drawing ratio, reduction, and thicknessto-diameter ratio. DR ¼ 138=75 ¼ 1:84 r ¼ ð138  75Þ=138 ¼ 0:4565 ¼ 45:65% t=Db ¼ 2:4=138 ¼ 0:017 ¼ 1:7% According to these measures, the drawing operation is feasible. The drawing ratio is less than 2.0, the reduction is less than 50%, and the t/Db ratio is greater than 1%. These are n general guidelines frequently used to indicate technical feasibility. Forces The drawing force required to perform a given operation can be estimated roughly by the formula:   Db F ¼ pDp tðTSÞ  0:7 ð20:12Þ Dp where F ¼ drawing force, N (lb); t ¼ original blank thickness, mm (in); TS ¼ tensile strength, MPa (lb/in2); and Db and Dp are the starting blank diameter and punch diameter, respectively, mm (in). The constant 0.7 is a correction factor to account for friction. Eq. (20.12) estimates the maximum force in the operation. The drawing force varies throughout the downward movement of the punch, usually reaching its maximum value at about one-third the length of the punch stroke. The holding force is an important factor in a drawing operation. As a rough approximation, the holding pressure can be set at a value ¼ 0.015 of the yield strength of the sheet metal [8]. This value is then multiplied by that portion of the starting area of the blank that is to be held by the blankholder. In equation form, n  2 o F h ¼ 0:015Yp D2b  Dp þ 2:2t þ 2Rd ð20:13Þ where Fh ¼ holding force in drawing, N (lb); Y ¼ yield strength of the sheet metal, MPa (lb/in2); t ¼ starting stock thickness, mm (in); Rd ¼ die corner radius, mm (in); and the other terms have been previously defined. The holding force is usually about one-third the drawing force [10]. E1C20 11/11/2009 16:3:57 Page 459 Section 20.3/Drawing Example 20.4 Forces in Drawing 459 For the drawing operation of Example 20.3, determine (a) drawing force and (b) holding force, given that the tensile strength of the sheet metal (low-carbon steel) ¼ 300 MPa and yield strength ¼ 175 MPa. The die corner radius ¼ 6 mm. Solution: (a) Maximum drawing force is given by Eq. (20.12):   138 F ¼ pð75Þð2:4Þð300Þ  0:7 ¼ 193; 396 N 75 (b) Holding force is estimated by Eq. (20.13): F h ¼ 0:015ð175Þ pð1382  ð75 þ 2:2  2:4 þ 2  6Þ2 Þ ¼ 86; 824 N n Blank Size Determination For the final dimensions to be achieved on the cylindrical drawn shape, the correct starting blank diameter is needed. It must be large enough to supply sufficient metal to complete the cup. Yet if there is too much material, unnecessary waste will result. For drawn shapes other than cylindrical cups, the same problem of estimating the starting blank size exists, only the shape of the blank may be other than round. The following is a reasonable method for estimating the starting blank diameter in a deep drawing operation that produces a round part (e.g., cylindrical cup and more complex shapes so long as they are axisymmetric). Because the volume of the final product is the same as that of the starting sheet-metal blank, then the blank diameter can be calculated by setting the initial blank volume equal to the final volume of the product and solving for diameter Db. To facilitate the calculation, it is often assumed that negligible thinning of the part wall occurs. 20.3.3 OTHER DRAWING OPERATIONS Our discussion has focused on a conventional cup-drawing operation that produces a simple cylindrical shape in a single step and uses a blankholder to facilitate the process. Let us consider some of the variations of this basic operation. Redrawing If the shape change required by the part design is too severe (drawing ratio is too high), complete forming of the part may require more than one drawing step. The second drawing step, and any further drawing steps if needed, are referred to as redrawing. A redrawing operation is illustrated in Figure 20.21. When the part design indicates a drawing ratio that is too large to form the part in a single step, the following is a general guide to the amount of reduction that can be taken in FIGURE 20.21 Redrawing of a cup: (1) start of redraw, and (2) end of stroke. Symbols: v ¼ punch velocity, F ¼ applied punch force, Fh ¼ blankholder force. E1C20 11/11/2009 460 16:3:58 Page 460 Chapter 20/Sheet Metalworking FIGURE 20.22 Reverse drawing: (1) start and (2) completion. Symbols: v ¼ punch velocity, F ¼ applied punch force, Fh ¼ blankholder force. each drawing operation [10]: For the first draw, the maximum reduction of the starting blank should be 40% to 45%; for the second draw (first redraw), the maximum reduction should be 30%; and for the third draw (second redraw), the maximum reduction should be 16%. A related operation is reverse drawing, in which a drawn part is positioned face down on the die so that the second drawing operation produces a configuration such as that shown in Figure 20.22. Although it may seem that reverse drawing would produce a more severe deformation than redrawing, it is actually easier on the metal. The reason is that the sheet metal is bent in the same direction at the outside and inside corners of the die in reverse drawing; while in redrawing the metal is bent in the opposite directions at the two corners. Because of this difference, the metal experiences less strain hardening in reverse drawing and the drawing force is lower. Drawing of Shapes Other than Cylindrical Cups Many products require drawing of shapes other than cylindrical cups. The variety of drawn shapes include square or rectangular boxes (as in sinks), stepped cups, cones, cups with spherical rather than flat bases, and irregular curved forms (as in automobile body panels). Each of these shapes presents unique technical problems in drawing. Eary and Reed [2] provide a detailed discussion of the drawing of these kinds of shapes. Drawing Without a Blankholder One of the primary functions of the blankholder is to prevent wrinkling of the flange while the cup is being drawn. The tendency for wrinkling is reduced as the thickness-to-diameter ratio of the blank increases. If the t=Db ratio is large enough, drawing can be accomplished without a blankholder, as in Figure 20.23. FIGURE 20.23 Drawing without a blankholder: (1) start of process, (2) end of stroke. Symbols v and F indicate motion and applied force, respectively. E1C20 11/11/2009 16:3:59 Page 461 Section 20.4/Other Sheet-Metal-Forming Operations 461 FIGURE 20.24 Common defects in drawn parts: (a) wrinkling can occur either in the flange or (b) in the wall, (c) tearing, (d) earing, and (e) surface scratches. The limiting condition for drawing without a blankholder can be estimated from the following [5]: Db  Dp < 5t ð20:14Þ The draw die must have the shape of a funnel or cone to permit the material to be drawn properly into the die cavity. When drawing without a blankholder is feasible, it has the advantages of lower cost tooling and a simpler press, because the need to separately control the movements of the blankholder and punch can be avoided. 20.3.4 DEFECTS IN DRAWING Sheet-metal drawing is a more complex operation than cutting or bending, and more things can go wrong. A number of defects can occur in a drawn product, some of which we have already alluded to. Following is a list of common defects, with sketches in Figure 20.24: (a) Wrinkling in the flange. Wrinkling in a drawn part consists of a series of ridges that form radially in the undrawn flange of the workpart due to compressive buckling. (b) Wrinkling in the wall. If and when the wrinkled flange is drawn into the cup, these ridges appear in the vertical wall. (c) Tearing. Tearing is an open crack in the vertical wall, usually near the base of the drawn cup, due to high tensile stresses that cause thinning and failure of the metal at this location. This type of failure can also occur as the metal is pulled over a sharp die corner. (d) Earing. This is the formation of irregularities (called ears) in the upper edge of a deep drawn cup, caused by anisotropy in the sheet metal. If the material is perfectly isotropic, ears do not form. (e) Surface scratches. Surface scratches can occur on the drawn part if the punch and die are not smooth or if lubrication is insufficient. 20.4 OTHER SHEET-METAL-FORMING OPERATIONS In addition to bending and drawing, several other sheet-metal-forming operations can be accomplished on conventional presses. We classify these as (1) operations performed with metal tooling and (2) operations performed with flexible rubber tooling. E1C20 11/11/2009 462 16:4:0 Page 462 Chapter 20/Sheet Metalworking FIGURE 20.25 Ironing to achieve a more uniform wall thickness in a drawn cup: (1) start of process; (2) during process. Note thinning and elongation of walls. Symbols v and F indicate motion and applied force, respectively. 20.4.1 OPERATIONS PERFORMED WITH METAL TOOLING Operations performed with metal tooling include (1) ironing, (2) coining and embossing, (3) lancing, and (4) twisting. Ironing In deep drawing the flange is compressed by the squeezing action of the blank perimeter seeking a smaller circumference as it is drawn toward the die opening. Because of this compression, the sheet metal near the outer edge of the blank becomes thicker as it moves inward. If the thickness of this stock is greater than the clearance between the punch and die, it will be squeezed to the size of the clearance, a process known as ironing. Sometimes ironing is performed as a separate step that follows drawing. This case is illustrated in Figure 20.25. Ironing makes the cylindrical cup more uniform in wall thickness. The drawn part is therefore longer and more efficient in terms of material usage. Beverage cans and artillery shells, two very high-production items, include ironing among their processing steps to achieve economy in material usage. Coining and Embossing Coining is a bulk deformation operation discussed in the previous chapter. It is frequently used in sheet-metal work to form indentations and raised sections in the part. The indentations result in thinning of the sheet metal, and the raised sections result in thickening of the metal. Embossing is a forming operation used to create indentations in the sheet, such as raised (or indented) lettering or strengthening ribs, as depicted in Figure 20.26. Some stretching and thinning of the metal are involved. This operation may seem similar to coining. However, embossing dies possess matching cavity contours, the punch containing the positive contour and the die containing the negative; whereas coining dies may have quite different cavities in the two die halves, thus causing more significant metal deformation than embossing. FIGURE 20.26 Embossing: (a) cross section of punch and die configuration during pressing; (b) finished part with embossed ribs. E1C20 11/11/2009 16:4:1 Page 463 Section 20.4/Other Sheet-Metal-Forming Operations FIGURE 20.27 Lancing in several forms: (a) cutting and bending; (b) and (c) two types of cutting and forming. (a) (b) 463 (c) Lancing Lancing is a combined cutting and bending or cutting and forming operation performed in one step to partially separate the metal from the sheet. Several examples are shown in Figure 20.27. Among other applications, lancing is used to make louvers in sheetmetal air vents for heating and air conditioning systems in buildings. Twisting Twisting subjects the sheet metal to a torsion loading rather than a bending load, thus causing a twist in the sheet over its length. This type of operation has limited applications. It is used to make such products as fan and propeller blades. It can be performed in a conventional punch and die which has been designed to deform the part in the required twist shape. 20.4.2 RUBBER FORMING PROCESSES The two operations discussed in this article are performed on conventional presses, but the tooling is unusual in that it uses a flexible element (made of rubber or similar material) to effect the forming operation. The operations are (1) the Guerin process, and (2) hydroforming. Guerin Process The Guerin process uses a thick rubber pad (or other flexible material) to form sheet metal over a positive form block, as in Figure 20.28. The rubber pad is confined in a steel container. As the ram descends, the rubber gradually surrounds the sheet, applying pressure to deform it to the shape of the form block. It is limited to relatively FIGURE 20.28 Guerin process: (1) before and (2) after. Symbols v and F indicate motion and applied force, respectively. E1C20 11/11/2009 464 16:4:2 Page 464 Chapter 20/Sheet Metalworking FIGURE 20.29 Hydroform process: (1) start-up, no fluid in cavity; (2) press closed, cavity pressurized with hydraulic fluid; (3) punch pressed into work to form part. Symbols: v ¼ velocity, F ¼ applied force, p ¼ hydraulic pressure. shallow forms, because the pressures developed by the rubber—up to about 10 MPa (1500 lb/in2)—are not sufficient to prevent wrinkling in deeper formed parts. The advantage of the Guerin process is the relatively low cost of the tooling. The form block can be made of wood, plastic, or other materials that are easy to shape, and the rubber pad can be used with different form blocks. These factors make rubber forming attractive in small-quantity production, such as the aircraft industry, where the process was developed. Hydroforming Hydroforming is similar to the Guerin process; the difference is that it substitutes a rubber diaphragm filled with hydraulic fluid in place of the thick rubber pad, as illustrated in Figure 20.29. This allows the pressure that forms the workpart to be increased—to around 100 MPa (15,000 lb/in2)—thus preventing wrinkling in deep formed parts. In fact, deeper draws can be achieved with the hydroform process than with conventional deep drawing. This is because the uniform pressure in hydroforming forces the work to contact the punch throughout its length, thus increasing friction and reducing the tensile stresses that cause tearing at the base of the drawn cup. 20.5 DIES AND PRESSES FOR SHEET-METAL PROCESSES In this section we examine the punch-and-die tooling and production equipment used in conventional sheet-metal processing. 20.5.1 DIES Nearly all of the preceding pressworking operations are performed with conventional punch-and-die tooling. The tooling is referred to as a die. It is custom-designed for the particular part to be produced. The term stamping die is sometimes used for highproduction dies. Typical materials for stamping dies are tool steel types D, A, O, and S (Table 6.5). E1C20 11/11/2009 16:4:3 Page 465 Section 20.5/Dies and Presses for Sheet-Metal Processes 465 FIGURE 20.30 Components of a punch and die for a blanking operation. Components of a Stamping Die The components of a stamping die to perform a simple blanking operation are illustrated in Figure 20.30. The working components are the punch and die, which perform the cutting operation. They are attached to the upper and lower portions of the die set, respectively called the punch holder (or upper shoe) and die holder (lower shoe). The die set also includes guide pins and bushings to ensure proper alignment between the punch and die during the stamping operation. The die holder is attached to the base of the press, and the punch holder is attached to the ram. Actuation of the ram accomplishes the pressworking operation. In addition to these components, a die used for blanking or hole-punching must include a means of preventing the sheet metal from sticking to the punch when it is retracted upward after the operation. The newly created hole in the stock is the same size as the punch, and it tends to cling to the punch on its withdrawal. The device in the die that strips the sheet metal from the punch is called a stripper. It is often a simple plate attached to the die as in Figure 20.30, with a hole slightly larger than the punch diameter. For dies that process strips or coils of sheet metal, a device is required to stop the sheet metal as it advances through the die between press cycles. That device is called (try to guess) a stop. Stops range from simple solid pins located in the path of the strip to block its forward motion, to more complex mechanisms synchronized to rise and retract with the actuation of the press. The simpler stop is shown in Figure 20.30. There are other components in pressworking dies, but the preceding description provides an introduction to the terminology. Types of Stamping Dies Aside from differences in stamping dies related to the operations they perform (e.g., cutting, bending, drawing), other differences deal with the number of separate operations to be performed in each press actuation and how they are accomplished. The type of die considered above performs a single blanking operation with each stroke of the press and is called a simple die. Other dies that perform a single operation include V-dies (Section 20.2.1). More complicated pressworking dies include compound dies, combination dies, and progressive dies. A compound die performs two operations at a single station, such as blanking and punching, or blanking and drawing [2]. A good example is a compound die that blanks and punches a washer. A combination die is less common; it performs two operations at two different stations in the die. Examples of applications include blanking two different parts (e.g., right-hand and left-hand parts), or blanking and then bending the same part [2]. A progressive die performs two or more operations on a sheet-metal coil at two or more stations with each press stroke. The part is fabricated progressively. The coil is fed E1C20 11/11/2009 466 16:4:4 Page 466 Chapter 20/Sheet Metalworking FIGURE 20.31 (a) Progressive die and (b) associated strip development. from one station to the next and different operations (e.g., punching, notching, bending, and blanking) are performed at each station. When the part exits the final station it has been completed and separated (cut) from the remaining coil. Design of a progressive die begins with the layout of the part on the strip or coil and the determination of which operations are to be performed at each station. The result of this procedure is called the strip development. A progressive die and associated strip development are illustrated in Figure 20.31. Progressive dies can have a dozen or more stations. They are the most complicated and most costly stamping dies, economically justified only for complex parts requiring multiple operations at high-production rates. 20.5.2 PRESSES A press used for sheet metalworking is a machine tool with a stationary bed and a powered ram (or slide) that can be driven toward and away from the bed to perform various cutting and forming operations. A typical press, with principal components labeled, is diagrammed in Figure 20.32. The relative positions of the bed and ram are established by the frame, and the ram is driven by mechanical or hydraulic power. When a die is mounted in the press, the punch holder is attached to the ram, and the die holder is attached to a bolster plate of the press bed. Presses are available in a variety of capacities, power systems, and frame types. The capacity of a press is its ability to deliver the required force and energy to accomplish the stamping operation. This is determined by the physical size of the press and by its power system. The power system refers to whether mechanical or hydraulic power is used and the type of drive used to transmit the power to the ram. Production rate is another important aspect of capacity. Type of frame refers to the physical construction of the press. There are two frame types in common use: gap frame and straight-sided frame. Gap Frame Presses The gap frame has the general configuration of the letter C and is often referred to as a C-frame. Gap frame presses provide good access to the die, and E1C20 11/11/2009 16:4:4 Page 467 Section 20.5/Dies and Presses for Sheet-Metal Processes 467 FIGURE 20.32 Components of a typical (mechanical drive) stamping press. they are usually open in the back to permit convenient ejection of stampings or scrap. The principal types of gap frame press are (a) solid gap frame, (b) adjustable bed, (c) openback inclinable, (d) press brake, and (e) turret press. The solid gap frame (sometimes called simply a gap press) has one-piece construction, as shown in Figure 20.32. Presses with this frame are rigid, yet the C-shape allows convenient access from the sides for feeding strip or coil stock. They are available in a range of sizes, with capacities up to around 9000 kN (1000 tons). The model shown in Figure 20.33 has a capacity of 1350 kN (150 tons). The adjustable bed frame press is a variation of the gap frame, in which an adjustable bed is added to accommodate various die sizes. The adjustment feature FIGURE 20.33 Gap frame press for sheet metalworking. (Photo courtesy of E. W. Bliss Company, Hastings, Michigan.). Capacity ¼ 1350 kN (150 tons). E1C20 11/11/2009 468 16:4:8 Page 468 Chapter 20/Sheet Metalworking FIGURE 20.34 Press brake with bed width of 9.15 m (30 ft) and capacity of 11,200 kN (1250 tons); two workers are shown positioning plate stock for bending. (Photo courtesy of Niagara Machine & Tool Works, Buffalo, New York.) results in some sacrifice of tonnage capacity. The open-back inclinable press has a C-frame assembled to a base in such a way that the frame can be tilted back to various angles so that the stampings fall through the rear opening by gravity. Capacities of open-back inclinable presses range between 1 ton and around 2250 kN (250 tons). They can be operated at high speeds—up to around 1000 strokes per minute. The press brake is a gap frame press with a very wide bed. The model in Figure 20.34 has a bed width of 9.15 m (30 ft). This allows a number of separate dies (simple V-bending dies are typical) to be set up in the bed, so that small quantities of stampings can be made economically. These low quantities of parts, sometimes requiring multiple bends at different angles, necessitate a manual operation. For a part requiring a series of bends, the operator moves the starting piece of sheet metal through the desired sequence of bending dies, actuating the press at each die, to complete the work needed. Whereas press brakes are well adapted to bending operations, turret presses are suited to situations in which a sequence of punching, notching, and related cutting operations must be accomplished on sheet-metal parts, as in Figure 20.35. Turret presses have a C-frame, although this construction is not obvious in Figure 20.36. The conventional ram and punch is replaced by a turret containing many punches of different sizes and shapes. The turret works by indexing (rotating) to the position holding the punch to perform the required operation. Beneath the punch turret is a corresponding die turret that positions the die opening for each punch. Between the punch and die is the sheetmetal blank, held by an xy positioning system that operates by computer numerical control (Section 38.3). The blank is moved to the required coordinate position for each cutting operation. Straight-sided Frame Presses For jobs requiring high tonnage, press frames with greater structural rigidity are needed. Straight-sided presses have full sides, giving it a E1C20 11/11/2009 16:4:12 Page 469 Section 20.5/Dies and Presses for Sheet-Metal Processes FIGURE 20.35 Several sheet-metal parts produced on a turret press, showing variety of possible hole shapes. (Photo courtesy of Strippet, Inc., Akron, New York.) FIGURE 20.36 Computer numerical control turret press. (Photo courtesy of Strippet, Inc., Akron, New York.) 469 E1C20 11/11/2009 470 16:4:18 Page 470 Chapter 20/Sheet Metalworking FIGURE 20.37 Straight-sided frame press. (Photo courtesy Greenerd Press & Machine Company, Inc., Nashua, New Hampshire.) box-like appearance as in Figure 20.37. This construction increases the strength and stiffness of the frame. As a result, capacities up to 35,000 kN (4000 tons) are available in straight-sided presses for sheet metalwork. Large presses of this frame type are used for forging (Section 19.3). In all of these presses, gap frame and straight-sided frame, the size is closely correlated to tonnage capacity. Larger presses are built to withstand higher forces in pressworking. Press size is also related to the speed at which it can operate. Smaller presses are generally capable of higher production rates than larger presses. Power and Drive Systems Power systems on presses are either hydraulic or mechanical. Hydraulic presses use a large piston and cylinder to drive the ram. This power system typically provides longer ram strokes than mechanical drives and can develop the full tonnage force throughout the entire stroke. However, it is slower. Its application for sheet metal is normally limited to deep drawing and other forming operations where these load-stroke characteristics are advantageous. These presses are available with one or more independently operated slides, called single action (single slide), double action (two slides), and so on. Double-action presses are useful in deep drawing operations where it is required to separately control the punch force and the blankholder force. There are several types of drive mechanisms used on mechanical presses. These include eccentric, crankshaft, and knuckle joint, illustrated in Figure 20.38. They convert the rotational motion of a drive motor into the linear motion of the ram. A flywheel is used to store the energy of the drive motor for use in the stamping operation. Mechanical presses using these drives achieve very high forces at the bottom of their strokes, and are therefore quite suited to blanking and punching operations. The knuckle joint delivers very high force when it bottoms, and is therefore often used in coining operations. E1C20 11/11/2009 16:4:20 Page 471 Section 20.6/Sheet-Metal Operations Not Performed on Presses 471 FIGURE 20.38 Types of drives for sheet-metal presses: (a) eccentric, (b) crankshaft, and (c) knuckle joint. 20.6 SHEET-METAL OPERATIONS NOT PERFORMED ON PRESSES A number of sheet-metal operations are not performed on conventional stamping presses. In this section we examine several of these processes: (1) stretch forming, (2) roll bending and forming, (3) spinning, and (4) high-energy-rate forming processes. 20.6.1 STRETCH FORMING Stretch forming is a sheet-metal deformation process in which the sheet metal is intentionally stretched and simultaneously bent in order to achieve shape change. The process is illustrated in Figure 20.39 for a relatively simple and gradual bend. The workpart is gripped by one or more jaws on each end and then stretched and bent over a positive die containing the desired form. The metal is stressed in tension to a level above its yield point. When the tension loading is released, the metal has been plastically deformed. The combination of stretching and bending results in relatively little springback in the part. An estimate of the force required in stretch forming can be obtained by multiplying the cross-sectional area of the sheet in the direction of pulling by the flow stress of the metal. In equation form, F ¼ LtY f ð20:15Þ where F ¼ stretching force, N (lb); L ¼ length of the sheet in the direction perpendicular to stretching, mm (in); t ¼ instantaneous stock thickness, mm (in); and Yf ¼ flow stress of the work metal, MPa (lb/in2). The die force Fdie shown in the figure can be determined by balancing vertical force components. FIGURE 20.39 Stretch forming: (1) start of process; (2) form die is pressed into the work with force Fdie, causing it to be stretched and bent over the form. F ¼ stretching force. E1C20 11/11/2009 472 16:4:22 Page 472 Chapter 20/Sheet Metalworking More complex contours than that shown in our figure are possible by stretch forming, but there are limitations on how sharp the curves in the sheet can be. Stretch forming is widely used in the aircraft and aerospace industries to economically produce large sheet-metal parts in the low quantities characteristic of those industries. 20.6.2 ROLL BENDING AND ROLL FORMING The operations described in this section use rolls to form sheet metal. Roll bending is an operation in which (usually) large sheet-metal or plate-metal parts are formed into curved sections by means of rolls. One possible arrangement of the rolls is pictured in Figure 20.40. As the sheet passes between the rolls, the rolls are brought toward each other to a configuration that achieves the desired radius of curvature on the work. Components for large storage tanks and pressure vessels are fabricated by roll bending. The operation can also be used to bend structural shapes, railroad rails, and tubes. A related operation is roll straightening in which nonflat sheets (or other crosssectional forms) are straightened by passing them between a series of rolls. The rolls subject the work to a sequence of decreasing small bends in opposite directions, thus causing it to be straight at the exit. Roll forming (also called contour roll forming) is a continuous bending process in which opposing rolls are used to produce long sections of formed shapes from coil or strip stock. Several pairs of rolls are usually required to progressively accomplish the bending of the stock into the desired shape. The process is illustrated in Figure 20.41 for a U-shaped section. Products made by roll forming include channels, gutters, metal siding sections (for homes), pipes and tubing with seams, and various structural sections. Although roll forming has the general appearance of a rolling operation (and the tooling certainly looks similar), the difference is that roll forming involves bending rather than compressing the work. FIGURE 20.40 Roll bending. 20.6.3 SPINNING Spinning is a metal-forming process in which an axially symmetric part is gradually shaped over a mandrel or form by means of a rounded tool or roller. The tool or roller applies a very localized pressure (almost a point contact) to deform the work by axial and radial motions over the surface of the part. Basic geometric shapes typically produced by spinning include cups, cones, hemispheres, and tubes. There are three types of spinning operations: (1) conventional spinning, (2) shear spinning, and (3) tube spinning. Conventional Spinning Conventional spinning is the basic spinning operation. As illustrated in Figure 20.42, a sheet-metal disk is held against the end of a rotating mandrel Side view FIGURE 20.41 Roll forming of a continuous channel section: (1) straight rolls, (2) partial form, and (3) final form. E1C20 11/11/2009 16:4:23 Page 473 Section 20.6/Sheet-Metal Operations Not Performed on Presses 473 FIGURE 20.42 Conventional spinning: (1) setup at start of process; (2) during spinning; and (3) completion of process. of the desired inside shape of the final part, while the tool or roller deforms the metal against the mandrel. In some cases, the starting workpart is other than a flat disk. The process requires a series of steps, as indicated in the figure, to complete the shaping of the part. The tool position is controlled either by a human operator, using a fixed fulcrum to achieve the required leverage, or by an automatic method such as numerical control. These alternatives are manual spinning and power spinning. Power spinning has the capability to apply higher forces to the operation, resulting in faster cycle times and greater work size capacity. It also achieves better process control than manual spinning. Conventional spinning bends the metal around a moving circular axis to conform to the outside surface of the axisymmetric mandrel. The thickness of the metal therefore remains unchanged (more or less) relative to the starting disk thickness. The diameter of the disk must therefore be somewhat larger than the diameter of the resulting part. The required starting diameter can be figured by assuming constant volume, before and after spinning. Applications of conventional spinning include production of conical and curved shapes in low quantities. Very large diameter parts—up to 5 m (15 ft) or more—can be made by spinning. Alternative sheet-metal processes would require excessively high die costs. The form mandrel in spinning can be made of wood or other soft materials that are easy to shape. It is therefore a low-cost tool compared to the punch and die required for deep drawing, which might be a substitute process for some parts. Shear Spinning In shear spinning, the part is formed over the mandrel by a shear deformation process in which the outside diameter remains constant and the wall thickness is therefore reduced, as in Figure 20.43. This shear straining (and consequent thinning of the metal) distinguishes this process from the bending action in conventional spinning. Several other names have been used for shear spinning, including flow turning, shear forming, and spin forging. The process has been applied in the aerospace industry to form large parts such as rocket nose cones. For the simple conical shape in our figure, the resulting thickness of the spun wall can be readily determined by the sine law relationship: tf ¼ t sin a ð20:16Þ where tf ¼ the final thickness of the wall after spinning, t ¼ the starting thickness of the disk, and a ¼ the mandrel angle (actually the half angle). Thinning is sometimes quantified by the spinning reduction r: r¼ t  tf t ð20:17Þ E1C20 11/11/2009 474 16:4:24 Page 474 Chapter 20/Sheet Metalworking FIGURE 20.43 Shear spinning: (1) setup and (2) completion of process. There are limits to the amount of thinning that the metal will endure in a spinning operation before fracture occurs. The maximum reduction correlates well with reduction of area in a tension test [8]. Tube Spinning Tube spinning is used to reduce the wall thickness and increase the length of a tube by means of a roller applied to the work over a cylindrical mandrel, as in Figure 20.44. Tube spinning is similar to shear spinning except that the starting workpiece is a tube rather than a flat disk. The operation can be performed by applying the roller against the work externally (using a cylindrical mandrel on the inside of the tube) or internally (using a die to surround the tube). It is also possible to form profiles in the walls of the cylinder, as in Figure 20.44(c), by controlling the path of the roller as it moves tangentially along the wall. Spinning reduction for a tube-spinning operation that produces a wall of uniform thickness can be determined as in shear spinning by Eq. (20.17). 20.6.4 HIGH-ENERGY-RATE FORMING Several processes have been developed to form metals using large amounts of energy applied in a very short time. Owing to this feature, these operations are called high- FIGURE 20.44 Tube spinning: (a) external; (b) internal; and (c) profiling. E1C20 11/11/2009 16:4:25 Page 475 Section 20.6/Sheet-Metal Operations Not Performed on Presses 475 FIGURE 20.45 Explosive forming: (1) setup, (2) explosive is detonated, and (3) shock wave forms part and plume escapes water surface. energy-rate forming (HERF) processes. They include explosive forming, electrohydraulic forming, and electromagnetic forming. Explosive Forming Explosive forming involves the use of an explosive charge to form sheet (or plate) metal into a die cavity. One method of implementing the process is illustrated in Figure 20.45. The workpart is clamped and sealed over the die, and a vacuum is created in the cavity beneath. The apparatus is then placed in a large vessel of water. An explosive charge is placed in the water at a certain distance above the work. Detonation of the charge results in a shock wave whose energy is transmitted by the water to cause rapid forming of the part into the cavity. The size of the explosive charge and the distance at which it is placed above the part are largely a matter of art and experience. Explosive forming is reserved for large parts, typical of the aerospace industry. Electrohydraulic Forming Electrohydraulic forming is a HERF process in which a shock wave to deform the work into a die cavity is generated by the discharge of electrical energy between two electrodes submerged in a transmission fluid (water). Owing to its principle of operation, this process is also called electric discharge forming. The setup for the process is illustrated in Figure 20.46. Electrical energy is accumulated in large capacitors and then released to the electrodes. Electrohydraulic forming is similar to explosive forming. The difference is in the method of generating the energy and the smaller amounts of energy that are released. This limits electrohydraulic forming to much smaller part sizes. FIGURE 20.46 Electrohydraulic forming setup. E1C20 11/11/2009 476 16:4:26 Page 476 Chapter 20/Sheet Metalworking FIGURE 20.47 Electromagnetic forming: (1) setup in which coil is inserted into tubular workpart surrounded by die; (2) formed part. Electromagnetic Forming Electromagnetic forming, also called magnetic pulse forming, is a process in which sheet metal is deformed by the mechanical force of an electromagnetic field induced in the workpart by an energized coil. The coil, energized by a capacitor, produces a magnetic field. This generates eddy currents in the work that produce their own magnetic field. The induced field opposes the primary field, producing a mechanical force that deforms the part into the surrounding cavity. Developed in the 1960s, electromagnetic forming is the most widely used HERF process [10]. It is typically used to form tubular parts, as illustrated in Figure 20.47. 20.7 BENDING OF TUBE STOCK Several methods of producing tubes and pipes are discussed in the previous chapter, and tube spinning is described in Section 20.6.3. In this section, we examine methods by which tubes are bent and otherwise formed. Bending of tube stock is more difficult than sheet stock because a tube tends to collapse and fold when attempts are made to bend it. Special flexible mandrels are usually inserted into the tube prior to bending to support the walls during the operation. Some of the terms in tube bending are defined in Figure 20.48. The radius of the bend R is defined with respect to the centerline of the tube. When the tube is bent, the wall on the inside of the bend is in compression, and the wall at the outside is in tension. These stress conditions cause thinning and elongation of the outer wall and thickening and shortening of the inner wall. As a result, there is a tendency for the inner and outer walls to be forced toward each other to cause the cross section of the tube to flatten. Because of this flattening tendency, the minimum bend radius R that the tube can be bent is about 1.5 times the diameter D when a mandrel is used and 3.0 times D when no mandrel is used [10]. The exact value depends on the wall factor WF, which is the diameter D divided by wall thickness t. Higher values of WF increase the minimum bend FIGURE 20.48 Dimensions and terms for a bent tube: D ¼ outside diameter of tube, R ¼ bend radius, t ¼ wall thickness. E1C20 11/11/2009 16:4:27 Page 477 References 477 FIGURE 20.49 Tube bending methods: (a) stretch bending, (b) draw bending, and (c) compression bending. For each method: (1) start of process, and (2) during bending. Symbols v and F indicate motion and applied force, respectively. radius; that is, tube bending is more difficult for thin walls. Ductility of the work material is also an important factor in the process. Several methods to bend tubes (and similar sections) are illustrated in Figure 20.49. Stretch bending is accomplished by pulling and bending the tube around a fixed form block, as in Figure 20.49(a). Draw bending is performed by clamping the tube against a form block, and then pulling the tube through the bend by rotating the block as in (b). A pressure bar is used to support the work as it is being bent. In compression bending, a wiper shoe is used to wrap the tube around the contour of a fixed form block, as in (c). Roll bending (Section 20.6.2), generally associated with the forming of sheet stock, is also used for bending tubes and other cross sections. REFERENCES [1] ASM Handbook, Vol. 14B, Metalworking: Sheet Forming. ASM International, Materials Park, Ohio, 2006. [2] Eary, D. F., and Reed, E. A. Techniques of Pressworking Sheet Metal, 2nd ed. Prentice-Hall, Inc., Englewood Cliffs, New Jersey, 1974. E1C20 11/11/2009 478 16:4:27 Page 478 Chapter 20/Sheet Metalworking [3] Hoffman, E. G. Fundamentals of Tool Design, 2nd ed. Society of Manufacturing Engineers, Dearborn, Michigan, 1984. [4] Hosford, W. F., and Caddell, R. M. Metal Forming: Mechanics and Metallurgy, 3rd ed. Cambridge University Press, Cambridge, UK, 2007. [5] Kalpakjian, S. Manufacturing Processes for Engineering Materials, 4th ed. Prentice Hall/Pearson, Upper Saddle River, New Jersey, 2003. [6] Lange, K., et al. (eds.). Handbook of Metal Forming. Society of Manufacturing Engineers, Dearborn, Michigan, 1995. [7] Mielnik, E. M. Metalworking Science and Engineering. McGraw-Hill, Inc., New York, 1991. [8] Schey, J. A. Introduction to Manufacturing Processes, 3rd ed. McGraw-Hill Book Company, New York, 2000. [9] Spitler, D., Lantrip, J., Nee, J., and Smith, D. A. Fundamentals of Tool Design, 5th ed. Society of Manufacturing Engineers, Dearborn, Michigan, 2003. [10] Wick, C., et al. (eds.). Tool and Manufacturing Engineers Handbook, 4th ed. Vol. II, Forming. Society of Manufacturing Engineers, Dearborn, Michigan, 1984. REVIEW QUESTIONS 20.1. Identify the three basic types of sheet metalworking operations. 20.2. In conventional sheet metalworking operations, (a) what is the name of the tooling and (b) what is the name of the machine tool used in the operations? 20.3. In blanking of a circular sheet-metal part, is the clearance applied to the punch diameter or the die diameter? 20.4. What is the difference between a cutoff operation and a parting operation? 20.5. What is the difference between a notching operation and a seminotching operation? 20.6. Describe each of the two types of sheet-metalbending operations: V-bending and edge bending. 20.7. For what is the bend allowance intended to compensate? 20.8. What is springback in sheet-metal bending? 20.9. Define drawing in the context of sheet metalworking. 20.10. What are some of the simple measures used to assess the feasibility of a proposed cup-drawing operation? 20.11. Distinguish between redrawing and reverse drawing. 20.12. What are some of the possible defects in drawn sheet-metal parts? 20.13. What is an embossing operation? 20.14. What is stretch forming? 20.15. Identify the principal components of a stamping die that performs blanking. 20.16. What are the two basic categories of structural frames used in stamping presses? 20.17. What are the relative advantages and disadvantages of mechanical presses versus hydraulic presses in sheet metalworking? 20.18. What is the Guerin process? 20.19. Identify a major technical problem in tube bending. 20.20. Distinguish between roll bending and roll forming. 20.21. (Video) According to the video on sheet-metal shearing, what is the blade rake angle? 20.22. (Video) According to the video on sheet-metal bending, what are the principal terms used to describe bending on a press brake? 20.23. (Video) According to the video on sheet-metal stamping dies and processes, what are the factors that affect the formability of a metal? 20.24. (Video) Name the four forming processes listed in the video clip on sheet-metal stamping dies and processes. 20.25. (Video) List the factors that affect the hold down pressure in a drawing operation according to the video on sheet-metal stamping dies and processes. MULTIPLE CHOICE QUIZ There are 21 correct answers in the following multiple choice questions (some questions have multiple answers that are correct). To attain a perfect score on the quiz, all correct answers must be given. Each correct answer is worth 1 point. Each omitted answer or wrong answer reduces the score by 1 point, and each additional answer beyond the correct number of answers reduces the score by 1 point. Percentage score on the quiz is based on the total number of correct answers. 20.1. Most sheet metalworking operations are performed as which one of the following: (a) cold working, (b) hot working, or (c) warm working? 20.2. In a sheet-metal-cutting operation used to produce a flat part with a hole in the center, the part itself is called a blank, and the scrap piece that was cut out to make the hole is called a slug: (a) true or (b) false? E1C20 11/11/2009 16:4:28 Page 479 Problems 20.3. As sheet-metal stock hardness increases in a blanking operation, the clearance between punch and die should be (a) decreased, (b) increased, or (c) remain the same? 20.4. A circular sheet-metal slug produced in a hole punching operation will have the same diameter as (a) the die opening or (b) the punch? 20.5. The cutting force in a sheet-metal blanking operation depends on which mechanical property of the metal (one correct answer): (a) compressive strength, (b) modulus of elasticity, (c) shear strength, (d) strain rate, (e) tensile strength, or (f) yield strength? 20.6. Which of the following descriptions applies to a Vbending operation as compared to an edge-bending operation (two best answers): (a) costly tooling, (b) inexpensive tooling, (c) limited to 90 bends or less, (d) used for high production, (e) used for low production, and (f) uses a pressure pad to hold down the sheet metal? 20.7. Sheet-metal bending involves which of the following stresses and strains (two correct answers): (a) compressive, (b) shear, and (c) tensile? 20.8. Which one of the following is the best definition of bend allowance: (a) amount by which the die is larger than the punch, (b) amount of elastic recovery experienced by the metal after bending, (c) safety factor used in calculating bending force, or (d) length before bending of the straight sheetmetal section to be bent? 20.9. Springback in a sheet-metal-bending operation is the result of which one of the following: (a) elastic 20.10. 20.11. 20.12. 20.13. 20.14. 20.15. modulus of the metal, (b) elastic recovery of the metal, (c) overbending, (d) overstraining, or (e) yield strength of the metal? Which of the following are variations of sheet metal-bending operations (two best answers): (a) coining, (b) flanging, (c) hemming, (d) ironing, (e) notching, (f) shear spinning, (g) trimming, and (h) tube bending? The following are measures of feasibility for several proposed cup-drawing operations; which of the operations are likely to be feasible (three best answers): (a) DR ¼ 1.7, (b) DR ¼ 2.7, (c) r ¼ 0.35, (d) r ¼ 0.65, and (e) t/D ¼ 2%? The holding force in drawing is most likely to be (a) greater than, (b) equal to, or (c) less than the maximum drawing force? Which one of the following stamping dies is the most complicated: (a) blanking die, (b) combination die, (c) compound die, (d) edge-bending die, (e) progressive die, or (f) V-bending die? Which one of the following press types is usually associated with the highest production rates in sheet-metal-stamping operations: (a) adjustable bed, (b) open-back inclinable, (c) press brake, (d) solid gap, or (e) straight-sided? Which of the following processes are classified as highenergy-rate forming processes (two best answers): (a) electrochemical machining, (b) electromagnetic forming, (c) electron beam cutting, (d) explosive forming, (e) Guerin process, (f) hydroforming, (g) redrawing, and (h) shear spinning? PROBLEMS Cutting Operations 20.1. A power shears is used to cut soft cold-rolled steel that is 4.75 mm thick. At what clearance should the shears be set to yield an optimum cut? 20.2. A blanking operation is to be performed on 2.0-mm thick cold-rolled steel (half hard). The part is circular with diameter ¼ 75.0 mm. Determine the appropriate punch and die sizes for this operation. 20.3. A compound die will be used to blank and punch a large washer out of 6061ST aluminum alloy sheet stock 3.50 mm thick. The outside diameter of the washer is 50.0 mm and the inside diameter is 15.0 mm. Determine (a) the punch and die sizes for the blanking operation, and (b) the punch and die sizes for the punching operation. 20.4. A blanking die is to be designed to blank the part outline shown in Figure P20.4. The material is 4-mm 479 50 25 25 85 25 FIGURE P20.4 Blanked part for Problem 20.4 (dimensions in mm). E1C20 11/11/2009 480 16:4:28 Page 480 Chapter 20/Sheet Metalworking thick stainless steel (half hard). Determine the dimensions of the blanking punch and the die opening. 20.5. Determine the blanking force required in Problem 20.2, if the shear strength of the steel ¼ 325 MPa and the tensile strength is 450 MPa. 20.6. Determine the minimum tonnage press to perform the blanking and punching operation in Problem 20.3. The aluminum sheet metal has a tensile strength ¼ 310 MPa, a strength coefficient of 350 MPa, and a strain-hardening exponent of 0.12. (a) Assume that blanking and punching occur simultaneously. (b) Assume the punches are staggered so that punching occurs first, then blanking. 20.7. Determine the tonnage requirement for the blanking operation in Problem 20.4, given that the stainless steel has a yield strength ¼ 500 MPa, a shear strength ¼ 600 MPa, and a tensile strength ¼ 700 MPa. 20.8. The foreman in the pressworking section comes to you with the problem of a blanking operation that is producing parts with excessive burrs. (a) What are the possible reasons for the burrs? (b) What can be done to correct the condition? Bending 20.9. A bending operation is to be performed on 5.00-mm thick cold-rolled steel. The part drawing is given in Figure P20.9. Determine the blank size required. 20.12. 35 t = 5.00 58 40° 20.13. R = 8.5 46.5 20.14. FIGURE P20.9 Part in bending operation of Problem 20.9 (dimensions in mm). 20.10. Solve Problem 20.9 except that the bend radius R ¼ 11.35 mm. 20.11. An L-shaped part is to be bent in a V-bending operation on a press brake from a flat blank 4.0 in by 1.5 in that is 5/32 in thick. The bend of 90 is to be made in the middle of the 4.0 in length. (a) Determine the dimensions of the two equal sides that will result after the bend, if the bend radius ¼ 3/16 in. For convenience, these sides should be measured to the beginning of the bend radius. (b) Also, determine the length of the 20.15. 20.16. 20.17. part’s neutral axis after the bend. (c) Where should the machine operator set the stop on the press brake relative to the starting length of the part? A bending operation is to be performed on 4.0-mm thick cold-rolled steel sheet that is 25 mm wide and 100 mm long. The sheet is bent along the 25 mm direction, so that the bend is 25 mm long. The resulting sheet metal part has an acute angle of 30 and a bend radius of 6 mm. Determine (a) the bend allowance and (b) the length of the neutral axis of the part after the bend. (Hint: the length of the neutral axis before the bend ¼ 100.0 mm). Determine the bending force required in Problem 20.9 if the bend is to be performed in a V-die with a die opening dimension of 40 mm. The material has a tensile strength of 600 MPa and a shear strength of 430 MPa. Solve Problem 20.13 except that the operation is performed using a wiping die with die opening dimension ¼ 28 mm. Determine the bending force required in Problem 20.11 if the bend is to be performed in a V-die with a die opening width dimension ¼ 1.25 in. The material has a tensile strength ¼ 70,000 lb/in2. Solve Problem 20.15 except that the operation is performed using a wiping die with die opening dimension ¼ 0.75 in. A sheet-metal part 3.0 mm thick and 20.0 mm long is bent to an included angle ¼ 60 and a bend radius ¼ 7.5 mm in a V-die. The metal has a yield strength ¼ 220 MPa and a tensile strength ¼ 340 MPa. Compute the required force to bend the part, given that the die opening dimension ¼ 15 mm. Drawing Operations 20.18. Derive an expression for the reduction r in drawing as a function of drawing ratio DR. 20.19. A cup is to be drawn in a deep drawing operation. The height of the cup is 75 mm and its inside diameter ¼ E1C20 11/11/2009 16:4:29 Page 481 Problems 20.20. 20.21. 20.22. 20.23. 20.24. 100 mm. The sheet-metal thickness ¼ 2 mm. If the blank diameter ¼ 225 mm, determine (a) drawing ratio, (b) reduction, and (c) thickness-to-diameter ratio. (d) Does the operation seem feasible? Solve Problem 20.19 except that the starting blank size diameter ¼ 175 mm. A deep drawing operation is performed in which the inside of the cylindrical cup has a diameter of 4.25 in and a height ¼ 2.65 in. The stock thickness ¼ 3/16 in, and the starting blank diameter ¼ 7.7 in. Punch and die radii ¼ 5/32 in. The metal has a tensile strength ¼ 65,000 lb/in2, a yield strength ¼ 32,000 lb/in2, and a shear strength of 40,000 lb/in2. Determine (a) drawing ratio, (b) reduction, (c) drawing force, and (d) blankholder force. Solve Problem 20.21 except that the stock thickness t ¼ 1/8 in. A cup-drawing operation is performed in which the inside diameter ¼ 80 mm and the height ¼ 50 mm. The stock thickness ¼ 3.0 mm, and the starting blank diameter ¼ 150 mm. Punch and die radii ¼ 4 mm. Tensile strength ¼ 400 MPa and yield strength ¼ 180 MPa for this sheet metal. Determine (a) drawing ratio, (b) reduction, (c) drawing force, and (d) blankholder force. A deep drawing operation is to be performed on a sheet-metal blank that is 1/8 in thick. The height (inside dimension) of the cup ¼ 3.8 in and the diameter (inside dimension) ¼ 5.0 in. Assuming the punch radius ¼ 0, compute the starting diameter of the blank to complete the operation with no 20.25. 20.26. 20.27. 20.28. 20.29. 20.30. 481 material left in the flange. Is the operation feasible (ignoring the fact that the punch radius is too small)? Solve Problem 20.24 except use a punch radius ¼ 0.375 in. A drawing operation is performed on 3.0 mm stock. The part is a cylindrical cup with height ¼ 50 mm and inside diameter ¼ 70 mm. Assume the corner radius on the punch is zero. (a) Find the required starting blank size Db. (b) Is the drawing operation feasible? Solve Problem 20.26 except that the height ¼ 60 mm. Solve Problem 20.27 except that the corner radius on the punch ¼ 10 mm. The foreman in the drawing section of the shop brings to you several samples of parts that have been drawn in the shop. The samples have various defects. One has ears, another has wrinkles, and still a third has torn sections at its base. What are the causes of each of these defects and what remedies would you propose? A cup-shaped part is to be drawn without a blankholder from sheet metal whose thickness ¼ 0.25 in. The inside diameter of the cup ¼ 2.5 in, its height ¼ 1.5 in, and the corner radius at the base ¼ 0.375 in. (a) What is the minimum starting blank diameter that can be used, according to Eq. (20.14)? (b) Does this blank diameter provide sufficient material to complete the cup? Other Operations 20.31. A 20-in-long sheet-metal workpiece is stretched in a stretch forming operation to the dimensions shown in Figure P20.31. The thickness of the beginning stock is 3/16 in and the width is 8.5 in. The metal has a flow curve defined by a strength coefficient of 75,000 lb/in2 and a strain hardening exponent of 0.20. The yield strength of the material is 30,000 lb/in2. (a) Find the stretching force F required near the beginning of the operation when yielding first occurs. Determine (b) true strain experienced by the metal, (c) stretching force F, and (d) die force Fdie at the very end when the part is formed as indicated in Figure P20.31(b). 20.32. Determine the starting disk diameter required to spin the part in Figure P20.32 using a conventional spinning operation. The starting thickness ¼ 2.4 mm. FIGURE P20.31 Stretch forming operation: (a) before, and (b) after (dimensions in inches). E1C20 11/11/2009 482 16:4:29 Page 482 Chapter 20/Sheet Metalworking 30° 200 50 FIGURE P20.32 Part (cross section) in conventional spinning (dimensions in mm). 20.33. If the part illustrated in Figure P20.32 were made by shear spinning, determine (a) the wall thickness along the cone-shaped portion, and (b) the spinning reduction r. 20.34. Determine the shear strain that is experienced by the material that is shear spun in Problem 20.33. 20.35. A 75-mm diameter tube is bent into a rather complex shape with a series of simple tube bending operations. The wall thickness on the tube ¼ 4.75 mm. The tubes will be used to deliver fluids in a chemical plant. In one of the bends where the bend radius is 125 mm, the walls of the tube are flattening badly. What can be done to correct the condition? E1C21 11/11/2009 15:44:1 Page 483 Part VI Material Removal Processes 21 THEORY OF METAL MACHINING Chapter Contents 21.1 Overview of Machining Technology 21.2 Theory of Chip Formation in Metal Machining 21.2.1 The Orthogonal Cutting Model 21.2.2 Actual Chip Formation 21.3 Force Relationships and the Merchant Equation 21.3.1 Forces in Metal Cutting 21.3.2 The Merchant Equation 21.4 Power and Energy Relationships in Machining 21.5 Cutting Temperature 21.5.1 Analytical Methods to Compute Cutting Temperatures 21.5.2 Measurement of Cutting Temperature The material removal processes are a family of shaping operations (Figure 1.4) in which excess material is removed from a starting workpart so that what remains is the desired final geometry. The ‘‘family tree’’ is shown in Figure 21.1. The most important branch of the family is conventional machining, in which a sharp cutting tool is used to mechanically cut the material to achieve the desired geometry. The three principal machining processes are turning, drilling, and milling. The ‘‘other machining operations’’ in Figure 21.1 include shaping, planing, broaching, and sawing. This chapter begins our coverage of machining, which runs through Chapter 24. Another group of material removal processes is the abrasive processes, which mechanically remove material by the action of hard, abrasive particles. This process group, which includes grinding, is covered in Chapter 25. The ‘‘other abrasive processes’’ in Figure 21.1 include honing, lapping, and superfinishing. Finally, there are the nontraditional processes, which use various energy forms other than a sharp cutting tool or abrasive particles to remove material. The energy forms include mechanical, electrochemical, thermal, and chemical.1 The nontraditional processes are discussed in Chapter 26. Machining is a manufacturing process in which a sharp cutting tool is used to cut away material to leave the 1 Some of the mechanical energy forms in the nontraditional processes involve the use of abrasive particles, and so they overlap with the abrasive processes in Chapter 25. 483 E1C21 11/11/2009 484 15:44:1 Page 484 Chapter 21/Theory of Metal Machining Turning and related operations Conventional machining Drilling and related operations Milling Other machining operations Material removal processes Abrasive processes Grinding operations Other abrasive processes Mechanical energy processes Nontraditional machining Electrochemical machining Thermal energy processes FIGURE 21.1 Classification of material removal processes. Chemical machining desired part shape. The predominant cutting action in machining involves shear deformation of the work material to form a chip; as the chip is removed, a new surface is exposed. Machining is most frequently applied to shape metals. The process is illustrated in the diagram of Figure 21.2. Machining is one of the most important manufacturing processes. The Industrial Revolution and the growth of the manufacturing-based economies of the world can be traced largely to the development of the various machining operations (Historical Note 22.1). Machining is important commercially and technologically for several reasons: FIGURE 21.2 (a) A cross-sectional view of the machining process. (b) Tool with negative rake angle; compare with positive rake angle in (a). E1C21 11/11/2009 15:44:1 Page 485 Section 21.1/Overview of Machining Technology 485 å Variety of work materials. Machining can be applied to a wide variety of work materials. Virtually all solid metals can be machined. Plastics and plastic composites can also be cut by machining. Ceramics pose difficulties because of their high hardness and brittleness; however, most ceramics can be successfully cut by the abrasive machining processes discussed in Chapter 25. å Variety of part shapes and geometric features. Machining can be used to create any regular geometries, such as flat planes, round holes, and cylinders. By introducing variations in tool shapes and tool paths, irregular geometries can be created, such as screw threads and T-slots. By combining several machining operations in sequence, shapes of almost unlimited complexity and variety can be produced. å Dimensional accuracy. Machining can produce dimensions to very close tolerances. Some machining processes can achieve tolerances of 0.025 mm (0.001 in), much more accurate than most other processes. å Good surface finishes. Machining is capable of creating very smooth surface finishes. Roughness values less than 0.4 microns (16 m-in.) can be achieved in conventional machining operations. Some abrasive processes can achieve even better finishes. On the other hand, certain disadvantages are associated with machining and other material removal processes: å Wasteful of material. Machining is inherently wasteful of material. The chips generated in a machining operation are wasted material. Although these chips can usually be recycled, they represent waste in terms of the unit operation. å Time consuming. A machining operation generally takes more time to shape a given part than alternative shaping processes such as casting or forging. Machining is generally performed after other manufacturing processes such as casting or bulk deformation (e.g., forging, bar drawing). The other processes create the general shape of the starting workpart, and machining provides the final geometry, dimensions, and finish. 21.1 OVERVIEW OF MACHINING TECHNOLOGY Machining is not just one process; it is a group of processes. The common feature is the use of a cutting tool to form a chip that is removed from the workpart. To perform the operation, relative motion is required between the tool and work. This relative motion is achieved in most machining operations by means of a primary motion, called the cutting speed, and a secondary motion, called the feed. The shape of the tool and its penetration into the work surface, combined with these motions, produces the desired geometry of the resulting work surface. Types of Machining Operations There are many kinds of machining operations, each of which is capable of generating a certain part geometry and surface texture. We discuss these operations in considerable detail in Chapter 22, but for now it is appropriate to identify and define the three most common types: turning, drilling, and milling, illustrated in Figure 21.3. In turning, a cutting tool with a single cutting edge is used to remove material from a rotating workpiece to generate a cylindrical shape, as in Figure 21.3(a). The speed motion in turning is provided by the rotating workpart, and the feed motion is achieved by the cutting tool moving slowly in a direction parallel to the axis of rotation of the workpiece. Drilling is used to create a round hole. It is accomplished by a rotating tool that typically has two E1C21 11/11/2009 486 15:44:1 Page 486 Chapter 21/Theory of Metal Machining Speed motion (tool) Work New surface Speed motion (work) Cutting tool Drill bit Feed motion (tool) Feed motion (tool) Work (a) (b) Speed motion Rotation Milling cutter FIGURE 21.3 The three most common types of machining processes: (a) turning, (b) drilling, and two forms of milling: (c) peripheral milling, and (d) face milling. New surface Feed motion (work) Milling cutter New surface Feed motion (work) Work Work (c) (d) cutting edges. The tool is fed in a direction parallel to its axis of rotation into the workpart to form the round hole, as in Figure 21.3(b). In milling, a rotating tool with multiple cutting edges is fed slowly across the work material to generate a plane or straight surface. The direction of the feed motion is perpendicular to the tool’s axis of rotation. The speed motion is provided by the rotating milling cutter. The two basic forms of milling are peripheral milling and face milling, as in Figure 21.3(c) and (d). Other conventional machining operations include shaping, planing, broaching, and sawing (Section 22.6). Also, grinding and similar abrasive operations are often included within the category of machining. These processes commonly follow the conventional machining operations and are used to achieve a superior surface finish on the workpart. The Cutting Tool A cutting tool has one or more sharp cutting edges and is made of a material that is harder than the work material. The cutting edge serves to separate a chip from the parent work material, as in Figure 21.2. Connected to the cutting edge are two surfaces of the tool: the rake face and the flank. The rake face, which directs the flow of the newly formed chip, is oriented at a certain angle called the rake angle a. It is measured relative to a plane perpendicular to the work surface. The rake angle can be positive, as in Figure 21.2(a), or negative as in (b). The flank of the tool provides a clearance between the tool and the newly generated work surface, thus protecting the surface from abrasion, which would degrade the finish. This flank surface is oriented at an angle called the relief angle. Most cutting tools in practice have more complex geometries than those in Figure 21.2. There are two basic types, examples of which are illustrated in Figure 21.4: (a) single-point tools and (b) multiple-cutting-edge tools. A single-point tool has one cutting edge and is used for operations such as turning. In addition to the tool features shown in Figure 21.2, there is one tool point from which the name of this cutting tool is derived. During machining, the point of the tool penetrates below the original work surface of the part. The point is usually rounded to a certain radius, called the nose radius. Multiple-cutting-edge tools have more E1C21 11/11/2009 15:44:1 Page 487 Section 21.1/Overview of Machining Technology 487 FIGURE 21.4 (a) A single-point tool showing rake face, flank, and tool point; and (b) a helical milling cutter, representative of tools with multiple cutting edges. than one cutting edge and usually achieve their motion relative to the workpart by rotating. Drilling and milling use rotating multiple-cutting-edge tools. Figure 21.4(b) shows a helical milling cutter used in peripheral milling. Although the shape is quite different from a singlepoint tool, many elements of tool geometry are similar. Single-point and multiple-cuttingedge tools and the materials used in them are discussed in more detail in Chapter 23. Cutting Conditions Relative motion is required between the tool and work to perform a machining operation. The primary motion is accomplished at a certain cutting speed v. In addition, the tool must be moved laterally across the work. This is a much slower motion, called the feed f. The remaining dimension of the cut is the penetration of the cutting tool below the original work surface, called the depth of cut d. Collectively, speed, feed, and depth of cut are called the cutting conditions. They form the three dimensions of the machining process, and for certain operations (e.g., most single-point tool operations) they can be used to calculate the material removal rate for the process: RMR ¼ vf d ð21:1Þ where RMR ¼ material removal rate, mm3/s (in3/min); v ¼ cutting speed, m/s (ft/min), which must be converted to mm/s (in/min); f ¼ feed, mm (in); and d ¼ depth of cut, mm (in). The cutting conditions for a turning operation are depicted in Figure 21.5. Typical units used for cutting speed are m/s (ft/min). Feed in turning is expressed in mm/rev FIGURE 21.5 Cutting speed, feed, and depth of cut for a turning operation. E1C21 11/11/2009 488 15:44:1 Page 488 Chapter 21/Theory of Metal Machining (in/rev), and depth of cut is expressed in mm (in). In other machining operations, interpretations of the cutting conditions may differ. For example, in a drilling operation, depth is interpreted as the depth of the drilled hole. Machining operations usually divide into two categories, distinguished by purpose and cutting conditions: roughing cuts and finishing cuts. Roughing cuts are used to remove large amounts of material from the starting workpart as rapidly as possible, in order to produce a shape close to the desired form, but leaving some material on the piece for a subsequent finishing operation. Finishing cuts are used to complete the part and achieve the final dimensions, tolerances, and surface finish. In production machining jobs, one or more roughing cuts are usually performed on the work, followed by one or two finishing cuts. Roughing operations are performed at high feeds and depths—feeds of 0.4 to 1.25 mm/rev (0.015–0.050 in/rev) and depths of 2.5 to 20 mm (0.100–0.750 in) are typical. Finishing operations are carried out at low feeds and depths—feeds of 0.125 to 0.4 mm (0.005–0.015 in/rev) and depths of 0.75 to 2.0 mm (0.030–0.075 in) are typical. Cutting speeds are lower in roughing than in finishing. A cutting fluid is often applied to the machining operation to cool and lubricate the cutting tool (cutting fluids are discussed in Section 23.4). Determining whether a cutting fluid should be used, and, if so, choosing the proper cutting fluid, is usually included within the scope of cutting conditions. Given the work material and tooling, the selection of these conditions is very influential in determining the success of a machining operation. Machine Tools A machine tool is used to hold the workpart, position the tool relative to the work, and provide power for the machining process at the speed, feed, and depth that have been set. By controlling the tool, work, and cutting conditions, machine tools permit parts to be made with great accuracy and repeatability, to tolerances of 0.025 mm (0.001 in) and better. The term machine tool applies to any power-driven machine that performs a machining operation, including grinding. The term is also applied to machines that perform metal forming and pressworking operations (Chapters 19 and 20). The traditional machine tools used to perform turning, drilling, and milling are lathes, drill presses, and milling machines, respectively. Conventional machine tools are usually tended by a human operator, who loads and unloads the workparts, changes cutting tools, and sets the cutting conditions. Many modern machine tools are designed to accomplish their operations with a form of automation called computer numerical control (Section 38.3). 21.2 THEORY OF CHIP FORMATION IN METAL MACHINING The geometry of most practical machining operations is somewhat complex. A simplified model of machining is available that neglects many of the geometric complexities, yet describes the mechanics of the process quite well. It is called the orthogonal cutting model, Figure 21.6. Although an actual machining process is three-dimensional, the orthogonal model has only two dimensions that play active roles in the analysis. 21.2.1 THE ORTHOGONAL CUTTING MODEL By definition, orthogonal cutting uses a wedge-shaped tool in which the cutting edge is perpendicular to the direction of cutting speed. As the tool is forced into the material, the chip is formed by shear deformation along a plane called the shear plane, which is oriented at an angle f with the surface of the work. Only at the sharp cutting edge of the tool does failure of the material occur, resulting in separation of the chip from the parent E1C21 11/11/2009 15:44:1 Page 489 Section 21.2/Theory of Chip Formation in Metal Machining 489 FIGURE 21.6 Orthogonal cutting: (a) as a three-dimensional process, and (b) how it reduces to two dimensions in the side view. material. Along the shear plane, where the bulk of the mechanical energy is consumed in machining, the material is plastically deformed. The tool in orthogonal cutting has only two elements of geometry: (1) rake angle and (2) clearance angle. As indicated previously, the rake angle a determines the direction that the chip flows as it is formed from the workpart; and the clearance angle provides a small clearance between the tool flank and the newly generated work surface. During cutting, the cutting edge of the tool is positioned a certain distance below the original work surface. This corresponds to the thickness of the chip prior to chip formation, to. As the chip is formed along the shear plane, its thickness increases to tc. The ratio of to to tc is called the chip thickness ratio (or simply the chip ratio) r: r¼ to tc ð21:2Þ Since the chip thickness after cutting is always greater than the corresponding thickness before cutting, the chip ratio will always be less than 1.0. In addition to to, the orthogonal cut has a width dimension w, as shown in Figure 21.6(a), even though this dimension does not contribute much to the analysis in orthogonal cutting. The geometry of the orthogonal cutting model allows us to establish an important relationship between the chip thickness ratio, the rake angle, and the shear plane angle. Let ls be the length of the shear plane. We can make the substitutions: to ¼ ls sinf, and tc ¼ ls cos (f  a). Thus, r¼ ls sin f sin f ¼ ls cos (f  a) cos (f  a) This can be rearranged to determine f as follows: tan f ¼ r cos a 1  r sin a ð21:3Þ The shear strain that occurs along the shear plane can be estimated by examining Figure 21.7. Part (a) shows shear deformation approximated by a series of parallel plates sliding against one another to form the chip. Consistent with our definition of shear strain E1C21 11/11/2009 490 15:44:2 Page 490 Chapter 21/Theory of Metal Machining FIGURE 21.7 Shear strain during chip formation: (a) chip formation depicted as a series of parallel plates sliding relative to each other; (b) one of the plates isolated to illustrate the definition of shear strain based on this parallel plate model; and (c) shear strain triangle used to derive Eq. (21.4). (Section 3.1.4), each plate experiences the shear strain shown in Figure 21.7(b). Referring to part (c), this can be expressed as g¼ AC AD þ DC ¼ BD BD which can be reduced to the following definition of shear strain in metal cutting: g ¼ tan (f  a) þ cot f Example 21.1 Orthogonal Cutting ð21:4Þ In a machining operation that approximates orthogonal cutting, the cutting tool has a rake angle ¼ 10 . The chip thickness before the cut to ¼ 0.50 mm and the chip thickness after the cut tc ¼ 1.125 in. Calculate the shear plane angle and the shear strain in the operation. Solution: The chip thickness ratio can be determined from Eq. (21.2): r¼ 0:50 ¼ 0:444 1:125 The shear plane angle is given by Eq. (21.3): tan f ¼ 0:444 cos 10 ¼ 0:4738 1  0:444 sin 10 f ¼ 25:4 E1C21 11/11/2009 15:44:2 Page 491 Section 21.2/Theory of Chip Formation in Metal Machining 491 Finally, the shear strain is calculated from Eq. (21.4): g ¼ tan (25:4  10) þ cot 25:4 g ¼ 0:275 þ 2:111 ¼ 2:386 n 21.2.2 ACTUAL CHIP FORMATION We should note that there are differences between the orthogonal model and an actual machining process. First, the shear deformation process does not occur along a plane, but within a zone. If shearing were to take place across a plane of zero thickness, it would imply that the shearing action must occur instantaneously as it passes through the plane, rather than over some finite (although brief) time period. For the material to behave in a realistic way, the shear deformation must occur within a thin shear zone. This more realistic model of the shear deformation process in machining is illustrated in Figure 21.8. Metal-cutting experiments have indicated that the thickness of the shear zone is only a few thousandths of an inch. Since the shear zone is so thin, there is not a great loss of accuracy in most cases by referring to it as a plane. Second, in addition to shear deformation that occurs in the shear zone, another shearing action occurs in the chip after it has been formed. This additional shear is referred to as secondary shear to distinguish it from primary shear. Secondary shear results from friction between the chip and the tool as the chip slides along the rake face of the tool. Its effect increases with increased friction between the tool and chip. The primary and secondary shear zones can be seen in Figure 21.8. Third, formation of the chip depends on the type of material being machined and the cutting conditions of the operation. Four basic types of chip can be distinguished, illustrated in Figure 21.9: å Discontinuous chip. When relatively brittle materials (e.g., cast irons) are machined at low cutting speeds, the chips often form into separate segments (sometimes the segments are loosely attached). This tends to impart an irregular texture to the machined surface. High tool–chip friction and large feed and depth of cut promote the formation of this chip type. å Continuous chip. When ductile work materials are cut at high speeds and relatively small feeds and depths, long continuous chips are formed. A good surface finish typically results when this chip type is formed. A sharp cutting edge on the tool and Chip FIGURE 21.8 More realistic view of chip formation, showing shear zone rather than shear plane. Also shown is the secondary shear zone resulting from tool–chip friction. Effective Tool Primary shear zone Secondary shear zone E1C21 11/11/2009 492 15:44:2 Page 492 Chapter 21/Theory of Metal Machining Discontinuous chip Continuous chip Continuous chip High shear strain zone Tool Tool Tool Low shear strain zone Tool Built-up edge Irregular surface due to chip discontinuities (a) Good finish typical (b) Particle of BUE on new surface (c) (d) FIGURE 21.9 Four types of chip formation in metal cutting: (a) discontinuous, (b) continuous, (c) continuous with built-up edge, (d) serrated. low tool–chip friction encourage the formation of continuous chips. Long, continuous chips (as in turning) can cause problems with regard to chip disposal and/or tangling about the tool. To solve these problems, turning tools are often equipped with chip breakers (Section 23.3.1). å Continuous chip with built-up edge. When machining ductile materials at low-tomedium cutting speeds, friction between tool and chip tends to cause portions of the work material to adhere to the rake face of the tool near the cutting edge. This formation is called a built-up edge (BUE). The formation of a BUE is cyclical; it forms and grows, then becomes unstable and breaks off. Much of the detached BUE is carried away with the chip, sometimes taking portions of the tool rake face with it, which reduces the life of the cutting tool. Portions of the detached BUE that are not carried off with the chip become imbedded in the newly created work surface, causing the surface to become rough. The preceding chip types were first classified by Ernst in the late 1930s [13]. Since then, the available metals used in machining, cutting tool materials, and cutting speeds have all increased, and a fourth chip type has been identified: å Serrated chips (the term shear-localized is also used for this fourth chip type). These chips are semi-continuous in the sense that they possess a saw-tooth appearance that is produced by a cyclical chip formation of alternating high shear strain followed by low shear strain. This fourth type of chip is most closely associated with certain difficult-to-machine metals such as titanium alloys, nickel-base superalloys, and austenitic stainless steels when they are machined at higher cutting speeds. However, the phenomenon is also found with more common work metals (e.g., steels) when they are cut at high speeds [13].2 21.3 FORCE RELATIONSHIPS AND THE MERCHANT EQUATION Several forces can be defined relative to the orthogonal cutting model. Based on these forces, shear stress, coefficient of friction, and certain other relationships can be defined. 2 A more complete description of the serrated chip type can be found in Trent & Wright [12], pp. 348–367. E1C21 11/11/2009 15:44:2 Page 493 Section 21.3/Force Relationships and the Merchant Equation 493 21.3.1 FORCES IN METAL CUTTING Consider the forces acting on the chip during orthogonal cutting in Figure 21.10(a). The forces applied against the chip by the tool can be separated into two mutually perpendicular components: friction force and normal force to friction. The friction force F is the frictional force resisting the flow of the chip along the rake face of the tool. The normal force to friction N is perpendicular to the friction force. These two components can be used to define the coefficient of friction between the tool and the chip: m¼ F N ð21:5Þ The friction force and its normal force can be added vectorially to form a resultant force R, which is oriented at an angle b, called the friction angle. The friction angle is related to the coefficient of friction as m ¼ tan b ð21:6Þ In addition to the tool forces acting on the chip, there are two force components applied by the workpiece on the chip: shear force and normal force to shear. The shear force Fs is the force that causes shear deformation to occur in the shear plane, and the normal force to shear Fn is perpendicular to the shear force. Based on the shear force, we can define the shear stress that acts along the shear plane between the work and the chip: t¼ Fs As ð21:7Þ where As ¼ area of the shear plane. This shear plane area can be calculated as As ¼ to w sin f ð21:8Þ The shear stress in Eq. (21.7) represents the level of stress required to perform the machining operation. Therefore, this stress is equal to the shear strength of the work material (t ¼ S) under the conditions at which cutting occurs. Vector addition of the two force components Fs and Fn yields the resultant force R0 . In order for the forces acting on the chip to be in balance, this resultant R0 must be equal in magnitude, opposite in direction, and collinear with the resultant R. FIGURE 21.10 Forces in metal cutting: (a) forces acting on the chip in orthogonal cutting, and (b) forces acting on the tool that can be measured. E1C21 11/11/2009 494 15:44:2 Page 494 Chapter 21/Theory of Metal Machining FIGURE 21.11 Force diagram showing geometric relationships between F, N, Fs, Fn, Fc, and Ft. None of the four force components F, N, Fs, and Fn can be directly measured in a machining operation, because the directions in which they are applied vary with different tool geometries and cutting conditions. However, it is possible for the cutting tool to be instrumented using a force measuring device called a dynamometer, so that two additional force components acting against the tool can be directly measured: cutting force and thrust force. The cutting force Fc is in the direction of cutting, the same direction as the cutting speed v, and the thrust force Ft is perpendicular to the cutting force and is associated with the chip thickness before the cut to. The cutting force and thrust force are shown in Figure 21.10 (b) together with their resultant force R00 . The respective directions of these forces are known, so the force transducers in the dynamometer can be aligned accordingly. Equations can be derived to relate the four force components that cannot be measured to the two forces that can be measured. Using the force diagram in Figure 21.11, the following trigonometric relationships can be derived: F ¼ F c sin a þ F t cos a ð21:9Þ N ¼ F c cos a  F t sin a ð21:10Þ F s ¼ F c cos f  F t sin f ð21:11Þ F n ¼ F c sin f þ F t cos f ð21:12Þ If cutting force and thrust force are known, these four equations can be used to calculate estimates of shear force, friction force, and normal force to friction. Based on these force estimates, shear stress and coefficient of friction can be determined. Note that in the special case of orthogonal cutting when the rake angle a ¼ 0, Eqs. (21.9) and (21.10) reduce to F ¼ Ft and N ¼ Fc, respectively. Thus, in this special case, friction force and its normal force could be directly measured by the dynamometer. Example 21.2 Shear Stress in Machining Suppose in Example 21.1 that cutting force and thrust force are measured during an orthogonal cutting operation: Fc ¼ 1559 N and Ft ¼ 1271 N. The width of the orthogonal cutting operation w ¼ 3.0 mm. Based on these data, determine the shear strength of the work material. Solution: From Example 21.1, rake angle a ¼ 10 , and shear plane angle f ¼ 25.4 . Shear force can be computed from Eq. (21.11): F s ¼ 1559 cos 25:4  1271 sin 25:4 ¼ 863 N E1C21 11/11/2009 15:44:3 Page 495 Section 21.3/Force Relationships and the Merchant Equation 495 The shear plane area is given by Eq. (21.8): As ¼ (0:5)(3:0) ¼ 3:497 mm2 sin 25:4 Thus the shear stress, which equals the shear strength of the work material, is t¼S¼ 863 ¼ 247 N/mm2 ¼ 247 MPa 3:497 n This example demonstrates that cutting force and thrust force are related to the shear strength of the work material. The relationships can be established in a more direct way. Recalling from Eq. (21.7) that the shear force Fs ¼ S As, the force diagram of Figure 21.11 can be used to derive the following equations: Fc ¼ Sto w cos (b  a) F s cos (b  a) ¼ sin f cos(f þ b  a) cos(f þ b  a) ð21:13Þ Ft ¼ St w sin (b  a) F s sin (b  a) ¼ sin f cos(f þ b  a) cos (f þ b  a) ð21:14Þ and These equations allow one to estimate cutting force and thrust force in an orthogonal cutting operation if the shear strength of the work material is known. 21.3.2 THE MERCHANT EQUATION One of the important relationships in metal cutting was derived by Eugene Merchant [10]. Its derivation was based on the assumption of orthogonal cutting, but its general validity extends to three-dimensional machining operations. Merchant started with the definition of shear stress expressed in the form of the following relationship derived by combining Eqs. (21.7), (21.8), and (21.11): t¼ F c cos f  F t sin f (to w=sin f) ð21:15Þ Merchant reasoned that, out of all the possible angles emanating from the cutting edge of the tool at which shear deformation could occur, there is one angle f that predominates. This is the angle at which shear stress is just equal to the shear strength of the work material, and so shear deformation occurs at this angle. For all other possible shear angles, the shear stress is less than the shear strength, so chip formation cannot occur at these other angles. In effect, the work material will select a shear plane angle that minimizes energy. This angle can be determined by taking the derivative of the shear stress S in Eq. (21.15) with respect to f and setting the derivative to zero. Solving for f, we get the relationship named after Merchant: f ¼ 45 þ a b  2 2 ð21:16Þ Among the assumptions in the Merchant equation is that shear strength of the work material is a constant, unaffected by strain rate, temperature, and other factors. Because this assumption is violated in practical machining operations, Eq. (21.16) must be E1C21 11/11/2009 496 15:44:3 Page 496 Chapter 21/Theory of Metal Machining considered an approximate relationship rather than an accurate mathematical equation. Let us nevertheless consider its application in the following example. Example 21.3 Estimating Friction Angle Using the data and results from our previous examples, determine (a) the friction angle and (b) the coefficient of friction. Solution: (a) From Example 21.1, a ¼ 10 , and f ¼ 25.4 . Rearranging Eq. (21.16), the friction angle can be estimated: b ¼ 2 (45) þ 10  2 (25:4) ¼ 49:2 (b) The coefficient of friction is given by Eq. (21.6): m ¼ tan 49:2 ¼ 1:16 n Lessons Based on the Merchant Equation The real value of the Merchant equation is that it defines the general relationship between rake angle, tool–chip friction, and shear plane angle. The shear plane angle can be increased by (1) increasing the rake angle and (2) decreasing the friction angle (and coefficient of friction) between the tool and the chip. Rake angle can be increased by proper tool design, and friction angle can be reduced by using a lubricant cutting fluid. The importance of increasing the shear plane angle can be seen in Figure 21.12. If all other factors remain the same, a higher shear plane angle results in a smaller shear plane area. Since the shear strength is applied across this area, the shear force required to form the chip will decrease when the shear plane area is reduced. A greater shear plane angle results in lower cutting energy, lower power requirements, and lower cutting temperature. These are good reasons to try to make the shear plane angle as large as possible during machining. Approximation of Turning by Orthogonal Cutting The orthogonal model can be used to approximate turning and certain other single-point machining operations so long as the feed in these operations is small relative to depth of cut. Thus, most of the cutting will take place in the direction of the feed, and cutting on the point of the tool will be negligible. Figure 21.13 indicates the conversion from one cutting situation to the other. FIGURE 21.12 Effect of shear plane angle f: (a) higher f with a resulting lower shear plane area; (b) smaller f with a corresponding larger shear plane area. Note that the rake angle is larger in (a), which tends to increase shear angle according to the Merchant equation. E1C21 11/11/2009 15:44:3 Page 497 Section 21.4/Power and Energy Relationships in Machining 497 FIGURE 21.13 Approximation of turning by the orthogonal model: (a) turning; and (b) the corresponding orthogonal cutting. TABLE 21.1 Conversion key: turning operation vs. orthogonal cutting. Turning Operation Orthogonal Cutting Model Feed f ¼ Depth d ¼ Cutting speed v ¼ Cutting force Fc ¼ Feed force Ff ¼ Chip thickness before cut to Width of cut w Cutting speed v Cutting force Fc Thrust force Ft The interpretation of cutting conditions is different in the two cases. The chip thickness before the cut to in orthogonal cutting corresponds to the feed f in turning, and the width of cut w in orthogonal cutting corresponds to the depth of cut d in turning. In addition, the thrust force Ft in the orthogonal model corresponds to the feed force Ff in turning. Cutting speed and cutting force have the same meanings in the two cases. Table 21.1 summarizes the conversions. 21.4 POWER AND ENERGY RELATIONSHIPS IN MACHINING A machining operation requires power. The cutting force in a production machining operation might exceed 1000 N (several hundred pounds), as suggested by Example 21.2. Typical cutting speeds are several hundred m/min. The product of cutting force and speed gives the power (energy per unit time) required to perform a machining operation: Pc ¼ F c v ð21:17Þ where Pc ¼ cutting power, N-m/s or W (ft-lb/min); Fc ¼ cutting force, N (lb); and v ¼ cutting speed, m/s (ft/min). In U.S. customary units, power is traditionally expressed as E1C21 11/11/2009 498 15:44:3 Page 498 Chapter 21/Theory of Metal Machining horsepower by dividing ft-lb/min by 33,000. Hence, HPc ¼ F cv 33; 000 ð21:18Þ where HPc ¼ cutting horsepower, hp. The gross power required to operate the machine tool is greater than the power delivered to the cutting process because of mechanical losses in the motor and drive train in the machine. These losses can be accounted for by the mechanical efficiency of the machine tool: Pg ¼ Pc E or HPg ¼ HPc E ð21:19Þ where Pg ¼ gross power of the machine tool motor, W; HPg ¼ gross horsepower; and E ¼ mechanical efficiency of the machine tool. Typical values of E for machine tools are around 90%. It is often useful to convert power into power per unit volume rate of metal cut. This is called the unit power, Pu (or unit horsepower, HPu), defined: Pu ¼ Pc RMR or HPu ¼ HPc RMR ð21:20Þ where RMR ¼ material removal rate, mm3/s (in3/min). The material removal rate can be calculated as the product of vtow. This is Eq. (21.1) using the conversions from Table 21.1. Unit power is also known as the specific energy U. U ¼ Pu ¼ Pc F cv Fc ¼ ¼ RMR vto w to w ð21:21Þ The units for specific energy are typically N-m/mm3 (in-lb/in3). However, the last expression in Eq. (21.21) suggests that the units might be reduced to N/mm2 (lb/in2). It is more meaningful to retain the units as N-m/mm3 or J/mm3 (in-lb/in3). Example 21.4 Power Relationships in Machining Continuing with our previous examples, let us determine cutting power and specific energy in the machining operation if the cutting speed ¼ 100 m/min. Summarizing the data and results from previous examples, to ¼ 0.50 mm, w ¼ 3.0 mm, Fc ¼ 1557 N. Solution: From Eq. (21.18), power in the operation is Pc ¼ (1557 N)(100 m/min) ¼ 155; 700 N  m/min ¼ 155; 700 J/min ¼ 2595 J/s ¼ 2595 W Specific energy is calculated from Eq. (21.21): U¼ 155; 700 155; 700 ¼ ¼ 1:038 N-m/min3 100(103 )(3:0)(0:5) 150; 000 n Unit power and specific energy provide a useful measure of how much power (or energy) is required to remove a unit volume of metal during machining. Using this measure, different work materials can be compared in terms of their power and energy requirements. Table 21.2 presents a listing of unit horsepower and specific energy values for selected work materials. The values in Table 21.2 are based on two assumptions: (1) the cutting tool is sharp, and (2) the chip thickness before the cut to ¼ 0.25 mm (0.010 in). If these assumptions are not met, some adjustments must be made. For worn tools, the power required to perform the cut is greater, and this is reflected in higher specific energy and unit horsepower values. As an approximate guide, the values in the table should be multiplied by a factor between 1.00 and 1.25 depending on the degree of dullness of the tool. For sharp tools, the factor is 11/11/2009 15:44:4 Page 499 Section 21.4/Power and Energy Relationships in Machining 499 TABLE 21.2 Values of unit horsepower and specific energy for selected work materials using sharp cutting tools and chip thickness before the cut to = 0.25 mm (0.010 in). Specific Energy U or Unit Power Pu Unit Horsepower Brinell HPu hp/(in3/min) Material Hardness N-m/mm3 in-lb/in3 Carbon steel Alloy steels Cast irons Stainless steel Aluminum Aluminum alloys Brass Bronze Magnesium alloys 150–200 201–250 251–300 200–250 251–300 301–350 351–400 125–175 175–250 150–250 50–100 100–150 100–150 100–150 50–100 1.6 2.2 2.8 2.2 2.8 3.6 4.4 1.1 1.6 2.8 0.7 0.8 2.2 2.2 0.4 240,000 320,000 400,000 320,000 400,000 520,000 640,000 160,000 240,000 400,000 100,000 120,000 320,000 320,000 60,000 0.6 0.8 1.0 0.8 1.0 1.3 1.6 0.4 0.6 1.0 0.25 0.3 0.8 0.8 0.15 Data compiled from [6], [8], [11], and other sources. 1.00. For tools in a finishing operation that are nearly worn out, the factor is around 1.10, and for tools in a roughing operation that are nearly worn out, the factor is 1.25. Chip thickness before the cut to also affects the specific energy and unit horsepower values. As to is reduced, unit power requirements increase. This relationship is referred to as the size effect. For example, grinding, in which the chips are extremely small by comparison to most other machining operations, requires very high specific energy values. The U and HPu values in Table 21.2 can still be used to estimate horsepower and energy for situations in which to is not equal to 0.25 mm (0.010 in) by applying a correction factor to account for any difference in chip thickness before the cut. Figure 21.14 provides values of this correction Chip thickness before cut to (in.) 0.005 0.010 0.015 0.020 0.025 0.030 0.040 0.050 1.6 1.4 FIGURE 21.14 Correction factor for unit horsepower and specific energy when values of chip thickness before the cut to are different from 0.25 mm (0.010 in). Correction factor E1C21 1.2 1.0 0.8 0.6 0.4 0.2 0.125 0.25 0.38 0.50 0.63 0.75 0.88 0.1 Chip thickness before cut to (mm) 1.25 E1C21 11/11/2009 500 15:44:4 Page 500 Chapter 21/Theory of Metal Machining factor as a function of to. The unit horsepower and specific energy values in Table 21.2 should be multiplied by the appropriate correction factor when to is different from 0.25 mm (0.010 in). In addition to tool sharpness and size effect, other factors also influence the values of specific energy and unit horsepower for a given operation. These other factors include rake angle, cutting speed, and cutting fluid. As rake angle or cutting speed are increased, or when cutting fluid is added, the U and HPu values are reduced slightly. For our purposes in the end-of-chapter exercises, the effects of these additional factors can be ignored. 21.5 CUTTING TEMPERATURE Of the total energy consumed in machining, nearly all of it (98%) is converted into heat. This heat can cause temperatures to be very high at the tool–chip interface—over 600 C (1100 F) is not unusual. The remaining energy (2%) is retained as elastic energy in the chip. Cutting temperatures are important because high temperatures (1) reduce tool life, (2) produce hot chips that pose safety hazards to the machine operator, and (3) can cause inaccuracies in workpart dimensions due to thermal expansion of the work material. In this section, we discuss the methods of calculating and measuring temperatures in machining operations. 21.5.1 ANALYTICAL METHODS TO COMPUTE CUTTING TEMPERATURES There are several analytical methods to calculate estimates of cutting temperature. References [3], [5], [9], and [15] present some of these approaches. We describe the method by Cook [5], which was derived using experimental data for a variety of work materials to establish parameter values for the resulting equation. The equation can be used to predict the increase in temperature at the tool–chip interface during machining: DT ¼ 0:4U vto 0:333 rC K ð21:22Þ where DT ¼ mean temperature rise at the tool–chip interface, C (F ); U ¼ specific energy in the operation, N-m/mm3 or J/mm3 (in-lb/in3); v ¼ cutting speed, m/s (in/sec); to ¼ chip thickness before the cut, m (in); rC ¼ volumetric specific heat of the work material, J/mm3C (in-lb/in3-F); K ¼ thermal diffusivity of the work material, m2/s (in2/sec). Example 21.5 Cutting Temperature For the specific energy obtained in Example 21.4, calculate the increase in temperature above ambient temperature of 20 C. Use the given data from the previous examples in this chapter: v ¼ 100 m/min, to ¼ 0.50 mm. In addition, the volumetric specific heat for the work material ¼ 3.0 (103) J/mm3-C, and thermal diffusivity ¼ 50 (106) m2/s (or 50 mm2/s). Solution: Cutting speed must be converted to mm/s: v ¼ (100 m/min)(103 mm/m)/(60 s/ min) ¼ 1667 mm/s. Eq. (21.22) can now be used to compute the mean temperature rise:   0:4(1:038)  1667(0:5) 0:333 C ¼ (138:4)(2:552) ¼ 353 C DT ¼ 50 3:0(103 ) n 21.5.2 MEASUREMENT OF CUTTING TEMPERATURE Experimental methods have been developed to measure temperatures in machining. The most frequently used measuring technique is the tool–chip thermocouple. This thermocouple consists of the tool and the chip as the two dissimilar metals forming the 11/11/2009 15:44:4 Page 501 References 501 RC-130B Titanium (T = 479v 0.182) 1600 FIGURE 21.15 Experimentally measured cutting temperatures plotted against speed for three work materials, indicating general agreement with Eq. (21.23). (Based on data in [9].)3 Cutting temperature, °F E1C21 1200 18-8 Stainless steel (T = 135v 0.361) 800 B1113 Free machining steel (T = 86.2v 0.348) 400 200 400 800 600 1000 Cutting speed (ft/min) thermocouple junction. By properly connecting electrical leads to the tool and workpart (which is connected to the chip), the voltage generated at the tool–chip interface during cutting can be monitored using a recording potentiometer or other appropriate data-collection device. The voltage output of the tool–chip thermocouple (measured in mV) can be converted into the corresponding temperature value by means of calibration equations for the particular tool–work combination. The tool–chip thermocouple has been utilized by researchers to investigate the relationship between temperature and cutting conditions such as speed and feed. Trigger [14] determined the speed–temperature relationship to be of the following general form: T ¼ K vm ð21:23Þ where T ¼ measured tool–chip interface temperature and v ¼ cutting speed. The parameters K and m depend on cutting conditions (other than v) and work material. Figure 21.15 plots temperature versus cutting speed for several work materials, with equations of the form of Eq. (21.23) determined for each material. A similar relationship exists between cutting temperature and feed; however, the effect of feed on temperature is not as strong as cutting speed. These empirical results tend to support the general validity of the Cook equation: Eq. (21.22). REFERENCES [1] ASM Handbook, Vol. 16, Machining. ASM International, Materials Park, Ohio, 1989. [2] Black, J, and Kohser, R. DeGarmo’s Materials and Processes in Manufacturing, 10th ed. John Wiley & Sons, Inc., Hoboken, New Jersey, 2008. [3] Boothroyd, G., and Knight, W. A. Fundamentals of Metal Machining and Machine Tools, 3rd ed. CRC Taylor and Francis, Boca Raton, Florida, 2006. [4] Chao, B. T., and Trigger, K. J.‘‘Temperature Distribution at the Tool-Chip Interface in Metal 3 The units reported in the Loewen and Shaw ASME paper [9] were  F for cutting temperature and ft/min for cutting speed. We have retained those units in the plots and equations of our figure. E1C21 11/11/2009 502 [5] [6] [7] [8] [9] 15:44:5 Page 502 Chapter 21/Theory of Metal Machining Cutting,’’ ASME Transactions, Vol. 77, October 1955, pp. 1107– 1121. Cook, N.‘‘Tool Wear and Tool Life,’’ ASME Transactions, Journal of Engineering for Industry, Vol. 95, November 1973, pp. 931–938. Drozda, T. J., and Wick, C. (eds.). Tool and Manufacturing Engineers Handbook, 4th ed., Vol. I, Machining. Society of Manufacturing Engineers, Dearborn, Michigan, 1983. Kalpakjian, S., and Schmid, R. Manufacturing Processes for Engineering Materials, 4th ed. Prentice Hall/Pearson, Upper Saddle River, New Jersey, 2003. Lindberg, R. A. Processes and Materials of Manufacture, 4th ed. Allyn and Bacon, Inc., Boston, 1990. Loewen, E. G., and Shaw, M. C.‘‘On the Analysis of Cutting Tool Temperatures,’’ ASME Transactions, Vol. 76, No. 2, February 1954, pp. 217–225. [10] Merchant, M. E., ‘‘Mechanics of the Metal Cutting Process: II. Plasticity Conditions in Orthogonal Cutting,’’ Journal of Applied Physics, Vol. 16, June 1945 pp. 318–324. [11] Schey, J. A. Introduction to Manufacturing Processes, 3rd ed. McGraw-Hill Book Company, New York, 1999. [12] Shaw, M. C. Metal Cutting Principles, 2nd ed. Oxford University Press, Oxford, UK, 2005. [13] Trent, E. M., and Wright, P. K. Metal Cutting, 4th ed. Butterworth Heinemann, Boston, 2000. [14] Trigger, K. J.‘‘Progress Report No. 2 on Tool–Chip Interface Temperatures,’’ ASME Transactions, Vol. 71, No. 2, February 1949, pp. 163–174. [15] Trigger, K. J., and Chao, B. T.‘‘An Analytical Evaluation of Metal Cutting Temperatures,’’ ASME Transactions, Vol. 73, No. 1, January 1951, pp. 57–68. REVIEW QUESTIONS 21.1. What are the three basic categories of material removal processes? 21.2. What distinguishes machining from other manufacturing processes? 21.3. Identify some of the reasons why machining is commercially and technologically important. 21.4. Name the three most common machining processes. 21.5. What are the two basic categories of cutting tools in machining? Give two examples of machining operations that use each of the tooling types. 21.6. What are the parameters of a machining operation that are included within the scope of cutting conditions? 21.7. Explain the difference between roughing and finishing operations in machining. 21.8. What is a machine tool? 21.9. What is an orthogonal cutting operation? 21.10. Why is the orthogonal cutting model useful in the analysis of metal machining? 21.11. Name and briefly describe the four types of chips that occur in metal cutting. 21.12. Identify the four forces that act upon the chip in the orthogonal metal cutting model but cannot be measured directly in an operation. 21.13. Identify the two forces that can be measured in the orthogonal metal cutting model. 21.14. What is the relationship between the coefficient of friction and the friction angle in the orthogonal cutting model? 21.15. Describeinwords whatthe Merchant equationtells us. 21.16. How is the power required in a cutting operation related to the cutting force? 21.17. What is the specific energy in metal machining? 21.18. What does the term size effect mean in metal cutting? 21.19. What is a tool–chip thermocouple? MULTIPLE CHOICE QUIZ There are 17 correct answers in the following multiple choice questions (some questions have multiple answers that are correct). To attain a perfect score on the quiz, all correct answers must be given. Each correct answer is worth 1 point. Each omitted answer or wrong answer reduces the score by 1 point, and each additional answer beyond the correct number of answers reduces the score by 1 point. Percentage score on the quiz is based on the total number of correct answers. 21.1. Which of the following manufacturing processes are classified as material removal processes (two correct answers): (a) casting, (b) drawing, (c) extrusion, (d) forging, (e) grinding, (f) machining, (g) molding, (h) pressworking, and (i) spinning? 21.2. A lathe is used to perform which one of the following manufacturing operations: (a) broaching, (b) drilling, (c) lapping, (d) milling, or (e) turning? 21.3. With which one of the following geometric forms is the drilling operation most closely associated: E1C21 11/11/2009 15:44:5 Page 503 Problems 21.4. 21.5. 21.6. 21.7. 21.8. (a) external cylinder, (b) flat plane, (c) round hole, (d) screw threads, or (e) sphere? If the cutting conditions in a turning operation are cutting speed ¼ 300 ft/min, feed ¼ 0.010 in/rev, and depth of cut ¼ 0.100 in, which one of the following is the material removal rate: (a) 0.025 in3/min, (b) 0.3 in3/min, (c) 3.0 in3/min, or (d) 3.6 in3/min? A roughing operation generally involves which one of the following combinations of cutting conditions: (a) high v, f, and d; (b) high v, low f and d; (c) low v, high f and d; or (d) low v, f, and d, where v ¼ cutting speed, f ¼ feed, and d ¼ depth? Which of the following are characteristics of the orthogonal cutting model (three best answers): (a) a circular cutting edge is used, (b) a multiplecutting-edge tool is used, (c) a single-point tool is used, (d) only two dimensions play an active role in the analysis, (e) the cutting edge is parallel to the direction of cutting speed, (f) the cutting edge is perpendicular to the direction of cutting speed, and (g) the two elements of tool geometry are rake and relief angle? The chip thickness ratio is which one of the following: (a) tc/to, (b) to/tc, (c) f/d, or (d) to/w, where tc ¼ chip thickness after the cut, to ¼ chip thickness before the cut, f ¼ feed, d ¼ depth, and w ¼ width of cut? Which one of the four types of chip would be expected in a turning operation conducted at low 21.9. 21.10. 21.11. 21.12. 21.13. 503 cutting speed on a brittle work material: (a) continuous, (b) continuous with built-up edge, (c) discontinuous, or (d) serrated? According to the Merchant equation, an increase in rake angle would have which of the following results, all other factors remaining the same (two best answers): (a) decrease in friction angle, (b) decrease in power requirements, (c) decrease in shear plane angle, (d) increase in cutting temperature, and (e) increase in shear plane angle? In using the orthogonal cutting model to approximate a turning operation, the chip thickness before the cut to corresponds to which one of the following cutting conditions in turning: (a) depth of cut d, (b) feed f, or (c) speed v? Which one of the following metals would usually have the lowest unit horsepower in a machining operation: (a) aluminum, (b) brass, (c) cast iron, or (d) steel? For which one of the following values of chip thickness before the cut to would you expect the specific energy in machining to be the greatest:(a) 0.010 in, (b) 0.025 in, (c) 0.12 mm, or (d) 0.50 mm? Which of the following cutting conditions has the strongest effect on cutting temperature: (a) feed or (b) speed? PROBLEMS Chip Formation and Forces in Machining 21.1. In an orthogonal cutting operation, the tool has a rake angle ¼ 15 . The chip thickness before the cut ¼ 0.30 mm and the cut yields a deformed chip thickness ¼ 0.65 mm. Calculate (a) the shear plane angle and (b) the shear strain for the operation. 21.2. In Problem 21.1, suppose the rake angle were changed to 0 . Assuming that the friction angle remains the same, determine (a) the shear plane angle, (b) the chip thickness, and (c) the shear strain for the operation. 21.3. In an orthogonal cutting operation, the 0.25-in wide tool has a rake angle of 5 . The lathe is set so the chip thickness before the cut is 0.010 in. After the cut, the deformed chip thickness is measured to be 0.027 in. Calculate (a) the shear plane angle and (b) the shear strain for the operation. 21.4. In a turning operation, spindle speed is set to provide a cutting speed of 1.8 m/s. The feed and depth of cut of cut are 0.30 mm and 2.6 mm, respectively. The tool rake angle is 8 . After the cut, the deformed chip thickness is measured to be 0.49 mm. Determine (a) shear plane angle, (b) shear strain, and (c) material removal rate. Use the orthogonal cutting model as an approximation of the turning process. 21.5. The cutting force and thrust force in an orthogonal cutting operation are 1470 N and 1589 N, respectively. The rake angle ¼ 5 , the width of the cut ¼ 5.0 mm, the chip thickness before the cut ¼ 0.6, and the chip thickness ratio ¼ 0.38. Determine (a) the shear strength of the work material and (b) the coefficient of friction in the operation. 21.6. The cutting force and thrust force have been measured in an orthogonal cutting operation to be 300 lb and 291 lb, respectively. The rake angle ¼ 10 , width of cut ¼ 0.200 in, chip thickness before the cut ¼ 0.015, and chip thickness ratio ¼ 0.4. Determine (a) the shear strength of the work material and (b) the coefficient of friction in the operation. E1C21 11/11/2009 504 15:44:5 Page 504 Chapter 21/Theory of Metal Machining 21.7. An orthogonal cutting operation is performed using a rake angle of 15 , chip thickness before the cut ¼ 0.012 in and width of cut ¼ 0.100 in. The chip thickness ratio is measured after the cut to be 0.55. Determine (a) the chip thickness after the cut, (b) shear angle, (c) friction angle, (d) coefficient of friction, and (e) shear strain. 21.8. The orthogonal cutting operation described in previous Problem 21.7 involves a work material whose shear strength is 40,000 lb/in2. Based on your answers to the previous problem, compute(a) the shear force, (b) cutting force, (c) thrust force, and (d) friction force. 21.9. In an orthogonal cutting operation, the rake angle ¼ 5 , chip thickness before the cut ¼ 0.2 mm and width of cut ¼ 4.0 mm. The chip ratio ¼ 0.4. Determine (a) the chip thickness after the cut, (b) shear angle, (c) friction angle, (d) coefficient of friction, and (e) shear strain. 21.10. The shear strength of a certain work material ¼ 50,000 lb/in2. An orthogonal cutting operation is performed using a tool with a rake angle ¼ 20 at the following cutting conditions: cutting speed ¼ 100 ft/min, chip thickness before the cut ¼ 0.015 in, and width of cut ¼ 0.150 in. The resulting chip thickness ratio ¼ 0.50. Determine (a) the shear plane angle, (b) shear force, (c) cutting force and thrust force, and (d) friction force. 21.11. Consider the data in Problem 21.10 except that rake angle is a variable, and its effect on the forces in parts (b), (c), and (d) is to be evaluated. (a) Using a spreadsheet calculator, compute the values of shear force, cutting force, thrust force, and friction force as a function of rake angle over a range of rake angles between the high value of 20 in Problem 21.10 and a low value of 10 . Use intervals of 5 between these limits. The chip thickness ratio decreases as rake angle is reduced and can be approximated by the following relationship: r ¼ 0.38 þ 0.006a, where r ¼ chip thickness and a ¼ 21.12. 21.13. 21.14. 21.15. 21.16. 21.17. 21.18. rake angle. (b) What observations can be made from the computed results? Solve previous Problem 21.10 except that the rake angle has been changed to 5 and the resulting chip thickness ratio ¼ 0.35. A carbon steel bar with 7.64 in diameter has a tensile strength of 65,000 lb/in2 and a shear strength of 45,000 lb/in2. The diameter is reduced using a turning operation at a cutting speed of 400 ft/min. The feed is 0.011 in/rev and the depth of cut is 0.120 in. The rake angle on the tool in the direction of chip flow is 13 . The cutting conditions result in a chip ratio of 0.52. Using the orthogonal model as an approximation of turning, determine (a) the shear plane angle, (b) shear force, (c) cutting force and feed force, and (d) coefficient of friction between the tool and chip. Low carbon steel having a tensile strength of 300 MPa and a shear strength of 220 MPa is cut in a turning operation with a cutting speed of 3.0 m/s. The feed is 0.20 mm/rev and the depth of cut is 3.0 mm. The rake angle of the tool is 5 in the direction of chip flow. The resulting chip ratio is 0.45. Using the orthogonal model as an approximation of turning, determine (a) the shear plane angle, (b) shear force, (c) cutting force and feed force. A turning operation is made with a rake angle of 10 , a feed of 0.010 in/rev and a depth of cut ¼ 0.100 in. The shear strength of the work material is known to be 50,000 lb/in2, and the chip thickness ratio is measured after the cut to be 0.40. Determine the cutting force and the feed force. Use the orthogonal cutting model as an approximation of the turning process. Show how Eq. (21.3) is derived from the definition of chip ratio, Eq. (21.2), and Figure 21.5(b). Show how Eq. (21.4) is derived from Figure 21.6. Derive the force equations for F, N, Fs, and Fn (Eqs. (21.9) through (21.12) in the text) using the force diagram of Figure 21.11. Power and Energy in Machining 21.19. In a turning operation on stainless steel with hardness ¼ 200 HB, the cutting speed ¼ 200 m/min, feed ¼ 0.25 mm/rev, and depth of cut ¼ 7.5 mm. How much power will the lathe draw in performing this operation if its mechanical efficiency ¼ 90%. Use Table 21.2 to obtain the appropriate specific energy value. 21.20. In Problem 21.18, compute the lathe power requirements if feed ¼ 0.50 mm/rev. 21.21. In a turning operation on aluminum, cutting speed ¼ 900 ft/min, feed ¼ 0.020 in/rev, and depth of cut ¼ 0.250 in. What horsepower is required of the drive motor, if the lathe has a mechanical efficiency ¼ 87%? Use Table 21.2 to obtain the appropriate unit horsepower value. 21.22. In a turning operation on plain carbon steel whose Brinell hardness ¼ 275 HB, the cutting speed is set at 200 m/min and depth of cut ¼ 6.0 mm. The lathe motor is rated at 25 kW, and its mechanical efficiency ¼ 90%. Using the appropriate specific energy value from Table 21.2, determine the maximum feed that can be set for this operation. Use of a spreadsheet calculator is recommended for the iterative calculations required in this problem. E1C21 11/11/2009 15:44:6 Page 505 Problems 21.23. A turning operation is to be performed on a 20 hp lathe that has an 87% efficiency rating. The roughing cut is made on alloy steel whose hardness is in the range 325 to 335 HB. The cutting speed is 375 ft/ min, feed is 0.030 in/rev, and depth of cut is 0.150 in. Based on these values, can the job be performed on the 20 hp lathe? Use Table 21.2 to obtain the appropriate unit horsepower value. 21.24. Suppose the cutting speed in Problems 21.7 and 21.8 is 200 ft/min. From your answers to those problems, find (a) the horsepower consumed in the operation, (b) metal removal rate in in3/min, (c) unit horsepower (hp-min/in3), and (d) the specific energy (in-lb/in3). 21.25. For Problem 21.12, the lathe has a mechanical efficiency ¼ 0.83. Determine (a) the horsepower consumed by the turning operation; (b) horsepower that must be generated by the lathe; (c) unit horsepower and specific energy for the work material in this operation. 21.26. In a turning operation on low carbon steel (175 BHN), cutting speed ¼ 400 ft/min, feed ¼ 0.010 in/ rev, and depth of cut ¼ 0.075 in. The lathe has a mechanical efficiency ¼ 0.85. Based on the unit horsepower values in Table 21.2, determine (a) the horsepower consumed by the turning operation and (b) the horsepower that must be generated by the lathe. 21.27. Solve Problem 21.25 except that the feed ¼ 0.0075 in/ rev and the work material is stainless steel (Brinell hardness ¼ 240 HB). 21.28. A turning operation is carried out on aluminum (100 BHN). Cutting speed ¼ 5.6 m/s, feed ¼ 0.25 mm/ rev, and depth of cut ¼ 2.0 mm. The lathe has a mechanical efficiency ¼ 0.85. Based on the specific energy values in Table 21.2, determine (a) the cutting power and (b) gross power in the turning operation, in Watts. 505 21.29. Solve Problem 21.27 but with the following changes: cutting speed ¼ 1.3 m/s, feed ¼ 0.75 mm/rev, and depth ¼ 4.0 mm. Note that although the power used in this operation is only about 10% greater than in the previous problem, the m